ID Questions

Lakukan tugas rumah & ujian kamu dengan baik sekarang menggunakan Quizwiz!

60 y/o M w/ CLL +f/ha/ataxia and AMS traveled from and outdoor family picni in rural virginia allergic to PCN T102 obtunded and nuchal rigidit WBC 25000 (30% bands) LP revelas WBC 1500 (50 N 50 L), glucose 30 and protien 200 GPC on CSF? abx recommended

A: TMP-SMX

An 18 yo man presents in March in Portland OR with several days of fever, cough, chest pain, tachypnea, hypoxia and conjunctivitis with this CXR. WBC 3.0, platelets 160, CRP 2.5, AST 75 2 days later he is in ICU on high levels of support. You suspect: A. Pneumococcal pneumonia B. Borrelia hermsii with capillary leak and ARDS C. Adenovirus D. Hantavirus pulmonary syndrome E. MRSA pneumonia F. Group A streptococcus with TSS

A: adenovirus conjuctivits + respiratory failure

Anisakis - raw or undercooked seaefood -eosinophilia - not in united states - we freeze the sushi - worm is still alive, burrow in GI - allergy to fish

toxocariasis (and baylisaascaris visercal larva migrans

A 46 year old male 18 months s/p HLA mismatched BMT. History of GVHD skin, GI tract, and BOOP 3 months ago, treated with steroids. One month s/p Parainfluenza 3 URI, with chest CT ‐ tree‐in‐bud opacities in LLL. Received levofloxacin for 10 days. He now has increasing shortness of breath and cough. Blood cultures no growth. Sputum - LF GNR. Serum galactomannan is negative. What is the most likely cause of his current process? A. Cryptococcus neoformans B. E. coli C. MRSA D. Aspergillus fumigatus E. Fusarium spp.

A: aspergillus fumigatus 1 month after viral infection several months after bad GVHD, even with negative galactomannan

50-year-old woman with newly diagnosed AML developed tender, pruritic papules and plaques on her neck. She had been febrile 38.7°C for the past several days and had received a dose of GCSF 3 days earlier, with rapid WBC increase (900 ANC). Most likely etiology: A.Candida albicans B.Sweet's syndrome C.Aspergillus niger D.Varicella Zoster Virus E.Pseudomonas aeruginosa

Answer B Sweet's sydnrome vasculitis rapidley recovering neutrophils

65 y/o M hx of jaundice 2 weeks before finished amox/clav for sinusitis hx of HTN on HCT and rosuvastavi ETOH 2 drinks per day TB 24, ALT 162, AST 97, ALk phos 235 IgM anti-HAV negative IgM anti-HBC neg HCV RNA neg, RUQ us neg which of the ofllowin gis most likely cause of hepatitis: A. toxicity from amox/clav B. EToh C. porphyria flare D. leptospirosis E. Statin

A: amoxicillin/clavulanate - cholestatic or mixed -often AFTER stopping med - 1/2500 rx - DRB11501 - Clavulanate > amoxicillin watch for hypersensitivites in dilantinc, abacavir, nevirapine

46 y/o M from pheonix Az trip to china 1 week of HA, stiffneck and vomitting no significant PMH and was sexually active with multiple partners PE - low grade fever and meningismus, otherwise negative CSF a WBC of 300, 60% eosinophils, glucose of 45, and protein of 150, GS negative cause? trepenoma pallidum mycobacterium tuberculosis coccidioides immitis angiostrongylus cantonensis lymphoma

A: angiostrongylus cantonensis all can give eosinophils but at this level should think of helmitnths

what test would support Strep pyogenes as the cause of acute glomerulonephritis in patient recently recovering from pyoderma?

Acute GN is immunological reaction Classically rise in anti-streptolysin ab (ASO) for antecedent GAS infection - measures 5 Ab; ASO, anti-hyaluronidase, anti-streptokinase, anti-NAD, and anti-DNase B after pharyngeal infection, ASO, anti-DNAse B, anti-NAD, AHase titres peak 4-5 weeks, decline in next several months after skin infection, only anti-DNAse B and Ahase titers increase

71 y/o M F/chills and pleuritic CP for 4 days a PPM implanted 15 months ago for SSS generator pocket is CDI +rhonchi and pleural rub no murmur 3/3 BCx + 24 hr for MRSA cxr nodules and pleural based lesions echo shows a 7 mm lead vegetation in right ventricle

A: remove entire device + vancomycin 4-6 weeks take out the device even if a pocket infection no data on addition of rifampin

39 y/o F, from brazil comes to ER with seizure CT scan is shown (shows a space occupying / trypasonma) HIV + CD4 - 20 VL 100,000 unsafe to perform LP started on sufla-pyrimthamine (toxo is most common organism in brazil) ARV are held after 10 days, she has not improved

A: traypanosomiasis lymphoma and toxo are common cause of space occupying disease chaga can look like toxo serology can help need CSF and brian biopsy

• 54 yo male 60 days post-cardiac transplant was treated for rejection with steroids when fever and a non-tender anterior cervical mass appeared. • Biopsy showed nodal replacement by lymphocytes, many of which stained positively for Epstein-Barr virus as well as for the B cell marker, CD20. • His plasma EBV viral load was 10,000 copies /ml. The most appropriate treatment for this condition is: A. Cidofovir B. Ganciclovir C. Acyclovir D. Cyclophosphamide E. Rituximab

Answer E

A 34-year-old woman who had lived in Georgia all of her life donated blood at her local blood bank and was found to be seropositive for HTLV-1. She had never previously been tested nor had she ever received a blood transfusion. She is referred to you for advice. She is in good health. Her complete physical examination and routine blood work are normal. She is interested in becoming pregnant. Her husband is HTLV-1 seronegative. Which of the following would you advise? She almost certainly acquired this from a prior sexual partnerHer husband is at no risk for sexual transmissionShould she become pregnant her baby should not be breast fedShe should have a bone marrow aspirate and biopsy to determine the need for therapy.She should treated with zidovudine plus interferon to reduce viremia if she becomes pregnant.

Correct Answer: Should she become pregnant her baby should not be breast fed. HTLV-1 infects 10-20 million persons globally, and thus it's important to know something about this virus. It is the cause of two potentially fatal diseases: HTLV-1 associated leukemia/lymphoma (2-5% of infected persons; ATL or acute T cell leukemia), and HTLV-1 associated myelopathy (also known as tropical spastic paraparesis-<2% of infected persons). In the US, less than 1% of the population is infected. Tropical spastic paraparesis typically occurs about 4 years after infection (range 4 months to 30 years) while ATL does not typically appear until 20-30 years after exposure. HTLV-1 is transmitted by breast-feeding, but also transplacentally, by sexual intercourse, and by blood (intravenous drug abuse or transfusion) or organ transplantation. See table below regarding HTLV-1 and 2 transmission. Thus breast-feeding is not recommended for infected mothers if other safer forms of infant nutrition are feasible. Transmission of Human T-Cell Lymphotropic Virus Types 1 and 2 Route of Transmission HTLV Type I HTLV Type II Mother to Child Transplacental Low efficiency Probable Breast milk Efficient Probable Sexual Heterosexual Efficient Efficient Male to male Efficient Unknown Parenteral Blood transfusion Very efficient Very efficient Injection drug use Efficient Efficient HTLV-1 and -2 are screened for in blood by a single test routinely in the United States. HTLV-1 and HTLV-2, unlike HIV-1 and HIV-2, are cell associated and therefore predominantly transmissible only with blood component transfusions. Screening for both types is done with a single test. The rate of transmission of HTLV-I decreased almost 10-fold (from 1 per 8500 to 1 per 69,000) after introduction of the screening test. Note in the table the differences between HTLV-1 and HTLV-2. The first choice is incorrect: she might have acquired this from her mother transplacentally or by being breast-fed. The second choice is incorrect because heterosexual and homosexual intercourse can transmit the virus. Condom use is recommended for discordant couples except for when they are trying to conceive. The fourth choice is incorrect: while the possibility of an HTLV-1 related leukemia lymphoma or HTLV-1 Associated Myelopathy is possible, neither is common (2-4% and 1-2% respectively) and a random bone marrow biopsy would not likely be useful in someone with no signs or symptoms or abnormalities on the blood count. The last choice is not correct: No therapy is known to be effective for eradicating virus.

62 y/o M w/ ITP on prednisone 20mg daily incarerated for 4 months +Fever, NS, wt loss, fatigue, loss of appetite, malaise, depression for 3 months VS: +fever, liver enlarged, spleen tip palpated ESR 75 , Alk phos 425 BCx/UCx NG, PPD / CXR negative / labs wnl CT = hepatosplenamogaly Disease? Pathogen? Treatment

D:Disseminated tuberculosis P: mycobateria tuberculoe T: RIPE

25 y/o M request for PrEP feels well no PMHx 2-3 different partner per month mostly unprotected oral and rectal intercourse two episodes of syphilis in the last year, one episode of GC/chlamydia normal renal function, what to prEP him

TDF / emtricitabine

79 y/o F 3 week of fever and fatigue jaw discomfort when chewing food episode of double vision one week ago became ill she attended a wedding at which she ate pork roasted on a spit T102 slight tendernes over left sclap mitral regurgitant mumur labs normal dx?

Temporal artery biopsy for giant cell arteritis FUO, Age > 50 scalp tenderness diplopia or transient visual law jaw or tongue fatige high sedimentation rate 50% associated with polymyalgia rhematic (PMR) -morning stiffnes

Which of the following mutations indicate high level resistance to elvategravir ? A. Q148R B. L68I C. L68V D. K67N E. K65R

Testing point: Every ID physician should know that Q148R is a common resistance mutation for efavirenz 1st Generation (Ral and Elva): Q148R and N155H most common; E92Q less common E681and V increase resistance but are not sufficient alone 2nd Generation (DTG) retains some activity vs Q148R and N155H, but presence of both yields DTG resistance Primary resistance mutation to Dolutegravir is R263K

35 y/o w/ recurrent UTI 3 episodes of F (103F), HA, Malaise, chills in last 4 weeks, each lasting 3-5 days; + found to have large opaque calculus, put on nitrofurantioin thereafter now with +rash , fever, eosinophilia, UA 0-4 WBC stop or not stop nitrofurantoin and add other pathogens?

stop nitrofurantoin

2 bipsies of lymph node, 34 y/o unexplained lymphadneopath wt loss the PAS stain is positive

tropheryma whipplei

38 y/o marine sergeant a week after shore leaves fever, melaise, 5 pustular lesions pain in flexion on shore leave in a port city in mexico, sex with a commercial sex worker, consumed alot of alcohol and passed out in an alley with rats and mice the most likely organism to grow from his BCx?

A: disseminated nisseria gonorrhea disease -tendonitis -migratory arthralgia - lesions -leptospirosis/syphilis/lyme disease (not grow in BCx) - gram negative bacillus (rat bite fever, Streptobacillus moniliformis, takes 3 weeks to grow)

45 y/o M HIV + CD4 <10, VL >100K been takaing TMP-SMX and efavirenz-tenofovir-emtricitabine only intermittently 3 week of fever, wt loss, and lesion aspiration many AFB rods, nonbranching, but after 6 weeks nothing grows the lesions is aspirated again how should it be grown?

A: mycobacteiium hemophilum can be cultured only at 32C with iron enriched medium

Rhodococcus buzzword Short gram + rods Cavitary pneumoinia Salmon PINk colonies Adhvanced HIV Horse

Actinomyces: Buzzwords Sulfur granules Dental work IUD Erosive mass Filamentous anaerobe

72 y/o F chronic cough normal CXR 1/3 sputum grew MAC what to do next? a)CT chest + AFB sputum b)Empiric Tx w/ Azith, Etham, and Rifapmin c) additional sputum AFB d) wait for in vitro susceptibilities

Answer A: X-ray usually normal, not specific for bronchiectasis Need CT Need more AFB (need at least 2 sputum) possibly due to transient sampling a lot of normal people have MAC

31M from tidewater Virginia presents in June with three days of fever and rash. Exam: unremarkable but T39.2°C, discrete black eschar on leg, scattered maculopapular rash elsewhere Which of the following is the most likely etiologic agent? A. Rickettsia rickettsia B. Ehrlichia chaffeensis C. Rickettsia parkeri D. Anaplasma phagocytophilum E. Rickettsia akar

Answer C american boutinee fever no eschar with HME or HGA R. rickettsii - R. akari - house mites

Echinocandin class of antifungals has which mechanism of action: A. inhibits synthesis of membrane sterols B. damages cytoplasmic membrane C. interferes with synthesis of fungal cell wall glucans D. inhibits fungal DNA synthesis E. interfere with synthesis of fungal cell wall chitin

Answer is C A - azoles B - amphotericin D - flucytosine E - ? not sure

A 42-year-old male with AIDS and a CD4 of 9 cells/cu mm was seen for a few weeks of pain over both maxillary sinuses and conjunctivitis. On examination, he was afebrile and had no tenderness over the sinuses. The ophthalmologist found bulbar and palpebral conjunctival suffusion with a thin discharge, but no corneal staining with fluorescein. The anterior chamber was normal. A conjunctival scraping sent for Gram stain was reported as showing 1-micron diameter, weakly Gram-positive spherical bodies. Which of the following is the most probable organism? Cyclospora cayetanensis Rhinosporidium seeberi Baylisascaris procyonis Encephalitozoon hellem Cryptosporidium parvum

Correct answer: Encephalitozoon hellem For viral keratitis, you should be thinking of HSV and adenovirus. For patients with contact lenses, consider bacteria (Staphylococcus aureus and epidermidis, Pseudomonas aeruginosa, diphtheroids, Streptococcus pneumoniae), and occasionally molds and microsporidia and acanthameba. This patient has keratoconjunctivitis and sinusitis. The only organism listed that can cause superficial keratoconjunctivitis and sinusitis is Encephalitozoon hellem. Encephalitozoon (Septata) intestinalis can also cause superficial keratoconjunctivitis. For the boards, don't worry about the taxonomy of microsporidia: it's confusing and it's in flux. By size, 1 micron organisms cannot be cyclospora (8-10 u) or Cryptosproidia (4-8u) and these latter protozoa would not be desribed as spherical bodies and would never cause eye disease . Bayliascaris is a round worm of raccoons that is associated with eosinophilic meningitis. Rhinosporidiosis is a distractor here that is not covered elsewhere in the course, but you should know what this is. Rhinosporidiosis causes fleshy, pedunculated lesions in the conjunctiva or nasal mucosa of normal persons in tropical areas. There have been a few cases in Texas and the South. The causative organism, Rhinosporidium seeberi is now thought to be a protozoon. Exposure to stagnant water is common. There is no medical therapy; lesions can be excised. Correct Response Encephalitozoon hellem

These skin lesions and the peripheral smear from an AIDS patient are most likely to be caused by: Candida albicansLeishmania donovaniAnaplasma phagocytophilumHistoplasma capsulatumCryptococcus neoformans

Correct answer: Histoplasma capsulatum The skin lesions are non specific and could be caused by fungi or viruses, as well as other organisms. The peripheral smear shows intracellular yeasts that are most compatible with Histoplasma by size and by clinical scenario. Cryptococcus neoformans could cause the same skin lesions but is rarely seen in peripheral blood smears. The skin lesions are not compatible with the other agents listed.

This blood agar plate inoculated with stool from a patient being treated with high dose steroids who developed pulmonary infiltrates and diarrhea. The most likely drug to benefit this patient is which of the following: IvermectinCiprofloxacinPraziquantelNitazoxanide

Correct answer: Ivermectin Filariform larvae of Stongyloides stercoralis can be seen to migrate on agar plates, dragging the stool flora along with them. Ivermectin is appropriate treatment. Correct Response Ivermectin

A 35-year-old patient with HIV (CD4 = 250, VL = 400K) comes in for an initial screening. He has no symptoms, but the following is seen on funduscopic examination. These lesions are most likely due to: Choices A HSV B VZV C CMV D Hepatitis C E Non-infectious infarcts

Correct answer: Non-infectious infarcts These are cotton wool spots, which are retinal infarcts that may or may not be related to HIV viremia. These are not related to herpes viruses or hepatitis virus. The photo shown has an unusually large number of cotton wool spots: usually only a few are seen. For HIV infected patients with CD4 counts >100, there are only a few retinal lesions associated with HIV to think of: HSV (acute retinal necrosis) and cotton wool spots. Other diseases that can occur in HIV negative individuals must also be considered such as syphilitic uveitis, and congenital toxoplasmosis . The absence of hemorrhage or vitreal inflammation should suggest cotton wools spots, especially in patients with CD4 counts >100. Diabetes mellitus and hypertension cause similar lesions.

An outbreak of nosocomial multi-drug resistant Acinetobacter baumannii infections has occurred in the burn unit. Evaluation by the microbiology laboratory demonstrates the epidemic strain produces both an extended-spectrum beta-lactamases and a carbapenemase. Which of the following antibiotics is most likely to be effective against this pathogen? Doripenem Cefepime Piperacillin/Tazobactam Polymyxin B or Polymyxin E (Colistin) Trimethoprim-Sulfamethoxazole

Correct answer: Polymyxin B or E A polymyxin is often the only treatment available for these MDR Gram negative infections that are resistant to most B-lactams and carbapenems. In addition, many strains demonstrate concomitant resistance to aminoglycosides, trimethoprim-sulfamethoxazole, and fluoroquinolones. If polymyxin therapy alone is used, resistance often develops within several days. Thus combination therapy is often employed. Given the limited therapeutic options and differing susceptibility patterns of individual isolates, there is no consensus regimen for treatment: a variety of drug combinations and pharmacokinetic approaches have been used and are the subject of ongoing clinical trials. At present, there is no convincing evidence that any one combination regimen is best for any specific multi drug resistant isolate. . Some isolates are sensitive to tigecycline or to an aminoglycoside. Despite in vitro activity, significant numbers of failures have occurred with tigecycline and hence its use is discouraged. It is postulated that the failures are due to the drug's low serum concentrations. Polymyxins: There are two parenteral polymyxins in clinical use. Polymyxin E is also known as Colistin. Polymyxin B is the second drug. Colistin is a "pro" drug and polymyxin B is not. There is no clear superiority in efficacy between the two. Polymyxin B is easier to use as its body clearance is by non-renal pathways and hence, there is no need to adjust dosage based on decreased renal function. Colistin is excreted by the urinary tract and dosage adjustment is necessary. Doripenem is a carbapenem and therefore is inactivated by carbapenemases. There is a theoretical therapy that combines doripenem with ertapenem. The carbapenemase binds to ertapenem with much greater avidity and thereby may protect the doripenem. There are no confirmatory controlled clinical trials using this approach. Cefepime and Piperacillin/Tazobactam are often ineffective in ESBL-producing organism even when reported as susceptible. Most of the multiresistant A. baumannii isolates are resistant to TMP/SMZ.

A previously healthy 24-year-old woman presented to the emergency room with a 2-day history of headache, vomiting, severe migratory arthralgias and a swollen left knee. She is employed in a pet store where she is responsible for dogs, cats, hamsters, and pet rats. She reports multiple scratches while working with all these animals. On exam, she is febrile to 101°F, hypotensive, has left knee synovitis and a faint, diffuse maculopapular rash. She is treated empirically with broad spectrum antibiotics and improves. The laboratory is asked to hold her blood cultures longer than the usual 5 days and after 7 days both aerobic and anaerobic blood cultures show thin Gram negative bacilli which vary from short to long filaments. Some of the longer filaments have bulbous swelling. The automated identification system did not have enough growth to report a species so the isolate was sent out for sequencing. What is the most likely causative agent? Fusobacterium necrophorum Streptobacillus moniliformis Brucella melitensis Francisella tularensis Borrelia recurrentis

Correct answer: Streptobacillus moniliformis Rat bite fever is an infectious disease caused by two different organisms. Both are fastidious pleomorphic, branching gram negative rods. Streptobacillus moniliformis causes rat bite fever in the United States. "Spirillum minus" causes rat bite fever in Asia and Africa. S. moniliformis is a filamentous gram negative bacillus that is a frequent colonizer of the nasopharynx of rats. Human infection follows rat bites or scratches; however, cases can also be seen after ingestion of food or drink contaminated with rat excrements (Haverhill fever). Infection also may result from handling rats, with no reported bite or scratch. Other rodents such as mice and gerbils may also spread the infection. Symptoms usually occur 2 to 10 days after exposure, and include an abrupt onset of chills and fever, vomiting, headache, myalgias and septic arthritis. A rash is seen in the majority of patients. The presentation of a systemic illness with severe arthralgias or septic arthritis and rash should raise the possibility of rat bite fever. S. moniliformis is difficult to grow, the microbiology laboratory should be alerted so that culture techniques are optimized. Intravenous penicillin is the treatment of choice. Presence of long filaments with fusiform swellings is seen with older cultures of S. moniliformis but not the other listed organisms. The presence of a maculopapular rash is not characteristic of what is seen with the other infections that are listed as possible answers. Further, the morphology and aerobic growth is inconsistent with F. necrophorum. The fever was not "recurrent" as in Borrelia recurrentis infection. Tularemia and Brucella are short coccobacillary organisms.

A 20-year-old patient from Jamaica with aplastic anemia received a cord blood transplant 5 months ago in Bethesda, Maryland with excellent engraftment, and is maintained on tacrolimus plus prophylactic antimicrobials. Two weeks before admission (4 months post-transplant) he developed progressive fever, shortness of breath, and a slight cough. He has bilateral crackles on lung exam but no wheezes. There is significant hypoxemia (pO2=90mmHg on room air) but no skin rash or diarrhea. He has not taken his trimethoprim-sulfamethoxazole, fluconazole, or acyclovir because he thinks they made him nauseated, but he did take his tacrolimus. His chest CT scan showed diffuse, bilateral ground glass infiltrates. WBC=5000 cells/uL (90% polys) Bronchoalveolar lavage: Direct stains negative for pneumocystis, bacteria, fungi and AFB BAL PCR positive for CMV, but blood CMV PCR negative BAL PCR positive for Toxoplasma What is the most likely cause of his pulmonary process? Cytomegalovirus Engraftment Syndrome Bronchiolitis obliterans Toxoplasmosis Candida

Correct answer: Toxoplasmosis pulmonary toxoplasmosis BAL PCR, very specific for detection of toxoplasma identically to PCP, and would not be identified by the usual direct tests or cultures. Had he taken his trimethoprim-sulfamethoxazole for PCP prophylaxis, he would have been protected from pulmonary toxoplasmosis. Engraftment syndrome occurs typically when the neutrophil count is returning. Thus this syndrome occurring months after the return of neutrophils is too late for this. Bronchiolitis obliterans is a late complication (>100 days post-transplant), usually in allogeneic transplants, occurring with other evidence of graft vs. host disease, and manifesting as airway obstruction (wheezing and restrictive disease), sometimes with diffuse infiltrates and fever. This is not likely to be due to Candida since Candida almost never causes pulmonary disease. ' CMV is not likely to be the cause in view of the negative blood PCR; a positive CMV PCR can occur with CMV reaction and/or cellularity, but one does not typically see pneumonitis without signs of peripheral reactivation as well.

54 y/o M, rectrack handyman, smoker - 2 weeks ago washing an empty stable Severe PNA in ICU - T 42 - WBC 19K, Cr 1.8, HIV - - Na 119 P?

P: Legionella is conmon contaminant of water system when water spray is inhaled - lung dx and smoker are predisposing factor other Pathogens: Coxiella burnetti (Q fever) - from tissue of slaugther animals and afterbirth Rhodocuccus equi - immunosuppres, usually no exposure to horses Burkholderia mallei (Glanders) - sick horses Histoplasma - dust from rich soil or bat guano

A 50 year old man who formerly engaged in injection drug use presented to your clinic with hepatitis C and HIV infection. At the time of his presentation he had both a liver biopsy and transient elastography, and was determined to have cirrhosis (Childs‐Pugh Class A). •He was started on dolutegravir plus tenofovir/emtricitabine •Three weeks later he was treated with sofosbuvir‐velpatasvir and had a sustained viral response (negative HCV PCR 12 weeks after the end of treatment). Which of the following should be done to monitor him for the development of hepatocellular carcinoma. A. He needs no monitoring since he was cured of HCV B. Transient Elastography every 6 months C. Liver ultrasound every 6 months D. Liver biopsy every 12‐24 months E. Serum Alpha‐fetoprotein every 6 months

Correct answer: C. Liver ultrasound every 6 months Per the IDSA/AASLD guidelines, all patients with advanced liver fibrosis (metavir F3‐F4) prior to HCV treatment should be monitored for hepatocellular carcinoma every 6 months with liver ultrasound. Since this patient has cirrhosis, he should be monitored by ultrasound, or alternatively perhaps CT or MR, every 6 months for life to detect the occurrence of hepatocellular carcinoma. Patients with HIV infection should be monitored similarly. Patients with HIV infection have accelerated risks of progression of HCV, even if their HIV is well controlled with effective antiretroviral therapy. Once patients have a sustained viral response (12 weeks after end of treatment) to the current drugs for HCV, viral relapse is unlikely, although monitoring the patient for HCV reinfection, or for HBV or HIV infection, would be appropriate if he were still engaging in risk behavior. Although dynamic contrast‐enhanced CT or MRI are more sensitive than ultrasound, the use of ultrasound every 6 months is commonly used since this approach has been shown to reduce mortality from hepatocellular carcinoma. For background information, not a testable issue: *Metavir is a classification used to assess inflammation and fibrosis by histopathological evaluation of a liver biopsy. The grade indicates the degree of inflammation, and the stage represents the amount of fibrosis or scarring. **Childs Pugh Score assess Total bili, serum albumin, prothrombin time, ascites, and hepatocellular encephalopathy to come up with class A (two year survival 85%), B (two year survival 60%), C (two year survival 35%)

This soft tissue gas in a patient with neutropenia and adenocarcinoma of the colon is most likely due to which organism: Choices A Clostridium septicum B Fusobacterium necrophorum C Bacteroides fragilis D Eikenella corrodens E Escherichia coli

Correct answer: Clostridium septicum Clostridial myonecrosis also called gas gangrene is a life-threatening process that develops either contiguously from an area of trauma or hematogenously from the gastrointestinal tract . Traumatic gas gangrene is usually caused by C. perfringens. Spontaneous gangrene is most commonly caused by C. septicum. Gas formation is a common development in clostridial myonecrosis and is not seen with the other bacilli listed.

A non-obese 60 y.o. male has severe COPD Required tracheostomy and residence in a SNF Now re-admitted with new RLL pneumonia Culture of tracheostomy aspirate 3 weeks ago was positive for P.aeruginosa susceptible to piperacillin-tazobactam Normal renal function No drug allergies Pending results of culture of new tracheal aspirate, Which one of the following empiric piperacillintazobactam regimens would you select ? A. 3.375 gms IV over 30 min. q 4 h B. 3.375 gms IV over 30 min q6h C.4.5 gms IV first dose; 4 hrs later start 4.5 gms IV over 4 hrs and repeat q8h D.4.5 gms IV over 30 min q8h

4.5 gms IV first dose; 4 hrs later start 4.5 gms IV over 4 hrs and repeat q8h

47 y/o M, lives in NY, + tick exposure + acute right faical droop, + HA, +malaise, + 7th nerve palsy, + erythema migrans, - nuchal rigidity Dx = disseminated lyme disease Tx- 14 day of doxycycline Q: Abx therapy effects 7th nerve palsy?

A: No! abx therapy prevents later complications (such as lyme arthritis) but no effect on 7th nerve palsy *early neurological lyme disease = cranial nerve palsy (7th most common, can be bilateral)

62 y/o from india, cam 20 years ago, traveled there a year ago +recurrent staphyloccoccal skin infections abscess growing (community staph aeurus) exam: healing abscess on his thigh, drained, one-half by two inch hypopigmeneted macular lesion in his upper arm (been there doesn't bother him) hypopigmeented skin area is almost completely without senssation? likel cause of hypopigment

A: mycobacterium leprae pinta (trepenoma pallidum pertenue infection), south america, not in india yaws (trepenema carateum)

45 y/o with 3 days of painful skin lesoins (bullous and pustule filled) which toxin is repsonsible?

A: Exfloiative toxin A (ETA) -targed to protein at stratum granulosium/cornium layer -systemic infection with staph aeurus not panton-valentine leukocidin

46 y/o HIV + M CD4 235, HIV rna undetec HBsAG +, no symptoms on TDF /FTC /RAL liver enzymes incresed from ALT of 46 to 1041 TB was 2.3 sexually active with men various ART regimens

A: acute HCV infeciton

27 y.o. male who is otherwise healthy presents with acute appendicitis and hypotension. At surgery, appendix is ruptured and severe fecal peritonitis is present. The hospital has run out of piperacillin-tazobactam In addition to crystalloid fluids and nor-epinephrine, which one of the following regimens offers the best combination of safety and efficacy ? Ceftazidime-avibactam plus metronidazole B. Ceftolozane-tazobactam plus clindamycin C. Ceftazidime plus tigecycline D. Cefepime alone

A:Ceftazidime-avibactam plus metronidazole clindamycin - half of B. fragilis are resistant

65 y/o with well controlled HIV received 13 valent 3 years ago, and 23 valent 5 years ago what vaccine needed?

A: he needs a PPSV23 alone >= 65 years old needed 19-64 w/ CLung/liverD, CHF, CSF leak, smokers, DM, Etoh functional or anatomic asplenia immunocompromisded

a couple in 30's plan to adopt 2 year old from ethiopia, they have a regular babysitter and a 7 year old who should receive the hepatitis A vaccine?

A: individual with close contact, all of them if exposure from high endemic area, 2 weeks prior to arrival who should get it: travelers MSM (can be transmitted sexually) IVDU work with nonhuman primates close contact with international adaptee chronic liver dx post-exposure PPx

38 y/o mexican migran worker in california painless lesions give him? thalidomide clarithromycin amikacin dapsone liposomal amphotericin

A: leprotemous leprosy - cresecent sticking out of skin treat with dapsone

38 y/o F 2 days of fever, HA, stiff neck, episdoes every 3-4 months with spontanous abatement sexual active with her husband of 8 years and 2 children +evidence of meningismus CSF 70 WBC, glucose 60, protein 100

A: molora syndrome HSV 2

57 y/o F liver transplant recipient transplant for ETOH steatohepatitis, CMV D+/R+ whole blood HHV-6 VL of 3.6x106 at 3 month post -up (done due to F x 4 days) receive done motnh of acyclovir PP post transplant receicinvg mycophenolate mofetil, prednisone and TMP-SMX post-trasnplant course was complication with episdeof of rejection on day 30 transpalnt PE unremarkable recommend? IV gan / PO valgan / oral cyclovir / IV foscarnet / no antiviral

A: no antiviral therapy

80 y/o M F, HA, personality changes T101F, disoriented CT w/o contrast negative CSF WBC 80 (95% poly), glucose 70, protein 120 , GS negative on acyclovir MRI shows enhancement of temporal lobe 3 days later CSF PCR for HSV 1 and HSV 2 negative now oriented and follows command? next step?

A: obtain a new CSF for HSV PCR

which of the following is a risk factors most predictive of complicated s. aureus bacteremia?

A: positive BCx on appropriate therapy + BCx positive after >48-72 on therapy (odds ratio 5.6) - community-onset (OR3.1) not sure how long of bacteremia, more seeding -fever > 3 DOT - skin findings -persittent or secondary focus -endocarditis - elderly (not sure) - MRSA

38 y/o F with AML admitted with fever underwent induction chemotherapy 2 weeks ago, complicated with neutropenic fever d/c to home developed fever without localizing symptom CBC WBC 12.250 with 20% bands T 101.4 BCx negative fever persists with interval development of raised , red-purple , tender non-pruruitic papules and nodules on her face neck an dorsum some papules are plaque-like biopys done shows neutriphlic infiltrate

A: sweet syndrome acute febrile neutrophilic(periph and path) dermatosis (plaques, on drosum of hand) AML

50 y/o m HIV CD4 40, VL 600,000 has CNS toxoplasmosis CT finding, and CSF PCR + also has cryptosporidosis with 4 stools per day + n/V limited food intake pharmacy can't obtain sulfadiazine or pyrimethamine best therapy for toxoplasmosis? Atovaquone clinda plus primaquine TMP-SMX Azithro Nitazoxanide

A:TMP-SMX trimethoprim not as active as pyrimethamine - due observation data form africa sulfa-trimpethoprim just as good clindamycin plus primethamine atovaquone - won't observe it

A previously healthy 24 year old man presents complaining of the acute onset of fever and myalgias. He is married and has an 18 month old child. On exam, has no adenopathy, pharyngeal exudate or rash. His AST and ALT are 2.5X normal. Peripheral smear is below: The likeliest pathogen is: A. CMV B. EBV C. HIV D. HHV-6 E. HHV-7

Answer is A no adenopathy, no pharyngeal exudate , toddler at home, atypical lymphocytes

Asymptomatic HIV + (CD4 = 10, VL 300,00) labs / CXR negative Toxo IgG positive *Sulfa allergy Recommendation in regards to PCP & Toxo prophylaxis?

Atovaquone Should have primary PPx Sulfa & dapsone have 50% cross-reactivity Aerosol pentamidine - ok for PCP but pyrimethamine not sufficient for Toxo

A 56 y.o man from southern Missouri presents in July with fever malaise, and rash of two days duration. Exam is only notable for T 38 and an annular "bulls-eye" 6 X 8 cm lesion on the lower back in area where a engorged tick was removed earlier in the week Which of the following is the most likely diagnosis? A. Lyme disease B. HME C. HGA D. Southern tick-associated rash illness E. B. lonestari infection

B. lonestarii (single case) Appears to occur after bite of Lone start tick B. burgdorferi tests including serology negative Unclear if doxycycline needed No sequelae

During an ongoing outbreak of Legionnaire's Disease at a nursing home, an 88‐year‐old resident of the nursing home is hospitalized for lobar pneumonia and respiratory distress. Her roommate at the nursing home was definitively diagnosed with Legionnaire's disease three days earlier. Which one of the following is the most appropriate isolation procedure for the newly ill 88‐year‐old? A. Standard precautions B. Droplet precautions C. Contact precautions D. Airborne precautions

Correct answer: A. Standard precautions Legionella is not transmitted person‐to‐person so droplet or special contact or airborne isolation precautions are not necessary for this patient.

This brain section was taken from autopsy of an HIV-infected patient. The origin of this infection was probably exposure to: Cat stoolHuman stoolMosquitoesTicksLice

Correct answer: Cat stool The structure is a cyst of Toxoplasma gondii, an infection acquired from ingestion of oocysts in cat stool or cysts in inadequately cooked meat from sheep or cows. Humans do not excrete oocysts in their stool. Insects are not vectors of toxoplasma.

This lung biopsy shows cells that stain pinkish-red with Mayer's mucicarmine stain. Blastomyces dermatitidis B Histoplasma capsulatum C Paracoccidioides brasiliensis D Cryptococcus neoformans E Histoplasma duboisii

Correct answer: Cryptococcus neoformans Mucicarmine stain is helpful in confirming that a yeast in tissue is a Cryptococcus, though not all cryptococcal cells on a pathology slide will stain clearly positive. None of the other fungi stain with mucicarmine. Histoplasma duboisii causes African histoplasmosis and is mucicarmine negative.

This previously healthy 35-year-old Maryland woman had the sudden onset of fever, convulsions and confusion after leaving her office in July. The most likely organism is: West Nile virusEastern equine virusHerpes simplex virusVaricella zoster virusHHV6 virus

Correct answer: Herpes simplex virus A lesion in the medial portion of the temporal lobe is seen shortly after onset in nearly all cases of herpes simplex encephalitis. While the other entities could mimic HSV, the location and presentation are characteristic of HSV encephalitis and prompt empiric therapy for HSV would also be appropriate until some other etiology is proven. Human herpesvirus 6 can cause encephalitis with lesions in the limbic system, including the temporal lobe, but encephalitis due to this virus appears confined to immunosuppressed patients. There is a long, long list of viral, protozoan and bacterial pathogens that could present with these findings, but this clinical scenario and this scan should always make HSV first on your list, and should require immediate acylcovir until some other entity is proven to be the cause.

The hypervirulent strain of C. difficile designated North American Pulse Field 1 (NAP1), 027 by PCR ribotyping, and BI by restriction endonuclease analysis (REA) is characterized by which of the following? Patient mortality in excess of 50% within 30 daysRecurrent C. difficile infection rates of over 35%High level toxin A and B productionVancomycin resistanceFidaxomicin resistance

Correct answer: High level toxin A and B production This hypervirulent strain of C. difficile is characterized by mutations in the tcdC gene which is a suppressor of toxin A and B production. Thus, with less suppression of toxin, there are high levels of toxin A and B production. These strains also carry the genes for binary toxin. These strains have epidemiologic importance but their diagnosis and therapy does not differ from the more common strains. Patient attributable mortality is high but not 50%, mortality is probably less than 7% overall, however, mortality rates in excess of 10% are found in the oldest patient populations over the age of 80. Disease occurs primarily in elderly hospitalized patients rather than younger community patients.

A man in his 20s presented with nine days of vomiting, diarrhea, fever, headaches. He lived on farm with goats, chickens, guinea pigs, turkeys, cats, dogs. He appeared acutely ill. T104.4°F. Exam otherwise normal. AST 111, ALT 79, Alk. Phos146. Liver biopsy showed doughnut granulomas A:Coxiella B. Cryptococcus C. Histoplasma D. Cyclospora E. Bartonella

Coxiella and "doughnut" granulomas Coxiella burnetii, the agent of Q fever, is contracted through exposure to infected ruminants (parturient goats, cattle, sheep). Transmission usually occurs by inhalation. Acute disease usually presents with high fever, pneumonia and/or hepatitis. "Doughnut" granuloma (dense fibrin ring, central lipid vacuole) characteristic of Q fever May also be seen in Hodgkin's disease, EBV and Cytomegalovirus infections, drug hypersensitivity

►Animal urine on intact skin: hiker, farmer, forestry, veterinarian, swimming, falling in water or rafting in contaminated water Leptospirosis ► Handling overseas animal hair, hides Anthrax ► Slaughterhouses, veterinarians, parturient cats, sheep handlers Q Fever

Culture negative endocarditis Homeless: Bartonella quintana Animal exposure: Coxiella burnetii Kaposi-like skin lesions: Bartonella henselae Tender lymph node: bartonellosis, tularemia, plague Fever + jaundice: leptospirosis Sacroiliitis: brucellosis

A 46-year-old newspaper reporter was seen at a local emergency room complaining of influenza-like symptoms for about 3 days and then onset of a high fever, cough, and SOB. On arrival his temperature was 39° C, pulse 124/min, BP 102/60 mmHg, respiratory rate of 30/min. He was agitated and slightly confused. According to a friend who accompanied him, the reporter had been working on a "hot" story but was highly secretive about the content. Chest X-ray revealed small bilateral pleural effusions. A chest CT without contrast showed enlarged mediastinal lymph nodes with possible necrotic centers and mild edema of the mediastinal tissue, and bilateral pleural effusions with small lower lobe infiltrates or atelectasis. Thoracentesis found the effusion to be bloody. Based on the above history and workup, the most likely diagnosis is: Plague pneumonia Marbug Ebola Inhalational anthrax Lassa fever

Correct Answer: Inhalational anthrax This question and answers lead the reader to assume that the reporter was exposed to an agent of bioterrorism and asks how different agents might present-no answers are given for more conventional pathogens that might logically be in your differential diagnosis. Inhalational anthrax is an agent of bioterrorism that presents as a two phase illness -a flu-like syndrome followed by a rapidly progressive illness. Most patients have fever, cough, and SOB. Chest x-rays are often non-diagnostic but CT scans often reveal necrotic mediastinal and hilar lymph nodes, mediastinal edema with minimal parenchymal involvement initially. An early and diagnostically useful sign is bloody pleural effusions, arising from necrotic mediastinal and hilar lymph nodes. It is crucial to consider this diagnosis and to initiate appropriate treatment as quickly as possible due to the very high mortality and the importance of notifying the proper authorities as well. None of the other options has a suggestion of necrotic mediastinal nodes or bloody pleural effusions. Plague pneumonia is a severe, with shock an early event. Marburg and Ebola present as fever, myalgia and headache, followed by pharyngitis, nausea, vomiting, and maculopapular rash. Hemorrhagic manifestations and liver failure may follow. Lassa causes a similar systemic illness to Ebola, but without rash.

19 y/o immigrant from Iraq 2 days of fever and severe abdominal pain similar episdoes at least 3 occasiona over past 7 years underwent appendectomy and was normal, other episodes resolved spontanously after 2-3 days T102, no rash, significant abdominal tenderens and gauarding CT abdomen and pelvis normal dx?

familial mediterranean fever sporadic, recurrent fever and serositis dx with genetic testing buzz word periodic episdoes (fever + ) colchicine repsonsive (though behcet's also responsive) ethnicity

47 WM executive referred from Baltimore because of severe headaches, diplopia, high fever of 1 wk's duration 4 wks PTA: Maui resort one week 3 wks PTA: ranch outside Tucson 1 wk 2 wks PTA: back at work in Baltimore 1 wk: PTA: Headache began Exam: Temp 38.5 C. Looks ill. Photophobia, nuchal rigidity, right CN6 palsy CBC, Chem 7 normal. CSF : Glucose 55, Protein 58, WBC 330 (20% eos). Negative cryptococcal antigen on CSF, serum Lyme serology and RPR. MRI with contrast normal. Worsens during 2 wks ceftriaxone. CSF cultures for bacteria, fungi, tbc neg to date. The most helpful diagnostic test would be: A. CSF cytology B. Stool O&P C. Dietary history D. Fungal serology E. Leptospirosis serology

fungal serology for coccoidomycosis

HIV + 3 week of cough, fever, 25lb wt loss, anorexia CD4 10, VL 500K cxr diffuse interstitial infiltrates BAL + PCP by immunofluresence x2 week later in hospital, the lab reports three BCx and BAL growing Mycobaterium Probe = mycobaterium avium complex type of isolatoin?

none spread by dust, dirt and food not person to person

Treatment of P. falciparum Uncomplicated (no organ dysfunction, low parasitemia, able to take po) if chloroquine sensitive area chloroquine if chloroquine resistant area artemether/lumefantrine (Coartem) x 3 days atovaquone/proguanil (Malarone) x 3 days 2nd line: quinine x 3 days + doxycycline x 7 days Severe IV artesunateavailable in U.S. on an IND (770-488-7788 or 7100) IV quinidine (or quinine) (ICU monitoring for QT prolongation) **NOTE: there is increasing artemisinin resistance in SE Asia but it has not yet emerged in Africa

Treatment of P. vivax chloroquine x 3 days THEN primaquine (30 mg base) x 14 days Need to check G6PD status before administering primaquine as primaquine can cause severe hemolysis in patients with G6PD deficiency Primaquine requires cytochrome P-450 2D6 to be effective. Therefore, clinical failure to cure P. vivax can be due to low host levels of CYP450-2D6. N Engl * note: single dose tafenoquine was approved for radical cure of P. vivax in July of 2018*

Finding an acid fast bacillus in a nasal smear is most consistent with: RhinoscleromaRhinosporidiosisMucocutaneous LeishmaniasisLepromatous leprosyTuberculosis

Correct answer: lepromatous leprosy Patients with lepromatous leprosy often have acid fast bacilli in their nasal smear. In endemic countries nasal mucosa along with skin snips from selected sites are sampled to classify leprosy as lepromatous and to determine the bacillary load. Leishmania, rhinoscleroma and rhinosporidiosis are not acid fast. Tuberculosis would rarely be found in a nasal smear.

21 M 8 d/a sinusitis->amoxicillin then 4 d/a +abd pain, fever, blood diarrhea colonoscopy: hemorrhagic colitis ascening colon w/ pseudomembrane abx stopped Stool/colonscopy cx: Klebsiella oxytoca -ve for slamonella, shigella, campy, EHEC (toxin assay), cdiff symptoms resolve D/P/T

D: antibiotic related hemorhagic colitis - usually young, PCN, right sided colitis P: Klebsiella oxytoca - EHEC cannot be excluded by toxin assay on stool or serotypic of sorbital negative colonies - K.oxytoca produces cyotoxin and fits the koch's postulates T: stop antibiotics?

39 y/o 10 days ago to indonesia no ppx against malaria now fever and HA blood smear with plasmodium in it;

plasmodium knowlesi - new 5th cauese of malaria - located in few placed of the world, mosquito bites monkey, then humans - sick very fast - looks like plasmodium malaria (very flimsy), unlike plasmodium knowlesi which is rapid and kills you

Low Dose Pathogens Commonly Cause Diarrhea Outbreaks in Day Care Center Which of the following doesn't fit? A. Shigella B. Cryptosporidium C. Giardia D. Campylobacter jejuni E. Norovirus

campylobacter jejuni high inoculum needed, not associated with outbreaks

What if he had HIV-2. What would be the best test to assess the presence of HIV-2? A. Send blood to CDC for specialized HIV-2 test B. Order "HIV-2" antibody test via commercial lab C. Order HIV-2 RNA PCR D. Do nothing. HIV-2 is part of routine HIV testing

do nothing, HIV is part of routine HIV testing New Generation of "Combo"HIV-1 / 2 Tests 4th generation—Synthetic peptide or recombinant protein antigens are used Antigen sandwich format: Detect IgM and IgG antibodies Monoclonal antibodies are included to detect p24 antigen. Tests usually do not distinguish antibody reactivity from antigen reactivity. Examples commercially available in US: HIV-1/HIV-2 enzyme immunoassay, HIV-1/HIV-2chemiluminescent immunoassay HIV-1/HIV-2 rapid test that uses separate indicators for antigen and antibody reactivity.

47 y/o F + recurrent bronchitis + bacterial pneumonia IgG - n, IgA - low, IgM-n Dx? Rx?

-CVID -Failure to respond is important! 1) Titers against tetatnus and pneumococcus 2) Immunize (Pnuemovax23) 3) Repeat titers (3-4 weeks later) 4) if low (<75% valent response), consider IVIG

Elderly Female w/ IDDM + 4 days of ear pain, + yellow drainge, + facial nerve palsy, NO fever Dx? Pathogen? Tx?

-Malignant (necrotizing) external otitis - pseduomonas aeruginosa - IV Or PO ciprofloxacin

Medicinal leeches are applied to a non-healing leg ulcer. Which one of the following pathogens is found in the "mouth" of the leech ? A. Alcaligenes xylosoxidans B. Aeromonas hydrophila C. Acinetobacter baumannii D. Arcanobacterium haemolyticum

Aeromonas hydrophila

Trichinellos undercooked pork food muscle pains periorbitla edema eosinophelia

Angiostrongylus the rat lung worm eisoniophilic meningitis ingestion of parasite in snails or slugs

Histo: TNFα blockers. Miliary infiltrates Malassezia furfur: culture needs oil Talaromyces marneffei: SE Asia. Skin lesions. Septum in dividing cells Fusarium: skin nodules. Blood culture with mould Ecthyma gangrenosa: aspergillosis, mucor, bacteria

Aspergillosis: halo sign, crescent sign Mucormycosis : Can mimic cavernous sinus thrombosis. Aseptate. Ampho. Crypto: LP for high OP. IRIS. Cocci: solitary lung cavity. Eosinophilic meningitis Blasto: indolent skin+lung lesion Candida: liver, spleen lesions in neutropenics

You are asked to see a patient who has had a copper IUD in place for several years and had a routine Pap smear that showed clumps of Gram positive bacilli consistent with Actinomyces species. The patient is totally asymptomatic, and her pelvic exam was normal. Which one of the following is the most appropriate recommendation for this patient? Culture the cervix for Actinomyces. Remove the IUD. Treat with antibiotics. Remove the IUD and treat with antibiotics. Tell patient to contact her physician if she develops symptoms.

Call doctor if symptoms develop common colonizer of copper IUD, not plastic ones

chronic granulomatous disease get recurrent infections; more common pathogens posses what? Capsule/catalase/exotoxin/fimbria/lipoteichoic acid

Catalase

20 y/o M Fever, nightsweats, and weight loss, w/ lymphadenopathy HIV Ab + CD4 20 Node biopsy: non-caseating ganuloma, + AFB seen Dx?

Disseminated MAC Start MAC therapy Encouarge Speciation 1/1000 incident CD4 <50 - 50/1000

A 50 y.o. previously healthy man developed a symptomatic urinary tract infection during a hospital stay for acute myocardial infarction. The most likely pathogen is:

Ecoli

Which of the following will most likely be missed on routine HIV testing? A. Group M B. Group N C. Group O D. Group X and which is the most common group?

Group O most likely to be missed - outlyer The most common group is group M - major group

Fundamentals ‐ risks - Early - mucositis, neutropenia - Late - GVHD (steroids, asplenia, T cell dysfunction) • Syndromes - Early pulmonary syndromes • Bacterial, fungal pneumonia • Non‐infectious: Alveolar hemorrhage, IPS - Late pulmonary syndromes • CMV, respiratory viruses, IFI • Non‐infectious: BOOP

Hemorrhagic cystitis • BK • Non‐infectious: conditioning • Diarrhea - colitis - hepatitis • Herpes viruses • Non‐infectious: GVHD • Neurologic syndromes - Herpes viruses (+HHV‐6), west nile, angio‐invasive, toxoplasmosis, PML (JCV) - Non‐infectious: PRES, antibiotics

42 y/o SCT 4 months ago multiple course of chemo for his lymphoma neutropenic for 4 months broad spectrum abx and voriconaole new lung cavitary lesion

Mucormycosis(cunninghamella, sacckina and rhizopus) - broad, no septate on GMS stain, right angle branched, variable width not fusarium

-47 y/o F -3 years of dyspnea -EF 25% -from Argentina -Impoverished Farming - Moved at age 16 Dx? TransX?

Trypanosoma Cruzi (chaga's disease) Reduvid bugs - defecating on food Mucosal transX form sugar can / acai berry juice contaminated by blenderized bugs

23 y/o nursing student 1st dose of hep B 18 month ago, 2nd dose 1 month therafter what do you do?

give 3 rd dose of HBV vaccine today hep B seorlogies not indicated, unless in geogrpahic regions >2%, MSM, PWID, immunosupprsed , liver disease all HCP should be vaccinated with 3 doses post vaccination anit-HBs>=10 (drawn 1-2 months after the last dose of vaccine)

35 y/o 68 post alogenic bone marrow trasnplant for myledysplastic, receiing meythylprednisolone for GVD of the GI , bcx growing mold?

fusarium multiforme

A 72 yr old man, status post extensive bowel resection, was receiving total parenteral nutrition through an implanted triple lumen catheter for the past 3 months, developed fever thought to be catheter acquired. However, blood cultures for aerobic and anaerobic bacteria, fungi and mycobacteria were negative. Which of the following additives to blood culture would help recover a likely pathogen: A. olive oil B. ferrous sulfate C. pyridoxal D. hydrogen gas E. carbon dioxide gas

meleassezia furfur - add oilve oil Malassezia furfur: budding yeast. Normal skin flora Catheter-sepsis in patients receiving lipid in TPN. Growth requires lipid (olive oil, Intralipid)

62 y/o M osteoarthritis 24 hr w/ f/c, odynophagia, and diarrhea sampling grape must T102.4 ill appearing, left tonsil swollen and erythametous left suppurative lymph node tender to palpations ?toxo? ?bartonellosis (cat scratch fever) ?tularemia ?EBV? ?scrofula?

oropharyngeal tularemia uncommon in the US -ingestion or rarely inhalation - inadequatly cooked game - contamined tap water (turkey) - rodent contamination of grape vines -exudative tonsilitis, ulcers, swollen LAN -dx culture (alert lab), serology -tx streptomycin and doxycline

Question 3: A 42 yo man from Bolivia presents with nasal stuffiness and is found to have nasal septal perforation. Biopsy demonstrates intracellular amastigotes consistent with Leishmania. Which is the most likely species? A. L. mexicana B. L. braziliensis C. L. peruviana D. L. infantum chagasi E. L. major

transmitted by sand flies (noiseless, active in evening Mucoutanous usually L. braziliensis L. guyanesis L. panamensis - treat systematically Amastigote Intracellular (macrophages) Round or oval Wright-Giemsa: dark-purple nucleus small rod shaped kinetoplast intracellular organisms with nucleus and kinetoplast

69 y/o M alcohalic went into shock 3 day after eating raw oyster bloody bullae on leg

vibrio

in patient with S. aureus bacteremia f/u bcx should be obtained until negative true/false

A:True

Prophylaxis to prevent an infection changed the incidence of infection from 10% to 5%. The number needed to treat (number of patients who would need to be treated to benefit one patient) would be which of the following: 40 20 15 10 5

Correct answer: 20 The number needed to treat is 1/(proportion infected without prophylaxis minus proportion infected despite prophylaxis) or 1/(0.1-0.05) or 20.

AHA 2007 guidelines on antibiotic ppx for dental extraction?

- IE felt to be more likely from frequent random bactremias than caused by dnetal, GI, GU procedures - Highest risk procedures: 1) manipulation of ginigival/periapical or perforation of the oral mucosa 2) respiratory tract incision or biopsy 3) GI procedure sin patients with active infection 4)intracardiac surgery PPx only for: 1)prosthetic cardiac valve 2)previous infectious endocarditis 3)valve regurge or abnormal valve in cardiac transplant 4) a) unrepaired cyanotic CHD (shunts / conduits) b) repaired CHD w/ prosthetic during first 6 months c) repaired CHD with defect at or adjacent to prosthetic patch or device

Ascaris lumbricoides • Large numbers of worms can cause abdominal distention and pain or intestinal obstruction • can cause "Loeffler's syndrome" - an eosinophilic pneumonitis with transient pulmonary infiltrates • cholangitis and/or pancreatitis b/c aberrant migration

- eosinophilia - migratory pulmonary infiltrates won't see eggs yet but once they do.. stool testing is good • MAJOR cause of ANEMIA and protein loss (b/c plasma loss) • pneumonitis associated with wheezing, dsypnea, dry cough (usually a few days to weeks after infection) • urticarial rash • mild abdominal pain If sensitized papulovesicular dermatitis at entry site "ground itch" If worms migrate laterally cutaneous larvae migrans (especially dog and cat hookworms, as late as 2-8 wks after exposure to A. braziliense)

• 24 yo man presents with mononucleosis-like illness • Febrile, faint rash on chest, pharyngeal edema, diffuse lymphadenopathy Exam Febrile, Skin rash, pharnygeal erythema, lymphadenopathy Lab WBC 2,300 - 43% lymphs, 12% mono, 45% polys Monospot Test: Negative QUESTION #1 Which if the following test will most likely diagnosis his condition: A. CMV Antibody Test B.Toxoplasma IgM Antibody Test C. Combination HIV-1 Antigen / Antibody Test D. HIV-1 EIA test E. Rapid Strept Test

C: combination HIV-1 antigen /ab test Testing point: Every ID physician should know how to diagnose HIV Seroconversion Syndrome

30 y/o HIV + pain on defacation bloody anal discharge anal sex one week ago empiric abx? CFX / azithro / doxy CFX/ azithro x3 weeks CFX / doxy x21 days CFX/ doxy x 7 d + oral acyclovir

CFX/ doxy x 7 d + oral acyclovir

27 y/o, 5 days ago from venezuela, touring rural sites and inland for bird watching and white water rafting 2 days of fever 102F, HA, myalgia in neck, faint macular rash over her trunk, no lymphadneopathy full immunizaiton prior to trip and taking doxycycline (for malaria) WBC 3.8, Platelets 137,000, Hg 12.3 g/dl Bili, Alkaline phosphatase, ALT and AST normal Chest X-ray: normal Malaria smear: pending cause of syndrome? Plasmodium vivax Leptospirosis Dengue Toxoplasmosis Scrub typhus (Orientia tsutsugamuchi)

Dengue could be caused by a variety of pathogens, but is perfectly compatible with Dengue. in the Americas incubation period (3-14 days) clue: taking doxycycline prevent leptospirosis, malaria, and scrub typhus. Note that there is NO scrub typhus in the Americas: this occurs in Asia and Oceana. In the Americas, for Rickettsial diseases, think more of murine typhus (Rickettsia typhi), or perhaps R. prowakezii (Epidemic typhus usually spread by lice), R. parkeri (spread by Amblyoma ticks), cat flea rickettsiosis (R. felis) or Rocky Mountain spotted fever (R. rickettsia spread by Dermacentor ticks). Especially with water exposure, this case could be leptospirosis except that doxycycline prophylaxis should have been protective. This syndrome could be malaria although the rash would push towards other diagnoses, and the doxycycline should have offered a high degree of protection. Note that the rash could be due to photosensitivity if it were only in sun-exposed areas, but in this case it occurred long after the doxycycline and sun exposure first occurred. Toxoplasmosis can be acquired worldwide from exposure to feline feces or to raw meat: there have been interesting cases associated with drinking water from streams infected with feces. However, the cytopenias here are more suggestive of a viral process, and there is no characteristic posterior auricular or cervical adenopathy, and fatigue should be more prominent than myalgias.

A woman from China in her 40s developed fever, epigastric pain, and nausea. One week later, she was brought to ED with confusion and fever. T 101°F. Right upper quadrant abdominal tenderness. Abdomen CT: 10 cm hypoattenuated liver lesion Aspirate on petri dish shows "string test" A:Entamoeba histolytica B. E. coli C. Streptococcus milleri D. Actinomyces E. Klebsiella pneumoniae

Klebsiella liver abscess Hypermucoid strain of Klebsiella pneumoniae associated with a distinctive clinical syndrome in Southeast Asia that includes primary liver abscess, bacteremia, and metastatic infection Risk factors: diabetes and Asian ancestry Colonies exhibit extreme "stickiness" on agar plates ("hypermucoviscosity phenotype") Positive String test: "string" of > 5 millimeters formed when one touches the colony

Clonorchis sinensis "Chinese Liver Fluke" • eggssnailsfreshwater fish • Acquisition by ingestion of undercooked fish • Flukes develop in duodenum then migrate to liver bile ducts • Can live for 50 years, making 2000 eggs/day • biliary obstruction • cholelithiasis • cholangiocarcinoma

Opisthorchis viverrini Southeast Asian Liver Fluke" • similar lifecycle • also acquired by eating fish • cholangiocarcinoma

40 y/o M with ESRD migrated from brazil to US, underwent cadaveric renal transplant Hx of recurrent epigatric pain with negativ endoscopy, WBC 6.5 K with 15% eosinophills after tranplant on pred/azathioprine, did well for 1 month, then presents fever 104, HA, AMS, and meningimus, hypoxic, diffuse interstitial infiltrates CSF WBC 2500, 98% neut, glucose 20, protien 450 CSF culture grew Ecoli, UCx no growth, chemistries normal ? diagnositc test CT scan of head and sinuses CT with oral contrast of the abdomen Serial stool examinations Fungal serology Urine exam for ova

Serial stool examinations E. coli bacteremia ordinarily urinary, biliary or gastrointestinal tract. Renal transplants prone urinary tract infections but urine culture negative. no cholangitis or diverticulitis. rare route of Gram negative meningitis is anterior sacral meningocele with an enterothecal fistula, an anomaly that could be suspected by presence of a "scimitar sacrum" on CT. more plausible route from the colon to the blood and CSF is through larvae migrating across the colonic mucosa. patient has hyperinfection syndrome, disseminated strongyloidiasis, immune suppression,with corticosteroids, followed by gram negative sepsis and meningitis is a typical presentation for Strongyloides hyperinfection. normal conditions, Strongyloides infection not involve the central nervous system, but hyperinfection has become more common as a result of widespread use of immunosuppressive drugs. uncontrolled infections, the filariform larvae will penetrate organs not normally involved in the life cycle, including the brain. carry organisms such as Escherichia coli, Klebsiella, Serratia, and enterococci in piggyback fashion on their surface as they exit the intestine, or the larvae may carry the enteric organisms within their own gastrointestinal tracts. Alternatively, the numerous Strongyloides larvae penetrating the intestinal tract during hyperinfection may predispose to gram negative bacteremia simply by disrupting the integrity of the intestinal epithelium The fact that larvae are seldom found within the central nervous system, even when bacterial meningitis has occurred, suggests that the meningitis can be secondary to seeding during bacteremia rather than secondary to direct invasion, although larvae occasionally have been found in the meninges. Definitive diagnosis, demonstration of Strongyloides stercoralis larvae in the feces or duodenal fluid; diagnostic sensitivity ~100% if seven serial stool samples are studied. also found in bronchoalveolar lavage in this patient Serology is not as useful: a negative serology would make chronic strongyloides unlikely, but a positive would not discern whether this acute syndrome is related to strongyloidiasis.

64 y/o M w/ DM 2 months, +fever, sweats, wt loss, cough 95% on RA CXR RUL w/ cavity LFT, Cr wnl, HIV - SputX + AFB GeneXpert MTB/RIF = MTB detected, no rif resistance Next step?

Start tx with once daily rifampin, isoniazid, pyrazinamide, and ethambutol *Having a positive GeneXpert MTB/RIF is enough for diagnosis of TB nowadays If rifapmin resistant can use Moxi/Ison/Pyraz/Amikacin Daily treatment preferred over biweekly

60 y/o M wiith CLL + fever, HA, ataxia, AMS travel to outdoor family picnic in rural virginia allergic to PCN has listeria meningitis what to use?

TMP-SMX for listeria

Which of the following results would indicate the highest likelihood of maraviroc activity in the regimen? A. Pure R5 virus B. Pure X4 virus C. Mixture of R5 and X4 viruses D. Dual Tropic (R5/X4) virus

Testing point: Every ID physician should know that maraviroc selectively blocks R5 virus and does not have activity against any X4 viruses

disease most associated with sweet's syndrome?

acute luekemia

27 y/o F to ED presents with fever and painless wrist ulcer healthy "spider bite" size of pencill, ulcerated, enlarged and a scab , fever for the past 2 days exposded to drum

cutanous antrhax

35 yr male 68 days post allogeneic bone marrow transplantation for myelodysplastic syndrome, receiving methylprednisolone 500 mg for Grade III GVHD of the gastrointestinal tract developed fever, several painful, red skin nodules and a blood culture growing a mold. The most likely fungus is which of the following: A. Scedosporium apiospermum (Pseudallescheria boydii) B. Scedosporium (Lomentospora) prolificans C. Apophysomyces elegans D. Fusarium multiforme E. Alternaria alternata

fusarium multiforme - blood cultures growing mold

a young man oral and genital ulcers suspect behcet disease ? diagnosis?

pustule at site of venipucture

slowly enlarging ulcerated skin lesion his shin after being hit by a soccer ball

pyoderma gangrenosum

19 y/o girl from dairy farm near fredrick MD sudden onset in july of fever, severe, HA, N/v, muscle aches one the 4 day developed the rash shown here

ricketssia, ricketssia rocky mountain fever give doxycyline give CFX ? not meningtitis as rash would develop right away

35 y/o diarrhea + bloody while snorkeling with family in cozumel mexico What is the Preferred Treatment for This Patient with Dysentery? A. Azithromycin 1,000 mg B. Ciprofloxacin 500 mg twice daily X 3 d C. Levofloxacin 500 mg D. Rifaximin 200 mg three times/d for 3 days E. Oral fluids only

shigellosis can be self limited, used of antibiotics decrease the duration and decrease the spread of organisms, very low inoculum abx therapy is always indicated w/ shigellosis excelling concentration in the upper GI tract A: azithromycin 1000 mg resistant to flouroquinolones are devloping, particulary from asia

You are consulted to see a 26-year-old man who was admitted yesterday with fever, headache, and a mildly stiff neck. In the emergency room, he had an LP and then was immediately given ceftriaxone, vancomycin and dexamethasone for suspected bacterial meningitis. The CSF gram stain was negative; there were 128 white blood cells of which half were neutrophils and half were mononuclear cells; glucose was normal; protein was 72. Today, blood and CSF cultures were reported to be negative; the patient was not clinically improved. The patient came to the United States from Japan a few months ago to attend graduate school. On your exam, you note three painful aphthous ulcers on the soft palate, and the patient says he has had intermittent ulcers in his mouth for at least four years. He also reports that he has had two episodes of painful penile ulcers but is quick to deny any sexual contact. You also note a 7mm pustular lesion in the left antecubital fossa, and the patient says it developed overnight at the site of venipuncture performed in the emergency room yesterday. His illness is most likely to respond to which one of the following? Ribavirin Acyclovir Ganciclovir Colchicine Ibuprofen

Correct Answer: Colchicine Behçet's Disease, a multisystem inflammatory disease characterized by recurrent painful aphthous oral ulcers, recurrent painful genital ulcers, and eye or skin lesions Pathergy, development of a red papule or pustule more than one-half centimeter in diameter is often seen one to two days after skin trauma, such as a needle stick. Behçet's is particularly prevalent in persons of Asian or Eastern Mediterranean descent. Aseptic meningitis, as in this patient, may also occur as can GI symptoms. Colchicine has long been the drug of choice for Behçet's Disease, although is best studied for mucocutaneous disease. Other agents that have been used for treatment include dapsone, corticosteroids, thalidomide, and immunosuppressive agents. Correct Response

A 70 year-old female in excellent health (HIV negative) had 3-day history of stabbing pain and itching in her right ear. On the second day of the pain she developed pain in her right jaw, spreading to her right cheek. Chewing caused some pain in the right cheek and she had some difficulty swallowing. On exam, she was afebrile, alert and oriented and had erythematous vesicles and moistness in the right external auditory canal plus some flattening of the nasolabial fold and lesser smile on the right. The tympanic membrane could not be visualized because of edema in the canal lining and pain on otoscopy. What diagnostic test on the ear exudate is most likely to yield the correct diagnosis: Routine bacterial culture of the right ear exudateMycobacterial smearPCR for HSVPCR for VZVFungal culture

Correct Answer: PCR of ear lesion swab for VZV This patient has Ramsay Hunt syndrome (Herpes zoster oticus), caused by VZV reactivation in the geniculate ganglion of cranial nerve VII, i.e. this is essentially zoster of CN VII. Thus, the correct answer is PCR of the exudate for VZV. Diagnosis is best established by PCR of lesion exudate. Very few labs do viral culture in this era, due to cost, but the diagnosis could be established by culture or by DFA. Ramsay Hunt can present with severe ear pain and reduced hearing or deafness. When vesicle are seen in the auditory canal or auricle, there are commonly abnormalities in cranial nerves VII, and sometimes VIII, IX or X. Acyclovir is usually recommended although it's not clear if it's effective. The facial paralysis is more severe and less likely to resolve than the usual HSV related Bells Palsy. Tuberculosis of the middle ear could present with loss of auditory function and exudate; however, the pain and ear lesions do not fit TB otitis. Fungi can be cultured from the middle ear but are rarely the cause of disease in immunologically normal hosts. The vesicles of HSV would have a similar appearance, but the distribution is much more typical Ramsey Hunt syndrome.

You are consulted about an outbreak of conjunctivitis. In late August, in a period of 10 days, 15 of 30 college students attending a summer French language camp in upstate New York developed eye pain and redness and preauricular adenopathy. Exams are notable for keratoconjunctivitis without purulent exudate or fever. All the students swim in the camp lake on a daily basis and eat meals together in a dining hall. The camp cat has recently had a litter and all the affected students say that they have played with the kittens. Which one of the following is the most likely cause of the outbreak? Enterovirus Adenovirus Bartonella Acanthamoeba Pneumococcus

Correct answer: Adenovirus Adenoviruses are the most common cause of epidemic keratoconjunctivitis, a syndrome characterized by eye pain and inflammation, fever, and preauricular lymphadenopathy. Outbreaks at camps are common. Enteroviruses are a common cause of usually painless conjunctivitis and can cause similar outbreaks but outbreaks in young adults are much less common than adenoviruses. Bartonella (cat scratch disease) can cause conjunctivitis and preauricular adenopathy, but cases are sporadic and do not occur as outbreaks. Acanthamoeba can cause keratitis in those who swim in lakes, but again, cases are sporadic. Pneumococcus has caused outbreaks of conjunctivitis on college campuses; conjunctivitis is typically purulent and cases are much less common than those due to adenoviruses.

During an ongoing outbreak of Legionnaire's Disease at a nursing home, an 88-year-old resident of the nursing home is hospitalized for lobar pneumonia and respiratory distress. Her roommate at the nursing home was definitively diagnosed with Legionnaire's disease three days earlier. Which one of the following is the most appropriate isolation procedure for the newly ill 88-year-old?Standard precautionsDroplet precautionsContact precautionsAirborne precautions

Correct answer: Standard precautions Legionella is not transmitted person-to-person so droplet or special contact or airborne isolation precautions are not necessary for this patient.

A 46-year-old woman is seen for fever and a rash. She was recently diagnosed with acute myelocytic leukemia. Following induction chemotherapy, she developed neutropenic fever that responded to antibiotics. Her marrow recovered, and she was well at home until four days ago when she developed fever with no localizing symptoms. An outpatient CBC showed a white blood cell count of 12,250. She was admitted, blood cultures were sent and she was started on broad spectrum antibiotics. Other than fever and tachycardia, her exam was unremarkable. On the second hospital day, she developed the acute onset of a rash consisting of multiple red to red-purple tender, non-pruritic papules predominantly on her face, but also involving her neck and the dorsum of her hands. Blood cultures were negative. Biopsy of two of the skin lesions was performed. Despite antibiotics her fever continued; some of the papules turned into plaque-like lesions with a central yellow discoloration. Today, the biopsy was reported as showing dense dermal perivascular infiltrates of neutrophils without evidence of vasculitis; stains for organisms were negative. Which one of the following is the most likely diagnosis? Ecthyma gangrenosum Pyoderma gangrenosum Sweet Syndrome Leukemic infiltrates Staphylococcal folliculitis

Correct answer: Sweet Syndrome acute neutrophilic dermatosis Sweet Syndrome. acute myelocytic leukemia Inflammatory bowel disease abruptly as painless papules or plaques, and are most often found on the face and extremities, particularly the dorsum of the hands. Biopsy shows neutrophilic dermal infiltrates without vasculitis. Most patients have antecedent fever. Treatment is corticosteroids. Ecthyma gangrenosumis a necrotic skin lesion associated with Pseudomonas or mold infections in neutropenic patients. This patient was no longer neutropenic. Pyoderma gangrenosumis associated with inflammatory bowel disease, and the lesions are ulcerative with undermined borders. Leukemic infiltrateswould be very unlikely given the good response to chemotherapy and the absence of blasts on CBC. Staphylococcal folliculitisis ruled out by the absence of organisms on biopsy stains

A 24-year-old woman has just sustained a puncture wound of her foot while walking barefoot in her backyard. She had full primary tetanus immunization in childhood, and her last "tetanus shot" was seven years ago. She is not aware of any other vaccinations since childhood other than influenza immunization. She has a six months old child at home. Which one of the following should be given to this patient? TdaP Tetanus immune globulin Tetanus toxoid and tetanus immune globulin Tetanus toxoid Td

Correct answer: Tdap. This patient has a tetanus prone wound, and she should get tetanus prophylaxis. Since she has had full primary immunization for tetanus, she doesn't need tetanus immune globulin. Td, with protection against diphtheria as well as tetanus, is recommended for adults receiving prophylaxis after a wound. There is concern about waning immunity of adults for pertussis. Infants are a special concern as well since there have been infant deaths due to pertussis during recent outbreaks. Mothers of children less than one year of age particularly need to be protected so that they do not transmit this infection to their infant. Because this mother did not receive Tdap with her recent pregnancy, she should receive Tdap now.

A 22-year-old man presents with fever, mouth pain, and skin rash. PE reveals 3 small oral ulcers and diffuse macular rash. Labs show WBC 3, plts 89K, monospot neg., RPR NR, HIV antibody negative, HIV RNA 1,876,000 cps/ml. Which statement is correct? A. ART should not be offered. B. ART would decrease his symptoms. C. ART has long-term virologic benefits in this setting. D. ART has long-term clinical benefits in this setting.

A: ART would decrease his symptoms - also reduce risk of transmissions • ART reduces symptoms and signs. • No long-term virologic, immunologic, or clinical data available. • If ART is started, use standard regimens with goal of full virologic suppression. • Obtain genotype prior to ART. • If ART is started prior to genotype results, use a PI-based or dolutegravir-based, rather than an NNRTI-based, regimen.

An 18 year old high school student develops chills, fever, cough, myalgia in January. She is prescribed azithromycin, rest and NSAIDS. Fever and cough continue and she becomes progressively dyspneic and weak. On admission T 39, P 150, RR 24-30, BP 120/50. She has crackles throughout both bases and a gallop. Influenza PCR positive • WBC =9000/mm3 (60% polys, 30% bands) • Creatinine 1.9 • BNP markedly elevated • CXR shows diffuse bilateral infiltrates and cardiomegaly What is the most likely cause of this influenza complication?: A. Pneumococcal pneumonia B. Staph aureus pneumonia with purulent pericarditis C. Influenza cardiomyopathy D. Pulmonary embolus due to hypercoagulable state E. Viral pericarditis with effusion

C - influneza cardiomyopathy influenza cardiomyopathy reyes syndrome usually in children - AMS, seizures, abn LFT's, high ammonia

34 y/o F, nurse, born in pakistan, in US for 5 years, hsa Crohn's on azathioprine and infliximab for 6 months 3 months of HA's, confusion, sweats, weight loss + nuchal rigidity + somnolent MRI leptomeningea enhancement and basal cistern involvement, no parenchymal or temporal lobe CSF fluid: GLucose 30 (serum 90), Protein 110 450 WBC (95%lymph) CSF GS -, fungal stain -, auramin stian for AFB -, crytpococcal Ag - 48 hours CSF bacterial, fungal, and mycobacterial cx negative Most diagnostic procedure to yield diagnosis ? Serum Toxo IgM? IgG? CSF mycobacterial cx CSF HSV PCR Urine Histoplasma Ag

Most sensitive microbiological test in CSF = Culture Duration and no MRI finding = not HSV Toxoplasma = parenchymal lesions No epidemeology for CSF histoplasma

Woman in her 50s presented with fatigue, confusion, word-finding difficulties and fever for 3 days Lived in Midwestern US. Avid outdoors person, frequently in wooded areas; husband recalls pulling a tick off her trunk recently T 101.3. Somnolent woman, oriented only to self CSF: WBC 146 (9% N, 56% L, 35% M); RBC 14; Glc 70; Pro 109 MRI: T2 hyperintensity left thalamus and substantia nigra; leptomeningeal enhancement A:Neisseria meningitides meningitis B. Herpes simplex virus encephalitis C. Lyme meningoencephalitis D. Powassan meningoencephalitis E. Lymphocytic choriomeningitis

CSF gram stain, fungal smear, bacterial and fungal cultures were negative CSF PCR tests for HSV, WNV, VZV, CMV negative CSF positive for immunoglobulin M against Powassan virus by ELISA. Confirmed at CDC Powassan Encephalitis Transmitted by Ixodid ticks Northeast, upper Midwestern (Great Lakes) US Transmission period April-December Incubation period up to 4 weeks Fever, confusion, seizures, focal neurologic deficits CSF: lymphocytic pleocytosis Diagnosis: MRI: T2 hyperintensity in thalamus, basal ganglia, brainstem Positive IgM antibody; confirmed at CDC with PRNT

60 yo M with L4-L5 and L5-S1 diskectomy for disc herniation in remote past and benign prostatic hypertrophy presented with a 6- month history of fever, chills and headache. He subsequently developed lower back pain, which worsened until it awoke him from sleep. Also noted anorexia and 12 lb weight loss. Patient was from Middle East (an urban area). He had eaten cheeses while in Europe several months earlier. He denied animal exposure and known TB exposure. On exam, he had point tenderness over lumbar spine with associated paravertebral muscle spasm. WBC 5.1. ESR 80. Blood cultures sterile. MRI: high signal intensity L4/5 vertebrae and disc space Pathology: epithelioid histiocytes, intracellular organisms A"E. coli vertebral osteomyelitis (VOM) B. Tuberculous VOM C. Staphylococcal VOM D. Brucella VOM E. Fungal VOM

Culture and serology positive for Brucella melitensis Prevalent in Mediterranean, Arabian peninsula, South Asia, Central and South America Although a systemic infection, localized osteoarticular complications may occur Lumbosacral spondylitis & sacroiliitis are the most common sites of osteoarticular disease Small gray non-pigmented Urease positive non-hemolytic colonies on Brucella agar with horse blood. Gram stain: gram-negative coccobacilli.

45 y.o.female alcoholic presents with an acute pneumonia during the winter months Respiratory PCR panel detects corona virus Sputum culture positive for Pseudomonas aeruginosa Lab reports evidence for production of ESBLs and metallocarbapenemases In vitro resistance to all tested antibiotics except the polymyxins Which one of the following treatment regimens would you recommend ? A. Polymyxin B alone? B. Polymyxin E (colistin) plus Meropenem ? C. Aztreonam alone ? D. Aztreonam + Ceftazidime-avibactam ?

Options: IF resistance is not due to beta-lactamase and likely due to porin closure and/or efflux pump, try ceftolozane-tazobactam If resistance is due to metallo-carbapenemase: Ceftazidimeavibactam plus aztreonam Potential Salvage Therapy : Pipeline drugs---aztreonam-avibactam, cefiderocol Monoclonal antibodies Bacteriophages

45 yr old male 6 weeks post stem cell transplant for myelodysplasia, with a history of chronic hepatitis C was discharged home to Florida on cyclosporine, mycophenylate, prednisone , Bactrim (tmp/smz), citalopram and voriconazole. Diffuse nonpruritic erythema developed over his sun exposed skin. The most probable cause was: A. porphyria cutanea tarda B. graft versus host disease C. drug interaction D. voriconazole E. Bactrim allergy

Photosensitivity, premature skin aging on solar exposure and squamous cell carcinoma can occur from prolonged voriconazole use. Graft versus host disease and drug allergy would not be confined to sun exposed areas. Hepatitis C increases incidence of porphyria cutanea tarda but lack of blistering, new onset and no precipitating events is against dx. Drug interaction with voriconazole can increase cyclosporine level but not cause solar hypersensitivity

55 year old man presents with complaints of fever / volume depletion HIV diagnosed in ER on rapid test Lymphadenopathy / splenomegaly / few petechiae / Oriented X 3 HIV RNA 340,000; CD4= 3 cells/ul On no medications Hb 8.2 gm/dl; Plt count 21,000; Creatinine 2.0 Rare schizocytes on peripheral blood smear Which of the following is the most effective intervention to increase the platelet count? A. Splenectomy B. Corticosteroids C. Plasmapheresis D. Ethambutol + Azithromycin E. Antiretroviral Therapy

-viral load really high -coats things, also platelets as well - plt sequastered into the spleen A. Splenectomy: Rarely needed for HIV ITP B. Corticosteroids: Will work, but not needed C. Plasmaphoresis: This is not TTP. Pentad not complete; rare schiztocytes D. Ethambutol + Azithromycin: He could well have MAC, but need BM bx to show pdtion pblm E. Antiretroviral Therapy: HIV coats platelets, attract AntiHIV Ab, leads to removal by spleen. ARV Rx lowers VL, plts rise to near normal levels

• A 71 yo man with COPD, history of MI is admitted in January with progressive dyspnea, cough, tachypnea, low grade fever. ROS is positive for rhinitis. • He has been spending time with young grandchild who has bronchiolitis • CXR shows bilateral perihilar infiltrates consistent with pneumonia.. The recommended strategy, pending more lab results, regarding isolation should be: A. Put him in a regular two bedded room with standard precautions B. Put him in a single room with standard precautions C. Put him in a single room with contact/droplet precautions D. Put him in an airborne isolation room with airborne isolation • Multiplex PCR of his nasal swab shows RSV. Which of the following is correct A. RSV is an incidental finding which might cause URI symptoms B. RSV likely accounts for infiltrate. He should be immediately started on palivizumab (Synagis) and ribavirin C. RSV likely accounts for infiltrate. Supportive care is appropriate D. He has high risk CAP and should be started on vancomycin and cefepime

1st Answer C 2nd Answer C • Exposure to children probably a risk factor • Nonsegmented, single stranded, negative sense RNA virus • Most common cause of LRTI in children • Common cause of URI with rhinitis in adults. AE-COPD, worsened CHF, asthma exacerbation and pneumonia in elderly and immunocompromised • Transmitted by large droplet and contact; nosocomial transmission in hospitals and ECF • Late fall to spring (usually December- April) • As common as influenza among hospitalized persons > 65

appropriate duration of therapy for S. aureus bactermeia? 4, 6, 8, or 16 weeks

8 weeks risk factor associated with recurrenc: ESRD MRSA and underained paravetebral/psoas abcess if MSSA 6 week if no abscess or undrained abscess 8 week if undrained abscess, esrd MRSA or ESRD at least 8 weeks

32 y/o M +penile discharge GS of discharge = intracullular GN Diplococci Allergy to PCN and cephalosporin CDC recommnds? Azithro Azithro + CFX Azithro + Gent Cipro Spectinomycin

? 2g azithrox1 + 5 mg/kg gentamicin IM when you can't use ceftriaxone azithromycin 2gx1, can't use due to resistance to macrolides

A 24-year-old man is referred for the management of Hepatitis B infection after his primary care physician discovered it incidentally during a routine visit. He is asymptomatic and feels well. He has no other medical problems and does not take any medications. He denies tobacco, alcohol, or any illicit drug use. His family history is remarkable for HTN in his mother. His physical examination is normal. His diagnostic results include the following: HBsAg + HBeAg + HBe Ab - Anti-HBs Ab - HAV IgG + HIV - Liver Biopsy: Mild inflammation, no viral inclusion bodies, negative immunofluorescent stains for hepatitis viruses Chemistries ALT 2X upper limit of normal AST Normal Given the data above, which of the following parameters would provide the strongest indication to initiate antiviral therapy in this patient? AgeHBeAg positiveGenotype AHBV DNA > 20,000 IU/mlMale sex

A 24-year-old man is referred for the management of Hepatitis B infection after his primary care physician discovered it incidentally during a routine visit. He is asymptomatic and feels well. He has no other medical problems and does not take any medications. He denies tobacco, alcohol, or any illicit drug use. His family history is remarkable for HTN in his mother. His physical examination is normal. His diagnostic results include the following: HBsAg + HBeAg + HBe Ab - Anti-HBs Ab - HAV IgG + HIV - Liver Biopsy: Mild inflammation, no viral inclusion bodies, negative immunofluorescent stains for hepatitis viruses Chemistries ALT 2X upper limit of normal AST Normal Given the data above, which of the following parameters would provide the strongest indication to initiate antiviral therapy in this patient? AgeHBeAg positiveGenotype AHBV DNA > 20,000 IU/mlMale sex Feedback Correct Answer: HBV vIral load This is a young patient who has active viral replication but no evidence of necroinflammation on liver biopsy despite an elevated ALT. Because he is HBeAg positive, his HBV viral load is likely to be high. According to guidelines, he should be treated if his HBV viral load exceeds 20,000 IU/ml and there is necroinflammation, which can be identified from either elevated ALT or liver biopsy. The standard therapy would be monotherapy with Tenofovir or entecavir in an HIV negative individual. Treatment of high DNA levels in the absence of necroinflammation in this young patient is not proven to improve outcomes. Gender is not an indicator to initiate therapy.

39 yr old man with severe aplastic anemia and absolute neutrophil count of 25/cu mm developed the sudden onset of fever and pulmonary infiltrates not responding to five days of ceftazidime. The CT is shown in the next slide. The most likely cause of his fever is which of following: A. Mulch in his garden B. Spray from the air conditioner water tower C. Pigeon droppings near the air conditioner inlet D. Visitor with a cough E. Reactivation of a prior infection

A. mulch in his garden (aspergillosis) B. Spray from the air conditioner water tower - legionella, not that kinda pulmonary infiltrate C. Pigeon droppings near the air conditioner inlet histoplasma - too rapid D. Visitor with a cough E. Reactivation of a prior infection

a foodborn outbreak occured among 100 school chilrden and teacher after a special luncheon incubation period 28 hrs vomitting seen in 70% diarrhea in 50% fever in 30% recovery occured in 12-60 bug? What is the likely cause of the outbreak? A. Norovirus B. Shigella sonnei C. Enterotoxin from Staphylococus aureus D. Clostridium perfringens E. Bacillus cereus

A. norovirus other organisms except enterotoxicc s. aureus and B. cereus don't hvae diarrhea

A 45 yr old Vietnamese business man came to the US to seek medical attention for an illness of 4 weeks duration, with low grade fever, weight loss, anorexia and the recent appearance of painless skin lesions. Biopsy of the skin lesions show is shown to the right. Which of the following is most likely: A. Talaromyces marneffei B. Histoplasma duboisii C. Fusarium oxysporum D. Cryptococcus gattii E. Paracoccidioides brasiliensis

A. talaromyces marneffei

31 y/o M, first grade school teacher +Fever, rhinorrhea, and malaise x days worsening cough, sick for 12 days cxr normal 5 days of azithro started on day 3 ( now completed 4 days ago) continues to cough nasopharyngeal swab = bordetella pertussis PCR + has received immunizaiton

A: All his household contact regardless o age and vaccine status should receive prophylaxis (5 days of azithromycin) 25% of pertusis are in adults immunity wanes, despite vaccination children should be vaccination

33 y/o M w/ AML s/p SCT 4 weeks ago (on cyclosporine and prednisone for GVHD) received acyclovir, and aerosolized pentamiidine on 21 day post transplant, neutrophils is 5K nodular infiltrates on chest xray BAL shows septated hyphae Voriconazole IV started renal and hepatic function normal if not dose adjustment, ? Subtherapeutic cyclosporine levels Subtherapeutic voriconazole levels Agitation or seizures due to supertherapeutic acyclovir levels Azotemia, headaches or seizures due to super-therapeutic cyclosporine levels Fulminant hepatic failure due to elevated (supertherapeutic) voriconazole levels

A: Azotemia, headaches or seizures due to super-therapeutic cyclosporine levels This patient is likely to get cyclosporine toxicity due to rising cyclosporine levels if his cyclosporine dose is not reduced when voriconazole is started. A 50% reduction in cyclosporine (and a 60% reduction in tacrolimus) would be in order, with subsequent close monitoring of cyclosporine (or tacrolimus) serum levels to be certain the modeling is correct and other interactions/absorption issues do not alter the kinetics. The first choice is incorrect because cyclosporine levels would not be reduced, they would be increased by voriconazole. Voriconazole levels are not reduced by cyclosporine, and thus the second choice is incorrect. Acyclovir is not metabolized in the liver. No interaction is expected with acyclovir or valacyclovir. ID physicians need to know that cyclosporine and tacrolimus have similar toxicities, most of which are dose related. These include renal (azotemia), neurologic (headaches, tremors, seizures, mutism) and hypertension. The fourth choice is correct because voriconazole in this patient would lead to elevated cyclosporine levels, and likely produce cyclosporine toxicity. Voriconazole can produce transaminase elevations, but this patient's voriconazole levels have no reason to be elevated. Thus the final choice is not the best answer.

72 y/o M in late august F, C, weaknes, 1 week ealrier, on DOA becomes confused in NJ, has horse farm, often mosquito and tick bites he is somnolent, denies HA stiffness T 103.1 no rash, neck suppple, no adenopathy, lung clear, heart w/o murmurs, abdomen normal neurologic exam, is oriented to person only, CN intact, motor strength 4/5 UE, and 3,5 LLE, and 2/5 RLE, sensation intact test to establish etiology? serum IgM, serum PCR, CSF IgM, CSF PCR, brain MRI

A: CSF IgM

34 y/o F w/ crohn's disease hospitalized after 3 months of HA and confusion, + sweats/wt loss born in pakistan nurse, immigrated 5 years ago on azathioprine and infliximab infusions 6 months ago +mild nuchal rigidity MRI shows leptomeningeal enhnancement with involement of the basal cistern, no parenchymal lesions, temporal lobes normal CSF fluid glucose 30, protien 110, 450 WBC 95%lymph Cx negative for bacteria fungal stain auramine stian negative for AFB , cyrptocccal ag negative Cx negative thus far. what's to yeild most?

A: CSF mycobacterial culture Most sensitive and most specific ?HSV, duration too long, MRI does not show temporal lobe Toxo, no parenchymal lesion Histo, no epidemeology

anthrax spores at GIant game ? true? A radio announcement should be made instructing all those at the game, and in the surrounding area, to go to their physician for a nasal swab to determine if they were exposed. In vitro antibiotic sensitivities are not yet available, but, since the first line prophylactic agents, ciprofloxacin and doxycycline, are contraindicated in pregnancy you should recommend amoxicillin to all women who are, or may be, pregnant. The spores on the grassy field will be viable for years. Ciprofloxacin or doxycycline should be given to all who were at the stadium

A: Ciprofloxacin or doxycycline should be given to all who were at the stadium Nasal swabs notindicated, all present in the stadium are presumed exposed to anthrax spores regardless of what results of a nasal swab reveal. Although contraindicated in pregnancy and in children, both ciprofloxacin and doxycycline are recommended for pregnant women and children for anthrax post exposure prophylaxis. When the sensitivity profile of the strain is known, substitution with penicillins, macrolides or other agents may be done in pregnancy and children. Since the exposure is outdoors, it is likely that most of the spores will be destroyed by UV light and would not remain viable for years. Although not FDA approved for anthrax post exposure prophylaxis, other available quinolones appear to be at least as active against B. anthracis as ciprofloxacin and could be substituted.

Diarrea outbreak among nuclear plant workers in argentina after workers attended a banquet incubation 10 hours +watery diarreha w/o blood NO vomitting, NO fever Pathogen?

A: Clostridium perfringens food poisoning -incubation of 8-12 hours (longer than S. aureus) - short incubation, suggest performed toxin - norovirus gastroenteritis, has vomitting in 50%, incubation 24 -48 hours -S. aureus, incubation 2-7 hours, + vomitting - Shigella and salmonella, longer incubation (>14 hrs) + dysentery and fever

organism found in sputum of a 62 y/o vietnamese government official +hemoptysis and cough for 1 year (picture of an egg) ? original source? Human stool Fish Crabs Walking barefoot on soil Pork

A: Crabs The egg is from the lung fluke, Paragonimus westermani, acquired from ingesting Paragonimus eggs contaminating pickled or inadequately cooked crab or crayfish in the Far East. In the chronic phase, multiple pulmonary lesions appear and may cavitate, causing hemoptysis. Radiologically, the lesions must be distinguished from tuberculosis.

24 y/o 33 week gestation +n/V and RUQ pain takes acetaminophen 1gm q46h, has a dog and bird, recent visit to NC 2/6 SEM; RUQ tendr, no rash Plt 103, HCt 26, WBC 6.6 10% L, Cr 0.6, ALT 225, AST 559, CRp 15.8, fibrinogen NL dx? HELLP / acute fatty liver/HAV / HSV / HEV

A: HELLP

24 y/o 33 week gestatin with N/V and RUQ pain taking acetaminophen 1 g q4-6, has dog and bird, recent visit to mom in NC RUQ tender, no rash, 2/6 SEM Plt 103, Hct 26, WBC 6.6 10% lymph, G 85, Cr0.6 ALT 225, AST 559, Tb 1.4, Pt wnl, fibrinogen nl best diagnosis? HELLP Acute fatty liver of pregnancy HAV infection HSV infection HEV

A: HELLP most importantly think: HSV , 50% mucocutanous lesions, 50% no rash, blood HSV PCR HELLP - hemolysis and elevated liver enzyme - HTN, can occur post partum - fibrinogen high vs. sepsis and AFLP AFLP - sever and low glucose, inc INR, low fibroinogen - can occur after deliver, delivery usually the curative approach HEV - exposure

24 y/o M high-risk sexual exposure, 6 days ago 3 months ago and 6 months ago and seronegative test for earliest evidence of HIV?

A: HIV RNA Viral load signs and symptoms of acute HIV infection appear 6 to 56 days after exposure Viremia can be detected as early as 4 days post exposure by RT-PCR of serum. RNA test becomes positive approximately 5 days before the p24 antigen, and 1 to 3 weeks prior to the appearance of measurable antibody formation HIV 1/2 serologies detect both IgM and IgG antibody but antibodies typically appear 22 to 27 days post infection Viral loads typically exceed one million copies/mL at its peak. Viral loads less than 1000 copies/mL should be viewed with suspicion as possible false positives in the setting of a seronegative individua CD4 enumeration is neither sensitive nor specific for early diagnosis of the acute retroviral infection Post exposure prophylaxis with antiretroviral therapy should be a consideration for this patient, though he is beyond the 72 hour window when prophylaxis is likely the most effective, based on animal studies.

51 black man from southern alabama W/ fever weakness and somnolence, and skin rash febrile raised psorisiform, scaly rash on his arms, lower legs diffuse lymphadenopathy WBC 11,900 - 73% lymphs (several w/ cerebroform nuclei) 24% polys, 3% monos Ca 14.3 flow cytometry reveals DDx?

A: HLTV-1 EIA test

37 y/o M, from philidelphia, on national gaurd duty for 6 weeks along adriatic coast in Serbia, + exposure to ticks and rodents 3 days of fever, myalgia, N/V, HA, drowsiness immediatly after returning WBC 1.6, Plt 60, transaminases 2x normal, Bili 2.0, Cr 2.6 Diarrehe worsen, HA --> delerium, PT/PTT rose to 5x , Plt fell to 8, CT showed hepatomegaly, bowel edema, splenomegaly and ascites not on any medicaitons, after 72 hours BCx, CSF and ascites negative, HIV ab negative ?viral disease? Lassa virus Enterovirus D68 Hantavirus Dengue virus Marburg virus

A: Hantavirus acquired in Eastern Europe from rodent exposure. febrile illness with cytopenias and coagulopathy, viral hemorrhagic fevers often overlooked. short incubations periods (most are one to two weeks but Hantaviruses can be five weeks); geographic niches (Ebola and Marburg and Lassa in Africa, Dengue in Asia, Africa, the Pacific and the Americas, but the Hantaviruses are worldwide); transmitted by bats (Ebola and Marburg), others by rodents (Lassa and Hanta) and some by ticks. The epidemiology is most consistent with Hemorrhagic Fever with Renal Syndrome (HFRS) which occurs worldwide and is caused by hantavirus, members of the Bunyaviruses, which occur worldwide Hemorrhagic fever may also be caused by Arenavirus (Lassa and South American Hemorrhagic fevers), Filovirus (Ebola and Marburg) or Flavirus (Dengue, Yellow Fever). We are more familiar with Sin Nombre disease, a New World hantavirus which causes a predominantly pulmonary illness with decreased myocardial function in people exposed to rodent urine and feces in places such as the Southwest or cabins in Yellowstone Park. Hantavirus pulmonary syndrome also occurs in Latin America, with hot spots in Chile and Argentina. There are a variety of Old World hantaviruses with similar manifestations; renal dysfunction is especially prominent among Old World hantaviruses, in contrast to hantavirus pulmonary syndrome. People with exposure to rodents are most susceptible, i.e., field biologists, agricultural workers, campers. This is diagnosed by PCR or antibody. There is no therapy that is clearly effective; ribavirin did not show efficacy in a trial. Person to person spread (and nosocomial transmission) is relevant with the African hemorrhagic fevers (Lassa, Ebola and Marburg) but not Hantaviruses. Eneterovirus D68 is not a hemorrhagic fever virus. This virus causes respiratory illnesses in the US, especially exacerbations of asthma, and has been associated with a few cases of polio like syndromes, though the association remains controversial

24 y/o health M smoker sexually active with both M and F and uses condom recs regards to HPV vaccine ?

A: He should receive 3 doses of HPV9 at 0, 1, and 6 monnth MSM and immunocompromisded >14 y/o and <26 should receive 3 doses , he's 24 if they had quadr, can still do trivalen to complete

34 y/o F, 7 month pregnant, visited paksitan to see ailing mother, spent 3 weeks 3 weeks after returning, +fatigue, fever x2 days, +nausea, vomitting, jaundice +hepatosplenomegaly, uterus nontender WBC 2.6K 72%, Hgb 9.6, Cr 1.2, AST 2460, ALT 3100, Aphos 494, Tbilli 14.4 (direct 12.2) Hep A IgM -(ive) Malaria smear -(ive) over 48 hr liver tests worsen Dx? HELLP syndrome Leptospirosis Hepatitis B Hepatitis E Hepatitis C

A: Hepatitis E food or waterborne via fecal contamination. Genotypes 3 and 4 pigs Genotypes 1 and 2 endemic Indian, Africa and Southeast Asia. a benign course in most cases. However, in pregnant women, particularly in the third trimester, a fulminant hepatitis with hepatic necrosis can ensue. Ribavirin may be effective therapy. Leptospirosis is less likely with highly elevated aminotransferases. She had no obvious exposure to hepatitis B or C such as needle exposure, transfusion from unscreened blood, or sexual exposure to someone with HBV Acute hepatitis C is rarely this severe. HELLP (hemolysis, elevated liver enzymes and low platelets) would not be this severe and absence of hypertension is against the diagnosis. Malaria can cause liver abnormalities, but these transaminases are high for malaria and there is no evidence of severe hemolysis, which would be expected.

42 y/o F malaise RUQ pain returned from 6 month at and IDP camp in uganda endorses tick and other bug bites 1st HAV vaccine 2 day befor edeparture, prior HBV vaccine no fever RUQ tender, mild icteric ALT 1245 TB 3.2 mg/dl WBC 3.2 K to do? Ebola PCR IgM anti-HEV IgM anti-HAV Borrelia PCR 16S RNA for rickettsial organsims

A: IgM anti-HEV acute jaundice in IDP camp = HEV next it's HBV next it's malaria sporadically, linked to undercooked meat (pork and deer) Hep A vaccine works even after 1st dose before going to airport

Which of the following mutations indicate high level resistance to efavirenz ? A. M184V B. K65R C. K219Q D. K67N E. K103N

A: K103N M184V: 3TC and FTC resistance B. K65R: TNF, ddI, abacavir resistance C. K219Q: TAM (AZT, D4T resistance) D. K67N: pan-nucleoside resistance E. K103N: Efavirenz Resisance

painless penile lesions 7 month duration Human papilloma virus Haemophilus ducreyi Treponema pallidum Klebsiella granulomatis Chlamydia trachomatis

A: Klebsiella granulomatis chronic ulcerated penile lesion is granuloma inguinale bacterium, difficult to culture, Calymmatobacterium granulomatisand now designated Klebsiella granulomatis. Granuloma inguinale is the classic cause of chronic genital ulcers that become increasingly disfiguring. Syphilitis ulcers generally heal within 3-12 weeks. Chancroid can be chronic, but painful. Diagnosis by organisms in macrophages, called Donovan bodies, using Wright-Giemsa stain. PCR, culture and serology are available only at research and referral centers. Trimethoprim-sulfamethoxazole or doxycycline are treatments of choice. The duration of therapy depends on clinical response.

13 y/o girl, from philippines 1 month of + swelling in neck 2x2 cm matted, non-tender posterior cervical lymph node, cool to touch and fixed to surrouding oropharynx normal, except marked gingivitis Dx/Tx? M. tuberculosis M. scrofulaceum Bartonella henselae Actinomyces israelii Kikuchi disease

A: M. tuberculosis M.scrofulaceumis a rare cause of mycobacterial lymphadenitis, despite the name, and would be much less likely than tuberculosis in a child from the Philippines. Further, lymphadenitis caused by NTM would be expected to occur in younger children (e.g. under age 5 years old). Bartonella henselae (catch scratch disease) would be more likely with cat exposure. While this diagnosis is possible, the history should point first to TB. Actinomycosis would not be in posterior cervical triangle but in the submandibular area or anterior cervical triangle. Kikuchi disease would ordinarily not be matted and fixed to the skin, as in this case. If the child is found to have tuberculosis, the health department should be alerted to check other family members.

29 y/o F R sided hip/thigh pain no trauma x2 months no F/NS gain abnormality soccer player, can't due to pain for past 2 weeks on exam: wide-based gait, pubic symphysis tenderness pain with resisted strength testing CBC wnl, x-ray shows bone remodeling of pubic symphysis MRI reveals subchondral sclerosis of pubic symphysis and small amount of fluid

A: NSAID's She has uncommon inflammatory disesae oseoitits pubis inflammation no infection, connective tissue athlete / pregnant / pelvic surgery lack of fever, gradual onset MRI helpful > bone scan maybe bone biopsy but try NSAID's first

23 y/o M in ED in kentucky in August +HA x1 day expsures: homemade cheese made from raw cow's milk jet ski accident on a man-made lake ingested lake water / injury to leg no head trauma alert and awake, + stiff neck CSF WBC: 1740 (82% neutrophils) RBC: 30 Glucose: 18 Protein: 420 Gram stain: negative Dexamethasone, vancomycin, and ceftriaxone were begun for suspected bacterial meningitis. The following day he was worse with confusion and vomiting. Cultures of the blood and CSF had no growth at 72 hours. ? etiology? Acanthamoeba castellanii Balamuthia mandrillaris Pythium insidiosa Naegleria fowleri Capillaria philippinensis

A: Naegleria fowleriprimary amebic meningoencephalitis. rare, always fatal Naegleria fowleri, a thermophilic freshwater ameba, water enters the nose; amebae migrate along olfactory nerves into the brain, Southern states during July through September, 1 to 14 days after water exposure; disease progresses rapidly. CSF findings mimic those of bacterial meningitis. Diagnosis, seeing actively moving trophozoites in a fresh wet-mount of centrifuged CSF. Trophozoites may be seen on Giemsa or Wright stain of CSF but are not seen with Gram stain. Balmuthia mandrillaris and Acanthamoeba castellanii are free living amoeba that cause granulomatous encephalitis and present as focal neurologic deficits of subacute onset. Capillaria is an intestinal helminth that causes diarrhea. Pythium causes deep tissue and vascular infection after trauma.

HIV + female CD4 260, HIV RNA <50 on ART exam T 38.3 and a macular rash on her trunk and extremities including her palms serum RPR is reactive at a titer of 1:2048, and FTA-ABS is reactive history of severe hives to PCN but has tolerated cephalosporins tx with? azithro Benzathine PCN G CFX Doxy

A: PCN G always in pregnancy with syphilis use PCN if early syphilis can have jarisch-herxheimer , causes early labor

32 y/o AML s/p induction therapy admitted for fever with neutropenia on levo ppx and G-CSF on admisison ill, T 39, multiple 5-10 mm painful tender red papules and plaques on arms dorsum and trunk oral mucosa and conjuctivae normal skin biopsy obtained hickman catheter in R infraclavicular region ANC 500, rise since clinic visit Ceftazidime began and levo d/c'd CT chest normal on 3rd day rash and fever unchanged, stbale, Cx -(ive), ANC now 750, biopsy reported numerous neutrophils with leukocytoclastic vasculitis, no organisms on fungal stain or on routine cx at 48 hrs Tx?

A: Patient has sweet's syndrome and treat with prednisone AML - common risk factor for sweet, raised violacoius nodules, rising WBC, skin biopsy neutrophilic vasculitis with negative stains no fungal since stains -(ive)

22 y/o M surgery from afghanistan 3 day fever, myalgia, some vomititng, epigastric pain facility garuding, sheep and goat around, purchased a wool rug flue like illness nearby exam normal T 102 WBC 3.3 Plt 21 Cr 1.2 Tbilli ok CXR - interstitial pna CFX started, but pt febrile CT shows pulmonary infiltrate answer?

A: Q fever brucellosis / visceral leishmenasis - does not give pulmonary infiltirates antrhax? pulmonary anthrax very rapid pneumonic plaque, getting a buboa (inoculation)- inoculation, lymphangitis, lymphadenopathy, rapidly fatal

65 y/o M, with PPM implanted 5 years ago for SSS 9 days of fever, malaise and NS UCx + 100K MSSA, no pyuria or urinary symptoms BCX + yeild MSSA PE unremarkable, generator pocket benign, -(ive) TTE Recs?

A: Remove the PPM and tx w/ Nafcillin for 4-6 weeks S. aureus bacteremia form an unknown source (not from urinary since no instrumentation / foley catheter or pyruria) raises question of intravascular lead infections - community acquired SAB with device = complicate SAB mural vegetation where the lead meets the endocardium - 45% in SAB - usual requires removal but risk of severe bradycardia

40 y/o M, travel to thialand + diarrhea, progresses to bloodystools +ill for 3 days + weak, +febrile, + tachy Tx while awaiting cultures?

A: Single dose of azithromycin (1000mg) - acute febrile and dysenteric diarrhea in a traveler is most likely shigella, campylobacter or salmonella - in south asia cipro-resistant campylobacter most likely cause of febrile dysentery - Tinidazole useful for giardiasis not campylobacter diarreha - rifaximin not effect for mucosally invasive bacterial infections

53 y/o M w/ HIV, 15 years of various ART regimen tested by tropism assay to determie suitability for maraviroc therpay CD4 - 20 100% R5 Virus Starte don maraviroc reigment 3 month later still viremic Repeat torfile shows 100% x4 virus What's happening?

A: The original tropism assay was likely correct and maraviroc therapy resulted in emergence of X4 tropic virus that was present at low levels prior to initiating the maraviroc chronically infected patients may have X4 or R5 coreceptor virus, or both viruses (D/M = dual/mixed viral strains). HIV1 enters via CD4 receptor and either the CXCR4 or CCR5 coreceptors. In late stage HIV-1 infection, approximately 50% using CXCR4 coreceptor, remaining 50% use either CCR5 coreceptor or a combination of the 2 coreceptors.

44 y/o fever x 3 months diarrhea for a year 30 lb loss intermittent arthralgia in his hands loud decrescendo blowing idastolic murmur at LL sternal BCx (6 sets) are drawn and remain negative after 21 days valvular tissu obtained at valve replacement reveals foamy macrophages by PAS stain organism?

A: Tropheryma whipplei PAS stain + foamy macrophages men 40-80 75% arthralgia tx: doxy + hydroxychlorquine 1 year or more (same as Q-fever)

mechanism of resistance in VRSA mecA vanA vanB vanC ileS-2

A: Van A all have mecA conveys methicillin resitance, van A from enterococcus species

complicated MRSA bacteremia on day 9 on dapto, but gets myalgia with a CPK of 1250 (upper limit 200) last positive BCx on day 3 of therapy vanc 2 dapto 0.5 dalba 0.25 tela 0.5 ceftaroline 1 which one?

A: Vancomycin vancomycin and daptomycin 2 only drugs indicated for bacteremia vancomcin MIC of 1.5 - 2 not a reliable predictor of clinical failure and not a reson to alter therapy vanc MIC > 2ug/mLl is a reliable predictor of nonsusceptibility and clinical faiulre and other agent should be used

55 y/o HIV +, virologic control w/ darunavir/ritonavir, abacavir/lamivudine requesting Veardenafil (levitra) neve rbeen on ED drugs (PDE-5 inhibitors Opinion?

A: Vardenafil is a substrate of CYP 3A4, thus, vardenafil dosage should be reduced ( levels of the PDE5 drugs increase dramatically prone to significant interactions with CYP3A4 inhibitors such as darunavir/ritonavir if these drugs are used for patients receiving protease inhibitors, especially boosted protease inhibitors, the dose must be started low

22 y/o F +pharyngitis, joint pain and fever x1 month daily fevers, then polyarticular joint pain and swelling to PCP, got blood and throat cx = -(ive) over next 2 weeks bcx and ucx negative no meds, other than ibuprofen exam T 39.2, erythema of nasopharyngeal, faint salmon colored macular eruption on chest and arms, synovitis both wrists, PIP and left knee, multiple 1.5-2cm cervical an axillary lymph nodes labs Hgb 10.8, WBC 15.2 (88%PMN) plt 450, ER 85, LFt normal, ferritin high, RF / ANA -(ive) monospot negative, CMV and EBV pending CXR and UA normal CT = splenomegaly

A: adult still's disease Buzz word: salmon rash ~ still's disesae duration of fever (months) kawaski = children lupus = ANA to be positive CMV = arthritis unlikley parvovirs = systemic illness

58 y/o w/ known MVP 3 months of low grade fever, chills and NS, 30 lb wt loss 2 weeks of myalgia and sever low back pain T 38, splinter hemorrhages, b/l roth spots, grave IV/VI blowing holosystolic murmur radiating from apex to axilla TTE 1.4 cm oscillating mass MV anteiror leaflet, mod MR after 3 days 6/6 BCx + strep gallolyticus(bovis) MIC to PC of 1 regimen?

A: ampicillin + gentamicin for 6 week PCN MIC = 1, out of realm of B-lactam alone or short course NVE, no reason for gentamicin, maybe enterococcus

21 y/o urethral discharge enrolled in study, NAAT done for mycoplasma genitallium, and test + chlamydia trachomatis and nisseria gonrrohea were not detected Abx to treat infection? Azithromycin Bactrim (Trimethoprim / sulfamethoxazole) Ceftriaxone Doxycycline Rifampin

A: azithromycin Mycoplasma genitaliumis, third most common cause of urethritis after gonorrhea and Chlamydia and is a sexually transmitted disease. urethritis, cervicitis, or pelvic inflammatory disease, and responds to azithromycin. Other sexually transmitted pathogens, such as C. trachomatis, N. gonorrhoeae, Trichomonas vaginalis, Ureaplasma urealyticum, and herpes simplex virus, can cause similar disease presentations. Treatment is different than the treatment of Chlamydia. The 7 day regimen with doxycycline is largely ineffective vs M. genitalium. Azithromycin was about 85% effective at treating M.genitalium but success has fallen to roughly 40 % in recent trials due to the widespread emergence of antibiotic resistance. Moxifloxacin was initially found to be 100 % effective. However, reports of resistance and clinical failure of moxifloxacin are now reported in the USA and from around the world. Mycoplasma do not have a cell wall, and hence, there is no target for beta-lactam antibiotics. Bactrim and rifampin have no useful activity.

34 y/o HIV + CD4 25, VL 3000 fever, 20lb wt loss not acutly ill erythamatous skin lesions awaiting biopsy hepatic lesions

A: bartonella henselae - can cause bacillary angiomatosis can look like Kaposi's sarcoma lesions on liver numerous, hypodense and vascular - charactersitic of peliosis hepatis KS (usually not febrile but when obstructing viscera), lymphoma and multicentric castleman's disease can invovle the liver, but not hypodense and not this many lesions c.albican distraction, skin and liver disesase, most often pt are luekopenic , very ill for a while, when WBC recovers they develop lesions

on day 9 of nacilllin for complicated s. aures bactermia has developed new neutropenia with 1000 neutrophils. MIC are PCN .12, cefazolin 0.5, vancomycin 1 (s), daptomycin 0.5 (S), ceftaroline 0.5 (s) which one to choose from?

A: cefazolin better tolerated than anti-staph PCN recommended by AHA as second line agent for NVE mortlaity no worst (maybe even better) failure rates and recurrence is similair beware of inocculum effect (10^7 vs. 10^5, MIC 0.5 vs 4-8), hence treat with Anti-S PCN then switch to cefazolin

36 y/o F IVDU R hip pain, decrease ROM 2/2 BCx MSSA CXR, CT abdomen, MRI, TTE all normal treated with empirical vanco Bcx sterile after 1 days of therapy , now on day 5 days of nafcillin, pain improved on day 7, but uses a can for ambulation which abx recommend for a 6 weeks course? dalba, CFX, vanco, ceffazolin

A: cefazolin - patient was only treated with 2 weeks , no R hip infection - for pt suspected complicated infection with no evidence, continued symptms +/- hip look harder: repeat echo MRI (maybe false negative in early disease) CT abd/pelvis PET-CT ultrasound (septic thrombphelbitis) look for spine, psoas, fibrous/ligamentous joints DV septic thrombosis 4-6 weeks initially

20 y.o. male quadriplegic with chronic Foley catheter Febrile with low BP and leucocytosis No evidence of pneumonia, diarrhea, or skin infection As expected, U/A consistent with infection After blood and urine cultures, patient is started on ciprofloxacin IV for suspected cystitis/pyelonephritis, and possible bacteremia Klebsiella oxytoca in urine and blood cultures Organism is Resistant, in vitro, to: All fluoroquinolones; TMP/SMX Aminoglycosides Extended spectrum cephalosporins and aztreonam Meropenem Production of carbapenemase documented by Rapid Carbapenem Inactivation Method (CIM) If the carbapenem-resistant K.oxytoca is susceptible to the following drugs,or drug combinaions, which one would you select ? a. Ceftazidime-avibactam b.Polymyxin E (colistin) alone c.Polymyxin E (colistin) plus Ampicillin-sulbactam d. Ceftazidime-avibactam plus aztreonam

A: ceftazidime-avibactam Therapy of Pathogens producing a Non-Metallo Carbapenemase Lab data consistent with MDR Klebsiella that is producing both: Non-metallo carbapenemase and an ESBL Avibactam protects ceftazidime from carbapenemase; no need to add aztreonam which is active in the presence of metallo- carbapenemases Want to avoid polymyxin toxicity. If used for pyelonephritis, would need colistin to achieve adequate urine concentrations of active drug

24 y/o F w/ pain and swelling on the rigth mandible over severla weeks unable to open mouth, denies F, HA, exam revealed swelling and erythem aalong her mandible MRI reveals mass eating at her mandible and going into her temporal brian Tx? Cefotaxime + Metro Vanco + cefepime TMP-SMX Voriconaozle liposomal amphotericin B

A: ceftoxamie + metronidazole or PCN and metronidazole actinomycosis

6 weeks old infant ? respiratory infeciton? +nasal discharge, +cough, worsened +irritable, cough (a staccato pattern) interfer with eating and sleeping ? acquiring infeciton at birth? Cervical culture for Chlamydia Throat culture for Chlamydia Oral NAAT test for Chlamydia Cervical NAAT test for Chlamydia Direct fluorescent antibody test for Chlamydia

A: cervical NAAT test for chlamydia C.trachomatiscan cause a self-limited subacute afebrile pneumonia with cough in neonates acquired from mothers with C. trachomatisin the birth canal. Ophthalmia neonatorum is another consequence of this perinatal infection but occurs earlier after birth (5-12 days compared to 1-3 months for pneumonia) NAAT is the most sensitive and specific test for Chlamydia from genital specimens. Culture of Chlamydia on tissue culture cells is a specific test for Chlamydia but few labs now perform this test. DFA tests are insensitive and associated with false-positive reactions.

19 y/o, no PMHx difficult to arouse by roommate + meningococcal meningitis -started on CFX -roomate received quadrivalent meningococcal conjugate vaccine (MCV4, protects against serogroups A,C,Y, and W135) one year earlier how to protect roommate?

A: chemo PPX (with rifampin, CFX, azithromycin, and ciprofloxacin [resistance to fluroquinolones reported] in Dakota and Minnesota]) - for close contacts ( typically > 8 hours, < 3 feet away, 7 days prior to onset of dx, and until 24 hours after initiation of effective abx) : 1)household members 2)day care contacts 3)contact in dormitory 4)exposed to oral secretions 5)index patient if not treated with CFX) serogroup not known within 24 hours - rate of secondary disease for close contact highest soon after onset, chemoprophylaxis immediately without waiting for culture -chemoprophylaxis >14 days after exposure not of value and not recommended - no need for booster, immunity wanes after 5 years, not 1 year - may need serogroup B meningeal ( accounts for 1/3 of cases) vaccine if outbreak

35 y/o M, agriultural visited zimbabwe for 3 weeks a day after return, developed +Fever, HA, diffuse myalgia, and joint pains in his hands, elbows, shoulder and knees and feet +macular non-pruritic rash on his face and neck that faded over several days stayed home for 4 days and fever abaated without therapy, joint pains persisted for 3 weeks later took mefloquine weekly during his stay in india, but stopped when fever and rash began labs and exam normal, malaria smear pending likely cause? Nipah virus Hepatitis A Chikungunya Mefloquine hypersensitivity Dengue

A: chikungunya Arthritis or arthralgias persist for weeks or months after a febrile illness = Chikungunya, O'nyong-nyong, or one of several other alphaviruses virus is transmitted by the Aedes mosquito in travelers from the Caribbean, South and Central America, India, islands in the Indian Ocean (e.g., Reunion) and Subsaharan Africa (e.g., Zimbabwe) Diagnosis - IgM Elisa and PCR. no specific therapy. distinction among dengue, chikungunya, Zika difficult Anopheles mosquitos transmit malaria, filaria and dirofilaria, but these diseases do not fit the symptoms of this patient. Nipah virus, Asia (including India and Malaysia), encephalitis, close contact with pigs. Hepatitis A is not likely, normal liver function tests, although hepatitis A or B can be associated with arthralgias. Dengue is a mosquito borne illness (several species of Aedes) with headache, high fever, rash and myalgias, but persistent arthralgias or arthritis would not be typical.

41 y/o F fever, worsenign respiratory symptoms long standing asthma placed on leukotriene receptor antagonist tapered off sterios on exam T 101.5 wheezing palpable purpura with nodules on elbows and legs WBC 15.23 (22% eosin) skin nodule bx: granuloma

A: churg-strauss syndrome - aka eosinophlic granulmoatsos with polyangitis long standing asthma new infiltrates and eosinophlila (>10%) as steriods being tapered rash (tender nodules, purpura) fever (uncommon)

Enterococcus faecalis is most likely to be resistant to which of the following abx? Daptomycin Linezolid Ampicillin Vancomycin Ciprofloxacin

A: ciprofloxacin Enterococci often sensitive to linezolid, daptomycin, vancomycin and ampicillin. all be therapeutic options for E. faecalis and E. faecium, although more resistance to vancomycin (especially E. faecium) and ampicillin. There is also emerging resistance to the newer drugs linezolid and daptomycin. Resistance to ampicillin and vancomycin occurs much more commonly in strains of E.faecium as comared to E. faecalis. Quinolones would be the least predictively active drug class vs. E. faecalis. Enterococci are uniformly resistant to all cephalosporins, oxacillin and other beta-lactamase resistant penicillins, and trimethoprim-sulfamethoxazole.

32 y/o HIV + on tenofovir / emtricitabine / efavirenz one month ago CD4 905, VL < 20 found to be pregnant in first timester what to do with ART?

A: continue current regimen for the mother's health and to prevent mother-to-child transmission of HIV If not already been on ART, some experts would wait until after the first trimester to start ART, although many would start ART right away. preferred for pregnancy: currently ABC (abacavir) /3TC or TDF/FTC or ZDV/3TC are recommended twoNRTI backbones. One can use either efavirenz (some would delay until after the first 8 weeks) or ATV/r or DRV/r or raltegravir as the companion drug/drug combination. Current U.S. guidelines recommend continuing an efavirenz-based regimen (despite rare reports of teratogenicity) if the mother is virologically suppressed and doing well on the regimen. This is based on the results from a large meta analysis demonstrating no increased risk of brith defects with efavirenz and is the current guideline recommendation.

A 26-year-old woman with HIV disease on abacavir/lamivudine + efavirenz with CD4 630 and VL suppressed below detection becomes pregnant. What do you recommend regarding ART? A. Discontinue ART until 2nd trimester. B. Change abacavir to zidovudine. C. Change efavirenz to dolutegravir. D. Continue current regimen.

A: continue current regimen • ART recommended for prevention of MTCT for all pregnant women, as early as possible, regardless of CD4 or VL level • Perform drug-resistance testing if VL >500-1000 cps/ml and adjust regimen, based on results • ART does NOT increase the risk of birth defects • Start (or continue) standard ART: • 2 NRTIs + a 3rd drug (PI, II, or NNRTI) •NRTI Preferred: • abacavir/lamivudine • tenofovir (TDF)/emtricitabine (or lamivudine) • Alternative: • zidovudine/lamivudine • Not recommended: zidovudine/lamivudine/abacavir (3 NRTIs) (insufficient virologic activity) • didanosine (toxicity) • stavudine (toxicity) • Insufficient data: tenofovir (TAF) - not recommended NNRTI • Alternative: • efavirenz (birth defects in primate studies were NOT borne out in human studies or extensive experience in pregnancy; screen for depression) • rilpivirine (not with baseline VL >100K or CD4 <200) • Not recommended: • etravirine (not for treatment-naïve) • nevirapine (toxicity, need for lead-in dosing, low barrier to resistance) PI Preferred: • atazanavir/ritonavir • darunavir/ritonavir (use bid) • Alternative: • lopinavir/ritonavir (use bid) • Not recommended: (pill counts and toxicity) • cobicistat • fosamprenavir • indinavir (boosted) • nelfinavir • ritonavir (as a single drug) • saqunavir/ritonavir • tipranavir (not for treatment-naïve) intergerase inhibitor • Preferred: • raltegravir • Alternative: • dolutegravir (neural tube defects described if taken during conception, but not later in pregnancy outside of 1st trimestor ) • Insufficient data: • bictegravir • Not recommended: • elvitegravir combinations (cobicistat) • Near delivery, if HIV RNA >1000 (or unknown), use intravenous zidovudine, and recommend Cesarean section at 38 weeks

two twin 15 y/o girls, both with diarrhea x 1week, told by family doc is viral, not improving, six girls with same thing six days prior GI illness started, exposure to recreational water park, swam in pool +water diarrhea, no fever, + nausea, abdominal cramps, all became ill 5-7 days after ?outbrak of gastroenteritis? Norovirus Listeria Giardia Cryptosporidium Rotavirus

A: cryptosporidium Cryptosporidium, a parasitic protozoan, watery diarrhea lasting 1-3 weeks in healthy individuals after an incubation period of 1-12 days. resistance to chlorination, Cryptosporidium has become the leading cause of gastroenteritis outbreaks associated with treated recreational water venues. incubation period here and duration too long for norovirus or rotavirus. Listeria after ingestion of contaminated food, but fever is a prominent part of such illness; the incubation period is a day and the illness lasts 2-3 days. Giardia outbreaks may occur following environmental exposure to water, but they are much less common in association with recreational water park exposures than are cryptosporidia.

29 y/o, 7 hrs ago at operating room with dislocated shoulder repair +fever +somnolent +flushied, diaphoretic and rigid BP elevated 1 amp of narcan, did not respond ? suggestion? IVIG High-dose corticosteroids Dantrolene Subcutaneous or IV epinephrine High-dose ceftriaxone.

A: dantrolene an example of the neuroleptic malignant syndrome (NMS) or malignant hyperthermia both of which should be treated with dantrolene. Neuroleptic malignant syndrome is due to an idiosyncratic reaction to neuroleptics and certain antiemetics and is characterized by four characteristics: mental status change, rigidity, fever, and dysautonomia. Extremities are described as "lead pipes." Labile hypertension, arrhythmias, and elevated liver function tests and CPK, plus hypoxia are characteristic. Look for this also in association with withdrawal or changes in anti-Parkinson medications where an NMS like syndrome occurs Neuroleptic Agents Aripiprazole Chlorpromazine Clozapine Fluphenazine Haloperidol Olanzapine Perphenazine Quetiapine Risperidone Thioridazine Ziprasidone

63 y/o M DM, CKD on day 7 of vanco, trough of 19.99 MRSA bacteremia, septic emboli, renal abscess, renal vein thrombosis BCx from HD 1-4 + for MRSA day 5 BC growing GPC in clsuters ABx recs? in addition to source control? No change combo vanc/gent/rifampin switch to dapto, televa,ceftaroline,to dapto+ ceftaroline

A: daptomcyin + ceftaroline ceftaroline is reserved for sickest not clear what to do but do dapto/ceftaroline

32 y/o M single, non-painful, clean based, penile ulcer that devleoped 3 days earlier in india for 3 week, 5 months ago serum RPR is negative most appropriate next step? obtain a tissue biopsy to evaluate for: klebsiella granulmoatis obtain serum FTA-abs dark microscopy on a swab from the ulcer no further testing; treat with doxycyline for two weeks serum glycoprotein G-based testing

A: dark microscopy on a swab from the ulcer look for syphilis too long for patient to get donovosis, or granuloma inguinnale best test for early test is dark field microscopy serum FTA-abs maybe reactive

35 y/o F painless ulcer on her vulva and one her soft palata unprotected vaginal and receptive oral sex 3 weeks earlier two ulcers with heaped up borders and a clean base which of these is inappropriate to obtain? serum RPR serum FTA-ABS serum trepenemal EIA darkfield microscopy on a specimen form oral ulcer dark filed microscopy on a specimen from the vulvar ulcer

A: darkfield microscopy on a specimen obtained form oral ulcer - in our mouth/rectum, tons of non-pathogenic trepenomes (specificity is 0) neurolgical/ocular may occur in any stage both trepenomal and non-trepenoemal tests may be negative in primary syphilis (in 30%) but they are ALWAYS positive in secondary and early latent syphilis (remember prozone reaction for non-trepenemal (RPR/VDRL) test in secondary syphilis) but the trepenemal test always positive trepenemal test are almost always positive in late syphillis (one positive always positive) irrespective of treatment history (in real life it may revert to negative after 10 years) PCN is the drug of choice to treat all stages of syphillis. No alternate agent should be used in pregnant women; in non-pregnant and non-CNS can use doxycycline

58 y/o F, nurse moved from NC to Ohio, healthy last CXR 18 years ago was normal eats sushi vounteers at dog shelter +smoker age / smoking, had a solitary pulmonary nodule resection pathology of nodule = a worm with musclaris layer surrounding a granulomatous reaction which of the following? Echinococcus Anisakis Toxoplasma Strongyloides Dirofilaria

A: dirofilaria, No drug therapy is indicated. Dirofilaria, the dogheartworm, may present as a solitary nodule in humans. The larval form transmitted to man from dogs by mosquito bites wind up in the right heart but, in humans, cannot develop into mature worms; the larvae die and are embolized to the lung, inducing a granulomatous reaction. Typically the diagnosis is made when a nodule is resected and the worm is seen. Echinococcus may cause asymptomatic lung infection but typically produces cystic disease. Anisakis is a roundworm that can cause human GI illness following ingestion of raw fish such as sushi, but would not cause a pulmonary nodule. Cysts and tachyzoites of Toxoplasma do not resemble worms and have no muscularis structures. Strongyloides larvae migrate through the lung and may cause a diffuse infiltrate in the hyperinfection syndrome but do not form solitary nodules in the lung.

73 y/o M form denver +f, NS x 1 week low appetite smoker prosthetic aortic vlave 2 years ago past 2 months received multiple itravescial BCG for bladder cancer visited brother in phoenix, tearing down chicken house in back yard ill, T 101.6, Hgb 12.8 UA 10-30 WBC, LFt x2 normal, BCx negative multiple new tiny nodular lesions in both lungs

A: disseminated mycobacterium bovis infectoin (BCgiosis) chicken? histo... but not in pheonix treat like disseminated TB, tx for 12 months could be disseminated mTB, look for it in their respiratory tract, don't get burned

35 y/o M from ethiopia +Fever, cough, and hemoptysis = found to have pulmonary TB + HIV ; no prior history CD4 = 23 VL = 350,000 Placed on 4-drug regimen; isoniazid, rifapmin, ethambutol and pyrazinamide (+pyridoxine) Assuming HIV is pansensitve, which drug best choice to combine with emtricitabine + tenofovir?

A: efavirenz Among the NNRTIs, rifampin may be used with efavirenz. efavirenz minimizing interactions when patients are to be started on both TB and ART increase the dose of efavirenz from 600 mg daily to 800mg daily if the patient weighs >60kg. rifampin is a potent inducer of CYP3A4 hepatic enzyme system responsible for the metabolic pathway of many protease inhibitors (PIs), other non-nucleoside reverse transcriptase inhibitors (NNRTIs), and maraviroc and these drugs should NOT be used concurrently. Rifampin is also a potent inducer of UGT1A1, the enzyme responsible for metabolism of dolutegravir and raltegravir. dose of the integrase inhibitor should be doubled, which increases cost considerably. Rifampin should not be used with elvitegravir-cobicistat and tenofovir-emtricitabine) because of complex drug-drug interactions with the CYP3A4 inhibitor, cobicistat. rifabutin could be used instead of rifampin: look up the contraindications and dose adjustments.

70 y/o piolot with 1 week of fever, diarrhea and sweats, then collapse tooth extraction 1 month before, E.shore of maryland and extensive travel, chelation "treatment" T 38.1, 97% on 2L, few small nodes soft systolic M, petechial rash on legs, neuro - wnl HCT 33%, WBC 1.4, plt 15 Cr 2.8 AST 495, ALT 159, Alk phos 47, TBR 0.8 CPK 8477 CXR: infiltrate LLL cause? leptospira Hep A EBV Ehrlichia chaffeensis Hepatitis G (GB virus C)

A: ehrlichia chaffeensis exposure and low platelets

48 y/o chicken farmer from oklahoma acute cihlls, fever, myalgia, w/o HA in early july 6 days later confusion and vomititng wife says she picked ticks off family cat , but no ticks on body received a shipment of baby chicks, not looked sick OA, T 39.4, confused, lungs full of rales, no rash or nuchal rigidity WBC 1100 43% bands, Plt 48, Hgb 11.2, BUN50, Cr 4.6, ALT 500, AST 700, bili normal, cxr normal, PCR + for?

A: ehrlichia chaffensis RMSF PCR of blood not available only PCR for Anaplasma vs. Ehrlichia much sicker than pt with ricketssia rickettsii, typhi or felis they usually have a rash morulae on peripheral blood, human monocytic ehrlichilosis in monocytes, for anaplasma phagocytophilum in granulocytes (more common) Ehrlichia is much more common in oklahoma, not anaplasma (goes in the ixodes tick area)

27 y/o AA F from DC, HIV + dx 2 months, ago with VL 861,000 and CD4 of 8, started on TMP-SMX PPx and emtricitabine/tenofovir, draunavir and ritonavir 3 weeks of abd cramping, + N / V x 2 days, no fever afebrile, obese, distended abdomen, slightly tender, no rebound VL falled to 54,000 and CD4 rise to 72 CT shows multiple necrotic nodes Empiric therapy?

A: empiric therpay for Mycobacterium avium complex (MAC) The clinical presentation of immune reconstitution syndrome from Mycobacterium avium mesenteric lymphadenitis. Some clinicians might opt to wait to see if this resolves without therapy for MAC, but since she has never been treated for MAC, the best option for those listed would be A. Some experts would use a regimen with activity against both MAC and MTb in this scenario. However, this patient has no history of TB exposure, and TB is uncommon to be acquired in the US except in prisons, homeless shelters, or certain other settings with high risk individuals (e.g. hospitals, long-term care facilities). Histoplasmosis does not present with such flagrant mesenteric lymphadenitis, though a biopsy is indicated to confirm the diagnosis. Stopping antiretroviral therapy for immune reconstitution syndrome (IRIS) is not recommended. Of course, a variety of cultures and serologies should be performed to attempt to identify the causative organism, but in this case none are likely to be positive

62 y/o vietnamese with hepatitis AST 688, ALT 580 TB 3.4 oncology suit getting cycle 5 of R-chop normal AST, ALT and tbilli total HAV detectable anti-HBc positiv HBsAg negative Anti-HCV neg What do you recommend? Hold rituximab Hold prednisone HCV PCR Entecavir

A: entecavir rituximab, prednisone and transplant have very high risk of HBV reactivation diagnosis of HBV would be with HBV DNA and HBsAg - if HBsAG positive, prophylaxis recommended in this case, he has seen HBV (anti-HBc +, HBsAg negative) so you could monitor but go with treatment - treatment with entecavir or tenofovir/TAF

38 y/o M , refused far advanced HIV admitted for ination, weakness, profound wt loss and chronic diarrhea picture of little dots in enterocytes

A: entercytozoon bieneusi species of the order Microsporidia which infects the intestinal epithelial cells. It is an obligate intracellular parasite cyclospora - would not be in enterocytes cryptosporidum would be in the enterocyte but picture dots are tiny

51 y/o M oversea trip south tip of africa to mediterrean sea, ate on shore after coming back for a week, have fever, 38.3 WBC 16000 without eosinophilia liver is enlarged and tender CT and MRI show multiple cysts?

A: enteric bacteria (bacterial liver abscess) fasciola hepatica parasitic trematode (fluke or flatworm, a type of helminth) of the class Trematoda, infected by eating raw watercress or other water plants contaminated with immature parasite larvae, not this kind of radiological figure, also eosinophilia echinococcus multiocularis, hytadid disease, got to be around dog, gradual disease cysticercosis paragonimus westermani - lung abscess

32 y/o F with HIV-1 on Tenofovir+emtricitabine (FTC)+Efavirenz, and has experienced rebound viremia K103N mutation which NNRTI - most virological activity?

A: etravirine Etravirine has good activity against viral isolates that contain only the K103N mutation, although the use clinically of etravirine following NNRTI failure is questionable - a boosted protease inhibitor-based regimen is preferred. Nevirapine would not be active against viral isolates that contain the K103N mutation. Resistance conferred by K103N to efavirenz is complete, so increasing the efavirenz dose would have no effect. Another mutation, A98G, is a NNRTI mutation that contributes to resistance to nevirapine and efavirenz but not to etravirine. It is unlikely but possible this would show up in a question. E138K is an NNRTI mutation that confers resistance to nevirapine, efavirez, rilpivirine AND etravirine

45 y/o HIV+ M, treatment experienced HDL = 38 LDL unable to calculate Trigly - 1200 on darunavir/ritonovir + tenofovir/emtricitabine trigylceride lowering approach?

A: fenofibrate Boosted darunavir will augment simvastatin, lovastatin, and pravastatin levels leading to enhanced toxicity When Triglycerides exceed 1000 mg/dl, either fibrate or niacin should be the first line therapy (see NCEP guidelines) and, therefore, "dietary counseling" is wrong. patient would likly benefit from fenofibrate plus a statin, plus omega 3 fatty acid, plus dietary counseling. Atorvastatin can be started with the lowest dose and titrated upwards. Pravastatin and fluvastatin have the least interactions and can be used safely with most protease inhibitors; however, darunavir has been shown to increase pravastatin levels substantially

33 y/o bank executive +intermittent watery diarrhea x 3 1/2 weeks +bloating, + abdominal cramping no fever, has 2 preschool children at day care several other children have diarrhea ?diarrhea cause? Norovirus Rotavirus Enterotoxigenic E. coli Giardia Cyclospora

A: giardia In healthy persons, infectious diarrheal disease rarely lasts more than two weeks except when due to Giardia or Cyclospora, or sometimes Amoeba. Post infectious irritable bowel can persist for many weeks. Giardiasis, diarrhe, day care centers. Illness due to norovirus, rotavirus and enterotoxigenic E. coli in healthy adults is short-lived, typically gone in a few days. Cyclospora may produce protracted diarrhea in healthy adults but is associated with ingestion of imported berries (raspberries) and not exposure to day care centers. If ameba had been offered as an answer, it would be possible but...it was not offered

44 y/o F hospitalized with anemia thrombocytopenia diagnoed with complement mediated HUS and treatment with eclizumab vaccination fo rmeningococcal vacine?

A: give both meningococcal conjugate vaccine (MCV4) and meningococcal B vaccines recs for meningococcal vaccine: mening B outbreak asplenia complement deficiency on eculizumab (soliris) microbioogist with exposure to nisseria meningiditis same vaccine should be used for all doses

38 yo M 6 week of cough, +f / NS nasal stuffiness lives in phildelphia, 6 months ago to cincinati, OH no pets T 100.2 saddle nose deformity with perforation of nasal septum, lungs clear , rest of exam normal CT chest: bilateral nodule with cativations Test to do:?

A: granulomatosis with polyangitis (GPA) formerly wegner's, order a C-ANCA pulmonary-renal syndrome dx clinical, C-ANCA / biopsy BUzz word: nasal symptoms (saddle nose and performation) CT nodules respiratory symptoms and hematuria

a man from west africa, hx of PCP pneumonia +HIV 1/2 ELISA VL <50 copies CD4 125 CBC/CMP - unremarkable - another lab confirmed HIV 1/2 ELISA + Interpertation and Tx?

A: has HIV-2, and should be treated with emtricitabine-tenofovir and darunavir-ritonavir An important issue is that commercial HIV-1 nucleic acid amplification tests do not amplify HIV-2. Some research labs and some new commercial kits can quantitate HIV-2 RNA levels Efavirenz has no activity against HIV-2. NNRTIs ( Doravirine, Rilpivirine, Etravirine, Efavirenz, Nevirapine) cannot be used for HIV-2, but NRTIs and PIs (and integrase inhibitors) are active against wild type virus.

27 y/o F aid worker, 2 month trip to nigeria in may recent graduate traveled to brazil 11 years ago received all childhood and yellow vaccine (11 years ago) which vacicnes should she get? yellow, hep a, typhoid, menin, japaenese enc, cholera

A: hep A , typhoid , meningococcal, and japanese encephalitis and cholera yellow fever - yellow fever virus, spread by mosquitos -for >=9 months areas of risky - single dose provides lifelong, no booster - exception is pregnancy when received (not responded to it), underwent HCT after initial vaccine, HIV+ meningococcal : >2 y/o in the meningis belt - mening B not recommended - 7-10 days to get antibodies (hence 10 days prior to hajj, also no more than 3 years for polysacchride, 9 for conjugate typhoid - south asia, west africa - oral vs. IM, 1-2 week prior to travel - delay vacicne >72 hr after antibacterial medications japanese encephalitis: >1 month, rurar areas, irrgated areas rice paddies or going to outbreak cholera vaccine approved since 2016 18-64, where cases have been reported

46 y/o M HIV+, being tx for disseminated histoplasmosis for past 2 months w/ amphoB, now itracon yesterday surgery under conscious sedation for infected wisdom tooth, night of surgery developed fever and received broad abx this morning w/ hypotension and abdomina pain, +pressors T 101.8 F, pulse 122 no rebound, +BS WBC 12K, 70% neut Na 125, K 5.7, Cr 1.9 BCx -(ive) x18 hrs Started on Ppip-tazo? oral source sepsis ? addition to his regimen? Dantrolene Caspofungin Liposomal amphotericin B Voriconazole Hydrocortisone

A: hydrocortisone This clinical picture of hypotension, fever, abdominal pain, hyponatremia and hyperkalemia is characteristic of an adrenal crisis in someone who was likely borderline adrenal insufficient. Disseminated histoplasmosis, like tuberculosis, can involve the adrenal glands and produce hypoadrenalism. The diagnosis of adrenal crisis should be considered in someone who has had a recent stress such as trauma or surgery and a reason to have underlying adrenal insufficiency such as TB or histoplasmosis. Fungemia, such as candidemia, from a dental source would be quite unusual, as would be the sudden onset of shock from uncontrolled histoplasmosis. Empirical addition of an antifungal would not be indicated. Malignant hyperthermia, which would prompt use of dantrolene, is usually triggered by a halogenated gas anesthetic or succinyl choline, neither of which would be used for conscious sedation. For malignant hyperthermia, muscle spasms, sometimes leading to rhabdomyolysis, are usual. This diagnosis would not be consistent with the presence of eosinophilia, hyperkalemia and shock.

A 49 year old man recently emigrated from Japan presents with rapidly progressing dementia. He underwent a meningioma resection with dura mater graft in Japan 35 years ago. He is an avid deer hunter and consumes venison. What is the most likely cause of his dementia: A. Iatrogenic CJD from the dura mater graft B. Iatrogenic CJD from eating deer. C. HTLV-I D. Spontaneous CJD

A: iatrogenic pituitary extracts - Human Growth Hormone - Gonadotrophin - Delay may be >30 y • Dura mater grafts - Mostly Lyodura brand • Transplants - Corneal - Pericardium - Liver • Instrumentation - Implantable NeurosurgicalEEG, stereotactic No links: -Vaccines • Feces • Saliva • Sputum • Bovine insulin • Semen, vaginal secretions

IVU fever x 4 days grade IV aortic insufficency murmur starte don vancomycin 1gm q12h after bcx MRS is grown form bcx and repeat bcx on day 2 and 3 on day 4 short of breath, rales on exam, most approprate next step ?

A: immediate aortic valve replacement aortic insuffiency, not abx insufficiency valve replacement once bcx are negative not likely

25 y/o peace corps work returns from nigeria felt well developed pruritic maculopapular rash dx with eczema, not resolving with topical steriods asymptomatic subQ nodule near her left elbow biopsy shows worm thingy she could develop? keratitis, lymphedema, eosinophilic pneumonia, encephalitis

A: kerititis river blindless Onchocerca volvulus adult filaria lives in nodules babies and microfliria move to skin (pruritis) move to cornea (die in cornea /opacified) not loa loa cause has migratory bumps, very difficult to find microfilarea eosinophilic pneumonia - strongyloides /filariform larvae through a transcutaneous route may result in Löffler's syndrome

48 y/o homeless lives in baltimore, found unresponsive 1 week of malaise, chills, HA, leg pain and weakness eats disposed food PMH ETOH, IDU , kidney stones T 39.1, sleepy Icteric, non-injected, no murmurs, or lymphadenopathy diffuse red maculopapular rash WBC 98K, Hgb 7.7, Plt 31K Cr 3.9, UA 1+ prob, INR 2.5, Tbilli 41 (direct 31), ALT/AST 146/213 HCV Ab pos, HIV ab neg dx: acute hepatitis A Babesia microti Ehrlichia chaffensis Leptospira icterahemorhhagiae Zika

A: leptospirosis exposure: fresh water water (rafting in hawaii, or costa rico) or rats systemic: kidneys, eyes, skin, muscle, lungs) labs: liver enzymes < bilirubin

A 37 yr female with diabetes mellitus is admitted for ketoacidosis, fever and sinus pain. Biopsy of a necrotic area of the turbinate shows wide, branching nonseptate hyphae. Serum creatinine is 2.5 mg/dl. Which of the following would be most appropriate? A. Voriconazole B. Anidulafungin C. Fluconazole D. Liposomal amphotericin B E. Itraconazolemiddle .

A: liposomal amphotericin B - even though she is azotemic broad branching NON-septate hyphea voriconaozle - not used for mucormycosis anidulafungin / fluconaozle no activity against mucor Amphotericin B is the only agent with proven clinical activity against the agents of mucormycosis. Posaconazole and isavuconazole have limited use. Fluconazole and echinocandins are inactive in mucormycosis There is no synergy between ampho and an echinocandin in mucormycosis (report was misleading). Deferasirox clinical trial failed.

34 y/o M, returned from a month lung misison trip, refugee camp in africa, no immunizations +Fever, cough, and diffuse rash initially Fever, cough, and coryza , 2 days later rash (initally on face then spread to chest arms and legs) +T101.6, tachy, and conjuctivititis, diffuse dark red maculopapular nonpruritic rash on face, neck, trunk and proximal extremiteis CBC/UA/CMP -nml CXR - interstitial infiltrates Dx? Measles Rift Valley Fever African tick-bite fever Chikungunya Lassa fever

A: measles conjunctivitis, coryza, and cough, three Cs of measles (rubeola) as the prodrome to the rash. rash typical starting on the face at risk since refugee camp where measles and other childhood infections are likely to occur never immunized. fever usually resolve 48 hours after onset of the rash. adults, with measleoften quite ill appearing did not have Koplik spots on his buccal mucosa assist diagnosis. Rift Valley Fever, in Subsaharan Africa, transmitted by mosquitos or close contact with small mammals, have fever, headache, myalgias, arthralgia, and coagulopathies. Rash is rare in Rift Valley Fever. In African tick-bite fever there may be several small eschars and petechiae, but not the rash described here Chikungunya typically has joint pain; a rash may occur which tends to be truncal, pruritic, fine, maculo-papular, pink and evanescent In Lassa Fever mucosal bleeding may occur but rash is not typical.

22 y/o college football septic shock from turf burn became infected BCx prior to abx, started on vacomycin 16 hr later three sets + for S. aureus, found to be oxacillin resistant, vanc/linezolid sensitive Responsible for oxacillin resistance? mecA gene encoding penicillin binding protein 2a Production of an oxacillin beta-lactamase Panton-Valentine leucocidin Altered porins causing decreased permeability Oxacillin efflux pumps

A: mecA gene Oxacillin resistance, always altered penicillin binding protein mecA gene encodes for production of penicillin binding protein 2a, which replaces penicillin binding protein 2. This 2a protein has low affinity for nafcillin and oxacillin and methicillin. MRSA also produce beta-lactamases that result in resistance to penicillin and ampicillin but not nafcillin or oxacillin (penicillinase resistant semisynthetic penicillins) whose beta lactam rings are not hydrolyzed by penicillinases. mecA is carried by SCC types 1-IV (staphylococcal chromosomal cassette). Types 1-III are more often associated with hospital acquired S. aureus; type IV is associated with community acquired MRSA. Panton-Valentine leukocidin is a S. aureus exotoxin that is found in most USA 300 strains, which have traditionally been community acquired. The PVL gene is responsible for a toxin that is relevant to its pathogenicity, but not its resistance to oxacillin. Efflux Pumps: There are no clinically significant oxacillin efflux pumps.

65 y.o. male has recurrent obstructive nephropathy, chills, and fever Started on empiric ciprofloxacin The laboratory reports E.coli in blood and urine E.coli is resistant to : Ciprofloxacin Cefazolin Ceftriaxone Aztreonam Gentamicin Patient has no history of drug allergy With this pattern of resistance, Which one of the following is your preferred therapy ? A. Piperacillin-tazobactam B. Meropenem C. Polymyxin B F. Tigecycline

A: meropenem clinical failure with Pip-tazo seen in marina trial not polymixin caue it's not renaly excerted If I say ESBL producing GNB, you think: Meropenem (carbapenem) is treatment of choice (CID 60:1326,2015) as all 3rd gen. cephs. hydrolysed Cefepime only potentially stable extended spectrum cephalosporin Alternatives in reserve: Ceftolozane/tazo and Ceftazidime/avibactam active Piperacillin-tazobactam: discordance; active in vitro but clinical failures; Inoculum effect ? Want to avoid polymyxins due to toxicity; in addition, polymyxin B urine concentration is very low Don't use tigecycline for anything

A 72 yr man with diabetes mellitus, renal failure and a central venous catheter developed fever and hypotension. Blood cultures grew Candida lusitaniae. On day 5 of liposomal amphotericin B 5 mg/kg he remained febrile and his creatinine rose from 4.5 to 6.0 mg/dl. In addition to changing his IV catheter, which of the following would be most appropriate?: A. Itraconazole B. Micafungin C. Amphotericin B lipid complex D. IV Voriconazole E. IV posaconazole

A: micafungin Itraconazole with CHF, impaires cardiac function lipid complex - more nephrotoxic IV vori and posaconazole, not to give with azotemia ; has sulfobutylcyclodextran

27 y/o M w/ sickle cell disease, w/ chest syndrome crisis, started on vanc/levo, CXR some b/l infiltrates CBC and chem 12: leukocytosis, hemolytic anemia, and mild LFT's has a tunneled double lumen subclavian line for 4 months at 96H, one bcx grew non-branching beaded GPB that is AFB + three additional BCx (one through each lumen) and one peripherally at 48H + for AFB nonbranching rod

A: mycobacterium mucogenicum lines AFB + should think mucogenicum (chelonae like) rapid grower can seed peripheral tissue (peritoneal) pull the line tx for several weeks

26 y/o M burning with urination, no discharge NAAT of urine -(ive) for gonorrhea or chlamydia wet mounts -(ive) wife tested for GC, chlamydia and trichomonas and -(ive) no other symptoms, HLAB-247 ag -(ive) tx with doxycycline unsccessful Dx? Chlamydia trachomatis Neisseria gonorrhea Trichomonas vaginalis Mycoplasma genitalium Reiter's Syndrome (reactive arthritis)

A: mycoplasma genitalium M. genitaliumis, emerging STD, urethritis, cervicitis, and PID, In men, no discharge. Doxycycline fails macrolides used to treat this infection. However, macrolide resistance widespread. drug regimen of choice Moxifloxacin for 10-14 days Fluoroquinolone resistance has been documented Diagnosis, difficult, nucleic acid amplification testing Antibodies for epidemiologic purposes only. Chlamydia and GC are unlikely by the negative workup. Motile Trichomonas may be seen on wet mount and can be cultured or tested by PCR, more sensitive tests but not done in this case. He had no associated symptoms of Reiter's syndrome and was HLA B-27 negative (positive in 80% of cases).

healthy female pain her calf red, tender rash o ncalf no trauma bleeding from an unsuccesful needle aspirate received cephalexin , pain progressed and increased and came to ER has a F 102, swollen tender calf a soft tissue film no gas WBC 15, 80% PMN's organism?

A: necrotizing fascitis by streptococcus pyogenes notice and call surgeon don't go for imaging gas gangrene = due to trauma usually, no gas here staph pyomyositis can occur but too rapid vibrio - no exposure to brackish water or eating sea food

24 y/o w/ HIV, poorly controlled CD4 110 VL 10,000 on abacavir/lamivduine, darunavi and ritonavir +fatigue, no fever or wt loss Hgb 7 WBC 500 Plt 45 Bone marrow normocellular, adequate irone stores utility of erythropoieten and G-CSF for this pt?

A: neither G-CSF or erythropoietin has been shown to improve survival Anemia often improves when effective ART is instituted. Erythropoietin can augment hemoglobin, especially in patients with serum erythropoietin levels <500 IU/mL. The augmentation in hemoglobin may have benefits in terms of functional ability and ability to tolerate marrow suppressive drugs, but there is no evidence that survival is improved due to the erythropoietin induced rise in hemoglobi G-CSF also has not been shown to improve survival in this patient population. Using this growth factor to raise neutrophil counts to >500 or >1000 cells/uL will increase neutrophil counts within two days in most patients, and probably decreases the frequency of infections, but there is no evidence that survival is improved

dentist request for abx ppx for endocarditis 63 y/o female with no significant PMH except calcified aortic sclerosis no prior history of IE NKDA PPx?

A: no ppx usually ppx regimen include: amox 2 gm po clind 600 mg po azithro 500 mg po ampicillin 1 gm IV

62 y/o F self reported hx of shingles 10 y/a and DM II received live -attenuated zoster vacine 2 years ago regarding vaccination?

A: now with have recombinant vaccine give recombinant zoster vaccine, recommende dover live attenuated vaccine ->50 y/o (as oppose to 60) no need to wait any specific period after Hz (just not during acute episode) -2doses 2-6 months apart

64 y/o M HIV + , treatment experienced on tenofovir, emtricitabine and unboosted atazanavir advised to switch to boosted atazanavir but he had never been willing to do so VL < 20 CD4 = 400 has GI bleed found to be helicobacter positive on gastric biopsy and treated with metronidazole, clarithromycin plus omeprazole two weeks and 6 weeks after hospitalization, his VL is >25000 copies on each occasion, and his CD4 count is falling despite perfect adherence Cause?

A: omperazole Atazanavir is no longer recommended as first-line treatment, and notat all without boosting by concurrent ritonavir or cobisistat. Unboosted atazanavir should not be used for patients who must take omeprazole ( acid-lowering agents) Boosted atazanavir would probably be adequate Atazanavir requires acidic gastric pH for adequate absorption The degree of impairment depends on the dose and half-life of the acid-lowering product, the duration of acid lowering effect, and the timing between taking these agents and atazanavir.

You are asked to advise your hem‐onc colleagues as to what prophylactic antifungal agent might be useful in preventing aspergillosis in their patients with acute graft‐vs‐host disease after allogeneic stem cell transplantation. According to the IDSA guidelines and literature you recommend: A. itraconazole solution B. posaconazole C. micafungin D. voriconazole E. caspofungin

A: posaconazole approved for PPx with prolonged neutropenia and aspergillosis Posaconazole FDA approved for GVHD and prolonged neutropenia Voriconazole not better survival than fluconazole in GVHD trial Posaconazole> voriconazole> micafungin recommended by guidelines for prolonged neutropenia and high risk aspergillosis

traveler to india 6 month ago sick since travel initially diarrhea, grossly bloody stools that responded to x3 days of cipro next 6 months, abdominal pain, bloating, loose stools, without diarrhea... exacerbated by eating DX?

A: post-infectious irritable bowel syndrome (IBS) * occurs in 5-10% after traveler's diarrhea, most common due to campylobacter and shigella - celiac / gluten-sensitive enteropathy, has other extraintestinal symptoms -cyclospora - chronic DIARRHEA

22 y/o F, pregnant and HIV+ - STD screen, VL ~ 1 million, CD4 450, resistance panel pending, 9 weeks pregnant ? what to include in her regimen? maraviroc efavirenz nevirapine D4T raltegravir

A: raltegravir ART to all pregnant women to reduce perinatal transmission Regimens based on raltegravir, atazanavir/ritonavir, or darunavir/ritonavir would all be preferred, with abacavir/lamivudine, AZT (zidovudine)-Lamivudine, or tenofovir-emtricitabine as the nucleoside components. Nevirapine:. if they received perinatal single dose nevirapine for a prior pregnancy, might have a nevirapine-resistant strain. In any pregnant patient, there is an increased risk of hepatic toxicity if they are started on nevirapine when their CD4 counts are above 250-350 cells/uL. On the board exam, stay away from nevirapine during pregnancy if CD4 counts are above 250 cells/uL. D4T and ddI are rarely used in USA associateid with lactic acidosis in pregnancy (with or without pancreatitis or hepatic steatosis) and should not be used in pregnant women. Efavirenzis teratogenic in non-human primates, although there is not strong evidence that it is teratogenic in humans and a large meta analysis found no increased risk. The current package insert for efavirenz still cautions about use during the first trimester 8 weeks of pregnancy. Women who are suppressed on EFV-based regimens and become pregnant should continue the regimen. There is insufficient data to support the use of maraviroc in pregnancy and this drug is not a recommended agent for initial therapy

51 y/o F in montana, hx of lyme disease (erythema migrans, tx with doxy 4 years ago) 6 months of: +fatigue, +insomnia, +myalgia, +difficulty concentrating labs: + B. Burgdoferi EIA, + IgM and IgG immunoblot recommendations?

A: reassure that her lyme diseaes acquired 4 years ago is not explanation of her symptoms *hx of lyme dx = + lym serology for 20 years including + IgM and IgG * lyme C6 ab is less specific than two tier testing; useful in IgM to determine false positive

45 y/o M one week of back pain afebrile, tenderness of the lower back MRI shows discitis, hyperemic marrow 1 of 3 BCx grow coNS reocommend? empiric therapy w/ vanc/cefe bone biopsy slide coagulat test of the blood PET -CT fungal serologies PPD

A: request a slide-coagulase test of blood isolate make sure your not dealing with staphylococcus lugdunesis -tube free coagulase test is negative -but latex bound coagulase test may be positive, a quick fast test (clumbing factor) virulant, aggrsive, similar to S. aureus bacteremia, NV and PV

40 y/o rural south east virgina +F, HA, myalgia, and arthralgia +multiple eschar on lower extremities denies travel, tick bite and exposure to mice several dog and cats developed erythamatous maculopapur rash on his trunk spread to his extremities Tx with cephalexin did not improve WBC 3300; Platelets 149,000; Hgb 13.5; LDH 1502 U/L; Bilirubin 2.4 mg/dL; ALT 195 U/L ? pathogen? Babesia microti Borrelia burgdoferi Scrub typhus (Orientia tsutsugamushi) Murine typhus (Rickettsia typhi) Rickettsia parkeri

A: rickettsia parkeri eschar followed by fever and rash is highly suggestive of rickettsial infections of the spotted fever group. The only one of those listed here is Rickettsia parkeri, a tick borne infection that occurs in the mid- and South Atlantic states, South Central and the Gulf coast states of the USA, as well as in South America. Eschars, or bite sites, on the lower extremities are typical of Rickettsia parkeri and distinguish it from Rocky Mountain Spotted fever, which has no eschar. A diffuse maculopapular rash would be unusual for babesiosis or Lyme disease, and Virginia would not be a common locale for babesiosis. Scrub typhus (Orientalis tsutsugamushi) does not occur in the United States: this occurs in Asia including China and India and Japan. R. conori (Mediterranean Spotted fever...not in US), R. africae (African tick bite fever, not in US), and O. tsutsugamushi (not in US) are associated with eschars; however, R. typhi is world wide. The organism R. typhi is spread when flea feces are rubbed into skin wounds. This is a mild illness, is seen most often in refugee camps and areas of poverty, but is seen domestically mostly in the southwest United States. Thus, this patient is not geographically typical. This case, therefore, could be R. typhi except the geography is atypical, and prominent eschars should point you to R. parkeri if the case was acquired in the United States.

18 y.o. male requires anti- seizure meds. for idiopathic epilepsy Fluctuant tender furuncle on right arm He develops fever and generalized erythroderma; wherever he is touched, a bullous lesion develops. A skin biopsy shows an intra-epidermal split in the skin. Which one of the following is the likely etiology of the skin bullae? A. S. aureus scalded skin syndrome? B. Bullous pemphigus? C. Drug-induced Toxic epidermal necrolysis (TEN)? D. S. pyogenes necrotizing fasciitis?

A: s. aureus scalded skin syndrome split in the skin for staph is at intra-epidermal split in the skin is at epidermal- dermal

48 y/o with RA on TNF-alpha inhinbitors never gets influenza vaccine, due to hives if he eats eggs and is immunosuppresed eats baked goods cooked with eggs and has no sequel ? can he get live influenza vaccine?

A: safely receive inactivated trivalent and quadrivalent influenza vaccine despite allergy safe to give any other forms of influenza vaccine even in individual with prophylaxis can be given with monitoring immunocompromised and should be vaccinated

37 y/o F fever and joint pain long distance runner 3 weeks ago noted R knee pain along a run treated with steriod injection developed bilateral ankle pain and redness notes subjective chills and sweats several tick bites last 2 months T101, bilateral synovial thickening of ankles with warmth and tendernes sto passive movement skin exam with painful pre-tibial nodules most likely diagnosis?

A: sarcoidosis, get a chest x-ray pt presents with lofgren syndrome - women - triad: arthralgia (ankles), lymphadenopathy on chest x-ray and erythema nodusom Buzzwords:: hilar lymphadneopathy erythema nodusm parotid enlargement uveitis aseptci meningitis with basilar enhancement non-caseating granulomas

27 y/o pregnant women G2P1 healthy 2 year old at home regards to Tdap in pregnnancy?

A: she should get the Tdap vaccination all adults all pregnant women during each preganancy, regardless of interval between 27-34 weeks

Which one of the following organism can occasionally give a positive shiga toxin 1 test? A. Shigella B. Campylobacter C. Norovirus D. Aeromonas E. E. histolytica

A: shigella • Some antibiotics mobilize phage (e.g. flouroquinolones, TMC-SMX), azithromycin and rifaximin do not • Antibiotics are not indicated in this infection but stay tuned • Controversy on utility of IV Eculizumab, a monoclonal antibody approved in U.S. for PNH that inhibits terminal complement. Some studies showed improved renal disease and is licensed in Europe (expensive ~ $500,000 per year in cost)

67 y/o F long-standing DM and CHF recieved 23- valent pneumococcal polysachrid vacicne at age 45 received 13 valent at age 65 next step?

A: should get another PPV 23 now got vaccinated before 65 with PPV23 (because of CHF, can be smoker, CLD) after age 65, 23-pneumo should be repeated, a year after 13 valent

26 y/o M 1 month of fever, NS, and fatigue seen by PCP negative mono thought viral illness lives in idiana with his wife and 2 y/o son 2 cats T 38.4 no lymphadenopathy palpable spleen tip most appropritae next step? flow cytometry, ANA,, EBV serology, soluble IL-2 receptor alpha level toxoplasma titer

A: soluble IL-2 receptor alpha level pt diagnosed with HLH -triggered by infections commonly EBV -elevated ferritin - cytopenia

44 y/o M +dyspnea, non-productive cough and diaphoresis, quiet ill, musician, exposed to drums CXXR b/l pleural effusion, cardiomegaly WBC 8 Starte don CFX, azithromycin nine hours later GPR in 4 sets of BCx ?recs? Immediate respiratory isolation Immunization of all healthcare workers coming into close contact with him with vaccine against the likely pathogen Doxycycline prophylaxis for all health care workers with close contact with the patient Standard precautions only

A: standard percautions only naturally occurring inhalational anthrax, hides used as drum heads were contaminated with anthrax spores The patient does not have spores, per se, in his respiratory secretions and is of no risk to those in his environment In theory, contaminated biologic fluid such as sputum could contain organisms that could incubate in the environment and produce spores, but this is quite unlikely Keep in mind that his friends or family members might have been in the same environment that he was when he was infected. Their exposure needs to be assessed and chemoprophylaxis must be considered for them with ciprofloxacin, levofloxacin, or alternatively doxycycline. The optimal therapeutic regimen of antibiotics for anthrax is not known: a combination of ciprofloxacin plus clindamycin is usually recommended, often with high dose penicillin if the organism is penicillin susceptible, or while awaiting susceptibility results. The FDA has approved raxabacumab, a monoclonal antibody that neutralized anthrax toxin for the treatment of inhalational anthrax.

36 y/o w/ lymphoma admitted for 3rd cycle of chemotherapy within 4 day so admission, developed pneumoina treated with vancomycin, ceftazidime and tobrmaycin sputum reveals MRSA, with vanc MIC 4 (intermediate resistance) BCx negative, isolate susceptible in vitro to linezolid, dapto, tigecycline and rifampin, tobra and ceftaizidme discontinued pt not responding after 2 days, vtrough at 12, GS of tracheal aspirate many GPC in clusters, ? to his treatment?

A: start linezolid and stop vancomycin pna with VISA strain, vancomycin at any dose not effective dapto maybe used in septic pulmonary embli and hematgenous PNA tigecycline - associated with increased mortality

22 y.o. pregnant female IVDU has tricuspid valve MRSA endocarditis complicated by multiple lung abscesses MRSA in vitro susceptibilities: Vanco-O.5 ug/ml, Dapto-1.0 ug/ml Vancomycin therapy: trough conc. 17 ug/ml Persistently febrile Blood cultures drawn on day 4 of vanco, positive on Day 6 and vanco MIC is now 2 ug/ml ; dapto MIC remains low at 1 ug/ml Which one of the following would you recommend ? A. Continue Vanco and add rifampin and low dose gentamicin B. DC vancomycin, start daptomycin in a dose of 8-12 mg/kg once a day C. DC vancomycin and start linezolid, 600 mg IV q12h D. DC vancomycin and start telavancin , 10 mg/kg IV q24h

A: stop vancomycin, start daptomycin in a dose of 8-12 mg/kg once a day developed in vivo resitance we want bactericidal, not bacteriostatic (linezolid) telavancin has it's own toxicity) Slowly bactericidal Three types of resistance: Hetero- VISA - thickened cell wall, target still present High level resistance due to target change (d-alanine - d-lac) Gram Positive Spectrum of the Vancomycin "Family" of Glycopeptides Aerobic & Anaerobic Gram Positive Cocci Selected aerobic gram positive bacilli: e.g., Listeria monocytogenes and Corynebacterium jeikeium Selected Anaerobic gram positive bacilli:e.g., Clostridium difficile and other Clostridium species Our 22 y.o. pregnant female with Tricuspid valve Infective Endocarditis remains seriously ill despite the high dose daptomycin Blood cultures collected after 5 days of Daptomycin return positive for MRSA 2 days later Daptomycin MIC is now 6 ug/m (resistant). Which one of the following salvage therapies would you recommend ? A . Continue daptomycin and add either oxacillin or ceftaroline B. Discontinue daptomycin, start ceftaroline alone C. Discontinue daptomycin, start tedizolid D. Continue dapto and Consult CV surgery; Consider removal of tricuspid valve to achieve source control Salvage therapies for dapto-resistant MRSA Nafcillin, oxacillin, or ceftaroline restores negative charge on cell membrane and thereby reverse daptomycin resistance Well-documented reports of clinical success with combination therapy: dapto plus a beta-lactam No data on monotherapy for IE with ceftaroline or linezolid/tedizolid

65 y/o F 2 day of fever, chills HA, confusion T40.5 obtundend stiff neck WBC 30, 000, plt 20,00 LP opening pressure of 280, WBC 2800, glucose 20 , protein 400. GS = GPC diplococci in addition to dexamethosone which of the following regimens? PCN G CFX Vancom + Amp Vanco + CFX Vanco + TMP-SMX

A: streptococcus pneumoina meningitis if bacterial meninngitis ? immunocompromised, hx of CNS dx, new onset seizures, papiledema, altered consciousness, or focal nurological deficits YES? 1)BCx stat 2)dexamethasone + empiric ABx ( adults =for pneumococcos, vanco + CFX, older = vancom +CFX, and ampicillin, after shunts / immunocompromised = cefepime or meropenem) 3)negative CT scan 4) LP NO? BCx / LP stat Dexa + empiric ABx CSF findings w/ bacterial meningitis / or +GS = dexa + targeted Abx

29 y/o HIV + CD4 205 VL 36,000 HLA-B*5701 + started on tenofovir/emtircitabine/elvitegravir/cobicstate 24week later, CD4 355, VL <20 3 months later, missing 2-3 doses a week, VL = 13,000 Two weeks later VL 32,000 genotype reveals M184V in RT and Y143R and Q148R in integrase ? once a day regimen of choice

A: tenofovir/emtricitabine + darunavir/ritonavir Q148R; integrase inhibitor mutations reduced activity to dolutegravir. the M184V would confer complete resistance to emtricitabine, so this is a suboptimal regimen. Efavirenz has a low barrier to resistance and must be combined with two fully active agents for maximal virologic efficacy and to avoid the emergence of resistance Also, the patient is HLA*B5701 positive which is a contraindication to abacavir

54 y/o M, anti-HCV + after elvated ALT noted by PCP IDU 20-21 y/o, moderate ETOH next step?

A: test for HCV RNA indication for screening: IDU, elevatedLFT of unkonwn cause now new is born 1945-1965 how to test: EIA, then RNA - if it's thought to b acute then do RNA, or both quesitoned continue: HCV RNA 4 milliion, genotyp 1A, ALT 42, AST 65, Tbili 1.6, ALb 3.9, Hgb 13.4, Plt 11000, Cr 1.2, HBsAG +, anti-HBc + now what? treat with oral regimen? check HCV 1a resistance test? elastography? confirm HCV ab test? repeat skin exam looking for acral necrolysis? elastography - to stage the disease before treat with oral regimen for 12 weeks - rule out cirrohsis / rule out HCC - and screen for varices resistance testing rarely in right answer in naive patient, but in treatment experinced

21 y.o. male presents with sore throat and fever PX: Exudative pharyngitis/tonsillits; tender cervical lymphadenopathy Rapid Strept test is positive Gives history of non-urticarial skin rash from Pen V-K prescribed for sore throat in high school and told he was forever allergic to penicillin A. Third generation oral cephalosporin B. Clarithromycin C. Pen VK or Amoxicillin D. Trimethoprim--sulfamethoxazole

A: third generation oral cephalosporins Positive history in 10 % of all patients in the US After formal testing, < 10 % of total at risk for IgE mediated acute reaction Penicillin IgE antibody disappears after 10 years Cross-reactivity in patients truly allergic to penicillin : < 5% if challenged with a cephalosporin < 2% if challenged with a 3rd or later generation cephalosporin < 1% if challenged with a carbapenem or aztreonam Immune-Mediated Allergic Reactions Reaction Type Pathogenesis Manifestations: Type l Beta-Lactam binds to membrane-bound IgE on mast cells Urticaria, angioneurotic edema, anaphylaxis (most worrisome) Type II Drug-induced antibody binds to cell-surface drug (antigen) Interstitial Nephritis; hemolytic anemia; neutropenia, thrombocytopenia Type III Circulating immune complexes Serum sickness like reaction: fever, urticaria, lymphadenopathy, arthralgia Type IV Activated T-cells; delayed hypersensitivity • Morbilliform rash • Stevens-Johnson • Toxic epidermal necrolysis • Drug-induced Hypersensitivity (DRESS): fever, rash, MOF, eosinophilia

45 y/o M serum RPR + tx for syphilis 15 y/a w/ PCN, does not remember his titers has few sexual partners over 25 years, does not recall chancres or STD healthy, normal physical RPR titer 1:64, FTA +, HIV ELISA -(ive) ?recs, persuming pt refuses LP? No further serologic follow up or treatment Treat with one dose of benzathine pencillin Treat with three doses of benzathine pencillin Treat with benazathine penicillin for 10-14 days Treat with IV penicillin for 10-14 days

A: treat with 3 doses of benzathine PCN treated for syphilis in past, RPR should fall to less than 1:4 within 5 years. His titer is above 1:4, so it must be presumed that he has either been re-infected or was inadequately treated. has syphilis of undetermined duration:could have early latent syphilis (syphilis acquired in the past one year) or late latent syphilis (syphilis acquired at least one year previously). could have neurosyphilis though there are no findings to support that diagnosis. risk factors including an RPR greater than 1:16 and a possible history of treatment failure. There is much debate about when to do an LP in late latent syphilis and how to interpret the results. The 2015 STD guidelines from CDC do not recommend an LP for adults with late latent syphilis unless there are findings consistent with neurosyphilis, including eye disease or deafness. In the absence of an abnormal LP or findings consistent with neurosyphilis, the guidelines recommend three weekly doses of benzathine penicillin G, 2.4 million units per dose.

22 y/o female vaginal discharge exam gray homogenous discharge swab reveals pH > 6.0, motile trichomonands, and the presecnece of 3 Amsel's criteria tx for her and partner?

A: trichomniasis and bacterial vaginosis -partners of bacterial vaginosis we don't treat - partner gets standard therapy for trichomniasis ( 2 g metronidazole) - BV can't be treatd MEtro x1 week for patient, and 2g x1 for partner

screening and treatment recommmendation of which of the following is different between HIV + and HIV - in a 20 y/o BV CT NG HSV Trichomnoas vaginalis

A: trichomonas vaginalis screen all HIV + women annually, and treatment is 2g metornidazole for longer (unlike HIV-)

50 y/o working in massachusetts a caretaker at a nantucket golf course, mowing the lawn and triming the bushes during the past 3 weeks of the summer painful ulcer on lesion, soon high fever, painful axillary adenopathy on same arm and a dry cough what was he doing?

A: tularemia / farnscilla tuleramia disposing of a dead squirl - fleas and tick on animals and get on you not plague / yersinia pestis (not in massachuset, in colorado and new mexico)

21 y/o F, no STD's, lives in massachusetts, school in SC + knee pain / swelling, +limited ROM x 3 weeks - fever, - trauma, - tickbites, - MRI joint aspiration = 18K with PMN predominance, no crystals, Cx (-)ive test to Dx Lyme?

A: two tier B. Burgdorferi serology *late lyme arthritis, knee common, recurrent, not ill but joint involvement, 100% positive serology *synovial fluid B. burgdorferi serology, 98-100% spec, 60-90% sens (<serum serology), not validated for synovial fluid or CSF)

33 y/o immigran from central america chronic ulcer of the leg leg ulcer progressively enlarged over 3 months bumpe dhis leg on a table course of oral abx no response past year and upset stomach with abdominal cramps and diarrhea intermittent fever likely diagnosis?

A: ulcerative colitis lesions is pyoderma gangrenosum neutrophilic dermatosis -pathergy - painful undermined ulcer with violaceous edges and necrotic base -unlike sweet's fever variably present -indolent disease - minimal trauma with skin lesion over time, does not responsd to abx -painful violaceous or red border and necrotic base

46 y/o DM low grade fever several urinary tract infection juvenile diabetes lost 5 KG left flank tenderness possibl left abdominal mass WBC 4,400, hg 10, Cr 1.4 UA 200 WBC , 150 RBC, 4+ protein, 3+ glucose UCx pending

A: urinary tract infections/xanthogranulomatous pyelonephritis a rare, serious, chronic inflammatory disorder of the kidney characterized by a destructive mass that invades the renal parenchyma. diabetic elderly XGP is most commonly associated with Proteus or Escherichia coli infection

63 y/o M, no medical history 1 week of fever, rigors, and progressive dyspnea BP 160/40 JVD+ loud idalostic decrescendo murmur left sternal bordre +quinckes pulse TTE 1.5 cm oscillating mass Bcx = (+) for GPC in cluster regimen?

A: vancomycin native valve endocarditis literature recommend against gentamicin and rifampin

70 y/o f/c, low back pain for 6 days underwent bioprosthetic AVR 9 months previously for cirtical aortic stensosis Temp left conjuctival petechiae rales grade II/VI SEM BCx: 3/3 positive at 18 hr for GPC in clusters while TEE pending which abx to start?

A: vancomycin plus gentamicin and (later add rifampin) vanc/gent first do decrease bacterial load

41 y/o, visited a community who doesn't belive in vaccination members of community had chicken pox (Varicella) healthy, and does not remmember if he ever had chicken pox, he is sure he didn't get varicalle vaccine Next step? Varicella-zoster immune globulin Valacyclovir Varicella vaccine Measure varicella antibody titer intravenous immune globulin

A: varicella vaccine The chickenpox (Varicella) vaccine is recommend for all healthy adults who are exposed to chicken pox and don't know their immune status. immunization within 3-5 days of exposure makes infection less likely or less severe. Varicella-zoster immune globulin (VariZIG) is recommended for susceptible pregnant women and susceptible immunosuppressed individuals who have had a significant exposure. most effective if given in the first 72-96 hours post exposure. . Valacyclovir is not indicated as prophylaxis for healthy adults. If used to protect immunosuppressed persons from chickenpox, it should be started no sooner than day 3 post exposure. You also can use acyclovir, valacyclovir or famciclovir to treat chickenpox in normal adults if begun in the first 24 hours of infection. Give varicella vaccine to HIV infected persons with a CD4 count of >200; if <200 give acyclovir. Measuring titers to see if exposed person was previously infected may not provide an answer in time to give vaccine and not cost effective.

44 y/o M w/ cardiomyopathy, receives a donor heart from a 26 y/o from colorado who drowned while white-water rafting patient did well for 3 days post-op, but suddently became confused and severe rapidly ascending paralysis over 24 hr WBC 800 w/ 60% PMN's Hgb-9.2 g/dl Plt-42,000/mm3 Cr- 1.6mg/dL AST/ALT- 94/96 U/dL Alk Phos-222U/dL Bilirubin- 1.6 mg/dL LP after platelet transfusion: WBC-16 (100% mononuclears) Glucose 66mg/dL Protein 210mg/dL most likely cause? Cryptococcus neoformans Fusarium solani JC Virus BK Virus West Nile virus

A: west nile virus West Nile infection transmitted by blood product or donated organs cause disease within a few days fever alone, fever and encephalitis, and fever and spinal cord disease. flaccid paralysis, an anterior horn cells, rapid onset asymmetric weakness of the limbs. flaccid paralysis, the viral entities, anterior horn cell disease are enteroviruses (Enterovirus, Coxsackie, Polio, D68) and West Nile. Tick Paralysis could cause anterior horn cell damage but only after exposure in Europe or the Far East and almost always in association with a biphasic disease and concurrent encephalitis. Polio, and now more recently enterovirus D-68, would also have paralysis following a viral prodrome/aseptic meningitis. Very few viral and fungal infections occur within the first few days after solid organ transplantation in patients who, largely, were not immunosuppressed before the organ procedure. Cryptococcus would cause headache, fever, obtundation but not flaccid paralysis. BK causes renal and bladder disease, not neurologic disease . Fusarium causes lung disease and skin lesions, and only very rarely focal neurologic disease. JC virus causes PML, not this syndrome.

CASE 2 34 yo woman diagnosed with HIV 3 years ago Initially presented with PCP Initial Lab values ◦ CD4 82 cells/uL ◦ VL 106,000 c/mL Started on TDF / FTC / EFV (FDC) Did well for a while, then the regimen failed The genotype shows a K65R mutation. Which nRTI drugs would you include? A. ZDV B. TDF C. ddI D. ABC

A: zidovudine tenofovir based mutation - only 1 situation to use zidovudine A. ZDV : Correct Answer B. TDF : TDF won't work well C. ddI : : Awful choice! (Sorry). The K65R pathway is precisely the reason why this combination failed to work as primary Rx D. ABC : Abacavir activity typically reduced with a K65R mutation

5 people ill 30-150 min after eating undercooked tuna steaks +rash, HA, diarreha and abd cramping with onset 30-150 minutes after tuna consumption index pt rash and HA 35 min after eating tuna illness in 5 people resolved without medical intervesion Pervent outbreak by?

A:Adequate refrigeration from time of catch to cooking typical scombroid fish poisoning. prevention proper refrigeration of fish at <40°F (<4.4°C) at all times between catching and consumption. Sensory examination (i.e., by smell and taste) is not sufficient to detect the absence or presence of histamine; chemical testing is required. Histamine is not destroyed when fish are frozen or cooked, making adherence to temperature requirements along all stages of the food supply chain essential. Treatment involves antihistamines and supportive care.

16 y/o M from baltimore, + hay fever, +allergic rhinitis uses nasal sprays with a decongestant or corticosteroids + hyphae seen in histopath of sinus content initially painless proptosis of L eye CT shows proptosis caused by ethmoid sinus mass EOM and VA normal transnasal ethmoidotomy and maxillary antrostomy, sticky dark mucus was curetted from the sinuses felt that the bony septae separating the ethmoidal air cells were soft but the maxillary sinus walls were intact sinus curettings contained occasional hyphae on periodic acid Schiff stain, mucus, strands of eosinophils, and occasional Charcot-Leiden crystals. Hyphae were seen in the mucus but not the submucosa. Culture at 5 days showed a few small colonies of a brown mold, which had not sporulated and could not yet be identified most appropriate therapy? Complete surgical removal of sinus contents Itraconazole Amphotericin B intravenously Posaconazole Amphotericin B nasal spray

A:Complete surgical removal of sinus contents Allergic fungal sinusitis occurs in older children or adults, often with a previous history of allergic rhinitis, with or without nasal polyps. Diagnosis depends on histopathology. Culture should be attempted but the dark-colored molds that cause this disease are diverse and difficult to identify. The most effective therapy is complete surgical extirpation of the infected tissue and establishing good drainage by generous sinusostomy. Systemic antifungal therapy does not diminish the extent of current disease but has a role when infection extends to or across the dura, a feature absent in this patient. Topical antifungals are not useful. Some ENT physicians prescribe corticosteroid nasal sprays after recovery from surgery, with the intent of keeping sinus passages open.

76 y/o HIV + on TDF -FDC + darunavir/ritonoavir originally VL 123,000, now suppressed CD4 originally 235, now 200-300 ?Continue current regimen. Change tenofovir/emtricitabine to abacavir/lamivudine. Change darunavir/ritonavir to efavirenz. Add maraviroc to current regimen. Add dolutegravir to current regimen

A:Continue current regimen The patient has experienced an optimal virologic response with antiretroviral therapy, but has a suboptimal CD4 cell recovery. Older age (among other reasons) has been associated with a suboptimal CD4 response. There is no proven strategy to treat a suboptimal CD4 response. Specifically, changing or adding antiretroviral drugs, including maraviroc or an integrase inhibitor (the other choices) has not been associated with be nefit. The patient should be reassured and the current antiretroviral drug regimen continued.

26 y/o F, receiving acupuncture for depression four monther later deveoped 1-3 cm lesions at accupuncture site biopsy shows granulomas with AFB, cx pending... best option? Do nothing until culture result is available. Treat with INH, rifampin, PZA, and ethambutol. Follow for several months and treat if they do not resolve. Perform excisional biopsy of the three lesions. Treat with rifampin and ethambutol.

A:Do nothing until culture result is available Most of the cases of cutaneous mycobacterial infection due to trauma, plastic surgery or minor surgery have been due to M. abscessus ,M. chelonae, or less often M. fortuitum.. The organisms can be recovered in culture if the diagnosis is considered but the isolates can be difficult to identify. A punch biopsy for AFB culture is indicated. M. chelonae is often susceptible to clarithromycin, ciprofloxacin and doxycycline. A two-drug regimen with clarithromycin and doxycycline for six months is usually adequate in nonimmunosuppressed patients. M. abscessus subsp.abscessus are frequently resistant to all oral antibiotics except clofazimine, with rare susceptibility to linezolid, flouroquinolones, or macrolides. Most M. abscessus and a high percentage of M. fortiutum have inducible macrolide resistance. The excision of three lesions seems extreme since these are on the neck. However, abscesses due to M. chelonae are not cured with simple drainage of the soft center. Dual-agent therapy is preferred to prevent acquired drug resistance. Acquired fluoroquinolone resistance has been demonstrated previously with ciprofloxacin monotherapy for cutaneous M. fortuitum infection and acquired clarithromycin resistance has been shown in the single-agent treatment of cutaneous infection caused by M. chelonae.

34 y/o M with HIV (CD4 802, VL <50) w/ +N/V and LUQ pain x 1 week; no fever, no skin disease pain worse with inspiration, + DOE CT shows, effusion involving the pleural space and pericardium, no mass Evaluation of the fluid shows mononuclear cells but no neutrophils, and all routine microbiologic evaluations have been negative. What virus is associated with a malignant process that would characteristically present with pleural effusion, pericardial effusion, and/or ascites in this type of patient? EBV CMV HHV 6 HHV 8 HSV

A:HHV 8 This is primary effusion cell lymphoma which is associated with HHV 8. This virus is also associated with Kaposi sarcoma and multicentric Castlemans disease. EBV is associated with primary CNS lymphoma in this population. Neither HSV nor HHV6 is associated with tumors in patients with HIV. The teaching point: when there are large pleural, pericardial, and peritoneal effusions in patients with HIV counts, consider HHV 8 related primary effusion cell lymphoma in the differential diagnosis.

A cardiac surgeon found HIV + (CD4 70), VL 300,000) now VL <50, feels well likes to keep operating, what do you tell him?

A:He may continue his practice if he adheres to hospital policies regarding HIV infected employees, has viral loads <50 copies, follows infection control guidelines, has satisfactory performance reviews, and there is no evidence of transmission of his infection. no state or Federal laws which prohibit the practice of surgery based on HIV status. there are guidelines issued by Society of Healthcare Epidemiologists of America (SHEA) that many hospitals have adapted. If he has a viral load >50 copies, there are professional society guidelines and some hospital policies that indicate a need to restrict the infected healthcare professional from "procedures with definite risk of blood borne pathogen transmission," which are hard to define (see table below for how the Society of Healthcare Epidemiologists of America define such procedures, but they are just one of many voices on the topic.) Since the sensational news about a Florida dentist transmitting HIV to his patients were reported in 1993, which has never been satisfactorily explained, there have been only 3 other likely cases of transmission from healthcare provider to patient. Thus, HIV transmission from healthcare provider to patient is very rare in the current era, and less common that HBV or HCV transmission. Take a quick look at this table regarding HBV, HCV and HIV with regard to managing infected health care practitioners. This is, however, only a guideline, and not a law.

60 y/o carpenter from china to US, healthy large circumscirbed raised, exophytic and crusted rash on his thigh , indolent for 27 years without pain or warmth remember getting splinter sin his thigh no F, wt loss, lesion not numb tender or pruritic x-ray, labs normal Dx? Tx? Trimethoprim-sulfamethoxazole Rifampin-clarithromycin Nifurtimox Itraconazole Dapsone

A:Itraconazole chromoblastomycosis. dark colored moulds. inoculated into the skin by wood splinters, and causes cutaneous plaques. histopathology brownish thick walled round fungal cells, called "copper pennies" in the subcutaneous tissue. Therapy : surgical excision, cryotherapy, and thermal therapy Itraconazole for many months is usually effective. lesion is too chronic for sporotrichosis and blastomycosis does not occur in China. Trimethoprim-sulfamethoxazole for Nocardia brasiliensis, but more subacute and present with lymphangitis spread. Rifampin-clarithromycin Mycobacterium marinum, but there is no exposure history to water lesion is too large and indolent Nifurtimox Chagas disease, but lesion too indolent for a chagoma and there is no Chagas'disease in China. Dapsone would be appropriate for leprosy. If this were lepromatous leprosy, it would not remain the only lesion for 27 years

26 y/o student fever and cervical lymphadenopthy well until 9 days ago when she had the acute onset of fever and vague neck discomfort no sore throat and no dental or scalp problem T 101.4 unilateral anterior and posterior cervicla enlarged lymph nodes, mobile , firm and mildly tender Hgb 13.9, WBC 4.9 (9% atypical lymphocytes) CXR normal ESR 72 monosopt negative serological studies: EBV -consistent with prior infeciton CMV, toxo, bartonella titers negative RF, ANA, ds-DNA negative lymph node pathology: necrotizing lymphadenitis with histiocytic infiltrate and phagocytosed debris stains for AFB and fungi negative

A:Kikuchi's disesae Buzzwords: acute onset of fever + cervical adenopathy -atypical lymphocytes (mono-like syndrome) -path: necrotizing adenitits with histiocytosis Fever Cervical adenopathy (unilateral) atypical lymphocytes and leukopenic (could mimic mono) dx: lymph node biopsy (histiocytes infiltrates)

HIv infected, resistant to most ART Trofile assay reveals dual-tropic (R5 + X4) virus Use of maraviroc in this setting?

A:Maraviroc should not be used in this patient Maraviroc, a co-receptor inhibitor, is only effective against HIV-1 that exclusively utilizes the R5 co-receptor should not be used for patients with X4 virus or dual-tropic virus

74 y/o F + fever for 2 weekd duration, + morning stiffness >30 min, pain in both shoulders (worse with movement and interfers with sleep) started on statin for HLD 2 months ago no joint swelling, muscles nontender normochromic normocytic anemia ESR 104, CRP >x5 normal Dx? Adult Still's disease Polymyalgia rheumatica Statin toxicity Rheumatoid arthritis Fibromyalgia

A:Polymyalgia rheumaticais >50 y/o, women. morning stiffness in shoulders, hip girdle and neck along with pain with movement in these areas. Muscles non-tender (sed. rate often >100mm/hr); mild anemia. Adult Still'sis associated with a rash, fever, and leukocytosis. Statin toxicitymay cause a diffuse myositis, but does not produce morning stiffness. Onset of rheumatoid arthritisat age 74 would be very unusual, and joint involvement is typically small joints of the hands and feet. Fibromyalgiatypically occurs in younger persons and physical exam is normal except for point tenderness; ESR and CRP are normal.

renal transplant receipient and related donor both from rural egypt await renal transplant both hx of shistosomiasis, S.mansoni and S. haematobium and both were treated on multiple occasions transplant surgeon asks? Serology would be helpful in ruling out active infection. Praziquantel can kill the schistome worms but not the eggs. Biopsy of rectum and bladder to search for eggs would help determine if schistosomiasis is active. If any worms are alive at transplantation in the recipient, they can multiply to cause a massive infection. Schistosoma haematobium might be transmitted by the donor kidney

A:Praziquantel can kill the schistome worms but not the eggs. Schistosomiasis, complicated life cycle, cannot multiply in the definitive host, humans. Immunosuppression does not result in worm multiplication. Likewise, schistosome ova do not multiply in the body. Active infections are characterized by the excretion of live eggs. Dead eggs in S. haematobium and to some extent in S. mansoni are retained in the tissues and shed in the feces or urine but do not constitute an active infection and require no treatment. Available serologic tests remain positive in inactive chronic infections. There is no reason not to treat active infections. Treatment is fast, safe and reasonably effective in killing the worms. Although the eggs are not killed, egg production stops with killing of the worms. The transplant has no bearing on treatment of the donor: his kidney will not transmit schistosomes to the recipient. He should be treated only to eradicate live worms in his body which might still be excreting eggs.

33 y/o F, from philippines, chronic nasal stuffiness, now w/ 3 episodes of cellulitis of the left ear 1st, 17 months ago 2nd 8 months ago most recent, 4 day ago two earlier episodes respond slow to abx on exam, pinnea is red, tender except for the lobe has ear peiricng since childhood, no ear trauma, + saddle-nose deformity ?Dx Recurrent streptococcal cellulitis Relapsing polychondritis Syphilis Granulomatosis with polyangiitis Leprosy

A:Relapsing polychondritis Relapsing polychondritis is an inflammatory condition of cartilage, particularly ears, nose, eyes, joints, and airways. Recurrent episodes lead to deformities such as "floppy" ears and saddle-nose deformities. Bacterial cellulitis of the pinna is very rare; recurrent episodes even more rare and it wouldn't explain the nasal stuffiness and saddle-nose deformity. Syphilis and Granulomatosis with polyangiitis (formerly Wegener's disease) may all result in a saddle-nose deformity but would not cause inflammation of the external ear. Lepromatous leprosy can cause deformity of the pinna but there is no loss of cartilage. Rather, there is nodular thickening of subcutaneous tissue in the ear and other subcutaneous tissues.

6 employs in oil refinary ill within 2 hours of the meal +facial flusshing, pruritis rashes on face, neck and trunk, heart palpitation and diarrhea ate mixed green, blue chees, tuna, wonton soup, and key lime pie Cause?

A:Scombroid Scombroid fish poisoning is an acute illness that occurs after eating coliform contaminated "spoiled" fish containing high levels of histamine or other biogenic amines. symptoms -facial flushing, sweating, rashes, burning or peppery taste in the mouth, diarrhea and abdominal cramps. More severe respiratory symptoms may occur. Ciguatera is a neurotoxic disease with neurologic symptoms (e.g. paresthesia and numbness) which were not reported. MSG toxicity can produce symptoms like scombroid but more often occur more quickly than 2 hours and rash is not part of this toxicity. Staphylococcal and Bacillus cereus food poisoning do not cause rash or facial swelling.

37 y/o M + needlestick exposure, 60 min ago, from HIV + man (the patient) the patient on ABC/3TC + darunavir/ritonoavir with VL of 62, and CD4 of 553 ? managment? No post-exposure prophylaxis (PEP). Start tenofovir/emtricitabine/efavirenz X 4 weeks. Start tenofovir/emtricitabine + atazanavir/ritonavir X 4 weeks. Start tenofovir/emtricitabine + dolutegravir X 4 weeks.

A:Start tenofovir/emtricitabine + dolutegravir X 4 weeks A needlestick exposure is a reason to start post-exposure prophylaxis (PEP). The source patient has a low, but not completely suppressed, HIV RNA. The CDC recommended regimen for PEP is tenofovir/emtricitabine + dolutegravir (or raltegravir).

23 y/o chillian male a week at chilli went to western paru, white water rafting, numerous insect bites two weeks after return noticed pruritic red papule on this arm, which enlarged and show did not repsond to cephalexin, biopsy showed chronic inflammation with abdundant histiocytes, plasma cells and lymphocytes but no gian cells Bacteria/fungi cx negative, no fungus over 4 weeks

A:cutanous leishmeniasis from sand fly

25 y/o spelunker bitten by a bad 6 days ago never rceived rabies vaccine: what to do?

A:he should receive HRIG + vaccine today, then in 3,7 and 14 days incubation 1-3 months, can be days to years pre-exposure (3 doses)on D 0,7,21 or 28 post-exposure (4 doses) 0,3,7,14 (if received should get 2 doses (0,3) immunocompromised, 5 doses, 0,3,7,14,28 if pre-vaccinated, no RIG

46 y/o AA 1 week of anorexia, nausea, and dark urine unprotected sex with male partners donated blood 6 months ago, no problems with blood ALT 1115, AST 846, Tbilli 6.2 HIV PCR -(ive) Hepatitis Panel: Hepatitis B anticore IgG and IgM positive Hepatitis B surface antigen and e antigen positive, antibody to surface antigen and e antigen negative Hepatitis B viral load 1,000,000 copies/mL Antibody to HIV, hepatitis A, C and D negative Appropriate treatment? Entecavir Lamivudine Adefovir plus lamivudine Alpha interferon 2b plus ribavirin Observation only

A:obersvation only has hepatitis B. Acute hepatitis B clears in >95% of immunocompetent adults, and thus treatment is not indicated for acute disease. Considerations are different for infants and children, and for immunocompromised adults. Distinction between acute hepatitis B and an acute exacerbation of chronic hepatitis B is difficult. He had donated blood six months ago, indicating that he was likely hepatitis core IgG antibody negative at that time. Also, his anticore IgM is still positive, consistent with acute hepatitis B. Viral load is not helpful in the distinction. Most patients will resolve acute hepatitis B spontaneously, developing antibody to surface and to e antigen and lose surface antigen positivity. Thus there is no urgent reason to treat this patient.

44 y/o anti-HCV and HCV RNA + , feels bad after recent etoh bing chronic rash on arms that is worse and elevated ALT and AST likely Dx Cirrohsis to HCV and etoh vibrio porphyria cutane tarda essential mixed cryoglobulinemia yirsinia

A:porphyria cutane tarda association with hepatitis C with porphyria cutanea tarda, cryoglobulin vasculitis, and lichen planus also rare syndrome is necrolytic acral erythema, respondive to zinc know for hepatitis B association with polyarteritis nodosa (PAN)

47 y/o F HIV + CD4 405 VL 213,000 HAV ab + HBV sAb - HBV sAg + HCV Ab - repeat CD4 352, VL 234,000 optimal initial ART? tenofovir/emtricitabine + rilpivirine raltegravir + efavirenz + maraviroc abacavir/lamivudine + zidovudine tenofovir/emtricitabine + darunavir + ritonavir

A:tenofovir/emtricitabine + darunavir + ritonavir needs optimal treatment of 2 infections: HIV (with 3 active drugs) and HBV (with 2 active drugs) emtricitabine, lamivudine, and tenofovir (both TDF and TAF) have activity against both viruses triple-nucleoside combination of abacavir/lamivudine/zidovudine is not recommended for HIV treatment due to suboptimal virologic activity. Also, rilpivirine-based regimens are NOT recommended for patients with baseline HIV RNA>100,000 copies/ml because of suboptimal virologic response

• 52 yo female S/P cadaveric renal transplant receiving tacrolimus, prednisone and mycophenylate. • Week 30 post transplant serum creatinine rose from 1.5 to 2.3 mg/dl. • Tacrolimus levels were in therapeutic range. • Urinalysis revealed one plus protein and no cells or casts. Which would be most helpful in understanding if BK virus was causing her renal failure? A. Presence of decoy cells in urine cytology B. Urine BK viral load C. Urine culture for BK virus D. Plasma BK viral load E. Demonstration of BK inclusions in renal tubular epithelium on renal biopsy

ANswer E Replication in urine precedes replication in blood precedes nephropathy Renal Bx - "Gold Standard" for diagnosis Blood PCR • Sensitive (100%) but less specific (88%) • Cannot rule out rejection • Useful as indicator for biopsy Urine Cytology, Electronmicroscopy, & PCR • Detection in urine: Low PPV but High NPV Hirsch et al, Transplantation 2005;7

A 20-year-old male with refractory lymphoma received an HLA-mismatched and T cell depleted allogeneic stem cell transplant He engrafted his WBCs on day 22, and developed a faint diffuse erythematous rash and low-grade fever on day 68, diagnosed as acute graft vs. host disease, for which he was treated with prednisone, which was ultimately tapered. The rash faded and he became afebrile. He is receiving trimethoprim-sulfamethoxazole three times a week and twice daily valacyclovir for prophylaxis. On day 110, he developed fever to 38°C, dyspnea, cough, and a faint erythematous rash. His WBC was 7,000, with a normal differential. Chest CT scan demonstrated bilateral ground glass opacities but cultures and/or stains of a bronchoalveolar lavage for bacteria, fungi and pneumocystis were negative. Alkaline phosphatase was 309 U/L, AST 488 U/L, ALT 430 U/L, total bilirubin 1.9 mg/dl. Urinalysis revealed hematuria: 1500 RBC, 20 WBC, with no bacteria on stain. His CMV PCR on peripheral blood was undetectable. What is the most likely cause of this syndrome? HSV VZV HHV6 Adenovirus BK Virus

Adenoviral diseases are well characterized in hematopoietic cell transplant (HCT) recipients. Manifestations include pneumonia, colitis, hepatitis, hemorrhagic cystitis, tubulointerstitial nephritis, encephalitis, and disseminated disease--and this patient has many of these manifestations. The T cell depleted graft and graft-versus-host disease are both risk factors for disseminated adenovirus disease. Diagnosis of adenovirus disease in this setting is complicated: serum PCR assays, sterile fluid PCR, bronchoalveolar lavage PCR and tissue biopsies all have roles. There is no specific therapy that is clearly effective although T cell infusions and perhaps cidofovir may have roles. Adoptive T-cell immunotherapy is in clinical trials. The hematuria suggests adenovirus or BK virus, but BK virus would not cause the pneumonitis, hepatitis, or other manifestations outside the bladder in a stem cell recipient. The skin rash is likely mild GVHD. The abnormal liver function and pulmonary infiltrates could be GVHD or adenovirus. HHV6 reactivation is common in patients post-transplant and may cause encephalitis but not cystitis. Diffuse pneumonia has not been clearly linked to HHV6 reactivation. HSV and VZV would be unusual causes of hematuria, and unusual in a patient receiving valacyclovir.

county health department seeks your advice about whether to purchase the quadrivalent meningococcal conjugate vaccine, considering that the cost per dose is so high that the money will have to come from cuts in other services ?Advise?

All adolescents ages 11 to 18 years old should be given this vaccine, thus the county should purchase this vaccine. quadrivalent vaccine that is effective against strains A and C and Y and W 135 as C and Y are responsible for much of the meningococcal disease in the United States. The quadrivalent vaccine does not provide protection against serogroup B, which accounts for a substantial fraction of disease in the United States in crowded situations such as dorms and barracks, but that is not a reason to avoid the vaccine. avoid in patients with a history of Guillain-Barré syndromes. ACIP recommended this vaccine for all adolescents age 11 to 18 years of age and for children 2 months to 11 years of age who may be at increased risk of meningococcal disease, for example, travel to endemic areas or asplenia. For otherwise healthy children the first doses should be given at age 11-12 and the second at age 16-18, for aplenic or some immunocompromised children initial 4 dose series at 2 months of age and then boosters every 5 years. In October 2014, a vaccine for meningococcus type B was approved. for the first time: this is indicated for individuals aged 10-25 years. At the moment, routine use of this vaccine is not recommended. It has been used to prevent spread of infection on college campuses during Group B outbreaks and among laboratory workers who might have been exposed.

40 y/o M, healthy 4 days +fever, chills WBC 3.7, HCt 34%, Plt 40, ALT 40, AST 97, peripheral smear blue cytoplasmic inclusions suspicious for morulae in a neutrophil ?ddx? Babesia microti Anaplasma phagocytophilum Ehrlichia chafeensis Southern Tick Associated Rash Illness (STARI) Bartonella elizabethae

Anaplasma phagocytophilum Anaplasmosis, a tick-borne rickettsial infection, peripheral blood smear showed blue cytoplasmic inclusions suspicious for morulae in a neutrophil. The presence of morulae in white blood cells rules out the other potential diagnoses, such as meningococcus (a gram negative diplococcus), babesia (a tick-borne protozoal infection of red blood cells, not white blood cells), histoplasmosis (a fungal infection which may appear as inclusions in white blood cells), or listeria (a gram positive rod). His PCR was positive for Anaplasma phagocytophilum. There are three main tick-borne diseases in the United States: Rocky Mountain Spotted Fever (RMSF); human monocytotropic ehrlichiosis (HME); and anaplasmosis. These infections usually present in spring and summer months (April - September). RMSF and HME are most common in southeastern & south-central US. Anaplasmosis is most common in New England, north-central states and focal areas of the West Coast. Approximately 30-40% of patients do not recall tick bite Anaplasmosis is caused byAnaplasma phagocytophilum, which is transmitted by the Ixodes scapularistick. Symptoms may include fevers, chills, malaise, myalgias and headache. Rash is present in <10%. Labs may show leukopenia, thrombocytopenia, and/or elevated transaminases. Diagnosis is made by visualization of intraleukocytic bacteria (morulae) on blood smear (present in 20-80% of cases), serology or PCR. Anaplasmosis is usually treated with doxycycline. Because they are transmitted by the same tick, coinfection with Lyme and Babesia should be considered and excluded.

A 43-year-old HIV+ man has CD4 600-900 and HIV RNA consistently <200 copies over the last 8 years. Do you recommend starting ART? A. Yes, all current guidelines recommend starting. B. Yes, there are theoretical benefits to starting. C. No, he should wait until his viral load level is confirmed >1000 copies/ml. D. No, he should wait until CD4 is confirmed <500.

Answer - A

A 68‐year‐old F with CLL s/p six cycles of fludarabine plus rituximab. Relapse 6 months later, received alemtuzumab. 2 months later with fevers, anorexia, and "hurting all over". Exam ‐ anterior cervical lymphadenopathy, liver span of 15 cm, and a palpable spleen. Hemoglobin 7 g/dl, WBC 1.9 X 109/L, platelet count 40 X 109/L, LDH 500, AST 30, ALT 50, tb 1.8. What is the most likely cause of her current symptoms? A. Cytomegalovirus (CMV) B. Epstein barr virus (EBV) C. Streptococcus pneumoniae D. Candida albicans E. Adenovirus

Answer A Anti‐CD52 Ab (Campath/alemtuzumab) • Profound defects in cell‐mediated immunity / like a chronic HIV • Risks last approximately 6 months • Reactivation of cytomegalovirus is the most commonly occurring infectious complication with use of this agent: reactivation from 10‐25% • Other risks: PCP, IFI (T cell)

• 18F c/o fever, dry hacking cough, malaise x 3d • Allergy: erythromycin (N/V) • Appears well, T38ºC, RR 16, P 80, BP 110/70 Oropharynx: normal TMs: normal Chest: some crackles left lower lobe • Azithromycin prescribed • Next day, full body rash and mucosal lesions develop What is the most likely etiology? A. Mycoplasma pneumoniae B. Enterovirus D68 C. Measles D. Candida albicans E. Drug reaction (azithromycin)

Answer A Enterovirus - walking pneumoina, no oral lesions measles - koplik spots candida - never pneumoina drug eruption - after 1 day, rather quick Mycoplasma pneumoniae • "Walking pneumonia" CXR: appears worse than patient • < 10% may have extra-pulmonary manifestations Stevens-Johnson syndrome (SJS), E. multiforme oMost common infectious cause (children/adolescents) oMale > female Hemolytic anemia Hepatitis CNS: encephalitis, meningitis diagnostic - PCR is gold senstiive - can have hemogluttin +

62M living in Arizona presents in early September with a three day history of fever, myalgia, headache and rash. He works as a electrical lineman for a utility company. He lives with his family in an older adobe home with dogs. He beginnings of petechial features on the wrists and ankles. Which of the following is the most likely diagnosis? A. Rocky Mountain Spotted Fever (RMSF) B. Human Monocytic Ehrlichiosis (HME) C. Human Granulocytic Anaplasmosis (HGA) D. Babesiosis E. Tularemia

Answer A acquired in town hide in walls dogs can carry them human monocytic ehrlichiosis nor human graulocytic anaplasmosis not found in arizona babesia not show up with petechiae

31M presents in January with 3d fever, HA, malaise, and myalgia. Works as counselor at wilderness camp in Pennsylvania. Flying squirrels common at camp including residing in the walls of his cabin. Exam is notable only for fever (39.60; no rash), tachycardia (P110) A diagnostic test for which of the following is most likely to be positive A. Murine typhus B. Epidemic typhus C. RMSF D. Tularemia E. Relapsing fever

Answer B

Which one ofthe following is FALSE regarding the 48 yo man admittedwith inhalational anthrax <12 hours ago? A. Azithromycin, although not the drug of choice, probably is an effective therapy for anthrax B. Staff who cared for the patient prior to the recognition of the anthrax dx should initiate ciprofloxacin or doxycycline prophylaxis C. Blood agar plates in the lab are potentially sources of 2˚ cases D. Most automated blood culture systems will identify B. anthracis in this patient's blood E. Switch antibiotics to ciprofloxacin, meropenem and linezolid

Answer B

42 year old M AML 20 days after a matched unrelated donor BMT (nonmyeloablative) develops fever, cough, pulmonary infiltrates. Pre‐transplant: HSV+, VZV+, CMV D+/R‐ Exam- 98% sat on 2L nc, T 38.3, crackles RLL Labs‐ Cr 2.2, WBC 1200 cells/mL, plt 122 He's currently receiving acyclovir and fluconazole for prophylaxis. What is the most likely cause of his current process? A. Candida albicans B. Staphylococcus aureus C. CMV D. Parainfluenza virus E. Hemorrhage

Answer B Candida - never lung disease Hemorrhage - more diffuse infiltrates , not common in non-ablative conditioning, but can happen with low platelets Parainfluenza virus , no URI symptoms Timeline neutropenia - inhale with mucositis, bacterial infecitons CMV usually later infection

A 20 year old woman is 18 days out from HSCT in January on and engrafted 3 days ago. • She develops fever, hypoxemia, bilateral lung infiltrates and is intubated. • A nasal swab is negative by rapid test for influenza. Which of the following is the most appropriate course of action (regardless of other actions you may take)? A. Do not initiate anti-influenza therapy due to result of rapid test. The timing suggests idiopathic pulmonary syndrome (engraftment) B. Initiate anti-influenza therapy empirically and send tracheal aspirate or BAL for influenza PCR C. Send IgG and IgM for influenza D. Send RSV EIA and initiate empiric IV ribavirin

Answer B Performance of all tests depends on prevalence of virus in community and specimen quality - a negative test has very bad NPV • Clinical diagnosis: up to 80% PPV during peak • Rapid influenza detection tests have low-moderate sensitivity 50- 70% (less for H1N1); reasonably specific • Positive test in peak season high PPV; negative test should not be used for decisions • PCR is the current gold standard, rapid platforms expanding • Serology useless for clinical diagnosis - can hav enegative test in immunocompromised, get a lower respiratory tract testing in immunocompromised RSV - potential, but no empiric ribivirn

A 54 yo woman presents with fever, chills, and oliguria one week after travel to Malaysia. Vitals: 39.0°C, HR 96/min, RR 24/min, BP 86/50 Notable labs: Hct 31%, platelets14,000/l, Cr of 3.2 mg/dL. Peripheral blood smear has intraerythrocytic forms that are morphologically consistent with Plasmodium malariae. The most likely infectious agent causing the patient's illness is: A. Plasmodium malariae B. Plasmodium knowlesi C. Plasmodium vivax D. Plasmodium falciparum E. Babesia microt

Answer B - clincially appeared like falciperum, but morphologically like malariae (usually doesn't cause severe disease) P. knowlesi diagnosed in over 120 people in Malaysian Borneo usually a parasite of macaques

67M COPD, alcoholic liver disease, diabetes, pancreatic CA POD #5 s/p Whipple developed nausea, vomiting, fever, cough, confusion and hypoxemia respiratory failure Labs WBC 18,000 15%B, 60%P Glucose 310 Na 128 sCr 1.7 Intubation ICU, respiratory sample: Heavy PMNs, no organisms on Gram stain Therapy: Vancomycin and piperacillin/tazobactam x 3 d No improvement, febrile, respiratory culture negative ID consultation called You are aware of a recent Legionella mcdadei outbreak in the hospital. Which test below, would most help you securing a diagnosis of L. mcdadei pneumonia? A. Legionella urinary antigen B. Legionella culture of respiratory secretions C. Legionella PCR, respiratory D. Legionella direct fluorescent antigen (DFA) stain of respiratory sample E. Paired Legionella acute/convalescent serology

Answer B - most diagnostic deal with legionella pneumophila serotype 1 - need legionella culture for any other ones -travel beyond home (e.g hotel, hospital, amusement park) - at risk > 50, CL&L d - environmental, but can be nosocomial via water systems Culture: sens 20-80, spec 100% Urinary Ag: sens 70-100, spec 95-100; only serogroup 1 PCR unknown DFA sen 25-75, spec 95%, demanding paired serology, sens 80-90, spec >99

• 43F visited southern Missouri on vacation, returns 7d later with fever, headache and diffuse myalgia x 3d • Physical examination: no findings • Laboratory evaluation : • WBC: 2.1/mm³ (80% PMNs, 10% lymphocytes, 8% monocytes • Hemoglobin: 7.0 g/dL, hematocrit: 24% • Platelets: 105,000/mm³ • AST: 364 U/L, ALT: 289 U/L • renal function: normal Which of the following is the most likely etiologic agent? A. Anaplasma phagocytophilum B. Ehrlichia chaffeensis C. Borrelia hermsii D. Babesia divergens E. Borrelia burgdorferi

Answer B ehrlichia chaffeensis - thrombocytopenia - luekopenia - elevated LFT - morulae in monocyte anaplasmosis does'nt occur in missouri If rocky spotted mountain fever also a consideration

23M cough, malaise, dyspnea, fever x 1 wk, just returning from overseas PMH: negative, no asthma Meds: atovoquone/proguanil ROS: no diarrhea, had rash on feet/legs post marathon now resolved SH: Laguna Phuket (Thailand) triathlon 3 wks earlier Non-smoker PE: Appears ill, BP 98/70, P 100 T 38.5ºC No lymphadenopathy Bronchial breath sounds lower fields, occasional wheezing No murmur No hepatosplenomegaly, abdominal tenderness No rash WBC 18,000 63N, 13L, 24E CXR: bilateral patchy infiltrates Blood smear: no parasites Which of the following is the most likely explanation? A. Allergic bronchopulmonary aspergillosis B. Hookworm infection C. Malaria D. Tropical pulmonary eosinophilia E. Drug reaction

Answer B - either most common helmith diseaes, ascaris lumbricoides - or either two of hookworms ancylostoma duodenale, and necator americanus - AAN - fecal oral transmission Löffler's syndrome • Fever, malaise • Respiratory symptoms: none—mild—moderate • Migratory pulmonary infiltrates • Peripheral eosinophilia • Migration of parasites • Dx: Larvae in respiratory specimen Stool O & P • Treatment Anti-helminthics Corticosteroids May spontaneously resolve Acute or chronic eosinophilic pneumonia • Helminthic Migration (Loffler's) o Ascaris o Hooksworms o Strongyloides Lung invasion o Paragonimiasis • Tropical Pulmonary Eosinophilia o Wuchereria bancrofti o Brugia malayi • Idiopathic hypereosinophilia • Acute eosinophilic pneumonia • Chronic eosinophilic pneumonia • Allergic bronchopulmonary aspergillosis (ABPA)

48M presents in October with fever and rash. Supervisor for apartment bldg in Queens, NY. Exam: T 390C brown-black 8mm eschar on RLE ~30 papulovesicular lesions on trunk Which of the following Is the most likely etiologic agent? A. R. rickettsii B. R. parkeri C. R. akari D. R. conorii E. Borrelia recurrentis

Answer C

53 year old F 7 yrs s/P allo BMT presents with fever, chills, rigors. H/O severe chronic GVHD skin. PE - T 39.2. tachycardia, tachypnea, hypotension. Skin thick, cracked (Sjogren-like). Social- dog and two cats, no recent exposures. Labs- WBC 8200 / mm3, platelet 43,000/mm3. CT of her chest, abdomen, pelvis - splenic atrophy. Blood cultures positive for gram-negative rods after 5 days. Most likely cause of her current condition: A.Fusobacterium nucleatum B.Eikenella corrodens C.Capnocytophaga canimorsus D.Acinetobacter baumannii

Answer C

70 yr old woman with AML, neutropenic for 15 days, s/p induction chemotherapy develops fever, diarrhea, and abdominal pain. Exam ‐ decreased bowel sounds and tenderness with deep palpation in her RLQ. CT shows inflammation in cecum. Levofloxacin and fluconazole prophylaxis. 4 days prior to her admission for chemotherapy, she ate Chinese food. Which is the most likely etiology? A. Norovirus B. Clostridium difficile C. Mixed anaerobic and aerobic bacteria D. Candida albicans E. Bacillus cereus

Answer C Neutropenic enterocolitis (typhlitis) - Necrotizing inflammation with transmural infection of damaged bowel wall - Mixed infection with gram‐negative, gram‐ positive, anaerobic bacteria, fungi - Can be accompanied by bacteremia • Hint: mixed, anaerobic (C. septicum, C. tertium, B. cereus) - Medical and (less often) surgical management

43 yr old F with AML with fever, cough and nodular lung lesions 20 days after induction therapy. On meropenem, fluconazole, acyclovir. Voriconazole begun for presumed aspergillosis. CT scan 10 days later showed lesion doubled in size with slight cavitation. ANC is 800. A. Change voriconazole to liposomal amphotericin B B. Change voriconazole to posaconazole C. Continue to follow on current therapy D. Add micafungin E. Bronchoscopy for diagnosis

Answer C cavitation - lesions get worst, cavitate then get better over time 2nd best answer is to get bronchoscopy mucor not in this population and lesion is cavitating and better Aspergillus species most common • Nodular, tracheobronchial abnormalities (sometimes with 'halo') that enlarge before necrotizing • Alternative microbial diagnosis - Fusarium, Scedosporium, others - Mucormycoses (voriconazole resistant, reverse halo sign)

Question 2: A 33 year old woman is traveling to Tanzania to do field studies in anthropology. She is two months pregnant. Which of the following do you prescribe for malaria prophylaxis? A. Doxycycline B. Chloroquine C. Mefloquine D. Atovaquone/progruanil E. No prophylaxis

Answer C usually pregnant patient shoudn't travel if travel, avoid mosquito bites only ones with experience is chloroquine or mefloquine in pregnancy choose mefloquine (not in SE asia) cause she's going to an area with chloroquine resistant malaria (anywhere besides central america and middle east)

A 32 year-old female with HIV infection VL = 100k, and a CD4 count below 10 cells/mm3has failed all available regimens. Her mother brings her to clinic because of confusion for 1-2 weeks. She is afebrile, oriented x 1, and slow to respond. She has nystagmus and CN VI palsy on the right. The MRI is showing periventricular white matter enhancement. Which PCR test would support the diagnosis that is most likely in this case? A. JC B. EBV C. CMV D. HHV6 E. HHV8

Answer C CMV encephalitis periventricular enhancement CD< 50 rapid onset focal CN findings or nystagmus CSF pleocytosis is neutrophilic rapid progression HIV dementia - large ventricles PML - white matter lesion Toxo - large focal mass - ? lymphoma CMV - periventricular enhacement

• 55 M 6d fever, malaise, severe headache, dry cough • PMH: HTN • Meds: Lisinopril/HCT • SH: Married, suburban Maryland, Works in long-term care facility Visited pet shop 10d earlier Confided infidelity in last month Exam: ill-toxic, 40ºC P88 BP100/70 RR18 O2 97% RA Lungs: clear Neck: supple Cor: no murmurs Skin: no rashes LP: pending Labs: WBC 5200, 26% B Sputum: Some PMNs, Gram-negative organisms Which antibiotic will lead to the most rapid improvement? A. Ceftriaxone B. Gentamicin C. Doxycycline D. Trimethoprim/sulfamethoxazole

Answer C - pet shop - headaches - temperature / pulse dissociation Chlamydia psittaci • AKA parrot fever, psittacosis, ornithosis Most "atypical pneumonia" • Risks: exposure to birds May be healthy or ill Pets, poultry, pigeons Native birds o Lawn mowing Range of illness: Mild, bronchitic to severe/ARDS Clue: temperature/pulse dissociation • Diagnosis: o Molecular, sputum (best) o Acute/convalescent serology (microimmunofluorescence, MIF) • Treatment: Preferred: doxycycline Alternatives: o Macrolides o Fluoroquinolones

19M from Wisconsin complained of feverishness, myalgia and headache x 5d. He works as a camp counselor. Several of his campers have had boils. Cephalexin has been given by the camp doctor without effect. He's afebrile & not acutely ill; exam is normal except for rash on leg Which would be the best next step: A. Order B. burgdorferi serology B. Administer ceftriaxone pending lumbar puncture C. Order trimethoprim-sulfamethoxazole x 7d D. Order doxycycline x 10-14d E. Obtain a skin biopsy & culture

Answer D

40 year old M day 60 after allogeneic BMT from unrelated donor, with bloody urine for 6 days. Has skin GVHD, receiving a prednisone taper (1 mg/kg/day). Exam, faint diffuse erythematous rash. Cr 1. LFTs normal. CMV pcr negative. The most likely etiology is: A. Cyclophosphamide B. CMV C. EBV D. BK E. JC virus

Answer D

A 6 yo boy from Indiana who has a pet dog and likes to play in a sandbox presents with fever, hepatosplenomegaly, wheezing, and eosinophilia. He has never travelled outside the continental U.S. The most likely causative agent acquired in the sandbox is: A. Anisakis simplex B. Onchocerca volvulus C. Enterobius vermicularis D. Toxocara canis E. Anyclostoma braziliens

Answer D

A 17‐year‐old man from Arizona presents with leg pain. He was in his usual state of good health until 8 months ago when he developed localized pain in his left leg just below the knee. He denied any antecedent trauma. He also denied any skin lesions, erythema, fevers, chills, sweats, weight loss, or fatigue. He is a competitive swimmer but he gave up the sport about four months earlier as a result of his leg pain. He denies tobacco, alcohol, or illicit drug use. He is sexually active with a single female partner. On examination, his vital signs are normal. The left leg appears normal on visual inspection. Deep palpation below the left knee over his tibia elicits mild discomfort. The knee joint is normal. Muscle strength and sensation are normal. Which of the following is most likely to be isolated from a biopsy of this lesion? A. Coccidoides immitis B. Mycobacterium marinum C. Pseudomonas aeruginosa D. Staphylococcus aureus E. Streptococcus pyogenes

Answer D Brodie's abscess is a subacute or chronic form of osteomyelitis caused by the hematogenous spread (or occasionally direct traumatic inoculation) of an organism to the bony metaphysis of long bones (most often involving the tibia). under the age of 25 only localized pain and no systemic symptoms Diagnosis radiographic penumbra sign is a thin layer of granulation tissue lining a bone abscess cavity which produces a higher signal on T1‐weighted magnetic resonance images; in one study, it had a sensitivity and specificity of 73 and 99 percent, respectively for the diagnosis of Brodie's abscess. While Streptococcus and Pseudomonas may be isolated from cultures in persons with Brodie's abscesses, S. aureus is the most commonly identified organism. Up to 50% of cultures from bone biopsies will not yield any pathogen. M. marinum has not been associated with this clinical presentation. Disseminated coccidioidomycosis can present in exactly this way, though an enhancing rim around the lytic lesion is unusual in coccidioidomycosis and often the chest xray shows a chronic infiltrate

A 6 yo boy from Indiana who has a pet dog and likes to play in a sandbox presents with fever, hepatosplenomegaly, wheezing, and eosinophilia. He has never travelled outside the continental U.S. The most likely causative agent acquired in the sandbox is: A. Anisakis simplex B. Onchocerca volvulus C. Enterobius vermicularis D. Toxocara canis E. Anyclostoma braziliense

Answer D visceral larva migrans

A 32 year old nurse is 34 weeks pregnant during influenza season. She develops influenza symptoms and is seen at an instacare where a rapid test is positive and she is given augmentin. • 72 hours after the onset she presents to the ED with fever, tachypnea, hypoxemia and decreased urine output. • CXR shows bilateral hazy infiltrates. She is hospitalized. Which of the following is correct? A. She should get supportive care only since she has had symptoms for >48 hours B. Oseltamivir is relatively contraindicated in pregnancy C. Zanamivir is clearly preferred because of low systemic absorption D. Oseltamivir should be started as soon as possible

Answer D • Antiviral treatment is recommended as soon as possible for: ▫ Patients with confirmed or suspected influenza who are hospitalized, or have severe, complicated or progressive illness regardless of duration of symptoms ▫ Outpatients with confirmed or suspected influenza who are at higher risk for influenza complications based on their age and/or medical conditions • Recommended medications: oseltamivir and zanamivir • Oseltamivir should be used, when indicated to provide treatment or chemoprophylaxis for infants younger than one year old • Empiric antiviral therapy should be offered to pregnant women and women up to 2 weeks postpartum • Pregnancy should not be considered a contraindication to oral oseltamivir or zanamivir use. • Treatment duration should be 5 days • Initiating treatment within 2 days of symptoms results in improved outcomes Chemoprophylaxis should not replace vaccination • 70-90% effective in trials • Prophylaxis may increase selection of resistant viruses • PEP is recommended to control influenza outbreaks in nursing homes • PEP can be considered for high risk persons with unprotected close contact with patient with flu • Post exposure prophylaxis should not be given after 48 hours from exposure • Post exposure prophylaxis for otherwise healthy persons is generally discouraged; prompt empiric therapy is preferable • Antivirals not effective after 48 hours in outpatients with uncomplicated flu but are effective later in hospitalized patients • Influenza B viruses are intrinsically resistant to adamantanes (rimantidine and amantidine) • Double dose oseltamivir not more effective • Resistance to oseltamivir occurs most often through a specific point mutation H275Y in H1N1 viruses (functionally same as H274Y in N2). This confers partial resistance ~40-fold to peramivir. Other mutations important in H3N2, H7N9 • Zanamivir remains active against H275Y mutant influenza and most oseltamivir resistant viruses • Peramivir licensed (600 mg IV x 1) but only for acute uncomplicated • Inhaled zanamivir can exacerbate asthma, not approved under 5 years • Using commercial powder of zanamivir in ventilator circuit has caused catastrophic ventilator failure

35 year old woman with AML (acute myloctic leukemia) day 15 after induction therapy. Fever, chills, diffuse erythematous rash. Blood culture + GPC in chains Exam - 100/62, HR 120, grade 2 oral mucositis, and a diffuse, blanching, erythematous rash. CXR - bilateral diffuse infiltrates. She is receiving levofloxacin and acyclovir This is most consistent with infection with which of the following organisms? A.Streptococcus pneumonia B. Coagulase-negative Staphylococcus C. Enterococcus faecalis D.Streptococcus mitis E.Stomatococcus mucilaginosus

Answer D - specific strep viridan seen in neutropenic hosts Key points: neutropenia, mucositis, high‐dose cytosine arabinoside, fluoroquinolone • Can present with fever, flushing, chills, stomatitis, pharyngitis • VGS shock syndrome: - After 24‐48 hours, hypotension in 1/3 of cases - Rash, shock, ARDS in 1/4 of cases (similar to toxic shock) • Endocarditis unusual (<10%) • S. mitis, S. oralis • Vancomycin • Mortality high (15‐20%)

A 45-year-old international agricultural researcher presents in June in the US with fever, diarrhea, myalgia, sore throat, and dyspnea. He is hypotensive and hypoxemic. • CBC shows mild leukopenia, chemistry panel and LFT's are normal. • Three days prior to the onset of his illness he was inspecting poultry operations Jiangsu Province, China Assuming the he acquired his severe respiratory illness from the poultry he was inspecting, the most likely influenza diagnosis would be: A. H1N1 B. H3N2 C. H5N1 D. H7N9 E. Influenza B

Answer D - US x2, canada, hong kong H5N1 - egypt, vietnam, indonesia • H7N9*: Caused >130 cases of severe disease 2013; >200 in second wave; ongoing • > 1500 cases in 5 years • 22% case fatality • Avian to human transmission • Few family clusters with human to human documented • Some intrinsic and some emergent oseltamivir resistance • Exported cases ▫ US x 2, Canada, Hong Kong, Taipei • H7N3: Isolated cases in farm workers • H7N7: Human cases associated with outbreak in Netherlands. H7 viruses associated with conjunctivitis

54 year old man with 4 weeks of cough, low grade fevers, & left-sided chest pain. Received a liver transplant 11 months ago, complicated by rejection, requiring high dose steroids 4 months ago. He receives TMP/SMX three times a week. On exam, he is stable, chronically-ill appearing, febrile (101.1oF), has clear lungs and benign abdomen. Labs reveal a normal white blood cell count, slight anemia, & normal creatinine. Chest radiograph reveals hazy opacity in left lower lung zone. Chest CT reveals nodular air-space consolidation in the left lower lobe with central cavitation (image). Gram strain of bronchoalveolar lavage fluid reveals beaded gram positive filamentous organisms (image). What is the most likely cause of this patient's pneumonia? A. Cryptococcus neoformans B. Histoplasma capsulatum C. Actinomyces israellii D. Nocardia farcinica E. Aspergillus fumigatus What are the most appropriate next steps in this patient's care? A. Initiate therapy with intravenous TMP/SMX B. Obtain a needle biopsy of the lung nodule to confirm the diagnosis C. Obtain a brain MRI & start amikacin, imipenem, & TMP/SMX D. Defer therapy until antimicrobial susceptibilities return

Answer D and C Nocardia Infections Beaded & branching gram-positive rods Partially acid-fast Aerobic (unlike anaerobic Actinomyces) . Immunocompromised 90%: slowly progressive pneumonia with cough, dyspnea, & fevers Can disseminate to any organ (brain in particular: get MRI) 10%: Skin infections from direct inoculation: Immunocompetent host in tropical region (N. brasiliensis) Immunocompromised patient who gardens or walks barefoot Sporotrichoid lesions Mycetomas: chronic, progres Diagnosis: Chest imaging: nodules, cavities, infiltrates with consolidation, effusions, ground-glass opacities MRI brain: single or multiple abscesses Blood culture, BAL, biopsy Gram stain, modified acid-fast stain, culture Species identification with nucleic acid amplification or MALDI: predictive of drug susceptibility

35 yr old F, 80 days after allogeneic BMT with 65 days of anorexia, nausea, epigastric pain, and diarrhea. CMV D‐/R+, HSV+, VZV+. Exam: faint maculopapular rash on upper body. Afebrile. Meds: acyclovir, TMP‐SMX and fluconazole. ANC 1000, ALC 250. LFTs normal. What is the most appropriate initial work‐up and management? A. Perform serum VZV PCR B. Empiric corticosteroid treatment C. Send C. diff toxin and start oral vancomycin D. CMV PCR, stool C. diff, bacterial culture E. #D and upper, lower endoscopy

Answer E 65 days post transplant low ANC either GVHD, or CMV of GI tract Acute (early after HSCT) - Fever - Rash - GI: hepatic, colon • Chronic (later after HSCT) - Skin changes (lichen planus, sceroderma) - Hepatic (cholestatic) - Ocular (keratoconjunctivitis) - GI (oral, dysphagia) - Pulmonary syndromes

• A 35 year old male with long standing HIV, untreated, is brought to the ER for a seizure. His CD4 has been < 20 cells. • The patient admits that he has had a slowly progressive left lower extremity weakness, and his performance at his accounting firm has deteriorated in the past few months. • MRI findings of a right parietal white matter lesion with no atrophy or ventricular dilation. • CSF shows wbc 20 (100% lymphs), protein 60 Which of the following CSF PCR tests would be the most useful? A. Jacob Creutzfeldt virus B. HIV C. EBV D. BK virus E. JC virus

Answer E Progressive Multifocal Leukoencephalopathy (PML) (JC Virus Encephalitis) • Polyomavirus (JC) — Transmission by respiratory route — >80% adults infection • serum antibody test has no clinical role • Predisposition: Immunosuppression — Highest risk • HIV, transplants, natalizumab, efalizumab, rituximab

42M just returned from a hiking trip Colorado, removing a tick on his arm 2d earlier. He is now heading out of town again on a beach vacation. There is some intense itching and redness at the site he thinks may be larger (~1cm) than yesterday. He is otherwise well. The best course of action would be: A. Doxycycline 200mg x single dose B. Doxycycline x 14d C. Doxycycline x 30d D. Cefuroxime x 14d E. Observation

Answer E colorado is not an endemic area small site and itching, likely hypersensitivity reaction I. scapularis tick bite prophylaxis B. burgdorferi transmittal • Tick attachment time < 24 h: 0/58 (0%) < 48 h: 4/50 (8%) < 72 h: 36/52 (69%) 200 mg given within 72h of tick bite Lyme disease: some pearls • No need for serology if diagnosing erythema migrans • B. burgdorferi IgM immunoblot most common cause of misdiagnosis • Late Lyme arthritis: always seropositive No evidence that seronegative Lyme exists in patients with long-term symptoms Lab evidence of LD essential unless hx of EM exists • Prolonged antibiotic treatment doesn't improve resolution of subjective symptoms

A 35 yo man is admitted to the ICU in July with fever, respiratory failure, hypotension. • 5 days PTA he complained of having the "flu;" fever, malaise, myalgia, mild abd pain. • History: Recently camped in cabins at Yosemite National Park which has had rodent infestations issues. • Has parakeet, dogs, cat had kittens recently, owns a hot tub. 2 kids in daycare have URI. • Labs: Hct 52; WBC 6.0 (20% bands, 45% polys, 2+ atypical lymphs), platelets 90K, • AST 105, PT 18, PTT 25 • CXR: Rapidly progressing bilateral infiltrates leading to white out Which of the following is the most likely cause of his illness? A. Adenovirus B. Influenza C. Anthrax D. Coxiella burnetii E. Hantavirus Pulmonary Syndrome

Answer E - thrombocytopenia - hemoconcentration - atypical lymphs, with left shift - elevated liver enzymes, abnormal LFTs • Recent outbreak in Yosemite. Endemic cases of HPS in much of US, Chile, Argentina Stages of Hantavirus Pulmonary Syndrome (HPS) • Incubation (4-30 days) • Febrile phase ▫ Fever, myalgia, malaise occasionally N, V, abd pain • Cardiopulmonary phase • Diuretic phase • Convalescent phase • Acute onset of cough an dyspnea • Presentation and rapid progression of shock and pulmonary edema (4-24h non-productive cough and tachypnea (shortness of breath) • Hypovolemia due to progressive leakage of high protein fluid from blood to lung interstitium and alveoli, decreased cardiac function

• 49F complains of four years of fatigue, headache, poor sleep and joint aches since trip to London UK PMH: TAH/BSO Medications: hormone replacement SH: Married, accountant. Lives in central Pennsylvania. Two dogs, often sleep in bed. PE: normal Labs: normal CBC, ESR, TSH o B. burgdorferi serology: EIA (not done), IgM WB 3/3 bands, IgG 1/10 • What is the best recommendation at this time? A. Doxycycline 100 mg x 14 days B. Doxycycline 100 mg x 28 days C. Repeat Lyme serology (two tier: EIA w/ reflex WB) D. Lyme C6 antibody assay E. Neither additional Lyme disease testing or treatment

Answer E -subjective c/o fatigue, HA, insomnia - we never know serology can be positive for while, even if amoxicillin - additional abx can't help -IgM+ very high false positive ; if more than 4 weeks of symptoms, don't pay attention to serology Two tier serology: not needed for EM First: total Ab screen - ELISA or EIA If positive, second tier reflexes to immunoblots (IB) o IgM: ≥ 2/3 bands, use only if < 4 wks of symptoms High rates false (+) o IgG: ≥ 5/10 bands, more reliable Alternative criteria (different bands): less specific Often negative in early infection (first 2-3 weeks) May need acute/convalescent for confusing rashes or neuroborreliosis Serology: may remain (+) for decades including IgM

31F fever, cough, myalgia, headache, dyspnea over 1 week ago • No help w/ azithromycin x 3d • 18 mos daughter, recent bronchitis PMH: not significant SH: ½ ppd smoker PE: ill T38.3, RR 35, BP 125/70, P 128 Coarse breath sounds, rales bilateral and decreased L base Data: WBC: 11, 300 38%P, 48%B RA ABG: 7.37/35/58 Sputum Gram stain: > 25 WBC/hpf Some Gram (+) cocci Sputum Cx: pending Respiratory Film Array: Influenza (+) RSV (+) Pt placed on oseltamivir, ceftriaxone and azithromycin. Which of the below should be recommended by the ID consultant? A. Disregard RSV as likely false positive B. Institute ribavirin PO for RSV C. Continue ceftriaxone, but replace azithromycin with moxifloxacin D. Change from oseltamivir to peramivir injection E. Attempt aspiration of left pleural fluid, start linezolid

Answer E post viral pneumonia Not much benefit swap azithro -> moxi same coverage )

59 year old woman with AML with neutropenia for 25 days. She has been febrile for 6 days, and is receiving meropenem, vancomycin, and acyclovir. New skin lesions that are small, papular, and tender, with no central ulceration. A. Rhizopus spp. B. Varicella zoster virus C. Cryptococcus neoformans D. Vancomycin resistant Enterococci E. Candida tropicalis

Answer E vre no lesions crypto - not neutrop vzv - wrong host Candida infections - Frequent in patients not receiving prophylactic antifungals • C. albicans, C. tropicalis • Mucositis, colonization, neutropenia • Acquired through GI tract or catheter • Organisms in patients receiving azole prophylaxis - C. glabrata, C. krusei - C. parapsilosis » catheter / intravenous infusates • Mold infections - Aspergillus fumigatus most common - Risk increases with duration of neutropenia or prior neutropenic episodes

44 year old farmer from Turkey is visiting family in the United States. He reports a several month history of fever and night sweats. One week ago he developed right low back and hip pain, worse with sitting. CT of the pelvis shows enhancement of the right sacroiliac joint capsule anteriorly Blood cultures held 5 days remain negative and IGRA is negative. What is the best next step? A. Initiate empiric treatment for tuberculosis B. Empiric treatment with vancomycin and ceftriaxone C. Start anti-inflammatories for spondyloarthropathy D. Open arthrotomy of SI joint for cultures and debridement E. Percutaneous sampling of SI joint

Answer E - sacroilitis / think brucella, aside from s. aureus - don't ititiate empiric abx prior to diagnosis

A 25 yo F reports passing thin, white, flat tissue fragments in her stool several times over the past few weeks. She is healthy and has been in Madagascar for 3 years as a Peace Corps volunteer. The microbiology lab confirms the tissue fragments are parts of a helminth. A long-term complication that can occur as a result of infection with certain species of this type of helminth is: A. HTLV-1 infection B. bladder cancer C. appendicitis D. liver abscess E. seizures

Answer E, seizures -taenia solium, ingestion of egg can lead to neurocysticercosis INTESTINAL TAPEWORMS Taenia solium tapeworm is acquired by eating larvae in pork adult tapeworm causes few symptoms Taenia saginatum acquired by eating larvae in undercooked beef causes few symptoms can grow to 10 m Diphyllobothrium latum (can grow > 10 m) acquired by ingesting fish with larvae *B12 deficiency in up to 40% of patients HTLV-1 - association with strongyloides sterocolis bladder cancer - s. hematobium appendicitis- pinworm, enerobius vermiculos echinococcus granulosis - liver cysts

A 59 y.o. white male from Missouri presents with fever (390), headache, myalgia, anorexia, nausea, one week after removing an engorged tick from his groin. No travel. Exam: unremarkable except ill appearing, no rash. Lab: wbc 2300 plt 42,000 ALT 111 Suspect ehrlichiosis (but no morulae on blood smear) After sending appropriate diagnostic tests the patient has not improved after three days of doxycycline. Which of the following is the most likely etiologic agent? A. R. rickettsii B. B. burgdorferi C. R. parkeri D. Heartland virus E. Severe fever with thrombocytopenia syndrome virus

Answer heatland virus - lone star tick - looks like HME

Which of the following protozoa can be treated successfully with TMP-SMX? A. Cyclospora B. Cryptosporidia C. Enterocytozoa D. Encephalitozoa E. Naegleria

Answer is A What Are the Most Common Causes of Diarrhea in Patients with HIV Infection in US? • Clostridium difficile 54% — Health care facilities — Antibiotics • Campylobacter 14% • Shigella 14% — Flexneri > Sonnei • Salmonella 7% • Staph aureus 4% • Mycobacteria 4% • Others 4%

Five days ago (January), a healthy 25 year old woman developed fever, myalgia, sore throat and malaise which was diagnosed as influenza. She was slowly improving. • Sixteen hours ago, she became hypotensive and hypoxemic, complained of diarrhea, abdominal pain, had a diffuse erythematous rash On exam she was slow to respond and had diffuse rales and mild abdominal tenderness that was non focal. • Chest xray shows diffuse infiltrates • WBC =5500/mm3 (60% polys, 30% bands) • Platelets = 40,000/mm3 with PTT 2 x normal • Creatinine 1.9 • ALT and AST 2 x normal with normal serum ammonia level What is the most likely cause of this influenza complication?: A. Reye's syndrome B. Staph aureus pneumonia with Toxic shock syndrome C. Gram negative sepsis with ARDS D. Pneumococcal meningitis E. Viral encephalitis

Answer is B Complication Exacerbation of underlying illness COPD, asthma, CHF Ischemic heart disease Viral pneumonia May be mild or severe hemorrhagic pneumonitis/ARDS Secondary bacterial infection Strep pneumoniae, GAS, S. aureus. Classically marked worsening after initial improvement. Account for large proportion of pandemic deaths Toxic Shock Syndrome Staphylococcal TSS most commonly described but GAS also reported

• 56 year old man presents to ED with 1-2 week history of atraumatic right knee pain and swelling and low grade fevers; weight bearing is now very uncomfortable. • He has poorly controlled diabetes. One month ago he travelled to the Dominican Republic where he swam in the ocean, recalls receiving several insect bites. He owns a pet kitten, last saw a dentist three months ago, and denies injection drug use. • On exam, he has pain with passive range of motion and a moderate effusion. • ESR 68 CRP 17 mg/dL Synovial fluid: 45,000 WBCs (82% neutrophils) Negative gram stain Culture growth at 3 days incubation What is the most likely organism? A. Pseudomonas aeruginosa B. Salmonella heidelberg C. Staphylococcus aureus D. Kingella kingae E. Pasteurella canis

Answer is B -poorly controlled diabetes - travel Pasteurella canis can cause septic arthritis - bites, inoculates - hematogoneous can occur very unlikely Kingella kingae - common in pediatrics < 4 y/o (most common cause) - rare in adults - not readily grow in cultures Pseudomonas - IVDU, abx exposure, indwwelling catheter

WhichoneofthefollowingisTRUEregardingthediplomaticcouplefrom Algeria hospitalizedintheUSwithpneumonicplague? A. Nasal swabbing for Y. pestis exposure for remaining staff in Algiers should be urgently initiated B. Since plague is endemic in Algeria this is unlikely to represent a bioterrorism incident C. Staff that initially cared for these patients do NOT need to be given prophylaxis with a quinolone or doxycycline D. A logical antibiotic regimen would be ciprofloxacin and gentamicin E. It is likely that the patients will develop bubo(es) during the course of their illness

Answer is D - endemic country, think of bioterrorism - should prophyx anyone exposed within 48hrs

For a patient with B‐cell chronic lymphocytic leukemia (B‐CLL ) who will be treated with alemtuzumab (Campath, anti CD52), what anti‐ infectious prophylaxis would be appropriate - A) None specifically due to the use of this drug - B) None unless the CD4 count fell below 100 cells/uL - C) TMP‐SMX only - D) TMP‐SMX and Valciclovir - E) INH and Voriconazole

Answer is D high risk at pneumocystis and viral reactivation (especially CMV)

• For a patient with rheumatoid arthritis who will be treated with rituximab, which of the following is the most likely to cause reactivation - A) Tuberculosis - B) Aspergillus - C) VZV - D) HBV - E) HCV

Answer is D - ritxucimab, more of a B cell problem Hepatitis B most worried about B-cell little contribution to TB, it's a T-cell problems

Question 4: A 41 yo woman presented to a local emergency department with a one day history of fever associated with swelling and redness in her groin four days after returning from safari in Tanzania. Peripheral blood smear is obtained. - shows flagellated organism What is the most likely diagnosis? A. Leishmania donovani B. Plasmodium vivax C. Trypanosoma brucei D. Wuchereria bancrofti E. Leptospira interrogans

Answer trypanosomiasis African Trypanosomiasis (sleeping sickness) Vector = tse tse fly (Glossina sp), very painful Trypanosoma brucei gambiense (W. Africa) • humans as reservoirs • progression over many months Trypanosoma brucei rhodesiense (E. Africa) • cattle and game park animals as reservoirs • progression over weeks DISEASE within 5 days: chancre at Tse Tse fly bite regional lymphadenopathy for weeks: fever, hepatosplenomegaly, lymphadenopathy, faint rash, headache Trypanosoma cruzi - reduviid bugs - rubbing fecal feces into the wound - perioribitla swelling - megaesophagus, megacolon and mega heart

A 42-year-old man presented to the emergency department with an oral temperature of 102°F and passage of 12 small grossly bloody stools over the last 24 hours. He has otherwise been well. Stool culture is sent. What empirical therapy is appropriate? Ceftriaxone Ciprofloxacin Metronidazole Rifaximin Azithromycin

Azithromycin is the best choice for empiric therapy. In the absence of an outbreak, an adult with dysentery and fever is more likely to have Campylobacter jejuni, Salmonella or Shigella than an STEC like E coli O157:H7. Because of widespread resistance of Campylobacter jejuni to fluoroquinolones this class of drugs would be inappropriate treatment. Resistance to ceftriaxone is also seen in C. jejuni and parenteral therapy such as ceftriaxone does not appeared to be required in the absence of comorbidities. Rifaximin is not recommended for diarrhea in the presence of fever or blood in the stool. Metronidazole is not useful in invasive bacterial gastroenteritis.

23 year old HIV-negative man with an HIV+ partner on ART with HIV RNA suppressed below detection asks about starting pre-exposure prophylaxis (PrEP). In addition to safer sex counseling, which of these do you recommend? A. Nothing - PrEP is not indicated. B. PrEP with tenofovir (TDF)/emtricitabine daily. C. PrEP with tenofovir (TDF)/emtricitabine "on demand". D. PrEP with tenofovir (TAF)/emtricitabine daily.

B - PrEP with TDF / emtricitabine daily • Before starting: • document HIV Ab negative and r/o acute infection within a week (assess symptoms) of starting • document CrCl >60, screen for STIs and HBV infection • Prescribe tenofovir (TDF)/emtricitabine 1 po daily X 90 days • provide risk reduction, adherence counseling, condoms • On treatment: • HIV testing every 3 months • check CrCl every 6 months • risk reduction, condoms, STI assessments/rx • evaluate the need to continue PrEP On demand regimen (french study): x2 pill,l 2-24 hr before sex, 1 within 24 hrs, 4th within 48 hrs after sex

A 23 year old man presents with a history of unprotected receptive anal sex with known HIV‐infected man, and one week of fever, adenopathy. HIV‐1/2 ELISA is reactive, viral RNA level 500,000 c/ml. He is started immediately on antiretrovirals. His confirmatory assay is negative, and repeat assays sent 3 weeks, 3 months, and one year after starting antiretrovirals are also negative. ELISA remains reactive. HIV‐2 assay is negative. Viral RNA on therapy is <40 c/ml. Which of the following is correct: A. The patient was infected with a strain of HIV‐1 that was not detected by the confirmatory assay B. The patient is HIV‐infected but did not develop a positive confirmatory assay because of the early antiretroviral therapy intervention C. The patient never had HIV infection. D. The patient had HIV but is now cured of HIV and antiretrovirals can safely be stopped

B is correct • Prompt reduction in HIV‐1 RNA • Potential blunting of humoral immune response -being driven by viremia • Confirmatory assay may become unreliable • HIV‐1 DNA PCR has been useful in documenting infection

A 30 year old man presents with dementia progressing over the last year. He was born in rural Indonesia, lived in London from 1990 - 2010, then moved to Philadelphia. Which of the following diseases is most likely the cause of his symptoms: A. Kuru B. variant Creutzfeldt-Jacob Disease C. Familial Creutzfeldt-Jacob Disease D. Rabies

B- epidemeology many relatively young patients pregoression is longer no typical EEG progress slower than sCJD predominantly in countries UK beef A- none in indonesia C- not as common in younger

In the United States, what is the prevalence of any baseline (pre-Rx) resistance mutations? A. < 1 % B. 5 -15 % C. 20-30% D. 30 -50% E. > 50%

B. 5-15% Testing point: Every ID physician should know that transmission of resistant virus occurs and should be checked for prior to initiation of ARV therapy

A 41 year old car salesman from Baltimore was admitted for a febile illness & found to have Brucella melitensis in his blood culture. He had attended a dinner a month prior where some family members from Greece had brought food from home. About two weeks prior to onset of fever, he had bought some lamb & beef at a farmer's market outside Baltimore. The most likely source of his brucellosis was which of the following: A. Home made sausage from Greece B. Home made goat cheese from Greece C. Cole slaw from a Baltimore delicatessen D. Beef tartar, meat from the farmer's market E. Lamb kabobs, meat from the farmer's market

BRUCELLOSIS ► Exposure to non-USA dairy or meat, unpasteurized cheese, uncooked meat ► Slaughterhouse worker, meat packer, veterinarian ► Acute or indolent onset fever, aches ► Nodes, liver, spleen may be enlarged Later onset lesions in bone (sacroilitis) and epidydomiditis

29 y/o M +Fever, HA x2 days 3 weeks ago non-pruritic, erythmatous, 8 cm circular skin lesions o nback of his left thigh, 10 days after returning from vacationing in nantucket islan, massachusetts x21 days of doxycycline, rash resolved, felt well until 2 days ago with fevers past 48 hrs, +chills, fever, fatigue, loss of appetite, mild abdominal tenderness, and hepatomegaly WBC 6100, Hgb 11.8, Plt 102, Billi 1.6, AST 106, LDH 368 ? dx? Borrelia hermsii Borrelia burgdorferi Babesia microti Ehrlichia chaffeensis Anaplasma phagocytophilum

Babesia microti Ixodes ticks,Northeast,carry three infections: Borrelia burgorferi, Babesia microti, and Anaplasma phagocytophila. A bite can transmit one, two, or three of these infections. Doxcycline treat lyme and Anaplasma and Ehrlichia, but not Babesia. patient's symptoms, co-infection Lyme disease Babesia microti, s acquired deer tick bite, initial symptoms erythema migrans lesion due to B. burgdorferi. While relapsing fever due to B. hermsii could account for his symptoms, this organism, and its vector (Amblyomma) are not endemic in the northeast. Re- infection with B. burgdorferi has been documented but the patient's current symptoms, physical and laboratory findings are all atypical for either acute or subacute Lyme disease.

40 yo M with diabetes was well until 6 weeks before admission when he developed fever and malaise. CXR showed RLL infiltrate. Treated with 2 wks of azithromycin. Initially felt better, but after stopping the antibiotic, he again felt poorly. On day of admission, he found vision in left eye was almost gone. Also complained of mild pain in left eye. Ill-appearing. T: 38.3°C. Left eye: conjunctival injection, reduced vision (hand motion). Decreased breath sounds on right. Tachycardic, S1/S2 +S3, soft diastolic murmur at RUSB. CXR: RLL infiltrate, right pleural effusion A:Staphylococcal keratitis B. Herpes simplex keratitis C. Bacterial endogenous endophthalmitis D. Pulmonary tuberculosis with chorioretinitis E. Cytomegalovirus retinitis

Bacterial endogenous endophthalmitis (in the setting of endocarditis) BCx: S. pneumoniae. TEE: aortic root abscess. Endogenous endophthalmitis results from hematogenous spread of bacterial or fungal infection to eye Commonly due to gram-positives (S. aureus, Strep); gram-negative infection may occur Acute onset of: Systemic illness Decreased visual acuity Eye pain Hypopyon (layer of pus in inferior portion of anterior chamber of eye) - seen in image Monitor pts with endocarditis for visual symptoms Staphylococcal and HSV keratitis: Patient did not have corneal symptoms. These diagnoses would not explain the signs/symptoms of systemic infection. Pulmonary TB with chorioretinitis: Would not explain initial improvement with azithromycin, diastolic heart murmur CMV retinitis: Would not explain initial improvement with azithromycin or diastolic heart murmur.

HIV + Cd4 10, VL 2 mil developed lesions on trunk, face and extremities in 8 months HHV-8 Ab negative low grade fever for months Picture of kaposi's sarcoma type rash (pigmented firm lesion) what else might it be?

Bartonella (jane's kaylor is expert) Not distinquishable between kaposi and bartonella rash on physical exam

25 yr WM presented in July with painful right inguinal mass of one week's duration. He is otherwise well. Married. Monogamous. No hx penile or skin lesion. Fishing last week in Northern Virginia creek, hiked through wooded area. Picked ticks off legs and neck. Has kitten and dog. Exam: T37C, 5 cm tender red mass in right midinguinal area, fixed to skin. Genitalia normal. Aspiration of soft center: 5 cc yellow pus. Gm stain neg. Keflex 250 mg qid. One week later: mass unchanged. Culture neg. VDRL and HIV neg. Most likely dx: A.Bartonella henselae B.Treponema pallidum C.Haemophilus ducreyi D.Francisella tularensis E. Klebsiella (Calymmatobacterium) granulomatis

Bartonella henselae- young cats Stellate abscess on bx. Warthin Starry stain - slow growing weakly gram (-) rod - in children - acute supprative lymphadenitis - proximal to bite, scratch, lick of young cat - 2014 guidelines, use azithromycin - B Baciliformis - sand fly,, oroya fever - B quintana, human to human by body louse pediculus humanus), trench fever, endocarditis, lysis centrifucation, held for 2-4 weeks - in HIV CD< 100, can cause bacillary angiomatosis, lesions bleed easily, ddx Kaposi Sarcoma ► Tick borne tularemia ("glandular") Systemically ill, more acute. Gram stain, routine culture negative Exposure to wild animals or their ticks ► Chancroid : painful genital ulcer. No suppurative lymph nodes in syphilis or granuloma inguinale (Klebsiella granulomatis) Staphylococcus aureus. Distal lesion often seen. Gram stain of pus & culture positive ► Lymphogranuloma venereum- Chlamydia trachomatis L1-L3: genital lesion usually inapparent "Stellate abscesses" on bx (+) Nucleic acid amplification test on urine or wound

39 y/o M in california eating food from dumpster (nearby stray cats dogs, racoons and rats) now to ED with HA, mental confusion, +Feer, mildely rigid neck WBC 10 (78% PMN, Hgb 12.6, Plt 210 Cr 1.2, LFT- wnl CSF OP 33, WBC 680 (42% eos) glucose 56, Protein 316, GS and AFB negative Etiology? Leptospira interrogans Angiostrongylus cantonensis Toxocara canis Gnathostoma Baylisascaris procyonis

Baylisascaris procyoni Baylisascariasis, gnathostoma and Angiostrongylus are causes of eosinophilic meningitis. Leptospirosis, rat urine,meningitis, not eosinophilia. ingestion of dog feces, toxocariasis, with fever and eosinophilia but not meningitis Baylisascariasis, intestinal helminth of raccoons, in children who ingest the eggs, raccoon infected stool. Angiostrongylus, Far East,Hawaii, the Caribbean but not in the continental USA. Gnathostomiasis, mostly in Asia, also Central and South America, but not North America]), ingesting infected molluscs or vegetables, eosinophilia and migratory localized erythematous skin lesions and nodules or with eosinophilic meningitis. The geography is not appropriate. Bartonella and rabies do not cause eosinophilic meningitis.

24 y/o from japan to ER with ulcers of mouth and peis hx of intemrittent painful oral ulcer for 3-4 years denies sexual contact for the past 5 year afebrile left eye is inflammed with hypopyon numerous ulcers on the oral mucosa no adenopathy, 0.5 cm ulcer on the penis 6mm papulo-pustular lesion on the rigtht antecubital fossa where they drew blood labs normal HIV negative

Behcet's disesae vasculitis clinical diagnosis: -pathergy (ulcer/lesions at the site of minimal trauma) -asian or mediterranean ancestory -recurrent oral genital ulcers (>3 per year) -uveitis

A 25 year old pregnant woman immigrant from southern Japan was referred to you for evaluation of a positive HTLV‐I western blot. Which of the following statements is true: A. The risk of HTLV‐I transmission can be entirely eliminated by caesarean section. B. The risk of HTLV‐I transmission will be entirely eliminated by not breastfeeding. C. Breastfeeding will provide sufficient immunity to prevent infection with HTLV‐I. D. The risk of HTLV‐I transmission will be significantly decreased but not entirely eliminated by avoiding breastfeeding. E. There is no risk of HTLV‐I disease. In this ethnic group, the HTLV‐I test was likely a false positive.

Breastfeeding • Prolonged duration: 20‐30% seroconvert if breastfed >12 mos • High maternal HTLV proviral load in breastmilk: 28.7 infections/1000 person months with 1.5% HTLV+ lymphs • Sexual • Transfusion • Risk of seroconversion: 40‐60%

A 62 yr old sheep rancher from Northern Australia was referred to a West Coast hospital because of refractory pneumonia that had failed to respond completely to multiple, prolonged courses of antibiotics over 3 months, leaving him with continued low grade fever, productive cough & asthenia Gram negative rods noted in moderate abundance on Gram stain of sputum & in sputum culture. Identification by automated system failed & isolate sent to referral lab Which of the following would have been a likely source of this infection? A. Hospital nebulizer while hospitalized in Australia (nosocomial superinfection) B. Water or soil from his ranch C. Coughing worker on his ranch D. Sick sheep on his ranch

Burkholderia pseudomallei possible Melioidosis Take-Aways SE Asia (Thailand)/Australia Soil/water exposure (inhalation/inoculation/rainy season) Pneumonia + severe sepsis/shock or multiple abscesses Can be years after exposure (not usually) Safety pins on Gram stain; Gram negative rods Glander's disease a varient, PNA and pustules from animals exposure Hospital nebulizer: Pseudomonas should be easily identified by lab & would be more acute Coughing worker not source of Gm neg pneumonia Sick sheep: Q fever (Coxiella) not seen on Gram stain

HIV Clinical Presentation: Question #3 A 49 year old woman from Guinea‐Bissau has a reactive HIV‐1/2 ELISA and a HIV multispot positive for HIV‐2 and negative for HIV‐1. CD4 cell count is 350 cells/µl. Which of the following is correct? A. HIV‐2 is less pathogenic than HIV‐1 so she only needs therapy with one antiretroviral drug B. She should not be treated with protease inhibitors because HIV‐2 is naturally resistant to PIs. C. She should not be treated with NNRTI therapy because HIV‐2 is naturally resistant to NNRTIs. D. Use of routine HIV‐1 viral load assays is useful in patient management

C - NNRTI PI work pretty well Routine HIV-1 viral load assay does not detect HIV -2 usually

A 23 yo Eastern European woman presents with persistent epigastric discomfort diagnosed as Hp+ gastritis by endoscopy. Fecal Hp antigen is also positive. As a child, she was treated repeatedly with PCN/amoxicillin for recurrent tonsillitis. What do you recommend for therapy? A. Clarithromycin + amoxicillin + PPI B. Metronidazole + erythromycin + PPI C. Bismuth subsalicylate + T CN + metronidazole + PPI (Bismuth quad therapy) D. Metronidazole + amoxicillin + PPI E. PPI therapy alone given her age After treatment of this patient for Hp gastritis, the H. pylori stool antigen test should be repeated: A. On the final day of H. pylori therapy B. Two weeks after completion of H. pylori therapy C. Eight weeks after completion of H. pylori therapy D. T he test should not be repeated to assess cure

C. Bismuth subsalicylate + T CN + metronidazole + PPI (Bismuth quad therapy), can use bismuth, ppi, amox, clarithyro Answer is C, eight week after completion of H. pylori therapy' How to diagnose: In most: Stool Hp antigen test, Urea breath test If alarm symptoms or signs then endoscopy is mandatory How to treat: Quadruple therapy (bismuth or nonbismuth) > Triple therapy Increasing emphasis on antibiotic resistance testing (fecal or biopsy genotypic testing for clarithromycin or MIC testing for clarithromycin, nitroimidazole, FQ resistance)

35 year old man presents with complaints of increasing fatigue, headache, SOB / DOE HIV diagnosed 4 mos ago with PCP Now on TDF / FTC / EFV + PCP Prophylaxis with Dapsone Claims adherence to all meds; "Doesn't miss a dose!" Normal PE Pulse Ox 85%; CXR no abnormalities ABG: 7.40 / 38 / 94/ 96% (room air) Which of the following is the most likely underlying cause of his symptoms? A. Recurrent PCP B. IRIS Reaction C. Drug toxicity D. Pulmonary Embolus E. Patent Foramen Ovale

C. Drug toxicity: Classic presentation for dapsone toxicity. "O2 Saturation Gap" - methemoglobinuria Discontinue offending agent Methylene blue - Action: reduces methemoglobin by NADPHpathway - Indication: methemoglobin level > 30% - Dose: 1-2 mg/kg IV given over 5 minutes - Avoid: do not give to patients with G6PD deficiency (won't work) A.Recurrent PCP: Unlikely with neg CXR and normal Pa02 B. IRIS Reaction: Similarly, unlikely with neg CXR and no other symptoms D. Pulmonary Embolus: Possible, but much less likely than dapsone E. Patent Foramen Ovale: Rare. Fixed Split S2

30 yo male business man from India presented with fever and dypnea while visiting Washington, DC Found to be HIV positive, with CD4 of 50. Diffuse infiltrate on chest xray, O2 sat of 65%, given trimethoprim-sulfa and prednisone. Failed to improve and went for BAL. This organism usually resides in which of the following: A. Sandflies B. Desert dust C. Rich, moist soil D. Cat feces E. Kissing bugs

C. Rich, moist soil (histoplasmosis) - top inches of yard too small for coccidoimycosis sand flies - leishmeniasis cat feces - toxoplasmosis kissing bugs -shistomiasis

29 y/o F first episdoe of meningococcal meningitis at age 17 and survied x2 more episdoes of meningococcal meningits, complete recover No howell-jolly bodies Reason for recurrenc? C7 deficiency Occult communication between sinuses and CSF Congenital asplenia Chronic Granulomatous Disease Job's Syndrome

C7 Deficiency Patients with a deficiency in the late complement components (C-5 to C-9) are at risk for repeated infections with meningococcus and gonococcus. Often, these patients have relatively mild episodes of meningococcal or gonococcal disease. Occult Communications between the sinuses and CSF can occur after trauma or surgery. When present, the etiology of the recurrent meningitis can vary and be due to different respiratory pathogens, e.g., pneumococcus or Haemophilus species. This type of anatomic defect would not explain repeated mild episodes of meningitis caused by meningococcus. Congenital asplenia can predispose to infections with encapsulated organisms such as meningococcus but episodes are usually severe and overwhelming with very high mortality. In the absence of functioning splenic tissue, Howell-Jolly bodies (nuclear remnants in RBCs) are evident in the peripheral blood. CGD and Job's syndrome (Hyperimmunoglobulin E Syndrome) are not associated with meningococcal infections

A previously healthy 45 y.o. man riding his bicycle in the rain skids, falling into a large muddy pool of rain water that has been traversed by many joggers, dog walkers, bicyclists. His elbow is lacerated by a submerged object. He "steri-strips" the laceration. Five days later the site is increasingly tender. Purulent drainage has Gram negative rods on Gram stain.. Which is the most likely pathogen? Vibrio vulnificus Mycobacterium marinum Burkholderia pseudomallei Aeromonas hydrophila Mycobacterium haemophilum

CORRECT ANSWER: Aeromonas hydrophila Rationale: Aeromonas hydrophila is a cause of freshwater injuries. Vibrio vulnificusand Mycobacterium marinumare primarily associated with salt water injuries. Moraxella catarrhalis is typically a pathogen of the respiratory tract and are gram-negative diplococci. B.pseudomallei can cause soft tissue infections but must have exposure to waters of SE ASIA Mycobacteria are weakly Gram positive.. Aeromonas can be found in Fresh water Estuaries ie (brackish) water, recreational water such as pools, drinking water, or polluted water. The most common manifestation of aeromonas infection is diarrhea which can be acute and secretory with vomiting, dysenteric with blood, chronic or travel related. Wound infections with appropriate epidemiology are well reported due to several Aeromonas species, not just hydrophilia. There were prominent clusters of cases after Hurricane Katrina in New Orleans and after the Tsunami in South East Asia. Aeromonas often produces beta lactamases, so that therapy, based on susceptibility testing, is often with carbapenems, quinolones, or trimethorprim-sufla

Bacteria with "safety pin" appearance Yersinia pestis Vibrio parahemolyticus Burkholderia mallei & pseudomallei Haemophilus ducreyi Klebsiella granulomatis Y. pestis (granuloma inguinale)

Causes of Sporotrichoid Lesions Nodular lymphangitis Sporothrix schenckii Gardening, soil, splinters, animal bites/scratches Nocardia brasiliensis Gardening, soil, splinters Mycobacterium marinum Aquarium, fish handling, water exposure Leishmania brasiliensis Living/traveling in endemic regions

AmpC Beta-Lactamases Chromosomal and inducible in SPICE-M enterobacteriaceae In presence of a beta-lactam, chromosomal gene de-repressed with 10 to 100 fold increased production of AmpC that can lead to emergence of resistance to ESCs during therapy More recently, identified AmpC constitutively produced from plasmid genes SPICE-M bacteria: Serratia, Providencia/Pseudomonas, indolepositive Proteus, Citrobacter, Enterobacter, Morganella. Greatest risk is repressed gene in Enterobacter sp

Class Subtypes Inhibitor ?? Substrates A ESBLs; KPCs;serine carbapenemases Clavulanic; Avibactam & Vaborbactam ESCs; Carbapenems B (BAD!) Metallocarbapenemases EDTA(lab testing only) All beta-lactams except aztreonam C AmpC None Cephalosporins D Oxa-48 Some ESBLs Serine carbapenemases (e.g., KPCs) None Clavulanic Avibactam & Vaborbactam Penicillins and Carbapenems ESCs & Aztreonam ESCs and Carbapenems

A 67-year-old woman with long-standing diabetes mellitus and congestive heart failure due to ischemic cardiomyopathy received the 23-valent pneumococcal polysaccharide vaccine at the age of 45. She received the 13-valent pneumococcal conjugate vaccine at the age of 65. What is the most appropriate next step? No additional pneumococcal vaccines13-valent pneumococcal conjugate vaccine now23-valent pneumococcal polysaccharide vaccine now13-valent pneumococcal conjugate vaccine in three years23-valent pneumococcal polysaccharide vaccine in three years

Correct Answer: 23-valent pneumococcal polysaccharide vaccine now The patient has two conditions that increase her susceptibility to pneumococcal infections: diabetes and CHF. She received the 23-valent polysaccharide vaccine at the age of 45. At the age of 65, she received the 13-valent conjugate vaccine. The latter is only given once. At this time, the recommendation is to give her another (final) dose of the 23-valent polysaccharide vaccine as it has been more than one year since she received the 13-valent vaccine.

A 47-year-old male with alcoholic cirrhosis (known varices but no ascites) is admitted for a variceal bleed which is confirmed by endoscopy. He is alert and has no signs of encephalopathy. Which of the following would you recommend? Administer no antibiotics unless there is a specific indication. 7 day course of IV ceftriaxone 7 day course of IV ceftriaxone and IV Fluconazole 14 day course of vancomycin, ciprofloxacin and gentamicin, given IV 14 day course of vancomycin, ciprofloxacin, gentamicin, and caspofungin given IV

Correct Answer: 7 day course of IV ceftriaxone Patients with cirrhosis who present with upper GI bleeding (from varices or other causes) should be administered prophylactic antibiotics, preferably before endoscopy (effectiveness has also been shown when administered post endoscopy). Antibiotics are more effective in patients with advanced liver disease (Child Pugh class B and C)) but all patients with cirrhosis should be treated. The most common sites are urinary tract infections, spontaneous bacterial peritonitis, respiratory infections followed by primary bacteremia . Antibiotics reduce the rate of infectious complications for patients with bleeding varices, and may also reduce mortality and re-bleeding. A 7-day course of oral norfloxacin or IV ceftriaxone is recommended, starting before endoscopy if feasible. IV ceftriaxone should be used if patients cannot tolerate oral drugs or if there is a reason to suspect quinolone resistant pathogens. Either would be acceptable but many experts prefer ceftriaxone since the causative organisms are gram positive and negative, and are often quinolone resistant. A variety of regiemens can be used: a quinolone, a cephalosporin or a carbapenem have been used. There is no evidence that broader spectrum agents such as vancomycin or gentamicin, or such agents with renal toxicity, are useful in patients with bleeding varices.

Treatment for asymptomatic bacteriuria is most appropriate for which one of the following? A 79-year-old man about to have transurethral resection of the prostate A 66-year-old woman with insulin dependent diabetes A 35-year-old man with paraplegia due to a spinal cord injury A 29-year-old woman about to receive steroids for systemic lupus erythematosus A 91-year-old woman who resides in a nursing home and is incontinent

Correct Answer: A 79-year-old man about to have transurethral resection of the prostate Screening for and treatment of asymptomatic bacteriuria is warranted only for pregnant women and individuals who are having urologic procedures that are likely to be associated with mucosal bleeding. Diabetic women are more likely to get UTIs than non-diabetics and their infections are more likely to be severe; however, studies show that treating asymptomatic bacteriuria in diabetics does not decrease risk of symptomatic infection. Similarly, patients with spinal cord injury have a high frequency of asymptomatic bacteriuria and UTIs, but this population does not benefit from treatment of asymptomatic bacteriuria. Steroid treatment is not associated with an increase in bacterial urinary tract infections. Nursing home residents have a high rate of asymptomatic bacteriuria, but treatment does not seem to impact infection rates, even in those who are incontinent of urine. Patients who are undergoing renal transplantation or who are post renal transplant would be more controversial candidates for screening.

20 year old male with osteosarcoma receiving chemotherapy, most recently 1 week before admission, thought he had an "insect bite" on his left forearm. On day of admission, he developed a fever to 102 and he noted that the skin lesion became larger and darker in the center. He had a fever of 101.2. He had a skin lesion on his forearm (shown). T he WBC count was 0.4; platelets 16,000. His skin lesion is most likely caused by: Bite of a brown recluse spiderA tick-borne infectionA gram-negative bacillusA mycobacteriumA spirochete

Correct Answer: A gram-negative bacillus Discussion: This patient has Ecthyma gangrenosum caused by Pseudomonas aeruginosa, a gram negative bacillus. Other diagnoses listed in the differential diagnosis include loxoscelism (caused by the bite of a brown recluse spider), tularemia (which can be tick-borne), aspergillosis (a fungal infection), and Lyme disease (a spirochetal infection). Ecthym gangrenosum often progresses through the following stages: an erythematous painless macule; hemorrhagic, bluish bullae; gangrenous ulcer with grey-black eschar and an erythematous halo. The lesion usually develops over 12-24 hours. Although most commonly associated with Pseudomonas aeruginosasepsis, ecthyma gangrenosum has been seen with other bacterial and fungal infections. Brown recluse spider bites may be complicated by a necrotic ulcer as well as systemic complications such as hemolytic anemia. The patient did not have a generalized rash to suggest a tick-borne infection. A disseminated fungal infection, such as aspergillosis or fusariosis, is possible in a neutropenic host and a skin biopsy is critical to ruling this out. Skin lesions of fusarium are usually mutlple may appear as papules which may become erythematous and necrotic; blood cultures may be positive in cases of disseminated fusariosis (in contrast to disseminated aspergillus infection, in which blood cultures are almost always negative). Erythema migrans due to Lyme disease (a spirochetal infection) presents with a lesion that is often larger than the one seen here and has a different appearance (e.g. it may be target-shaped).

A 77‐year‐old woman with insulin‐dependent diabetes is seen for 4 days of severe ear pain. The pain is worsened by chewing. She has no previous history of ear problems and has not had fever. She says that the ear feels wet, and that there is a yellow stain on her pillowcase in the morning. On examination, she is afebrile. The pinnae appear normal and symmetrical, but tugging on the right external ear produces pain. The right ear canal appears moist and is partially occluded by heaped‐up granulation tissue. The part of the tympanic membrane that can be seen is normal. There is no mastoid tenderness and hearing is grossly normal. There is mild facial nerve palsy on the right side. The rest of the exam is unremarkable except for the sequelae of diabetes. Pending culture results, which one of the following antimicrobials is most appropriate for this patient? A. IV or PO Ciprofloxacin B. IV Amphotericin B C. IV or PO Clindamycin D. Gentamicin ear drops E. PO Amoxicillin‐clavulanate

Correct Answer: A. Ciprofloxacin Malignant (necrotizing) external otitis (also called malignant otitis externa) is an invasive infection of the external auditory canal and skull base, which typically occurs in elderly patients with diabetes mellitus. Pseudomonas aeruginosa is nearly always the responsible organism. Patients with malignant external otitis classically present with ear drainage and severe otalgia, which is not responsive to topical antibiotics used to treat simple external otitis. With progressive infection, osteomyelitis of the base of the skull and temporomandibular joint can develop. Progression of the osteomyelitis can be associated with cranial nerve palsies, most frequently the facial nerve. Of the antimicrobial choices, the only systemic agent with anti‐pseudomonal activity is ciprofloxacin which has become the drug of choice for this infection. Mucormycosis, even in the presence of diabetes mellitus, would be extraordinarily rare as a cause of otitis externa. Other molds would be unlikely in the absence of immunosuppression, so amphotericin B would not be indicated.

A 30‐year‐old male with HIV (CD4 =210 cells/uL, HIV VL = 200,000 copies) has been receiving ART intermittently, and comes in for care and drugs sporadically. He has been complaining of shortness of breath. He has no fever or weight loss. Physical examination reveals no fever, and clear lung, but a large pleural effusion and no infiltrate in the lung. The rest of the exam is normal. The most likely test to be useful among the following is: A. Flow cytometry of pleural fluid B. PCR of pleural fluid for TB C. PCR of pleural fluid for HHV8 D. PCR of pleural fluid for EBV E. PCR of pleural fluid for HHV6

Correct Answer: A. Flow cytometry of pleural fluid This patient has large pleural and pericardial effusions. Primary effusion cell lymphoma (PEL) presents as effusions characteristically in the absence of a mass lesion. A mass lesion would suggest another diagnosis such as a more common form of lymphoma or another tumor although a variety of variants can be seen. The key diagnostic criterion for PEL is the presence of HHV‐8 in the nuclei of the malignant cells. Flow cytometry usually shows characteristic cell markers: the cells express CD30 and 45 but lack characteristic B cell or T cell markers. They do have cytologic features suggesting lymphoma. Primary effusion cell lymphomas are associated with HHV‐8 and EBV. Some patients may have positive PCR for HHV 8 and EBV without PEL. This malignancy represents about 4% of HIV associated tumors, occurs usually in males with a median CD4 count of 200 cells (higher than other lymphomas), and is very difficult to treat. There is no standard therapy with a high degree of efficacy

A 29‐year‐old female developed her first bout of meningitis at age 24. Since then, she has had 3 other episodes. Each is associated with headache, stiff neck, photophobia, and occasional radicular pain down her right lower extremity. Each has been treated with appropriate doses of antibiotics and resolved over several days. Her last CSF results at the time of an acute episode revealed a WBC of 230/mm3 (92% lymphs with some large and atypical appearing cells), normal glucose, and protein of 110 mg/dL. All routine bacterial cultures and Gram stains have been negative. The patient has been well otherwise but has a history of both oral and genital herpes simplex virus infection, and a remote history of treated syphilis. She has refused permission for an HIV test in the past. This young woman's recurrent meningitis is likely due to: A. HSV 2 B. VZV C. CMV D. Syphilis E. HIV

Correct Answer: A. HSV This is a typical presentation of recurrent benign lymphocytic meningitis - referred to as Mollaret's recurrent meningitis. Mollaret's meningitis is a type of benign recurrent aseptic (i.e., nonbacterial) meningitis that is most often due to herpes simplex virus type 2 (HSV‐2) infection. Genital lesions are usually absent at the time of presentation. None of the other choices is known to cause recurrent meningitis over years.

Which one of the following bacteria have a high frequency of intrinsic resistance to polymyxin antibiotics? A. Serratia marcescens B. Escherichia coli C. Klebsiella pneumonia D. Pseudomonas aeruginosa E. Acinetobacter baumannii

Correct Answer: A. Serratia marcescens The polymyxins are not active vs. all the aerobic Gram‐negative bacilli. Polymyxins (colistin and polymyxin B) bind to cell wall lipopolysaccharide. In some bacteria, the binding polymyxins fails to occur. A helpful acronym is M(A)PPS for bacterial genera frequently resistant to polymyxin antibiotics: M = Morganella A = "and" P = Providencia P = Proteus S = Serratia

You are asked to see a 38-year-old white male for evaluation of a severe occipital headache, which progressively worsened over the last 5 days. Three days ago he developed anorexia and nausea, which was followed yesterday by 2 episodes of vomiting. Today he has the worst headache of his life with continued nausea. Past history is negative except for recurrent "cold sores." He works as a computer consultant and returned 3 days ago from a 10-day business trip to Jamaica. On physical examination: He is in pain from his headache. Temperature: 37.7°C There is a small vesicular sore at the vermillion border of the left lower lip. The neck is stiff. Laboratory studies reveal: Hct: 42% Hgb:14.2 g/dL WBC: 8,200 (neutrophils 68%, lymphocytes 22%, monocytes 2%, eosinophils 8%) Blood Glucose: 106 mg/dL CSF protein: 64 mg/dL CSF Glucose: 63 mg/dL CSF WBC: 428/mm3 (neutrophils 16%, mononuclear cells 36%, eosinophils 48%) Gram stain: negative for organisms Which of the following is the most likely cause of this patient's illness? Balyisascaris procyonis Listeria monocytogenes Angiostrongylus cantonensis Herpes simplex virus Mycobacterium tuberculosis

Correct Answer: Angiostrongylus cantonensis This patient presents with a clinical picture of subacute, aseptic meningitis. Remember three organisms likely to be on the boards as a cause of eosinophilis meningitis: Angiostrongylus, Baylisascaris, and Gnathostomiasis. There are several features of his history and laboratory findings which are characteristic of a patient with eosinophilic meningitis caused by Angiostrongylus cantonensis. Most important among these are the findings of peripheral and CSF eosinophilia in a patient who has an epidemiologic history compatible with exposure to this organism, which is the rat lung worm. Infection in humans results from ingesting infected mollusks, principally snails, prawns and crabs. Leafy vegetables contaminated by slugs can also be a source. This organism is endemic in the South Pacific and Asia where infection is most commonly acquired but can also be seen in some Caribbean islands and Louisiana. Ingested larvae migrate from the intestine to the brain resulting in eosinophilic meningitis in people. Abdominal pain and vomiting can occur during larval migration. Larvae do not develop into adult worms in humans. Baylisascariasis is acquired from accidental ingestion of raccoon stool and the few reported cases have been in children. Thus, this is less likely in this adult with another logical exposure. Gnathostomiasis (occurs mostly in Asia, but also in other continents, but not North America) is acquired from ingesting infected molluscs or vegetables and may present with eosinophilia and migratory localized erythematous skin lesions and nodules or with eosinophilic meningitis. The geography, i.e., travel to Jamaica, is not appropriate. Patients with eosinophilic meningitis are typically not as toxic or severely ill as those with acute bacterial meningitis caused by organisms such as Streptococcus pneumoniae or Listeria monocytogenes. Similarly, unlike Herpes simplex encephalitis, patient's mental status is usually normal. Reactivation of herpes simplex, such as the lip lesion of this patient, is well known to occur during other stresses and infections. This reactivation does not suggest that HSV is the cause of the central nervous system syndrome in this patient. The CSF formula argues strongly against other causes of meningitis including Mycobacterium tuberculosis where the CSF protein is almost always more elevated and a marked eosinophilic pleocytosis is almost never observed. Correct Response Angiostrongylus cantonensis

A 44 year old male had blunt trauma to the abdomen and had an emergency splenectomy. He was previously in excellent health and has no evidence for HIV infection or an immunosuppressive disorder. What would you recommend to reduce the likelihood of overwhelming sepsis? Penicillin VK daily for 1 year plus appropriate immunizations Cephalexin daily for 5 years plus appropriate immunizations Moxifloxacin for life plus appropriate immunizations Appropriate immunizations but no antibiotic prophylaxis No antibiotic prophylaxis and Immunizations only if was unvaccinated as a child

Correct Answer: Appropriate immunizations but no antibiotic prophylaxis Asplenic patients have a small risk of overwhelming infection. Fulminant sepsis in asplenic patients is usually caused by encapsulated organisms, i.e. Streptococcus pneumoniae as well as Hemophilus influenzae and Neisseria meningitidis. Fulminant sepsis in asplenic patients has also been caused by Capnocytophaga canimorsus, which usually occurs after close contact with canine bites, scratches, or saliva exposure.. Asplenic individuals are also at risk for severe babesiosis, Antibiotic prophylaxis is usually recommended in asplenic children, i.e daily prophylaxis with oral penicillin VK or amoxicillin) until at least age five and for at least one year following splenectomy. However, such prophylaxis is not routinely recommended for adults since sepsis is less common in asplenic adults than in asplenic children and there is the possibility of antibiotic resistance and adverse effects if antibiotics are given prophylactically. Highly immunocompromised individuals and survivors of pneumococcal sepsis might be candidates for prophylaxis for life, but this is controversial. For patients undergoing elective splenectomy, the vaccines routinely recommended are pneumococcal, meningococcal, and Haemophilus influenzae vaccines. They should be administered at least 14 days prior to or if that is not feasible, after the 14th postoperative day Although not a testable point, splenectomized adults should also carry amoxicillin or moxifloxacin to take empirically if they develop fever or rigors while urgently seeking medical care.

A 46-year-old patient with hepatitis B, cirrhosis and ascites is being considered for transplantation. He is not currently bleeding, and has no fever or encephalopathy and his bilirubin and creatinine are only slight elevated. His labs show: Hepatitis B Sag positive, Hepatitis C antibody and PCR negative Creatinine 0.9 g/dl Bilirubin 2.0 mg/dl Transaminases 4 x normal Ultrasound: enlarged liver and splenomegaly with considerable ascites Diagnostic paracentesis show 100 wbc and no organisms on gram stain or culture. Which of the following would be the most appropriate indication for prophylactic antibiotics: Ascites Ascites only with documented cirrhosis Ascites with a prior episode of spontaneous bacterial peritonitis All patients awaiting liver transplantation regardless of liver histology or ascites

Correct Answer: Ascites with a prior episode of spontaneous bacterial peritonitis The best supported indications for antibiotic prophylaxis are either the presence of variceal bleeding, or the presence of ascites plus a history of at least one prior episode of spontaneous bacterial peritonitis. Long-term fluoroquinolone or trimethoprim sulfamethoxazole would be reasonable drug choices. Some experts would give long term prophylaxis to patients with ascites and some indicator of severity such as a bilirubin >3 mg/dl, Creatinine >1.2 mg/dl, or a Child Pugh >9. Patients such as this one who are admitted to the hospital should have a diagnostic paracentesis even if asymptomatic. They should be treated empirically if their ascitic wbc is >250 cells/mm3 and if they develop signs and symptoms of peritonitis even if their cell count is low.

A 56 year old white male with cavitary pulmonary disease due to Mycobacterium abscessus is transferred to your care. He is thought to be failing therapy. He is currently being treated with azithromycin, clofazimine, and moxifloxacin. He started this regimen 6 weeks ago with intravenous amikacin, but had sudden onset of tinnitus and the amikacin was stopped a month ago. His antibiotic susceptibility testing reports "susceptibility" to each of the medications within the current regimen except the moxifloxacin which has an MIC of 4.0. Tigecycline and linezolid were tested as possible additional agents for this patient and were found to be in "susceptible" range. Which of the drugs being tested may fail because of induced resistance in an isolate that appears susceptible? Clofazimine Tigecycline Moxifloxacin Azithromycin Linezolid

Correct Answer: Azithromycin Treatment of M. abscessus subsp. abscessus with azithromycin or clarithromycin has often been inadequate, despite appearing susceptible in vitro to these macrolides. Isolates of M. abscessus in the United States are most frequently those of the subspecies abscessus, as opposed to the sub-species bolletii or massiliense. Isolates of M. abscessus sub-species abscessus almost always carry a functional ERM gene that confers inducible macrolide resistance in-vivo. Azithromycin being used in the patient above likely has little or no activity against his organism due to inducible resistance. Most laboratories in the US do not test for the ERM gene or evaluate the isolate for inducible macrolide resistance, but rather simply report a macrolide to be "susceptible" in in-vitro testing. For the other therapies listed, there are no reports of inducible resistance. Nearly all isolates have low MIC values to clofazimine and tigecycline and they are reported as "susceptible." MIC values and reporting of linezolid and moxifloxacin are more variable, although a small proportion of isolates are deemed "susceptible" based on low MIC values. All of these agents have been used in therapies constructed against M. abscessus.

A 27‐year‐old pregnant female presents to the clinic for evaluation. She recently immigrated to the United States and is being evaluated for the first time at approximately 7 weeks of gestation. She is known to be HIV infected and had received nevirapine (alone) prophylaxis during a previous pregnancy. She denies having been on antiretrovirals, other than nevirapine. Her current CD4 count is 480 and her viral load is 10,000. Which of the following would be the best option for this patient? A. Start her on tenofovir (TDF)/lamivudine and nevirapine B. Start her on tenofovir/emtrictabine and darunavir/ritonavir C. Start her on tenofovir/emtricitabine/efavirenz D. Recommend no therapy currently but recommend the use of zidovudine (AZT plus lamivudine plus lopinavir‐ritonavir) at 36 weeks of pregnancy

Correct Answer: B. Start her on a tenofovir/emtricitabine and darunavir/ritonavir The keys for pregnancy are as follows: Antiretroviral therapy is recommended in all pregnant women, regardless of virologic, immunologic, or clinical parameters, for the purpose of preventing mother to child transmission. Both reduction of HIV RNA levels and use of antiretroviral therapy appear to have an independent effect on reduction of perinatal transmission. Antiretroviral therapy also benefits the mother, and should be administered for mother's health as well. Genotypic resistance testing is recommended for everyone initiating therapy, including all pregnant women. Tenofovir (TDF)/emtricitabine is among the recommended nucleoside combinations in pregnancy (along with abacavir/lamivudine and zidovudine/lamivudine). However, zidovudine is associated with significant GI toxicity and is given twice daily. Zidovudine related anemia is a particular concern among pregnant women who may develop anemia for reasons related to their pregnancy. Note TAF is not yet recommended for use in pregnancy because of insufficient data on its safety in pregnancy. Darunavir‐ritonavir is recommended for pregnancy, (atazanavir/ritonavir also is recommended; raltegravir is also recommended). Thus, regimen B is a good choice. With the history of use nevirapine monotherapy use, drug resistance to both nevirapine (and efavirenz) would be expected and so A and C are wrong. Also, nevirapine would be a poor choice due to her high CD4 count, putting her at high risk for hepatotoxicity. Moreover, nevirapine is seldom used for ART in the United States. Since all pregnant women should receive ART, D is wrong.

healthy 50‐year‐old never immunized for hepatitis B cleaning her son's bedroom when she was stuck by a blood tinged needle she found in a jar under his bed. Her son has been in and out of jail for drug abuse, and is known to have chronic hepatitis B (HbsAg positive) that has not been treated. You send off blood for liver function tests and HBV serologies Her prior liver function tests have been normal. On the assumption that this patient is HBV seronegative, and never vaccinated, which of the following would be the best recommendation for prevention of HBV? A. Immunize her for HBV as the only intervention B. Immunize her for HBV and provide HBIG C. Immunize, provide HBIG, and treat her with entecavir plus tenofovir for at least 4 week D. Provide HBIG alone E. Provide Entecavir plus tenofovir for at least 4 weeks and no other intervention

Correct Answer: B. Immunize her for HBV and provide HBIG all non‐immune persons exposed to HBsAg positive person by percutaneous, sexual, ocular or mucous membrane exposure including human bites that penetrate the skin. For this patient, one or two doses of HBIG given a month apart are appropriate if this can be begun in the first 24 hours (preferably within 12 hours of injury, accompanied by the first dose of vaccine in a different site, followed by the remaining doses of a vaccine series (one month and six months later). There would be no harm in giving this patient HBIG while awaiting the serologies, though the considerable expense would be wasted were she already infected. Treatment with drugs active against HBV (entecavir and tenofovir for at least 4 weeks) might be plausible, but is not currently recommended.

58 year old male with longstanding poorly controlled diabetes and neuropathy. He has had previous neuropathic ulcer and osteomyelitis. He developed a new midfoot ulcer that failed to heal with conservative measures, including two prior antibiotic courses for soft tissue infection. The ulcer measures 3x3 cm and does not probe to bone. What is the best way to diagnose osteomyelitis in this patient? A. 3‐phase bone scan B. MRI C. CT Scan D. White blood cell scan E. Deep wound cultures at bone surface

Correct Answer: B. MRI Diagnosis of Osteomyelitis in the Diabetic Foot 50‐60% of serious diabetic foot infections are complicated by osteomyelitis Presence of osteomyelitis can inform: - need for surgical debridement - route and duration of antibiotic therapy - Increased risk of limb amputation Consider evaluation for osteomyelitis if: - Severe infection - Longstanding (>30 days) or large (>2 cm) wounds - Recurrent infection around ulcer

Over a 3‐week period, 5 patients in a 12‐bed ICU have infections with a carbapenem‐resistant Klebsiella pneumoniae (KPC): Two have symptomatic urinary tract infections, 2 have ventilator‐associated pneumonia, and 1 has a line‐related bacteremia. These are the only KPC infections recognized in this ICU in the past 6 months. Whole genome sequence (WGS) analysis of the isolates shows that four are nearly identical and one probably genetically unrelated. The most likely epidemiologic explanation for these infections is that this cluster represents which of the following: A. Is a pseudoepidemic B. Results from lapses in infection control C. Results from common source medication contamination D. Represents a water‐borne outbreak E. Represents a food‐borne outbreak

Correct Answer: B. Results from lapses in infection control Pseudoepidemics usually reflect surveillance, diagnostic, or lab errors — unlikely in this scenario. So, the occurrence of a cluster of KPC likely is a real epidemic. The occurrence of a predominant WGS‐type and the multiple different sites of infection suggest cross‐infection due to poor infection control practices. The other explanations - water, food, or medication contamination — would not explain the sites of infection and are unlikely based on the known current behavior of KPC. The existence of 2 WGS types suggests 2 separate introductions of KPC into the ICU.

A 60-year-old man presented with a 6-month history of fever to 102-104°F, chills, headache and the indolent onset of lower back pain, which worsened until it awoke him from sleep. He also noted anorexia and 12-pound weight loss. Patient had recently emigrated from Turkey, where he lived in an urban area in Arizona. On exam, he had point tenderness over the lumbar spine with associated paravertebral muscle spasm. The WBC was 5.1. The ESR was 80. Blood cultures were sterile. The MRI is shown below. A needle biopsy of the L5 vertebral body showed epithelioid histiocytes but no organisms on acid fast and Gram stain. Cultures for bacteria and mycobacteria were negative. What test would best support the most likely etiologic agent? PPD skin testCoxiella burnetii phase 1 and 2 antibodyPCR for Tropheryma whipplei on the fixed tissue from the biopsyBrucella agglutination serologyCoccidioides CF antibody

Correct Answer: Brucella agglutination serology Brucellosis is rare in the United States, but common in Turkey and many other countries. Osteomyelitis is one of the more common late manifestation of infection, with vertebral or sacro-iliac bones one of the more common sites. The agglutination test is almost always positive in active brucellosis and should prompt treatment. Neither Whipple's disease (Tropheryma whipplei) nor Q fever (Coxiella burnetii) is a well-described cause of spondylitis. A positive PPD would be consistent with a diagnosis of tuberculosis but would not be sufficiently diagnostic to permit treatment. Coccidioidomycosis is not found in the Near Eastbut is a plausible distractor given his residence in Arizona. This patient has brucella spondylitis. A L4 laminectomy with decompression and aspiration of the epidural abscess produced free-flowing malodorous purulent material. Pathological exam showed focal epithelioid histiocytes with intracellular coccobacilli. Operative culture grew small gram-negative coccobacillli, identified as Brucella melitensisbiovar 3 by the CDC. The Brucellastandard agglutination test was positive with a titer of 1:320. Testing for Brucellaantibodies was also positive (IgM 1:320, IgG 1:640). Brucella spondylitis is prevalent in the Mediterranean, Arabian peninsula, South Asia,andCentral and South America. Although a systemic infection, localized osteoarticular complications may occur. Lumbosacral spondylitis and sacroiliitis are the most common sites of osteoarticular disease. Although Staphylococcal infection, tuberculosis and, less commonly, fungal and anaerobic infections can cause vertebral osteomyelitis, he is from an endemic area (the Middle East) and has a typical presentation (involvement of the lumbosacral spine; presence of intracellular organisms) forBrucella vertebral osteomyelitis. Correct Response

A 28-year-old male with history of type 2 diabetes mellitus comes in with 12 days of fevers, cough, and muscle aches. He recently returned from a community service mission in Vietnam. The last week there, he assisted with construction of a clinic in a rural area. On examination he has a fever of 103°F, and signs of consolidation in the lungs. His exam also shows multiple skin abscesses. He rapidly decompensates and is transferred to the intensive care unit. A CT of the chest reveals cavitary pneumonia and a liver abscess. The abscess is aspirated and reveals many gram negative organisms which on culture were an aerobic, oxidase positive bacillus which the automated system was unable to identify. Which one of the following is the most likely etiologic bacteria? Burkholderia pseudomallei Pseudomonas aeruginosa Burkholderia cepacia Aeromonas hydrophilia Pseudomonas stutzeri

Correct Answer: Burkholderia pseudomallei The patient likely has melioidosis, which is a disease caused by Burkholderia pseudomallei, a facultative intracellular gram negative organism that is endemic in Southeast Asia and Northern Australia. It is usually acquired after being directly exposed to contaminated soil or water. Inhalation of the organism often follows rain storms. It has also been described after near drowning, or after tsunamis. The organism grows well in culture but is often difficult to identify on automated systems and may be misidentified as B. cepacia, P. stutzeri, or other Pseudomonas species. Common clinical manifestations include skin lesions, pneumonia, and abscesses. It can involve virtually any organ. The illness may have an acute, sub-acute or chronic course. Melioidosis is more common in patients with underlying illness such as diabetes or chronic renal disease. The recommended initial therapy is parenteral, with ceftazidime or imipenem. Most aminoglycosides are inactive. Once the patient has improved, prolonged therapy with an oral agent such as trimethroprim/sulfamethoxazole is recommended.

A 55‐year‐old man chronic HBV, HBV sAg positive, and genotype 1a HCV coinfection presented for treatment of HCV. His HIV serologic test was negative. MRI abdomen demonstrated evidence of cirrhosis and portal hypertension. Clinically, the patient had compensated cirrhosis with Child‐Pugh class A liver disease. The pretreatment HCV RNA viral load (VL) was 1.3 million IU/mL. Treatment with sofosbuvir and ledipsavir was initiated and resulted in a rapid decline in HCV VL. After 7 weeks of treatment the patient began to experience malaise, nausea, and epigastric abdominal pain. Physical examination at week 8 revealed significant jaundice with tender hepatomegaly. Liver function tests (LFTs) were now significantly abnormal, with an AST of 1792 IU/L, ALT of 1495 IU/L, total bilirubin of 12.2 mg/dL, and INR of 1.96. Which of the following pre HCV treatment laboratory tests on peripheral blood would be most likely to explain his symptomatic illness occurring during successful HCV treatment? A. EBV PCR B. CMV PCR C. HBV PCR D. HIV PCR E. Hepatitis A IgM

Correct Answer: C. HBV PCR Chronic HCV infection suppress HBV replication patients co infected with HCV and HBV. Both intrahepatic and peripheral HBV antigen levels are reduced in patients chronically coinfected with HCV compared to individuals with HBV alone. Similarly, patients with chronic HBV have higher rates of surface antigen clearance following HCV superinfection], and viral replication in acute HBV infection may be attenuated in patients with preexisting HCV. One way to look at this, despite uncertainty of mechanism, is that HCV and HBV compete with each other: if HCV is removed, HBV may become more active. HBV DNA levels must be closely monitored in coinfected (HBV and HCV, with or without HIV) patients during HCV treatment with interferon‐free, direct‐acting antiviral regimens. In patients with a positive HBcAb but a negative HBsAg and HBsAb, HBV VL should be obtained prior to treatment and monitored during therapy. These patients are at less risk of reactivation of HBV, however, than patients who are HBsAg positive compared to those who are only HB core antibody positive. Early initiation of anti‐HBV therapy in the setting of an increasing HBV VL should be strongly considered Since the patient was asymptomatic when he began HCV therapy, he was unlikely to have hepatitis A (ie. Positive Hepatitis A IgM). Chronic HIV would not explain this hepatic flare, and this would be unusual for acute HIV.

A 68‐year‐old diabetic woman is admitted for pneumonia. A rapid RT‐PCR test is positive for influenza A. Sputum Gram‐stain shows Gram‐positive cocci in clusters. The culture is growing methicillin‐resistant Staphylococcus aureus susceptible to vancomycin, daptomycin, linezolid, and trimethoprim‐ sulfamethoxazole (TMP/SMX). She has been receiving vancomycin 1 gm once daily for the past 3 days based on a calculated creatinine clearance of 45 ml/min and has a trough concentration of 22 μg/ml. The primary physician is concerned because a vancomycin susceptibility result has returned with MIC = 2 μg/ml and has asked you to recommend an alternative agent. Which one of the following alternative regimens would you recommend? A. Ceftaroline 600 mg IV q12h B. Daptomycin 6 mg/kg IV once daily C. Linezolid 600 mg PO twice daily D. Telavancin 7.5 mg/kg IV once daily E. TMP‐SMX 10 mg/kg (TMP component) per day in 2 divided doses

Correct Answer: C. Linezolid 600 mg PO twice daily. Vancomycin could have been continued because an MIC of 2 mcg/ml is still considered susceptible, though some have questioned whether failure is more frequent in isolates with that MIC. Linezolid is FDA approved for MRSA pneumonia. Ceftaroline is not approved for MRSA pneumonia despite anecdotal reports of success. Clinical trials are in progress. Until results are available, caution is indicated in the use of ceftaroline for pneumonia as the dosage and efficacy are unclear. Daptomycin is not indicated for the treatment of pneumonia based on clinical trails wherein patients with S. pneumoniae CAP failed, presumably due to binding of the drug to pulmonary surfactant. Whether daptomycin would fail treatment of S. aureus pneumonia is unknown as S. aureus infections are necrotizing and destroy surfactant while S. pneumoniae infection does not result in degradation of surfactant. In patients with right‐sided endocarditis and embolic S. aureus pneumonia, daptomycin seemed efficacious. None the less, probably it is good to avoid treatment of pneumonia with daptomycin until more data are available. Telavancin has a black box warning from FDA because of observed increased mortality versus vancomycin in patients with creatinine clearances < 50 ml/min. TMP‐SMX is not FDA approved for treatment of MRSA pneumonia, the proper dose is not defined and it is associated with hyperkalemia in diabetics with pre‐existing renal insufficiency.

A 48 year old woman is evaluated for the sudden onset of painful blisters on her vulva 2 days earlier. She denies any previous history of genital lesions or any sexually transmitted infections. She has been married for 28 years and she denies any extramarital contacts. To her knowledge, her husband is monogamous and has never been diagnosed with any STI. On examination, she has 8 superficial ulcers with erythematous bases covering her vulva. No lymphadenopathy is noted and the remainder of the genitourinary examination is unremarkable. Her HIV antibody test is non‐reactive as are her tests for chlamydia, gonorrhea, trichomoniasis, and syphilis. HSV‐2 PCR from one of the vulvar lesions is positive for HSV‐2. You tell the patient that two potential explanations for her clinical presentation are a new infection or the first clinical presentation of a long‐standing infection. Which of the following test results would best support the explanation that her infection is a recent one? A. Non‐reactive serum IgM antibodies to HSV‐2 B. Reactive serum IgM antibodies to HSV‐2 C. Non‐reactive serum glycoprotein G IgG antibodies to HSV‐2 D. Reactive serum glycoprotein G IgG antibodies to HSV‐2

Correct Answer: C. Non‐reactive serum glycoprotein G IgG antibodies to HSV‐2 It can be difficult to distinguish between a recent HSV infection and the first clinical presentation of a long‐ standing infection. One must be very careful as misinterpretation of diagnostic tests in this setting may lead to unnecessary strain on a relationship. A positive HSV‐2 PCR would be expected in both scenarios so it does not help distinguish between the two. Serum IgM antibodies are neither sensitive nor specific for a recent HSV infection. As such, they should never be used to diagnose an incident infection. Don't order or try to interpret these tests. Serum glycoprotein‐G based IgG antibodies are sensitive and specific tests. As with all antibody tests, they will be nonreactive early in the course of the infection. Consequently, non‐reactive serum Glycoprotein G IgG antibodies early in the course of the infection in the setting of a positive PCR for HSV‐2 is suggestive of a recent infection (one can repeat the Glycoprotein‐G based IgG antibodies 4‐8 weeks later and a reactive test result at that time would confirm the diagnosis of a recent infection). Reactive IgG HSV‐2 antibodies very early after the onset of lesions would support the diagnosis of first clinical presentation of a long‐standing infection (although early seroconversion in persons with a recent infection can occur‐ so be careful). The best answer is answer C: finding nonreactive serum Glycoprotein G IgG antibodies in a patient with a positive PCR for HSV‐2. So, does this mean that her husband was unfaithful? Not necessarily: He could have been one of the 70% of persons infected with HSV‐2 who are asymptomatic and only now, did he transmit the infection to his wife‐ 29 years into their marriage (or, he could have been unfaithful)!

A 66 year old patient in the ICU is day 6 post‐operative following a pancreatectomy for pancreatic carcinoma. He is recovering uneventfully with improving renal and hepatic function. On the evening of his 6thpost‐operative day, he develops a fever of 38.50 C. The surgeons draw three cultures from an indwelling port that was placed preoperatively for chemotherapy that has not yet started. No cultures are drawn through his peripheral IV or percutaneously Piperacillin tazobactam is started. On Day 7 the patient remains intermittently febrile but is otherwise stable with no new findings. Labs are remarkable only for a WBC that continues to decline following surgery and is now 7800 cells/uL with 70% neutrophils The lab calls to say that after 14 hours of incubation, all three cultures are growing gram positive cocci in clusters. By the time the surgeons call you the next morning, the patient has been stable, the port and the peripheral IV look fine, there are no other concerning physical findings or lab values, and the organisms have been identified by MALDI‐Tof as Staphylococcus epidermidis. The surgeon is very eager to retain the port. Assuming that there are no unusual resistance patterns, what would you recommend pending the results of follow up cultures: A. Vancomycin should be started, the port should be removed B. Oxacillin should be started, and the port should be removed C. The port can be retained, vancomycin plus lock therapy can be administered for 10‐14 days, and the port left in place if the patient is stable with no further Staphylooccus epidermidis bacteremia D. The port can be retained, vancomycin plus lock therapy can be administered for 4‐6 weeks, and the port left in place if the patient is stable with no further Staphylooccus epidermidis bacteremia E. Only if an echocardiogram is negative, the port can be retained, vancomycin plus lock therapy can be administered for 10‐14 days, and the port left in place if the patient is stable with no further Staphylooccus epidermidis bacteremia

Correct Answer: C. The port can be retained, vancomycin plus lock therapy can be administered for 10‐14 days, and the port left in place if the patient is stable with no further Staphylooccus epidermidis bacteremia This patient likely has a port infection due to Staphylococcus epidermidis. This catheter has a high likelihood of being salvaged. The risk of endocarditis is extremely small with this organism. 10‐14 days of therapy should be sufficient. There are excellent tables in the IDSA Catheter Management Guideline https://academic.oup.com/cid/article/49/1/1/369414 58 Long‐term catheters should be removed from patients with CRBSI associated with any of the following conditions, none of which this patient appears to have: severe sepsis; suppurative thrombophlebitis; endocarditis; bloodstream infection that continues despite >72 h of antimicrobial therapy to which the infecting microbes are susceptible; or infections due to S. aureus, P. aeruginosa, fungi, or mycobacteria. For patients with CRBSI for whom catheter salvage is attempted, additional blood cultures should be obtained, and the catheter should be removed if blood culture results (e.g., 2 sets of blood cultures obtained on a given day) remain positive when blood samples are obtained 72 h after the initiation of appropriate therapy. For long‐term and short‐term CRBSI due to less virulent microbes that are difficult to eradicate (e.g., Bacillus species, Micrococcus species, or Propionibacteria), catheters should generally be removed after blood culture contamination is ruled out on the basis of multiple positive culture results, with at least 1 blood culture sample drawn from a peripheral vein.

A 34 y/o female with Crohn's disease is hospitalized after three months of headache and progressive confusion. She has had sweats and weight loss. She was born and raised in Pakistan, where she trained as a nurse. She immigrated to the U.S. five years ago, has lived in Massachusetts since then, and has no other travel. Home medications include azathioprine, as well as regular infliximab infusions started 6 months ago. On exam she is somnolent and oriented only to person. There is mild nuchal rigidity. Brain MRI shows leptomeningeal enhancement with involvement of the basal cisterns; no parenchymal lesions, temporal lobes normal. Cerebrospinal fluid findings: glucose 30 mg/dL (serum glucose 90 mg/dL), protein 110 mg/dL, 1 RBC/cu mm, 450 WBC/cu mm (95% lymphocytes), Gram stain negative for bacteria, fungal stain negative, auramine stain negative for acid fast bacilli, cryptococcal antigen negative. CSF bacterial, fungal, and mycobacterial cultures are negative so far after 48 hours of incubation. The diagnostic procedure most likely to yield the underlying diagnosis is: serum toxoplasma IgG assayserum toxoplasma IgM assayCSF mycobacterial cultureCSF HSV PCRUrine histoplasmosis antigen

Correct Answer: CSF mycobacterial culture Tuberculosis meningitis is the most likely diagnosis in this patient who has epidemiologic risks for M. tuberculosis exposure and is receiving an anti-TNF agent (infliximab). Anti-TNF agents increase the risk of active TB in someone who has been infected with M. tuberculosis, and increase the risk of severe and non-pulmonary forms of TB. Tuberculosis meningitis typically causes a lymphocyte-predominant CSF pleiocytosis, with moderately elevated protein and moderately decreased glucose. Typically there are very few M. tuberculosis bacteria detectable in the CSF; smear microscopy typically is negative, and the most sensitive test of CSF is mycobacterial culture (the yield is increased with increasing volume of CSF, and increasing number of cultures). Nucleic acid amplification tests of CSF can be useful adjuncts but are less sensitive than cultures. Mycobacterial culture of CSF is not 100% sensitive, and if clinical suspicion is high (as in this case), therapy should be initiated. Central nervous system toxoplasmosis typically causes parenchymal lesions and not basilar meningitis. This patient has no epidemiologic risk factors for histoplasmosis. HSV meningoencephalitis is associated with CSF lymphocytic pleiocytosis, but in this patient the long duration of symptoms and the absence of temporal lobe abnormality on MRI make HSV meningoencephalitis less likely.

A 19-year-old young woman has had a slowly progressive ascending paralysis over the last two days. She can no longer walk. One week before she developed neurologic symptoms, she had an episode of fever, cramping abdominal pain, and diarrhea that lasted three days. Which one of the following was the most likely cause of her gastrointestinal illness? Shigella sonnei Clostridium botulinum Enterovirus Campylobacter jejuni

Correct Answer: Campylobacter jejuni Guillain-Barre is the most common cause of acute neuromuscular paralysis in the world. Many cases follow an obvious antecedent infection and many such infections have been described. One of the most common of these, if not the most common, is gastroenteritis due to Campylobacter jejuni. Campylobacter jejuni infection is the most commonly identified precipitant of Guillain Barre Syndrome. Cytomegalovirus, Epstein-Barr virus, HIV, and Zika virus have also been associated. The presumptive mechanism for this syndrome is that an antecedent infection that evokes an autoimmune response. This immune response cross-reacts with peripheral nerve components because of sharing of cross-reactive epitopes: (ie, molecular mimicry). An acute polyneuropathy results. . This immune response is directed towards the myelin or the axon of peripheral nerve. Shigellaproduces febrile diarrhea and may be associated with seizures; Guillain-Barre like illness would be reportable. Clostridium botulinumis a very rare cause of GI symptoms other than constipation as part of infant botulism; botulism is a descending paralytic illness. Many patients with invasive listeriosis have a history of antecedent GI illness but listeria is more often associated with meningoencephalitis, not ascending paralysis. CMVis not associated with a Guillain-Barré- like syndrome. Enteroviruses can cause aseptic meningitis but not diarrhea or Gullain-Barré syndrome.

A previously healthy 30-year-old woman presented with right temporal headache, eye pain, diplopia and decreased vision in the right eye. On exam, her temperature was 102°F. Her left eye had normal extraocular movement and vision. On the right, there was periorbital edema, proptosis, chemosis, and ptosis; the right eye was fixed in the midline. Vision in the right eye was reduced to count fingers at 3 feet. She underwent a lumbar puncture; CSF glucose 69 mg/dL, protein 180 mg/dL, 3,000 WBC/mm3(82% P, 18% L). An MRI is ordered. The most likely diagnosis is: Superior sagittal sinus thrombosis Cavernous sinus thrombophlebitis Bacterial endophthalmitis Mucormycosis Right ethmoid sinusitis

Correct Answer: Cavernous sinus thrombophlebitis The patient underwent MR angiography of the head, which revealed thrombosis of the right cavernous sinus. Cultures of blood and CSF grew methicillin-sensitive Staphylococcus aureus. The source was unknown. The facial veins and pterygoid plexus drain into the cavernous sinus via the inferior and superior ophthalmic veins; as a result, infections of the face can spread to the cavernous sinus. The cavernous sinus encompasses CN III, IV, V1, V2; CN VI is located more medially, near the internal carotid artery. Cavernous sinus thrombosis presents with headache, eye swelling, proptosis, diplopia, ptosis and usually visual loss; the findings often become bilateral as thrombosis spreads to the contralateral sinus through a venous plexus that crosses the midline. The other diagnoses on the list would not be expected to give such prominent proptosis, ptosis, and ophthalmoplegia. Moreover, the CSF formula is consistent with a parameningeal focus; the other diagnoses on the list would not be expected to have such a CSF formula. Mucormycosis would not be expected in a nondiabetic.

A 45- year-old woman with a history of asthma presented to her primary care physician in January with twodays of fever, chills, and sore throat. Because influenza was circulating in the community, the physician prescribed oseltamivir, 75 mg po twice daily. She defervesced but remained fatigued. Five days later, she developed high fevers, cough, chest pain, and shortness of breath. She presented to the ED where she is hypotensive, hypoxemic and tachycardic. The WBC is 17,000 with 20% band forms, andthe chest X-ray shows infiltratesin both lower lobes with a largeleft pleural effusion with possible loculations. She is admittedto the ICU. Which of the following is the most appropriate therapy while awaiting results from blood and pleural fluid cultures? IV peramivir Ceftriaxone and azithromycin Levofloxacin Ceftriaxone and vancomycin Piperacilin/Tazobactam

Correct Answer: Ceftriaxone and vancomycin Ceftriaxone and vancomycinis the best option ofthose given. Chest pain, leukocytosis andpleural effusion strongly suggest bacterial pneumonia as a complication of influenza. Staphylococcus aureusinfections are an importantconsideration in bacterial infections complicating influenza, in addition to S. pneumoniaeand S. pyogenes. The probability of staphylococcal pneumonia is increasedafter influenza. The severity of illness and the possible empyema make staphylococcus, including MRSA an important consideration. Vancomycin or linezolid should be usedwhen MRSA pneumonia is suspected. IV peramiviris incorrect. The presentation suggests bacterial superinfection rather than the development of resistance so treating for resistant influenza is not critical. Moreover, the patient has already received a course of a neuraminidase inhibitor. Peramivir may not be active against oseltamivir resistant strains Ceftriaxone and azithromycinisnot optimal. Ceftriaxone and azithromycin have limited activity against S. aureus. Levofloxacinwould be appropriate for a non ICU patient with CAPbut not the best choice for possible MRSA pneumonia. Piperacillin/Tazobactamis not optimal - it lacks activity against MRSA.

A 25-year-old female from California presents with the acute onset on severe pain in her right wrist. She is sexually active with multiple male partners, who only intermittently use condoms. On presentation vital signs were temperature 102oF, pulse 100, respirations 16, and blood pressure 110/70 mmHg. Examination of the right wrist revealed swelling, erythema, and pain on motion. She has a few painless papules on examination of the skin of the lower extremities. Which of the following tests is most useful in establishing the etiologic diagnosis in this patient? Blood culture Synovial fluid Gram stain Synovial fluid culture Cervical NAAT

Correct Answer: Cervical NAAT This patient has disseminated gonococcal infection (DGI), which may present as a purulent gonococcal arthritis or a syndrome of tenosynovitis, dermatitis, and polyarthralgia without purulent joint infection. Fewer than 50% of patients have a true septic arthritis. In patients with gonococcal arthritis, the synovial fluid Gram stain is positive in only 25% of patients and cultures are positive in only 50%; in those without suppurative arthritis, synovial fluid cultures are only positive in 20-30% of cases. Blood cultures yield positive results in <30% of patients. Yield is greatest from mucosal sites - cervical swabs are positive in up to 80% of women and urethral swabs are positive in up to 70% of men. Testing of extragenital sites (throat and rectum) will enhance the yield so it is important to test at all mucosal sites. NAATs are the most sensitive tests to use at all mucosal sites.

Which one of the following is the most likely mechanism of the resistance of aerobic gram-negative bacilli to the polymyxins: polymyxin B and colistin (polymyxin E): Enzymatic modification of the polymyxin Change in the drug site of action Shedding of capsular polysaccharides Presence of a plasmid-mediated efflux pump

Correct Answer: Change in the drug site of action Polymyxins act by binding to Lipid A in the bacterial cell membrane. The known mechanisms of polymyxin resistance act by modifying Lipid A so the polymyxins bind less well. Until recently, the resistance genes were found in the chromosome. Recently, a gene,mcr1(mobile colistin resistance), was found in China that causes polymyxin resistance that is transmissible between bacteria via plasmids The gene is now detected worldwide. So far, there is no documentation of enzymatic alteration of the polymyxins. A few papers have described efflux pumps. Since polymyxin acts on the surface of the bacterial cell, there is, at least in theory, no internal drug to efflux. Some K. pneumoniaestrains are able to shed their capsular polysaccharides (CPS) that includes polymyxin-binding targets. The released CPS could trap or bind polymyxin and might lessen the amount of drug that reaches the cell surface. As the poymyxin target is the deeper lipid A, shedding of polysaccharide low in lipid A content seems a less important mechanism of resistance.

A 19-year-old male with no significant past medical history was found difficult to arouse by his roommate in his college dormitory room and was taken to the emergency room. Blood and cerebrospinal fluid cultures grew Neisseria meningitides serogroup pending. The patient's roommate received the quadrivalent meningococcal conjugate vaccine 1 year earlier Which is the following is the most appropriate approach to prevent infection in the patient's roommate? Chemoprophylaxis Booster immunization with the meningococcal conjugate vaccine Vaccination with the serogroup B meningococcal vaccine Empiric antimicrobial therapy for meningococcal infection No intervention is required

Correct Answer: Chemoprophylaxis Although the patient's roommate was vaccinated with the quadrivalent meningococcal conjugate vaccine, that vaccine only provides protection against infection caused by meningococcal serogroups A, C, Y, and W135, but not Serogroup B. In this case you do not know what serogroup the patient had. Chemoprophylaxis is necessary for contacts of patients with invasive meningococcal disease. Those at increased risk are household members, day care center contacts, those in contact through dormitory living (11-fold increased risk), persons directly exposed to the patient's oral secretions (e.g., through kissing, mouth-to-mouth resuscitation, endotracheal intubation, or endotracheal tube management), and the index patient if he/she is not treated with a third generation cephalosporin because some antimicrobial agents (e.g., high-dose penicillin and chloramphenicol) do not reliably eradicate meningococci from the nasopharynx of colonized patients. Serogroup B accounts for approximately one-third of cases of invasive meningococcal disease in the United States. The Advisory Committee on Immunization Practices recently recommended that persons >10 years of age who are at increased risk for meningococcal disease should receive the serogroup B meningococcal vaccine; this includes those at increased risk because of a serogroup B meningococcal outbreak. However, because an outbreak has not been demonstrated in this instance, vaccination would not yet be indicated and the roommate should receive chemoprophylaxis. Options for chemoprophlaxis are rifampin, ceftriaxone, azithromyin, and ciprofloxacin. However, it should be noted that cases of fluoroquinolone-resistant N. meningitidis disease have been reported among residents in the North Dakota/Minnesota border and the CDC does not currently recommend ciprofloxacin as empiric antimicrobial chemoprophylaxis of meningococcal disease in selected counties of North Dakota and Minnesota.

A 25-year-old man comes to the emergency room complaining of numbness. He returned from a honeymoon in Puerto Rico earlier today. He reports an episode of nausea, vomiting and diarrhea yesterday afternoon after eating lunch (salads, fruits, chicken, fish, beef). This morning he developed numbness in legs, arms, and pain in his teeth. He also reports that a cup of iced water felt warm to touch. On examination his heart rate is 55 and he is mildly diaphoretic. There is no weakness, deep tendon reflexes are slightly decreased, and he is diffusely tender to palpation. What is the diagnosis? Ciguatera poisoning Tetrodotoxin poisoning Scombroid poisoning Organophosphate intoxication Botulism

Correct Answer: Ciguatera poisoning Ciguatera poisoning accounts for over half of fish-related foodborne disease tropics and subtropics; different toxins that are formed by dinoflagellates, which are consumed by fish, and then predatory fish concentrate the toxin in their organs and flesh. The most common sources are barracuda, moray eel, amberjack, and certain types of grouper, mackerel, parrotfish, and red snapper. The ciguatera toxin-containing fish does not taste, smell, or appear unusual. Cooking, marinating, freezing, and stewing fish does not destroy the toxins. Gastrointestinal symptoms are common a few hours after the exposure, with vomiting, diarrhea, and abdominal cramping. Neurologic symptoms begin 3-72 hours after the meal, and include paresthesias, painful teeth, blurred vision, nerve palsies, and hot/cold temperature reversal. Occasionally bradycardia or hypotension are seen. The neurologic symptoms persist from a few days to several weeks or months. The diagnosis is made clinically. There is no commercially available diagnostic test. Mannitol may help when given within 48 hours, however one randomized trial found no benefit. Amitryptiline, fluoxetine, and gabapentin have also been used. Incorrect Answers: Scombroid poisoning is common throughout the world and presents shortly after ingesting contaminated fish, the symptoms resemble an IgE-mediated allergic reaction with flushing, an erythematous rash, and palpitations. Tetrodotoxin poisoning occurs after ingesting some species of pufferfish. Tetrodotoxin is one of the deadliest natural toxins, producing weakness, dizziness, paresthesias, severe hypotension and, in some cases, general paralysis, respiratory failure and death. Organophosphate intoxication is characterized by manifestations of cholinergic excess, such as salivation, tearing, weakness and convulsions. Botulism produces descending weakness and cranial neuropathies.

A woman in her 60s presented with three months of non-productive cough, night sweats and a 10-pound weight loss. She had received empiric clarithromycin, but the symptoms persisted. The patient had traveled in the past year to South America, Europe, Arizona and Australia. A Chest CT showed a left upper lobe lung nodule. Pathology from a lung biopsy is shown. The most likely diagnosis is which of the following: HistoplasmosisCryptococcosisPneumocystis jiroveciiCoccidioidomycosisParacoccidioidomycosis

Correct Answer: Coccidioidomycosis Discussion: This patient has pulmonary coccidioidomycosis. Pathology of the lung revealed multiple caseating and non-caseating granulomas with fungal yeast forms present. Special stains showed spherules with endospores, consistent with coccidioidomycosis. Culture revealed a filamentous organism, and gene probe was positive for Coccidioides immitis or Coccidioides posadasii. C. posadasii is endemic to the Southwestern U.S., Mexico, Central and South America. C. immitis is found in the central valley of California. After entering by inhalation, C. posadasii and C. immitis can cause a febrile illness with cough, chest pain and shortness of breath. CXR may show unilateral pulmonary infiltrates and hilar adenopathy. Risk factors for systemic dissemination include immunocompromised state (e.g. HIV), male sex, 3rd trimester of pregnancy or post-partum, African or Filipino ancestry. The morphology of the organism in the pathologic specimen is key is distinguishing between the potential diagnoses. The pathologic specimen shows a large spherule with endospores, which is characteristic of Coccidioides. By contrast, Histoplasma, Cryptococcus and Paracoccidioides are budding yeast in tissue. The trophozoites and cysts of Pneumocystis jirovecii are round, nonbudding spheric structures with no internal spores.

A 35-year-old Hispanic American female in Brownsville, Texas, who is in the fourth month of pregnancy is admitted to the hospital because of low-grade fever and malaise. Physical examination is unremarkable except for a temperature of 38°C and her pregnancy. Chest x-ray, WBC and routine blood chemistries were normal. Cefotaxime was begun. By the end of the second hospital day, the patient was afebrile and was discharged home on cefaclor. The next morning, the laboratory reported that an admission blood culture was growing on aerobic incubation a slender, catalase-positive, Gram-positive rod. Colonies were white and had a small zone of beta hemolysis. Which of the following is the most likely source of infection? A wool rug from Pakistan Desert dust Coleslaw Her own intestinal tract Sexual intercourse

Correct Answer: Coleslaw The clues or buzz words to think of instantly: wool (anthrax), desert dust (coccidioides), coleslaw (listeria), own intestinal tract (E. coli), sexual intercourse (gonorrhea). Gram-positive aerobic bacilli would include Bacillus anthracis, other Bacillus species, Erysipelothrix rhusiopathiae, Rhodococcus equi, Corynebacterium diphtheriae, other Corynebacteria, Arcanobacterium haemolyticum and Listeria monocytogenes. Presentation of bacteremia with no apparent source and the cultural characteristics are most consistent with listeriosis. Pregnancy increases susceptibility to Listeria bacteremia. Although maternal infection is often mild, bacteremia can lead to fatal disseminated infection in the fetus. This patient probably improved spontaneously, not from the cephalosporin. Food, such as dairy products and undercooked meat, is a common source of listeriosis. In a Canadian outbreak, 41 cases were attributed to ingestion of coleslaw. This patient should be treated with ampicillin. Listeria infection in pregnancy can resolve spontaneously, but they can be associated with considerable fetal morbidity and mortality. An important teaching point: the patient must have been exposed in the past 3 months as this will not occur in someone hospitalized for many, many months barring some unusual in hospital exposure to food brought in by the family, for instance. The incubation period for Listeria gastroenteritis is substantially shorter than the incubation period for invasive disease: mean 24 hours (range 6 hours -10 days) for gastroenteritis whereas for invasive listeriosis the median incubation is 35 days (range 1 -91 days).

A macramé worker who uses wool products provided by a friend in her home country of Pakistan has a 3cm painless forearm eschar with a necrotic center draining large amounts of secretions that are soaking through her wound dressings. The forearm is swollen. What is the appropriate precaution/isolation for this patient? Respiratory Contact Droplet Standard Strict (Respiratory & Contact)

Correct Answer: Contact Based on the exposure (wool presumably contaminated with anthrax spores) and clinical findings (painless eschar and edema), this patient may have cutaneous anthrax. Care is in standard precautions unless there are uncontained secretions, as in this case, which warrant contact precautions because of possible transmission through any non-intact skin contact of healthcare workers who are exposed to the patient's draining lesions. Type of Anthrax/Type of Isolation/Comments Gastrointestinal/Standard/Infected patients do not generally pose a transmission risk. Cutaneous/Standard/Transmission through non-intact skin contact with draining lesions possible, therefore use Contact Precautions if large amount of uncontained drainage. Hand washing with soap and water preferable to use of waterless alcohol based antiseptics since alcohol does not have sporicidal activity. Pulmonary/Standard/Not transmitted from person to person.

Owners of an aquatic park were notified by the Public Health Department that 8 children had developed diarrheal disease in the week following their visit to the park. The children had profuse, watery diarrhea, abdominal cramping, and low-grade fevers. The illnesses were all self limiting. Parasitology examination of their stool specimens was positive with acid-fast organism, 4-6 µm in diameter. A biopsy also demonstrated the organism on the mucosal surface. The Public Health Department closed the park, and collected specimens from the food service and water pools. Which of the following is the most likely source of these infections? Basil in saladChickenContaminated waterHotdogsSeafood appetizer

Correct Answer: Contaminated water This outbreak is caused by Cryptosporidium, the only acid-fast parasite that is 4 to 6 µm in diameter. The most common source for Cryptosporidium infections in the United States has been contaminated water. Keep in mind the relative sizes of three acid fast parasites: cryptosporidium (4-6u), cyclospora (8-10u), and isospora (20-35u).

A 36-year-old female who was 2 years post-cadaveric renal transplantation for renal failure due to chronic glomerulonephritis presented with fever of five days duration. She had some nausea but no urinary, respiratory or abdominal symptoms. She had presented to an outside hospital three days previously where a chest x-ray, urinalysis and blood culture had been negative. She was given levofloxacin but remained febrile with malaise. Current medications included mycophenylate, sirolimus and prednisone 20 mg. Examination found a fever of 39.2°C grade 1 systolic ejection murmur over the left sternal border, and a non-tender transplanted kidney in the right lower quadrant. Renal ultrasound of the transplanted kidney was normal. Urine culture grew 100,000 colonies of E. faecalis, susceptible to ampicillin. Urinalysis found 10 WBC/ul, nitrate and protein negative. WBC was 10,200. Creatinine 1.3 mg/dl Abdominal CT with contrast showed a lobe of the kidney which did not perfuse well with contrast and was swollen (Fig). There is no evidence of abscess formation. There was no evidence of urinary obstruction. Ampicillin 2 gm IV q 6h was begun but the patient remained febrile the next 24 hours. Which of the following is the most appropriate management? CT-guided biopsy of the affected kidney Add gentamicin Wedge resection of affected area of kidney Check urine for "decoy" cells of BK virus Continue ampicillin at same dose

Correct Answer: Continue ampicillin at same dose The findings are consistent with "lobar nephronia" or acute focal bacterial nephritis. This form of acute pyelonephritis is characterized by edema and inflammation of one or occasionally 2 kidney lobes. If not treated adequately, the affected lobe can liquefy and form a renal cortical abscess. Thus, this patient does not currently have an abscess that needs to be drained, but an abscess could ultimately develop. There is no reason at this point to do a biopsy or a surgical resection. If an abscess were present (this likely would be suggested by the ultrasound which would identify fluid/pus) that would have to be drained. Response to therapy is usually somewhat slower than pyelonephritis and relapse is probably more common if less than three weeks of treatment are given. Levaquin would not have been a good therapeutic choice. Response of enterococcal pyelonephritis to Levaquin is suboptimal, but may reduce abnormalities of the urinalysis and urine culture. BK viruria causes renal failure but not fever in renal allograft recipients. Unlike enterococcal endocarditis, addition of an aminoglycoside is not necessary in treating enterococcal urinary tract infection.

A 24 year old male has chronic myelocytic leukemia that has been refractory to conventional therapy. After an unsuccessful peripheral stem cell transplant from his brother, his bone marrow biopsy and peripheral smear show relapsed B lymphoblastic acute lymphocytic leukemia. He is referred for CART T cell therapy (Chimeric antigen receptor T cells) which he receives following a preparative regimen consisting of fludarabine plus Cytoxan. He is maintained on the following anti‐infective prophylaxis: levofloxacin, acyclovir, micafungin, and aerosol pentamidine On day 10 following CAR T cell infusion, the patient develops fever to 39C on several serial measurements, distributive shock, and diffuse pulmonary infiltrates. He is transferred to the ICU where his is administered 2 liters of saline, given low dose norepinephrine to bring his mean blood pressure to >60mm/Hg, and placed on supplemental oxygen since his O2 saturation on room air was 90%. . Labs reveal that he is profoundly neutropenic (Absolute neutrophil count < 200), with serum creatine rising from 1.3mg/dl to 2.4mg/dl, and transaminases rising from 1.5 x normal to 3x normal. What is the best initial course of action after blood cultures are drawn and pulmonary is being consulted about bronchoscopy: A. Administer high doses corticosteroids (1g solumedrol q12h x 6doses) B. Administer tocilizumab daily for 4 days C. Provide supportive care pending measurement of IL6 and Tumor necrosis factor‐alpha levels D. Administer broad spectrum anti‐bacterials +/‐ anti‐candida therapy E. Administer infliximab for three consecutive days

Correct Answer: D. Administer broad spectrum anti‐bacterials +/‐ anti‐candida therapy neutropenic patient fever and shock septic shock promptly receive broad spectrum antimicrobials, the first hour could have CAR T cell "cytokine storm," which is admittedly a bad name for the immunologic consequences of certain cell basd therapies, but septic shock needs to be the first consideration as well as other causes of shock such as cardiogenic shock, hemorrhagic shock, adrenal insufficiency, and anaphylaxis. Chimeric antigen receptor (CAR) T cells are genetically modified T cells engineered to express a chimeric antigen receptor (CAR) targeting antigen (CD19, CD22) expressed on the surface of, in this case, B cells. Recognition of a specific cell surface antigen activates T cell response independently of MHC recognition. 84 A cytokine release syndrome does occur 3‐14 days following cell infusion. This syndrome can present with dysfunction of virtually any organ: fever and myalgias are common, but distributive shock, ARDS, respiratory failures, renal failure, and hepatic failure can occur. There is interest in treating this syndrome with steroids or tocilzumab (IL 6 blocker) or siltuximab (also IL6 blocker) but these are not standard therapies and could not be on the boards There is no cytokine profile that would allow the clinician to distinguish this syndrome from septic shock even if such a profile were available in real time.

32 year old man with active daily injection drug use presents with a two month history of slowly progressive lower back pain He has no fevers, sweats, or weight loss MRI showed abnormality in the lumbar spine, seen in the image below Blood cultures collected off antibiotics are negative What radiographic feature would be most suggestive that this lesion is infectious rather than traumatic, degenerative, or neoplastic? A. Lumbosacral spine location B. Air in disc space C. Isolated involvement of one vertebrae D. Discitis and endplate irregularities of two contiguous vertebrae

Correct Answer: D. Discitis and endplate irregularities of two contiguous vertebrae See lecture on pyogenic osteomyelitis and septic arthritis. Any joint can be involved with pyogenic osteomyelitis or septic arthritis. Septic arthritis may be polyarticular. - If sternoclavicular joint ‐ think IDU Organisms which cause osteomyelitis or septic arthritis: - Staph aureus, incl. MRSA, most common (skin flora) - Pseudomonas less common now than it once was (today's heroin requires heating to dissolve) - Candida (lemon juice used to dissolve brown heroin, skin flora) - Clostridium (black tar heroin) see soft tissue infection - Eikenella and viridans Streptococci (needle‐licking) Pyogenic Vertebral Osteomyelitis: Radiographic Clues and Pitfalls Pyogenic infection - Crosses two disc spaces - Early discitis and endplate erosions Isolated involvement of one vertebral body less likely to be infection Vacuum phenomenon (air in disc space): - Signifies degenerative disease, not infection Tuberculous / fungal ‐ "atypical" - Relative sparing of the disc space - TB: multi‐level disease, large paraspinal abscess

A 30 year old HIV‐infected gay man (CD4 count 780 cells/mm3 with an undetectable HIV RNA) with no significant past medical history complains of pain and decreased vision in his right eye. He was well until three days prior to presentation when he developed discomfort in his eye and blurry vision. He denied any history of trauma. He had just returned from a 10‐day trip to North Africa and Western Europe one week prior to the onset of symptoms. On examination, he has a maculopapular rash on his trunk and diffuse lymphadenopathy. He is referred to an ophthalmologist and is diagnosed with panuveitis. A CBC, complete metabolic panel, and chest radiograph are unremarkable. He had a negative ppd three months earlier. His RPR is nonreactive. Which of the following tests is most likely to be abnormal? A. Toxoplasma serum IgG B. Cerebrospinal fluid JC virus PCR C. Cerebrospinal fluid TB PCR D. Serum treponemal EIA E. Serum Quantiferon

Correct Answer: D. Serum treponemal EIA This patient had a nonreactive RPR because of a prozone effect. .Diluted serum found a RPR titer of 1:128. EIA is not affected by the prozone. The patient has secondary syphilis with ocular involvement. Panuveiutis is one of the most common ocular manifestations of syphilis‐ particularly, early stage syphilis. Approximately 70% of patients with ocular syphilis will have one or more CSF abnormalities. Of those who have CSF abnormalities, most will have CSF pleocytosis but only about half will have an abnormal CSF VDRL. Among patients with secondary syphilis, nearly 100% should have reactive serum treponemal and non‐ treponemal antibodies (barring a prozone reaction). The history does not provide any clues that he is at risk for TB infection. Uveitis could be caused by TB but is not likely here. Toxoplasmosis could explain the uveitis but not rash or lymphadenopathy JC virus is the cause of JC virus encephalitis, which would be unlikely to cause uveitis and unlikely to cause any disease in a patient with such a high CD4 count. Ocular syphilis is treated the same as CNS syphilis with a 14 day course of intravenous penicillin G.

A 28‐year‐old woman who is 9 days post receipt of allogeneic HSCT for acute myeloid leukemia presents with 2 days of altered mental status. Last night, her nurse witnessed what may have been a self‐limited focal seizure. She is lethargic and confused but complains of headache. She is still severely neutropenic. MRI with FLAIR imaging is shown below. She is from Haiti and has a history of latent TB, which was treated for 9 months prior to transplant with INH. Post‐transplant, she has had slow engraftment, and her current serum creatinine is 3.2. Her blood pressures have been moderately high, ranging from 140-170 systolic. Her current medications include tacrolimus (her last level was within the therapeutic range), and prednisone at 10 mg. She is receiving fluconazole and valacyclovir prophylaxis. Which of the following is the best explanation of her current process? A. Tuberculosis B. HHV‐6 C. Cryptococcosis D. Tacrolimus toxicity E. Polyoma virus

Correct Answer: D. Tacrolimus toxicity - very rapid, 2 days While reactivation TB is possible in a transplant recipient, it is less likely in people who have been treated for latent disease. The radiographic image is very atypical for tuberculosis or cryptococcosis. She is receiving fluconazole, making cryptococcosis even less likely. Neutropenia is not predisposing to either infection. Lesions in the white matter may be seen with polyoma virus and HHV6. Polyoma virus encephalitis (usually JC virus) is much more indolent and would be rare in this setting. HHV6 can cause limbic encephalitis but rarely confined to this cortical distribution or prior to engraftment. the patient has posterior reversible encephalopathy which has many possible causes but can result from tacrolimus neurotoxicity post‐transplant. Tacrolimus levels may or may not be elevated. Diagnosis is made by the clinical presentation and subcortical edema which includes the parietal or occipital cortex, best seen on MRI by FLAIR (fluid‐attenuated inversion recovery) imaging Supportive evidence of that diagnosis includes: timing relative to the tacrolimus infusion, relative hypertension, symptoms, and radiographic findings.

A 60‐year‐old woman with a history of diabetes mellitus presented with a month of anorexia and nausea followed by 1 week of fever and right upper quadrant abdominal pain. She had undergone an endoscopy with biopsy of a gastric ulcer 2 months before. On exam, she was febrile (101°F). She had RUQ abdominal tenderness. WBC count: 12,000. Alkaline phosphatase: 195. An abdominal CT scan showed hypoattenuated lesions in the liver. A gram stain of the aspirate is shown. Aerobic culture is negative The most likely diagnosis is: A. Mycobacterium tuberculosis B. Nocardia C. Streptococcus milleri (anginosus group) D. Actinomyces E. Aspergillus

Correct Answer: D. Actinomyces This patient has hepatic actinomycosis. Direct fluorescent antibody stain and culture of the liver aspirate were positive for Actinomyces israelii. Actinomyces most commonly involves the jaw, the lung and intra‐abdominal sites. Hepatic actinomycosis represents about 5% of cases of actinomycosis and may result either from direct extension from contiguous focus or hematogenous spread. It is usually related to a breach in the gastrointestinal mucosa, although extension from pleuro‐ pulmonary infection is sometimes seen. Other companion organisms may be present. The gram stain of the liver aspirate shows branching, filamentous gram positive rods, a characteristic appearance for Actinomyces. Although Nocardia can have a similar appearance on gram stain, it usually infects immunocompromised hosts; moreover, Nocardia is modified AFB positive whereas Actinomyces is not. nocardia, aerobic and beaded, also with modified AFB. The other organisms on the list (M. tuberculosis, Aspergillus, Streptococcus milleri) would not have the morphologic appearance of branching, filamentous gram positive rods on gram stain.

A previously healthy 60 year old man presented with a few hours of severe pain in the right upper extremity. The exam was normal and he was discharged. Over the next few hours, he developed progressive swelling of the right upper extremity. There was no history of trauma. On exam, he appeared anxious, with cold and clammy skin. BP55/30. The right upper extremity was diffusely swollen with a deep‐red discoloration; there were several bullae (shown). No pulses were palpable in the right upper extremity. WBC 8,900 (47% polys, 38% bands). An X‐ray showed air in the soft tissues The most likely diagnosis is which of the following: A. Vibrio vulnificus B. Group A streptococcal necrotizing fasciitis C. Mixed aerobic/anaerobic necrotizing fasciitis D. Clostridial gas gangrene E. Ecthyma gangrenosa

Correct Answer: D. Clostridial gas gangrene -clue against necrotizing fascitis with anaerobes was the gas on x-ray Discussion: This patient has spontaneous gas gangrene. Surgical cultures grew Clostridium septicum. In retrospect, patient reported a several month history of bright red blood per rectum. Subsequent evaluation revealed an invasive colonic carcinoma. Gas Gangrene (Clostridial myonecrosis) may present with acute onset of severe pain, sometimes without abnormal physical findings. The patient generally appears quite ill. There is subsequent rapid progression of skin discoloration, tense edema, crepitus and development of bullae containing thin, serosanguinous or 44 hemorrhagic fluid. Gram stain of fluid or surgical specimen demonstrates large gram‐positive or gram‐ variable rods. Traumatic gas gangrene is generally due to C. perfringens, and sometimes other Clostridial species. Spontaneous (non‐traumatic) gas gangrene is most commonly due to C. septicum. Predisposing factors for spontaneous gas gangrene include intestinal disease (tumor, ulceration, inflammation) or systemic factors (malignancy, cirrhosis, neutropenia, diabetes mellitus). C. septicum infection is associated with malignancy: In one series, 81% of patients had malignancy and in 37% the cancer was occult. The most common cancers associated with C. septicum are colorectal and hematologic malignancies. Vibrio vulnificus can cause a rapidly progressive soft tissue infection but usually afflicts patients with liver disease, iron overload or an immunocompromising condition. Group Streptococcal necrotizing fasciitis would not result in air in the soft tissues. Mixed aerobic/anaerobic necrotizing fasciitis often occurs after trauma or surgery. Ecthyma gangrenosa would be rare in the absence of profound neutropenia or comparable immunosuppression.

A 59‐year‐old woman with HIV‐1 infection has had multiple courses of antiretroviral therapy all of which failed due to poor adherence. On her current therapy, her genotypic analysis reveals RT: M41L, F77L, K103N, Q151M, M184V Protease: M46I, I50L, V82A, L90M Trofile: X4 virus Which of the following regimens is likely to have the most virologic activity in this circumstance? A. dolutegravir, AZT, 3TC, maraviroc B. dolutegravir, tenofovir, FTC, enfuvirtide, etravirine, atazanavir/ritonavir C. dolutegravir, tenofovir, FTC, etravirine, maraviroc, darunavir/ritonavir D. dolutegravir, tenofovir, FTC, etravirine, darunavir, ritonavir

Correct Answer: D. dolutegravir, tenofovir, FTC, etravirine, darunavir, ritonavir This is a hard question: what can you eliminate? Maraviroc is not an option since there is X4 virus, thus the first and third choices are wrong. I50L alone confers resistance to atzanavir, so the second choice also is wrong. In addition, enfuvirtide, while active here, requires twice‐daily subcutaneous dosing, so is not well tolerated. That only leaves...the final choice. NRTI: Q151M complex has multidrug resistance - tenofovir is the only viable option in the nucleoside class. NNRTI: K103N: efavirenz has no activity; while etravirine has activity. PI: I50L is a signature resistance mutation for atazanavir resistance, but not other protease inhibitors; none of those listed are major darunavir‐associated resistance mutations. Coreceptor Inhibitor: X4 virus‐ maraviroc has no durable activity. Enfuvirtide, darunavir, and dolutegravir all should be active agents here

A 32-year-old woman presents with two days of swelling of her submandibular area bilaterally with low-grade fever. Her temperature is 38.5°C. The area is brawny and board-like. She can open her mouth, though she has mild trismus, and her tongue appears to be pushed up so that the rest of her oral cavity is difficult to visualize. She has not been able to drink or eat for a day, and she reports that breathing through her mouth is difficult. She has no masses or nodes that can be palpated. The rest of her examination is unremarkable. She is pictured below. Her initial WBC= 22,000 (95% neutrophils). Which of the following is the most likely cause of the infection? ActinomycosisDiphtheriaRetropharyngeal abscessDental abscessPeritonsillar abscess

Correct Answer: Dental abscess This is Ludwig's angina, which is caused most often by an abscess of the second or third mandibular molar. The infection is unilateralbut edema rapidly become bilateral, involving the submandibular and sublingual areas, spreads quickly without abscess formation or lymphatic involvement. These patients can be quite toxic and can have complications from involvement of adjacent structures, including airway compromise. Surgical drainage of the abscess or intubation is often not necessary if broad spectrum antibiotics are started promptly. Diphtheria can cause submandibular edema but not without pharyngeal exudates. Peritonsillar and retorpharyngeal abscess do not cause edema in this space. Actinomycosis is indolent and more localized in the submandibular area. Correct Response

A 47-year-old man with recurrent episodes of bronchitis is referred for possible immune defect. Recently he has had more bronchitis exacerbations as well as an episode of documented bacterial pneumonia and sinusitis. His primary care physician obtained immunoglobulin levels which showed: IgG 200 (normal 523-1482), IgA 412 (normal 51-375), and IgM 165 (normal 37-200). Which one of the following tests is most likely to solidify the diagnosis and thereby facilitate specific therapy? Draw IgG subclasses. If the subclasses are low (IgG1 and 3 or IgG 2 and 4), consider immunoglobulin therapy. Repeat the IgG levels. If they remain low, the diagnosis of hypogammaglobulinemia is established and immunoglobulin replacement should be initiated. Apply skin tests for candida, mumps, tetanus, and trichophyton. If anergic, draw specific antibody levels against tetanus and pneumococcus. If low, consider immunoglobulin therapy. Draw specific antibody levels against tetanus and pneumococcus, immunize, and repeat titers at 2 weeks. If response is low, initiate immunoglobulin therapy. Order a test for serum alpha 1-antitrypsin levels; if low administer immunoglobulin therapy.

Correct Answer: Draw specific antibody levels against tetanus and pneumococcus, immunize, and repeat titers at 2 weeks. If response is low, initiate immunoglobulin therapy. The clinical syndrome of recurrent bacterial respiratory infections and low IgG levels is consistent with common variable immunodeficiency (CVID), but is also seen frequently in smokers and those with other forms of bronchial damage. The low serum IgG concentration dictates the need for further evaluation. The most appropriate test is measurement of specific antibody titers before and after immunization. Even if the repeat IgG concentrations are in the (low) normal range, the important question is his ability to produce antibody on appropriate antigen stimulation. Random titers do not predict the ability to respond to antigen now. In the event that he is already on IgG replacement before you are asked to evaluate his antibody production, you would have two choices. You could take him off replacement and then recheck. As an alternative, you could use a novel antigen, like keyhole limpet hemocyanin or the bacteriophage 174. Low IgG subclasses per se are not a sufficient reason to recommend prophylactic immunoglobulin therapy. Serum alpha 1-antitrypsin deficiency has been linked to emphysema but not repeated respiratory infections. Immunoglobulin therapy would have no role in patients with serum alpha 1-antitrypsin deficiency.

A 66‐year‐old male comes to the Emergency Room with severe nausea and vomiting for 6 hours with diarrhea for 2 hours. He has been in relatively good health but has adult onset diabetes, mild heart failure, and hypercholesterolemia. He has no history of prior GI problems and no one else in his family is ill, including the two toddlers that his wife cares for in his home. On physical examination, he is febrile to 38.3°C, BP 150/80, RR 20, P 100. He has a mildly tender abdomen with no rebound. Laboratory: WBC 11,000 (60% polys), Hg 13 g/dl, Plat 220,000 Chemistry Profile: Na 145 meq/L, K 3.8 meq/L, CO2 22 meq/L, BUN 20 mg/dl, Creat 2.0 mg/dl Lactate 0.9 mmol/L Stool cultures sent: rotavirus screen negative The ER starts vancomycin and ciprofloxacin plus metronidazole and calls you for a consultation. What is the appropriate response to the finding of pneumatosis intestinalis in this patient? A. Immediate laparotomy B. Change antibiotic regimen to meropenem and vancomycin C. Add caspofungin to ciprofloxacin and metronidazole D. Colonoscopy E. Observation of clinical course: no change in current management

Correct Answer: E. Observation of clinical course: no change in current management The pneumatosis intestinalis refers to gas within the wall of the small or large bowel and has also been referred to as pneumotosis coli, intestinal emphysema, and pneumatosis cystoides. This is a radiologic sign and should not by itself alter the plan developed on the basis of history, physical examination, and laboratory markers. The condition can be managed medically unless there are clinical features suggesting an acute abdominal emergency such as bowel obstruction, perforation or peritonitis. Thus, this patient could have food poisoning, a gastrointestinal virus such as Norovirus, a bacterial pathogen such as Shigella, or a host of other infectious and non‐infectious syndromes. At this point, no data are given suggesting that more aggressive diagnosis or therapy is necessary.

A 15 year old high-school student develops Shiga toxin producing E. coli infection and has sudden onset of rising serum creatinine and very low platelet count. The diagnosis is atypical hemolytic uremic syndrome (HUS). Which one of the following is known to improve renal function in HUS patients? Rituximab Plasmapheresis Eculizumab IV fluoroquinolones High dose corticosteroids

Correct Answer: Eculizumab The treatment of choice of STEC HUS is supportive and use of dialysis if the renal failure is severe enough. No data has shown rituximab to be effective. Plasmapheresis is not used nor is it helpful unless the condition is TTP not HUS. Eculizumab is a terminal complement inhibitor shown effective in two placebo control trials in improving the renal function in HUS The major problem is that eculizumab is extremely expensive. Fluoroquinolones are reported to mobilize a phage that upregulates Shiga toxin production and may worsen HUS. Corticosteroids are of no value in the treatment of HUS. Correct Response Eculizumab

A 33-year-old man with advanced HIV infection (most recent CD4 =50 cells/uL, VL =500,000 copies/uL) comes to the emergency room complaining of "the worst sore throat of my life." He is hoarse but has no coughing. He is seen intermittently in the HIV clinic but refuses to take antiretroviral therapy or Pneumocystis prophylaxis claiming that "those medicines" killed a friend of his. His temperature is 102.6F. He looks flushed; his heart rate is 120. His oral exam shows thrush on his tongue and buccal mucosa, but his throat appears normal. There is no cervical adenopathy. His wbc is 14.7 cells/uL and his monospot test and a rapid test for Group A Strep are negative. He is given oral fluconazole and discharged with clinic follow-up. He returns to the emergency room three days later saying in an extremely hoarse voice that his sore throat is worse and he is having pain on swallowing. His temperature is 102.2. His thrush appears slightly improved; again the pharynx appears normal. He is observed spitting out his saliva into a tissue because swallowing is so painful. Which one of the following is the most likely cause of his sore throat? Peritonsillar abscess Retropharyngeal abscess Esophagitis EpiglottitisLudwig's angina

Correct Answer: Epiglottitis Epiglottitis should be considered in any patient complaining of "the worst sore throat of my life" who has a normal pharyngeal exam. The consideration of epiglottitis obviously merits severe caution in examining the airway, and an urgent need to assess the degree of airway compromise, and the likelihood that airway compromise will progress. A lateral xray of the neck can be useful ("thumb sign"). Waiting for a CT may be too time consuming and the trip to CT with an unsecured airway can be dangerous if the patient in fact has epiglottitis. Attempts to visualize the epiglottis with a tongue depressor can lead to sudden, complete obstruction of the airway, particularly in children. An emergent ENT consultation for endoscopic visualization of the epiglottis is essential. The sore throat with normal initial examination is a key here to suspecting epiglottitis. A Peritonsillar abscess would be seen by bulging of the tonsillar and would not cause hoarseness, nor would a retropharyngeal abscess or esophagitis cause hoarseness. Ludwig's angina is identified by the prominent submandibular swelling. This infection typically arises from an infected second or third mandibular molar tooth. Ludwigs angina is an aggressive, rapidly spreading cellulitis that is different from Group A strep in that there is no lymphadenopathy or pharyngitis. Ludwigs does have potential for airway obstruction and requires careful monitoring and rapid intervention for prevention of asphyxia and aspiration pneumonia. Epiglottitis is essentially a cellulitis of the epiglottis and surrounding tissues. Since the adult airway is much larger than infants and small children, the severe manifestations of airway compromise, drooling, and stridor are less common, although airway compromise clearly occurs and must be carefully assessed and managed. Adults typically are not as toxic looking as children and less likely to be bacteremic. There is a long list of likely bacterial and viral causes of epiglottitis. Despite widespread immunization, Haemophilus influenza group B still seems to be a frequent cause in immunized and unimmunized persons, although clearly the total number of adult and pediatric cases has fallen dramatically since Haemophilus type B immunizations were instituted. Unless the patient is bacteremic it is difficult to decide what the causative organism is, and adults are usually NOT bacteremic

A 23-year-old previously healthy (HIV negative) male presents with a two day history of a progressively red and painful right eye and low grade fever. His vision is slightly blurred. He has no history of local trauma or recent surgery, and is aware of no other recent illness. On physical examination he is febrile to 38.3°C, he has a moderate ophthalmoplegia involving cranial nerves III, IV, and VI. His pupil reacts sluggishly to light. There is marked chemosis, periocular edema and proptosis. His laboratory examination is normal except for WBC = 17000 (90% neutrophils). The most likely process which led to this ocular presentation is: Choices A Facial cellulitis B Pseudomonas conjunctivitis C Hematogenous seeding of his globe D Ethmoidal sinusitis E Herpetic keratitis

Correct Answer: Ethmoidal sinusitis This patient has orbital cellulitis. It is very important to distinguish orbital cellulitis from preseptal cellulitis. Preseptal cellulitis involves the skin and soft tissue outside the orbit. In the diagram below preseptal cellulitis is defined as involvement external to the ocular septum. It arises from the sinus or from facial cellulitis. Because the orbit itself is not involved, there is no pain on eye movement, no loss of vision, and no extraocular paresis, which this patient clearly had. Preseptal cellulitis is caused by the flora of the sinuses or skin and can be treated as an outpatient. A CT scan may be useful to be certain there is no orbital involvement. Facial cellulitis can lead to cavernous sinus thrombosis particularly with Staphylooccus aureus. Sinusitis is a more frequent source of cavernous sinus thrombosis than facial lesions and this patient may also have cavernous sinus thrombosis, but this cannot explain all the findings. For orbital cellulitis, the orbit itself is involved, usually due to extension of ethmoidal sinusitis across the thin bony septum separating it from the orbit. For orbital cellulitis there is pain on eye movement, and there may be thrombosis of ocular veins or arteries leading to catastrophic vision loss. Many adults with this syndrome have neither fever nor leukocytosis. After delineating the anatomy with a CT scan, these patients require IV antibiotics and in some cases surgical intervention to drain the sinus and sometimes the orbit if there is an abscess present. Bacterial conjunctivitis including cases caused by Pseudomonas or MRSA causes itching and a scratchy sensation over the eye, mild pain, and purulence, but there would be no neurologic involvement, and no fever or leukocytosis. Pain is unusual and would suggest another diagnosis, or perhaps a viral etiology involving the mucosa of the eye lid. Herpetic keratitis presents with tearing and photophobia, but there should be no paresis of extraocular muscles and minimal inflammation of the surrounding soft tissues. There should be no leukocytosis Herpetic kertatitis can be primary or recurrent. Regarding answer C, hematogenous seeding of the globe (endophthalmitis) is unusual in immunocompetent patients. Patients may have a source such as endocarditis or IV drug abuse. They typically have an acute onset of eye pain and decreased vision, and have prominent vitritis and hypopyon. Herpetic keratitits should not be associated with ophthalmoplegia or proptosis. There is chemosis, watery discharge, and blurred vision with conjunctival injection adjacent to the limbus. There are characteristic dendritic lesions on the cornea. This figure shows the relationship of the orbital septum. Syndromes involving periorbital cellulitis are often characterized as pre-septal (anterior to the septum) or post-septal (posterior to the septum). You can see the anatomic difference.

A patient with HIV (CD4 = 27, VL = 70k) comes to your office complaining of cloudy vision in his right eye. He has taken his antiretroviral agents and TMP-SMX only intermittently. He had dermatomal zoster on the right side of his face two months previously. He has a history of recurrent genital herpes simplex, with no current lesions. You refer the patient to an ophthalmologist who reports round retinal lesions which resemble progressive outer retinal necrosis (PORN). The ophthalmologist offered to do a vitreal aspirate to try to confirm the diagnosis but the patient refused. The empirical treatment most likely to stop progression of the disease is which of the following: Ganciclovir intravenously alone Ganciclovir ocular implant alone Foscarnet intravenously alone Ganciclovir plus foscarnet, both IV plus intravitreal ganciclovir Ganciclovir: both IV and implant

Correct Answer: Ganciclovir plus foscarnet, both IV plus intravitreal ganciclovir While CMV is by far the most common cause of retinal disease in HIV infected patients with low CD4 counts, you should be aware of syndromes cause by HSV and VZV. Progressive outer retinal necrosis (PORN) typically occurs in patients with very low CD4 counts, and can be distinguished from CMV by its distribution and its retinal appearance if an experienced ophthalmologist is available. A vitral tap may reveal VZV by PCR. Prognosis is very poor. While there are no good controlled trials, an aggressive approach with both ganciclovir and foscarnet IV, and probably intravitreal ganciclovir, is recommended. Be aware that the ganciclovir implant cannot be the correct answer because that device is no longer marketed in the US. Distinguish PORN from acute retinal necrosis, a VZV related retinal process that occurs in patients with higher CD4 counts which has a better prognosis.

26 year old HIV+ man on his first ART regimen, tenofovir (TDF)/emtricitabine + raltegravir, for 2 years. HIV RNA originally 203,000 copies/ml, then decreased to <50 copies/ml by 4 months. On his most recent routine lab tests, HIV RNA was 13,900 copies/ml, repeated 2 weeks later after adherence counseling at 11,400 copies/ml. What lab test(s) would you now order? Drug level testing Genotype testing (reverse transcriptase/protease and integrase) Phenotype testing (reverse transcriptase/protease and integrase) Genotype and phenotype testing (reverse transcriptase/protease and integrase) CCR5 tropism testing

Correct Answer: Genotype testing (reverse transcriptase/protease and integrase) This patient is experiencing virologic failure (likely due to adherence issues) on a nucleoside reverse transcriptase inhibitor- and integrase inhibitor-containing regimen and should have drug resistance testing done. For first regimen failure, genotypic testing (alone) is the recommended test. For multiregimen failure, both genotypic testing (reverse transcriptase/protease and integrase) AND phenotypic testing (reverse transcriptase/protease) is recommended. Drug level testing and phenotypic testing of integrase inhibitors are not routinely recommended. CCR5 tropism testing is only recommended if maraviroc is being considered as a treatment option. Correct Response Genotype testing (reverse transcriptase/protease and integrase)

A 32-year-old man presents with a 3-day history of right upper quadrant abdominal pain. He had been in his usual state of good health until 3 months prior to admission when he noted that his bowel movements were becoming loose (1-2 episodes per day) with associated abdominal cramping. Over the next few months, he experienced extreme fatigue, and a 25-pound weight loss. He reports that the ingestion of milk seemed to worsen his GI symptoms. His past medical history is significant for childhood asthma and a diagnosis of syphilis treated two years earlier. He is sexually active with two male partners and he uses condoms inconsistently. He is on PrEP. In the previous year, he traveled to India, Arizona, California, Cuba, and Switzerland. He is an avid outdoorsman and has participated in several triathlons. On examination, he is a thin man who appears fatigued. His vital signs were normal. His sclerae were anicteric. Examination of his oral mucosae revealed several apthous ulcers. Abdominal examination demonstrated right upper quadrant tenderness but no rebound or guarding. His bowel sounds were normal. Digital rectal examination was unremarkable yielding a small amount of brown stool that was hemoccult negative. He had a normal CBC. He had a normal serum creatinine, a mildly elevated total bilirubin (1.5 mg/dl), an elevated alkaline phosphatase (150 U/L), and low albumin (3.0 gm/dl). A right upper quadrant ultrasound revealed a thickened gallbladder wall. A lactose breath hydrogen test was diagnostic of lactose malabsorption. What is best next diagnostic test: Small intestinal biopsy Giardia stool antigen String test Stool acid fast smear .Helicobacter pylori stool antigen

Correct Answer: Giardia stool antigen This patient is likely to have chronic giardiasis because of the duration of symptoms, low albumin and weight loss. . Both his sexual behavior and his travel increase the risk of ingesting Giardia cysts. Up to 40% of patients with chronic giardia infection can develop lactose intolerance. Cholangiopathy is an uncommon but well described complication of giardiasis, though mostly patients with an underlying immune disorder. This patient's loose stools, weight loss, abdominal cramps and low albumin could be celiac sprue or Whipple's disease , but that would not explain his bilirubin and alkaline phosphatase elevation. Diagnosis of sprue or Whipple's disease would require intestinal biopsy. Acid fast smear would detect cystoisosporiasis, cryptosporidiosis and cyclosporiasis . These illnesses can last weeks and be associated with weight loss but chronic diarrhea in a normal host would be uncommon. Helicobacter pylori infection does not present as chronic diarrhea. The simplest, mos useful test for giardiasis is the stool antigen test. Small intestinal biopsy or stool PCR can be diagnostic of giardiasis but is rarely required. Three fresh stool sent for O&P are the most sensitive test but requires an experienced microscopists and was not offered as a choice.

A 29-year-old female developed her first bout of meningitis at age 24. Since then, she has had 3 other episodes. Each is associated with headache, stiff neck, photophobia, and occasional radicular pain down her right lower extremity. Each has been treated with appropriate doses of antibiotics and resolved over several days. Her last CSF results at the time of an acute episode revealed a WBC of 230/mm3(92% lymphs with some large and atypical appearing cells), normal glucose, and protein of 110 mg/dL. All routine bacterial cultures and Gram stains have been negative. The patient has been well otherwise but has a history of both oral and genital herpes simplex virus infection, and a remote history of treated syphilis. She has refused permission for an HIV test in the past. This young woman's recurrent meningitis is likely due to: HSV 2VZVCMVSyphilisHIV

Correct Answer: HSV 2 This is a typical presentation of recurrent benign lymphocytic meningitis - referred to as Mollaret's recurrent meningitis. Mollaret's meningitis is a type of benign recurrent aseptic (i.e., nonbacterial) meningitis that is most often due to herpes simplex virus type 2 (HSV-2) infection. Genital lesions are usually absent at the time of presentation. None of the other choices is known to cause recurrent meningitis over years.

A 42-year-old man is referred for asymptomatic elevation of his liver function tests. He underwent a living-related donor kidney transplantation 14 months earlier secondary to end-stage renal disease from uncontrolled hypertension (CMV D-/R-). Six months after his transplant, his physicians noted an asymptomatic increase in aminotransferases, with aspartate aminotransferase (AST) 8 times the upper limit of normal (ULN), alanine aminotransferase (ALT) 6 x ULN, and gamma glutamyl transferase (GGT) 5 x ULN. His total bilirubin was mildly elevated and his alkaline phosphatase was normal. Hepatitis A virus, Hepatitis B virus (HBV), hepatitis C virus (HCV), human immunodeficiency virus (HIV)- 1, Epstein-Barr virus, herpes simplex virus 1 and 2, and cytomegalovirus serologies were negative. DNA HBV and RNA HCV were undetectable. Liver autoimmunity panel was negative. Abdominal ultrasound was normal. He denied alcohol consumption . He recently returned from living the past year in Germany and is an avid consumer of sausage. His immunosuppressive regimen included tacrolimus, mycophenolate mofetil, and prednisolone. His liver function tests have continued to be elevated over the past 9 months despite changes in his immunosuppressive regimen and antihypertensive medications. His physical examination was unremarkable. His BMI was 20 kg/m2. No scleral icterus was noted and no stigmata of cirrhosis were noted. A liver biopsy demonstrated lobular hepatitis without fibrosis. Which of the following entities is most likely responsible for his hepatitis? Coxiella burnetii Hepatitis D Hepatitis E Leptospira interrogans Non-alcoholic hepatosteatosis

Correct Answer: Hepatitis E The most likely etiology for his persistently elevated liver function tests is chronic hepatitis E infection. The most likely etiology for his persistently elevated liver function tests is chronic hepatitis E infection. The four genotypes of hepatitis E vary significantly, though knowledge is explanding rapidly. Genotype 3 hepatitis E largely has been observed in Europe, where it is endemic in swine. Although illness is self limited in previously health patients, some immunosuppressed patients, largely solid organ transplants to date, have become chronically infect, with hepatic function abnormalities and persistent viremia. Oral ribavirin has been reported to reduce viremia. Eating undercooked pork has been identified as a risk factor for infection by genotype 3 HEV. Genotypes 1 and 2 HEV are reported from Southeast Asia, where they are transmitted by the fecal oral route and cause acute hepatitis. Fatal infections have been reported in pregnant women. The patient is not infected with Hepatitis B so cannot be infected with Hepatitis D. While Coxiella may cause hepatitis, the patient has no other manifestations of Q fever and the liver biopsy did not reveal any granulomas. He has no other manifestations to suggest Weil's disease due to Leptospira. The biopsy did not reveal evidence of hepatic steatosis

A 23-year-old student in St Louis has returned in January from a trip to the Far East the prior September to November and now presents with fever and confusion. The patient has complained of temperature elevations for several days, and a headache that has become progressively worse. On the fourth day of illness, when his friends visited his apartment and found him to be disoriented and belligerent, they brought him to the emergency room. The student was part of an academic program that visited Southern China for 3 months, teaching English in rural and urban areas. He left China 6 weeks ago, spent a week in Tokyo and then visited his parents in San Francisco. On physical exam, the patient has a fever to 39°C, but no rash or bites are noted. He reacted only to pain and not verbal stimuli. Routine labs were normal, including CBC. MRI scan showed focal enhancement in the right medial temporal lobe. Lumbar puncture: opening pressure normal; CSF WBC: 400 cells (98% lymphocytes) with normal glucose, protein 110 mg/dL. PCR of CSF for various pathogens is pending. Which of the following viruses would be the most likely cause of this disease? Japanese encephalitis virus (JEV) St. Louis virus West Nile virus California virus Herpes simplex virus

Correct Answer: Herpes simplex virus The clinical picture and MRI are strongly suggestive of HSV encephalitis. Thus E is the correct answer. Even for the exam, common diseases should occur commonly unless there is a compelling reason to suspect a less common diagnosis. The PCR for HSV should be positive in almost all cases within the first 24 hours of presentation: false negatives may occur if the lab is using an insensitive technique, or occasionally very early in the clinical course. This patient traveled in rural areas of China in which Japanese encephalitis virus infection is hyperendemic, with infections particularly common in November through March. The usual incubation period is 4-21 days, making ten weeks beyond the range. Always consider the potential incubation period for travel-related diseases. Despite the name, Japanese encephalitis is now rare in Japan. Mosquitoes, the vectors of Japanese B encephalitis, would be rare in the chilly Tokyo winter and a high percent of the population in Japan has been vaccinated. Also, the MRI of Japanese, St Louis and West Nile virus encephalitis, all flaviviruses, shows lesions in the basal ganglia, cerebellum, midbrain, pons and spinal cord. West Nile, California and St. Louis virus encephalitis are mosquito borne illnesses that are very uncommon in the Northern hemisphere during midwinter.

A 23-year-old female was admitted for severe abdominal pain and fulminant hepatic failure. She had been followed in hematology clinic after an allogeneic stem cell transplant for acute lymphocytic leukemia done 6 months prior. Repeated episodes of graft versus host disease of the gut had prompted periods of high-dose corticosteroid therapy with the current prednisone dose being 40 mg per day. On admission, the patient was in acute distress with midepigastric pain, prostration and fever of 39°C. Bowel sounds were normal. The abdomen was distended and without rebound tenderness, though with some guarding. A few small vesicles were noted on the anterior abdomen but no other rash was noted. Blood chemistries showed an AST, 5600 U/L; ALT, 3630 U/L; total bilirubin, 2.2 mg/dL, with a direct of 1.9 mg/dL; alkaline phosphatase, 276 mU/mL; and partial thromboplastin time (PTT), 44 sec. Amylase and lipase were normal. Prior to transplantation she was known to be CMV seropositive and HSV seronegative. Whole blood PCR for CMV was nondetectable. IgM antibody to hepatitis A, hepatitis B core antigen and hepatitis C were negative. HBV serum PCR is also negative. She was born in Guatetmala but had lived in the United States for five years. The most likely diagnosis is which of the following? Graft versus host disease of the liver Herpes simplex virus hepatitis Disseminated BK virus Hepatitis B Disseminated tuberculosis

Correct Answer: Herpes simplex virus hepatitis Herpes simplex hepatitis should be considered when there is marked transaminase elevations in a patient who is either pregnant or immunosuppressed. Onset is sudden with fever, abdominal pain and profound rises in aminotransferases. There are usually no active signs of herpes simplex in the oral mucosa, vagina or skin. There may be a role for measuring blood HSV viral load but diagnosis is often by liver biopsy, liver tissue at transplantation or autopsy. The dramatic elevations of ALT and AST in this case are incompatible with the other diagnoses listed except hepatitis B but that disease would be rare in someone with a negative antibody to hepatitis B core antigen and negative HBV PCR. Adenovirus can cause fulminant hepatitis in HSCT patients but is not offered as a choice here.

This 9-month-old infant had the sudden onset of irritability and fever ranging up to 104°F. On the second day of illness, the child was examined by a physician and found to still be febrile to 103°F but not to appear critically ill. There were a few palpable anterior cervical nodes. The oropharynx appeared clear. Three days later, the mother returned with the child because, while the child no longer had a fever, an erythematous rash developed on his trunk, face and extremities. The child appeared much less ill. The infant did not attempt to scratch the lesions. The oropharynx was clear. A WBC showed a modest leukopenia and a few atypical lymphocytes. Of the following entities, which would best explain this illness? Human herpes virus 6Epstein barr virusCytomegalovirusToxoplasmosisHuman herpes virus 8

Correct Answer: Human herpes virus 6 While the ID boards do not test on pediatrics, you should probably know at least how HHV 6 presents in normal humans, since these children can transmit HHV to adults, and since there are more cases being recognized in abnormal adults, ie immunocompromised adults, who presumably reactivate latent HHV 6 which they acquired during childhood or acquire primary infection as adults. Herpes virus 6 causes roseola infantum, also called sixth disease or exanthema subitum. This is an acute viral disease of very young children, characterized by the acute onset of fever, followed in about three days by defervescense and appearance of rash. Although herpesvirus 6 has been divided into 6A and 6B, most cases of roseola are due to 6B. EBV, CMV and toxoplasmosis can cause an infectious mononucleosis syndrome, with acute onset of fever and atypical lymphocytes, usually accompanied by lymphadenopathy but typically without rash. Onset of rash with drop in fever should suggest roseola. Acquired CMV or EBV would present with a mono like syndrome. Rash could be part of the syndrome but would be unusual. Similarly, HHV 8, the etiologic agent of Kaposi sarcoma, can be acquired in childhood and can be associated with fever and rash, but this biphasic illness would be atypical.

A 23-year-old woman with uknown HIV status (test is pending) is concerned about her risk of developing HPV related cervical cancer after her first ever sexual encounters last month which involved unprotected vaginal intercourse with several male friends. Which of the following is correct? If she had been vaccinated with one of the commercially available HPV vaccines before her first ever sexual encounters, she is protected against all HPV serotypes that cause cervical cancer and thus is at no risk for HPV associated cervical cancer. If she acquires HPV serotype 16 or 18, she will almost certainly develop a malignant lesion (>85%). If she acquires HPV by vaginal sexual intercourse, the only HPV related tumor she would be at risk for would be cervical squamous cell cancer. If this patient were HIV infected with CD4 count = 20 cells/ul, HPV vaccine would be safe to administer although its efficacy might be compromised somewhat. Both commercial vaccines, Cervarix and Gardasil, protect against genital warts

Correct Answer: If this patient were HIV infected with CD4 count = 20 cells/ul, HPV vaccine would be safe to administer although its efficacy might be compromised somewhat. HPV is transmitted primarily be penetrative sexual intercourse. None of the commercially available vaccines protect against all HPV strains, Most infections with HPV are self limiting, or become latent and undetectable, even in patients with HIV infection. Thus B and C are incorrect Not all women infected with serotype 16 or 18 develop cancer. Most of the infections are self-limiting, and in most patients cancer never develops. Cervical and vulvar squamous cell cancer are associated with at least 12 serotypes of HPV. While serotypes 16 is responsible for 50% of cases, and serotype 18 for another 10-15%, many other serotypes can be associated with malignancies. The new Gardasil 9 includes more types associated with cervical and vulvar cancers and dysplasias: 6, 11, 16, 18, 31, 33, 45, 52, and 58: HPV is associated predominantly with cervical squamous cell cancer but vaginal and vulva carcinomas and cervical adenocarcinoma are associated as well. Rectal cancer occurs in males and females, and penile occurs in males. HPV is not a live virus vaccine: it is safe to give immunosuppressed patients, although its effectiveness may be diminished with more advanced immunosuppression. Cervarix, unlike Gardasil, does not protect against genital warts and is not approved for males.

A 19-year-old female previously healthy college student, presented with one week of cough and mild dyspnea. Three months prior she returned from a 3-month school trip to India. In India she worked with a charity organization caring for patients in a Calcutta hospital. Prior to the trip and immediately upon return she was PPD negative. She lives with 5 roommates in a small house on campus. She is afebrile. Physical examination is normal, including lung exam. PPD placed by primary physician 2 days prior shows 20 mm induration. Routine blood work is normal. HIV serology is negative. Induced sputum is sent for smear, bacterial, fungal and AFB cultures. AFB smear is negative. Two more induced sputums for mycobacteria are requested. Her chest x-ray is shown. Which of the following is true regarding this patient's pulmonary infection. Because of concern about MDR TB in India, therapy should be held pending the results of susceptibility tests. In the absence of cavitary disease, treatment can be begun with three times weekly directly observed therapy with a standard regimen If untreated, her youth makes her more prone to progress to cavitary disease. Because AFB smear is negative, she can resume her college classes immediately. Because the PPD skin test is positive, isoniazid plus vitamin B6 should be given while awaiting results of mycobacterial cultures.

Correct Answer: If untreated, her youth makes her more prone to progress to cavitary disease. She currently has a left upper lobe infiltrate but no cavity. Primary progression to cavity is more likely in adolescent/ or young adult. 23% if 15-19 years old 13% if 20-24 years old 4% if 25-29 years old 2% if >= 30 years old Treatment for active TB diseaseis not postponed in patients with previously untreated tuberculosis, though additional drugs may be added when there is strong suspicion of MDR tuberculosis. Treatment for active TB disease is always begun with two months of daily therapy. She is infectious and should not return to class until at least two weeks of therapy have been completed. A four drug regimen should be begun for treatment for active TB. In individuals with a positive PPD skin test (and/or positive IGRA), treatment for latent TB infection should NOT be started if active TB is suspected, since treatment of active TB with a single drug or otherwise inadequate regimen can engender resistance to that drug(s).

A 60 year old male with diabetes presents with 6 hours of right upper quadrant pain, low grade fever, and nausea. On physical examination he has tachycardia (p=120 bpm), he is febrile (T=101C) and he has right upper quadrant guarding. His laboratory values include WBV 16000/mm3 with 90% neutrophils and 5% bands Bilirubin 3.6 mg/dl (Direct 2.0, indirect 1.6) ALT 60 units/L, AST 150 units/L Alkaline phosphatase 180 units/L A CT scan of the abdomen is shown below and reveals air in the gall bladder wall The best approach to this patient is Medical therapy alone including piperacillin-tazobactam for the initial 72 hrs Medical therapy alone including piperacillin-tazobactam plus daptomycin for the initial 72 hours Observation alone for the initial 1-3 days Immediate surgical approach without antibiotics Immediate surgical approach plus piperacillin-tazobactam

Correct Answer: Immediate surgical approach plus piperacillin-tazobactam This patient has emphysematous acalculous cholecystitis. This patient has no evidence for a gall stone, and thus has emphysematous, acalculous cholecystitis. About 10% of all cholecystitis does not involve a stone. Acalculous cholecystitis is most common in critically ill, hospitalized patients, but can occur in outpatients as well, as in this case. Creptitus in the right upper quadrant can be a clue to the complication that this is emphysematous disease, but is recognized in a minority of patients. While this patient had a CT scan quickly, ultrasound is often the first diagnostic test available, and demonstrates a thickened gall bladder wall (>3.5 mm). In stable patients in whom the diagnosis is unclear after ultrasonography, a technetium scan using iminodiacetic acid (HIDA) can be useful. Intravenous HIDA is taken up by hepatocytes, and excreted into bile with concentration in the gallbladder. Failure to opacify the gallbladder suggests cholecytitis, but the sensitivity of the test is not high. The treatment of acalculous cholecystitis should be the prompt initiation of antibiotics and either cholecystectomy or cholecystostomy tube. Because it is less invasive, cholecystostomy is often preferred . Acalculous cholecystitis in an outpatient is usually due to enteric flora and thus in this patient piperacillin tazobactam would hve been a reasonable choice. If the patient has been hospitalized, a wide variety of gram positive (staph aureus and enterococci) and gram negative organism (enteric bacilli, Pseudomonas) can be involved. In the latter case, a more complex antibiotic regimen would be appropriate. Expeditious treatment is imperative because without it, gallbladder gangrene may develop and can result in gallbladder perforation. Moreover, this patient has air in his gall bladder, ie emphysematous cholecystitis. Emphysematous cholecystitis is a surgical emergerncy: this acute infection of the gallbladder wall caused by gas-forming organisms, most commonly , Clostridium or Escherichia coli. The mildly elevated indirect bilirubin should at least raise the possibility of Clostridium induced hemolysis.

One of your long time patients, a healthy 50-year-old woman never immunized for hepatitis B, was cleaning her son's bedroom when she was stuck by a blood tinged needle she found in a jar under his bed. Her son has been in and out of jail for drug abuse, and is known to have chronic hepatitis B (HbsAg positive) that has not been treated. You test the patient on the day of the injury: you send off blood for liver function tests and HBV serologies. Her prior liver function tests have been normal. On the assumption that this patient is HBV seronegative, and never vaccinated, which of the following would be the best recommendation for prevention of HBV? Immunize her for HBV as the only interventionImmunize her for HBV and provide HBIGImmunize, provide HBIG, and treat her with entecavir plus tenofovirfor at least 4 weekProvide HBIG aloneProvide Entecavir plus tenofovir for at least 4 weeks and no other intervention Submit

Correct Answer: Immunize her for HBV and provide HBIG This patient should receive HBIG and a primary series of vaccine injected into a different site. This is recommended for all non-immune persons exposed to HBsAg positive person by percutaneous, sexual, ocular or mucous membrane exposure including human bites that penetrate the skin. For this patient, one or two doses of HBIG given a month apart are appropriate if this can be begun in the first 24 hours (preferably within 12 hours of injury, accompanied by the first dose of vaccine in a different site, followed by the remaining doses of a vaccine series (one month and six months later). There would be no harm in giving this patient HBIG while awaiting the serologies,, though the considerable expense would be wasted were she already infected. Treatment with drugs active against HBV might be plausible, but is not currently recommended.

A 40-year-old woman recently visiting her daughter in New Orleans from Taiwan developed fever, epigastric pain, and nausea. One week later, she was brought to the emergency department because of confusion and fever. On examination, her temperature was 101°F. She had right upper quadrant abdominal tenderness. An abdominal CT scan showed a 10 cm hypoattenuated liver lesion. An aspirate was performed; the culture showed mucoid colonies of an aerobic, Gram-negative rod with a positive string test. Her daughter likes to serve raw oysters and has a cat. The most likely organism is: Vibrio parahaemolyticus Aeromonas hydrophila Fusobacterium necrophorum Pasteurella multocida Klebsiella pneumoniae

Correct Answer: Klebsiella pneumoniae Discussion: The only organism in the list to give a mucoid colony is K. pneumoniae. In addition, a hypermucoid strain of Klebsiellahas been associated with a distinctive clinical syndrome in Taiwan and some other parts of Southeast Asia that includes primary liver abscess, bacteremia, and metastatic infection. Risk factors for this infection include diabetes and Asian ancestry. Klebsiellacolonies can exhibit extreme "stickiness" on agar plates ("hypermucoviscosity phenotype"), and the string test is positive (i.e. a "string" of more than 5 millimeters is formed when one touches the colony). The other organisms on this list would not be expected tohave a hypermucoid phenotype or a positive string test; Fusobacterium necrophorum is an obligate anaerobe. . Vibrio parahaemolyticushas caused outbreaks of gastroenteritis from raw oysters but would be a rare cause of liver abscess. Aeromonas can cause diarrhea but would rarely be the sole organism in a liver abscess. Pateurella multocidacan cause sepsis after a cat bite but liver abscess would be rare.

Several children in your community have been reported to have bloody diarrhea followed by hemolytic uremia syndrome. With the likelihood that this is a foodborne outbreak, which variety of food is most likely to be responsible? Leafy greens Dairy products Fruits and nuts Poultry Unpasteurized apple juice

Correct Answer: Leafy Greens The children are likely to have acquired HUS (hemolytic uremic syndrome) by infection with a strain of Shiga toxin producing E. coli. Most infections due to E coli O157:H7 are associated with bloody diarrhea. Non-)157 STEC strains are less likely bloody but cause HUS with the same frequency. Fever may not be present in either. The presence of multiple cases and occurrence of HUS are highly suggestive of STEC. With improvements in the handling of poultry and general knowledge of the need to cook poultry well have led to a reduced risk of STEC infection by poultry consumers. Currently leafy greens are the most important vehicles of transmission of STEC where run off of excreta from cattle and wild animals enters crops with STEC strains internalized in the root system of the plant, leading to contamination of most leafs of a head, making it difficult to clean. Dairy products and contaminated unpasteurized fruit juices are uncommon sources of STEC infection.

A 50-year-old man was admitted with nephrolithiasis and E. coli urosepsis treated with piperacillin-tazobactam for 7 days. His course was complicated by acute respiratory distress syndrome developing on day one of hospitalization, requiring prolonged ventilatory support and a tracheostomy. During the hospitalization, he developed a nosocomial pneumonia due to MRSA, treated with vancomycin. One the fourteenth hospital day, after cessation of piperacillin-tazobactam therapy, he was only receiving vancomycin. On hospital day 15, he developed a new rash and fever. On exam, he had erythematous areas over the trunk and thighs as well as bullae, which expressed yellow, serous, non-purulent fluid when opened. The most likely diagnosis is which of the following: Dermatitis herpetiformisBullous pemphigoidLinear IgA bullous diseaseHerpes zosterStaph scalded skin syndrome

Correct Answer: Linear IgA bullous disease Linear IgA bullous disease (LABD) is a rare autoimmune subepidermal blistering disorder, which is usually idiopathic. The syndrome is characterized by a vesiculobullous eruption of the skin and mucous membranes. On pathology, there is linear IgA deposition along the basement membrane. Drug-induced LABD is most often associated with the use of IV vancomycin. It generally resolves with discontinuation of vancomycin, but may recur with rechallenge. The other entities in the differential diagnosis are less likely. Herpes zoster presents with grouped vesicles, which are very different in appearance than the large bullae seen here. Dermatitis herpetiformis is usually extremely pruritic and is characterized by grouped excoriations, erythematous plaques and papules with vesicles as opposed to bullous formation. Dermatitis herpetiformis is an autoimmune blistering disorder that is frequently associated with a gluten-sensitive enteropathy . The usual location for dermatitis herpetiformis lesions is on the extensor surfaces of the back, elbows, knees, and buttocks.. Treatment is ususally a gluten free diet and dapsone. Bullous pemphigoid is a chronic, inflammatory disease that is subepidermal and blistering. This entity, it can persist for months or years, with periods of spontaneous remissions and exacerbations. Treatment is anti-inflammatory or immunosuppressive drugs. Bullous pemphigoid may be precipitated by drugs, but the immunofluorescent staining is different than that of linear IgA bullous disease (LABD). Staphylococcal scalded skin syndrome (SSSS) is a toxin-mediated disease, usually in children, in which the skin easily detaches by rubbing, which is known as Nikolsky's sign. The toxin acts on the stratum granulosum of the epidermis. SSS differs from toxic epidermonecrolysis since SSS almost never involves the mucosa while TEN typically usually involves the mucous membranes. The severity of SSS varies from a few blisters that are localized at the site of infection to a severe exfoliation affecting the entire body. Treatment is supportive, plus treatment for the underlying MSSA or MRSA. Many clinicians add a ribosomal inhibitor such as clindamycin for a few days to reduce toxin production

A previously healthy 60-year-old woman presents to the emergency room complaining of headaches, fevers, nausea, and vomiting for the past 4 days. Two days prior to her ER presentation, she noted difficulty with balance. Earlier this morning, she noted left facial numbness. She denied any night sweats, photophobia, or neck stiffness. She has not traveled outside of the United States and has lived her entire life in Baltimore, Maryland. She lives with her husband and two daughters in a row house. They own 2 cats and a dog. She denies smoking, alcohol use, or injection drug use. On physical examination, her temperature is 38.3°C, blood pressure is 120/60 mm Hg, heart rate is 109 beats/minute, and respiratory rate is 13 breaths/minute. She is alert and oriented but appears uncomfortable. Higher cortical functions, extraocular movements and visual fields were intact. Limb tone, motor strength and reflexes were normal. Plantar responses were flexor. Abnormalities included left facial hypoaesthesia, right facial weakness and left gaze-evoked nystagmus. She was noted to have gait ataxia. The remainder of her physical examination was unremarkable. Her peripheral white cell count and differential were normal. A comprehensive metabolic panel was normal. Lumbar puncture and CSF examination revealed a lymphocytic pleocytosis with 500 white blood cells/mm3 (94% lymphocytes). CSF glucose level was normal and protein level was slightly elevated. No organisms were visualized on Gram's stain of the CSF. CT scan without IV contrast was unremarkable. Postgadolinium T1-weighted MRI images of the brain showed multiple ring-enhancing abscess-like lesions in the brainstem with mild meningeal enhancement. Which of the following is the most likely etiology of her clinical presentation? Behcet's disease Cytomegalovirus (CMV) Herpes Simplex Virus type 2 Listeria monocytogenes Mycobacterium tuberculosis

Correct Answer: Listeria monocytogenes The most likely diagnosis is rhombencephalitis. The most common infectious cause is L. monocytogenes that often affects otherwise healthy individuals. Other causes of infectious rhombencephalitis are Enterovirus 71, HSV-1, HSV-2, HHV-6, EBV, and M. tuberculosis. Behcet's is the most common non-infectious cause. Rhombencephalitis as a result of a paraneoplastic process also occurs. The classic clinical syndrome of L. monocytogenes-associated rhombencephalitis is a biphasic illness characterized by a prodrome of fever, headache, nausea, and vomiting, followed by the sudden onset of progressive asymmetrical cranial nerve abnormalities, cerebellar signs, and hemiparesis or hemisensory defects, with or without meningeal signs. The majority of patients have a lymphocytic CSF pleocytosis (only 40% have a CSF dominated by polymorphonuclear cells). Blood cultures are more likely to be positive (~60%) than CSF cultures (~30%). MRI is superior to CT for demonstrating rhombencephalitis. Finding abscesses with ring enhancement can be useful in making a diagnosis of Listeria-associated rhombencephalitis. Immediate antimicrobial therapy is the most important predictor of a favorable outcome (Clauss and Lorber. Curr Infect Dis Rep. 2008;10(4):300-6). None of the other pathogens listed can explain the entirety of the clinical presentation: Neither CMV nor HSV are likely to present with ring enhancing abscesses of the brainstem in an immunocompetent adult. While M. tuberculosis is a rare cause of rhombencephalitis, the involvement of the basal cisterns with leptomeningeal enhancement after intravenous injection of contrast is the finding most characteristic of tuberculosis-associated rhombencephalitis. She has no known risk factors for TB. She does not have any other clinical manifestations to suggest Behcet's disease.

A 36-year-old female injection drug user is admitted for 4 days fever, neck pain and stiffness. She has a temperature of 38.3°C, pulse of 110, respiratory rate of 16, and blood pressure of 120/70. On admission physical examination, she is alert and oriented. She has no cervical spine tenderness but does note pain with flexion and extension of the neck. The rest of the physical exam is unremarkable, other than signs of needle track marks, including the cardiac exam is normal. Blood cultures have been drawn and you are asked to recommend further work-up. Which one of the following studies is most important to optimize the outcome for this patient? Lumbar puncture for cerebrospinal fluid studies Transthoracic echocardiogram (TTE) CT of chest and abdomen CT scan of the neck MRI of the cervical spine

Correct Answer: MRI of the cervical spine a spinal epidural abscess, most likely due to S. aureusin an injection drug user. management usually requires prompt drainage, especially in patients with neurologic deficit. antimicrobial therapy alone can be considered in patients who have localized pain or radicular symptoms without long-tract findings, but these patients require frequent neurologic examinations and serial MRI studies to demonstrate resolution of the abscess. The clinical presentation does not suggest meningitis. CSF findings, even if abnormal, are likely to show a nonspecific pleocytosis due to a parameningeal focus of infection and unlikely to establish a diagnosis. Although the patient could have endocarditis, most patients with epidural abscess do not. An echocardiogram should be ordered if blood cultures are positive, but at this juncture the greater and more urgent threat is spinal epidural abscess. Although a chest x-ray and perhaps a chest CT is indicated in a patient with a fever and history of injection drug use, any results are unlikely to explain the neck findings on exam. A CT of the neck may show evidence of soft tissue or bony infection, but it is insensitive for an epidural abscess. The most sensitive test for diagnosis of epidural abscess is MRI.

A 36-year-old automobile repairman from West Virginia presented with a six-month history of progressive easy fatiguability and dyspnea on exertion. A CT scan found an anterior mediastinal and perihilar mass that on mediastinoscopy was a hard mass that on biopsy showed only fibrosis and a few lymphocytes. Upper extremity venogram showed complete obstruction of the superior vena cava with abundant collaterals. Echocardiography was consistent with pulmonary hypertension. Thoracic surgery did not recommend attempted excision of the mass. The most appropriate management would be which of the following: Empirical therapy with fluconazoleEmpirical therapy with itraconazoleEmpiral therapy with antituberculous chemotherapyLong-term therapy with prednisoneMedical follow-up with no current therapy

Correct Answer: Medical followup with no current therapy Mediastinal fibrosis is most commonly due to prior histoplasmosis, though the organisms are difficult to find in biopsy specimens and cultures are routinely negative. Rare cases are due to tuberculosis or molds, organisms that are usually seen histologically but are culture negative. Surgical excision is rarely possible and may jeopardize valuable collateral venous return to the right heart. No medical treatment, including antifungal therapy has been shown to alter to slow progression of this indolent disease. Stenting of constricted bronchi or accessible veins may be helpful. In this case, stenting the SVC is unnecessary because collaterals are allowing return to the right heart and the symptoms are due to pulmonary hypertension, not SVC obstruction. Medical treatment for complications, such as right heart failure, can warrant follow-up.

Which of the following correctly ranks tests, as applied to sputum, from most sensitive for detection of pulmonary tuberculosis to least sensitive for detection of pulmonary tuberculosis: Most sensitive > Least sensitive Nucleic acid amplification tests > mycobacterial culture > smear microscopy Mycobacterial culture > nucleic acid amplification tests > smear microscopy Smear microscopy > nucleic acid amplification tests > mycobacterial culture Mycobacterial culture > smear microscopy > nucleic acid amplification tests

Correct Answer: Mycobacterial culture > nucleic acid amplification tests > smear microscopy Lower limits of detection, in colony forming units per mL of sputum, are approximately 10,000 cfu/ml for smear microscopy, 100 cfu/ml for nucleic acid amplification tests, and 10 cfu/ml for culture. The clinical corollary is that negative sputum smears do not exclude a diagnosis of pulmonary TB; only about 50% of pulmonary TB patients have a positive sputum smear for acid fast bacilli. Nucleic acid amplification tests are more sensitive than smear microscopy, but still miss some pulmonary TB patients, especially those in whom there are few bacilli in communication with the respiratory tract. In the U.S., the gold standard is considered to be 3 sputum specimens for mycobacterial culture. Of note, unlike mycobacterial cultures, nucleic acid amplification tests can be positive even if the M. tuberculosis bacteria are dead. Therefore nucleic acid amplification tests should not be used routinely to monitor progress on therapy.

A 72 year old US born, white female reports a history of needing antibiotic therapy for repeated respiratory infections over the last 12 months. With each treatment she improves to near her baseline, but within several weeks her cough has worsened again, became more productive, and she complains of fatigue. Overall, she notes a decline in exercise capacity, 10 lbs weight loss, and progressive fatigue the last 6 months. She is a life-long non-smoker and has no risk factors for tuberculosis. She is otherwise healthy and takes no medications. Her chest radiograph is normal, but a chest computed tomograph (CT) reveals right middle lobe bronchiectecasis with scattered tree-bud infiltrate, mucous plugging, and a small right upper lobe cavity with a fungus ball present within the cavity. The most likely cause of her syndrome and progressive decline is: Mycobacterium. gordonae chronic necrotizing aspergillosis Mycobcterium. tuberculosis Mycobacterium avium complex Nocardia farcinica

Correct Answer: Mycobacterium avium complex In this case, the patient has both symptoms, history, and radiographic features suggesting chronic pulmonary non-tuberculous mycobacterial disease. The fungus ball is a consequence of her lung disease, not a cause. The overwhelming majority of NTM disease in the US is caused by M. avium complex. It frequently co-exists with bronchiectasis and causes all of the radiographic features mentioned above. This patient's condition has been called "Lady Windermere's syndrome", M. gordonae is a non-pathogenic NTM except in very rare or extreme circumstances, and it should be regarded as a contaminant. Nocardia is rarely as chonic as 6 months. Chronic necrotizing aspergillus appears in patients with severe underling lung disease. M. tuberculosis should be considered in the differential, but is much less common in the US, unlikely to be confined to the right middle lobe, and unlikely to occur in an individual lacking TB risk factors

A 27-year-old male with sickle cell disease presents with a chest syndrome crisis: he has his typical fever, chest pain, and leukocytosis. He is cultured, and started on vancomycin and levofloxacin. His initial chest radiograph shows bilateral infiltrates: an x-ray one week prior showed only some chronic scarring. Sputum gram stain, acid fast stain, and culture show only modest amounts of normal flora. His CBC and Chem 12 show a leukocytosis, hemolytic anemia, and mild LFT abnormalities. Careful physical examination reveals no localizing physical findings: a tunneled double lumen subclavian line, in for 4 months, appears unremarkable. After 96 hours, the laboratory reports that one blood culture is growing a non-branching beaded gram positive bacillus that is acid fast stain positive. Three additional blood cultures are drawn (one through each lumen of the subclavian line) and one peripherally: at 48 hours they are all reported to be positive for an acid fast non branching rod. Which of the following organisms would be most likely? Nocardia asteroidesMycobacterium mucogenicumRhodococcus equiLegionella mcdadeiMycobacterium tuberculosis

Correct Answer: Mycobacterium mucogenicum The beaded, acid fast organism appears to be some type of rapid growing mycobacterium. Microbiologically, this could be Mycobacterium fortuitum, Mycobacterium chelonae, M. mucogenicum or Mycobacterium abscessus. These organisms rarely appear in blood cultures except for catheter acquired sepsis. The subclavian catheter, which has been in place for 4 months, is the likely source. None of the listed organisms are likely to enter the bloodstream from the lung, including Nocardia species. Also, the acid fast smear of the sputum showed no organisms. Management of catheter acquired sepsis from rapidly growing mycobacteriashould involve removal of the catheter. Immunosuppressed and immunocompetent patients are often treated but metastatic foci of infection are rare. Susceptibilities can be determined in most laboratories and used to guide multi-drug treatment if the patient is immunosuppressed or has other indications for treatment. The optimal length of therapy is unclear and is dictated by extent of infection and immune status of the host.

You are asked to see a 32-year-old AIDS patient who has recently completed an uneventful two-week course of amphotericin B and flucytosine for cryptococcal meningitis and is transitioning to oral fluconazole. A PICC (peripherally inserted central catheter) line tip culture, sent three days ago based on hospital protocol when the line was removed, is growing methicillin-resistant coagulase-negative staphylococci with vancomycin MIC = 1 μg/ml, daptomycin MIC = 0.5 μg/ml, linezolid MIC = 1 μg/ml, trimethoprim-sulfamethoxazole (TMP/SMX) MIC < 2/38 μg/ml. The patient has no fevers or evidence of infection at the former PICC site and wants to go home. Which of the following would you recommend? A 3-day course of vancomycin. A 3-day course of daptomycin. A 7-day course of linezolid. A 7-day course of TMP/SMX. No antibacterial therapy.

Correct Answer: No antibacterial therapy Tip cultures of intravascular catheters should not be routinely performed upon discontinuation of the catheter. In the absence of signs of infection, no therapy is indicated. If a catheter tip is cultured and grows > 15 cfu of Staphylococcus aureus, many clinicians would treat with an active agent for 5-7 days even in the absence of signs or symptoms of infection. Almost all other pathogens that grow in the absence of signs or symptoms have no diagnostic relevance although they may provide information on organisms colonizing the patient or they may be irrelevant contaminants.

A 35-year-old woman complains of 6 days of watery diarrhea without blood or fever. She denies travel and exposure to antibiotics. She has an 8-week-old infant at home who is well and has not been to childcare. You diagnose Clostridium difficile infection by fecal PCR for the toxin B gene. She responds clinically to treatment with oral vancomycin. Which of the following should you do to follow-up this infection? Repeat fecal C. difficile PCR for toxin A 2 weeks after her infection. Repeat fecal C. difficile PCR for toxin B 2 weeks after her infection. Repeat fecal C. difficile PCR for toxin A 8 weeks after her infection. Repeat fecal C. difficile PCR for toxin B 8 weeks after her infection. No follow-up fecal testing is warranted.

Correct Answer: No follow-up fecal testing is warranted. No follow-up fecal testing should be done after treatment of C. difficile disease. Continued detection of the organism and/or its secreted toxins (A and B) is possible after successful therapy and has no prognostic significance for development of recurrent C. difficile disease that occurs in 20-30% of individuals.

A healthcare worker sustains a deep needle stick with a hollow bore needle that was just used to drawn blood from a patient with hepatitis B (HBsAg positive) who was being assessed for hepatitis B treatment, but who had not yet started therapy. The healthcare worker previously was immunized for hepatitis B seven years ago and was documented immediately after that immunization series to have an adequate antibody response (antibody level >10 units/uL) Which of the following would be the best option for this exposed worker? HBIGLamivudine plus entecavir for at least 4 weeksHBIG and a minimum of four week course of Lamivudine and EntecavirHBIG and Lamivudine and Entecavir plus an HBV vaccine boosterNo therapeutic intervention

Correct Answer: No therapeutic intervention This patient was immunized to hepatitis B, as all healthcare workers should be, and was documented to have an adequate serologic response. Some experts would provide a booster dose of HBV vaccine (this alone was not an option in this question), but neither HBIG nor chemotherapy for HBV is indicated for this patient.Recommended Treatment for According to Source of Needle Stick Injured Health Care Worker Hepatitis B Status Exposed Person HbsAg Pos HbAg Neg Unknown Unvaccinated HBIG plus vaccine Vaccine Vaccine Previously Vaccinated Responder None None None Non Responder HBIG plus re-vaccinate None If high risk, treat as if source is positive Unknown Response Test for Ab and then per above None Test for Ab and then per above

A 51 year-old male with an 18 year history of well controlled HIV was seen on a routine visit and found to have a rising creatinine. One month prior, his antiretroviral medication has been changed at his request for a once daily regimen. His viral load had been undetectable and his CD4 750 while taking emtricitabine-tenofovir disoproxil fumarate (TDF)-lopinavir-ritonavir. He was HLA-B *5701 negative. His new regimen for the past month was abacavir/dolutegravir/lamivudine. His serum creatinine one month prior had been stabile at 1.45 mg/dl but after one month had risen to 1.79 mg/dl. His urinalysis now is still normal as was his serum phosphate and electrolytes. He had no unusual physical activity, continuing his desk job. He was taking no other medications and had no history of renal stones, hypertension or diabetes mellitus. What is needed now to address his rising creatinine. NothingRenal UltrasoundUrine cultureChange back to his prior regimenCT of the abdomen and pelvis

Correct Answer: Nothing Rationale: A slight rise (0.1- 0.2 mg/dl) is expected from dolutegravir because it decreases creatinine clearance without an effect on glomerular filtration rate. Dolutegravir affects the function of the organic cation transporter 2 (OCT2) in the renal tubular cell which effluxes creatinine. The creatinine rise is seen in the first month of treatment and then the serum creatinine stabilizes.

A 26-year-old woman who emigrated to the United States 5 years ago from the Sudan was admitted to a hospital with pre-eclampsia. An incidental finding on admission was a painless rounded, raised lesion on the palm of her right hand. There was no surrounding erythema or edema. This had been present for two weeks She had not left the United States since emigrating from Sudan, and had had no such lesions before. Through an interpreter, it was determined that three weeks previously she had obtained a sheep's head from a halal butcher to be used in cooking. While preparing the head she accidently cut her hand with a spicule of bone from the front part of the sheep's face. She subsequently developed the described lesion. The most likely cause of this lesion would be: Anthrax Brucella ORF Mycobacterium ulcerans Leishmania mexicani

Correct Answer: ORF ORF is a poxvirus (specifically a parapoxvirus) that causes painless lesions at inoculation sites in 14-21 days after inoculation and which ultimately resolve spontaneously over 4-8 weeks. They do not cause systemic symptoms or disseminate except in unusual cases of immunosuppression. There is no therapy. This a painless ulcerated nodule. The differential might include anthrax, leishmania, or Buruli ulcer: patients are not necessarily systemically ill with any of these entities (with cutaneous anthrax systemic manifestations may develop. This woman had been in the United States for 5 years, making Leishmania and anthrax unlikely -anthrax is seen primarily in the United States in bison, not domestic animals. Buruli ulcers, due to M. ulcerans, are acquired in Africa, not the United States. ORF is common in herds of goats and sheep: contact with oral lesions of animals, or with milking them, can lead to this type of lesion. Presumably this woman had contact with an oral lesion in the sheep head and the virus was then inoculated into her skin by the minor trauma. Similar cases can occur with cows, goats, deer, or with elk. See the lesion in the goat oropharynx below.

A 66-year-old male comes to the Emergency Room with severe nausea and vomiting for 6 hours with diarrhea for 2 hours. He has been in relatively good health but has adult onset diabetes, mild heart failure, and hypercholesterolemia. He has no history of prior GI problems and no one else in his family is ill, including the two toddlers that his wife cares for in his home. On physical examination, he is febrile to 38.3°C, BP 150/80, RR 20, P 100. He has a mildly tender abdomen with no rebound. Laboratory: WBC 11,000 (60% polys), Hg 13 g/dl, Plat 220,000 Chemistry Profile: Na 145 meq/L, K 3.8 meq/L, CO2 22 meq/L, BUN 20 mg/dl, Creat 2.0 mg/dl Lactate 0.9 mmol/L Stool cultures sent: rotavirus screen negative CT Scan: The ER starts vancomycin and ciprofloxacin plus metronidazole and calls you for a consultation. What is the appropriate response to the finding of pneumatosis intestinalis in this patient? Immediate laparotomyChange antibiotic regimen to meropenem and vancomycinAdd caspofungin to ciprofloxacin and metronidazoleColonoscopyObservation of clinical course: no change in current management

Correct Answer: Observation of clinical course: no change in current management The pneumatosis intestinalis refers to gas within the wall of the small or large bowel and has also been referred to as pneumotosis coli, intestinal emphysema, and pneumatosis cystoides. This is a radiologic sign and should not by itself alter the plan developed on the basis of history, physical examination, and laboratory markers. The condition can be managed medically unless there are clinical features suggesting an acute abdominal emergency such as bowel obstruction, perforation or peritonitis. Thus, this patient could have food poisoning, a gastrointestinal virus such as Norovirus, a bacterial pathogen such as Shigella, or a host of other infectious and non-infectious syndromes. At this point, no data are given suggesting that more aggressive diagnosis or therapy is necessary.

In September, a 38-year-old year old man working in the rice fields located in south eastern Korea became ill with a fever of 104°F, sweats, severe headache, dry cough, and lethargy during his second week on the job. He received amoxicillin from a local physician but did not feel better. Over the next day, he developed generalized macular rash which started on the trunk and then spread to the extremities. He was admitted to a hospital, where he became hypotensive, hypoxic and oliguric and was transferred to the ICU. An ICU nurse incidentally noted a 6-mm skin lesion with a necrotic dark center on his upper back while she was turning him. Which of the following is the most likely etiology? Rickettsia parkeri Rickettsia rickettsii Leptospira interrogans Orientia tsutsugamushi Rickettsia akari

Correct Answer: Orientia tsutsugamushi Based on the clinical presentation and geographic setting, scrub typhus caused by Orientia tsutsugamushi would be high on the list of possible diagnoses. This potentially serious infection occurs in rural areas in a wide region; Korea is an endemic area. The vector and reservoir is the thrombiculid mite larva also known as a chigger. Scrub typhus does not occur in the USA. Diagnosis is by serology. Culture and PCR of blood, skin or lymph node are other potentially useful diagnostic modalities. Doxycycline is the preferred treatment. The ABIM usually avoids questions about patients whose illness presents overseas, such as this man and scrub typhus is unusual in travelers. Rickettsia ricketsii and Rickettsia parkeri do not occur in Korea. R. rickettsii and Leptospira are not associated with an eschar.

A 19-year-old man in St. Louis presents to the emergency department with a three-week history of cough, hemoptysis, and fevers. Chest x-ray shows a left lower lobe infiltrate with a small pleural effusion. Labs are remarkable for a white blood cell count of 15,200 cells/mm3 with 48% neutrophils, 24% lymphocytes, 5% monocytes, 22% eosinophils, and 1% basophils. The patient just graduated high school and reports that four months ago he and his friends went on a camping trip where he ate two raw crayfish on a dare. The organism most likely for the patient's symptoms is: Paragonimus kellicotti Trichinella spiralis Anisakis simplex Taenia saginata Diphyllobothrium latum

Correct Answer: Paragonimus kellicotti Paragonimiasis is a trematode infection of the lungs that is acquired by eating undercooked crabs or shellfish. During the late stage of infection, which occurs weeks to years after initial exposure, patients can develop chronic pulmonary disease with pulmonary infiltrates, cough, hemoptysis, and eosinophilia. Chest x-ray can show focal infiltrates as well as an eosinophilic pleural effusion. Paragonimus westermani, the most common cause of the disease in humans, is common in Southeast Asia. Most cases in the United States occur because of acquisition abroad or due to consumption of imported crabs or shellfish containing the parasite. In the past several years, though, there have been reports of indigenous acquisition of P. kellicotti in the United States, with over a dozen cases reported to date (the majority in Missouri). Many of these have occurred in individuals consuming raw crayfish. None of the other organisms listed cause pulmonary disease. T. spiralis causes intestinal symptoms followed by dissemination of newborn larvae into the muscles and associated edema, eosinophilia, and myositis. A. simplex is acquired by eating undercooked fish and causes an intestinal disease in which the larval form burrows into gastric or intestinal mucosa. T. saginatum and D. latum are both intestinal tapeworms.

A 45-year-old male is diagnosed with Helicobacter pylori infection by endoscopy and antral gastric biopsy performed for weight loss and abdominal pain. There is a family history of gastric cancer. He is treated for 14 days with omeprazole, clarithromycin, and amoxicillin. What would be best option to evaluate this patient regarding Helicobacter infection/disease after completing antibiotic therapy? No further testing is necessary for one year Perform the stool Helicobacter pylori antigen test 8 weeks after treatment Perform the urea breath test 3 weeks after treatment Repeat endoscopy, biopsy and rapid urease test (RUT) 6 weeks after treatment

Correct Answer: Perform the stool Helicobacter pylori antigen test 8 weeks after treatment This patient should have testof cure (TOC) for H. pylori therapy to confirm eradication of the organism. The H. pylori stool antigen test performs with high sensitivity and specificity. It is non-invasive and is FDA-approved for TOC, making this the optimal approach. The test should be performed at least fourweeks after completion of antibiotic and at least twoweeks off PPI therapy to avoid false negatives. Both the urea breath test and endoscopy with rapid urease testing are acceptable alternatives. However, the urea breath test is dependent on live organisms. Three weeks after treatment is too soon to repeat this test as a false negative result may occur. Given the recent endoscopy and biopsies, it is not necessary to again perform endoscopyto evaluate for gastric cancer, and it is not necessary to perform an invasive test to confirm cure of H. pylori infection.

A 28-year-old woman with HIV-1 infection has persistent viremia after three years of good control on abacavir /lamivudine (3TC) + atazanavir/ritonavir. Viral RNA level = 25,000 copies/ml and CD4 count = 320 cells/µl Her genotype shows: RT: M184V and Protease: I50L Which of the following statements is most correct about I50L? I50L is a common polymorphism found in drug naïve individuals that does not contribute to protease inhibitor resistance. Phenotype of viruses with an I50L mutation will show resistance to atazanavir, darunavir, and lopinavir. Phenotype of viruses with an I50L mutation will show resistance to atazanavir but will be sensitive to protease inhibitors darunavir and lopinavir. Phenotype of viruses with an I50L mutation will show resistance to atazanavir, but the IC50 will not be high, and ritonavir boosting will suppress the virus.

Correct Answer: Phenotype of viruses with a I50L mutation will show resistance to atazanavir but will be sensitive to protease inhibitors darunavir and lopinavir. I50L is a relatively specific primary mutation that confers high level resistance to atazanavir. Boosting with ritonavir will not overcome this, so D is incorrect. This I50L is specific for atazanavir and in fact confers hypersusceptiblity to darunavir and lopinavir. Thus B is not correct. This mutation has only emerged since atazanavir was introduced, so A is incorrect. Correct Response Phenotype of viruses with an I50L mutation will show resistance to atazanavir but will be sensitive to protease inhibitors darunavir and lopinavir.

A 37-year-old female nurse comes to your office to get advice. She recently spent four weeks on a mission trip in Chad, and one week prior to coming back to the United States she received a minor bite on her lower leg from a stray dog. She cleaned the wound and took a few days of amoxicillin/clavulanic acid. Efforts to obtain rabies vaccine and rabies immune globulin in Chad were unsuccessful. The wound healed without complication and she is feeling well. She had a tetanus shot last year. When she was a child living overseas she had serum sickness after receiving horse tetanus immune globulin. She has been back in the United States for a week, so it has been two weeks since she had the bite and the site is not inflamed. What treatment is indicated? Nothing else is indicated at this late date Rabies vaccination only Rabies vaccination and rabies immune globulin in the buttocks Rabies vaccination and rabies immune globulin in the bite site Rabies vaccine and skin testing for allergy to horse serum prior to administration of rabies immune globulin

Correct Answer: Rabies vaccination and rabies immune globulin in the bite site Post-exposure prophylaxis for rabies should begin as soon as possible after the exposure. It should be noted that, since long latency periods between exposure and onset of disease have been reported, postexposure prophylaxis should be given following a rabies exposure, regardless of the length of the delay. Rabies immune globulin should be given with the vaccine in order to provide immunity before the response to the vaccine will occur. It should be given no later than 7 days after vaccine series is initiated in order to prevent interference with vaccine effect. As much as possible should be infiltrated into the area of the bite, even at this late date. Human rabies immune globulin is available in the United States so that allergy to horse serum is not an issue.

A 51-year-old woman is referred with complaints of more than 6 months of fatigue, poor sleep, musculoskeletal pains and difficulty with concentration. She denies depressive symptoms but frustrated she cannot accomplish her normal activities including exercise that appears to especially precipitate exhaustion. She has a history of Lyme disease presenting as erythema migrans, treated with doxycycline 9 years earlier with full recovery. Her exam reveals no remarkable findings. Her recent laboratory studies document a normal CBC, ESR, routine chemistries and thyroid stimulating hormone levels. Her primary care physician also obtained Lyme serology with a positive B. burgdorferi EIA with both positive IgM and IgG immunoblots. Your recommendation should include: Reassuring that active infection with Borrelia burgdorferi acquired 9 years ago is not an explanation for her illness Repeating two-tier Lyme serology Obtaining Lyme C6 antibody Doxycycline x 14 days Ceftriaxone IV x 28 days followed by doxycycline x 60 days

Correct Answer: Reassuring that active infection with Borrelia burgdorferi acquired 9 years ago is not an explanation for her illness Patients who have a history of treated Lyme disease may maintain positive serologic evidence for more than 20 years, including positive IgM or IgG immunoblots. Although IgM immunoblots can be used to help diagnose patient with early Lyme disease with symptoms of less than four weeks duration, it should not be used alone in patients with long-term symptoms due to high rates of false positivity. The Lyme C6 antibody is less specific than traditional two tier testing but could be helpful in assessing whether a Lyme IgM immunoblot is a false positive; however, in a patient with known prior Lyme disease, this is less helpful although a negative assay would confirm well-treated prior infection as this titer declines over time in a majority patients (best studied in patients with early Lyme disease, erythema migrans of whom ~70% document C6 titer decline two years after initial antibiotic therapy). Although acquiring Lyme disease a second time is possible, there is no objective evidence of new, active infection in this scenario given just subjective complaints. Moreover, multiple randomized, blinded trials of long-term antibiotics versus placebo have found no benefit in durably aiding the relief of subjective symptoms such as fatigue or musculoskeletal pains in patients with a history of previously treated Lyme disease.

Over a 3-week period, 5 patients in a 12-bed ICU have infections with a carbapenem-resistant Klebsiella pneumoniae (KPC): Two have symptomatic urinary tract infections, 2 have ventilator-associated pneumonia, and 1 has a line-related bacteremia. These are the only KPC infections recognized in this ICU in the past 6 months. Whole genome sequence (WGS) analysis of the isolates shows that four are nearly identical and one probably genetically unrelated. The most likely epidemiologic explanation for these infections is that this cluster represents which of the following: Is a pseudoepidemicResults from lapses in infection controlResults from common source medication contaminationRepresents a water-borne outbreakRepresents a food-borne outbreak

Correct Answer: Results from lapses in infection control Pseudoepidemics usually reflect surveillance, diagnostic, or lab errors — unlikely in this scenario. So, the occurrence of a cluster of KPC likely is a real epidemic. The occurrence of a predominant WGS-type and the multiple different sites of infection suggest cross-infection due to poor infection control practices. The other explanations - water, food, or medication contamination — would not explain the sites of infection and are unlikely based on the known current behavior of KPC. The existence of 2 WGS types suggests 2 separate introductions of KPC into the ICU.

A 59-year-old woman from Myanmar (Burma), now two months status post deceased donor kidney transplant for hypertensive nephrosclerosis and diabetic nephropathy presents with fever, right anterior chest pain, and a RML infiltrate. Her post-transplant course has been complicated by repeated episodes of graft rejection, for which she received 2 courses of pulse-dosed steroids, in addition to her maintenance doses of sirolimus and mycophenylate mofetil. She has a history of latent TB and was treated for 9 months with INH prior to transplantation. She lived in Burma as a child, but moved to Detroit, MI 35 years ago. She has been transferred to your institution from an outside hospital, with persistent fever despite 10 days of levofloxacin and 5 days of meropenem. Her evaluation from outside hospital included negative blood cultures, sputum cultures with growth of E. coli, and bronchoscopy with lavage and biopsy performed within 1 day of antibiotics, which revealed growth of Candida albicans but no bacteria and a core biopsy that showed "benign lung tissue, early organizing hemorrhage within alveolar spaces, and reactive pneumocytes." Additional laboratory analyses include negative galactomannan assays from blood and BAL, CMV serum PCR results of 1,200 copies and growth of Vancomycin-Resistant Enterococci from rectal swabs at admission. At the time of transfer, exam reveals tenderness and erythema on the right anterior chest wall, but no skin lesions, decreased breath sounds in that area, and CT scan that shows an increased RML infiltrate, now more central cavitation. An ultrasound of her chest wall shows no fluid collection, but signs of inflammation within the soft tissues. Which of the following is the most likely? Rhodococcus equi Penicillium species Rhizopus spp. Candida species Burkholderia pseudomallei

Correct Answer: Rhizopus spp. This patient has a rapidly evolving necrotizing pneumonia that is locally invasive into soft tissues. Of the above organisms, the most likely to cause this would be mucormycosis. Penicillium species are an unlikely cause of a locally invasive necrotizing pneumonia, and Candida in this setting was likely an uninvolved colonizing organism. While Rhodococus equi could cause pneumonia, one would expect to recover this organism from BAL in the setting of a large necrotizing pneumonia. Invasion of the chest wall would be atypical. Burkholderia pseudomallei would be a consideration given her endemicity from Burma, but she moved out of the endemic country long ago, and this would as well likely be recovered from cultures of respiratory secretions, lavage or biopsy. Agents of mucormycosis can cause locally invasive necrotic pneumonias that are difficult to diagnose by culture evaluation, and her risks include her receipt of transplant, immunosuppressive regimen for chronic graft rejection, and multiple courses of pulse-dose steroids.

A 58-year-old healthy woman from Boston returned from South Africa two days ago, staying for 2 weeks, visiting the major cities and spending 5 days in a game park. She received no pre-travel counseling or immunizations. She had never traveled before except to the Caribbean. On return, she began to have low grade fevers within 2 days. She noted myalgias, and headaches as well as a scab on her right thigh which was painless. She saw a travel medicine expert 2 days after these symptoms occurred and she was feeling even worse. On exam, she was uncomfortable, but with a normal exam except for a temperature of 38.6°C, a scabbed lesion on the right thigh , a few papular skin lesions on both legs, and a slightly tender 1cm node in the right inguinal region. Labs: WBC-3600/mm^3(40% PMN, 54% Lymphs, 4% eos); Hgb-11.6 Gm/dL; Platelets-94,000/mm3.Creatinine -1.2 ; AST/ALT- 69/84 U/dL; Alk Phos-98U/dL, Bili-1.0mg/dL. The most likely diagnosis is: Dengue Fever Typhoid Fever Plague Babesiosis Rickettsia africae

Correct Answer: Rickettsia africae The eschar, leukopenia and rash is suggestive of a rickettsiosis. E is correct. R. africae is the most common rickettsial illness in travelers going to S. Africa. It is transmitted by a tick bite and causes an eschar at site of bite and then systemic symptoms of fever and malaise with a maculopapular rash several days after onset. Treatment is with doxycyline. Dengue fever does not have an eschar or local adenopathy. Typhoid fever would not be the best answer due to the presence of the eschar. The fever and "rose spots" would be typical, but this patient appears to have papular extremity lesions, not rose spots. Rose spots (bacterial emboli to skin) typically appear on the torso or abdomen, not the extremities. Babesiosis is not associated with an eschar or rash. In addition, no mention is made in the stem of exposure that would make one suspect babesia, i.e. travel to Martha's Vineyard, Nantucket, Shelter Island (New York), or some other similar coastal area from Rhode IslandNew Jerseynorth. Some cases of babesiosis also have been reported in California, associated with cattle, and in the Midwest.

A 45 year old woman is referred to you by the public health department for ongoing management of recently diagnosed pulmonary tuberculosis which is confirmed susceptible to all first-line antituberculosis drugs. Three days ago she started a regimen of daily rifampin, isoniazid, pyrazinamide, and ethambutol (plus pyridoxine). She reports that she takes warfarin for atrial fibrillation. Which statement is correct regarding drug-drug interactions between the anti-tuberculosis drugs and warfarin? No clinically important drug-drug interaction Rifampin will increase the clearance of warfarin Rifampin will decrease the clearance of warfarin Pyrazinamide will increase the clearance of warfarin

Correct Answer: Rifampin will increase the clearance of warfarin There is a clinically important drug-drug interaction between rifampin and warfarin (as well as between rifampin and hormonal contraceptives, fluconazole, corticosteroids, methadone, most HIV NNRTIs, HIV PIs, HIV INSTIs, HIV CCR5 inhibitors). Rifampin induces a number of enzymes involved in drug metabolism(e.g, cytochrome p 450 system) , which increases the metabolism of those other drugs and can result in sub-therapeutic blood levels and hence reduced efficacy of those drugs (thus requiring increased dose of the other drug(s) to maintain therapeutic effect). Pyrazinamide has no clinically important effect on enzymes involved in drug metabolism and has no clinically important drug-drug interaction with warfarin. Correct Response Rifampin will increase the clearance of warfarin

A 48-year-old physician presents with complaints of severe fevers, abdominal pain, diarrhea, and back pain for 5 days. The patient returned from a 6-month medical mission to Sudan 2 weeks ago. The patient took doxycycline daily for malaria prophylaxis while there, but reports she would occasionally forget a dose. She experienced frequent insect bites, especially when she took hikes along the banks of the White Nile River. She was usually careful about what she ate, but about once a week would eat home cooked meals prepared by coworkers at the medicine clinic. On exam, her heart rate is 110 bpm, BP is 100/70, respiratory rate is 24/min, and temperature is 38.6 °C. Lung sounds are clear to auscultation bilaterally. Abdomen is soft with moderate tenderness in the right upper and right lower quadrants. Abnormal laboratory values include a white blood cell count of 18,400/mm3 with 45% neutrophils, 24% lymphocytes, 6% monocytes, 24% eosinophils, and 1% basophils. AST is 158 units/L and ALT is 144 units/L. Ova and parasite examinations on stool and urine samples, sent by the patient's primary physician three days ago, are negative. Which of the following organisms is most likely causing her illness? Salmonella typhi Plasmodium falciparum Onchocerca volvulus Schistosoma mansoni Ancylostoma duodenale

Correct Answer: Schistosoma mansoni acute Schistosomiasis, also known as Katayama syndrome. sudden onset of fever, abdominal pain, and marked eosinophilia in travelers with exposure to freshwater in endemic areas. contact with freshwater when walking along the banks of the White Nile River. In Schistosomiasis, cercariae released by freshwater snails directly penetrate human skin. Once inside a person, the parasites become schistomula and migrate through the circulation to mesenteric vessels of the bowel (S. mansoni) or bladder (S. haematobium). Acute Schistosomiasis occurs at the onset of egg laying by the mature parasites, which is typically 4 to 8 weeks after infection. The diagnosis of acute Schistosomiasis is often a clinical one, as initial diagnostic studies (including both ova and parasite exams as well as antibody studies) can be negative early in the course of infection. S. typhi and P. falciparum can present with similar symptoms of abdominal pain and fevers. However, neither would be expected to cause a high level of eosinophilia. O. volvulus is the cause of river blindness and causes skin and eye manifestations. A. duodenale, a cause of hookworm disease, can cause abdominal pain but is much less likely than acute Schistosomiasis to cause fever.

Four adults became ill after having lunch together at a restaurant. They all ate raw clams followed by a tuna-spinach salad and rice. Within two hours of the meal all four experienced symptoms which included a burning sensation in the mouth, facial flushing, palpitations, nausea, sweating and diarrhea. All symptoms resolved within a few hours. Which one of the following was the most likely cause of illness? Ciguatera toxin Neurotoxic shellfish poisoning Scombroid toxin Tetrodotoxin Staphylococcal

Correct Answer: Scombroid toxin Scombroid poisoning is the most common seafood associated illness in the U.S. It is often related to ingestion of dark meat fish such as tuna, mackerel, skip-jack and bonito; clusters are common. Illness is due to histamine produced when bacteria overgrow on improperly stored fish. Symptoms usually begin within an hour of ingestion and resemble an IgE-mediated allergic reaction. Incorrect Answers Ciguatera is associated with ingestion of reef-dwelling tropical fish like barracuda, amberjack, grouper and red snapper and is due to dinoflagellate toxins consumed by the fish. Symptoms occur 3-6 hours after ingestion and include GI symptoms such as abdominal pain, nausea and diarrhea; later neurological symptoms occur such as paresthesias, painful teeth, blurred vision, nerve palsies, and hot/cold temperature reversal. Neurotoxic shellfish poisoning is associated with red tides and the ingestion of contaminated shellfish. Symptoms are similar to those seen with ciguatera. Tetrodotoxin (pufferfish poisoning) causes rapid onset of weakness, dizziness, paresthesias of face and extremities, nausea, and loss of reflexes; high doses may result in hypotension and paralysis. Staphylococcal food poisoning has an incubation of about 4-6 hours and produces severe nausea and vomiting.

A 26-year-old pregnant woman is being seen at her first prenatal visit. She is otherwise healthy and asymptomatic. She has been married for four years; she and her husband are monogamous. She was never diagnosed with a sexually transmitted infection. Her Papanicolaou smears have always been normal. She lives in a state that requires syphilis screening at the time of delivery. In addition to the mandatory screening at the time of delivery, which of the following syphilis screening strategies for this patient is most consistent with current CDC recommendations? No additional screening Screening at the first prenatal visit Screening early in the third trimester (~28 weeks) Screening at the first prenatal visit and early in the third trimester

Correct Answer: Screening at the first prenatal visit This is a low-risk patient given her history. The 2015 CDC STD Treatment Guidelines recommends the following: A serologic test for syphilis should be performed for all pregnant women at the first prenatal visit. Women who are at high risk for syphilis should be screened again early in the third trimester (at approximately 28 weeks' gestation), and at delivery. Some states require all women to be screened at delivery. Neonates should not be discharged from the hospital unless the syphilis serologic status of the mother has been determined at least one time during pregnancy and preferably again at delivery if at risk. Any woman who delivers a stillborn infant should be tested for syphilis.

A 48 year old Cambodian female presents with 5 weeks of pain and swelling of the right wrist. Five years ago she was diagnosed with disseminated mycobacterial infection with cultures of sputum, blood, and T10 thoracic epidural abscess fluid aspirate all positive for Mycobacterium avium complex (MAC). HIV antibody test was negative. She was treated with 24 months of azithromycin, rifabutin, and ethambutol with improvement of symptoms. Tissue biopsy of the wrist demonstrates poorly formed granulomata and a few acid fast bacilli (AFB). Broth culture is positive for AFB on day 18; DNA-hybridization probe is negative for Mycobacterium tuberculosis. Which is one of the following is most likely to establish the underlying disease? HIV antibody test Molecular test for mutation in interferon gamma receptor gene Molecular test for mutation in the signal transducer and activator of transcription 3 [STAT3] gene PCR for M. tuberculosis 16s ribosomal RNA Serology for anti-interferon gamma auto-antibody

Correct Answer: Serology for anti-interferon gamma auto-antibody Disseminated MAC rarely if ever occurs in a normal host. Although HIV is a risk factor, the patient had a negative HIV test with the initial MAC diagnosis. It may be reasonable to repeat the test, but HIV cannot explain the original infection and as this is a recurrence, some other immunocompromising condition is more likely. Given the patient is from Southeast Asia and that her infection presented in adult life an acquired defect (i.e., an auto-antibody) in interferon gamma function is the most likely possibility. Disseminated MAC is associated with interferon gamma receptor deficiency due to a genetic mutation, but onset is in childhood and such a late presentation as an adult is unlikely. False-negative hybridization tests for M. tuberculosis (Mtb) are rare, especially in this case with the prior history of disseminated MAC, therefore further molecular testing to exclude a phantom diagnosis of Mtb is not necessary. STAT3 mutations are associated with autosomal dominant hyper-IgE syndrome (i.e., Job's syndrome), characterized by recurrent candidiasis, bacterial pneumonia, and skin abscesses.

A 25-year-old woman comes to the Emergency Department with a complaint of watery diarrhea for several days but today she noted blood in her stools. She denies undercooked food exposure, antibiotic exposure or travel. She has not had prior intestinal complaints. She is not febrile or orthostatic. Laboratory studies reveal a WBC 13,000, hematocrit 34%, platelets 80,000 and creatinine 2.4 mg/dl. You are most concerned about: Campylobacter jejuni infection Clostridium difficile infection Shiga toxin-producing Escherichia coli infection (STEC) Inflammatory bowel disease Shigella sonnei infection

Correct Answer: Shiga toxin-producing Escherichia coli infection (STEC) STEC infections commonly cause watery diarrhea followed by the onset of bloody diarrhea in association with evidence of an early hemolytic-uremic syndrome (HUS). An elevated WBC is a risk factor for development of HUS with STEC infection. HUS is defined by hemolytic anemia, thrombocytopenia and renal insufficiency. Initiation of this pathogenic process is suggested by the patient's laboratory values. A blood smear will show evidence of hemolysis. C. jejuni is the most common cause of bloody diarrhea in this age group but is not expected to induce the laboratory abnormalities detected in this case. Similarly, although community-associated C. difficile disease is occurring more frequently, this individual lacks risk factors including antibiotic exposure, healthcare facility exposure, infant exposure, pregnancy and/or inflammatory bowel disease. On careful history, acute presentations of inflammatory bowel disease commonly are preceded by earlier, milder intestinal complaints that this individual does not report. Shigella sonnei is the Shigella spp. that most commonly causes diarrheal disease in the United States. Most often these illnesses are characterized by watery diarrhea and do not exhibit hematologic abnormalities. only Shigella dysenteriae, a Shigella species that is not endemic in the United States, expresses classic Shiga toxin that is related to the toxins secreted by STEC.

A 28-year-old male college student is seen at his student health clinic for headache and fatigue one week after returning from Sudan, in Eastern Africa. The patient spent two years in the Peace Corps in Tanzania teaching English. He had unprotected sexual intercourse on several occasions. He returned to the United States 7 days ago and reports that he has had intermittent fevers for one week before leaving Africa. He now feels sluggish with headaches and fatigue that are slowly worsening. On further questioning he recalls that shortly before leaving the area at the end of his tour he noted a 2 cm ulcer on his upper arm. On exam his temperature is 38.2°C. His has a healing scar on his right upper arm, and patches or erythema on his trunk and left thigh- each with a central patch of clearing. Laboratory reveals a Hgb of 11.3 gm/dL; WBC of 3900/mm3 (64%N, 34%L, 2% Eos); platelet count of 100,000/mm3; chemistry values are normal. When he returns 1 week following his first visit to you (two weeks after returning from Africa) he feels worse and now seems to have trouble answering questions. Which of the following tests is likely to reveal the diagnosis? VDRL on CSF Isolator blood cultures for fungi HIV serology Smear of centrifuged CSF MRI of brain

Correct Answer: Smear of centrifuged CSF The patient has an illness compatible with East African trypanosomiasis (sleeping sickness) The illness is transmitted by the bite of the tsetse fly and caused by T. brucei rhodesiense or, in West Africa, by T. brucei gambiense. Those in East Africa have a more severe initial disease. The history of the "chancre" that he reports to spontaneously resolve is helpful in the diagnosis of East African trypanosomiasis. A patchy erythematous rash is also a common finding on exam. The progressive neurological decline requires a spinal tap where the parasite can be seen on wet mount of centrifuged specimen of CSF. It is also diagnosed by finding the trypomastigote on thick and thin blood smears. Rapid card tests (CATT) are available for rapid diagnosis, only for T.b. gambiense and new PCR tests are being developed. MRI of the brain is not diagnostic. Since Lyme disease is not usually seen in Africa where this started, and is not associated with a chancre which this patient had on his arm, it is unlikely the diagnosis. Neurosyphilis and HIV don't explain the chancre and prolonged fever. Here are some photos in case the exam shows a photo clue; two chancres, a geographic map, a blood smear and a table comparing east and West African Trypanosomiasis. Table 1 The principal features of West and East African sleeping sickness disease West African sleeping sickness Trypanosoma brucei gambiense riverine tsetse flies (Palpalisgroup) Insidious onset, slow progression, death in stage II after many months or years Parasitaemia scanty, Winterbottom's sign, serology East African sleeping sickness Trypanosoma brucei rhodesiense savannah tsetse flies (Morsitans group) Acute onset, chancre frequent, rapid course, death frequently in stage I (cardiac failure) Parasitaemia usually higher and easily detectable, serological tests unreliable

A 27-year-old pregnant female presents to the clinic for evaluation. She recently immigrated to the United States and is being evaluated for the first time at approximately 7 weeks of gestation. She is known to be HIV infected and had received nevirapine (alone) prophylaxis during a previous pregnancy. She denies having been on antiretrovirals, other than nevirapine. Her current CD4 count is 480 and her viral load is 10,000. Which of the following would be the best option for this patient? Start her on tenofovir (TDF)/lamivudine and nevirapine.Start her on tenofovir/emtrictabine and darunavir/ritonavir.Start her on tenofovir/emtricitabine/efavirenz.Recommend no therapy currently but recommend the use of zidovudine (AZT plus lamivudine plus lopinavir-ritonavir) at 36 weeks of pregnancy.

Correct Answer: Start her on a tenofovir/emtricitabine and darunavir/ritonavir The keys for pregnancy are as follows: Antiretroviral therapy is recommended in all pregnant women, regardless of virologic, immunologic, or clinical parameters, for the purpose of preventing mother to child transmission. Both reduction of HIV RNA levels and use of antiretroviral therapy appear to have an independent effect on reduction of perinatal transmission. Antiretroviral therapy also benefits the mother, and should be administered for mother's health as well. Genotypic resistance testing is recommended for everyone initiating therapy, including all pregnant women. Tenofovir (TDF)/emtricitabine is among the recommended nucleoside combinations in pregnancy (along with abacavir/lamivudine and zidovudine/lamivudine). However, zidovudine is associated with significant GI toxicity and is given twice daily. Zidovudine related anemia is a particular concern among pregnant women who may develop anemia for reasons related to their pregnancy. Note TAF is not yet recommended for use in pregnancy because of insufficient data on its safety in pregnancy. Darunavir-ritonavir is recommended for pregnancy, (atazanavir/ritonavir also is recommended; raltegravir is also recommended). Thus, regimen B is a good choice. With the history of use nevirapine monotherapy use, drug resistance to both nevirapine (and efavirenz) would be expected and so A and C are wrong. Also, nevirapine would be a poor choice due to her high CD4 count, putting her at high risk for hepatotoxicity. Moreover, nevirapine is seldom used for ART in the United States. Since all pregnant women should receive ART, D is wrong.

You are consulting on a hospitalized 25 y/o female from Kenya with one month of fever, sweats, weight loss. She is ill-appearing and hypotensive. Chest X-ray shows a miliary pattern, HIV testing is positive, and CD4=10 (ten). GeneXpert MTB/RIF is performed on sputum and the result returns as "MTB DETECTED" ,Rif resistance: NOT DETECTED". What is the most appropriate management approach with regard to tuberculosis treatment and HIV antiretroviral therapy? Start tuberculosis treatment immediately, and defer antiretroviral therapy until after tuberculosis treatment is completed at least 6 months from now Start antiretroviral therapy immediately, and defer tuberculosis treatment until after the viral load has decreased by one log Start tuberculosis treatment immediately, and then start antiretroviral therapy in about 8-12 weeks Start tuberculosis treatment immediately, and then start antiretroviral therapy within 2 weeks Rifamycins should not be used to treat tuberculosis in HIV-infected patients

Correct Answer: Start tuberculosis treatment immediately, and then start antiretroviral therapy within 2 weeks There are two issues here, namely whether tuberculosis therapy should be started, and the timing of antiretroviral therapy initiation in this patient with HIV-infection. In this patient with advanced HIV-associated immunosuppression, the diagnosis of tuberculosis has been confirmed by GeneXpert MTB/RIF testing and the clinical presentation is compatible with disseminated tuberculosis, which has a high mortality rate. In patients with HIV, treatment for tuberculosis should be started immediately if tuberculosis is microbiologically confirmed or strongly suspected. With regard to the timing of antiretroviral therapy, clinical trials have demonstrated that, in HIV-infected patients with advanced immunosuppression (CD4<50) initiation of antiretroviral therapy within about two weeks of initiation of tuberculosis therapy has a mortality benefit (compared with later initiation of antiretroviral therapy). Therefore responses A and C are incorrect. In HIV-infected TB patients with CD4 ≥ 50, antiretroviral therapy should be started within 8-12 weeks after tuberculosis therapy. E is not correct because rifamycins should be used to treat tuberculosis in all patients unless there is drug resistance, toxicity, or severe intolerance. Against M. tuberculosis, rifamycins have sterilizing activity that cannot be replaced by other drugs. Rifabutin (a rifamycin) is a weaker inducer of drug metabolic enzymes than is rifampin, and rifabutin can be used with many HIV protease inhibitors.

A 50 y/o 5'10", 278 lb (BMI 40) male is admitted for treatment of weeping bilateral lower leg "cellulitis not responding to outpatient therapy with oral amoxicillin/clavulanate." Leg swelling is of several months' duration but the fluid oozing is of recently origin and worse after standing all day at his job. He recently quit work for that reason. On exam, temp 37.2C with otherwise normal vital signs. Both lower legs and feet are red, edematous, pit slowly to pressure, and minimally tender. The skin has some loose blebs that are oozing sticky fluid. There is an underlying brownish hue to the skin. Pulses are detectable with Doppler. There is evidence of both tinea pedis and onychomycosis. Which one of the following is the most likely diagnosis: Erysipelas Stasis dermatitis Erythrasma Staphylococcal scalded skin syndome

Correct Answer: Stasis dermatitis It may be difficult to distinguish venous stasis dermatitis from erysipelas, and, rarely, the two may co-exist. Stasis dermatitis is a chronic edematous state with inflammation due to the transudation of protein-rich fluid into the interstitium. Erysipelas is an invasive streptococcal infection. Some distinguishing features: Feature Stasis dermatitis Erysipelas Pain Minimal Moderate to severe Bilateral Yes Very, very rare Associated Tinea pedis Yes Yes Lymphangitis/inguinal lymphadenopathy No Yes Potential for toxic shock syndrome No Yes Associated lymphedema Yes Rarely Soft tissue deposition hemosiderin (brown pigment) Yes Yes, if concomitant venous stasis Erythrasma is caused by a bacteria, Corynebacteria minutissimum that fluoresces coral red with a Woods lamp (UV light). Found in inguinal area, confused with Tinea cruris. Lesions are usually well demarcated, dry, asymptomatic, and located in intertriginous areas. Oral erythromycin or topical clindamycin or erythromycin are the usual therapies. Staphylococcal scalded skin syndrome, usually an acute disease of children, would not be confined to the lower extremities or this chronic. . Erythrasma http://dermatologyoasis.net/erythrasma- Erysipelas With erysipelas or bacterial cellulitis, patients are often febrile and have unilateral disease of the lower extremities. Bilateral involvement should lead to consideration of other causes. Cellulitis involves the deeper dermis and subcutaneous fat while erysipelas involves the upper dermis and superficial lymphatics. Cellulitis may present with or without purulence; erysipelas is nonpurulent [1-3]. Patients with erysipelas tend to have acute onset of symptoms with systemic manifestations including fever and chills, whereas patients with cellulitis tend to have a more indolent course with localized symptoms over a few days. In addition, in erysipelas there is clear demarcation between involved and uninvolved tissue which is not the case for cellulitis. . Bacterial Skin Infections: Can You Make the Diagnosis? Carly A. Elston; Dirk M. Elston, MD | April 26, 2016 Staphylococcus scalded skin Staphylococcal scalded syndrome is a reaction to a toxin released by Staphylococcus aureus. This most often occurs in infants and young children but also occurs in adults with renal failure. Fever, malaise, and skin tenderness occur before the eruption. The fragile subcorneal bullae demonstrate the appearance of wrinkled skin with subsequent desquamation Biopsy with examination of a frozen tissue specimen of sloughed epidermis can be used to rapidly distinguish staphylococcal scalded skin from the epidermal sloughing of toxic epidermal necrolysis. Frozen sections taken from staphylococcal scalded skin syndrome demonstrate cleavage at the granular layer The full-thickness epidermal necrosis and cleavage at the dermal-epidermal junction characterize toxic epidermal necrolysis. Staph aureus can usually be cultured from the nasopharynx or conjunctiva. Impetigo, due to Group A streptococci, is clearly more superficial and localized, and not part of this differential.

A 39-year-old man presents to the internal medicine clinic in November with complaints of chronic abdominal pain. He said he has had these abdominal pains for "as long as I can remember." He describes the pain as a dull ache and says that the intensity of it waxes and wanes from day to day. He has normal bowel movements and has not seen blood in his stool. He denies fevers, chills, sweats, or weight loss. He immigrated to the United States from Guatemala when he was 24 years old, and returns for one month every summer to visit friends and family. Routine CBC, blood chemistries, and liver enzymes are all normal. Abdominal CT scan was also normal. The following was visualized on stool ova and parasite exam: Image taken at 100x magnification. Which of the following is the most likely organism? Clonorchis sinensis Enterobius vermicularis Necator americanus Ascaris lumbricoides Strongyloides stercoralis

Correct Answer: Strongyloides stercoralis Chronic vague abdominal pain is a common clinical manifestation of intestinal helminth infections, especially of those that cause some degree of invasion of the intestinal mucosa. These include intestinal nematodes such as Strongyloides stercoralis, Trichuris trichiuria, Necator americanus, and Ancylostoma duodenale as well as intestinal flukes such as Fasciolopsis buski and Metagonimus yokagawi. Liver and biliary flukes can also cause chronic abdominal pain, but would often be associated with abnormalities in liver enzymes or bilirubin levels. The image shows two rhabditiform larvae of Strongyloides stercoralis. Unlike most intestinal helminths, S. stercoralis releases eggs which immediately hatch and are passed as larvae in the stool. Offspring of adult N. americanus, A. lumbricoides, and C. sinensis all exit the human body as eggs, not larvae, in stool specimens. The image above can potentially be confused with adult worms of E. vermicularis, which cause pinworm disease and can sometimes be seen on stool ova and parasite exams. However, pinworms are non-invasive and rarely, if ever, cause chronic abdominal pain. Pinworm is typically diagnosed by visualizing adult worms or eggs on a peri-anal touch prep. In terms of size, male pinworms are generally larger than adult Strongyloides worms. Adult male pinworms average 2-5mm in length and females are 8-13 mm in length. Adult male Strongyloides worms are about 1 mm length and adult females reach 2.5 mm in length.

A 32-year-old woman is diagnosed with bacterial meningitis due to Streptococcus gallolyticus (S. bovis). She has two pet dogs and had lived in rural parts of Southeast Asia as a Peace Corp volunteer over a decade prior. Generally she has been in excellent health, but two weeks before the onset of meningitis she sustained head trauma as the result of a bicycle accident. Five days before she presented with meningitis she was started on high dose corticosteroids to treat a severe case of poison ivy acquired while gardening. Which one of the following is the most likely underlying problem that led to her meningitis? Colon cancer CSF leak Strongyloidiasis Common variable hypogammaglobulinemia Accidental ingestion of dog fleas

Correct Answer: Strongyloidiasis Strongyloidiasis Southeast Asia, skin to larvae in fecally contaminated water asymptomatic, roundworm may complete its life cycle within the human host, infection may persist for years to decades. Steroids may result in dissemination of worms that leave the GI tract carrying along with them GI flora. This may result in gram negative rod bacteremia, and less commonly bacteremia due to other gut organisms including Streptococcus gallolyticus or Enterococcus. However, this patient's young age and the onset with corticosteroids make strongyloidiasis more likely. CSF leak is a high risk factor for bacterial meningitis, but most cases are due to pneumococci or viridans streptococci; S. gallolyticus would be very unlikely. Common variable hypogammaglobulinemia is typically associated with a history of recurrent respiratory tract infections, a history absent in this case. Accidental ingestion of dog fleas may result in human infection with the dog tapeworm, Dipylidium caninum, but meningitis is not seen.

A 66 year old patient in the ICU is day 6 post-operative following a pancreatectomy for pancreatic carcinoma. He is recovering uneventfully with improving renal and hepatic function. On the evening of his 6thpost-operative day, he develops a fever of 38.50C. The surgeons draw three cultures from an indwelling port that was placed preoperatively for chemotherapy that has not yet started. No cultures are drawn through his peripheral IV or percutaneously Piperacillin tazobactam is started. On Day 7 the patient remains intermittently febrile but is otherwise stable with no new findings. Labs are remarkable only for a WBC that continues to decline following surgery and is now 7800 cells/uL with 70% neutrophils The lab calls to say that after 14 hours of incubation, all three cultures are growing gram positive cocci in clusters. By the time the surgeons call you the next morning, the patient has been stable, the port and the peripheral IV look fine, there are no other concerning physical findings or lab values, and the organisms have been identified by MALDI-Tof as Staphylococcus epidermidis. The surgeon is very eager to retain the port. Assuming that there are no unusual resistance patterns, what would you recommend pending the results of follow up cultures: Vancomycin should be started, the port should be removedOxacillin should be started, and the port should be removedThe port can be retained, vancomycin plus lock therapy can be administered for 10-14 days, and the port left in place if the patient is stable with no further Staphylooccus epidermidis bacteremiaThe port can be retained, vancomycin plus lock therapy can be administered for 4-6 weeks, and the port left in place if the patient is stable with no further Staphylooccus epidermidis bacteremiaOnly if an echocardiogram is negative, the port can be retained, vancomycin plus lock therapy can be administered for 10-14 days, and the port left in place if the patient is stable with no further Staphylooccus epidermidis bacteremia

Correct Answer: The port can be retained, vancomycin plus lock therapy can be administered for 10-14 days, and the port left in place if the patient is stable with no further Staphylooccus epidermidis bacteremia This patient likely has a port infection due to Staphylococcus epidermidis. This catheter has a high likelihood of being salvaged. The risk of endocarditis is extremely small with this organism. 10-14 days of therapy should be sufficient. There are excellent tables in the IDSA Catheter Management Guideline https://academic.oup.com/cid/article/49/1/1/369414 Long-term catheters should be removed from patients with CRBSI associated with any of the following conditions, none of which this patient appears to have: severe sepsis; suppurative thrombophlebitis; endocarditis; bloodstream infection that continues despite >72 h of antimicrobial therapy to which the infecting microbes are susceptible; or infections due to S. aureus, P. aeruginosa, fungi, or mycobacteria . For patients with CRBSI for whom catheter salvage is attempted, additional blood cultures should be obtained, and the catheter should be removed if blood culture results (e.g., 2 sets of blood cultures obtained on a given day) remain positive when blood samples are obtained 72 h after the initiation of appropriate therapy . For long-term and short-term CRBSI due to less virulent microbes that are difficult to eradicate (e.g., Bacillus species, Micrococcus species, or Propionibacteria), catheters should generally be removed after blood culture contamination is ruled out on the basis of multiple positive culture results, with at least 1 blood culture sample drawn from a peripheral vein.

The patient is a 56-year-old retired professional football player. His chronic low back pain has gotten worse over the last two weeks and he has fevers to 39oC. A CT scan shows a 4 cm psoas abscess. Admission blood cultures grew methicillin-susceptible S. aureus (resistant only to erythromycin and penicillin, oxacillin MIC = 1 μg/ml, vancomycin MIC = 2 μg/ml). He was initially treated with vancomycin 1 gram IV every 8 hours. On hospital day 4 treatment was changed to nafcillin 2 gm IV q4h. His fever is trending down but still present. Blood cultures are positive on admission and day one and two. No blood cultures were drawn on days three and four. One of two blood cultures obtained on day 5 of hospitalization because of a fever to 38.5°C is reported as positive for Gram-positive cocci in clusters. TEE is negative. Which of the following is the most likely explanation for the positive blood culture? Treatment failure due to emergence of vancomycin resistance on therapy. His PICC line has become contaminated with MSSA . Tricuspid valve endocarditis that was missed on TEE. The undrained psoas abscess. Contamination of the blood culture with coagulase-negative staphylococci.

Correct Answer: The undrained psoas abscess The most common identified reasons for persistent staph aureus bacteremia are 1) an un-drained source of infection and 2) endovascular infection, such as endocarditis. Many patients will have persistent staph bacteremia (MSSA or MRSA) for several days without an identifiable cause, but remedial causes should be sought. Keep in mind that MSSA bacteremia typically persists 3 to 4 days and 30% persist more than 7 days. MRSA bacteremia is at least as long Assuming his antibiotics were given through the PICC line, a contaminated catheter focus is less likely than the psoas abscess as a source of bacteremia. Although echocardiogram can be falsely negative, with a sensitivity of 90% or higher endocarditis is unlikely to be the explanation particularly given a known un-drained focus of infection. Persistent bacteremia is a relatively common occurrence in invasive staphylococcal infection, hence subsequent positive blood cultures are not likely to be due to contamination. Vancomycin and nafcillin resistance are mediated by vanA and mecA, respectively, and do not emerge de novo. The possibility that resistance was missed on the original isolate or is due to co-infection with susceptible and a resistant strains, although theoretically possible, is extremely remote. Persistence bacteremia should always suggest the possibility of inadequate source control.

A 36-year-old woman presents 10 days after being diagnosed and treated for Trichomonas vaginalis infection. Her nucleic acid amplification tests (NAATs) for C. trachomatis and N. gonorrhoeae were negative. She was treated with metronidazole 2g PO x1. Her sex partner also was treated and she has not had any sexual encounters since then. Following therapy, she says that her symptoms persisted. Today, her examination reveals a thin discharge. Vaginal pH is 6.0 and wet mount reveals motile trichomonads. Which is the most appropriate regimen to use at this time? Metronidazole 2g single dose Tinidazole 2g single dose Topical metronidazole gel Topical clindamycin gel Tinidazole 2g daily for 5 days

Correct Answer: Tinidazole 2g single dose The patient has failed therapy with the single 2g dose of metronidazole. This is not likely due to reinfection since she has not had sex. Most likely, she is infected with a strain that has lower susceptibility to nitroimidazoles. Up to 5% of strains in the United States may demonstrate similar characteristics. Tinidazole is a nitroimidazole that has a longer half-life and achieves higher tissue concentrations than metronidazole (but it's more expensive). In this case, after failing the 2g metronidazole regimen, there are two options: metronidazole 500mg PO BID X 7 days, or tinidazole 2 g PO X1 dose. If the patient fails either one of these regimens, metronidazole 2g po QD X 5 days or tinidazole 2g QD X 5 days may be used. Therfore, the second choice is correct. Topical metronidazole or clindamycin gel is not recommended for the treatment of trichomoniasis. Correct Response Tinidazole 2g single dose

A new immigrant from India presents complaining of painless lesions shown below. She has been in the Untied States for one week. Which of the following tests will best allow you to make a specific, etiologic diagnosis? NAAT on urine for Chlamydia Serum RPR Culture of lesion Tissue biopsy Serological titers

Correct Answer: Tissue biopsy Painful ulcers are typical of HSV and chancroid, while painless ulcers are associated with syphilis, LGV, and granuloma inguinale. Since this patient has painless ulcers, start thinking about syphilis, LGV and granuloma inguinale, although in clinical practice such distinctions are not always obvious and valid. This patient has granuloma inguinale or Donovanosis. This infection is caused by Klebsiella granulomatis.Look for friable, beefy red lesions with raised margins or the so called "kissing lesions" due to autoinoculation on the adjacent thighs. The diagnostic modality is tissue biopsywhich allows visualization of dark staining Donovan bodies after Wright-Giemsa staining. These are rod-shaped oval organisms that are seen in the cytoplasm of mononuclear phagocytes. K. granulomatisis very difficult to culture. Serological titersare not used to make a diagnosis of granuloma inguinale. Based on the history and the characteristics of the ulcer, it is unlikely to be syphilis, chlamydia or HSV. HSV ulcers should be painful. Syphilis and chlamydia would present with painless ulcers as with this patient, but not beefy red lesions. Chancroid would be painful with ragged borders.

75 y/o male with diabetes mellitus and ankylosing spondylitis treated with prednisone 20 mg daily, admitted with 3 weeks of fevers to 39°C, lethargy, and weight loss of 10 lbs. He underwent transurethral resection of a bladder cancer three months prior, and recently completed a six-week course of intravesical Bacille Calmette Guerin (BCG) administered once weekly. He lives in Tucson, Arizona. Urinalysis shows protein, nitrite, and leukocytes; routine bacterial culture is negative. Chest X-ray is normal. Chest CT scan shows innumerable tiny (1-4 mm) nodules. What diagnostic procedure is most likely to reveal the diagnosis? Bacterial blood culture Silver stain of induced sputum Ziehl-Neelsen stain of induced sputum Trans-bronchial biopsy Serum antibody testing for Coccidioides

Correct Answer: Trans-bronchial biopsy The presentation and history are most consistent with disseminated BCG infection as a complication of intravesical administration. Routine bacterial blood cultures are not incubated sufficiently long enough to detect growth of Mycobacterium bovis BCG. When mycobacterial infections present with a miliary pattern on chest imaging, the bacillary burden in respiratory secretions typically is very low; Ziehl-Neelsen staining of sputum has very poor sensitivity, and is not likely to be positive in this context. Multiple respiratory specimens should be sent for mycobacterial culture, which is the most sensitive approach for microbiologic confirmation of mycobacterial disease, although sensitivity is less than 100% in miliary disease. Pneumocystis jiroveci is in the differential diagnosis in this patient on corticosteroids, although a miliary pattern on chest imaging would be very unusual for PJP, and molecular nucleic acid amplification tests of induced sputum are substantially more sensitive than silver stains. Coccidiomycosis is a consideration given the patient's residence in Arizona and it can present with a miliary pattern, usually in AIDS patients. However, serum antibody testing for Coccidioides can be falsely negative early in disease and in immunosuppressed patients. A positive serology might represent remote infection as the serology can remain positive for years.

Three United States Special Forces officers reported to sick call with a similar complaint of moderately severe, diffuse muscle pain, low-grade fever and malaise. Physical examination was normal except for diffuse muscle tenderness and some puffiness around the eyes. Routine laboratory work was normal except for total eosinophil counts of 700-1000/cu mm and a CPK twice the upper limit of normal. All three patients had been on a joint training mission in Brazil, during which they had spent nights in the jungle, waded in streams and had numerous insect bites. At the end of the mission they had eaten at a barbecue of roast pig with other local foods. Mefloquine was used for malaria prophylaxis. Symptoms began approximately two weeks after their return to the United States. The most likely cause of their illness would be which of the following? Trypanosoma cruzi Trichinella spiralis Leishmania brasiliensis Wuchereria bancrofti Leptospira interrogans

Correct Answer: Trichinella spiralis eating undercooked pork. encysted nematode larvae in muscle tissue. variety of serologic tests becomes positive beginning the third week after exposure, as symptoms are waning spontaneously. Mebendazole or albendazole, if effective at all, has to be begun in the first few days of symptoms. Steroids may be useful in severe cases in addition to the antiparasitic agent. None of the other infectious agents listed causes muscle pain and eosinophilia.

A 22-year-old, HIV-negative, Korean male presents with two years of painless purulent drainage and progressive hearing loss in his right ear which is now profound. He has been treated numerous times with various antibacterial drugs for Staph aureus and Pseudomonas without improvement. He is afebrile and denies anorexia, night sweats or weight loss. There are no remarkable physical findings or laboratory findings except for extensive granulation tissue and drainage in his right external auditory canal and a large tympanic membrane perforation. Imaging of his chest is normal. Brain CT scan indicates he has partial opacification of the right mastoid air cells. What is the most likely cause of this chronic otitis? Granulomatosis with polyangiitis (Wegener's) Syphilis Aspergillosis Tuberculosis Polychondritis

Correct Answer: Tuberculosis Chronic suppurative otitis bacterial chronic perforation of the tympanic membrane chronic mastoiditis and responds to antibiotics when given along with appropriately aggressive surgery. Cholesteatomas and infected ossicles require removal and mastoidectomy is often required. Failures should raise the possibility of fungal or mycobacterial infection, including tuberculosis. Fungal otitis is usually caused by Aspergillus and is not invasive, except in neutropenic patients. Granulomatosis with polyangiitis (Wegener's granulomatosis) can cause a similar picture but absence of disease in the lung, paranasal sinus or kidney makes this less likely. TB otitis is usually unilateral, relatively painless, and progresses over many months. The tympanic membrane shows a large perforation or multiple small perforations. Mastoid air cells may be partially pneumatized. Preauricular adenopathy or ipsilateral facial palsy may occur. As with chronic bacterial otitis media, extension to the petrous pyramid or temporal lobe of the brain may occur. Keep in mind that while 50% have evidence of TB on chest imaging, 50%...don't. Diagnosis can be established by smear and culture (and probably PCR) of otic canal drainage or biopsy of granulation tissue. This entity is readily treatable.

A 21-year-old female college varsity track and field athlete noted onset of right knee pain and swelling for three weeks that has persisted despite taking anti-inflammatories, rest and icing. She feels otherwise well without fever and denies any trauma or prior history of joint swelling. An MRI of the knee showed no meniscal tears and mild synovial thickening. An orthopedist aspirated the joint last week revealing 18,000 WBC/ml with PMN predominance. No crystals were identified and cultures were negative. She is a resident of Massachusetts and attends school in South Carolina. She has no prior past medical history including sexually transmitted diseases, and doesn't recollect any history of rash consistent with erythema migrans or history of tick bites recently. Her exam in normal with the exception of her right knee that has some limited range of motion and a moderate effusion. Which selection below would be most supportive of a diagnosis of late Lyme arthritis? Therapeutic trial of antibiotics (doxycycline or ceftriaxone) Synovial fluid B. burgdorferi immunoblot (IgM or IgG) Synovial fluid culture for Borrelia spp. Two tier B. burgdorferi serology, including IgG immunoblot

Correct Answer: Two tier B. burgdorferi serology, including IgG immunoblot Late Lyme arthritis almost always involves the knee and is a recurrent mono- or oligo-articular arthritis, although some patients will present with the first bout of knee swelling as in this case. Patients are not typically ill but experience local complaints attendant to the involved joint. Synovial fluid is typically inflammatory with WBC counts averaging 24,000 cells/mL with neutrophil predominance. Two tier B. burgdorferi serology is nearly 100% positive in patients with Lyme arthritis, and with a compatible clinical picture and epidemiology as in this case is enough to implicate and prompt treatment. Although synovial fluid B. burgdorferi PCR is highly specific, it is less sensitive in detecting the pathogen than customary serology. This test was not offered as a choice here: a positive result would be supportive: a negative result would not rule out Lyme disease. Culture for Borrelia is difficult and not routinely performed; even in research laboratories, late Lyme arthritis rarely yields positive culture results. Use of immunoblot testing in fluids other than serum (such as synovial fluid or CSF) is not validated, and results should not be viewed as a reliable means for diagnosis alone. Lastly, about 10-15% of patients have persistent inflammatory Lyme arthritis despite antibiotics, which appears to be on a pathogen-precipitated autoimmune basis. Therefore, a subset of patients will not fully respond to antibiotics alone and may require DMARDS (disease modifying anti rheumatic drugs, i.e. methotrexate, leflunomide, plaquenil), intra-articular steroids or synovectomy.

Over a 7-day period, 4 patients in your 250-bed community hospital have Pantoea (formerly Enterobacter) agglomerans bacteremia. Quick record review shows no prior episodes in the past 12 months and only the four bacteremias (i.e., no other sites positive in bacteremic patients); the lab is very confident in their microbiology work-up. Which of the following is most likely to provide insight into the cause of this outbreak: Obtain pulse-field gel electrophoresis (PFGE) of the four patient isolates Begin an environmental culture survey on the affected wards Obtain daily blood cultures (active surveillance culturing) of all patients on the affected wards Undertake a case-control study Check the four patients' charts for common medications

Correct Answer: Undertake a case-control study A case-control study is the most helpful step to identify the source of this outbreak of "primary bacteremia". A cluster of primary bacteremias, i.e., no other infection sites positive in bacteremic patients, suggests infection by a contaminated infusate, a product administered intravenously, or an intravascular device. Checking the charts for common medications is likely to find numerous identical medications that other patients not affected were also receiving. PFGE is most useful with more common isolates, to determine clonality. Although PFGE likely would be performed on these isolates, the time-space tightness of this cluster and the lack of prior isolates of this pathogen suggest a clonal outbreak. Environmental cultures may be helpful once there are epidemiologic clues to a source. Blood culture surveillance in afebrile patients is unlikely to be helpful with a relatively pathogenic gram-negative bacillus.

Which of the following has the strongest and most consistent evidence linking it to Mycoplasma genitalium infection? Cervicitis Male infertility Pelvic inflammatory disease Proctitis Urethritis

Correct Answer: Urethritis The 2015 CDC STD Treatment Guidelines feature a new section on M. genitalium, an emerging infection. The guidelines suggest that although strong evidence has linked M. genitalium to urethritis in men, it remains unknown whether this infection can cause male infertility. The organism has been detected in men with epididymitis in a limited number of cases, but this has not been extensively investigated. Similarly, M. genitalium has been found in the rectum, but detection is infrequently accompanied by rectal symptoms, and its presence does not appear to cause a syndrome of clinical proctitis. At this time, the data linking M. genitalium infection to cervicitis are moderately strong, yet not as consistent as those linking it to urethritis.

A 75-year-old male with a history of diabetes mellitus, hypertension, and coronary artery disease presents with a 2-day history of fever and headache. One day prior to admission, his family noted that he was lethargic. On presentation, temperature was 102°F, and he was unresponsive. Neck was supple and there were no focal neurologic abnormalities. WBC 25,000 (30% bands). In the emergency room, the patient was treated empirically with vancomycin, ampicillin, ceftriaxone, and acyclovir. He was then sent for a non-contrast emergent CT scan of the head, which was negative. Four hours later, the patient underwent CSF analysis, which revealed an opening pressure of 300 mm H2O, WBC 2,200/mm3(98% segs), glucose 30 mg/dL, and protein 200 mg/dL. CSF Gram stain revealed gram-positive diplococci. What is the best regimen at this point? Dexamethasone + vancomycin + ceftriaxone + ampicillin Dexamethasone + vancomycin + ceftriaxone Dexamethasone + vancomycin Vancomycin Vancomycin + ceftriaxone

Correct Answer: Vancomycin + ceftriaxone The patient presents with symptoms and signs of meningoencephalitis. Given the significant alteration in consciousness at the time of presentation, it was determined that the patient should undergo a CT scan of the head prior to lumbar puncture. Prior to sending the patient for CT scanning, he was treated empirically with antimicrobial therapy directed against the likely causes of bacterial meningitis (based on the patient's age and underlying disease status) and for herpes simplex encephalitis. Once the CSF analysis was performed, the patient was diagnosed with pneumococcal meningitis. The recommendation for treatment of pneumococcal meningitis in adults would be to administer adjunctive dexamethasone in combination with vancomyin and a third-generation cephalosporin (either cefotaxime or ceftriaxone). However, since the diagnosis was not made until more than 4 hours after administration of the first doses of antimicrobial therapy, adjunctive dexamethasone should not be administered because it is unlikely to offer benefit in terms of morbidity and mortality. When used, adjunctive dexamethasone should be administered concomitant with or just prior to the first dose of antimicrobial therapy. At this point, the patient only needs to receive antimicrobial therapy for treatment of pneumococcal meningitis caused by highly penicillin- or cephalosporin-resistant strains (i.e., vancomycin + a third-generation cephalosporin), pending results of in vitro susceptibility testing.

A 19-year-old female in her 10th week of pregnancy was seen in the obstetrics clinic for routine follow-up when the history was elicited that she had taken care of her friend's child all day three days prior and that child had now developed chickenpox. The patient was a recent immigrant from rural Nigeria, had no recollection of chickenpox or shingles and no knowledge that she had ever been immunized or exposed to chickenpox previously. The OB clinic has called to see if anything should be given as post-exposure prophylaxis. You cannot obtain a varicella titer for at least 3 more days since this call comes...Friday afternoon. Your advice is which of the following: Nothing Chickenpox vaccine Varicella zoster immune globulin Varicella zoster immune globulin plus chickenpox vaccine Intravenous immune globulin (IVIG)

Correct Answer: Varicella zoster immune globulin About 72 hours prior, this pregnant female was exposed to a child capable of transmitting chickenpox for a sufficient period of time (defined as more than one hour indoors) and she has no history to suggest immunity. She is at risk of developing severe disease and of transmitting the virus to her child causing congenital varicella syndrome. The CDC recommendation is to give susceptible pregnant women or immunosuppressed patients a single dose of VariZig, varicella immune globulin within 10 days but preferably within 96 hours of exposure. A pretreatment blood should be obtained to assess whether the patient should be vaccinated after delivery, at least 5 months after VariZig. Valacyclovir, a Class B agent, is not recommended for post-exposure prophylaxis during pregnancy but is usually given for chickenpox during pregnancy because of the high incidence of complications, including chickenpox pneumonia. The chickenpox vaccine is a live virus vaccine and is not recommended during pregnancy. If this woman were not pregnant, the vaccine would be effective if administered within 3-5 days of exposure. It should not be given at the same time as VariZig. Intravenous immune globulin may have some efficacy, and was used during a period when no varicella immune globulin was licensed in the United States, but is not the treatment of choice.

A 56 year-old man is admitted with 3 days of fever, rash, confusion, and leg weakness in August. He lives in Alabama. He is febrile to 104°F and is confused. He has a faint maculopapular rash in the trunk. His neurologic exam reveals mild tremors, a strength of 5/5 in the arms, 3/5 in the lower extremities, normal deep tendon reflexes, and normal sensory exam. CBC, chemistry tests, urinalysis, chest-X-ray and head CT without contrast are normal. HIV test is negative. A lumbar puncture reveals 1000 WBC, 70% polys, 30% lympho/monos, protein 81 mg/dL, glucose 92 mg/dL. Gram stains and Cryptococcal antigen are negative. He is treated with vancomycin and ceftriaxone. Over the next three days the weakness progresses to the point that he is paraplegic. Which of the following is the most likely diagnosis? Poliovirus West Nile virus Saint Louis encephalitis virus Acute HIV infection Guillain-Barre syndrome

Correct Answer: West Nile virus West Nile virus seen in Africa, the Middle East, parts of Europe and Asia, Australia, and in recent years has become endemic in the continental United States. seasonal incidence, peak in late summer or early fall. The virus is transmitted by the Culex mosquitoes, blood transfusion, organ transplantation, transplacental transmission or breastfeeding. 80% of the infections are asymptomatic. West Nile fever is seen in 20%, and is characterized by nonspecific symptoms such as fever, headache, malaise, back pain, myalgias, anorexia, or in some outbreaks, a faint rash. Neuroinvasive disease is seen in 1 out of 150 individuals who are infected. The neurological manifestations include encephalitis, meningoencephalitis, meningitis, poliomyelitis (acute flaccid paralysis). In endemic areas, flaccid paralysis or weakness with or without encephalitis is very suggestive of West Nile virus. diseaseis based on detection of IgM antibody in CSF. Viral cultures and PCR become negative too early to be useful in previously normal patients; initial serology can be negative so it is important to check convalescent titers. Risk factors for severe disease are older age, diabetes, alcohol abuse, and being immunocompromised. Poliovirus has been eradicated from the United States. Acute HIV infection can occasionally present with a Guillain-Barre-like syndrome (GBS). GBS is an acute immune mediated polyneuropathy, with progressive ascending weakness and albumin-cytologic dissociation where the CSF protein is very high and the cell count is not. St. Louis encephalitis virus is one of etiologic agents of viral encephalitis in the United States, and presents as encephalitis, not as poliomyelitis.

56 year old woman presents with shortness of breath, fever, and CXR with bilateral interstitial infiltrates. Bronchoscopy reveals cysts of Pneumocystis and she is started on trimethoprim-sulfa and corticosteroids. Subsequent HIV antibody test is positive, CD4 is 105 cells/uL, and HIV RNA is 213,000 copies/ml. She improves over the next 2 weeks and is started as an outpatient on tenofovir (TDF)/emtricitabine and atazanavir/ritonavir. Two weeks later, she develops jaundice, and labs reveal AST 32 IU/L, ALT 45 IU/L, alkaline phosphate 134 IU/L, bilirubin 3.7 (direct 0.4) mg/dL. What is the most likely cause of her jaundice and hyperbilirubinemia? trimethoprim-sulfa tenofovir (TDF) emtricitabine atazanavir ritonavir

Correct Answer: atazanavir This patient developed jaundice with hyperbilirubinemia (all indirect) which is a known effect of atazanavir, due to its inhibition of the hepatic UGT1A1 enzyme. Atazanavir-associated indirect hyperbilirubinemia is intermittent and NOT associated with other liver function test abnormalities, but the jaundice may cause concerns for patients, occasionally requiring a change in ART.

A 36-year-old woman developed a productive cough with low grade fevers in August. Over the next 2 months, she received several courses of antibiotics without improvement. In October, she presented to the hospital with higher fevers and worsening cough. A chest radiograph revealed bilateral scattered infiltrates worse in the upper lobes. CT of chest revealed bilateral nodular infiltrates and several small cavities bilaterally. Exam: T-39.4C; P-110/min; BP102/66 mmHg She appeared toxic with a productive cough producing thick gray sputum. Chest had diffuse rhonchi and scattered crackles bilaterally most prominent in the upper lung fields. The remainder of exam was normal. Labs: WBC- 16,000 (84%PMN, 16%Lymphs); Hgb- 9.8Gm/dL; Plt-426,000/mm3 Creatinine 2.9mg/dL; BUN-48mg/dL ; AST/ALT- 64/72U/dL; Urinalysis showed 3+ hematuria with red cell casts on the microscopic exam ESR-126mm/hr; CRP-36; ANA-negative; Anti-DNA -Negative C-ANCA ( antiProteinase 3)- positive. Microbiology data: stains and cultures of sputum are unrevealing for bacteria, fungi, and mycobacteria. This patient most likely has which of the following: Sarcoidosis Tuberculosis Anti-glomular basement membrane antibody (Goodpasture's) disease granulomatosis with polyangiitis (Wegener's) Mycobacterium avium complex disease

Correct Answer: granulomatosis with polyangiitis Granulomatosis with polyangiitis (GPA) is a multi-system disease due to small-vessel vasculitis. Pulmonary abnormalities on imaging are common and include nodules, patchy infiltrates, cavitary lung lesions, and mediastinal lymphadenopathy. GPA often mimics other granulomatous diseases, however renal involvement, and specifically red cell casts diagnostic of acute glomerulonephritis, is not seen in these disorders. In contrast, glomerulonephritis is a common finding in Anti-GBM antibody disease. Pulmonary involvement with anti-GBM antibody disease is usually associated with hemoptysis and patchy pulmonary infiltrates, and fever and other systemic symptoms are uncommon with this disease. Laboratory findings supportive of GPA include a positive Anti-Neutrophil Cytoplasmic antibody of the Proteinase 3 type.

A 54-year-old female from San Antonio, Texas, presented with a two-day history of scattered vesicles on her trunk and face and both upper extremities, together with a low grade fever and malaise. The vesicles look like chickenpox except that they are all in the same stage of development. She worked as a dental assistant but had just returned a week prior to onset from a vacation in the Yucatan, Mexico. She was otherwise well, taking no medications, had three young children in the home, her husband was well and she had a clear history of chickenpox as a child. An important part of her visit would be to inquire about which of the following. new sexual partners cold sores in her mouth or on her genitals new pets in the home exposure to wool rugs contact with children who had chickenpox

Correct Answer: new pets in the home The increasing acquisition and trading of exotic pets among the United States public is creating new risks for pet owners, including a well-documented outbreak of monkeypox from pet prairie dogs exposed to an imported Gambian giant rat, an illness which this patient could have. Monkeypox lesions are generally in the same stage of development, in contrast to varicella; the patients are rarely very ill, in contrast to smallpox. A clear history of childhood chickenpox makes this patient very unlikely to have chickenpox now. Though she is at risk of reactivation of varicella zoster virus infection as shingles, the failure to begin with a dermatomal distribution would be very unusual for shingles. Herpes simplex can cause focal collection of lesions in persons who have eczema or wrestlers who have multiple areas inoculated from an open lesion. Sexually transmitted diseases do not present at scattered vesicles. Allergy to an alternative medicine could cause erythema multiforme but the lesions in this patient were scattered vesicles, not bullae in the center of erythematous macules.

A 23-year-old healthy female develops severe abdominal pain and vomiting several hours after eating sushi prepared from raw wild-caught Pacific salmon. Upper endoscopy revealed a 2 cm long nematode embedded in the gastric mucosa. The patient was most likely infected with: Dracunculus medinensis Trichinella spiralis Taenia solium Toxocara canis Anisakis simplex

Correct answer : Anisakis simplex Anisakis simplex is a zoonotic infection acquired by consumption of raw or undercooked fish. After ingestion, the larvae of A. simplex typically burrow into the gastric or small intestinal mucosa, causing an acute illness with symptoms such as abdominal pain, fever, vomiting, and diarrhea. Recently, allergic reactions to A. simplex ingestion have been increasingly reported, with patients developing wheals, hives, and anaphylaxis after eating Anisakis-contaminated fish. Cases of Anisakiasis have been reported from Japan, Europe, Africa, and North and South America. A. simplex is present in many types of ocean fish, including wild-caught Pacific salmon. While infection of fish with this parasite is common, live infections from eating sushi are uncommon as freezing of fish (which is recommended by the FDA and done by most sushi restaurants) typically kills these nematodes. Of note, freezing does not prevent allergic reactions to Anisakis.

A 34-year-old female with diabetes mellitus presents with fever, chills, and left flank pain, Her urinalysis shows many neutrophils. A flat plate of the abdomen is shown, below with findings in her left kidney. The most likely cause of this syndrome is: Aspergillus fumigatusBacteroides fragilisEscherichia coliMethicillin resistant Staphylococcus aureusClostridium septicum

Correct answer Escherichia coli This patient has emphysematous pyelonephritis, with gas in the renal collecting system, parenchyma, and renal vein. This syndrome is associated with necrosis of tissue. This syndrome is most often due to E. coli or Klebisella, and most likely occurs in patients with urinary tract infections and diabetes mellitus or urinary tract obstruction. Air may also be seen in the renal pelvis or bladder. In the past, nephrectomy was indicated, but antimicrobial therapy and relief of obstruction (if any) can salvage some patients.

The effect of iron overload on the prevalence of infection with a specific bacterium was estimated by comparing the prevalence of infection in patients who did not have iron overload, using a national database of hospital discharge diagnoses. The prevalence of the infection in patients with iron overload was 2 per 1000 persons. The prevalence of the infection in those without iron overload was 0.5 per 1000 persons. The hazard ratio of iron overload for this infection was which of the following: 4 2.5 2 0.4 0.2

Correct answer: 4.0 The hazard ratio is the 1/prevalence without iron overload divided by 1/prevalence with iron overload, or (1/0.005)/(1/0.002) or 2000/500=4.

A new diagnostic test for streptococcal pharyngitis yielded 20 patients with a true positive result, 90 patients with a true negative, none with a false negative and 10 with a false positive. The specificity of this test is which of the following: 100% 90% 82% 50% 33%

Correct answer: 90% Specificity is the proportion of uninfected patients who did not have a positive test result. The test measures the proportion of negatives who are correctly identified as such, i.e., in this example, 90% of healthy people are correctly identified as not having streptococcal pharyngitis. Specificity is calculated as all the true negatives divided by all the true negatives and false positives, or 90/(90+10) or 90.

Which of the following is the most likely indication of extension of a mycotic aneurysm of the sinus of Valsava (i.e. interventricular septal abscess development) in a patient with native valve Staph aureus aortic valve endocarditis? A) A-V Block B) Atrial Fibrillation C) Ventricular Fibrillation D) Sinus tachycardia

Correct answer: A These EKGs show the following patterns: A) A-V Block B) Atrial Fibrillation C) Ventricular Fibrillation D) Sinus tachycardia When the infectious process of endocarditis spreads beyond the valve leaflets, mycotic aneurysm of the sinus of Valsalva may form, which can rupture into the interventricular septum and injure the AV node, leading to complete heart block. Although mycotic aneurysms of the sinus of Valsalva can involve other coronary cusps, rupture into the septum most often results from aneurysms in the noncoronary cusp. Transesophageal echocardiogram is the most reliable imaging technique to detect aneurysms of the sinus of Valsalva. Surgical repair of a ruptured mycotic aneurysm, usually with aortic valve replacement, can be lifesaving.

Which of the following is true about nucleic acid amplification (NAA) tests for tuberculosis when used to assess respiratory samples? False positive tests occur in approximately 12% of tests Specimens with negative acid fast smears are rarely positive Mycobacterium avium-intracellulare can be detected in sputum with this test A negative test on a smear-positive specimen suggests a Mycobacterium other than M. tuberculosis complex Use of radioisotopes in this test has restricted use to specialized laboratories

Correct answer: A negative test on a smear-positive specimen suggests a mycobacterium other than tbc NAA tests have a greater sensitivity in AFB smear positive sputums than AFB smear negative sputums. NAA tests that are positive precede the culture results by many weeks. Unlike sputum smears, they are also able to differentiate M. tuberculosis from most other species of Mycobacteria. The most recent DNA test, Xpert® MTB/RIF, not only provides the result in hours, but is almost as sensitive as culture and provides rifampin resistance information. False positive tests are rare. False negative NAAT are common in smear negative samples but sensitivity is still adequate to warrant use. Radioisotopes are not employed.

This fundoscopic picture was obtained from a 45-year-old patient who had been receiving parenteral alimentation following complicated abdominal surgery for complications of lymphoma. He had been receiving high dose prednisone for hemolytic anemia but not currently receiving cytotoxic drugs. The most likely cause is related to: An oocyst found in cat stoolAn acid fast organism found in sputum from a patient with fever and coughA yeast often found in normal oral floraA worm egg found in human fecesA virus found in normal oral secretions

Correct answer: A yeast often found in normal oral flora This is Candida endophthalmitis that has extended from the retina into the vitreous humor as vitreous abscsses (white balls) surrounded by intense vitritis (cloudiness). Infection follows candidemia, usually from an intravenous catheter, in hosts that are usually not neutropenic. Patients with Candida endophthalmitis note clouding or floaters in one eye initially, if they are not intubated or too ill to notice. Candida is part of the normal flora of humans. Toxoplasma acquired from cat stool, tuberculosis or Staphylococcus aureus can cause endophthalmitis, but not vitreous abscesses. Herpes viruses include herpes simplex, herpes zoster and cytomegalovirus, all of which can cause retinal diseases, but none with vitreous abscesses. Toxocara canis from ingesting a worm egg canrarely causeretinal lesions but not vitreous abcesses.

A 32‐year‐old woman in Wisconsin who worked as a farmer, presents to her family physician in June with a 3‐week complaint of a low‐grade fever, myalgias, a productive cough, and a skin lesion on her arm that developed during the previous week. The physician found an infiltrate on chest x‐ray, collected sputum and a biopsy from the skin lesion for culture and histopathology. The organisms shown here were seen when the tissue was stained with a silver stain (thick walled structure, broad based budding yeast). After 14 days, a mold grew in the fungal culture. Which of the following is the most likely pathogen? A. Blastomyces dermatitidis B. Candida albicans C. Coccidioides immitis D. Cryptococcus neoformans F. Histoplasma capsulatum

Correct answer: A. Blastomyces dermatitidis This patient has blastomycosis with dissemination to the skin, a common site of dissemination. Blastomyces species is a dimorphic fungus with yeast cells observed in tissue and a filamentous mold when grown in culture. The large yeasts are characterized by a thick wall and a broad base when budding is observed. Candida and Cryptococcus species are not dimorphic and grow as yeasts both in tissue and culture. Coccidioides and Histoplasma species are dimorphic fungi; in tissue, Coccidioides species appears as non‐ budding spherules and Histoplasma species as yeast with narrow based buds; both grow as a mold in culture.

A 22‐year‐old male computer engineer was found to be HIV‐infected during a routine ER visit for minor trauma. (CD4 count = 450 cells/uL, VL = 50,000 copies/uL). He was referred to HIV clinic and started on efavirenz, tenofovir, and emtricitabine‐the clinic preferred this regimen over the current recommended regimens. His most recent sexual contact was 90 days ago with an anonymous partner. He was RPR negative at that time and again when he started ART. Nine days after starting therapy, he developed a mildly itchy rash over his trunk, face and extremities including palms and soles. He has no blistering lesions and no lesions on his oral mucosa. A. Continue the ART regimen with no other intervention B. Switch the efavirenz to darunavir‐ritonavir; no other therapy C. Switch tenofovir‐emtricitabine to zidovudine‐lamivudine; no other therapy D. Continue ART and treat with benzathine penicillin

Correct answer: A. Continue the ART regimen. This pruritic rash is most likely due to efavirenz (NNRTI). Rashes due to efavirenz are common (~10% of patients starting the drug), usually mild and self‐limiting, and occur at a median time of 9 days after starting therapy. Progression to Stevens Johnson is extremely uncommon with efavirenz, and this patient has no such suggestive findings. Rashes to tenofovir and emtricitabine are far less common. A rash on the palms and soles could be syphilis, but the pruritus and the two negative RPRs make this less likely. Manifestations of syphilis occur at a median time of 21 days, and thus this would be unusually late, occurring 90 days after his last exposure.

A man in his seventies with a bioprosthetic aortic valve presents with fever, dyspnea and anorexia. He lived in the Northeast US and had no recent travel. On exam, his temperature was 101 F. There was a non‐tender lesion on his thumb (below). Janeway There were no other lesions. Which of the following is most likely? A. Endocarditis B. Herpes zoster C. Tache noir (Rickettsial infection) D. Fusariosis -not immunosupprsed E. Herpetic whitlow

Correct answer: A. Endocarditis Discussion: Multiple blood cultures were positive for Staphylococcus aureus. An echocardiogram revealed a vegetation on the prosthetic aortic valve. The patient has a Janeway lesion in the setting of endocarditis

Patients with pulmonary cavities and this xray (fungus ball) are most prone to which of these serious complications: A. Hemoptysis B. Brain abscess C. Spread to contiguous bone D. Spread within the lung E. Empyema

Correct answer: A. Hemoptysis A pulmonary fungus ball due to Aspergillus—or aspergilloma—is a solid mass of hyphae growing in an existing lung cavity. Usually, Aspergillus fungus balls of the lung develop in preexisting cavities in the pulmonary apex of patients with chronic lung disease such as bullous emphysema, sarcoidosis, or sequelae of tuberculosis. Detection of Aspergillus in sputum cultures or high titers of Aspergillus antibodies are additional evidence that the radiographic findings are consistent aspergilloma. A biopsy is not necessary except to diagnose the underlying lung disease, if needed. Definitive therapy is surgery for lobectomy, although such procedures can be hazardous due to pleural fibrosis and tenuous pulmonary status. Control of hemoptysis may be accomplished through embolization in many cases. Massive hemoptysis is an infrequent but potentially lethal complication of fungus ball of the lung. Spread of the fungus, usually Aspergillus, beyond the cavity is extremely rare. Empyema is an occasional complication of surgical removal of a fungus ball.

An 18‐year‐old woman in Portland, Oregon presented to an acute care office because of a 4‐day history of redness and pain in her upper ear. The pain began two days after an ear piercing performed at a shopping mall. She saw a pediatrician two days prior and was given cephalexin prescription. Although compliant with the prescription, the ear piercing site has become increasingly painful. She was otherwise healthy and has no history of boils or other notable skin infections. On examination she was afebrile. There is marked erythema, tenderness and swelling on the upper margin of her left ear around a small gold stud. No pus was seen or could be expressed by gentle pressure on the wound site. There was no local adenopathy. The stud was removed. Based on the most likely etiologic organism, which one of the following oral drugs would you add to her empiric antibiotic therapy? A. add ciprofloxacin B. add metronidazole C. add valacyclovir D. change cephalexin to amoxicillin‐clavulanate E. Add Trimethoprim‐sulfamethoxyzole

Correct answer: A. add ciprofloxacin The two most worrisome possibilities for this infection, which is progressing despite cephalexin, are methicillin‐resistant Staphylococcus aureus (MRSA) and Pseudomonas aeruginosa. MRSA would typically lead to abscess formation, but it can take a while if starting in the cartilage. Ps.aeruginosa has a predilection for cartilage of the pinna as exemplified by malignant external otitis in diabetics. Ps.aeruginosa is a recognized contaminant of antiseptic solutions applied to the ear prior to piercing. Ciprofloxacin would be useful due to its predictable activity vs Ps.aeruginosa. The more problematic issue is empirical management of MRSA in communities where this is common. Clindamycin or trimethoprim‐sulfamethoxazole could be considered for that possibility. AS above, the absence of purulence makes S.aureus less likely. The activity of TMP/SMX vs S. pyogenes is debated. The choice would be simpler if there was a suitable specimen for culture and sensitivity. Metronidazole to cover anaerobic infections would not be useful in a highly vascular site such as the ear rim. Herpes simplex virus would be so rare in this site that acyclovir would not be helpful. Changing cephalexin to amoxacillin‐clavulanate would not provide coverage for the likely organisms here which are pseudomonas or MRSA. The clinical presentation is not consistent with relapsing polychondritis, a systemic disease that affects cartilage, rather than soft tissue, and which usually presents symmetrically. Inflammation and tenderness of the ear cartilage would be consistent with relapsing polychondritis, as would involvement of the cartilage of the nose, respiratory tract, sternum, and other areas.

A 56‐year‐old man six months post‐heart transplant complicated by steroid‐induced diabetes and transplant rejection, requiring rituximab three months prior, sustained a laceration on the lower leg after cutting his leg on a lawn mower blade. Three days later, he developed erythema of the tissues surrounding the lesion associated with increasing pain and fever. He was started on Vancomycin empirically. Despite the vancomycin, the cellulitis progressed and he developed a black eschar over the wound (see image). There was no crepitus. A punch biopsy of the wound was performed in the emergency department and the KOH preparation of the tissue biopsy is shown below The best therapeutic intervention, in addition to debridement, would be: A. Intravenous liposomal amphotericin B alone B. Intravenous voriconazole alone C. Intravenous liposomal amphotericin B plus deferoxamine D. Intravenous voriconazole plus deferoxamine E. Oral posaconazole alone

Correct answer: A. amphotericin B alone This is cutaneous mucormycosis which is caused by fungi from the subphylum Mucoromycotina. Note the broad hyphae and absence of septae. This calcofluor stain under UV light would show septae very clearly but none are present. Mucoromycotina are ubiquitous in nature, and can be found on decaying vegetation and in the soil. Infection of the skin and soft tissues usually results from inoculation of the fungal spores into the dermis and accordingly is almost always associated with trauma or wounds. Cutaneous mucormycosis usually appears as a single, painful, indurated area of cellulitis that develops into an ecthyma‐like lesion. Patients who have suffered trauma with an open wound that was contaminated with spores can develop rapidly progressive tissue necrosis reflecting the presence of ischemic infarction. Dissemination and deep tissue involvement are unusual complications of cutaneous mucormycosis, but need to be considered in the immunocompromised host. Radiographic imaging is useful to help determine the extent of involvement, but should not delay surgical intervention. Treatment of mucormycosis involves a combination of surgical debridement of involved tissues and antifungal therapy. Amphotericin remains the drug of choice for mucormycosis; most experts would use a lipid formulation to deliver high doses with less nephrotoxicity, particularly in a transplant recipient who is likely also receiving a calcineurin inhibitor. Posaconazole could potentially be used in the intravenous form or the new tablet formulation, however, the in vitro activity of posaconazole is not predictable. Further, posaconazole is probably not as effective as amphotericin B for the therapy of mucormycosis and is not FDA‐approved for primary or salvage therapy of mucormycosis. Deferoxamine is not useful in treating mucormycosis, based on a small clinical trial.

A 40-year-old-woman, a United States resident, returned from a vacation in Kenya with fever. Blood smear revealed 2.4% parasitemia with Plasmodium falciparum. The patient was treated with quinine and doxycycline. On day 2 of therapy, parasitemia was now 1.1% and the patient had only a 38.0°C temperature. On the third day of therapy, she began to feel short of breath which worsened over the day. Chest radiograph showed bilateral interstitial infiltrates and bilateral small pleural effusions. O2 saturation on room air was 84% but rose to 92% on 40% supplemental oxygen by face mask. She was moved to the intensive care unit where her temperature was 37.9°C, blood pressure-138/86 mm Hg, pulse-116/min. WBC was 4100/mm3 (56% PMN, 41% lymphs,3% mono), Hgb-9.4Gm/dL, platelets -101,000.D-Dimer was negative. Blood smear showed rare gametocytes only. The most likely cause of this patient's respiratory difficulty is: ARDS (Acute Respiratory Distress Syndrome) Pulmonary emboli Nosocomial pneumonia Quinine-resistant malaria Quinine toxicity

Correct answer: ARDS (Acute Respiratory Distress Syndrome) ARDS may occasionally occur in patients who are already responding to treatment. The negative D-Dimer make pulmonary embolism less likely. Nosocomial pneumonia should cause higher fever and would be unlikely to have the chest x-ray findings that are described in the text above. Quinine resistance is not apparent since the parasitemia is already reduced and gametocytes may be seen long after ring forms resolve since they do not respond to treatment. Quinine has no specific pulmonary toxicity. Patients with falciparum malaria treated with quinine may develop hyperinsulinemia with symptomatic hypoglycemia, presenting as deteriorating consciousness, not dyspnea.

A 20-year-old Caucasian woman with HIV infection has been given a variety of antiretroviral regimens in the past and was started on abacavir three weeks ago plus lamivudine, darunavir and ritonavir (all new to her except lamivudine). She is also taking trimethoprim-sulfamethoxazole prophylaxis. A week ago she developed nausea, vomiting, diarrhea, and fever and stopped all her medications. Today, feeling better, she restarted all her medications but an hour later felt faint and sought medical attention. The figures show her cutaneous lesions that developed within the first 24 hours of her feeling faint, and shows them resolving one week later.The most likely drug to cause this syndrome with these skin lesions evolving over a week would be: DarunavirRitonavirTrimethoprim-sulfamethoxazoleAbacavirLamivudine

Correct answer: Abacavir This syndrome is typical for abacavir hypersensitivity syndrome and is not typical for any of the other drugs listed. The syndrome occurs abruptly, usually in the first 6 weeks after starting the drug, and gets worse with each dose. It resolves within 2 to 3 days of stopping the drug. If the drug is stopped due to such symptoms, and then the drug is restarted with the mistaken presumption that the syndrome was viral rather than abacavir related, the resulting hypersensitivity syndrome can be especially severe as in this patient and hypotension and even death have been reported. Almost all (94%) of patients with abacavir hypersensitivity syndrome have haplotype HLA-B*5701. Currently guidelines recommend that patients be screened for this allele (8% Caucasians, 2% blacks, rare in Asians) and that the drug NOT be given to positive individuals. This patient should have been screened prior to staring abacavir. Questions rarely mention race unless race identification is a clue to the correct answer, as it was here.

This 26-year-old patient with keratitis originating underneath a contact lens would be most likely due to which species of amoeba, among the following choices: Naegleria fowleriAcanthamoeba castellaniEntamoeba disparBalamuthia mandrillarisEntamoeba moshkovskii

Correct answer: Acanthamoeba castellani Acanthamoeba species have caused corneal ulcers but none of the other amoebae listed here have done so. Acanthamoeba also can cause subacute/chronic granulomatous meningocephalitis most often but not always in immunosuppressed patients. Naegleria causes acute amebic meningitis/encephalitis typically in children with exposures to fresh water ponds such as diving. This syndrome is usually fatal. Entamoeba moshkovskii causes diarrhea but not keratitis. Entameba dispar is an amoeba that is considered a commensal and non pathogen: the following are usually considered non pathogens. If they are on the exam, they are distracters most likely: Entamoeba coli or hartmanni, Endolimax nana, Trichomonas hominis, and Chilomastix mesnilii. Iodamoeba butschlii is usually a commensal but occasionally causes diarrhea. Balamuthia mandrillaris is a rare cause of subacute encephalitis in both immunocompetent and immunosuppressed patients.

This 22-year-old parking lot attendant who emigrated from India three years prior presented with the indolent onset of midthoracic back pain of a month's duration. He was afebrile and otherwise well. The most likely organisms is which of the following: Anaerobic branching Gram positive bacillusAnaerobic Gram negative bacillusAerobic acid fast bacillusAerobic Gram negative coccobacillus

Correct answer: Acid fast bacillus This patient has vertebral osteomyelitis with a paravertebral abscess. Multiple organisms can cause this, most commonly Staphylococcus (Gram positive cocci not listed), but tuberculosis would be the most common among those listed. Brucella (aerobic Gram negative coccobacillus) in unlikely after three years in the United States.

A 57-year-old man with history of advanced AIDS (CD4=10 cells, VL= 1 million copies) presents with complaints of three days of fever, malaise, and a pruritic painful rash covering most of his body. He looks moderately ill. The rash started on his face and over several days spread to most of his body. Some lesions have scabbed, but new papules and vesicles can be seen on examination. This illness started about 10 days after he returned from a visit to Eastern Europe to see relatives. He has not started any new medications in the past 6 months. His rash is shown in the figures and includes vesicles and papules in various stages of maturation. Which of the following medications is most likely to benefit this patient? Lipid formulation amphotericin B Acyclovir Ganciclovir Penicillin Cidofovir

Correct answer: Acyclovir The rash in this patient is vesicular and shows lesions in different stages. The presentation would be classic for chickenpox. While most patients in the United States have been immunized if they were of school age since the vaccine appeared in 1995, or had been exposed before the vaccine, there are some seronegative individuals who might be exposed to the occasional cases in the United States, or when traveling abroad to countries with high rates. This patient's age suggests that he was in school prior to introduction of the vaccine. There is no stated exposure to smallpox or monkeypox. While this could be a drug eruption, there is no history of starting any new drugs recently. A smear of a skin lesion could confirm this as varicella by immunofluorescence or PCR. There is no description of this starting in a dermatomal distribution, so this is more likely primary varicella than disseminated zoster, although the virus is the same, varicella zoster virus. The patient should receive parenteral acyclovir for treatment since he is moderately ill and since he is an immunocompromised adult. Ganciclovir or cidofovir would be effective but more toxic, and thus there is no reason to use these drugs. The vesicular rash of chickenpox is pruritic, and appears in successive crops over several days. The lesions begin as macules that rapidly become papules and then vesicles and occasionally pustules. Amphotericin B could be used treat histoplasma or cryptococcal disease, but these lesions appearing in crops are much more likely to be varicella. Cryptococcal and histoplasma skin lesions are papular or ulcerated and not vesicular. Penicillin could be used to treat syphilis, but a vesicular rash with fever would be atypical, but of course...not impossible.

A 26-year-old injection drug user is being treated for endocarditis due to methicillin-resistant Staphylococcus aureus with vancomycin 1 gram bid. His creatinine is normal and the Staphylococcus M.I.C. for vancomycin is 1 mcg/ml. On day three of treatment, he complains of discomfort and decreased vision in the right eye. An ophthalmologic evaluation shows clouding of the vitreous consistent with endophthalmitis. Which one of the following is the best approach to his endophthalmitis? Change vancomycin to daptomycin .Add rifampin to the vancomycin Add gentamicin to vancomycin Add intravitreal vancomycin.

Correct answer: Add intravitreal vancomycin. Bacterial endophthalmitis secondary to hematogenous seeding is best treated with intravitreal antibiotic injections in addition to systemic antibiotics. Vitrectomy may also be needed. Systemic antibiotics alone are almost never adequate to treat this infection. There is no advantage to changing vancomycin to daptomycin with an MIC of 1 mcg/ml. There is no advantage of adding rifampin or gentamicin to control his bacteremia or trying to penetrate the eye without intravitreal injection.

A 47-year-old male with lymphoma has been hospitalized for an allogeneic myeloablative stem cell transplant with a conditioning regimen of cyclophosphamide and total body irradiation. He is now 65 days post-transplant, receiving cyclosporine, mycophenolate, trimethoprim-sulfamethoxazole, fluconazole, and acyclovir, and has received several courses of pulse steroids for graft-versus-host disease. His white blood count is 3500 cells/µL. Sixty days post transplant he developed gross hematuria with extreme pain on urination and he is passing clots of blood in his urine. His urinalysis shows rare leukocytes, the nitrate test is negative and his urine culture is negative for routine bacterial and fungal pathogens. His urine cytology for BK virus "decoy cells" is negative, as is BK PCR of his blood and urine. Which one of the following would be the most likely cause of hemorrhagic cystitis in this patient? CyclosporineCyclophosphamideJC VirusAdenovirusCytomegalovirus

Correct answer: Adenovirus Adenovirus and BK virus are important cause of hemorrhagic cystitis in this patient population, as is cyclophosphamide (cytoxan). In this case, BK was ruled out by the PCR tests in the stem. Hematuria due to cytoxan (cyclophosphamide) usually starts within 48 hrs of the dose, making that unlikely in this case. Radiation also causes hemorrhagic cystitis. Adenovirus is an important cause of hemorrhagic cystitis which can be latent in an HSCT recipient from a remote infection and reactivate during periods of intense immunosuppression. Active adenovirus infection may be asymptomatic and detected only by PCR of peripheral blood, or may present clinically as hepatitis, or diffuse pneumonia or cystitis. Diffuse cystitis may be so severe that the patient has urethral obstruction from blood clots and such intense pain on voiding that narcotics may be required. There is no specific therapy for adenovirus: some clinicians might use cidofovir which has activity in vitro, but toxicities are problematic. It is important to minimize immunosuppression, as feasible, but this is complicated by underlying control of GVHD. CMV, JC and cyclosporine do not cause hematuria. Cyclosporine causes renal dysfunction with proteinuria. This patient was BK negative. However, BK, a polyoma virus may reactivate under the same circumstances as adenovirus and appear in the urine in high copy number. Although BK may cause renal tubular damage in transplanted kidneys, the role of this virus as a cause of hematuria remains a subject of diagnostic debate: some cystitis and hematuria is clearly due to BK virus, but how to recognize such cases without biopsy or cytology is difficult because BK viruria in high quantities may be present without cystitis.

A group of teenagers travel from New York and are surfboarding in Florida during the presence of a red tide. Two individuals with a prior history of asthma and two with no such history complain of sore eyes and wheezing soon after getting to the beach. They had eaten steamed crabs 3 days earlier but no other seafood. They had eaten at a Chinese restaurant the night before. They go to the emergency room and are found to have a worsening of pulmonary function (decrease FEV1 and peak expiratory flow) and are treated for worsening asthma. 48 hours later they are symptom-free. What is the diagnosis? Ciguatera Chinese restaurant syndrome Scrombroid Aerosolized dinoflagellate toxin Exposure to toxigenic Vibrio parahaemolyticus

Correct answer: Aerosolized dinoflagellate toxin These patients have inhalational dinoflagellate toxin (brevetoxin) related to the red tide. Aerosolized brevetoxin can be inhaled via wave action during algal blooms where dinoflagellate Karenia brevis toxin is inhaled and exposed to eyes. The illness is mild and short lasting. The steamed crabs are low risk, having been consumed 3 days earlier. Ciguatera produces impressive neurologic features and the incubation period is short (30 minutes to 36 hours).Disease is usually more severe, than here and the pulmonary symptoms would be atypical. The Chinese restaurant syndrome due to monosodium glutamate (MSG) could cause bronchospasm, but the incubation period is 5 to 30 minutes, ruling this out as an explanation. Scombroid can explain the ocular and respiratory symptoms but would only last a few hours and shouldn't worsen respiratory function. Vibrio parahaemolyticus causes diarrhea.

A 23-year-old college student is seen for intermittent fevers, headaches and arthralgias. He came to the US from the Central African Republic (central Africa) two months ago to attend college. He says his symptoms have been present for at least the last four months, and it is hard for him to concentrate on his studies. On exam his temperature is 100.6F; he has a soft, moveable posterior cervical node 3cm by 3cm; and his liver and spleen are palpable. Which one of the following is the most likely diagnosis? Malaria Rift Valley Fever Rickettsia africae infection Typhoid Fever African trypanosomiasis

Correct answer: African trypanosomiasis West African trypanosomiasis due to Trypanosoma brucei gambiense is seen in West and Central Africa and is more chronic in presentation than the East African form due to Trypanosoma brucei rhodesiense. Patients may have intermittent fevers, headaches, and arthralgias for months before developing CNS infection (sleeping sickness). Posterior cervical adenopathy, known as Winterbottom's sign, is common in the West African form. Patients may have hepatoslenomegaly. Malaria may cause hepatosplenomegaly but is not associated with adenopathy. This illness is too long in duration for Rift Valley Fever, Rickettsia africae infection, or Typhoid Fever.

A 25-year-old male presents with fatigue and abnormal liver function tests. He has been healthy, jogging 4 miles per day, is at ideal body weight, is HIV negative, and denies alcohol or drug use, although you are not confident he is being forthright. He was treated with amoxicillin-clavulanate for otitis 2 weeks ago at which time he also ingested a large amount of acetaminophen. He received all required childhood immunizations. On exam he is a thin, afebrile, and has no abdominal pain or hepatosplenomegaly. Laboratory AST (SGOT) = 450u/L (normal 5-40) Alk Phos = 150 ( 38-126u/L) ALT (SGPT) = 200u/L (normal 7-56) Bilirubin total = 1.9 ( <0.3mg/dl) Fasting glucose and lipid panel: normal What is the most likely cause of the abnormal liver function tests? Non-alcoholic steatohepatitis Alcoholic hepatitis Hepatitis E Acetaminophen toxicity Amoxicillin clavulanate toxicity

Correct answer: Alcoholic hepatitis AST is more than 2 times the ALT, and both are under 500, which is characteristic of alcoholic hepatitis and not typical of viral hepatitis or other drug toxicities. However, in real life these patterns are not universally true. Hepatitis E is rare in the US,is an acute infection, and has higher aminotransferases. Non-alcoholic steatohepatitis would be unlikely in this patient since there are no risk factors (diabetes, lipid abnormality, obesity), no hepatomegaly, and an AST/ALT ratio that would be unusual. Acetaminophen toxicity is surprisingly common but should have resolved by now, and should not have this pattern of AST/ALT. This timing would be typical of amoxicillin-clavulanate hepatotoxicity, but the AST/ALT ratio would be less characteristic. Such toxicity can occur weeks or even months after the drug is discontinued.

A 17-year-old girl in rural Louisiana presented in July with a 5 cm diameter fiery red, round, well demarcated macular lesion on the back of her left knee. The lesion had been present at least 2 days and was accompanied by moderate headache and malaise but not fever. The lesion was not painful or pruritic. She had removed a tick from the area of the lesion. No inguinal adenopathy was palpable. Routine laboratory work was normal. Which tick would be the most likely vector for this illness? Dermacentor andersoni Ixodes pacificus Amblyomma americanum Ixodes scapularis Ornithodorus parkeri

Correct answer: Amblyomma americanum The lesion is compatible with erythema migrans, but the Ixodes ticks responsible for Lyme disease are not found in the deep South. The cause is mostly likely STARI, Southern Tick Associated Rash Illness, an infection most likely transmitted by Amblyomma americanum, the lone star tick. Despite the name, Borrelia lonestari, that spirochete does not appear to be usual cause of STARI and the agent or agents are currently unknown. Ornithodorus ticks can transmit tickborne relapsing fever. Dermacentor ticks can transmit rickettsial diseases.

A patient with HIV (CD4 = 22 cells) has had fever and shortness of breath for 3 weeks. Chest x-ray and transbronchial biopsy are shown. The optimal therapy for this patient would be: ItraconazoleFluconazoleVoriconazoleLiposomal Amphotericin B aloneLiposomal Amphotericin B plus Flucytosine

Correct answer: Amphotericin B alone This is HIV-associated pulmonary coccidioidomycosis. This is a coccidioides spherule seen in the lung biopsy. These spherules are large, will not be detected by gram stain, and are usually detected in wet mounts or by stains such as calcofluor. These are 20-70 micra in size which is huge compared to histoplasma which are closer to the size of a rbc, i.e., 2-4 micra, and cryptococcus would be 3-6 micra and encapsulated, Coccidioides also can be seen or grown from sputum or bronchoalveolar lavage. Therapy for diffuse pulmonary disease should always be amphotericin B rather than an azole. If meningitis is also present, an azole or perhaps intrathecal amphotericin B will have to be added. Either conventional amphotericin B, or lipid formulations of amphotericin B can be used.

A 55-year-old male presented with fever and malaise of two weeks duration and was diagnosed as sub-acute bacterial endocarditis based on a fever, a new aortic murmur and a vegetation on echocardiography of his aortic valve. He has no allergies. Three blood cultures were all positive on the automated blood culture device at 24 hours and had Gram positive cocci in chains on Gram stain. Subculture onto blood agar had no growth but chocolate agar showed rapid growth of Gram positive cocci in chains. Susceptibility tests could not be done since the plates had no growth. Which one of the regimens below would the guidelines recommend in the absence of MIC data? Ceftriaxone for 2 weeks Ceftriaxone and gentamicin for two weeks Penicillin for 4 weeks Vancomycin for 4 weeks with gentamicin for two weeks Ampicillin and gentamicin for 4-6 weeks

Correct answer: Ampicillin and gentamicin for 4-6 weeks Endocarditis on a native valve due to nutritionally deficient streptococci responds less well to therapy than the more common Viridans viridans streptococci. Most of the Granulicatella adiacens (formerly Abiotrophia adiacens) and Abiotrophia defectiva isolates, the usual nutritionally deficient streptococcal species, have penicillin MIC's between.0.12 and 0.5 mcg/ml, the intermediate range, but some are as high as 4-8 mcg/ml. Those in the intermediate range might be treated with 4 weeks of penicillin or ceftriaxone plus 2 weeks of gentamicin. Streptococcal endocarditis with a penicillin MIC'sabove 0.5 mcg/ml has been recommended to be treated with 4-6 weeks of either ampicillin or penicillin with gentamicin for the entire period. Vancomycin plus gentamicin is an alternative for patients allergic to penicillin but not recommended for other patients. In the absence of an MIC, the severity of endocarditis with nutritionally deficient streptococci would indicate that the last regimen in the list above is the most conservative. Susceptibility testing on a medium that will support growth of nutritionally deficient streptococci would assist management of this case. For example, an "E test" susceptibility test on chocolate agar might be done. Diagnostic laboratories are reluctant to report non-approved tests.

An 18-year-old male had the acute onset of sore throat, followed in two days by high fever. On presentation in the emergency room he was acutely ill, with a temperature of 105°F. His right neck (CT shown below) was tender. Chest CT shown in the top image showed nodules in his lung fields. Swelling and tenderness in the right anterior cervical triangle led to the CT with IV contrast shown here. Note the filling defect in the right jugular vein (No catheter had been in his vein). Figure A shows a lung nodule and Figures B and C show clot in right jugular Blood cultures were likely to reveal which of the following: Aerobic Gram positive rodAerobic Gram negative rodAnaerobic Gram positive rodAnaerobic Gram negative rodEndemic mycosis

Correct answer: Anaerobic Gram negative rod This patient has Lemierre's syndrome due to Fusobacterium necrphorum consisting of suppurative phlebitis of the , internal jugular vein and a lung nodule due to septic embolus. Lemierre's begins with a sore throat with high fever. Infection extends from the tonsillar area to the internal jugular vein, causing septic phlebitis, bacteremia and septic emboli to the lung, sometimes complicated by empyema. The organism is almost always a Fusobacterium species, an anaerobic Gram negative rod, usually Fusobacterium necrophorum. Most Fusobacterium isolates are susceptible to beta-lactams, metronidazole and clindamycin. Not all patients have swelling in the anterior cervical triangle, as did this patient, and in some the pharyngitis will begin to respond to antimicrobial therapy while the septic phlebitis is progressing. Peritonsillar abscess may be seen on imaging of some patients.

Pus from a draining sinus over a homeless man's right submandibular area has the following microscopic finding. What microorganism is most likely to be grown from this pus? Acid fast bacilliAnaerobic Gram positive bacilliAerobic Gram positive bacilliAnaerobic Gram negative bacilliFungal hyphae

Correct answer: Anaerobic Gram positive bacilli Draining submandibular nodes may suggest mycobacterial infection or actinomycosis. The small yellow particles in the drainage shown here are "sulfur granules" characteristic of actinomycosis, caused by anaerobic Gram positive bacilli. The source of cervical actinomycosis is usually a dental abscess. Scrofula, or mycobacterial lymphadenitis, would not have the particles in the drainage. With regard to hyphae, fungi can cause mycetoma, a chronic subcutaneously infection with draining sinuses with grains in the pus. Mycetoma arises from minor trauma and is almost always on an extremity. A submandibular site would not be consistent with mycetoma.

This is a stool smear of a young child from rural Georgia who was seen by a pediatrician for listless behavior. This smear could explain which of the following findings in this child? DiarrheaAppendicitisHepatomegalyAnemiacholangitis

Correct answer: Anemia Hookworm infection is often asymptomatic except for development of anemia from the blood consumed by the worm attached to the intestinal wall. The human hookworms include the nematode species Ancylostoma duodenale and Necator americanus. Treatment is albendazole with mebendazole and pyrantel pamoate as alternatives. Hookworm was once widespread in the United States, particularly in the southeastern region, but eradication efforts in the 20th century and improvements in living conditions have greatly reduced hookworm infections. Hookworm, Ascaris, and whipworm are known as soil-transmitted helminths (parasitic worms). Appendicitis and cholangitis can be seen in ascariasis but not hookworm. Hepatomegaly is not seen with any of the intestinal worms but occurs in visceral larval migrans (Toxocara canis).

A 24-year-old female, in prior excellent health, developed a severe sore throat threedays ago with fever. Hersore throat has gotten progressively worse. She now has chest pain and shortness of breath. On exam she has no cervical adenopathy; her throat is red with tonsillar exudate; her lungs are consolidated. Her WBC = 22,000/mm3(95% neutrophils). Her labs are otherwise unremarkable, andher HIV test is negative. A CT of the neck and chest are shown. The intensivist could not place a central venous catheter in the right internal jugular; after looking at the CT of the neck taken beforehis effort, a clot in the internal jugular was visible. CT of the clotted internal jugular vein (red arrow). Dilated left internal jugular vein indicated (blue arrow). From Brook I: http://tonsilitisunderstood.blogspot.com/p/lemierre-syndrome.html Chest CT shows bilateral pleural effusion and multiple areas of consolidation with cavitation (arrows) Lai, N Engl J Med 2004; 350:e14 The most appropriate therapy on the first day of this presentation would be: Surgical removal of the clotted vein plus antibioticsTonsillectomy plus antibioticsAntitoxin and antibioticsAntibiotics aloneAntibiotics plus corticosteroids

Correct answer: Antibiotics alone This case is a classic example of Lemierre's Syndrome that consists of bacterial pharyngitis associated with internal jugular septic phlebitis and septic pulmonary emboli. The organism that purists say is the only organism causing Lemierre's syndrome would be Fusobacterium necrophorum, although cases in the literature use the term with such cases of necrobacillosis due to other pathogens including other Fusobacterium species, anaerobic streptococci, and other anaerobes. In this patient, a septic embolus yielding pulmonary cavitation progressed to communicate with the pleural space, causing an empyema, which can sometimes be a feature of this syndrome. Appropriate management is antibiotic therapy alone at least initially: ampicillin-sulbactam or a carbapenem would be satisfactory therapy. Surgery to remove infected clot is rarely indicatedunless there is ongoing bacteremia. Anticoagulation is unnecessary unless the thrombosis is extensive or extending on antibiotic therapy. This patient had no evidence of a peritonsillar abscess. If such a collection were present, drainage should be considered. EBV can cause severe pharyngitis, but patients usually have marked cervical adenopathy plus malaise, and there is no association with septic phlebitis or septic pulmonary emboli. Arcanobacter causes pharyngitis and is often associatedwith a rash. Note the lab will not routinely look for fusobacterium in the throat culture (throat cultures are not set up to look for anaerobes). Fusobacteria havebeen recently describedas responsible for a subset of suppurative pharyngitis in adolescents and young adults, butdedicated cultures or PCR for detection are not widely available. Group A streptococci are a common cause of pharyngitis but are not associatedwith this syndrome. Diphtheria would be treatedwith an antitoxin, but there is no exposure history, and diphtheria would present with a gray pharyngeal membrane, septic emboli would not be typical.

5.The following lesions were seen in an asymptomatic male presenting for the first time for evaluation of HIV (CD4 = 125 cells, VL = 1 million). Lesions have been present for months, but are increasing in size and number. Which of the drugs listed is the best drug therapy for these lesions: Choices A Acyclovir B Valganciclovir C Voriconazole D Amphotericin B E Antiretroviral therapy alone

Correct answer: Antiretroviral therapy alone This is Molluscum contagiosum, which is caused by a pox virus that is spread by skin to skin contact. These lesions can become numerous and large in patients with HIV infection, especially those with low CD4 counts. There is no specific chemotherapy. The lesions often resolve with ART, or can be removed by physical methods such as liquid nitrogen, curretage, or electrosurgery although such therapies may have to be repeated every few weeks for lesions that are cosmetically disturbing. Some reports suggest a role for cidofovir, but that is controversial and was not offered as an option. Disseminated cryptocococcal infection can cause skin lesions that look like molluscum, but the patient would typically be ill with persistent fever.

A 31-year-old female with headaches, low grade fevers for 3 weeks, had diffuse sore throat, malaise and nonpruritic maculopapular rash 3 months ago which resolved. Neuro exam is nonfocal, CT of brain normal; serum RPR is reactive 1:32, treponamal EIA IgGs reactive. HIV serology is negative. She is three months pregnant and has a history of an itchy rash when she was given amoxicillin two years ago. CSF: WBC 42/mm3 (100% mono's), protein 105mg/dl, glucose 55 mg/dl and VDRL nonreactive What would you recommend? Desensitize and give benzathine penicillin G 2.4 million units IM weekly x 3 dosesDoxycycline 100 mg bid x 14 daysCeftriaxone 2 gm IV qd x 21 daysDesensitize and give aqueous penicillin G 3 million units IV q4h x 14 daysAzithromycin 500 mg daily for 14 days

Correct answer: Aqueous penicillin G 3 million units IV q4h x 14 daysThe patient most likely had secondary syphilis three months ago and now has meningitis as a late complication. A negative CSF VDRL does not exclude the diagnosis.Parenteral penicillin regimen indicated here reaches treponemocidal levels in the CSF and is the drug of choice. Pregnancy puts the fetus at risk for congenital syphilis and requires desensitization and penicillin . Her prior reaction was not anaphylaxis, making desensitization safe enough to warrant the risk.Penicillin levels in the CSF are too low with benzathine penicillin to treat CNS syphilis.Doxycycline is an option for treating syphilis at some stages in the patient with penicillin allergies.It also treats infections caused by other spirochetes which might have been considered here, but the patient has serologic evidence of syphilis infection and it is unproven therapy for neurosyphilis.Ceftriaxone does have efficacy against T. pallidum, but we have far less experience with this agent than with aqueous penicillin, and a 21-day course would be more compatible with CNS Lyme than with syphilis.Azithromycin is not recommended for syphilis.Aseptic meningitis as a presentation of syphilis can occur within a few months to one year following syphilis exposure and overlap with either primary or secondary lesions; it can even occur more than one year, though this is uncommon.

This 70-year-old female from a rural area on the Eastern shore of Maryland had a low grade fever and expanding, painless rash on her arm, shown here. Which of the following would be the most likely later complication of this infection if it is not treated promptly? Transverse myelitisMeningitisUveitisArthritisHepatitis

Correct answer: Arthritis The lesion is typical of erythema migrans due to Lyme disease. The causative spirochetal agent, Borrelia burgdorferi, is usually acquiredfrom an infected black-legged (deer )tick bite in an endemic area. Days to weeks after the skin lesion appears if patients are not treated, then complications of disseminated infections may occur including cardiac or neurologic manifestations. Late Lyme arthritis may begin weeks to months after acquiring the infection. This characteristic arthritis typically always involves the knee with relative cool but large effusions. Even without treatment, arthritis may remit after weeks or months but then flare in the other knee or other joints. Uveitis and transverse myelitisare not associated with Lyme, though neuroretinitis and encephalitis can occur. Liver function tests may be transiently elevated during acute infection when fever is present but does not persist even without antibiotic therapy.

A 24-year-old man from rural Ecuador presents with a chief complaint of worsening abdominal pain. He reports a 1-month history of right upper quadrant pain, which has become suddenly worse over the past day and is now associated with vomiting and low grade fever. He exhibits tenderness over the right upper quadrant. Total white blood cell count is 12,000/ml with 60 % neutrophils, 24 % lymphocytes, 10 % monocytes, and 6 % eosinophils. Total bilirubin is elevated at 4.5 mg/dL and the alkaline phosphatase level is 255 IU/L. AST and ALT are twice normal. Which of the following helminths is most likely to cause this clinical presentation? Fasciolopsis buski Taenia saginata Strongyloides stercoralis Wuchereria bancrofti Ascaris lumbricoides

Correct answer: Ascaris lumbricoides acute cholangitis Ascaris lumbricoides routinely causes this migration of the Ascaris worm into the biliary duct Diagnosis ultrasound or CT scan, which are fairly sensitive means of detecting worms in the biliary tree. Medical treatment with an anthelmintic can sometimes cure the disease, though surgery or ERCP are often required (especially when there are stones or strictures which prevent the worm from exiting back into the duodenum). Other helminths that can cause biliary disease include the liver flukes Fasciola hepatica, Clonorchis sinensis, and Opisthorchis viverrini. Fasciolopsis buski, the giant intestinal fluke, can cause intestinal inflammation and ulcerations with heavy infections where the flukes attach to the intestine, but do not typically affect the biliary tree. Taenia saginata, the beef tapeworm, rarely causes abdominal symptoms in adults and is not known to cause cholangitis. Strongyloides stercoralis worms in the small intestine can cause abdominal pain due to duodenitis, but do not typically cause cholangitis. Adult worms of Wuchereria bancrofti, a causative agent of lymphatic filariasis, live in lymphatic vessels and do not cause cholangitis.

You are asked to see a 24-year-old Peace Corps volunteer who passed the following worm in his stool. The volunteer is perfectly healthy and has been in West Africa for two years. This worm is most likely to be: Whipworm (trichuris trichuris)Pinworm (Enterobius vermicularis)Ascaris lumbricoidesDiphylobothrium latumTaenia saginata

Correct answer: Ascaris lumbricoides This is a round worm, so it cannot be Diphylobothrium or Taenia which are tapeworms which are flat. Taenia can be several meters long but may be excreted in smaller segments that break off. Pin worms are round worms, but they are rarely much over 1 cm, Whipworms are also round worms but they are also short, typically 3-5 cm. The size is more consistent with the round worm Ascaris, which are 15-30 cm in length. Treatment is albendazole, mebendazole, or ivermectin. Ascaris are round worms, as shown below. Familiarize yourself with:

You are asked to see a patient in the Bone Marrow Transplantation Unit for fever, neutropenia and pleuritic pain. The patient, a 45-year-old woman, had an allogeneic stem cell transplant five weeks ago for acute leukemia. She has been receiving prophylaxis with acyclovir and fluconazole. Three days ago she developed fever without localizing signs or symptoms. At that time, she had failed to engraft and remained neutropenic with an ANC of zero. Cefepime was begun. Fever persisted and vancomycin was added. On day 5 of fever, she complained of right-sided pleuritic pain. She has no cough. A chest CT scan showed a peripheral right-sided lung lesion which was described by the radiologist as having a " halo sign," that is, a nodular dense consolidation with hazy ground glass infiltrate surrounding it. Which one of the following is the most likely cause of the lung problem? AspergillosisCandidiasisLegionellosisBronchiolitis obliteransDiffuse alveolar hemorrhage

Correct answer: Aspergillosis This halo sign is very suggestive (but not absolutely diagnostic) of Aspergillosis. Aspergillus species can form a "halo sign" (central dense consolidation surrounded by ground glass infiltrate) seen in Figure A below because they invade and thrombose blood vessels. The nodule is fungus and infarcted lung; the halo is due to hemorrhage into the alveoli. The dense nodule often cavitates (Figure B) with effective antifungal therapy, especially when the neutrophil count returns, as shown below. Another example of a halo sign (JCO 2001; 19:253) is below: Keep in mind that this is different from the "reversed halo sign" seen below which hopefully will not be on the exam because, while described in the literature, is not as well documented as the halo and crescent signs. The reverse halo sign is a dense margin of a pulmonary infiltrate with a less dense center and has been reported to be associated with mucormycosis. Candida almost never causes pulmonary disease in any patient, and thus is not the best answer Cryptococcus is not likely is a patient taking fluconazole prophylaxis, and is not associated with a halo sign. Bronchiolitis obliterans is associated with graft versus host disease, occurs late, usually after 100 days, and presents with wheezing and sometimes progresses to bilateral ground glass infiltrates, but not nodules. Diffuse alveolar hemorrhage occurs within the first 100 days post-transplant, and presents with bilateral, diffuse ground glass. To make this diagnosis the bronchoalveolar lavage should be bloody throughout the lavage, and other causes of hemorrhage should be excluded.

A 56-year-old woman with poorly controlled type 2 diabetes mellitus presents with fever, rigors and near syncope. In the Emergency Room she is confused, hypotensive, and febrile to 38.5°C. Her WBC is 18,000 with 90% neutrophils. She is given bolus IV fluids for shock, started on vancomycin and piperacillin-tazobactam and admitted. CT showed air in her left atrium (see figure) and multiple brain abscesses. The left atrium shows air in the atrium (black arrow) and the esophagus (white arrow). Her past history is remarkable for a dental extraction one week prior. No prophylaxis was given. Four weeks prior she had her second catheter radiofrequency ablation for atrial fibrillation. The most likely cause of this syndrome is: Aortic valve endocarditisAtriobronchial fistulaAtrioesophageal fistulaMucormycosis of the left atriumAtrial myxoma

Correct answer: Atrioesophageal fistula As more and more patients get radiofrequency ablations for arrhythmias, health care providers must be aware of the complications of the ablation procedure which are rare (1-2% of cases) but which include pericardial tamponade and atrioesophageal fistula. Patients are often febrile after ablations for a day or two (perhaps due to local pericarditis), and thus such fever does not necessarily signal infection, but they rarely have leukocytosis or chills from this non-infectious post procedure process. The esophagus can be contiguous to the ostium of the pulmonary vein where it enters the left atrium. Thermal coagulation at this site to treat atrial fibrillation can create a fistula which introduces bacteria and sometimes air into the left heart. Sudden onset of profound sepsis is the usual presentation. (BMJ Case Rep 2013; Nov. 20) Neurologic manifestations due to brain abscesses or air emboli are common. Performing a transesophageal echo is contraindicated because this can lead to air emboli. The fistula must be repaired surgically or by some less invasive measure, such as an esophageal stent. The bronchi are too far away from the ablation site to create a fistula. Neither an atrial myxoma, mucormycosis or endocarditis would cause air in the atrium.

A 30-year-old traveler went to Goa, India for vacation and developed diarrhea and fever on the fifth day at the resort. After 36 hours of diarrhea stools begin to contain bright red blood and mucus. Which of the following drugs would be the most appropriate empiric therapy for this patient, pending results of stool studies. Ciprofloxacin Azithromycin Nitazoxanide Metronidazole Rifaximin

Correct answer: Azithromycin The invasive pathogens (Shigella, Salmonella, and Campylobacter) occur more commonly in travelers to Asia compared with destinations in Africa and Latin America. Ciprofloxacin would be a poor choice in this patient because of the importance of fluoroquinolone-resistant Campylobacter as a causative agent of dysenteric travelers' diarrhea occurring in Asia Metronidazole would be appropriate for persistent diarrhea caused by Giardia or Entamoeba histolytica but would not be useful in most patients with travelers' diarrhea where the typical enteropathogenic agents are bacterial in origin. Nitazoxanide would be useful for giardiasis or cryptosporidiosis but neither would cause bloody diarrhea. Rifaximin is of value for the common watery diarrhea of travelers but is of no value for the treatment of mucosally invasive enteropathogens as the drug is very poorly absorbed (< 0.4%). Azithromycin should be preferentially used for dysenteric travelers' diarrhea or when breakthrough diarrhea occurs in a patient taking rifaximin to prevent travelers' diarrhea.

In your community, there is an outbreak of cases of diarrhea often associated with low grade fever and resolving after several days. For many of the affected patients, household contacts developed diarrhea 3‐7 days later. The cases are characterized by diarrhea without prominent nausea or vomiting. Which of the following organism is most likely responsible for this outbreak of diarrhea with high rates of secondary spread? A. Salmonella typhimurium B. Shigella sonnei C. Campylobacter jejuni D. Vibrio parahaemolyticus E. Aeromonas hydrophila

Correct answer: B. Shigella sonnei The two most communicable enteric pathogens are Shigella and norovirus. Since norovirus is not an option in the question and there was no reported vomiting, Shigella sonnei is the best answer. The incubation period of Shigellosis averages 3 days (range 1‐7 days) and the average case lasts for 7 days untreated. Shigella sonnei, the answer here, typically cause mild, self‐resolving disease that may not be associated with fever. Other strains such as Shigella dysenteriae or Shigella flexneri may cause high fevers and many bloody stools per day. Salmonella, other than S typhi, rarely is associated with secondary person‐to‐person spread. Campylobacter, non‐cholera Vibrios and Aeromonas spp. even less likely causes secondary spread due to the higher inoculum size required for illness to develop. Of the organisms listed only Shigella regularly infects people at very low inoculum size.

In February, 27‐year‐old man is seen for fever, myalgias, arthralgias, and headache of one‐day duration. Two days before he became ill, he returned from a skiing vacation with friends in California. They stayed in a cabin in the woods that had been uninhabited for many months. Rodent droppings were evident in the cabin when they arrived. He has been taking NSAIDS for muscle pain precipitated by skiing. Exam is notable for fever and meningismus. A CSF exam shows 400 white blood cells, 97% lymphocytes. The CSF protein is 98, and the glucose is 30 with simultaneous blood glucose of 88; Gram stain is negative. Which one of the following is the most likely cause of his illness? A. NSAID‐induced aseptic meningitis B. Lymphocytic choriomeningitis virus C. Enterovirus D. Hantavirus E. Listeria

Correct answer: B. Lymphocytic choriomeningitis virus Lymphocytic‐predominant meningitis winter rodent exposure, lymphocytic choriomeningitis virus headache, meningismus myalgias. CSF white blood cell count higher than with other viral‐induced meningitis and may be more than 1000. The CSF glucose is low in about a quarter of cases. NSAID‐induced meningitis is typically neutrophilic and the CSF glucose is normal. Enterovirus meningitis is seen in summer and early fall. Hantavirus illness is associated with exposure to rodent excreta but does not produce meningitis. Listeria meningitis may be associated with lymphocytic predominance in the CSF but would be very unusual in an otherwise healthy young man.

A 40‐year‐old man from Ghana was visiting his daughter in the United States. She sought medical care for her father who was constantly scratching himself, with generalized pruritus. A 4 cm painless nodule was found on his right hip. Biopsy of the nodule reveals cross‐sections of very thin nematodes. The most likely cause of the nodule is: A. Ascaris lumbricoides B. Onchocerca volvulus C. Schistosoma mansoni D. Trichinella spiralis E. Loa loa

Correct answer: B. Onchocerca volvulus Onchocerca volvulus River blindness Blackflies filarial nematode itching, subcutaneous nodules, and blindness Western Central Africa, South America. curled up on themselves, within subcutaneous nodules such as the shin and forehead. Microfilariae, which are released from adult worms, migrate within the skin and conjunctiva. inflammatory responses to dying microfilariae cause of symptoms i Incorrect answers: Ascaris lumbricoides, the most common nematode infection of humans, migratory stage travels to the lungs not typically migrate through skin or subcutaneous tissu Schistosoma mansoni is a fluke portal veins. Trichinella spiralis tissue‐invasive nematode larvae encyst muscle cells. Loa loa, like O. volvulus, chrysops flies gh subcutaneous tissues (causing transient swellings called Calabar swellings) and across the eye. do not remain stationary and thus do not cause long‐lasting subcutaneous nodules or generalized pruritus.

A 19-year-old autistic male is admitted with 3 week history of fevers and constipation, decreased oral intake and decreased activity. He has severe constipation and abdominal pain. This patient lives with his mother in Washington, DC. He has had no recent sick contacts/travel but he has very poor personal hygiene and often lives on the streets for days at a time. He has two cats at home, but no recent bites/scratches; no TB contacts. He is febrile to 103°F, anxious and moaning but has no other findings. His routine laboratories are unremarkable: his WBC is 12,500 and his ESR is 96. An image of his liver is shown in the figure. The most likely organism is a: BacillusCoccusYeastProtozoonVirus

Correct answer: Bacillus This patient has bacillary peliosis, caused by the bacillus, Bartonella henselae. The organism is a gram negative rod that is very fastidious: it is best seen not with a Gram stain but with a Warthin Starry silver stain, and requires fresh blood and special conditions for growth. The vascular lesions in liver and spleen are associated with granulomas and are acquired from his cat. The lesions can be diagnosed by PCR or histology of a liver biopsy specimen. Radiologic appearance is very distinct from bacterial liver abscesses, amebiasis or viral hepatitis. The appearance is compatible with hepatosplenic candidiasis but this diagnosis is unlikely in an immunologically normal young adult. There is no prior history of neutropenia and no reason for him to have had candidemia. Are these lesions due to louse borne Bartonella quintana, or cat-acquired Bacillary henselae? With poor hygiene and life on the streets he may have lice; with two cats at home, he may have exposure to cat fleas or scratches. The answer: Peilosis hepatis is always (or almost always) related to B. henselae. Peliosis hepatis is most likely to be seen in patients with HIV and low CD4 counts but can be seen in other populations.

A 26-year-old man was visiting a friend in upstate New York in January. The friend lived on a small farm with 2 dogs, a kitten, and a horse. The patient went horseback riding and cross country skiing over the weekend visit. About 3 days after returning, he noted that his left eye was irritated and reddened. The eye worsened and he noted a swelling in front of his left ear. He felt mildly fatigued and thought that he had a low grade fever. On physical exam: T: 38.1° C, BP: 124/82 mmHg, P: 84/min HEENT: Left conjunctiva was markedly reddened; 1.5 cm Lymph node was palpated in the left pre-auricular area Right eye was normal. The remainder of the exam was normal. The patient most likely has a disease caused by: Borrelia burgdorferi Ehrlichia chaffeensis Anaplasma phagocytophilum Pasteurella multocida Bartonella henselae

Correct answer: Bartonella henselae This is a case of oculoglandular fever of Parinaud. This syndrome consists of follicular conjunctivitis, localized adenopathy (cervical, submandibular, pre auricular) and fever. This syndrome is caused by several agents including Bartonella henselae (Cat Scratch Disease), adenovirus and tularemia. Presumably the organism comes into contact with the conjunctiva, often inoculated by the patient's fingers. If the patient had contact with a rabbit or similar animal, Francisella tularensis would be a consideration. There are more cases being linked to Bartonella henselae, possibly due to inoculation of cat flea feces when fingers come into contact with eyes. Diagnosis of Bartonella depends on PCR or culture of the conjunctiva, or serologies. These tests are not all widely available. Lyme, ehrlichia, and anaplasma would be unlikely to cause this syndrome, and January would be an highly unusual time to have a tick-borne disease such as Lyme, ehrlichiosis or anaplasmosis in New York state. Pasteurella multocida infection is caused by a bite or scratch of a cat (occasionally a dog) and presents as a cellulitis, sometimes with bacteremia but not as Parinauds syndrome.

A 34-year-old male with HIV disease (CD4 count = 25 cells/uL, VL 300,000 copies/ul) has had fever and 20 lb weight loss although he is not acutely ill. He has erythematous skin lesions that are awaiting biopsy and he has the hepatic lesions shown here. Assuming that the skin lesions and these low density hepatic lesions are caused by the same disease process, they are most likely due to: Kaposi sarcomaLymphomaMulticentric Castleman's diseaseBartonella henselaeCandida albicans

Correct answer: Bartonella henselae The illness and low-density lesions on CT are characteristic of peliosis hepatis, a term used to describe gray lesions on the cut surface of the liver. In HIV patients, a form of peliosis hepatis called bacillary peliosis should be suspected when low-density lesions are seen as in this case. The lesions are highly vascular and caused by Bartonella henselae. Patients do not invariably have a history of contact with cats. Kaposi's sarcoma, multicentric Castleman's disease and lymphoma could cause lesions in the liver, but the lesions would not be low-density, vascular lesions and would not be this numerous. Disseminated candida can cause skin lesions and hepatic lesions. The skin lesions would be more typical in an acutely ill, neutropenic patient or a patient with catheter sepsis. Hepatic lesions would be typically in a different patient population, namely those with recently resolving chemotherapy induced neutropenia.

A 45-year-old male from Los Angeles with HIV (CD4=20 cells/ul, VL= 500,000) has had a low-grade fever, skin lesions and right wrist pain. His right arm and an x-ray are shown. The x-ray shows periosteal inflammation and cortical bone erosion. This patient volunteers at a shelter for stray dogs and cats, and is also fond of consuming raw fish (sushi). The most likely infectious agent to cause both the skin and bone lesions would be: Balamuthia mandrillarisBartonella henselaeMycobacterium genavenseBlastomyces dermatitidisRhodococcus equi

Correct answer: Bartonella henselae These lesions are typical for Bacillary angiomatosis. Skin lesions are often impossible to distinguish on inspection from Kaposi sarcoma. Biopsy readily makes the distinction. The most likely cause is Bartonella henselae which is transmitted by cat bites or licks and between cats and humans by cat fleas. Balamuthia mandrillaris is a free living ameba found in the soil which is a rare cause of amebic encephalitis in transplant recipients and occasionally in other immunosuppressed patients. Balmuthiia has no particular association with HIV infection or with this type of skin lesion. Mycobacteria genavense is a rare, slow growing Mycobacterium that does not characteristically cause skin lesions. Blastomyces dermatitidis rarely causes infections in the Western United States, occurring largely in the Southeastern United States and Central United States and Canada. Blastomycosis does cause skin and lytic bone lesions, but the skin lesions tend to have an ulcerated center. Blastomycosis is not an HIV related opportunistic infection unlike several other endemic fungi. Rhodococcus equi has been reported to cause sepsis or pneumonia in patients with HIV, but would not be associated with these skin or bone lesions.

An 18-year-old young man who lives in a group home in California and has severe developmental disabilities is admitted to a hospital in coma with hypertonia and hyperreflexia. The people at his residence say he had been sleepy and "wobbly" for the two previous days. Initial laboratory studies showed peripheral eosinophilia (18% of 14,200 white blood cells), and he had CSF eosinophilic pleocytosis (40% of 42 white blood cells); CSF gram stain and cultures were negative. He was treated with cefepime, amphotericin B, and ivermectin but remained unconscious. An investigation of the residential home by public health officials showed evidence of raccoon feces in the yard in which the patient played regularly. Which one of the following is the most likely cause of his encephalitic illness? Coccidioides Angiostrongylus Rabies West Nile virus Baylisascaris

Correct answer: Baylisascaris The raccoon roundworm, Baylisascaris, is a rare cause of eosinophilic meningitis and encephalitis. children infected by ingesting infective eggs shed in raccoon feces from the GI tract to viscera, the eyes and the CNS. no known effective treatment but albendazole is often used. Coccidioides can cause eosinophilic meningitis but the onset here is too acute and the encephalitic picture would be rare. The rat lung worm, Angiostrongylus, is a common cause of eosinophilic meningitis in Southeast Asia and tropical Pacific islands. ingestion of raw or undercooked infected snails or slugs; or pieces of snails and slugs accidentally chopped up in vegetables or salads; or foods contaminated by the slime of infected snails or slugs. West Nile virus is not associated with peripheral or CNS eosinophilia. Toxocara does not invade the CNS.

A 36-year-old man returned from a fishing trip in Colorado. He presented to the emergency department of a local hospital with complaints of 4 days of watery diarrhea, crampy epigastric pain, and foul-smelling flatulence. A stool specimen submitted for ova and parasite examination was positive for the parasite in the figure. Which of the following is the most common reservoir host? BeaversDogsRabbitsSnakesTrout

Correct answer: Beavers The organism in the figure is Giardia lamblia. Giardia trophozoites are 9 to 12 µm long and 5 to 15 µm wide. Flagella are present, as are two nuclei with large central karyosomes, a large ventral sucking disk for attachment to the intestinal villi, and two oblong parabasal bodies below the nuclei. Cysts are smaller -8 to 12 µm long and 7 to 10 µm wide. Four nuclei and four parabasal bodies are present. Giardia has a worldwide distribution with streams and lakes contaminated in mountainous areas. The sylvatic distribution is maintained in reservoir animals such as beavers and muskrats. In this setting, giardiasis is acquired through the consumption of inadequately treated contaminated water. The other animals listed in this question (i.e., dogs, rabbits, snakes, trout) have not been implicated in disease caused by this parasite. Correct Response

These painful recurrent lip and mouth lesions are from a woman with prior painful labial ulcers, arthritis of the knee and decreased vision in one eye The most likely diagnosis is: Acute reactive arthritis (previously Reiter's syndrome)Antiiphospholipid syndromeSystemic lupus erythematosus (SLE)Behcet's syndromeHerpes simplex virus infection

Correct answer: Behcet's Syndrome Recurrent painful ulcers in the mouth, lips or genitalia should suggest Behcet's syndrome, a clinical entity that can include skin lesions, iritis or uveitis, and pathergy, which is an inflammatory response to mild skin trauma. Reactive arthritis (previously termed Reiter's syndrome), is a combination of recurrent episodes of arthritis, conjunctivitis and urethritis typically triggered by a gastrointestinal (eg, Salmonella, Shigella, Campylobacter) or genital (Chlamydia) infection. Other manifestations include keratoderma blennorrhagica, which are hard nodules on the soles or palms, uveitis and painless ulcers (circinate balinitis) on the tip of the penis. Antiphospholipid syndrome is characterized by venous thromboses in unusual sites but not mucosal lesions. SLE may cause arthralgias and painless (as opposed to the painful lesions of Behcets disease) oral ulcers, but not genital ulcers. Herpes simplex viral infection can cause oral ulcers (HSV-1) and genital lesions (HSV-1 or 2) but not arthritis

A 32-year-old woman in Wisconsin who worked as a farmer, presents to her family physician in June with a 3-week complaint of a low-grade fever, myalgias, a productive cough, and a skin lesion on her arm that developed during the previous week. The physician found an infiltrate on chest x-ray, collected sputum and a biopsy from the skin lesion for culture and histopathology. The organisms shown here were seen when the tissue was stained with a silver stain. After 14 days, a mold grew in the fungal culture. Which of the following is the most likely pathogen? Blastomyces dermatitidisCandida albicansCoccidioides immitisCryptococcus neoformansHistoplasma capsulatum

Correct answer: Blastomyces dermatitidis This patient has blastomycosis with dissemination to the skin, a common site of dissemination. Blastomyces species is a dimorphic fungus with yeast cells observed in tissue and a filamentous mold when grown in culture. The large yeasts are characterized by a thick wall and a broad base when budding is observed. Candida and Cryptococcus species are not dimorphic and grow as yeasts both in tissue and culture. Coccidioides and Histoplasma species are dimorphic fungi; in tissue, Coccidioides species appears as non-budding spherules and Histoplasma species as yeast with narrow based buds; both grow as a mold in culture.

This is the blood smear of a previously healthy commercial photographer with a fever after being in Tanzania and then Colorado. He went to Tanzania from 42 days previously to 35 days previously, where he spent most of the time in the city but did go to a rural orphanage to take some photographs. He saw many rats near his hotel and in the rural area. After he returned to Chicago, he left for a week's trip to Colorado from 14 days previously to 7 days previously, where he stayed with his friends in his uncle's cabin in the mountains, doing white water rafting and driving an all-terrain vehicle through the woods. About 3 days after returning to Chicago, he developed a fever, headache and myalgia. He came to a travel clinic the next day, after developing a fever (day 4 after returning from Colorado), where this blood film for malaria was observed. The blood for CBC and routine chemistries clotted. When he was called about the results of his blood smear, he said he was feeling OK, was busy, and wouldn't return to the clinic. The most likely identity of this organism is: Borrelia hermsiiBorrelia recurrentisLeptospira interogansBorrelia burgdorferiSpirillum minus

Correct answer: Borrelia hermsii Relapsing fever, caused by spirochetes of the Borrelia genus, is an arthropod-borne infection and occurs as tick-borne and louse-borne. Incubation periods are important: both types of relapsing fever (louse borne and tickborne) have incubation periods of 7-18 days. Thus the incubation period is too long for Borrelia recurrentis (and this patient is not a likely candidate for a louse borne disease), and of the other 4 options, seeing Borrelia in the blood would be most unlikely except for Borrelia hermsii transmitted in this case by a tick. Tick-borne relapsing fever is a zoonosis and occurs worldwide involving many different ticks and at least 15 species of Borrelia. The two main Borrelia species in North America are Borrelia hermsii (in the Western mountains) and Borrelia turicatae (in the South). Louse-borne relapsing fever is caused by only one species. Borrelia recurrentis is spread only by lice from human to human, and is related to socio-economc factors, now largely refugee camps, most recently in Subsaharan Africa although in World War II this was a huge problem in Europe and Northern Africa (50,000 deaths). Neither Leptospira interrogans nor Borrelia burgdorferi appear in blood smears. Relapsing fever usually resolves after a few febrile episodes of decreasing intensity, but deaths do occur. Jarisch Herxheimer reactions occur during therapy (tetracyclines, macrolides, or penicillin). Spirilum minus is an uncommon cause of rat bite fever: and has not been reported in the Western Hemisphere: there is no history of rat bites. This organism has been seen in blood smears.

A 56-year-old man is seen for low back pain that has been present for a month. He is afebrile and x-rays show abnormalities of the left sacroiliac joint suggestive of infection. Two months before his pain began, he spent a two-week vacation in Spain where he enjoyed eating local cheeses made from unpasteurized cow, goat, and sheep milk. He has had no gastrointestinal or genitourinary symptoms. Which one of the following is the most likely cause of his sacroiliitis? Brucella Listeria Yersinia Salmonella Campylobacter

Correct answer: Brucella Acute brucellosis presents with insidious onset of fever, night sweats, arthralgias and myalgias, low back pain, and and non specific weight loss and malaise. Many patients have dyspepsia, abdominal pain, and cough. Hepatomegaly, splenomegaly, and/or lymphadenopathy are often present. . The fever in untreated acute brucellosis can be high and usually lasts for days to weeks. Localized infection due to Brucella can occur anywhere but osteoarticular involvement is the most common presentation especially the sacroiliac joints and large joints of the lower limbs. Such joint involvement is more prevalent in older patients and patients with prolonged illness. The lumbar vertebrae are involved more frequently than the thoracic and cervical vertebrae. Paravertebral, epidural, and psoas abscesses occur. Chronic and relapsing disease are typical. Local infections in which the organism can be cultured need to be distinguished from reactive arthropathies. Reactive arthritis is conventionally defined as a mono or oligo arthritis that follows by a few days or a few weeks an infection, although the pathogens cannot be cultured from the affected joints. It is generally regarded as a form of spondyloarthritis. Associated infections follow: Chlamydia species especially Chalmydia trachomatis Salmonella species Shigella species Yersinia enterocolitica and Yersinia pseudotuberculosis Campylobacter, especially Campylobacter jejuni Clostridium difficile

The hypervirulent strain of C. difficile designated North American Pulse Field 1 (NAP1), 027 by PCR ribotyping, and BI by restriction endonuclease analysis (REA) is characterized by which of the following? A. Patient mortality in excess of 50% within 30 days B. Recurrent C. difficile infection rates of over 35% C. High level toxin A and B production D. Vancomycin resistance E. Fidaxomicin resistance

Correct answer: C. High level toxin A and B production This hypervirulent strain of C. difficile is characterized by mutations in the tcdC gene which is a suppressor of toxin A and B production. Thus, with less suppression of toxin, there are high levels of toxin A and B production. These strains also carry the genes for binary toxin. These strains have epidemiologic importance but their diagnosis and therapy does not differ from the more common strains. Patient attributable mortality is high but not 50%, mortality is probably less than 7% overall, however, mortality rates in excess of 10% are found in the oldest patient populations over the age of 80. Disease occurs primarily in elderly hospitalized patients rather than younger community patients.

A 65‐year‐old man is seen for pneumonia. He has had a non‐productive cough, fever, headache, and anorexia for five days. He says two friends currently are hospitalized for pneumonia at another institution. The last time he saw his hospitalized friends was three weeks ago when they gathered with two other friends to have dinner at a friend's rural home. Now three of 6 individuals at that gathering have acute, febrile pneumonias. He remembers the evening well because the family cat gave birth to a litter in the same room during the dinner. T 102.2°F; P 70; BP128/88; RR 28. Chest clear. WBC 6,200 with normal differential. Platelet count 55,000 AST and ALT 3x normal X‐ray = infiltrate right lower lobe. Which one of the following is the most likely cause of pneumonia in the three friends? A. Streptococcus pneumoniae B. Legionella pneumophila C. Coxiella burnetii D. Listeria monocytogene E. Brucella species

Correct answer: C. Coxiella burnetii The incubation period for Q Fever (Coxiella burnetii) is usually 14‐22 days (range 9‐39 days) while Legionella is shorter (2‐10 days). Listeria and Brucella don't cause pneumonia typically, so they don't fit although Brucella has an incubation period similar to Q fever. The cluster and incubation period do not fit pneumococcal disease. Humans acquire Q Fever pneumonia due to Coxiella burnetii by inhaling infected aerosols from animals, particularly cattle, goats and sheep, but also cats, dogs, and rabbits. In infected animals, high concentrations of Coxiella are found in the placenta, so aerosols at the time of parturition are often the source of human infection. The incubation period is about 3 weeks. Simultaneous pneumonia and hepatitis should always raise the possibility of Q Fever. Thrombocytopenia and a temperature‐pulse disparity are common. Common source outbreaks of pneumococcal pneumonia are rare and usually seen with severe crowding as in jail settings. Legionella can cause pneumonia, hepatitis, and a temperature‐pulse disparity, but the incubation period here is too long and most patients with Legionnaire's have leukocytosis.

You are consulted about three rugby players from the same team who have skin lesions. The skin lesions have been present for two to three days. Each player has 10 to 20 raised, clustered lesions on the face, neck, and arms that are about 2 to 5 mm in diameter and filled with a clear yellow fluid; there is a small ring of erythema around the base of each lesion. The athletes say the lesions are mildly uncomfortable but not pruritic; they are very minimally tender. Three days before the lesions were noted by the first athlete, they had engaged in a rugby match after which they attended a party and bathed in a hot tub. Which one of the following is the most likely cause of the skin lesions? A. Pseudomonas B. Mycobacterium C. Herpes simplex D. Contact dermatitis E. Molluscum

Correct answer: C. Herpes simplex Uncomfortable vesicular lesions suggest a herpes virus. However, the cluster of lesions is the key to the clinical presentation. So‐called "herpes gladiatorum" refers to outbreaks of herpes simplex skin lesions seen in wrestlers and rugby players who have prolonged traumatic skin‐to‐skin contact. Lesions are most common on the head, neck and arms. Pseudomonas folliculitis causes pruritic lesions in areas covered by bathing suits (also known as "hot tub buns," which are pruritic but not painful). Mycobacterial infectious can occur associated with hot tubs but the lesions are not vesicular, cases are sporadic, and the incubation period is longer than seen here. Contact dermatitis may produce small vesicular lesions but they are pruritic. Molluscum lesions are not vesicular and not readily transmitted person to person. MRSA can spread among football players and wrestlers but would not cause clusters of vesicular lesions

Which one of the following tests would provide the most sensitive and specific test for the diagnosis of Clostridium difficile diarrhea? A. Common antigen B. Anaerobic culture C. PCR D. Enzyme immunoassay (EIA) E. Latex agglutination test

Correct answer: C. PCR Most laboratories do either a single PCR test, or a combination of common antigen (glutamate dehydrogenase) test plus EIA (enzyme linked immunosorbent assay for toxin). For the latter combination performed on one sample, concordant results are reported as positive (both tests are positive) or negative (both tests are negative). If there is discordance (one positive and one negative) then PCR is done to resolve this discordance. The FDA approved PCR test for the C. difficile toxin B gene is the most sensitive and specific test available. Cytotoxicity testing in tissue culture was formerly the gold standard for detection of toxins but is associated with some false‐negative and false‐positive reactions. Moreover, such testing is expensive, non‐standardized, and time consuming. The EIA tests have been found to be specific but insensitive. There is no evidence that more than one test is required (in contrast to detection of enteric parasites). The latex test for C. difficile GDH by itself is sensitive but not specific. Culture for C. difficile is sensitive but nonspecific as a second test is required to detect either the toxin gene or toxin secretion by the isolated C. difficile strain

A 57‐year‐old medical school research scientist is seen for a febrile illness. Four days ago he was bitten on his hand by a laboratory rat. Last evening he had a fever, and today he has fever, chills, myalgias, and a painful left knee. On exam he is febrile. The bite wound is largely healed and has no evidence of infection. His left knee is swollen with obvious effusion and some pain on both active and passive motion. He has a petechial rash over both shins, and it is also present on the soles of his feet. Which one of the following is the most likely cause of his illness? A. Leptospira interrogans B. Spirillum minus C. Streptobacillus moniliformis D. Hantavirus E. Pasteurella canis

Correct answer: C. Streptobacillus moniliformis Rationale: Rat‐bite fever due to Streptobacillus moniliformis is one of the most common zoonosis associated with laboratory rats. While few cases are reported, it is estimated that 22,000 rat bites occur in the US annually, and that 10% are associated with rat bite fever. S. moniliformis has also caused waterborne outbreaks, called Haverhill Fever. Patients have acute, systemic illness with fever that may be relapsing. Many patients have rash which may be morbilliform or petechial commonly occurring on the extensor surfaces of the extremities and may involve the palms or soles. Many patients have frank arthritis of one or more joints. Given the severity of rat bite fever and the proposed frequency of this illness following rat bites, prophylaxis with 3 days of penicillin following the bite would be reasonable but is not known to be effective and is not a universally endorsed approach. Spirillum minus, the other cause of rat‐bite fever which occur in Asia, has an incubation period typically of 1‐3 weeks and is associated with an ulcerative lesion at the site of the rat bite. Leptospirosis is not associated with arthritis nor is hantavirus. Pasteurella would be expected to have evidence of infection at the site of the bite.

An HIV‐infected patient (CD4 = 20) presents with the acute onset of deteriorating visual acuity without ocular pain. He has been seronegative for CMV and Toxoplasmosis as recently as two months ago. Funduscopic appearance is shown. The most likely etiology of this is: A. Bartonella B. CMZ C. VZV D. EBV E. HHV 6

Correct answer: C. VZV This is Peripheral Outer Retinal Necrosis (PORN) which is most commonly caused by VZV in patients with CD4<50 cells. HSV and occasionally CMV can also cause this syndrome. This syndrome involves the outer layer of the retina. Acute retinal necrosis is caused by VZV or HSV, but occurs in patients with CD4>100. The serologies suggest that CMV or Toxoplasma are unlikely to be the cause of this syndrome. Admittedly the ophthalmologist will provide details about the funduscopic findings, but the clear visualization of the retina suggests that there is no aqueous or vitreal inflammation which would be more typical of syphilis. Comparison of Acute Retinal Necrosis (ARN) and Progressive Outer Retinal Necrosis (PORN) Acute Retinal Necrosis Immunocompetent or CD4 > 100 VZV most common, but also HSV, CMV Resents peripherally with pain, floaters Unilateral but can become bilateral if untreated Retinal detachments common Acyclovir effective for arresting disease Maintenance with valacyclovir Peripheral Outer Retinal Necrosis HIV patients with CD4<50‐100 Multifocal with little inflammation Often involves optic nerve Often associated with retinal detachment, blindness Therapy rarely successful: ganciclovir plus foscarnet recommended. Intravitreal injections also have been used.

A 27-year-old female presents with a painful, swollen knee and fever of three days' duration. A knee aspirate reveals 100,000 wbc/hpf and no organisms. Culture is pending. She has had several recent sexual contacts. A cervical swab for NAAT is positive for gonorrhea She has a similar episode one year prior that was documented to be gonococcal arthritis. Which of the following tests is most likely to reveal the reason for her gonorrhea to disseminate on two occasions? Serum IgASerum IgGHIVCH50Nitroblue tetrazolium (NBT) test on neutrophils

Correct answer: CH50 Disseminated gonococcal (DGI) is occasionally associated with deficiency of terminal complement components that can be detected through screening the total hemolytic complement level. DGI has not been associated with other host defense deficiencies. Thus, recurrent DGI should suggest the possibility of a complement disorder. Empiric management is appropriate since the causative organism is isolated in a minority of patients in terms of blood or joint fluid cultures. However, urethral or rectal or oral cultures may be useful: half of patients have a positive urethral or cervical or rectal culture or NAAT (nucleic acid amplification test) at the time of dissemination.

This retinal lesion in an AIDS patient with a CD4 count of 20 cells/uL is most characteristic of which organism: CMVPneumocystisSyphilisToxoplasmosis

Correct answer: CMV The "mustard and catsup" retinal lesions with contiguous spread along perivascular areas, plus necrosis and hemorrhage in the absence of vitreous flare is typical of CMV retinitis in HIV-infected patient with CD4 count below 50 cells/uL.

Look at this funduscopic photo from a patient with HIV infection and a CD4 count = 10 cells/uL The most likely cause of this lesion in a patient with HIV infection, CD4 count =10 cells/uL, would be: ToxoplasmosisSyphilisCMVHSVVZV

Correct answer: CMV There is a long list of causes of retinitis in a severely immune deficient patient with HIV. CMV is by far the most common, especially in a patient with little anterior or posterior chamber inflammation (the fundus can be clearly visualized here) and with the combination of hemorrhages and exudates. This is probably the only one you would be asked to recognize on an exam, although you might be expected to think of other entities especially syphilis if vitreal haze makes the funduscopic exam blurry. Infectious disease specialists should know the differential diagnosis, and the basic findings of the common retinal syndromes of CMV, HSV, VZV, Toxoplasma, and syphilis.

An outbreak of illness was reported among approximately 50 persons eating at an area restaurant. The illness consisted of nausea (97%), vomiting (97%), abdominal cramps (86%), chills (78%), muscle aches (67%), fever (64%), headache (61%) and diarrhea (58%).The median incubation period was 31.3 hours, one ill person was hospitalized and 10 sought medical care.Eating a "sub" sandwich at the restaurant was associated with illness (Relative Risk = 14.5).One person seeking medical attention tested positive for E. coli O157:H7. The illness lasted approximately 48 hours with some malaise thereafter.What was the likely cause of the outbreak? Calicivirus/norovirus (e.g. Norwalk virus) Shiga toxin-producing E. coli O157:H7 (STEC) Campylobacter Enterotoxigenic E. coli (ETEC) Preformed Staphylococcus aureus enterotoxin (food poisoning)

Correct answer: Calicivirus/norovirus (e.g. Norwalk virus)The illness meets Kaplan criteria for a norovirus: vomiting predominated as a finding seen in >50%, the incubation period was between 24-48 hours and illness lasted between 12-60 hours. This could be confirmed by stool PCR.This is not STEC (also called EHEC, enterohemorrhagic E. coli), which is more severe. In the face of a STEC outbreak affecting this many persons, hospitalization and hemolytic uremic syndrome (HUS) likely would have been seen. Vomiting is not the predominate finding in STEC disease and a high percentage of persons would have dysentery (passage of bloody stools). While the one person who tested positive for E. coli O157:H7 probably had infection by an STEC strain, the laboratory finding did not predict cause of the outbreak.For Campylobacter, diarrhea and dysentery should have been reported in many, vomiting would not predominate as a finding and illness would have lasted longer. This is not ENTEROTOXIGENIC ESCHERICHIA COLI (ETEC): ETEC typically causes watery diarrhea, but not prominent nausea and vomiting. The incubation period is too long for Staphylococcal food poisoning which is between 2-7 hours.

A 29-year-old woman has the rapid onset of ascending paralysis. Her CSF exam is normal except for an elevated protein (90 mg/dL). Two weeks before her weakness began, she had an influenza immunization. Ten days prior to the onset of weakness, she had a febrile gastroenteritis for which she received a fluoroquinolone. Her fever and diarrhea resolved after three days. Which one of the following is the most likely cause of her paralytic illness? Influenza vaccine Cytomegalovirus Campylobacter Mycoplasma Fluoroquinolone

Correct answer: Campylobacter The most common cause of acute ascending neuromuscular paralysis is Guillain-Barré Syndrome. The disease is symmetrical and slowly progressive from most distal to proximal. Lack of reflexes and dysautonomia are characteristic. There is no specific diagnostic test although the CSF often shows an elevated protein but normal number of cells (albuminocytologic dissociation). The most common associations/causes of Guillain-Barré Syndrome are antecedent campylobacter, HIV, or influenza, although many different triggering events have been reported. Zika virus infection has been reported to cause Guillain-Barré Syndrome. An increased incidence of Guillain-Barré Syndrome followed influenza immunization for swine flu 1976-77. Subsequent studies have demonstrated no risk of influenza immunization or a very tiny risk (less than one additional case per million vaccinations). Fluoroquinolones may produce a variety of neuropsychiatric side effects, but not Guillain-Barré.

A 78-year-old man complains of pain on the roof of his mouth when he inserts his upper dentures. He has worn dentures for many years and has never had this problem before. He has no pain on swallowing. He was recently placed on a non-steroidal for pain secondary to osteoarthritis. On exam, after removal of his upper denture, there is diffuse erythema of his hard palate without any evidence of white plaques or ulcerations. The rest of the oral cavity appears normal. Which one of the following is the most likely cause of his problem? Herpesvirus Denture trauma Cytomegalovirus NSAID reaction Candida

Correct answer: Candida Oral candidiasis (thrush) typically presents as the so-called pseudomembranous form with white plaques on the buccal mucosa, palate, tongue, or oropharynx. "Denture stomatitis," or the atrophic form, is seen in older adults who wear dentures. They commonly present with pain and erythema under upper dentures without white plaques. The location and absence of vesicles or ulcers is against herpes virus. Denture trauma would be unlikely in someone who has worn dentures for many years. Cytomegalovirus would not cause hard palate infection in an otherwise normal host. An NSAID reaction

A 24-year-old female had completed her fifth course of cytotoxic chemotherapy for a poorly differentiated osteosarcoma, had an absolute neutrophil count of 5/cu mm and platelet count of 7,000/cu mm when she developed the sudden onset of fever to 40C° but was otherwise stable. Two blood cultures were drawn through the tunneled subclavian catheter, and one culture was drawn peripherally. The Hickman catheter exit site and tunneled area were non-tender and look unremarkable on physical examination. The laboratory reports the next morning that the 2 blood cultures drawn through the line are positive for an organism. The peripheral culture is negative at 24 hours. Lack of another source of infection has raised the likelihood that the patient has catheter-acquired sepsis. The patient and the oncologist are hopeful that the tunneled subclavian catheter might be salvaged, particularly while the patient is thrombocytopenic. Which of the following organisms would make removal of the Hickman tunneled catheter mandatory? Candida albicans sensitive to fluconazole Enterococcus faecalis sensitive to ampicillin E. coli sensitive to cephalosporins Streptococcus mitis sensitive to penicillin Staphylococcus epidermidis sensitive to vancomycin

Correct answer: Candida albicans sensitive to fluconazole All temporary lines that appear to be a source for sepsis should be removed, regardless of the likely or documented causative organism. This patient had a tunneled subclavian catheter, which requires different management. For tunneled catheters, if the patient has refractory shock (the definition of which varies from physician to physician), the catheter should be removed promptly if it could be the source of sepsis, assuming that some other venous access can be obtained. Data suggest that many cases of uncomplicated line-related bacteremia can be managed without removing the line although wise management always supports removing the line if that is technically feasible. This patient is not hypotensive, nor does the patient have evidence of persistent bacteremia or embolic phenomena or a tunnel infection. Thus, the infection could be managed medically for most bacteria, but medical management is so unlikely to be successful for Candida that the line should be removed at the earliest possible time.

A 72-year-old diabetic male from Chicago, recently on etanercept for psoriasis, is seven days post-resection of an adenocarcinoma of the descending colon. He is intubated and has a central venous line in his internal jugular vein for hyperalimentation. He spikes a fever to 39.8°C and two blood cultures are collected, one through his central line and the other peripherally. After two days of incubation, yeasts are growing in the aerobic bottles of both blood cultures. Which of the following fungi is the most likely? Blastomyces dermatitidis Candida parapsilosis Cryptococcus neoformans Histoplasma capsulatum Malassezia furfur

Correct answer: Candida parapsilosis The yeasts in this list that would be most likely to grow within two days are Candida species and thus Candida parapsilosis is the best answer. This could be any Candida species; C. parapsilosisis is commonly associated with hyperalimentation. M. furfur is also associated with hyperalimentation but would not typically grow in two days; further, the culture medium should be supplemented with fatty acids for optimal recovery of this organism. C. neoformans grows in routine blood culture bottles but generally requires three to five days of incubation. H. capsulatum and B. dermatitidis will grow in blood cultures but as a mold and require prolonged incubation. A better method for recovering dimorphic fungi would be to collect blood in an Isolator tube and culture the sediment on fungal media incubated at 25-30°C. Dimorphic fungi will grow as molds (filamentous fungi) in one to two weeks.

Consultation was requested because of low-grade fever in an 18-year-old man from St. Louis, Missouri. He had successful remission induction for acute lymphocytic leukemia with return to normal of peripheral neutrophil counts. He had become febrile while profoundly neutropenic. The fever had not responded to 5 days of ceftazidime, but began to respond when amphotericin B was added and his neutrophil count rose to normal. Daily low-grade temperatures of 38°C began after he had been discharged on no therapy. Physical examination was unremarkable except for the Hickman catheter, which was still in place. Routine laboratory work had shown a slight increase in his alkaline phosphatase to 180 U/L but no other liver function abnormalities. Chest x-ray was normal. Ultrasound found numerous nodular lesions in the liver ranging up to 15 mm diameter. Which is the most likely diagnosis? Candidiasis Histoplasmosis Aspergillosis Fusariosis Acute leukemia

Correct answer: Candidiasis This patient has a high likelihood of having chronic disseminated (hepatosplenic) candidiasis which is seen when neutrophils return after a period of neutropenia. Diagnosis can be confirmed by liver biopsy but empiric therapy is often done. Other mycoses rarely cause this clinical scenario with nodular liver lesions. CT of the abdomen should be done when fever of unknown origin follows recovery from neutropenia, particularly in light of abnormal liver function. For hepatosplenic candidiasis, liver, spleen, or kidney may have necrotic lesions, ranging in size from barely visible to 2-3 cm diameter. Margins are usually contrast enhancing. Although ultrasound can visualize some lesions, contrast-enhanced CT is more sensitive. Appearance of these lesions in the appropriate clinical setting is sufficient to warrant amphotericin B therapy. Fluconazole has been used late in the course of treatment, provided the neutropenia does not return. Although bone marrow biopsy would be helpful in determining early relapse of leukemia, fever from acute lymphocytic leukemia would be uncommon in relapse not yet detectable in the peripheral blood. Acute leukemia in relapse often causes diffuse infiltration of the liver but nodular lesions don't occur. Blood culture by lysis centrifugation is particularly helpful with recovery of fastidious organisms, such as certain fungi and mycobacteria, but adds little to the management of this case.

A 47-year-old male presents with confusion and fever for 2 days. The patient's family tells you he was bitten by a dog on his left thumb 3 days prior to developing symptoms. He has a history of splenectomy as an adolescent after blunt abdominal trauma. On physical exam, the temperature is 102.4°F and blood pressure 90/60 with norepinephrine vasopressor support. The patient is intubated and mechanically ventilated due to altered mental status. His skin is notable for palpable, non-blanching, purplish lesions over the arms and legs. There is a laceration in the left thumb with mild surrounding erythema and no purulence. His WBC count is 16500, platelets are 19,000, he has acute renal failure, and he has a laboratory picture of disseminated intravascular coagulation. The mostly likely microbial cause of this syndrome is which of the following: Pasteurella multocida Streptococcus mitis Capnocytophaga canimorsus Fusobacterium spp. Pasteurella canis

Correct answer: Capnocytophaga canimorsus Capnocytophaga canimorsus is a fastidious Gram negative rod that is part of the normal oral flora of dogs and cats. It is a cause of fulminant sepsis in those with history of splenectomy, chronic alcohol abuse, or immunosuppression. The associated mortality can be as high as 35%. This organism may take several days to grow in cultures. Peripheral blood smear may show fusiform rods. Combinations of β-lactam/β-lactamase inhibitor agents is the treatment of choice, other alternatives are ceftriaxone, carbapenems, or clindamycin. The microbiology of infected dog bites is complex, mixed aerobic and anaerobic bacteria are present in the majority of infections if appropriate culture techniques are utilized. Pasteurella spp. is the most common isolated pathogen isolated from both dog and cat bites. Pasteurella canis is the most common isolate of dog bites. Pasteurella multocida subspecies multocida is the most common isolate of cat bites. The next most frequently encountered aerobic organisms are streptococci and staphylococci. The most common anaerobic organisms isolated are Fusobacterium spp., Bacteroides spp. (in particular B. tectum), Porphyromonas spp., Prevotella spp., Propionibacterium acnes, and Peptostreptococcus spp. The clinical picture seen with the listed Pasturella species and Fusobacterium infections is less severe and there is no increased risk of severe disease in those with a history of splenectomy.

This 4-year-old child was brought to a rural clinic in Brazil because his mother noticed this swelling around his eye. The child had a low grade fever and, according to the mother, had been irritable. The eye appeared to be normal. If untreated, this child may present many years later with a: EncephalitisCardiomyopathyCranial Nerve PalsyLymphedemaEosinophilia

Correct answer: Cardiomyopathy Swelling around the eye of children in Brazil and other endemic areas of South America should raise the question of chagoma, called Romaña's sign. The lesion arises when a reduviid bug crawls out of mud walls and other dark corners of poorly constructed adobe huts. The bug defecates while taking a blood meal on an exposed skin surface, often the face. Young children are most likely to not swat the bug away. Trypanosoma cruzi in the excreta are rubbed into the bite wound, conjunctiva or abraded skin surface and cause a local inflammation with swelling of adjacent nodes, such as the preauricular node. Acute Chagas' disease symptoms range from mild to severe. Many years later, chronic Chagas' disease manifests as cardiomyopathy, megacolon or megaesophagus. Encephalitis can be seen in acute Chagas' but not the chronic phase. Lymphedema and cranial nerve palsies are not seen. Eosinophilia occurs with helminths, not protozoa.

This man with AIDS presented with low grade fever and tender skin lesions. Assume that these are not due to Kaposi sarcoma. The microorganisms most likely to be causing these lesions have also been found in which of the following: Dog's bloodTicksLiceCat's blood

Correct answer: Cat's blood This is bacillary angiomatosis or BA, caused by Bartonella henselae, the agent of cat scratch disease. The fleshy red, well-circumscribed lesions can be mistaken for Kaposi's sarcoma but are entirely different on biopsy. Vascular proliferation with bacilli evident on immunoperoxidase or Warthin Starry stain are characteristic of BA. Low grade fever accompanies onset of skin lesions. Most patients have had HIV infection with CD4 counts <200 cells/uL. In many regions of the United States, B. henselae has been isolated from the blood of apparently healthy feral cats. Transmission between cats is by fleas. The exact route of transmission from cats to humans is less clear, but perhaps by nips or scratches by the cat or cat fleas.

A 60-year-old male who had chronic lymphocytic leukemia (CLL) presented with twoweeks of fever and chills and 0.5-2.0 cm purple papules on his extremities. These papules appeared just after the onset of fever. Lab: WBC 30,000 with 95% lymphocytes and 5% neutrophils; Chem 12 normal. The patient was receiving prednisone 10 mg daily and chlorambucil for his CLL. He had several recent anonymous sexual exposures,and enjoyed fishing and camping. He had recently helped clean an alley behind an inner city property he was renovating. Hehad some scratches and perhaps bites from some rats, mice, and feral cats. The routine stain of the skin biopsy (hematoxylin and eosin stain) showed lobular collections of plump endothelial cells with vascular proliferation interspersed with an inflammatory infiltrate composed mainly of neutrophils. Fungal stains and tissue Gram stain showed no bacteria. Warthin-Starry staining identified numerousbacilli. The pathogen causing this lesion was probably transmitted to the patient by: Cats Ticks Lice Rat bites Mosquitos

Correct answer: Cats This patient has bacillary angiomatosis, usually due to Bartonella henselae. The vector appears to be cats that have chronic bacillemia without illness. Fleas can transmit the infection among cats, butflea transmission to humans occurs less frequently. Human body lice can also transmit the other agent of bacillary angiomatosis, Bartonella quintana. Although most patients with bacillary angiomatosis have HIV/AIDS, other immunosuppressed patients have been reported to develop the disease including those with hematologic malignancies. Bacillary angiomatosis typically presents as purple vascular lesions that appear similar to Kaposi sarcoma. Most often this disease involves the skin, but it can affect other organs such as the lung, bone, lymph nodes, gastrointestinal tract, and very occasionally the brain. Patients may have constitutional symptoms including fever, chills, anorexia, malaise, headache, and weight loss. Bacillary angiomatosis is adifferent manifestationof bartonella infection than hepatic peliosis, which are cystic blood filled spaces in the hepatic parenchyma seen predominantly in AIDS patients. Patients may have both manifestations concurrently. Bartonella species may also cause other illness including cat scratch fever (acute infection with lymphadenopathy), endocarditis, trench fever and Carrion's disease. Diagnosis is not easy: PCR is often helpful. Histopathology is usually characteristic with the Warthin-Starry stain revealing theorganisms usually in clumps or chains.Blood cultures sometimes are positive if specialculture media especially if isolator tubes or EDTA or fresh blood are used. Serology is supportive but not definitive. Therapy of bacillary angiomatosis is usually doxycycline or erythromycin or azithromycin. Gentamicin or rifampin can be addedfor severe disease or bacteremia.

A 22-year-old previously healthy male presents to the Emergency Room with a 24 hours history of right lower quadrant pain, nausea and vomiting, low-grade fever, and WBC=15,000 (90% neutrophils). A CT diagnoses a non-perforated appendicitis. The surgeon excises an inflamed but non-perforated appendix at laparoscopic surgery. The patient is not known to be colonized with any special pathogen, and has no allergies. What would be the optimal recommendation for antibiotic therapy? Cefazolin for one dose given within one hour preoperatively Cefazolin for one dose preoperative and for 48 hours afterwards Ampicillin-Sulbactam within one hour pre-operatively and for 7-10 days Ampicillin-sulbactam within one hour preoperatively and for 48 hours afterwards Imipenem within one hour preoperatively and for 48 hours afterwards

Correct answer: Cefazolin for one dose given within one hour preoperatively For excision of a non-perforated appendicitis, antibiotic therapy is not clearly indicated over and above routine skin prophylaxis. Thus, a single dose of cefazolin would be adequate given within one hour prior to skin incision, although some clinicians give cefazolin plus metronidazole, or cefotetan, or ampicillin sulbactam as a single dose. The major point here is that there is no need to give more than a single dose of prophylactic drug, regardless of whether you "prefer" to include coverage for bowel flora. Thus, the other answer choices include longer courses of drugs which are not necessary. For a perforated appendix, the initial antibiotic choice should include likely bowel flora: the choice of drug and duration of administration will be driven by culture results, adequacy of drainage, and clinical course.

An 18-year-old woman presents to her primary care physician's office complaining of lower abdominal pain with low-grade fevers and chills. She denies any other symptoms. Her last menstrual period was two weeks earlier, and it was normal. She is in a monogamous relationship with a male partner. On examination, her temperature was 38.3°C, and she has mild abdominal tenderness on deep palpation, and cervical motion tenderness on bimanual examination. Her urine pregnancy test was negative, and a wet mount of vaginal secretions was unremarkable. In addition to doxycycline, which of the following antibiotics is most appropriate to treat her infection as an outpatient? Ertapenem Cefotetan Azithromycin PO Levofloxacin PO Ceftriaxone IM

Correct answer: Ceftriaxone IM The patient has pelvic inflammatory disease based on the presence of cervical motion tenderness on examination. A single physical finding (cervical motion tenderness, uterine tenderness, or adnexal tenderness) is sufficient to make a clinical diagnosis of PID. The presence of fever (a minor criterion) does increase the specificity of the diagnosis, but it decreases the overall sensitivity of the diagnosis (that's why minor criteria are no longer part of the clinical diagnostic criteria). The patient has mild to moderate infection, is not pregnant, is not vomiting, and there is nothing to suggest that she will not be able to follow-up within 72 hours. As such, outpatient management is appropriate. She needs to be covered for both gonorrhea and chlamydia +/- anaerobic coverage. Doxycycline appropriately covers chlamydia. 250mg of intramuscular ceftriaxone with 14 days of oral doxycycline is a recommended regimen. Cefotetan and ampicillin/sulbactam are both appropriate to use but they are intravenous. There is no reason to use an intravenous regimen in this patient. There is also no reason to use a broad spectrum drug such as ertapenem. Azithromycin plus doxycycline is not a first-line recommended regimen due to the limited efficacy data and the gastrointestinal side effects of azithromycin when used to treat gonococcal infections. Given the increasing resistance of gonorrhea to fluoroquinolones, the regimen of Levofloxacin PO is no longer recommended for the treatment of PID.

A 72-year-old man develops fever, abdominal pain, and unstable blood pressure 4 days after a subtotal colectomy for carcinoma of the colon. Empiric therapy with ceftriaxone and vancomycin is initiated.. Within hours the Clinical Microbiology laboratory reports that the patient's blood cultures are positive for enteric gram-negative rods, preliminarily identified as Klebsiella species. Susceptibility results are not yet available. What would be the best recommendation for antibiotic management pending further susceptibility data? Change ceftriaxone to piperacillin-tazobactam .Change ceftriaxone to colistin Continue ceftriaxone and add ciprofloxacin. Change ceftriaxone to cefepime. Change ceftriaxone to meropenem

Correct answer: Change ceftriaxone to imipenem Klebsiella pneumoniaeis the most common pathogen to carry plasmids encoding extended spectrum beta-lactamases (ESBLs). In an unstable patient with a nosocomial infection, the possibility of such a resistant strain should be considered. The most reliable coverage of such ESBL-producing strains is a carbapenem, such as meropenem or imipenem. A less likely possibility is infection due to a K. pneumoniaeproducing carbapenemase (KPC). As KPCs spread, the answer to this question may be colistin or polymyxin B as part of combination therapy. . At present, the toxicity of colistin and polymyxin B is sufficient to withhold polymyxins until susceptibility results are provided. The production of ESBLs consistently results in hydrolysis of the beta-lactam rings of ceftriaxone and aztreonam. The activity of cefepime versus ESBL positive Enterobacteriaceae depends on the in vitro MIC. The MIC reflects the summation of all the resistance mechanisms that may be present:e.g., ESBls, efflux pumps, changes in drug target, and impaired cell wall permeability. Ticarcillin clavulanate is no longer available anywhere on planet earth. Pipercillin-tazobactam may appear active in vitro but fail clinically. It is believed this discordance is due to the low inoculums used in vitro vs a high inoculums during active systemic infection. Finally, ESBL resistance has been linked epidemiologically and biologically to ciprofloxacin resistance. Thus, adding ciprofloxacin to ceftriaxone would also not reliably cover a possible ESBL-containing strain of K. pneumoniae.

A 25-year-old HIV infected man is seen for routine HIV care. He has received his initial regimen with tenofovir (TAF), emtricitabine, and efavirenz for the past year but has a history of a poor response and currently has a viral load that is 10,000 copies on repeated determinations despite his assurances of adherence. Resistance profile on his current regimen includes: Nucleoside: M184V Non nucleoside K103N Phenotype: fully susceptible to all protease inhibitors, AZT, D4T He is hepatitis B surface antigen (HBsAg positive). The most prudent management strategy for this patient would be: Discontinue his current regimen and repeat the genotype after he has been off antiretrovirals for four weeks .Change his regimen to Abacavir, darunavir-ritonavir, and Raltegravir. Change his regimen to Zidovudine, Lamivudine, darunavir-ritonavir .Change his regimen to Tenofovir, FTC, darunavir-ritonavir. Change his regimen to Abacavir, Lamivudine, darunavir-ritonavir.

Correct answer: Change his regimen to Tenofovir, FTC, darunavir-ritonavir. After virologic failure on an NNRTI-based regimen, boosted protease inhibitor regimens offer significant virologic activity. However, in this patient, you want to optimally treat his 2 viral infections: HIV and HBV. If this patient is started on an effective antiretroviral regimen and has a substantial rise in CD4 and fall in VL, a hepatitis B flare would be a major concern. Thus, prudent management would dictate that he receive at least two active drugs against hepatitis B. The second and third regimens have NO HBV-active drugs and the fifth choice, with lamivudine (3TC) alone might work for his HIV, but would select eventually for resistance to HBV. The first choice is incorrect: resistance testing should always be done while the patient is on the current regimen. The longer a patient is off the regimen, the more likely wild type virus will emerge on testing, obscuring the archived resistant virus that will reemerge when there is selective drug pressure of a new regimen. The following drugs have Hepatitis B activity: Emtricitabine, Lamivudine, Tenofovir (both TDF and TAF), Adefovir, Entecavir, and Telbivudine. At least two drugs with HBV activity should be used concurrently to reduce the likelihood of HBV drug resistance

A 25-year-old non-pregnant woman is diagnosed with pulmonary tuberculosis, based on positive smears and cultures. She is also found to be HIV-positive (CD4 = 80 and Viral Load = 125000). She was started on daily INH/Rifampin/Ethambutol/PZA, was found to have a pansensitive organism, and after 18 weeks was switched to twice weekly INH and Rifampin. Her internist initiated tenofovir/emtricitabine/darunavir/ritonavir at the standard dosages recommended in the guidelines for patients without TB, not realizing the drug interactions could be an issue. He only looks at the tables in the guideline summary, and does not have time to read the text!! What regimen would you advise, assuming that she is using barrier contraception consistently? Continue current ARVs plus once-weekly INH and rifapentine. Continue current regimen plus twice-weekly INH and rifampin. Continue current regimen plus twice-weekly INH and rifabutin. Continue current regimen plus daily INH and rifampin. Change lopinavir-ritonavir to efavirenz with daily INH and rifampin.

Correct answer: Change lopinavir-ritonavir to efavirenz with daily INH and rifampin. There are two issues here: 1. What is the best ART regimen for a patient who must receive rifampin? 2. What tuberculosis regimen is appropriate? TB drugs must be given at least 3 times per week with CD4 counts <100 cells, efavirenz is more appropriate to give that darunavir/ritonavir, because rifampin significantly reduces the concentrations of allprotease inhibitors. . Given that the patient has <100 CD4 cells, treatment for TB must be given at least 3 times weekly, or rifampin-resistant TB strains may develop. Rifapentine should never be used in anyone known to be HIV-positive, as it has been found to be associated with failure, and the development of rifampin-resistant tuberculosis in that population. This is another reason why A is incorrect. Rifampin should not be used with any HIV protease inhibitor.

This 12-year-old boy was referred because of recurrent skin and deep staphylococcal infections and this abnormal peripheral blood smear, shown in the figure: www.health.auckland.ac.nz504 ×331 What white blood cell abnormality does this patient most likely have? Oxidative burstChemotaxisAdhesionMyeloperoxidase

Correct answer: Chemotaxis This boy has Chediak Higashi syndrome Giant neutrophil granules are characteristic of Chédiak-Higashi syndrome, an autosomal recessive disorder of the lysosomal trafficking gene characterized by recurrent staphylococcal infection, oculocutaneous albinism, and easy bleeding. Peripheral neuropathy and an accelerated lymphoproliferative syndrome develop late in life. Neutrophil chemotaxis is impaired but not the other functions listed.

A 35-year-old man presented with a rash followed by malaise and fever. The rash started on his face and spread to the trunk, extremities and genitalia over 2 days. He has no underlying illnesses. He does not know which childhood immunizations he had. He has no pets, no sick contacts, no travel, no promiscuous sexual activity. Exam: Temperature 37.8°C Pulse 90 BP122/80 He is iIl appearing with injected conjunctiva, oral and genital ulcers, and a vesicular rash mainly on face, back and chest (see images). WBC is 7.5 (normal diff), routine blood work normal. The most likely diagnosis is which of the following: Measles Disseminated HSV Chicken Pox (primary VZV) Smallpox Pemphigus vulgaris

Correct answer: Chicken Pox (primary VZV) This is chicken Pox (Varicella): Constitutional symptoms start AFTER rash (or mild before rash). He likely was vaccinated as a child if he was born after 1995 but...this person was born in 1980 and while the vaccine is highly effective, no vaccine is 100% effective. Characteristic vesicles on erythematous base at DIFFERENT stages. Lesions begin in more central locations. Can have mucous membranes (oral and genital ulcers) involved. Measles Fever and upper respiratory prodrome before rash. Rash starts 2-3 days after onset of fever on head and progresses down, but much different appearance (no vesicles, see image below) and lasts for a week; pruritic. 3 C's: conjunctivitis, coryza, cough. Koplik spots on buccal mucosa on third or fourth day of illness. Rare in this country even if not immunized. He probably was immunized in order to enter school. Rubella: Mild disease that only last 2-3 days Conjunctivitis and runny nose plus arthralgias, especially in younger women Occipital lymphadeopathy Rash starts on face and spreads to trunk and is fine vs. blotchy measles rash Small Pox: Lesions more centrifugal, (see image below) Vesicles/pustules at SAME stage Febrile prodrome BEFORE rash Not possible in the current era unless bioterrorism attack suspected

A 37-year-old woman from St. Louis is referred to you by her rheumatologist for treatment of possible Lyme arthritis of six month's duration. Her symptoms began shortly after a week's vacation in Jamaica. Towards the end of her stay she developed fever, malaise, muscle aches, headache and then a nonpruritic rash over her trunk and extremities. She took a picture of the rash with her cell phone (see figure). As her rash and fever faded away, she developed joint pains in her hands, feet, ankles and wrists. This has persisted to the present. The most likely useful diagnostic test is a serology for: O'nyong-nyong virus STARI Measles virus Chikungunya virus Borrelia burgdorferi

Correct answer: Chikungunya virus Chikungunya is a mosquito borne illness, once formerly in Asia but now in South and Central America, and in the Caribbean including Jamaica and Puerto Rico, and extending into Southern Florida. Illness resembles dengue, with fever and rash, but is followed by arthritis, particularly of the extremities, that can be severe and last up to a year or two. O'nyong-nyong causes a similar illness in Africa but not the Caribbean. STARI, or Southern Tick Associated Rash Illness, has fever and rash but not arthritis. Measles rash appears at onset of fever, is accompanied by cough, coryza and conjunctivitis and not by arthritis. There is no Lyme disease in St. Louis or Jamaica and the acute rash is not erythemia migranas.

A resident working in the emergency room consults you about two young gay males he had seen recently with proctitis, associated with bloody mucus and pain so severe on defecation that the patients became constipated. They also reported low-grade fever and malaise. Both patients, who presented a week apart, were friends and admitted to having unprotected sex in "leather scene" meeting places. Anoscopy caused such severe pain that the physician could not visualize the rectal mucosa. The remainder of the physical examination was normal, with no skin lesions, lymphadenopathy, or genital or oral lesions. He suspected rectal herpes simplex on the first patient but there was no response to valacyclovir and the rectal swab sent for HSV and GC was negative. He tried an injection of ceftriaxone on the second patient but the patient returned a few days later without improvement, at which time he prescribed valacyclovir. They are scheduled for a return visit, and he asks what tests to perform. You advise him to test for which of the following? Chlamydia trachomatis Entamoeba histolytica Haemophilus ducreyi Helicobacter cinaedi Treponema pallidum

Correct answer: Chlamydia trachomatis Proctitis in MSM is most often caused by gonorrhea or chlamydia. v Cases of lymphogranuloma venereum (LGV), i.e., C. trachomatisof serovars 1, 2 or 3, have been described throughout the U.S. mainly among men who have unprotected rectal intercourse with other gay men. The usual painless penile ulcer and painful inguinal adenopathy of LGV in males is absent, and the urethral discharge and urine test for C. trachomatisis usually negative. Diagnosis can be made during the acute illness by rectal swab using PCR . Most commercial laboratories perform a multiplex PCR that distinguishes the LGV strains from the more typical D-K C. trachomatisstrains. The NAAT on a rectal swab can also be used, although this test is not approved by the FDA. A serum complement fixation titer of at least 1:64 is highly suggestive. Treatment is with doxycycline 100 mg po bid for 21 days. Proctitis caused by the D-K strains is usually treated for only 7 days. Sexual partners should be screened for both gonorrhea and chlamydia even if asymptomatic. Helicobacter cinaedicauses diarrhea in gay men but not proctitis or colitis. Amoebiasis ulcers and syphilis chancres in the rectum are not as painful as proctitis due to LGV, HSV or gonorrhea. Chancroid, due to H ducreyi, does not cause proctitis.

A 32-year-old man with a history of urethritis 3 weeks earlier presents with the manifestations seen in the 3 images below. He has swelling of one knee, sausage digits, oral ulcers and nail changes: Prompt treatment of which acute infection would likely have prevented (or reduced the likelihood) of these complications which occurred: Campylobacter jejuni Yersinia pseudotuberculosis Salmonella enteritidis Chlamydia trachomatis Herpes simplex virus

Correct answer: Chlamydia trachomatis The diagnosis is reactive arthritis (formerly known as Reiter's syndrome). The patient presents with conjunctivitis, keratoderma blennorrhagica (hyperkeratotic lesions on palms and soles), and enthesitis (classic symptoms are heel swelling and sausage digits). Other manifestations include oral ulcers, nail changes (that resemble psoriatic changes), and occasionally circinate balanitis. The classical pathogens associated with reactive arthritis are: Chlamydia trachomatis, Yersinia, Salmonella, Shigella and Campylobacter. Typically, the arthritis is an asymmetrical mono or oligoarthropathy. Studies suggest that less than 50% of patients with reactive arthritis are HLA-B27 positive. Prompt treatment of C. trachomatis infection has been shown to decrease the risk of reactive arthritis in that setting; long-term antibiotics for reactive arthritis have not been shown to improve clinical outcomes. Treatment of no other infections have been shown to decrease the risk of reactive arthritis. NSAIDS are the drugs of choice to control inflammatory symptoms. Systemic corticosteroids are rarely needed.

A 40-year-old male, a recent emigrant from Central America previously in good health, presented with 6 months of sore throat, dysphagia, shortness of breath, nasal congestion and a non-productive cough. He had no fever. Swallowing became difficult and liquids often came out his nose. Physical examination was not remarkable except for fleshy masses filling both nasal cavities. His throat was normal. Labs: unremarkable including complete blood count and differential. On endoscopy, erythematous masses partially obstructed both nares and the epiglottis. A biopsy of the epiglottis shows plump gram negative rods in macrophages but only normal flora grew from the biopsy. The best medical course of therapy would be: CiprofloxacinStreptomycinDapsone and RifampinINH, PZA and RifampinItraconazole

Correct answer: Ciprofloxacin The gram negative rods within macrophages suggests rhinoscleroma, caused by Klebsiella pneumoniae ssp rhinoscleromatis. It is not unusual for cultures to be negative. The treatment of choice is long courses of quinolones or tetracyclines. These lesions can be seen anywhere in the upper respiratory tract, including the oropharynx, nose and sinuses. The other answers point toward leprosy (rifampin plus dapsone) or fungi (itraconazole) or a vasculitis such as granulomatosis with polyangiitis (Wegener's), but that would not explain the gram negative rods. It would be important for the clinician to make sure appropriate stains were done in a suggestive case of rhinoscleroma. Most cases of rhinoscleroma are documented in Latin America and Asia. It is important to be aware of this disease since stains can demonstrate the organism if they are requested, and the histopathology is often the major clue since cultures can be negative in a substantial number of cases, at least in the geographic locales where cases are reported. Lymphoma, midline granulomatosis, rhinosporidiosis, granulomatosis with polyangiitis, mucocutaneous leishmaniasis and paracoccidioidomycosiscan cause granulomatous lesions in the nasal cavity, but the gram negative bacilli in this specimen make the diagnosis most likely to be rhinoscleroma.

This soft tissue film of a person's leg several days following penetrating trauma while digging a house foundation. He had not sought medical attention. The x-ray is most consistent with which of the following? Staphylococcal pyomyositisStreptococcus pyogenes necrotizing fasciitisClostridial myonecrosisAeromonas myonecrosisVibrio vulnificus

Correct answer: Clostridial myonecrosis Gas in multiple fascial planes is highly suggestive of clostridial myonecrosis and would not be expected in the other entities listed. This radiograph could also be found in patients with anaerobic streptococcal (peptostreptococcal) myonecrosis or synergistic non clostridial mixed anaerobic/aerobic necrotizing fasciitis with myonecrosis(e.g. Meleney's synergistic necrotizing fasciitis). . Aeromonas myonecrosis is associated with fresh water or fish, and is NOT associated with gas formation in tissue. For Vibrio vulnificus, look for liver disease and raw oyster ingestion or soft tissue trauma with brackish or salt water exposure. Gas gangrene caused by clostridia occurs in two situations: 1) muscle injury and dirt contamination with the spores of Clostridium perfringens and others species; 2) bacteremia related to neutropenia or a cryptogenic colonic lesion complicated by bacteremia due to Clostridium septicum. Note that pyomyositis is a primary muscle infection, usually caused by Staphylococcus aureus, and more common in the tropics. However, this can occur after penetrating injury, or vascular insufficiency but soft tissue gas would be an extremely rare presentation.

A 22-year-old woman comes to the emergency room with 20 hours of progressive abdominal pain, nausea, and vomiting. Four days prior she had a medically induced abortion (6 weeks pregnant) with mifepristone followed by vaginal misoprostol. She has no past medical or surgical history and is not receiving any medications. On examination she is afebrile, tachycardic with a heart rate of 146, blood pressure is 68/43, 26 respirations, and saturating 90% on room air. She has moderate abdominal tenderness. Her WBC is 88000, hematocrit is 53%, normal platelet count, she has a metabolic acidosis, her renal and liver function tests are normal. Chest-X-ray shows bilateral infiltrates and pleural effusions. Over the following 10 hours she develops refractory hypotension and dies. Which of the following is the most likely organism? Group A Streptococcus Group B Streptococcus Clostridium sordellii Staphylococcus aureus Pseudomonas aeruginosa

Correct answer: Clostridium sordellii Infected abortion retained products commonly caused by Clostridium perfringens, polymicrobial infections with other anaerobes and Gram negatives were also commonly seen. C. sordellii is an anaerobic spore-forming Gram positive rod which is an uncommon reported cause of pneumonia, endocarditis, arthritis, peritonitis, genitourinary infections and myonecrosis. C. sordellii bacteremia and sepsis occur rarely, primarily among patients with serious underlying conditions. In recent years there have been multiple case reports of Clostridium sordellii- associated toxic shock syndrome occurring within a few days after medically induced abortions (women who received mifepristone and misoprostol). The pathogenesis involves two large cytotoxins that disrupt crucial cell processes such as cell-cycle control, apoptosis, gene transcription, and the structural functions of actin. Cytoskeletal disruption leads to vascular leak. Characteristic findings have included absence of fever or rash, dramatic leukemoid reaction, capillary leak and fluid sequestration with associated hemoconcentration, refractory tachycardia and hypotension, and marked edema of infected tissues without gas production or extensive myonecrosis. Cultures many times have been negative, and the diagnosis has been confirmed with molecular techniques. The reported mortality is very high. Besides adequate antibiotics, emergency hysterectomy may be required.

Which of the following agents are considered a serious hazard to laboratory staff? Neisseria gonorrhoeae Haemophilus ducreyi Cryptococcus neoformans Coccidioides immitis Corynebacterium diphtheriae

Correct answer: Coccidioides immitis C. immitis/posadasii may be a risk for microbiology staff. The other organisms listed do not pose risk if they are handled appropriately. Laboratory workers must be careful with other organism-types as well. For instance, C. immitis/posadasii, Neisseria meningitidis, Brucella species, Francisella tularensis, Yersinia pestis, Bacillus anthracis, Burkholdheria pseudomallei and Burkholdheria mallei should be handled in a biological safety cabinet. C. immitis/posadasii, along with N. meningitidis, Brucella species, F. tularensis, Y. pestis, B. anthracis, and Burkholdheria species will grow on routine bacteriology media, so the laboratory should be notified if these are suspected. Laboratory workers who inadvertently work with these agents outside of a biological safety cabinet may require antimicrobial prophylaxis. It is your responsibility to notify the laboratory if you suspect infection with any of these organisms to help protect those that provide a service for you. Organisms that Pose Special hazard to Laboratory Workers Group 3 Risk Group 3 pathogens can be transmitted by the airborne route and characteristically need only a low infectious dose to produce serious or life-threatening disease Bacillus anthracis Brucella species Coxiella burnetii Francisella tularensis Mycobacterium tuberculosis Lymphocytic choriomeningitis virus Hantaan virus St Louis encephalitis virus Japanese encephalitis virus Western equine encephalitis virus West Nile virus SARS coronavirus Prions Coccidioides immitis Blastomyces dermatitidis Histoplasma capsulatum Paracoccidioides brasiliensis 4 Containment Level 4 agents have the potential for aerosol transmission with a low infectious dose, can produce very serious and often fatal disease and have no licensed treatment or vaccine available Lassa virus Marburg virus Ebola virus Herpesvirus simiae Table adapted from Noble, M.A., Manual of Clinical Microbiology, 10th edition (ed. Versalovic et al), Chapter 10, 2011. Correct Response

A 39-year-old woman is seen for painful, erythematous bumps on the anterior surfaces of both legs. The lesions have been present for a week and have been associated with some arthralgias and intermittent fever. She recently returned from a three-week trip with her church group to Northern Mexico when she helped to construct a new church. During her stay she ate local cheeses made from unpasteurized milk and had one episode of unprotected sex with a fellow church member. She also reports having trauma to her toe from a rock while she was wearing open-toed shoes. Her exam is completely normal except for tender, red-purple nodules over both shins. There is no detectable abnormality at the site of her toe trauma and no adenopathy. Which one of the following is the most likely cause of her illness? Listeria monocytogenes Nocardia brasiliensis Brucella abortus Coccidioides posadasii Treponema pallidum

Correct answer: Coccidioides posadadii Coccidioidomycosis, also called Valley Fever, has been shown to be caused by two genetically distinct species (Coccidioides immitis and C. posadasii): these species are similar clinically and in terms of drug susceptibility but differ in geographic distribution. Coccidioidomycosis is common in Northern Mexico, where the usual species is Coccidioides posadasii. "Desert rheumatism," "desert bumps" describe the arthralgia and erythema nodosum that can accompany coccidioidomycosis in otherwise healthy persons. Women are particularly prone to erythema nodosum with coccidioidomycosis. Illness usually resolves spontaneously. The leg lesions are most consistent with a diagnosis of erythema nodosum. The associations with erythema nodosum to consider are: inflammatory bowel disease, sarcoidosis, oral contraceptives and other drugs, TB, leprosy, histoplasmosis, coccidioidomycosis, blastomycosis, yersiniosis, Q fever, psittacosis, and Chlamydia trachomatis. Association with Streptococcus pyogenes infection remains controversial. If there is associated hilar adenopathy, think of sarcoidosis, TB, endemic fungi, and Yersinia enterocolitica or Yersinia pseudotuberculosis (especially in Europe). Nocardia brasiliensis can cause a nodular lymphangitis and may follow trauma to the skin, but bilateral lesions would be unlikely. Listeriosis and brucellosis both may occur following ingestion of unpasteurized milk, but neither is associated with erythema nodosum. Reports of secondary syphilis as a cause of erythema nodosum exist, but they are quite rare and largely in the older literature.

A 22-year-old medical student is seen for fever of one day duration. He just returned from a hiking vacation in Nevada and New Mexico. While there he drank water from local streams and was bitten by numerous mosquitoes, deer flies and ticks. He cut his vacation short because he became ill. Six days before returning home he had the acute onset of fever, chills, headache, photophobia, and myalgias but after 72 hours these resolved. His symptoms have returned. On exam he was febrile with a mild tachycardia; the spleen tip was palpable. There was no rash, and the rest of the exam was normal. His WBC count was 1200 with 55% lymphocytes; platelet count was 90,000. Which one of the following is the most likely diagnosis? Colorado Tick Fever Rocky Mountain Spotted Fever Dengue Fever West Nile Virus Leptospirosis

Correct answer: Colorado Tick Fever a saddleback fever illness with leukopenia and thrombocytopenia, all characteristic of Colorado Tick fever, a tick (Dermacentor andersoni-the Rocky Mountain Wood tick) transmitted viral illness. Fever typically lasts 2-3 days, then, after 1-3 days without fever, it recurs in 50%. Splenomegaly may be seen. This entity can be seen in the Western US and Canada, and has a natural reservoir in rodents. short incubation period (1-14 days) like many viral diseases. Fever, headache and myalgias are typical. Rash and cytopenias occur in a minority of patients. Fatigue can be striking and prolonged. Diagnosis can be made by serology or PCR. The patient could have Rocky Mountain Spotted Fever but the patient does not have a severe illness and RMSF is not biphasic. Dengue is not endemic in the areas this patient traveled. West Nile Virus does not cause a relapsing fever illness. Leptospirosis may be associated with exposure to river water and may cause a biphasic illness, but leukopenia and thrombocytopenia are quite rare.

A 27-year-old patient with HIV infection (CD4 = 29 cells) has been asymptomatic. He has noted a slowly growing lesion in his anal area, which is embarrassing but which does not cause pain. The lesion is shown. This lesion is most likely: Squamous carcinomaCondyloma lataHSVCondyloma accuminataKaposi's sarcoma

Correct answer: Condyloma accuminata This is a giant condyloma accuminata caused by HPV (human papilloma virus). Condyloma lata lesions, caused by syphilis, and squamous carcinoma are much flatter. Kaposi's lesions are flat and dark colored. HSV causes painful acute genital ulcers.

This 44-year-old man with AIDS has an organism in his skin. Which of the following sites is the most likely site for the virus to spread to? ConjunctivaBuccal mucosaBrainBlood streamDraining lymph nodes

Correct answer: Conjunctiva The papule with a shiny center is typical of molluscum contagiosum, an inoculation viral disease seen in immunosuppressed and, less often, in otherwise healthy patients. Molluscum can spread to other areas of the patient's skin or conjunctiva by inoculation. Spread to buccal mucosa would be rare. Lymphatic spread or hematogenous dissemination to brain or blood stream does not occur. Skin lesions of disseminated Cryptococcus can be mistaken for molluscum. Cryptococcus is most likely to involve the meninges or lungs. These options were not offered.

A 24-year-old woman is admitted with suspected bacterial meningitis. She is treated empirically on admission with intravenous ceftriaxone and vancomycin. Cerebrospinal fluid culture obtained in the Emergency Department grows Neisseria meningitidis on hospital day two. The patient's doctor calls you. The patient has received ceftraxone for 48 hours and is improving but has not been on any isolation precautions. The doctor asks which of the following options would be most appropriate at this point. Continue standard precautions. Place the patient on airborne isolation. Place the patient on contact precautions. Place the patient on contact precautions & airborne isolation.

Correct answer: Continue standard precautions. Neisseria meningitidisis transmitted person-to-person by respiratory droplets, which travel about 3 feet before being removed from the air by the force of gravity. N. meningitidisis a common cause of bacterial meningitis, and therefore, patients with suspected bacterial meningitis are placed on droplet precautions -- mask when within 3 feet of the patient. After 24 hours of effective antimicrobial therapy, patients with meningitis due to Neisseria meningitidisare no longer contagious; hence in this case, in which the patient had been treated for >24 hours, isolation precautions were no longer indicated.

A patient with subacute endocarditis had the infected heart valve replaced. Blood cultures collected at the time of surgery and cultures of the vegetation on the heart valve are negative. The patient did not receive antimicrobial agents prior to collections of these specimens for culture. Which one of the following organisms will not grow in conventional blood culture bottles? Abiotrophia defectiva (formerly known as nutritionally deficient streptococci) Aggregatibacter actinomycetemcomitans Eikenella corrodens Cardiobacterium hominis Coxiella burnetii

Correct answer: Coxiella burnetii Rationale: A routine blood culture set typically includes an aerobic and an anaerobic bottle. Most organisms grow in one or both broths. Routine blood culture incubation for five days is sufficient to detect the HACEK group organisms. Abiotrophia and Granulicatella species (formerly, "nutritionally deficient streptococci") will grow in the broth cultures although they will not grow on subculture to routine blood agar; they will grow on subculture to chocolate agar. C. burnetii will not grow in routine blood culture bottles; although it can be grown in specialized cell cultures, this testing is not routinely performed, primarily for safety reasons. The preferred approach to the diagnosis of Q fever is serology; NAAT testing of whole blood and/or heart valve tissue may be considered. Other organisms that will not grow in routine blood culture bottles include Borrelia species, Leptospira species, and Tropheryma whipplei. Of these, only the last is typically considered as a cause of endocarditis. Bartonella species, which may cause endocarditis, are rarely isolated in blood culture bottles; serology is preferred, and NAAT testing of whole blood and/or heart valve tissue may also be considered. Brucella species will grow in blood culture bottles and may, but don't typically, require prolonged incubation. The laboratory should be notified of suspect cases of brucellosis to prevent laboratory-acquired brucellosis. Nocardia and Mycobacterium species may grow in blood cultures, but may require special blood culture bottles and extended incubation. Aggregatibacter actinomycetemcomitans (previously named Actinobacillus actinomycetemcomitans) is a Gram-negative rod which isand part of the HACEK group. This oral commensal is often found in association with aggressive periodontitis,

Three days after attending a wedding reception in Saint Louis, 32 guests became ill with symptoms that included diarrhea, anorexia, abdominal cramping, and a low-grade fever. Cultures for bacteria and viral pathogens were negative. However, coccoid forms, 8 to 10 µm in diameter that stained acid-fast were seen on ova and parasite examination (figure). The cause of this diarrhea is most likely which of the following: CandidaCryptosporidiumCyclosporaIsosporaMicrosporidia

Correct answer: Cyclospora Of the organisms listed, all are acid-fast except Candida. The easiest way to differentiate the acid-fast parasites is by their size: microsporidia are 1-2 µm in diameter; Cryptosporidium is 4-6 µm; Cyclospora is 8-10 µm; and Isospora is 10-19 µm wide and 20-30 µm long. Candida does not cause any gastrointestinal syndromes in immunocompetent individuals.

Approximately 100 people having eaten at a country club buffet in Houston in May developed diarrhea with a median incubation period of one week. Their symptoms included diarrhea (99%), weight loss (91%), abdominal bloating or gas (84%), fever (54%) and vomiting (27%). The median duration of illness was 1 week. Stool samples from 30 persons were sent to the laboratory for routine bacterial culture and O & P. Results of stool studies are pending. What was the likely cause of the outbreak? Calicivirus/norovirus (e.g. Norwalk virus) Cyclospora cayetanensis' Bacillus cereus Giardia lamblia Enterocytozoon bieneusi

Correct answer: Cyclospora cayetanensis Calicivirus/norovirus (e.g. Norwalk virus) is not the best answer: - an incubation period of one week rather than the shorter incubation period (24 to 48 hours) -Moreover, norovirus is associated mainly with vomiting. Cyclospora cayetanensis -This was the beginning of a multi-state epidemic of Cyclosporiasis traced to consumption of Guatemalan raspberries, eventually being shown to affect approximately 1,500 persons from 20 states and the District of Columbia. During routine testing for ova and parasites, stool specimens are not usually examined for Cyclospora. Many laboratories do not have the expertise to detect the organism even when suspected. The long incubation period suggested a parasitic agent which only slowly invades the intestinal enterocytes. With a modified acid fast procedure or other specialized stains, the organism can be seen as an oocyts approximately twice the size of Cryptosporidium. Bacillus cereus causes food poisoning from a preformed toxin and would not require a week to appear. Giardiasis is spread by the fecal oral route and has not caused foodborne outbreaks. Enterocytozoon bieneusi causes diarrhea in immunosuppressed patients and does not cause foodborne outbreaks. Correct Response Cyclospora cayetanensis

A 25-year-old man, infected with human immunodeficiency virus (HIV), presented to his physician with a 10-week history of profuse, nonbloody, watery diarrhea, and 18-pound weight loss. This patient travels all over the world although he is vague about why and where he travels. Stool exam for ova and parasites demonstrated the organism in the figure. The parasite measured 25 µm long and 15 µm in width with tapering ends. The parasites also were observed when the stool specimen was stained with an acid-fast stain. Which of the following organisms is the most likely responsible for this patient's disease? CryptosporidiumCyclosporaEntamoebaGiardiaCystoisospora

Correct answer: Cystoisospora Cystoisospora is the largest acid-fast parasite responsible for human disease. Cryptosporidia are 5-7 µm in diameter and Cyclospora are 8-10 µm in diameter. This organism is acquired from water or food that is contaminated by human stool that incubated for a day or two outside the host before being ingested. Neither Entamoeba or Giardia are acid-fast. Each of the parasites causes diarrheal disease, with Entamoeba histolytica responsible for a hemorrhagic colitis and the other parasites causing a watery diarrhea. Cytoisospora can be acquired worldwide, but most commonly in tropical areas. It is unlikely to be acquired in North America.

A 27‐year‐old male presented with two (2) months of progressive dyspnea. He had noted the appearance of purple indurated lesions over his anterior chest. Chest CT found nodular perihilar lung lesions, prominent interlobular septae, and pleural effusions. (Fig) He was remarkably hypoxemic with room air pO2=59mmHg. Bronchoscopy revealed some purple lesions in the bronchus; bronchoalveolar lavage revealed no pathogens on special stains; cytology was unremarkable. BAL CMV PCR was positive at 2000 copies/mL (log104.40IU/ml). Skin biopsy found endothelial cells with nuclei that stained positive for HHV8 on immunocytochemistry. He was found to be HIV positive with a CD4 of 8 cells/uL and a viral load of 80,000 copies/uL. Pleural fluid was bloody but cytology did not show malignant cells. Blood PCR results: CMV 5000 copies/uL (log10 4.82 IU/ml); EBV 4500 copies/mL (log10 2.40 IU/ml); HHV 8 = 200 copies/mL; HHV6 = 500 copies/mL. Antiretroviral therapy was begun. The most useful additional drug for treatment of his diffuse pulmonary disease would be which of the following: A. Cidofovir B. Ganciclovir C. Foscarnet D. Liposomal doxorubicin E. Cyclophosphamide

Correct answer: D. Liposomal doxorubicin This patient most likely has pulmonary disease due to Kaposi sarcoma: the radiologic appearance is typical, as are the intrabronchial lesions and prominent interlobular septae. Pleural effusions are classically described as bloody, but any type of effusion can be seen. It is important to note that there is no specific cytologic abnormality for KS, and thus the cytology would be expected to be negative. Bronchoalveolar lavage HHV8 PCR is not helpful. Most patients with Kaposi sarcoma in the lung also have extensive skin disease, although rare patients will have visceral disease only at time of presentation. The recommended therapy for HIV related Kaposi sarcoma is antiretroviral therapy plus either local or systemic therapy as indicated by the clinical situation. Liposomal doxorubicin is the most commonly recommended systemic therapy for Kaposi sarcoma. Therapy directed at HHV 8 with ganciclovir, foscarnet, or cidofovir will reduce HHV8 levels but has no clinical benefit and thus is not recommended. HHV8 levels are not helpful for diagnosis: most (but not all) patients will have positive serum PCRs for HHV8, but the test is not sensitive or specific, and is not helpful for following the effectiveness of therapy (for Multicentric Castleman's disease, some experts feel that HHV8 levels can be followed to assess the effectiveness of therapy). HHV‐6 almost never causes pulmonary disease; EBV is associated with lymphoproliferative syndromes in transplant recipients but would not be associated with pulmonary disease in HIV infected patients except related to EBV associated lymphoma. Does this patient have CMV lung disease? CMV disease of the lung is surprisingly uncommon in patients with HIV, as opposed to stem cell or solid organ transplant recipients. Many patients with HIV and low CD4 counts will have positive serum CMV PCR, but this test has low specificity for predicting the occurrence of disease. Thus, the positive CMV and EBV tests in the stem are distracters and do not help define the cause of the patient's pulmonary disease: a biopsy would be required showing many CMV inclusions, and even that is a poor test due to sampling error and the occurrence of low level, clinically insignificant CMV infected cells in the lung. Note that laboratories are changing from copies/ml for CMV and EBV to log10 international units/ml because copy number varied between laboratories

A 20‐year‐old Caucasian woman with HIV infection has been given a variety of antiretroviral regimens in the past and was started on abacavir three weeks ago plus lamivudine, darunavir and ritonavir (all new to her except lamivudine). She is also taking trimethoprim‐sulfamethoxazole prophylaxis. A week ago she developed nausea, vomiting, diarrhea, and fever and stopped all her medications. Today, feeling better, she restarted all her medications but an hour later felt faint and sought medical attention. The figures show her cutaneous lesions that developed within the first 24 hours of her feeling faint, and shows them resolving one week later The most likely drug to cause this syndrome with these skin lesions evolving over a week would be: A. Darunavir B. Ritonavir C. Trimethoprim‐sulfamethoxazole D. Abacavir E. Lamivudine

Correct answer: D. Abacavir This syndrome is typical for abacavir hypersensitivity syndrome and is not typical for any of the other drugs listed. The syndrome occurs abruptly, usually in the first 6 weeks after starting the drug, and gets worse with each dose. It resolves within 2 to 3 days of stopping the drug. If the drug is stopped due to such symptoms, and then the drug is restarted with the mistaken presumption that the syndrome was viral rather than abacavir related, the resulting hypersensitivity syndrome can be especially severe as in this patient and hypotension and even death have been reported. Almost all (94%) of patients with abacavir hypersensitivity syndrome have haplotype HLA‐B*5701. Currently guidelines recommend that patients be screened for this allele (8% Caucasians, 2% blacks, rare in Asians) and that the drug NOT be given to positive individuals. This patient should have been screened prior to staring abacavir. Questions rarely mention race unless race identification is a clue to the correct answer, as it was here.

A 31‐year‐old, previously healthy man was admitted to a Baltimore hospital with bilateral community‐ acquired pneumonia. He has no prior history of lung infections and has not responded to a week of ceftriaxone and azithromycin. Repeated WBC counts have all been slightly below normal. Sputum Gram stain and cultures for bacteria, fungi, and mycobacteria on BAL were unremarkable. Legionella culture on the BAL was not done but urine antigen was negative. His exposure history during the past two weeks included a guided tour of a horse farm and staying two nights with a friend who had two parrots. Both his friend and the parrots are well. He complained of a persistent headache. A soft spleen tip was palpable, and several non‐tender red macular lesions a few millimeters diameter were seen on the abdomen. Which one of the following should you recommend? A. Add ciprofloxacin B. Add vancomycin C. Add linezolid D. Add doxycycline E. Send urine antigen for histoplasmosis

Correct answer: D. Add doxycycline. sick or well psittacine birds, parrots, has two months before pneumonia, psittacosis should be considered and doxycycline included in the regimen. turkeys, pheasants, chickens, and geese birds do not necessarily look sick when The incubation period for humans is relatively short, usually 5‐14 days. Ciprofloxacin or azithromycin have activity in vitro,but clinical experience is too limited to be trusted. Severe headache, slightly low WBC, and Horder's spots (look like rose spots on the face) on admission and splenomegaly appearing after a week or so of illness should increase the probability of psittacosis. Visiting a horse farm by a healthy person is a weak reason to treat empirically for Rhodococcus equi. Legionellosis is still possible despite the negative antigen test, though splenomegaly and skin lesions are unusual. There are not enough findings to suggest acute pulmonary or disseminated histoplasmosis and the skin lesions are not consistent with histoplasmosis.

A 47‐year‐old male with lymphoma has been hospitalized for an allogeneic myeloablative stem cell transplant with a conditioning regimen of cyclophosphamide and total body irradiation. He is now 65 days post‐transplant, receiving cyclosporine, mycophenolate, trimethoprim‐sulfamethoxazole, fluconazole, and acyclovir, and has received several courses of pulse steroids for graft‐versus‐host disease. His white blood count is 3500 cells/µL. Sixty days post‐transplant he developed gross hematuria with extreme pain on urination and he is passing clots of blood in his urine. His urinalysis shows rare leukocytes, the nitrate test is negative and his urine culture is negative for routine bacterial and fungal pathogens. His urine cytology for BK virus "decoy cells" is negative, as is BK PCR of his blood and urine. Which one of the following would be the most likely cause of hemorrhagic cystitis in this patient? A. Cyclosporine B. Cyclophosphamide C. JC Virus D. Adenovirus E. Cytomegalovirus

Correct answer: D. Adenovirus Adenovirus and BK virus cause of hemorrhagic cystitis in this patient , as is cyclophosphamide (cytoxan). In this case, BK was ruled out by the PCR tests in the stem. Hematuria due to cytoxan (cyclophosphamide) usually starts within 48 hrs of the dose, making that unlikely in this case. Radiation also causes hemorrhagic cystitis. Adenovirus is an important cause of hemorrhagic cystitis which can be latent in an HSCT recipient from a remote infection and reactivate during periods of intense immunosuppression. Active adenovirus infection may be asymptomatic and detected only by PCR of peripheral blood, or may present clinically as hepatitis, or diffuse pneumonia or cystitis. Diffuse cystitis may be so severe that the patient has urethral obstruction from blood clots and such intense pain on voiding that narcotics may be required. There is no specific therapy for adenovirus: some clinicians might use cidofovir which has activity in vitro, but toxicities are problematic. It is important to minimize immunosuppression, as feasible, but this is complicated by underlying control of GVHD. CMV, JC and cyclosporine do not cause hematuria. Cyclosporine causes renal dysfunction with proteinuria. This patient was BK negative. However, BK, a polyoma virus may reactivate under the same circumstances as adenovirus and appear in the urine in high copy number. Although BK may cause renal tubular damage in transplanted kidneys, the role of this virus as a cause of hematuria remains a subject of diagnostic debate: some cystitis and hematuria is clearly due to BK virus, but how to recognize such cases without biopsy or cytology is difficult because BK viruria in high quantities may be present without cystitis

A 31‐year‐old female with headaches, low grade fevers for 3 weeks, had diffuse sore throat, malaise and nonpruritic maculopapular rash 3 months ago which resolved. Neuro exam is nonfocal, CT of brain normal; serum RPR is reactive 1:32, treponemal EIA IgGs reactive. HIV serology is negative. She is three months pregnant and has a history of an itchy rash when she was given amoxicillin two years ago. CSF: WBC 42/mm3 (100% mono's), protein 105mg/dl, glucose 55 mg/dl and VDRL nonreactive What would you recommend? A. Desensitize and give benzathine penicillin G 2.4 million units IM weekly x 3 doses B. Doxycycline 100 mg bid x 14 days C. Ceftriaxone 2 gm IV qd x 21 days D. Desensitize and give aqueous penicillin G 3 million units IV q4h x 14 days E. Azithromycin 500 mg daily for 14 days

Correct answer: D. Aqueous penicillin G 3 million units IV q4h x 14 days most likely had secondary syphilis three months ago and now has meningitis as a late complication. A negative CSF VDRL does not exclude the diagnosis. Parenteral penicillin regimen indicated here reaches treponemocidal levels in the CSF and is the drug of choice. Pregnancy puts the fetus at risk for congenital syphilis and requires desensitization and penicillin . Her prior reaction was not anaphylaxis, making desensitization safe enough to warrant the risk. Penicillin levels in the CSF are too low with benzathine penicillin to treat CNS syphilis. Doxycycline is an option for treating syphilis at some stages in the patient with penicillin allergies. It also treats infections caused by other spirochetes which might have been considered here, but the patient has serologic evidence of syphilis infection and it is unproven therapy for neurosyphilis. Ceftriaxone does have efficacy against T. pallidum, but we have far less experience with this agent than with aqueous penicillin, and a 21‐day course would be more compatible with CNS Lyme than with syphilis. Azithromycin is not recommended for syphilis. Aseptic meningitis as a presentation of syphilis can occur within a few months to one year following syphilis exposure and overlap with either primary or secondary lesions; it can even occur more than one year, though this is uncommon. A nonreactive VDRL does not rule out neurosyphilis (30‐50% sensitive only).

A 27‐year‐old female presents with a painful, swollen knee and fever of three days' duration. A knee aspirate reveals 100,000 wbc/hpf and no organisms. Culture is pending. She has had several recent sexual contacts. A cervical swab for NAAT is positive for gonorrhea She has a similar episode one year prior that was documented to be gonococcal arthritis. Which of the following tests is most likely to reveal the reason for her gonorrhea to disseminate on two occasions? A. Serum IgA B. Serum IgG C. HIV D. CH50 E. Nitroblue tetrazolium (NBT) test on neutrophils

Correct answer: D. CH50 Disseminated gonococcal (DGI) is occasionally associated with deficiency of terminal complement components that can be detected through screening the total hemolytic complement level. DGI has not been associated with other host defense deficiencies. Thus, recurrent DGI should suggest the possibility of a complement disorder. Empiric management is appropriate since the causative organism is isolated in a minority of patients in terms of blood or joint fluid cultures. However, urethral or rectal or oral cultures may be useful: half of patients have a positive urethral or cervical or rectal culture or NAAT (nucleic acid amplification test) at the time of dissemination.

60 year old man was well until day of admission when he developed lethargy and confusion. Over the course of the day, his hands and feet grew cold and numb and he developed a rash. The patient lived in a rural area (mountain‐lion territory) and drank well‐water. He had a history of alcohol abuse. The patient reportedly rode horses and had dogs, one of whom bit him a few days before. On exam, his temperature was 102 F. Nonblanching, nonpalpable, purpuric patches on head, trunk, thighs; puncture wounds on dorsal aspect of hand; edema, cyanosis of nose (below). Which of the following is most likely? A. E. coli 0157:H7 B. Yersinia pestis C. Pasteurella canis D. Capnocytophaga canimorsus E. Leptospira interrogans

Correct answer: D. Capnocytophaga canimorsus Discussion: The patient's blood cultures were positive for C. canimorsus. This organism is a facultative, fastidious gram‐negative bacillus found in the mouth of dogs and cats. Risk factors for infection include male sex, dog‐bite, alcohol abuse, asplenia, and immunosuppression. In patients with septicemia, 20‐40% develop a rash (maculopapular, progressing to purpura fulminans). Differential diagnosis E coli 157:H7: typically presents with abdominal cramping, diarrhea; fever is usually absent Yersinia pestis: usually presents as bubonic plague, with regional lymphadenitis Pasteurella canis: may follow a cat or dog bite; usually presents with cellulitis; septicemia uncommon, though more common with Pasteurella multocidacfrom a cat or dog bite. Leptospira interrogans: contact with urine or tissue of infected animals; in acute phase, patients may have conjunctival suffusion; purpura fulminans, as in this case, would be unusual.

A 25‐year‐old graduate student penile ulcers been present for two weeks and are painful ulcers began as "red bumps" that developed into "pimples" and then eroded into ulcers past 2‐3 days he noted a tender lump in his groin. began during a trip to Africa from which he returned just three days ago. Africa he had vaginal intercourse with several commercial sex workers while he was inebriated and did not always use a condom. There are two 1cm adjacent "kissing" ulcers in the coronal sulcus. They are tender and filled with a yellow purulent exudate. There is a tender, large lymph node in the left groin. Which one of the following is the most likely cause of his problem? A. Treponema pallidum B. Herpes simplex C. Chlamydia trachomatis D. Haemophilus ducreyi E. Klebsiella (Calymmatobacterium) granulomati

Correct answer: D. Haemophilus ducreyi painful ulcers due to HSV or chancroid (haemophilus ducreyi) painless ulcers associated will be syphilis, LGV (chlamydia trochomatis) , or granuloma inguinale (klebsiella / calymmatobacterium granulomatis) ‐lab confirmation should be performed. PCR has been reported to be more sensitive than culture. The classic presentation of chancroid due to Haemophilus ducreyiis a deep painful ulcer with purulent exudate; painful inguinal lymphadenitis is common. rare in the United States but is endemic in sub‐Saharan Africa. H. ducreyi has been increasingly recognized as a cause of nongenital skin ulcers in tropical areas, including in children. Primary syphilis due to Treponema pallidum produces a painless ulcer (chancre). Herpes simplex produces a cluster of painful vesicles, typically on the penile shaft, which quickly ulcerate. Tender inguinal lymphadenopathy Is atypical. Lymphogranuloma venereum due to Chlamydia trachomatis presents as inguinal adenopathy. An initial genital ulcer may be noted but typically heals within a few days followed by adenopathy which is usually unilateral. Granuloma inguinale or donovanosis due to Klebsiella granulomatis presents as a painless, slowly progressing ulcer without regional adenopathy.

This 29‐year‐old Hassidic Jew from New York presented with seizures. He had no prior history of fever, overseas travel or pets. He claims to eat only Kosher food, i.e., no pork or shellfish, but you are not certain how strict he is about eating when he is away from home. He has probably eaten which of the following: A. Poorly cooked pork B. Raw hamburger C. Gefilte fish D. Human stool E. Unpasteurized soft cheese

Correct answer: D. Human stool The solid, possibly calcified lesion is consistent with neurocysticerosis as it appears cystic with a possible protoscolex inside of it. Cystericercosis lesions may be in muscle or brain, and may be multiple or shrunken to form a calcific nodule. Cysticerosis is acquired from eating the eggs of the pig tapeworm, Taenia solium, which are in the stool of a patient with a tapeworm. Ingesting poorly cooked pork causes tapeworm infection. When a human with tapeworm infection contaminates food with their stool, an unsuspecting person ingesting can acquire cysticercosis. In this question, the food preparer may have had tapeworm infection and contaminated the food by not washing hands after defecating. In regard to the other sources of infection listed above: Diphyllobothrium latum is a tapeworm acquired by eating inadequately cooked fish, such as gefilte fish. Raw hamburger can lead to beef tapeworm. Unpasteurized cheese is linked to brucellosis, listeriosis and mycobacterium bovis infection, none of which cause brain lesions like this.

A 19‐year‐old female was brought to the emergency room by police after being raped by several men at a party earlier that evening. She complained of vaginal and rectal pain. She did not know the men who raped her, but knew that some of them had been "shooting up" in a room at the party. She had never been tested for HIV and is extremely concerned about having been infected by the rape. She asks if she can be given drugs to prevent acquisition of HIV, as well as the morning after pill. In addition to testing her for HIV and other sexually transmitted diseases, your approach to this question is which of the following? A. Ask the patient to return in two days when her HIV serology is available B. Offer a one‐week regimen of tenofovir plus emtricitabine C. Offer a single dose of Atripla (efavirenz, emtricitabine, tenofovir) to be taken now D. Offer a four‐week antiretroviral regimen of TDF/FTC + RAL (or DTG) E. Offer a 6‐month antiretroviral regimen of TDF/FTC + RAL (or DTG)

Correct answer: D. Offer a four‐week antiretroviral regimen of TDF/FTC + RAL (or DTG). Patients who have a non‐occupational exposure to HIV, either known or highly likely, in the past 72 hours are candidates for a four‐week regimen of using one of the regimens recommended for therapy of HIV‐ infected individuals. Clinics that can do the rapid test for HIV on saliva find it useful to check for preexisting HIV if the patient has had high‐risk behavior. Although the exact window of time in which antiretroviral prophylaxis is effective remains unknown, evidence points to starting prophylaxis as soon as possible, and ideally within 72 hours based on animal data. Waiting until the HIV serology returns to begin antiretroviral is not recommended. The correct answer is a 3‐drug regimen of TDF/FTC + RAL (or DTG) X 4 weeks. Regimens that use darunavir‐ritonavir, and more often raltegravir, or dolutegravir are among those recommended based on lack of drug resistance in the community, in combination with tenofovir (TDF)‐ emtricitabine. Single‐ or dual‐drug regimens are undesirable because of the possibility that the acquired virus is already resistant, or that the acquired virus will become resistant on therapy. HIV infection can be acquired without mucosal breaks, though trauma increases the risk. Keep in mind management for other STDS including HBV. The Centers for Disease Control and Prevention (CDC) recommend empiric antibiotic prophylaxis. The risks are substantial for chlamydial infection (3% to 16%), pelvic inflammatory disease and bacterial vaginosis (11%), trichomoniasis (7%). Empiric therapy with ceftriaxone and azithromycin or equivalent drugs will cover these pathogens. Post‐exposure HBV vaccine is also recommended.

A 68-year-old woman was admitted one week ago for an upper GI bleed. A gastric carcinoma was found on endoscopy. While awaiting gastric resection she developed methicillin-resistant S. aureus bacteremia of a right internal jugular central venous catheter which was complicated by thrombophlebitis. The central venous catheter was removed and a PICC line placed in her left arm. She has been treated with vancomycin for six days. She has persistent fevers. A blood culture drawn on day 4 of vancomycin therapy is positive for MRSA despite documented trough serum concentrations of vancomycin of 17 μg/ml. The vancomycin MIC is 1 μg/mL for the pre-treatment isolate. The referring physician wishes to change from vancomycin and asks you to recommend another regimen. Which one of following options is most appropriate for this patient? Add gentamicin to the vancomycin. Ciprofloxacin as a single agent. Daptomycin as a single agent. Linezolid as a single agent. Add rifampin to the vancomycin.

Correct answer: Daptomycin as a single agent. This patient's isolates is susceptible to vancomycin (<2 mcg/ml) but Staph aureus septic phlebitis, which this patient probably has, can cause prolonged bacteremia and fever. Many clinicians would not change the current vancomycin regimen, despite some concerns with "heteroresistance" in isolates with a vancomycin MIC of 1 mcg/ml. Responding to the query for an alternative, ceftaroline, not listed above, is a possible answer. Although approved for methicillin susceptible Staph aureus pneumonia and MRSA complicated skin and soft tissue infections (SSTI), little data are available on treatment of MRSA sepsis. Oritavancin and dalbavancin are only approved for MRSA SSTI and would not be appropriate here. Daptomycin is the only antibiotic other than vancomycin that is FDA approved for complicated staphylococcal bacteremia. Vancomycin and gentamicin are synergistically nephrotoxic and the advantage of the combination over vancomycin alone is unproven especially for suspected vancomycin treatment failure. Linezolid is not indicated for catheter-related infections and is bacteriostatic, which may be disadvantageous in endovascular infections. Telavancin(not listed) is approved for complicated skin and soft tissue infections and for hospital-acquired or ventilator-associated bacterial pneumonia when alternative treatments are not suitable. Experience with telavancin for complicated bacteremia is too limited to recommend use for front line therapy. The advantage of the combination of vancomycin and rifampin over vancomycin alone is unproven especially for suspected vancomycin treatment failure and IDSA guidelines for MRSA do not recommend addition of rifampin for bacteremia. Quinolones are not appropriate for serious staph aureus infections due to the rapid emergence of resistance. Many MRSA isolates are also ciprofloxacin resistant.

A 23-year-old female, previously in good health, has received 5 days of azithromycin for a clinical diagnosis of community-acquired pneumonia manifest as low-grade fever, non-productive cough, and fatigue. She is clinically getting worse. You elicit a history of a fever and cough several months earlier which resolved. She lives in Maryland, and has not traveled. She has been mulching in her garden. Her physical examination was normal except for a temperature of 38.5°C and diffuse crackles. Her WBC is 17,000 with 90% neutrophils. Her chest x-ray shows small patchy nodules in 5-lobes. Due to progressive deterioration, an open lung biopsy was performed. The results demonstrate the presence of necrotizing granulomas; stains show a mold consistent with Aspergillus. Her mother reports that the patient and her brother had both been seen for several episodes of cervical lymphadenitis when they were younger; these episodes were attributed to cat scratch disease. The patient has used IV illicit drugs in the past. Which one of the following tests is most likely to establish an etiology of the patient's abnormal host defense mechanism? Quantitative immunoglobulins Bone marrow biopsy Dihydrorhodamine 123 oxidation test Complement levels CD4 cell count

Correct answer: Dihyrorhodamine 123 oxidation test The diagnosis of chronic granulomatous disease (CGD) should be suspected in a patient with invasive aspergillosis and no obvious predisposing factor. Testing granulocytes for ability to oxidize nitroblue tetrazolium on light microscopy or, now more commonly, oxidation of dihydrorhodamine by fluorescence-activated cell sorting, should be diagnostic. The prior history of lymphadenitis, which likely was erroneously diagnosed as cat scratch, suggests a familial disorder for both the brother and the patient. CGD patients typically develop pyogenic infections due to Staphylococcus, Nocardia, molds (especially Aspergillus) as well as Burkholderia and Serratia. Outside the United States, BCG and Salmonella are common associations with CGD. Although the X-linked form of this inherited disease is symptomatic only in males, females are also affected by other mutations which lead to defective oxidative capacity of neutrophils. Although most cases are more severe and present and are diagnosed in childhood, milder forms manifest in young adults. Complement disorders and immunoglobulin deficiencies are not associated with aspergillosis. A bone marrow biopsy is not likely to reveal a cause for this: it would be unusual for lymphoma or leukemia to present as aspergillosis and also unusual for them to present with a relatively normal CBC. Drug use could suggest HIV but aspergillosis is rarely the presenting manifestation of HIV/AIDS. Correct Response Dihydrorhodamine 123 oxidation test

This 30-year-old previously healthy exchange student from Shanghai, China was in college in Phoenix, Arizona when he had the sudden onset of left chest pain and had this CT finding. He had been afebrile, feeling well, and attending class. The agent is most likely which of the following? Normal oral floraDimorphic fungusAerobic soil bacillusAmoebaAcid fast bacillus

Correct answer: Dimorphic fungus The cavity which caused pneumothorax was caused by Coccidioides posadasii or Coccidioides immitis, two species that can be separated by genomic sequencing but seem to differ in geography, with C. Immitis in California and C.posadasii being more broadly distributed. Both are dimorphic fungi, meaning they have a different form in the tissue (spherules) than in culture (mold). Healing of coccidioidomycosis may leave a single, thin walled cavity. The cavity is often asymptomatic but found on routine chest x-ray or causes hemoptysis or pneumothorax. Half of the lesions clear spontaneously during the first year but others either close more slowly or not at all, being found years after the patients leave the endemic area of the Southwestern United States. A cavity from normal flora would include actinomycosis, which causes chronic pneumonia in addition to any cavity that might be present. An aerobic soil bacillus would include bacillus species, which do not cause lung cavities. Amoebic abscess can extend from the liver across the diaphragm but amoebiasis causes an abscess often with empyema and not a solitary cavity. Mycobacteria would be rare causes of an isolated thin walled cavity.

Which of the following is the most common route of transmission for infections caused by this organism? Direct inoculationDirect skin penetrationFecal-oralInhalationMosquito bite

Correct answer: Direct skin penetrationThe filariform larva of Strongyloides stercoralis is infectious and can infect humans by directly penetrating through the skin. Autoinfections also can occur in patients with severe disease when the noninfectious rhabditiform larva develops into the infectious filariform larva, penetrates through the intestine or perianal skin, and initiates a new cycle of infection. Sexual transmission also can occur when larvae are transferred to the skin of the sexual partner.

This 21-year-old African American male college student in Tucson, Arizona was seen because of low grade fever, malaise and scalp lesions progressing over the past 3 weeks. He had visited Nogales, Mexico with some of his fraternity brothers six months earlier and had sex with a prostitute. About a month ago, he was drunk at a party, fell into a pond and required resuscitation. A skin biopsy is shown below. The most likely etiologic agent is found in which of the following locations: Pond scumDirtGenital lesionsBat droppingsPigeon droppings

Correct answer: Dirt The spherule indicates the diagnosis of coccidioidomycosis. African Americans are a much higher risk of dissemination than Caucasians. Sites of dissemination prominently include bone, joint, soft tissue, meninges and skin, though other organs may be affected. The source of infection is inhalation from the soil. Coccidioides spores are extremely resistant to desiccation and heat of summer in the Southwest United States. Arizona has the highest attack rate of coccidioidomycosis, with California coming in second. Tucson is in a highly endemic area.

A 47-year-old male from Maryland with myelodysplastic syndrome and prolonged neutropenia underwent ablative chemotherapy in Bethesda and an allogeneic bone marrow transplant. Post bone marrow transplant he was placed on acyclovir, and fluconazole. Two years ago, he spent 3 months on an island off the coast of Venezuela. On day 5 following transplant, with an absolute neutrophil count of zero, he became febrile to 40°C with hypotension. Piperacillin-tazobactam and vancomycin were begun. The next day, new necrotic skin lesions were noted. The lesions were 2 to 3 cm in diameter and deep in the subcutaneous tissue and reddish purple in color. Chest CT was normal. Voriconazole was added. On day 6 the laboratory reported two routine blood cultures positive for septated hyphae. The most likely diagnosis is: Disseminated aspergillosis Disseminated mucormycosis Disseminated histoplasmosis Disseminated penicillium marneffei Disseminated fusariosis

Correct answer: Disseminated fusariosis report of hyphae growing from blood should strongly suggest fusariosis. Fusarium can cause skin or corneal disease in normal hosts, albeit rarely. With prolonged neutropenia, or following bone marrow transplantation, dissemination can occur, probably from skin or lungs in most cases. Necrotic skin lesions occur in 60-80% of cases. For aspergillosis, or mucormycosis, blood cultures are almost always negative; in contrast, positive blood cultures are seen in 40-50% of cases of fusariosis. Thus, this case is far more likely to represent fusariosis than aspergillosis. If this patient were in Southeast Asia, Penicillium marneffei might be considered. The skin lesion is atypical for penicilliosis or histoplasmosis and neither would grow in routine blood cultures in six days.

A 42-year-old Greek man was visiting family in the USA when he was seen in clinic for cough. He had been employed for several decades herding sheep in the mountains of Northern Greece He had been living and cooking his meals outdoors, drinking spring water and eating an occasional fish he caught from streams. The left photo is his chest x-ray. The right photo shows specimens from a fine needle aspirate of a lung lesion. The infection was likely acquired from: Choices aSpring water bDog stool c Undercooked pork d Undercooked fish

Correct answer: Dog stool - cystic disease of the lung!!! hytadid The first photo shows a hydatid lung cyst in chest xray. The second photo shows the hooklets that appear in the sputum when a cyst ruptures into a bronchus. The hooklets may also be seen in fine needle aspirates of a hydatid cyst. This patient acquired echinococcosis from incidental ingestion of stool from dogs who have eaten uncooked meat from sheep infected with Echinococcus granulosus. Cysts are often single, in the lung or liver, and asymptomatic as they enlarge over several years. Echinococcus species are widely distributed worldwide as a tapeworm (taenia) of carnivores. Humans are infected by eating food contaminated with stool from a carnivore excreting Echinococcus eggs. Echinococcus multilocularis (alveolar echinococcosis) cause lesions which on CT imaging more resembles a tumor than a cluster of cysts. Alveolar echinococcus is found worldwide, including in Alaskan natives with dogs fed bear or other animal meat.

The patient with this peripheral blood smear was bitten by which of the following: Deer tickDog tickDeer flyKissing bugFlea

Correct answer: Dog tick Monocytic ehrlichioisis is transmitted by dog ticks. Note that the morula is in a monocyte, not a neutrophil. If the cell were a neutrophil, the answer would be a deer tick. Deer flies can transmit tularemia. Fleas can transmit Rickettia felis and Rickettsia typhi. Kissing (reduviid) bugs can transmit Chagas' disease, but none of these last three infections produce intracellular inclusions.

A woman has erythema migrans. Three years ago she had erythema migrans that cleared entirelywith two weeks of doxycycline. Which one of the following is the appropriate treatment for this second episode? Ceftriaxone for six weeks Azithromycin for two weeks Azithromycin for four weeks Doxycycline for two weeks Doxycycline for eight weeks

Correct answer: Doxycycline for two (2) weeks Reinfection with Borrleia burgdorferi can occur following the successful treatment of early Lyme disease. Reinfection arising from an infected, new tick bite still presents as erythema migrans (EM). Patients may suffer third or even fourth bouts of EM from newly acquired bites. Interestingly, erythema migrans as reinfection probably does not occur following late infection such as Lyme arthritis, probably because a more robust immune response developed. Recent data show convincinglythat recurrent episodes of erythema migrans occurring more than a year apart are due to reinfection and not due to relapse of a putative latent infection. Therefore, second episodes should be treatedwith the same regimen recommended for first episodes that 14 days of doxycycline. Macrolides are less effectivethan doxycycline. Ceftriaxone is not superior to doxycycline for early Lyme.

A college student with a severe cough is hospitalized for what was thought to be a "community-acquired pneumonia." She has been treated with ceftriaxone and azithromycin since admission. You are asked to see her on the third hospital day, and you suspect, and then confirm, a diagnosis of pertussis. What type of isolation would you recommend for this patient? Contact isolation Airborne isolation Droplet isolation No isolation

Correct answer: Droplet isolation The 2007 CDC guidelines recommend droplet precautions for hospitalized patients with pertussis at least until they have had effective therapy for five days. Remember, classically airborne precautions (N95 masks and single rooms that are negative pressure or HEPA-filtered) are only indicated for TB, varicella, and measles.

An 81-year-old man, afebrile and of normal mental status, underwent surgical debridement of chronic osteomyelitis of his prosthetic hip which was implanted one year previously. MRSA was recovered from a deep bone biopsy, and he was started on linezolid and rifampin. Hip prosthesis remains in place. He had been receiving paroxetine (Paxil) for depression during the 3 weeks prior to admission to the hospital, as well as chronic daily digoxin, amiodarone, and furosemide. After 1 week of linezolid therapy, the patient returned to the hospital because of changes in mental status. He became increasingly tremulous. In the ensuing 48 hours he developed fever, high blood pressure, tachycardia, confusion, hyperreflexia and tremors without localizable neurologic signs. Lab studies showed a platelet count of 190,000/mm3, a rising serum lactate, a falling serum bicarbonate to 9 meq/l, and alanine aminotransferase 66 U/L; total creatine phosphokinase, 766 U. The syndrome occurring in this patient is most likely the result of which one of the following: "Flu " syndrome from rifampin Mitochondrial toxicity due to linezolid Amiodarone toxicity Drug interaction between amiodarone and linezolid Drug interaction between paroxetine and linezolid

Correct answer: Drug interaction between paroxetine and linezolid direct toxicity of linezolid. Thrombocytopenia and transaminase elevations occur due to Linezolid Thrombocytopenia probably occurs after 7 to 10 days of therapy, drug-induced inhibition of the synthesis of mitochondrial proteins. a serotonin syndrome (fever, agitation with mental status changes, and tremors), which occurs when linezolid is used concurrently with selective serotonin reuptake inhibitors (SSRIs), such as paroxetine (Paxil), fluoxetine (Prozac), sertraline (Zoloft), citalopram, or venlafaxine. Linezolid is a weak monoamine oxidase inhibitor; normally monoamine oxidase breaks down synaptic serotonin. Hence, the potential additive effect of the combination of a SSRI and linezolid. Lactic acidosis and hepatic dysfunction can be part of this syndrome. The features that distinguish serotonin toxicity from other drug-induced toxic syndromes (e.g., neuroleptic malignant syndrome) are the combination of hyperreflexia, hypertonia, myoclonus, fever, and agitation. Clinically, the onset of serotonin toxicity is often rapid; at first, the patient reports tremor and hyperreflexia. Clonus and myoclonus start initially in the lower limbs and may become generalized. Autonomic features (tachypnea, tachycardia, and hypertension) fluctuate and are not usually difficult to manage. Other symptoms may include shaking, shivering, chattering of the teeth, and trismus. Pyramidal rigidity is a late development and may impair respiration if it affects truncal muscles. Rigidity and fever (temperature, >39.5°C) indicate serious toxicity. Rifampin flu like syndrome occurs with periodic patient noncompliance with chronic rifampin therapy as part of the multidrug therapy of influenza

Extension of infection from this patient's sphenoid sinusitis into an adjacent structure is most likely to result in which physical finding: https://radiopaedia.org/images/21870 Monocular blindnessFacial numbnessDysconjugate gazeBell's palsyUnilateral hearing loss

Correct answer: Dysconjugate gaze This patient could have extension from the sphenoid into the cavernous sinus, as can be seen from the carotid artery in the upper portion of the sinus. Cranial nerves 3, 4, and 6 traverse the cavernous sinus and lead to early oculomotor palsy. The second, seventh, and eighth cranial nerves do not traverse the cavernous sinus so monocular blindness, Bell's palsy, and hearing loss are not seen. Look at this figures to get a sense of the anatomic relationship of the sphenoid sinus and the cavernous sinus. See arrows:

A 22‐year‐old male from Trinidad has had aplastic anemia since 2003. He is being prepared for a stem cell transplant from his brother after receiving a long course of horse anti‐thymocyte globulin (ATG) and prednisone. He is chronically neutropenic with a current absolute neutrophil count of 75/cu mm and platelet count of 15,000/cu mm. Renal and hepatic function are normal. He is admitted to the ICU from clinic with fever, hypotension, and abdominal tenderness and distension with some rebound tenderness and only a few bowel sounds He was started on vancomycin, and imipenem plus fluconazole. Surgical consultation was obtained and a CT scan with oral and intravenous contrast was ordered: showed submucosal edema in cecum What would you recommend be added to his regimen of vancomycin/imipenem/fluconazole at this time? A. Ivermectin B. Surgical resection C. Linezolid D. Liposomal Amphotericin B E. Nothing. Continue present management

Correct answer: E. Nothing. Continue present management The x‐ray showing intense submucosal edema with only a narrow channel containing oral contrast material is typical of typhlitis, or neutropenic enteritis. This typically involves the cecum, but can also involve the terminal ileum or ascending colon. This entity occurs during periods of neutropenia, and often follows intensive chemotherapy. While perforation can occur, many patients can be successfully managed medically. The presence of typhlitis by itself is not an indication for immediate surgery. There is nothing in this presentation that mandates the escalation of antifungal therapy, or therapy for a wider spectrum of gram‐positive cocci. Strongyloidiasis can produce abdominal pain by invading the bowel wall or occluding the appendix. Ivermectin might be useful in treating this patient from Trinidad if he had hyperinfection strongyloidiasis but cecal edema is not particularly suggestive of that diagnosis. The presenting clue would be bacteremia, or more typically polymicrobial bacteremia and/or meningitis, in someone from an endemic area. Pulmonary infiltrates or Larvae in the BAL are sometimes seen because the nematode migrates through the lungs. Stronglyoidiasis can be prevented by screening for a positive serology or prophylaxis with ivermectin in high‐risk patients.

Bacteria have developed high level resistance to carbapenems ( imipenem, meropenem, doripenem, or ertapenem) by which one of the following mechanisms? A. Enzymatic alteration of the carbapenem B. Active efflux of antibiotic out of the bacterial cell C. Modification of peptidoglycan cell wall target of carbapenems D. Modification of ribosomal binding site target of carbapenems E. Production of a zinc‐ or serine‐requiring beta‐lactamase or carbapenemase

Correct answer: E. Production of a zinc or serine‐requiring beta‐lactamase or carbapenemase Carbapenem antibiotics (e.g., imipenem, meropenem, ertapenem) are potent antibiotics active against virtually all groups of organisms, with only a few exceptions: e.g., MRSA, VRE, Stenotrophomonas sp. High Level resistance to carbapenems , especially by gram‐negative bacilli, is most commonly due to production of a carbapenemase that can hydrolyze the beta lactam ring. Some of the carbapenemases require the presence of zinc at the active site and are termed metallo carbapenemases. Other carbapenemases require serine at the active site. Currently in the United States, serine requiring carbapenemases are more common than the metallocarbapenemases. The rapid CarbaNP test can detect presence of carbapenemases within 2 hrs. Strains that produce carbapenemases may also possess antibiotic efflux pumps. If the only mechanism of resistance, efflux pumps lead to low level, and not high level, resistance. Carbapenemases producing aerobic gram‐negative bacilli are generally also resistant to all the penicillins, carbapenems, cephalosporins, quinolones and aminoglycosides. Enzymatic alteration of an antibiotic, e.g., aminoglycosides, can result in high level resistance. So far, there are no carbapenem modifying enzymes identified Modification of ribosome binding sites can result, depending on the specific site, to high level resistance to macrolides, lincosamides, tetracyclines, and aminoglycosides. Alteration of carbapenem cell wall binding sites, if it occurs, is extremely rare; a change in target would lead to high level resistance Carbapenemase producing isolates are suspected by their antibiotic resistance pattern. Definitive diagnosis, often used for epidemiology of outbreaks, uses gene sequencing.

49 year old man with AIDS (CD4 count 43, HIV RNA 225,000) presented with 4 weeks of pain on defecation. His physical exam was notable for a tender, boggy prostate. The urinalysis showed 5‐10 WBC/hpf. The urine culture was without growth. A pelvic CT scan showed a prostate abscess. Aspirate of the abscess revealed the findings below. (yeast with capsule) Which of the following is the correct diagnosis? A. Blastomyces dermatitidis B. Pneumocystis jiroveci C. Histoplasma capsulatum D. Candida albicans E. Cryptococcus neoformans

Correct answer: E. Cryptococcus neoformans Discussion: The prostate aspirate showed evidence for Cryptococcus: On hematoxylin and eosin stain, there are yeast surrounded by a thick capsule (below). H & E stain Cryptococcus neoformans enters body via the respiratory route. The organism has a tropism for the central nervous system, and can cause meningitis. Other focal sites of infection may include the skin, bone or prostate. Cryptococcal prostate infection may be asymptomatic. The prostate may serve as a sanctuary from antifungal therapy; as a result, prolonged therapy may be required to cure the infection Differential diagnosis ¢ Blastomyces: may involve prostate; yeast form usually has thick wall, broad‐based budding ¢ Histoplasma: multipolar budding, sometimes with narrow buds ¢ Candida albicans: in tissues, yeast forms intermingled with pseudohyphae ¢ Pneumocystis: extra pulmonary disease may very rarely occur, but prostate involvement not typical

29 y/o from rhode island refused all immunization presents with malaise, loss off appetite, mild fever, jaundice 10 day camping trip to northern minnesota, no tick bites has some diarrhea cares for infants and toddler sin a low cost unlicensed program Sclera are icteric right upper quadrant tenderness with abdominal discomfort and hepatomegaly. No rash is detected except a few papules and small nodules with surrounding erythema that the woman attributes to bug bites. Labs: HGB 11.6, Platelets 240,000, WBC 3.4 (45% L, 44% PMN, 6% Monos, 5% Eos) Electrolytes and BUN/Creatinine ‐ normal ALT‐1235, AST‐677, Total Bilirubin 13.5, Alkaline Phosphatase 213 CPK 122, Amylase 89 Chest radiograph: normal cause> A. Anaplasma phagocytophilum B. Babesia microti C. Ehrlichia chaffeensis D. Leptospira icterohaemorrhagiae E. Hepatitis A virus

Correct answer: E. Hepatitis A virus typical for hepatitis A: ALT>AST, transaminases over 1,000 IU/l and total bilirubin over 10. supported by IgM and IgG serologies. Babesiosis, anaplasmosis and ehrlichiosis often cause elevated transaminases but not this high. Thrombocytopenia is usual in these three diseases, but this patient did not have thrombocytopenia. For leptospirosis, disease other symptoms and mild liver enzyme elevations, compared to a high bilirubin and alkaline phosphatase. Day care is one of the leading risk factors for HAV infection. Her immigrant clientele could be expected to have a low rate of immunization, and to have a higher than normal risk of importing hepatitis A from outside the United States. Children with HAV are often asymptomatic.

A 45‐year‐old male whom you are following for long‐term treatment of chronic osteomyelitis returns to your office complaining of decreasing vision of two weeks' duration. After an initial course of vancomycin, long term suppression with linezolid 600 mg po bid was begun three months ago. Four weeks ago numbness and tingling of his fingertips led to a diagnosis of peripheral neuropathy and pregabalin (Lyrica) was started. Two weeks prior, he noted blurring of vision in his left eye. He consulted an ophthalmologist who reported vision of 20/400 in both eyes. The anterior chamber, vitreous and retinas were normal except for some blurring of the disc margins bilaterally and a pale sector in the right disc. On visual fields, a scotoma was found in both eyes. The patient drinks 4 to 6 beers a day, and was treated for syphilis two decades previously. His medications are linezolid and pregabalin. The patient admits to a sexual encounter with a prostitute while on a business trip 6 weeks ago. Cuddled with the neighbor's cat a few weeks ago Which one of the following is the most likely diagnosis? A. Vitamin B deficiency B. CNS syphilis C. Bartonella henselae D. Varicella zoster vasculopathy without skin lesions E. Linezolid toxicity

Correct answer: E. Linezolid toxicity Linezolid can cause polyneuropathy and optic neuritis after several weeks or months of use. Hypesthesias and dysesthesias are more prominent in the hands than in the feet. Visual loss from optic neuritis is gradual and asymmetrical. The peripheral neuropathy may be reversible after stopping linezolid. However, reversal of the optic neuropathy is less frequent. Vitamin B6 might be useful for prevention. Inflammation of the optic nerve due to syphilis can occur during any stage of the infection, including tertiary syphilis. In 50% of cases, it is bilateral. This patient had syphilis 20 years earlier so he could have a relapse and may very well have tertiary ocular syphilis However, the co‐presence of a neuropathy favors linezolid as the cause. The other choices can cause optic neuritis but do not fit with the patient's clinical syndrome Tertiary syphilis most often presents as uveitis, iridocyclitis, or iritis, but can cause optic neuritis but would not explain the peripheral neuropathy Deficiencies of vitamin B12 or thiamine can cause peripheral neuropathy but not optic neuritis. Pregabalin does not cause optic neuritis or peripheral neuropathy.

70 year old man presented with an ulcerative lesion over the dorsum of his left wrist. He had been gardening 10 days earlier, and scraped his hand on a piece of metal in weeds near the house. The lesion had grown in size with surrounding erythema; two nodules had developed on his forearm, proximal to the wrist lesion. Epidemiologic history: he did not know of any rose bushes in the area. Although he had seen rabbits occasionally, he had not seen them near his house. Denied fresh‐ or salt‐water exposures. On exam (below): T 99.2°F (37.2°C). 1‐2 cm ulcer over dorsum of wrist; multiple erythematous subcutaneous nodules extending in a chain proximally. 2 x 3 cm tender, warm mass just above the elbow. Axilla: 2‐3 non‐tender lymph nodes. Remainder of exam normal. Which of the following is most likely? A. Staphylococcus aureus B. Mycobacterium marinum C. Leishmania braziliensis D. Francisella tularensis E. Nocardia brasiliensis

Correct answer: E. Nocardia brasiliensis nodular lymphadenitis diseaese: 1. sporotrichosis (rose bushes) 2. mycobaterium marinum - fish 3. francisella tularensis - rabbit but also systemic symptoms 4. leishmania braziliensis - south america Gram stain of skin biopsy showed beaded, branching gram‐positive rods; the organisms were also seen on a modified AFB stain. Culture grew Nocardia brasiliensis. 46 Sporotrichoid ("lymphocutaneous") pattern of lesions may be caused by: Nocardia: found in soil and water; may present after direct inoculation, e.g. gardening or trauma. Sporothrix schenckii: found in similar geographic areas as Nocardia. Mycobacterium marinum: usually follows exposure to organism growing in water, such as a pool, ocean or aquarium. Leishmaniasis: inoculation by bite of sand fly; typically acquired in Central or South America Francisella tularensis :ulcer is often particularly painful; patients may have fever, systemic symptoms, localized adenopathy.

A 48‐year‐old insurance company executive is seen for six days of unremitting fever and "aching all over." He says his leg muscles hurt when he walks, his arms hurt when he combs his hair, and his "eyes hurt" when he looks from side to side. He has no joint pain and no respiratory, urinary or gastrointestinal symptoms. On examination his temperature is 101.2°F. He has periorbital edema, a conjunctival hemorrhage, and subungual splinter hemorrhages in five fingers. There is tenderness on palpation of the trapezius and gastrocnemius muscles. Two months ago he went to Arizona on a business trip and played golf on a desert course. Two weeks ago he returned from a ten day vacation in Texas where fished for and ate fresh trout and hunted and ate wild boar, cooked rare. His hemoglobin is 14.2, his WBC is 18,000 (69% segs, 12% lymphs, 14% eos); his platelet count is normal. Which one of the following is the most likely explanation for his illness? A. Chikungunya B. Coccidioidomycosis C. Dengue D. Eosinophilic granulomatosis with polyangiitis (Churg -Strauss syndrome) E. Trichinellosis

Correct answer: E. Trichinellosis Muscle pain and tenderness, splinter and conjunctival hemorrhages, periorbital edema and ocular pain, and eosinophilia are all cardinal features of trichinellosis (formerly trichinosis). Fever is often present and may be high. Trichinellosis has been reported worldwide; the prevalence is highest in China, Thailand, Mexico, Argentina, Bolivia, the former Soviet Union, and other parts of Central Europe. Infection is acquired by eating meat containing cysts of Trichinella spp., usually from inadequately cooked pork from domestic pigs outside the United States ‐ domestic pigs in the United States are rarely the cause in recent years. Wild carnivorous animals harbor and perpetuate infections: walrus, bear, cougar, and wild boar. Undercooked wild game has emerged in recent years as a predominant source of infection in the United States; during 1997‐2001, pork products were associated with 40% of cases, and wild game was associated with 60% (most frequent was bear meat). The incubation period is generally 7 to 30 days‐this is an important clue here. Albendazole or mebendazole is the therapy of choice. Steroids may be used in severe cases. The other diseases listed explain some but not all the features of this patient's illness. Polyarteritis nodosa may cause fever, muscle pain and weakness. Dermatomyositis causes skin changes but not like this. Coccidioidomycosis, acquired in Arizona, can cause eosinophilia and fever, but is typically associated with pulmonary disease and would not generally cause periorbital edema or muscle tenderness. Eosinophilic granulomatosis with polyangiitis could explain blood eosinophilia and fever but asthma, a characteristic feature, is missing. As Dengue, Chikungunya, and Zika viruses spread to regions in the US, they will become increasingly part of the differential diagnosis, but the blood counts do not suggest these viral diseases, i.e., there are no cytopenias. These viral diseases would also not cause eosinophilia, periorbital edema, or subungual hemorrhages.

A 41‐year‐old man with recently diagnosed HIV disease is eager to start antiretroviral therapy. Initial work‐up reveals HIV RNA 1.7 million copies/ml, CD4 677, HLA‐B*5701 positive, genotype wild‐type, tropism test with dual/mixed virus, on proton pump inhibitor (PPI) for chronic gastric reflux. Which of the following regimens do you recommend? A. tenofovir/emtricitabine + rilpivirine B. abacavir/lamivudine + efavirenz C. tenofovir/emtricitabine + atazanavir D. tenofovir/emtricitabine + maraviroc E. tenofovir/emtricitabine + dolutegravir

Correct answer: E. tenofovir/emtricitabine + dolutegravir Individualization of an ART regimen based on patient factors is key for long‐term treatment success. Of those listed, the best choice is the U.S. DHHS recommended regimen of tenofovir/emtricitabine combined with the integrase inhibitor. Because the patient takes PPI, both rilpivirine and unboosted atazanavir are contraindicated. Regarding PPIs and boosted ritonavir, the answer is more complicated: if a PPI is used it should be administered at least 12 hours before the boosted atazanavir and probably at low doses, i.e., omeprazole 20 mg daily or equivalent. In addition, rilpivirine‐based regimens or abacavir/lamivudine regimens (except when combined with dolutegravir) demonstrated suboptimal virologic responses in patients with baseline HIV RNA levels >100,000 copies/ml (and his is much higher than that at 1.7 million copies/ml). Also, because he is HLA‐B*5701 positive, he is at greater risk of abacavir hypersensitivity reaction, and this drug should not be used. Finally, the patient has dual/mixed virus which means that maraviroc (active only against R5 virus) would not have sufficient virologic activity. Questions on the optimal choice for antiretroviral can be difficult board questions. It's easier to ask about regimens that should NOT be given. As of October 2017 these are the recommendations from the DHHS Guidelines; however, these recommendations change periodically. The Panel classifies the following regimens as recommended regimens for antiretroviral‐naive patients: 65 Integrase Strand Transfer Inhibitor‐Based Regimens: Dolutegravir/abacavir/lamivudine—only for patients who are HLA‐B*5701 negative (AI) Dolutegravir plus tenofovir disoproxil fumarate/emtricitabine (AI) Elvitegravir/cobicistat/tenofovir alafenamide/emtricitabine—only for patients with pre‐ antiretroviral therapy CrCl ≥30 mL/min (AI) Elvitegravir/cobicistat/tenofovir disoproxil fumarate/emtricitabine—only for patients with pre‐ antiretroviral therapy CrCl >70 mL/min (AI) Raltegravir plus tenofovir/emtricitabine (AI) Protease Inhibitor‐Based Regimen: Darunavir/ritonavir plus tenofovir disoproxil fumarate/emtricitabine (AI)

A 40-year-old male with HIV infection and CD4 =40 cells/uL presents with the tongue lesion that is shown. Scraping of the tongue were evaluated: Wet mount with Calcofluor stain for fungus and Gram Stain for bacteria were negative. The patient is asymptomatic and has no notable symptoms related to his tongue or mouth. Which of the following is linked etiologically to this lesion? HSVVZVEBVHHV-6HHV-8

Correct answer: EBV The white patches on the side of the tongue are oral hairy leukoplakia. Oral hairy leukoplakia usually involves the lateral tongue, but buccal mucosa floor of mouth, palate also may be involved. The lesion is EBV associated. No treatment is necessary although the lesions often regress with acyclovir. These lesions often disappear with effective antiretroviral therapy.

A 30-year-old man from Washington, DC and no recent travel developed non-bloody diarrhea with abdominal cramps so severe that an acute abdominal emergency was considered. After a day or two, gross blood became visible in the stools. He had no fever, his leukocyte count was mildly elevated, and he had only rare leukocytes found in the stool by microscopy after methylene blue staining. Colonoscopy with biopsy revealed submucosal edema, hemorrhage and ulcerations consistent with ischemic colitis. A stool sample sent to the laboratory was negative for the Clostridium difficile toxin B gene by PCR and for giardia by EIA. Stool cultures at 48 hours are negative for Shigella, Salmonella, and Campylobacter. Stool culture on sorbitol McConkey agar did not detect O157:H7. Which of the following tests would most likely be useful? EIA for shigatoxin in stool Stool antigens for Entamoeba histolytica PCR for C. difficile toxin A gene in stool Stool Gram stain for microsporidia PCR on stool for enteroinvasive E. coli

Correct answer: EIA for shigatoxin in stool bloody diarrhea, with severe abdominal pain, lack of fever and rare leukocytes in stool all suggest Shigatoxin producing enterohemorrhagic E. coli. For bloody diarrhea, shigatoxin producing E. coli should be the first thought, followed by Campylobacter, Shigella, Salmonella, or non-infectious causes. In the United States, E. coli 0157:H7 and other serotypes are both possible with geographically diverse epidemiology. For example, non-O157 STEC predominate in the mid-Atlantic region whereas the classic E. coli O157:H7 remains more common in the Pacific Northwest.. The EIA, while less sensitive than laboriously checking the O and H serotype of all sorbitol negative colonies, can detect shigatoxin produced by any of the serotypes. In addition, sorbitol positive strains of non-0157:H7 serotypes have been described. Although not offered as a choice, a number of multiplex PCR panels can detect the shigatoxin gene in STEC from stool. The acute onset and bloody diarrhea are unusual for amebiasis and not seen with microsporidiosis. There is no history of travel outside the United States to suggest amebiasis or HIV disease to suggest microsporidiosis. Nor does microsporidiosis cause bloody diarrhea. C. difficile toxin A does not cause disease in the absence of toxin B, which was negative by PCR. Enteroinvasive E coli is similar to Shigella. It causes diarrhea that can occasionally be bloody but fever and fecal leukocytes would be expected.

A 55-year-old man from Afghanistan was being evaluated for a complex cystic mass in his liver. While waiting for his next appointment, he rapidly develops shortness of breath 20 minutes after falling onto his abdomen while running to catch a train. In the emergency department, he is noted to be tachycardic, hypotensive, and diaphoretic. His tongue and lips are swollen, he has diffuse wheezing on pulmonary auscultation, and he has hives all over his body. Before he left Afghanistan five years ago, he had raised sheep in a small rural village. Hepatomegaly found on a routine physical exam led to an ultrasound examination which discovered a liver cyst. He had been afebrile and liver function tests had been normal. His clinical condition is most likely due to an immune response to which of the following? Echinococcus granulosus Entamoeba histolytica Fasciola hepatica Fasciolopsis buski Taenia solium

Correct answer: Echinococcus granulosus When this patient fell, he ruptured a hepatic cyst due to Echinococcus granulosus, and then had an anaphylactic response to the released cyst contents. E. granulosus is a tapeworm infection of dogs which utilizes sheep, goats, and other animals as intermediate hosts. Humans are accidental hosts that can become infected with the larval stage of the parasite after ingestion of E. granulosus eggs in food or water contaminated with dog feces. In humans, the parasites travel to visceral organs, primarily the liver and/or lungs, and grow into large cysts known as hydatid cysts. The inner membrane of hydatid cysts is composed of a germinal layer of pluripotential cells, each of which can develop into its own cyst. Echinococcal cysts frequently have multiple septations and daughter cysts within them. While hydatid cysts are often asymptomatic, they can cause symptoms by compression of vital structures and by rupture. Common clinical presentations include secondary cholangitis due to rupture into the biliary tree, biliary obstruction by compression, peritonitis by intraperitoneal rupture, and pneumonia symptoms by rupture into the bronchial tree. As with most helminths, Echinococcus induces the production of parasite-specific IgE antibodies. Sudden rupture of an echinococcal cyst can thus induce an anaphylactic reaction due to the rapid release of large quantities of helminth antigens. The patient is at risk of developing new daughter cysts throughout the abdomen and should be treated with albendazole. Entameba histolytica can cause amebic liver abscesses. However, amebic liver abscess is usually an acute disease (occurring 8-20 weeks after acquisition) which he would not likely have acquired in the United States. (Cases of presentation years after exposure have been reported but are unusual). Additionally, symptomatic amebic liver abscess is usually accompanied by fever. If an amebic liver abscess ruptured due to trauma, the rupture might cause shock but the rupture would not cause an allergic reaction. Fasciola can cause an acute inflammatory lesion in the liver, and then chronically be associated with biliary flukes which may lead to biliary obstruction and pyogenic liver abscess, but not the acute allergic syndrome described in this case. Fasciolopsis buski is an intestinal fluke which causes occult blood loss. Paragonomus most often causes pulmonary nodules or hemoptysis.

This skin lesion (necrosis and hemorrhage in a neutropenic patient) is most typical of which one of the following entities: Choices A Purpura fulminans B Impetigo C Pyoderma gangrenosum D Ecthyma gangrenosum

Correct answer: Ecthyma gangrenosum The sharply delimited border, central necrosis and pale outer border is typical of ecthyma gangrenosum. Ecthyma gangrenosum is classically caused by Pseudomonas aeruginosa but can be caused by invasive molds, such as aspergillosis and mucormycosis or by a variety of other bacteria and yeasts. Culture of a punch biopsy is needed to establish the causative agent since blood cultures are often negative. Ecthyma gangrenosum typically begin as painless red macules that quickly evolve into areas of induration with pustules or bullae. These lesions progress to gangrenous ulcers. Ecthyma lesions typically progress within 12 to 18 hours). Ecthyma gangrenosum results from perivascular bacterial invasion of the media and adventitia of arteries and veins. There is secondary ischemic necrosis. The term "Ecthyma" refers to an ulcerative lesion that extends through the epidermis and deep into the dermis. The lesion appears as a "punchedout" ulcer covered in crust surrounded by raised violaceous margins. Although ecthyma gangrenosum can occur at any anatomic location, the anogenital and axillary areas are most often involved.

A 26-year-old female from Jamaica visiting family in New York City sought medical attention for facial swelling and pain of one day's duration. She had a temperature of 39°C and swelling with tenderness over the submental area. The tongue was somewhat raised but she could talk and swallow normally. Opening her mouth beyond a centimeter or two was painful. No lymphadenopathy could be felt in the submandibular or submental area, which was diffusely tender and firmly indurated. Her right mandible was also tender in the midportion. Dentition, as best it could be seen, was poor, with severe caries. The most appropriate management at the moment is which of the following: Administer high dose corticosteroid therapy for 5 days Surgical consultation for drainage of the submandibular area Dental consultation for drainage of probable periapical abscess Empirical therapy with piperacillin-tazobactam Administer diphtheria antitoxin

Correct answer: Empirical therapy with piperacillin-tazobactam Ludwig's angina is an infection spreading from a periapical abscess around the first molar tooth and extending into the sublingual space. Therapy is directed towards the aerobic and anaerobic oral flora. Hence, needs drug with predictable activity vs anaerobic pathogens: Pipaeracillin -tazobactam, ampicillin-sulbactam, or, if severe pen-allergy, clindamycin. Resistance of aerobic gram-negative bacilli, e.g., E. coli, to ampicillin-sulbactam is increasing and is now often around 20%; Hence, some advantage of piperacillin-tazobactam Surgical drainage is usually not helpful because there is usually s no focal abscess in the soft tissue. Emergent protection of the airway may be important if treatment is delayed but response to antibiotics is usually rapid. A root canal or other treatment of the dental abscess can wait until the acute problem has subsided. Diphtheria causes a "bull neck" but the predominant finding is posterior pharyngitis with white plaques over the tonsils, posterior oropharynx, uvula and contiguous structures.

This red lesion in the ocular fundus is most consistent with which of the following: Miliary tuberculosisCandidemiaEndocarditisSystemic lupus erythematosus

Correct answer: Endocarditis A retinal hemorrhage with a pale center is called a Roth spot, and occurs in endocarditis. Retinal lesions due to candidemia, military tbc and SLE (cytoid bodies) are pale with no surrounding hemorrhage. Make certain you can distinguish the following: Roth spots - Exudative, edematous hemorrhagic lesions of the retina which have pale centers Janeway lesions - Nontender erythematous macules on the palms and soles. Janeway lesions are more common in acute endocarditis than subacute disease. They reflect microabscesses and demonstrate neutrophil infiltration of capillaries. Osler nodes are Tender tender subcutaneous violaceous nodules which occur on the pads of the fingers and toes, and/or the thenar and hypothenar eminences. They are rarely seen now that most endocarditis is diagnosed sooner than in an earlier era. Osler nodes and Roth spots occur most frequently in the setting of prolonged bacteremia. They are due to vascular occlusion by microthrombi leading to immune-mediated vasculitis in a very focal area.

A 53 year old diabetic man is admitted for suppurative cellulitis and contiguous osteomyelitis of the right foot. Intraoperative cultures grow methicillin-resistant Staphylococcus aureus (vancomycin MIC=2 mcg/ml, daptomycin MIC=0.25 mcg/ml). Admission blood cultures are no growth. He is treated with daptomycin 6 mg/kg IV once daily and transferred to a skilled nursing unit to complete a 6-week course of therapy. On day 22 he develops fever, dyspnea, and blood oxygen saturation of 65%. Lung exam shows diffuse rales. Peripheral white blood cell count, 5,800/mm3 a week ago, is 15,400 with 80% neutrophils, 12% lymphocytes, and 8% eosinophils. Chest radiograph shows interval development of patchy bilateral airspace consolidations. Mixed oral flora are seen on sputum Gram stain. What is the most likely explanation for this clinical picture. Acute MRSA endocarditisAcute pulmonary embolismCardiogenic pulmonary edemaHospital-acquired acute bacterial pneumoniaEosinophilic pneumonia

Correct answer: Eosinophilic pneumonia Eosinophilic pneumonia (in this case marked eosinophila of > 1,200 eosinophils per mm3 is an uncommon, but well recognized side effect of daptomycin). This degree of eosinophilia should not be ignored. None of the other conditions (endocarditis, bacterial pneumonia, pulmonary embolism, cardiogenic pulmonary edema) can account for the eosinophilia. MRSA endocarditis given negative blood cultures at the time of admission is extremely unlikely. The chest radiographic findings and the eosinophilia are atypical for pulmonary embolism.. . Except for the eosinophilia, the infiltrates and hypoxemia could be due to cardiogenic pulmonary edema, but in absence of a prior history of cardiac failure this would be lower down the list. Nosocomial pneumonia is a consideration here as well, but the sputum gram stain is not suggestive. This of course does not rule out community acquired pneumonia or HAP (hospital acquired pneumonia), but in this case, the unremarkable gram stain and the peripheral eosinophilia should make an eosinophilic, drug related pneumonia more likely.

You are called by a family physician about a patient, a 17-year-old whom she saw two days earlier for severe sore throat and malaise of five days duration. The patient was well until he developed the sore throat accompanied by low grade fever and "feeling tired and sick." He doesn't know anyone else who is sick. He is sexually active with a single partner and always uses condoms. On exam, his temperature was 100.8°F; pulse 86, BP 112/78. He had periorbital edema and bilateral anterior and posterior cervical nodes that were more prominent posteriorly. His throat was red with small exudates. The spleen tip was palpable. A rapid strep test performed in the family physician's office was negative. The doctor thought the young man had mononucleosis and ordered a CBC and Monospot test (heterophile antibody). The WBC count was 12,000; there were 32% lymphocytes and 12% atypical lymphocytes and the platelet count was slightly low at 120,000. The Monospot test was negative. Which one of the following is most likely responsible for the young man's illness? Cytomegalovirus HIV Epstein-Barr virus Toxoplasma Human herpes virus 6

Correct answer: Epstein-Barr virus This patient has a classic mononucleosis syndrome. About 90% of mononucleosis like illnesses in adolescents and young adults are due to Epstein-Barr virus, and the peak age for EBV mononucleosis in the United States is 16 to 17 years of age. The Monospot test, a rapid agglutinin test for heterophile antibody, is highly specific but not highly sensitive The important point is: The false negative rates are highest during the beginning of clinical symptoms (25 percent are falsely negative in the first week; 5 to 10 percent in the second week, 5 percent in the third week) Measurement of EBV-specific antibodies is usually not necessary since the majority of patients are heterophile positive. However, testing for EBV-specific antibodies can confirm the diagnosis if the heterophile test is negative and there is a need for a definitive diagnosis. IgM and IgG antibodies directed against viral capsid antigen (VCA) have high (>95%) sensitivity and specificity for the diagnosis of mononucleosis. Ten percent of mononucleosis syndromes are due to other agents including HIV, CMV (sore throat less likely), toxoplasmosis (no sore throat), HHV-6 and HHV-7. For strep pharyngitis there should be no splenomegaly and no severe fatigue. Acute HIV infection is a consideration but less likely. It can cause a mononucleosis like syndrome with symptoms that can include fever, sore throat, lymphadenopathy, rash, myalgia/arthralgia, diarrhea, weight loss, headache, and mucocutaneous ulcers. Typically, the total WBC count and the absolute lymphocyte count are low, unlike this patient. More importantly, if we take his sexual history at face value, he is at low risk because of consistent condom use.

A 56-year-old commercial crab fisherman on the Chesapeake Bay is seen for a painful, red hand. Three days ago he noticed a red dot on his index finger that was became increasingl painful. The lesion progressed to a red-purple involvement of his entire index finger, his middle finger, and most of the dorsum of his hand looking like a cellulitis. He is afebrile and says the involved area is quite painful but only slightly tender to the touch. He says the finger joints feel stiff although there is no joint swelling on exam. Which one of the following is the most likely cause of his problem? Erysipelothrix rhusiopathiae Mycobacterium chelonae Sporothrix schenckii Aeromonas Pseudomonas aeruginosa

Correct answer: Erysipelothrix rhusiopathiae Persons in contact with shellfish, pigs or occasionally other animals are at risk for erysipeloid, a subacute cellulitis due to inoculation with this Gram positive bacillus that can persist for long periods on the surface of shellfish or cause chronic infection in the skin or pigs and some other domestic animals. Erysipeloid, a subacute infection of the skin that begins with a painful red dot and progresses over days to a red-purple discoloration of the skin with minimal swelling and pain often out of proportion to the visible lesion. Bacteremia is uncommon but when it occurs is usually associated with aortic valve endocarditis. None of the other entities fit this background or clinical picture. Nontuberculous mycobacteria, such as M. marinum, cause a much more indolent course.

A 54-year-old woman has had multiple urinary tract infections, and now comes in with fever and right flank pain. On physical examination, you find a mass in the right flank which is tender, and obtain the abdominal image shown below. There are staghorn calculi present. Her urinalysis reveals pyuria. Because of concern for renal cell carcinoma, a nephrectomy is performed. The pathologist finds no tumor, but does find numerous lipid laden macrophages in the yellow colored renal tissue, along with granulomas and inflammatory cells. These masses most likely represent a pathologic response to: Methicillin resistant staphylococcus aureus Candida albicans Nocardia asteroides Escherichia coli Adenovirus

Correct answer: Escherichia coli This patient has xanthogranulomatous pyelonephritis which occurs in middle aged women with recurrent urinary tract infections. The presentation clinically and in the urine is similar to pyelonephritis. The imaging typically shows a non-functioning kidney with or without calculi. The most common causes of this syndrome are: Escherichia coli, Proteus mirabilis, Pseudomonas aeruginosa, Enterococcus faecalis, and Klebsiella species. Xanthogranulomatous pyelonephritis is characterized by a destructive mass that invades the renal parenchyma. The kidney is usually nonfunctional. Most cases of XGP involve the entire kidney but 20% are focal. Pathologically, XGP demonstrates lipid-laden foamy macrophages and is usually associated with urinary tract obstruction, infection, nephrolithiasis, diabetes or some other form of immunosuppression. Therapy is usually removal of the kidney.

A 17-year-old girl from the suburbs near Seattle was brought to the emergency room in July because she began stumbling while walking and over the next 24 hours became completely unable to stand up. Her arm strength also appeared to be decreasing. She had no recent travel. She played in the woods near her home with friends but none of them were ill. She had eaten honey purchased at the county fair the prior day but nothing else out of the ordinary. She had received all the childhood vaccinations. On examination, she was alert and oriented. Vital signs were normal. Her motor strength and deep tendon reflexes were severely reduced in the legs and somewhat reduced in the arms. Sensation appeared intact, as were cranial nerves. MRI of the head and lumbar puncture were normal. The test most likely to reveal the cause of the paralysis is which of the following: Examination of her scalp Serum lead EMG Serum IgM antibody to Borrelia Serum analysis of botulism toxin

Correct answer: Examination of her scalp The Dermacentor ticks usually must attach for several days. Patients complain of fatigue and paresthesias, and then progress to weakness which can be symmatical, asymmetrical, ascending, or bulbar. There is no fever and CSF is normal. The symptoms may improve within a few hours of tick removal, or may not improve for a day or two. Ascending paralysis can be due to Guillain-Barré syndrome, myasthenia gravis or tick paralysis. Careful examination of the skin, including the scalp, with removal of the tick can provide both diagnosis and cure. The normal CSF and absence of sensory findings is atypical for Guillain-Barré. The onset is too rapid for myasthenia gravis. Lack of sensory findings and rapid progression is against lead neuropathy. Ascending paralysis has not been seen with CNS Lyme disease. Botulism causes paralysis of the cranial nerves before appearing in the extremities. Honey in infant formula has been implicated in infant botulism, but not in older persons. Honey contains the spores that are thought to germinate in an infant intestine, but honey does not contain botulism toxin. A variety of Dermacentor, Amblyomma, and Ixodes ticks can transmit the causative neurotoxin.

A 29-year-old travels to Papua New Guinea. He takes mefloquine 250 mg weekly starting two weeks before and for four weeks after travel as recommended. He was not told to take other drugs. Six months after the trip, he becomes febrile and is diagnosed with smear positive Plasmodium vivax infection. The explanation for this is most likely: Primary infection with mefloquine-resistant P. vivax Lack of adequate adherence with the mefloquine regimen Heavy exposure to P. vivax Failure to eradicate the hepatic hypnozoite

Correct answer: Failure to eradicate the hepatic hypnozoite This is a relapse of P. vivax due to failure to eradicate the hepatic hypnozoite. Malaria prophylaxis does not prevent malaria infection. Mefloquine acts in the blood only, so this drug provides only suppressive prophylaxis for either P. falciparum or P. vivax. Mefloquine doesn't penetrate the liver and is not active against the dormant hypnozoites of P. vivax in the liver. The only drug active against hypnozoites (the stage that causes relapses of P. vivax or P. ovale) (the only two species capable of relapse) is primaquine. This patient never received terminal prophylaxis with primaquine after travel to a very high-risk area. P. vivax primarily resistant to mefloquine in the erythrocytic stages has not been described. If the patient had been non-compliant a primary clinical illness with P. vivax almost always occurs earlier than six months after exposure.

A 62-year-old female just returned from Bolivia with 3 months of fever, malaise, loss of appetite, 10 kg weight loss, right upper quadrant pain and icterus. While in Bolivia, she had heard of the beneficial health effects of watercress which she consumed regularly before the onset of her illness She is on no medication. Physical examination discloses an afebrile woman with a non-tender liver 3 cm below the costal margin and with no splenomegaly. Lab: Hct 31. WBC 15,600 (21% PMN, 18%L, and 56% eos) Blood cultures negative on three consecutive days Hepatic transaminases 5 times normal with normal alkaline phosphatase Hepatitis A, B, C, serology negative Stool O & P X 3 is negative Chest x-ray is normal Abdominal CT scan shows multiple hypodense lesions throughout the liver parenchyma without any abnormalities of the biliary ducts. The most likely diagnosis is: Schistosoma mansoniToxocara canisAscaris lumbricoidesClonorchis sinensisFasciola hepatica

Correct answer: Fasciola hepatica The Altiplano of Peru and Bolivia highly endemic for fascioliasis. The differential diagnosis of hypereosinophilia with destructive hepatic lesions is limited. For Toxocariasis (dog round worm acquired by humans, usually children, who accidentally ingest dog feces) the pathology is due to granulomas around individual non-migrating larvae and is non-destructive, i.e. this amount of transaminase elevation would be unusual. Liver masses and hepatomegaly are more common. Schistosoma mansoni does not exist in Bolivia and pathology is due to egg deposition in the liver with eventual peri-portal fibrosis. Ascariasis (human round worm) causes physical obstruction of large biliary ducts. Eosinophilia can be marked during the stage of pulmonary migration, but is not common with adult worms in the biliary duct. Clonorchis is restricted to Asia. The presentation could be similar to Fasciola except this patient was not in Asia. Clinically, hepatic fascioliasis can be divided into acute and chronic phases. During the acute phase, which occurs 6-12 weeks after ingestion, migrating parenchymal larvae cause fever, hypodense lesions in the liver, eosinophilia, right upper quadrant pain and especially significant anorexia. Vomiting and weight loss of 20 kg or more may develop, which usually abates when the larvae mature to adults. The adult flukes in the biliary tree are generally asymptomatic but some patients develop chronic manifestations including right upper quadrant pain, nausea, vomiting, and hepatomegaly. Eosinophilia and abnormal liver function may develop but are less common than with acute disease. Malignant degeneration and cholangiocarcinoma such as results from chronic infection with the oriental liver fluke, Clonorchis sinensis, has not been reported with Fasciola hepatica. Diagnosis is best made by serology as egg production by the adult flukes is scanty and intermittent. This is the only trematode infection for which praziquantel is not the drug of choice. Triclabendazole is the recommended treatment.

Which of the following is the most common route of infection for the parasite whose egg is shown in the figure? Direct inoculationDirect skin penetrationFecal-oralInhalationMosquito bite

Correct answer: Fecal-oral Enterobius vermicularis is commonly known as the pinworm. Its unusual symptom is anal itching. The adult worm lays her eggs in the perianal area and person-to-person spread occurs most commonly by the fecal-oral route. Eggs become infective after only 6 hours of exposure to air. The organism is unable to cause disease by direct inoculation, direct skin penetration, inhalation, or mosquito bites.

This is a light microscopic image of a stool sample obtained from a 72-year-old Caucasian West Virginia coal miner who was referred for an unexplained eosinophilia of 18% with a normal WBC of 8,000. He had retired from the mines 18 years ago and spent most of his time gardening at home. He had been receiving isoniazid, rifampin, and ethambutol for tuberculosis for six months and the referring physician did not want to stop the patient's anti-tuberculous medications to determine if the eosinophilia was due to an allergic reaction to medication. The likely source from which the patient picked up this organism is which one of the following: Poorly cooked porkFecal contamination of ground water in the minesHis dogInsect bite

Correct answer: Fecaly contamination of ground water in the mines This is the larva of Strongyloides stercoralis, an infection that can persist for many years, probably in part to autoinfection. Eosinophilia is often a clue to the diagnosis in asymptomatic or mildly asymptomatic patients. Infection is acquired from walking on soil or mud contaminated by human feces. A common practice in the coal mines was to defecate in an unused tunnel without returning to the surface. Water constantly present in mine shafts moved enough of the excrement to working areas of the mines to penetrate intact skin of workers. Strongyloides larvae can be difficult to demonstrate in feces so that multiple stool examinations for larvae are necessary. Serodiagnosis is helpful if there is a good exposure history.

A 40-year-old white male was switched to a darunavir+ritonavir+tenofovir (TDF)/emtricitabine regimen, secondary to virologic failure and multiple PI mutations including I50L. He presents to the clinic 6 months after the switch for a routine evaluation. Current CD4 = 350 and VL = <50 copies Lipids drawn during the evaluation reveal Total cholesterol of 260 LDL 130 HDL 35 Triglycerides 1200 What is the most appropriate management for his hypertriglyceridemia? Switch patient to an atazanavir-based regimen .Increase exercise regimen .Fenofibrate Fat-free diet

Correct answer: Fenofibrate This patient should be started on fenofibrate to reduce his triglycerides. He should not be put on atazanavir based regimen because of his I50 L mutation, which is an important part of this question. Atazanavir based regimens are attractive for patients with lipid abnormalities. For the boards, however, you should recognize that I50L mutation is associated with non-response to atazanavir. This patient needs some other drug, not atazanavir. Switching to an integrase inhibitor based regimen might be an option, but that choice is not listed in the answers. This lipid profile reveals a predominant hypertriglyceridemia. Statins are not the preferred agents for the treatment of predominant hypertriglyceridemia. Fenofibrate would be reasonable choices, so this is a good answer. Niacin is usually used as a second line drug for triglycerides if fibrates are not effective or not tolerated. With his triglyceride level >1000 he is at increased risk for pancreatitis and should be treated. An important fact not tested here: if pravastatin therapy were a consideration: despite the lack of Cyp 3A4 metabolism, the use of pravastatin with darunavir results in significant increases in the pravastatin AUC and if used is recommended to be started at the lowest dose.

A 17-year-old man vacationing in Virginia is seen in the emergency room for purulent conjunctivitis. On exam his temperature is 100°F; pulse is 88. His right eye has markedly inflamed conjunctivae and a grossly purulent discharge. The eyelid is slightly tender. Vision is intact. A tender preauricular node is present on the right. The left eye is normal as is the rest of the exam. He reports that two days before his eye began to trouble him he was removing ticks from his pet dog and killing them by squeezing them between his thumb and forefinger. He remembers getting squirted in the right eye with some "tick juice." Which one of the following is most likely the cause of his eye infection? Adenovirus Francisella Ehrlichia Leptospira Rickettsia

Correct answer: Francisella Tularemia due to Francisella tularensis usually is transmitted through direct animal contact, but may be transmitted through the bites of deer flies, mosquitos, and ticks. Direct inoculation of Francisella into the eye through rubbing or "tick juice" results in a purulent conjunctivitis with regional (preauricular) adenopathy. Adenovirus is a common cause of conjunctivitis and may be associated with preauricular adenopathy, but gross purulence does not occur. Ehrlichia and Rickettsia are tick-borne pathogens but do not produce this syndrome. Leptospirosis causes conjunctival suffusion, but this disease is usually acquired from animal urine or contaminated water, and the suffusion would be bilateral without an adjacent lymph node enlargement. Sarcoidosis may cause an oculoglandular syndrome, but the conjunctival inflammation is not purulent. This syndrome could have been due to Bartonella henselae (Cat Scratch Disease) but this was not offered as a choice since it would have been a second possible correct answer. Parinaud's oculoglandular syndrome is an unusual form of cat scratch disease that includes conjunctivitis, with adjacent preauricular lymphadenopathy. The inoculation of the Bartonella henselae occurs via a cat bite or lick near the eye or self-inoculation from another site. The presentation of this case is consistent with this except for the purulence of the conjunctivitis but the epidemiologic clues point to Francisella. For the exam look for a case of Parinaud's oculoglandular syndrome! There are many causes, including bacteria (e.g., Bartonella, TB, tularemia, and syphilis), fungi, viruses and inflammatory diseases (e.g., sarcoid)-but look for Bartonella and Francisella on an exam).

A 16-year-old girl with acute lymphoblastic leukemia presents with fever and multiple red skin nodules up to 1 cm diameter. Histopathology of the skin lesion reveals acute-branching septate hyphae. A hyaline mold is isolated from the skin biopsy and also from two blood cultures. What is this fungal genus involved? CandidaAspergillusBlastomycesHistoplasmaFusarium

Correct answer: Fusarium Although molds such as Aspergillus species can cause disseminated systemic infections in immunocompromised patients such as this one, relatively few molds are recovered in blood cultures. Fusarium species is an important exception in that half the cases in severely immunosuppressed patients have a positive blood culture. Blastomyces and Histoplasma species would show yeast in tissue, not hyphae. Candida species would grow as a yeast.

A 53-year-old rancher from a rural area of Colorado sought medical care in a New York City Emergency Department after two days of fever, fatigue, and painful left axillary swelling. There was a scratch on his left hand, which he thought happened when he tried to pick up his sick cat. He denied cough or shortness of breath. On clinical examination: He appeared ill with diaphoresis, rigors, and lower extremity cyanosis. His temperature was 104.4° F (40.2°C), blood pressure was 78/50 mm Hg, and oxygen saturation was 98% on room air. He had tender left axillary adenopathy with overlying erythema and edema. White blood cell (WBC) count was 24,700/µL (and platelet count was 72,000. Chest X-ray was normal. An aspirate of the inguinal node revealed short gram-negative rods. He had arrived in New York City two days previously to attend a meeting. Which of the following would be most appropriate to include in his initial antibiotic regimen? Imipenem Piperacillintazobactam Ceftazidime Gentamicin Trimethoprim-sulfamethoxazole

Correct answer: Gentamicin Suppurative lymphadenitis with septic shock due to a Gram-negative bacillus in a previously healthy patient should raise the question of tularemia or plague. Cat scratch disease due to Bartonella henselae is never this severe and cannot be seen on Gram stain. The drug of choice for tularemia or plague would be streptomycin or gentamicin. Lymphadenitis from either Yersinia pestis or Francisella tularensis follows inoculation into the skin. In tularemia, infection follows exposure to ticks, flies, or tissues of infected animals. Plague can be inoculated into the skin by the bite of rat fleas, or occasionally, by the scratch or saliva from an infected cat. Outdoor cats become infected during encounters with infected rodents. Respiratory transmission from infected cats can also occur. The laboratory to which specimens are submitted should be notified that either disease is suspected so that infection of laboratory personnel can be avoided.

In July, a woman who is 34 weeks pregnant is seen for an expanding ovoid rash on her anterior abdomen in the belt line. She lives in an area of high endemicity for Lyme Disease, and she takes daily walks with her dog in the woods behind her home. A diagnosis of erythema migrans is made. Which one of the following is the most appropriate approach to this patient? Give amoxicillin. Give doxycycline. Induce labor. Order Lyme serology.

Correct answer: Give amoxicillin. Diagnosis and treatment for Lyme disease during pregnancy is the same as in the non-pregnant patient, except that tetracyclines should be avoided. Amoxicillin and cefuroxime have been shown to be as effective as doxycycline for early Lyme Disease; azithromycin is less effective. There is no reason to use parenteral therapy here. There is no reason to terminate the pregnancy since there is no definable congenital Lyme disease syndrome, and, if adequately treated, Lyme disease occurring during pregnancy is not known to predispose to congenital anomalies or fetal demise. Erythema migrans is a clinical diagnosis; serology is often negative in early Lyme disease presenting as EM.

A 68-year-old man with metabolic syndrome is hospitalized for same-day surgery to repair an inguinal hernia. Six hours post-op, he develops fever and ankle pain. His medications are an oral hypoglycemic, a statin, and a thiazide diuretic. He has had no recent travel. He has a new kitten that frequently scratches him. On exam, he is obese, in pain, and has a temperature of 101.4°F. He has scratches on both hands without evidence of infection. His right ankle is swollen, warm and red, and painful on active and passive motion. His white blood cell count is 13,350 (90% polys). His uric acid is normal. An ankle joint fluid aspirate has a WBC count of 51,400 (90% polys). Gram stain of the joint fluid shows many WBCs but no organisms. The most likely diagnosis in this patient is which one of the following? Bartonella henselae arthritis Pasteurella multocida arthritis Gout Pseudogout Staphylococcus aureus arthritis

Correct answer: Gout Gout should be thought of when a patient presents with an inflammatory arthritis but gram stains of joint fluids are negative. Surgery is a common precipitating event for gout and uric acid levels, even in patients with hyperuricemia, are often normal during attacks of acute gouty arthritis. Staphylococcal arthritis with a negative Gram stain in the absence of antibiotic therapy would be uncommon. Pseudogout (caused by calcium pyrophosphate crystals) may also be precipitated by surgery but fever and leukocytosis are uncommon when compared with gout, and the knees are disproporationally involved with this disease. Bartonella henselae causes osteomyelitis, but acute febrile monoarticular arthritis would be unusual. Pasteurella bacteremia and arthritis would generally be accompanied by celllulitis at the site of a cat bite or nip.

This 40-year-old crab fisherman working in the Chesapeake Bay waters came in with low grade fever and a painful rash on his hand of three days' duration. He cut his hand several days ago on a crab spine. The probable organism based on his exposure and the image is which of the following: S. aureus impetigoS.pyogenes ecthymaS. iniaeErysiplothrixVibrio vulnificus

Correct answer: Gram positive bacillusErysipelothrix rhusiopathiae causes "erysipeloid" lesions following scratches in brackish water. Bacteremia is uncommon, but when it occurs, is usually accompanied by endocarditis. Erysipelothrix rhusiopathiae is a pleomorphic, gram-positive bacillus capable of causing self-limited soft tissue infection or serious systemic infection. E. rhusiopathiae is widespread in nature, occurring in domestic and marine animals including cattle, chickens, crabs, and fish. Crab pickers are classic, but swine farmers may be more common. Infection in humans is usually due to occupational exposure. Thus, slaughterhouse workers, butchers, fishermen, farmers, and veterinarians are at risk. The typical manifestations are localized or diffuse skin lesions, or bacteremia with or without endocarditis. Why is this not Mycobaterium marinum? The incubation period is too short. M. marinum has an incubation period of 2-3 weeks. The exposure is also more likely to be a non-chlorinated swimming pool or a fish tank. Erysipeloid is also flatter and more diffuse than the nodular, discrete lesions of M. marinum.

A 70-year-old male presents to the Emergency Room with confusion, slurred speech and a right sided weakness of 3 hours duration. He had previously been healthy except taking methotrexate and infliximab for rheumatoid arthritis. He has no history of headaches and no pain on palpation of his forehead. MRI with gadolinium contrast showed restricted diffusion in the left posterior basal ganglia extending to the internal and external capsule, compatible with an acute stroke. LP showed: 90 wbc (90% mononuclear) 5 rbcs Glucose: 50 mg/dl Protein: 60 mg/dl The patient's wife reports that he had shingles on his left forehead and around his eye 7 weeks ago. This began while he was on a Mediterranean cruise, delaying medical attention. While extremely painful, the rash had improved over three weeks with famciclovir and prednisone. If this lesion were caused by an infectious agent, which of the following would be the most likely etiologic agent? West Nile virus CMV HSV VZV Tick borne encephalitis virus

Correct answer: Granulomatous angiitis from VZV Clinicians must be aware of neurologic complications of zoster. This case is most likely large vessel vasculitis (granulomatous arteritis) characterized by the development of acute stroke weeks or months after zoster ophthalmicus. Most patients are over 60 years old, immunosuppressed or normal and typically have their event 7 weeks after the localized zoster, although intervals up to 6 months are documented. Transient ischemic attacks may also occur with granulomatous arteritis, as will less well-defined cognitive changes. The lesion is due to focal constriction and segmental narrowing of the internal carotid, or anterior or medial cerebral arteries. Histology reveals multinucleated giant cells, inclusion bodies, and viral particles. Other vessels can be involved if the zoster occurs in other cranial distributions. LP, if done, shows a modest lymphocytosis, less than 100 cells/cu mm. Although there are not controlled trials, given the pathology, acyclovir for 7-10 days and prednisone (60-80mg qd x 3-5 d) is recommended. For the ID boards, remember CNS complications of VZV, such as Ramsey Hunt syndrome (zoster of the geniculate ganglion presenting as vesicles in the internal or external ear or palate or tongue associated with cranial nerve VII palsy and treated with acyclovir/prednisone), cerebellar ataxia, transverse myelitis and encephalitis in immunocompetent adults, and small vessel disease encephalitis, especially in HIV infected patients. Some of these syndromes occur in the absence of a clinically apparent rash. Other parts of the differential, not listed here, would be an ischemic stroke or temporal arteritis. An MR angiogram should define the arterial abnormalities and are distinct for temporal arteritis vs ischemic stroke vs granulomatous arteritis. Tickborne encephalitis occurs in Europe as well as Siberia and the Far East, mostly in campers or workers exposed to ticks. However, focal cerebral lesions are uncommon, and there are no clues here about a biphasic illness or tick exposure. West Nile causes encephalitis and muscle weakness, not hemiplegia. On MRI, but look for basal ganglia or thalamic lesions as well as extrapyramidal signs for the encephalitis manifestation of this infection. HSV encephalitis should not present with hemiparesis and the lesion on imaging should be in the temporal lobe. HSV encephalitis should be associated with bizarre behavior and a temporal lesion, at least classically.

A 42-year-old male with HIV (initial CD 4= 10, initial viral load -500,000 copies but current CD4 =50, VL <50 on darunavir/ritonavir, tenofovir, emtricitabine started at another clinic 1 month ago) presents with 2 months of apathy, mild memory loss, bilateral hand tremors , but no fever. Evaluation reveals of negative serum IgGs for toxoplasma and CMV and negative RPR and FTA. LP reveals 20wbc (100% lymphocytes), protein 11mg/dl, glucose 80g/dl) crypt antigen negative. VDRL negative. MRI shown with dialeted ventricles, cortical atrophy, but no other abnormalities. The most like cause of this patient's cognitive decline is: HHV-6 encephalitis Progressive multifocal leukoencephalopathy (JC encephalitis) Syphilis HIV encephalopathy HHV 8 encephalopathy

Correct answer: HIV encephalopathy classic HIV dementia with mild ventricular enlargement and periventricular enhancement. triad of cognitive loss (memory, concentration, and executive function), behavioral issues (apathy) and motor abnormalities (tremor) HHV 8 does not cause encephalopathy. HHV 6 has occasionally been associated with an acute or subacute encephalopathy, but this is uncommon. JC encephalitis, also known as Progressive Multifocal leukoencephalopathy, usually manifests with multiple focal white matter lesions. A CSF JC PCR would confirm this diagnosis if the clinical history and MRI were compatible. With negative serum RPR, CNS syphilis is not likely in a patient of this age. The negative CSF VDRL does not, however, rule out neurosyphilis.

An HIV infected patient with CD4 = 24 presents with painful ulcerating skin and mucosal lesions of several weeks' duration: typical lesions are shown here. The most likely cause of these lesions are: CMVHSVEBVHHV-6Adenovirus

Correct answer: HSV These are most likely due to Herpes simplex virus. After chronic treatment, the virus may become resistant to acyclovir and valacyclovir. CMV, adenovirus and HHV6 do not characteristically cause ulcerating lesions, especially in these locations which are typical for HSV. Perirectal lesions are the most common sites; fingers can be involved due to direct spread from contact with an infected mucosal surface or a secre

For the facial syndrome shown, what would be the most common etiologic agent? HSVVZVCMVEBVHHV6

Correct answer: HSV This patient has facial palsy, also known as Bells palsy. The most likely cause of this lesion among those listed is HSV. HIV and Lyme disease are less common causes. Of note, Lyme is rarely the cause of Bells palsy unless there are other manifestations of Lyme. VZV can also cause this syndrome, as in the Ramsey Hunt Syndrome (involvement of otic canal). For typical facial palsy, prednisone is the preferred therapy, optimally given within 3 days of onset, for one week duration (prednisone 60-80mg qd). Acyclovir alone is not better than placebo, although there might be some rational (unproven) to add acyclovir or valacyclovir to prednisone. CMV is an extremely rare cause of facial paralysis and thus not the best answer.

A 20-year-old woman with no significant past medical history presented in the winter at 30 weeks gestation with cough, fever, nausea and vomiting, and mild right upper quadrant pain. Symptoms had been progressing over one week. She acknowledged several sexual partners, lived in the mid-Atlantic region of the United States, had not traveled outside this area since becoming pregnant. She has a 4-year-old child who was recently ill with fever and cold sores. Examination revealed: T of 40.7 C and BP 110/70; she was anicteric. There were several < 1cm supra-clavicular lymph nodes. Abdominal exam was mildly tender diffusely and her uterus consistent with her gestational stage. Labs: WBC = 5.400 (84% PMN); Hb 12.8 g/dl; platelets 120,000/mm3 AST = 1130 IU/L; ALT 720 IU/L; CPK 430; Alk Phos 99 Hepatitis A IgM, and Hepatitis C antibody negative Hepatitis B surface and core antibody positive, PCR negative HSV-1 IgM positive: Serum HSV PCR positive What is the most likely diagnosis? Acute hepatitis B HELLP syndrome Hyperemesis gravidarum HSV hepatitis Acute fatty liver of pregnancy

Correct answer: HSV hepatitis In this case, HSV was diagnosed by PCR and thus is the correct answer. HSV-1 and HSV-2 hepatitis is a rare but a rapidly progressive and frequently fatal infection in the third trimester of pregnancy in previously healthy women. Either HSV1 or HSV2 may be the cause but a prior history of HSV is usually not obtained. Mucocutaneous lesions are present in less than half the cases. HSV hepatitis also occurs in immunosuppressed patients. Coagulopathy, thrombocytopenia, seizures or coma may occur. Intravenous acyclovir is the drug of choice but liver transplantation may be required. Acute hepatitis B is not likely in someone who is at the time of presentation HBs antibody positive, and was likely immunized as a child. Hyperemesis gravidarum can cause nausea and hepatitis during pregnancy but typically occurs during the first trimester. Acute fatty liver of pregnancy can occur late term and present with hepatitis. However, there is typically jaundice and encephalopathy as well as leukocytosis and a sepsis like presentation. Aside from elevated liver enzymes, in this case there are few of the cardinal findings of HELLP such as platelets < 100,000/mm3 or hemolysis. While gonorrhea is not offered as an answer in this case, one might consider: perihepatitis (Fitzhugh Curtis Syndrome). However, this diagnosis would not fit well because in perihepatitis the transaminases are either normal or elevated to a lesser degree than in this case. Also, symptoms of pelvic inflammatory disease are often present.

A 30-year-old female lawyer, a native of Trinidad, is referred to you for recurrent strongyloidiasis. She has been well all her life until about two years ago when she developed mild ataxia. She has had progressive weakness of her legs with some paresthesias. She is receiving vitamin B12 for possible pernicious anemia. Her strongyloidiasis has been confirmed on six different occasions. She has received courses of albendazole and ivermectin. The patient lives in Manhattan and has no exposure that you can discern since she emigrated from Trinidad. She works 70 hours per week and rarely leaves Manhattan. Which of the following would be most useful for establishing the cause of her recurrent current disease? CD4 count HIV serology Pharmacokinetic studies to document absorption of albendazole or ivermectin HTLV-I antibody Serum IgA level

Correct answer: HTLV-I antibody This is a case of HTLV-1 associated spastic paraparesis. Spastic paraparesis occurs in about 0.3% of HTLV-1 infected individuals. HTLV-1 occurs in lifelong residents of the US, but is more common in some areas of the Caribbean and Japan. Although strongyloidiasis was in the original CDC AIDS definition, in fact it is probably not a true HIV associated opportunistic infection. However, it is clearly more common among patients with HTLV-1 where it presents as persistent and recurrent disease. HTLV-1 is not associated with low CD4 counts. CD4 counts are not really useful in any population except for patients with known HIV infection: these counts are not sensitive and/or specific enough in other populations. Serum levels of IgA would not be useful.

A 25-year-old graduate student is seen for penile ulcers. The ulcers have been present for two weeks and are painful. He says the ulcers began as "red bumps" that developed into "pimples" and then eroded into ulcers. Over the past 2-3 days he noted a tender lump in his groin. The problem began during a trip to Africa from which he returned just three days ago. While in Africa he had vaginal intercourse with several commercial sex workers while he was inebriated and did not always use a condom. On exam he is afebrile and findings are confined to his genital area. There are two 1cm adjacent "kissing" ulcers in the coronal sulcus. They are tender and filled with a yellow purulent exudate. There is a tender, large lymph node in the left groin. Which one of the following is the most likely cause of his problem? Treponema pallidumHerpes simplexChlamydia trachomatisHaemophilus ducreyiKlebsiella (Calymmatobacterium) granulomatis

Correct answer: Haemophilus ducreyi For the boards, painful ulcers are going to be due to HSV or chancroid, while painless ulcers associated will be syphilis, LGV, or granuloma inguinale. However, a clinical diagnosis made solely by clinical criteria may be misleading-lab confirmation should be performed. PCR has been reported to be more sensitive than culture. The classic presentation of chancroid due to Haemophilus ducreyiis a deep painful ulcer with purulent exudate; painful inguinal lymphadenitis is common. The infection is rare in the United States but is endemic in sub-Saharan Africa. H. ducreyihas been increasingly recognized as a cause of nongenital skin ulcers in tropical areas, including in children. Primary syphilis due to Treponema pallidumproduces a painless ulcer (chancre). Herpes simplexproduces a cluster of painful vesicles, typically on the penile shaft, which quickly ulcerate. Tender inguinal lymphadenopathy Is atypical. Lymphogranuloma venereum due to Chlamydia trachomatispresents as inguinal adenopathy. An initial genital ulcer may be noted but typically heals within a few days followed by adenopathy which is usually unilateral. Granuloma inguinale or donovanosis due to Klebsiella granulomatispresents as a painless, slowly progressing ulcer without regional adenopathy.

A 22-year-old female has had frequent episodes of lower urinary tract infections. She has frequent intercourse with a single partner, who always uses condoms with spermicide. Which one of the following actions would be most likely to decrease the frequency of her infections? Have her partner discontinue spermicide use with condoms Vaginal douching after intercourse Discontinue wearing of pantyhose Urinate after intercourse Drink cranberry juice for prophylaxis

Correct answer: Have her partner discontinue spermicide use with condoms Spermicide use, whether with condoms or diaphragms, has been shown to increase the risk of UTIs from E. coli and Staphylococcus saprophyticus 2-3 fold. Cranberry juice has been shown to be ineffective in preventing recurrent UTIs in a recent well-controlled randomized trial. Pantyhose, urinating after intercouse, douching, and wiping patterns have been shown to have nothing to do with frequency of UTIs.

A 42-year-old asymptomatic male has the following laboratory studies: AntiHBc positive HBsAg positive ALT, AST, Alkaline Phos: normal Serum HBV DNA negative He is diagnosed with lymphoma and is about to get CHOP chemotherapy plus rituximab. You are asked to consult regarding whether HBV relapse is possible and what should be done. Which of the following is the best approach to his HBV status: He is at no risk for HBV reactivation and needs no HBV monitoring He is at no risk for HBV reactivation but should receive HBV immunization with three injections of double standard dose HBV vaccine. He is at risk for HBV reactivation, started on lamivudine and monitored with serial HBV DNA and LFT testing He is at risk for HBV reactivation and should receive HBIG (HBV immune globulin), then monitored with serial HBV DNA and LFT testing and treated if evidence of reactivation is documented. He is at risk for HBV reactivation and should be started on entecavir or tenofovir and monitored with serial HBV DNA and LFT testing.

Correct answer: He is at risk for HBV reactivation and should be started on entecavir plus tenofovir and monitored with serial HBV DNA and LFT testing. Observational studies indicate that patients with HBV who receive chemotherapy for B cell lymphoma with CHOP plus rituximab (CHOP-R) are at high risk for developing exacerbation of hepatitis B, including those who have chronic HBV, those who are in the inactive HBsAg carrier phase, or even those with only anti-HBc. Rituximab is one of the medications most strongly associated with HBV relapse (Table below , from Perilllo http://onlinelibrary.wiley.com/doi/10.1002/cld.448/full). A variety of other immunosuppressive conditions seem to predispose to HBV reactivation, particularly allogeneic stem cell transplant recipients. Because he is HBsAg positive, there is a clear indication for preventative treatment with entecavir or tenofovir (or TAF) during immunosuppression. Lamiviudine is not preferred because of failure due to development of drug resistance.Those who are HBsAg positive have a higher risk of reactivation than those who are HBsAg negative. However, even those who are anti-HBc positive can relapse. The options in that instance are to monitor ALT and HBV DNA levels regularly and treat if increasing/detectable or to treat empirically with entecavir plus tenofovir. There are some differences in expert guidelines on which is best. The following table is one expert's approach to the risk for HBV reactivation:

A 48-year-old with rheumatoid arthritis on TNF- alpha inhibitors presents in the Fall of 2017 for routine follow-up. He states he NEVER gets the influenza vaccine because he develops severe hives if he eats eggs and is immunosuppressed. On further questioning he states he can eat baked goods cooked with eggs and has no allergic sequelae. What would you advise this patient about influenza vaccination: He should be given the Live Attenuated Influenza VaccineHe may safely receive Inactivated trivalent or quadrivalent Influenza VaccineHe should not receive any influenza vaccine due to his egg allergyThe only safe option is to receive Flucelvax (ccIIV), the mammalian Cell Culture Inactivated Influenza Vaccine or Flublok (rIIV), the Recombinant Influenza Vaccine

Correct answer: He may safely receive Inactivated trivalent or quadrivalent Influenza Vaccine Incorrect. The live influenza virus vaccine ys not recommended by the ACIP for 2016-17 influenza season In those who can eat eggs but develop only hives the ACIP has deemed IIVs safe to use. He could also be given the vaccine with virus raised in cell culture or recombinant hemagglutinin protein from the three viruses grown in cell culture. Since he is immunosuppressed it is even more important that he receive annual influenza immunization.

A 25-year-old male from Saudi Arabia, in the United States for 3 years, is admitted to the ICU with pulmonary edema. He had been well for the past several years - going to school and working part time in a child care center. In Saudi Arabia he had several episodes of chest pain and fever, or joint swelling and fever between the ages of 5 and 16 years old. He has severe mitral stenosis, is managed medically, and discharged. For long-term management as an outpatient, in addition to chemoprophylaxis for endocarditis, what would you recommend regarding prophylaxis against rheumatic fever? No prophylaxis to prevent rheumatic fever is necessary at his age He should receive intramuscular benzathine penicillin monthly for 12 months He should receive intramuscular benzathine penicillin monthly for 24 months He should receive intramuscular benzathine penicillin monthly until he is at least 40 years old He should receive intramuscular benzathine penicillin until his ASO titers have been normal for at least 1 year

Correct answer: He should receive intramuscular benzathine penicillin monthly until he is at least 40 years old This patient probably had episodes of acute rheumatic fever that caused his arthralgias and chest pain. His antistreptolysin titer is likely to be over 200 U for at least 2 months following a streptococcal infection: this is helpful diagnostically for determining if he has had streptococcal disease (and such disease obviously can be followed by episodes of rheumatic fever), but is not helpful for determining the interval when chemoprophylaxis is useful. The risk of a recurrence declines with age and with time since last occurrence of rheumatic fever. This patient is probably at considerable risk because, although he is no longer a child, he has had recent episodes of acute rheumatic fever, and has valvular heart disease. Chemoprophylaxis, consisting of monthly benzathine penicillin, or daily oral penicillin VK or sulfadiazine, should be continued to prevent rheumatic fever, as follows, according to the American Heart Association: Rheumatic Fever without carditis, 5 years since most recent episode of rheumatic fever or age 21 years, whichever is longer Rheumatic Fever with carditis, but no valvular damage, 10 years since most recent episode, or well into adulthood, whichever is longer Rheumatic Fever with residual heart disease, at least 10 years since last episode and until at least age 40 years This patient is at special risk for streptococcal disease, incidentally, since he works with children. Thus, he would be a poor candidate for a short course of chemoprophylaxis. Keep in mind, there are two kinds of prophylaxis: prophylaxis against endocarditis, and prophylaxis against rheumatic fever.

A 29-year-old female day care worker from Providence, Rhode Island, whose family refused all immunizations, leaving her unimmunized as an adult, presents in August with malaise, loss of appetite, mild fever, and jaundice. She recently completed a 10-day camping trip in Northern Minnesota during which time she had diarrhea for 2 days. She did not notice ticks on her body. Her boyfriend accompanied her on the trip but did not have a similar illness. She is taking no medications. Her job includes care for many infants and toddlers in a low cost unlicensed program with many migrant workers. On examination: T=100, P-88, R-20, BP-102/66. Sclera are icteric and there is right upper quadrant tenderness with abdominal discomfort and hepatomegaly. No rash is detected except a few papules and small nodules with surrounding erythema that the woman attributes to bug bites. Labs: HGB 11.6, Platelets 240,000, WBC 3.4 (45% L, 44% PMN, 6% Monos, 5% Eos) Electrolytes and BUN/Creatinine - normal ALT-1235, AST-677, Total Bilirubin 13.5, Alkaline Phosphatase 213 CPK 122, Amylase 89 Chest radiograph: normal Which of the following is the most likely cause of her illness? Anaplasma phagocytophilumBabesia microtiEhrlichia chaffeensisLeptospira icterohaemorrhagiaeHepatitis A virus

Correct answer: Hepatitis A virus The woman is at some risk for all these infections, and all can cause hepatic inflammation. This case is typical for hepatitis A: ALT>AST, transaminases over 1,000 IU/l and total bilirubin over 10. The diagnosis would be supported by IgM and IgG serologies. Babesiosis, anaplasmosis and ehrlichiosis often cause elevated transaminases but not this high. Thrombocytopenia is usual in these three diseases, but this patient did not have thrombocytopenia. For leptospirosis, disease characteristically causes other symptoms and mild liver enzyme elevations, compared to a high bilirubin and alkaline phosphatase. Day care is one of the leading risk factors for HAV infection. Her immigrant clientele could be expected to have a low rate of immunization, and to have a higher than normal risk of importing hepatitis A from outside the United States. Children with HAV are often asymptomatic.

An HIV-infected patient (CD4 = 80 cells), not currently on medications, presents with the following lesions on his feet. These have developed over several weeks. They are very itchy. The dermatologist does not know any of his additional history. He otherwise feels well. The dermatologist leaves a note that this is lichen planus. Which of the following is associated most characteristically with lichen planus: CMVHHV 6Hepatitis CTreponema pallidumHistoplasma capsulatum

Correct answer: Hepatitis C Lichen planus can be associated with severe liver disease, especially hepatitis C liver disease. Lesions often arise during interferon therapy, and will regress when interferon is stopped. However, the lesions are not necessarily interferon related. The syndrome also can be caused by a variety of drugs including quinine and quinidine. Lichen planus progresses insidiously, and can be localized or generalized, involving skin, mucosal surfaces, or genitalia. Lesions can be very pruritic. The disease appears to be autoimmune, directed against keratinocytes. Lesions can be treated with topical steroids or occasionally oral corticosteroids.

A 37-year-old woman is seen for fever and a generalized rash. She had a long history of atopic dermatitis but was otherwise well. She has had a new sex partner for the past month. Four days ago she developed a painful rash starting symmetrically on her neck and chest that quickly spread to the rest of her body, sparing her palms, soles and genitalia. Two days ago, she went to an emergency room at another hospital where she was thought to be having an exacerbation of her atopic dermatitis and was sent home on prednisone and hydroxyzine. The rash worsened and her temperature rose to 103°F. On exam, she is febrile and clearly in pain with a diffuse erythematous eruption characterized by numerous vesicles in different stages of development. Her pain was increased with any movement or touching of her skin. Which one of the following is the most likely cause of her rash? Staphylococcus aureus Herpes simplex Variola Varicella Treponema

Correct answer: Herpes simplex The key parts of this stem are the painful vesicular rash starting symmetrically on the neck and chest and the atopic dermatitis. The painful rash should make one think first of Herpes simplex or perhaps disseminated varicella zoster, or a bacterial superinfection. She likely has been vaccinated against varicella, but she could of course develop disseminated zoster. However, for this patient with atopic dermatitis, HSV is the most likely cause. Cutaneous pain and a vesicular eruption in a patient with preexisting atopic dermatitis should immediately bring to mind eczema herpeticum (also known as Kaposi's varicelliform eruption) usually caused by herpes simplex virus type 1. Staphylococcus aureus may cause a bullous impetigo, but the lesions are not monomorphic as in this patient and they are not painful. Variola (smallpox) produced a vesicular eruption with lesions all in the same stage of development, but there has been no naturally acquired smallpox since 1977. Varicella is characterized by lesions in different stages of development, but does not spread into areas of eczema and pain is unusual. Generalized rash is characteristic of secondary syphilis, but the palms and soles are typically involved and vesicles do not occur in adults.

This 35-year-old Peruvian woman with chronic myelogenous leukemia in blast crisis was admitted for allogeneic hematopoietic stem cell transplantation from her sister. On admission, this extensive painful rash was found on the gluteal area of both buttocks. She is chronically ill but does not appear to have an acute deterioration, i.e. she is not septic and has no new major organ dysfunction other than her blast crisis. She complained that the area had been painful for several days. She was afebrile but normotensive and not acutely ill; she was markedly granulocytopenic from prior chemotherapy. The most likely diagnosis is which of the following: Herpes zosterHerpes simplexEcthyma gangrenosaAspergillosisSweet's syndrome

Correct answer: Herpes simplex This lesion is due to Herpes simplex virus. Herpes simplex, unlike herpes zoster, can spread by local inoculation and present, as in this case, on both sides of the midline. Skin that is moist, such as intertriginous areas, are particularly susceptible to inoculation. This patient was febrile, sweating, and lying long periods on her back. In the process she inoculated herpes simplex from an apparent lesion, probably genital, into the skin of her back. This lesion does not resemble ecthyma gangrenosa, an infection usually due to hematogenous dissemination to one or several sites of Pseudomonas aeruginosa, Aspergillus or an agent of mucormycosis. These lesions are initially red, then purple and finally necrotic in the center from invasion of dermal blood vessels. Ecthyma gangrenosa can be hematogenous or locally inoculated in granulocytopenic patients. With hematogenous lesions the patients are critically ill. With local inoculation, the patient is often not septic. Sweet's syndrome would not likely be this large and would not likely be on the buttock. Sweet's syndrome is characterized by abrupt onset, characteristic pathology, and may be associated with fever and leukocytosis. The lesions tend to be smaller, and more likely on the face or hands. They are painful. This syndrome is associated with a long list of infectious and autoimmune and malignant diseases.

You are consulted about three rugby players from the same team who have skin lesions. The skin lesions have been present for two to three days. Each player has 10 to 20 raised, clustered lesions on the face, neck, and arms that are about 2 to 5 mm in diameter and filled with a clear yellow fluid; there is a small ring of erythema around the base of each lesion. The athletes say the lesions are mildly uncomfortable but not pruritic; they are very minimally tender. Three days before the lesions were noted by the first athlete, they had engaged in a rugby match after which they attended a party and bathed in a hot tub. Which one of the following is the most likely cause of the skin lesions? PseudomonasMycobacteriumHerpes simplexContact dermatitisMolluscum

Correct answer: Herpes simplex Uncomfortable vesicular lesions suggest a herpes virus. However, the cluster of lesions is the key to the clinical presentation. So-called "herpes gladiatorum" refers to outbreaks of herpes simplex skin lesions seen in wrestlers and rugby players who have prolonged traumatic skin-to-skin contact. Lesions are most common on the head, neck and arms. Pseudomonas folliculitis causes pruritic lesions in areas covered by bathing suits (also known as "hot tub buns," which are pruritic but not painful). Mycobacterial infectious can occur associated with hot tubs but the lesions are not vesicular, cases are sporadic, and the incubation period is longer than seen here. Contact dermatitis may produce small vesicular lesions but they are pruritic. Molluscum lesions are not vesicular and not readily transmitted person to person. MRSA can spread among football players and wrestlers but would not cause clusters of vesicular lesions.

A 38-year-old man with a history of inflammatory bowel disease diagnosed during his teenage years has been treated with an anti-TNF agent for the past two months. He is referred to you by his gastroenterologist because of a new rash on his palms and soles. The rash is characterized by multiple papulopustular red lesions that do not itch or hurt. He feels well and has no fever. He has never had anything like this before. Other than the rash his exam is completely normal. Which one of the following is the most likely cause of his rash? His inflammatory bowel disease His anti-TNF medication Herpes simplex Treponema pallidum Parvovirus

Correct answer: His anti-TNF medication This rash is suggestive of palmoplantar pustulosis. A recent report (Ann Intern Med 2016;164:10-22.) showed that as many 30% of patients on anti-TNF agents for IBD developed skin lesions. Lesions included eczema, psoriasiform eczema, xerosis cutis, and palmoplantar pustulosis. Inflammatory bowel disease may be associated with a variety of rashes, most typically pyoderma gangrenosum, but palmoplantar pustulosis is not associated with IBD. The lesions of herpes simplex would be unlikely to cause this particular distribution and typically are associated with pain. Secondary syphilis may cause lesion on the palms and soles; these are typically associated with other signs or symptoms like fever and adenopathy, and the lesions are papulosquamous and not pustular. Parvovirus rash in children is typically the red face; in adults, rash is rare and is associated with arthropathy.

A 23-year-old man recently returned from a camping trip in Mexico. He presented to the emergency department suffering from abdominal pain, nausea, fever, and bloody diarrhea. Stool specimens were collected and submitted to the laboratory for bacterial cultures and ova and parasite examination. Bacterial cultures were negative for enteric pathogens but the ova and parasite examination revealed the organism seen in the figure. On the left is an iodine stain and on the right is a Giema stain showing ingested erythrocytes. Which of the following is the primary reservoir host for this parasite? CockroachesDogsFliesHumansMosquitoes

Correct answer: Humans The parasite observed in the figures is the trophozoite form of Entamoeba histolytica, the etiologic agent of amebiasis. The first figure is an iodine stain; the second is a giemsa stain that shows phagocytosis of erythrocytes. To be certain these were not other pathogenic or nonpathogenic species of Entamoeba, PCR or species specific antigen testing would be required Characteristically, the trophozoites are motile and vary in average size from 15 to 30 µm. The single nucleus is round with a central karyosome and an even distribution of chromatin granules around the nuclear membrane. Ingested erythrocytes may be in the cytoplasm. The cysts are smaller (15 to 20 µm) and contain one to four nuclei (usually four). Round chromatoidal bars may be in the cytoplasm. Patients infected with E. histolytica pass noninfectious trophozoites and infectious cysts. The infection is spread when cysts in stool are ingested in contaminated food or water. Therefore, the main source of water and food contamination is the asymptomatic carrier who passes cysts.

A 56-year-old alcoholic woman with adult onset diabetes mellitus and depression fractured her right hip falling down the stairs. She had an open reduction and internal fixation repair one week ago, but developed redness and drainage of the wound secondary to a vancomycin resistant E. faecium (VRE) infection of the operative site. She was started on linezolid. You are called to see her on day 10 of hospitalization (day 8 of linezolid) because of anxiety, tremulousness, fever to 39.6°C, agitation and confusion. The patient takes metformin (Glucophage) and glipizide for her diabetes mellitus and citalopram (Celexa) for her depression. Which one of the following is most likely? Delirium tremors IgE mediated allergic reaction IgG mediated allergic reaction Hyper-serotonin syndrome Malignant hyperthermia

Correct answer: Hyper-serotonin syndrome This patient most likely has a serotonin excess syndrome due to the combined effects of linezolid and citalopram SSRI. Linezolid is a weak monoamine oxidase inhibitor (MAOI) and can rarely interact with selective serotonin reuptake inhibitors (SSRI's) such as citalopram causing a hyper-serotonin syndrome. You should be aware of this interaction since intervention can be life saving, and since this is easy to test on the board examination. This is too late to be delirium tremens assuming that she has not been drinking in the hospital: delirium tremens occurs within 2-4 days of alcohol cessation and while it may last for 5-7 days, it would not start on the tenth day of hospitalization. Malignant hyperthermia is usually related to inhalational anesthetics (halothane) or succinycholine and occurs in the operating room or recovery room in most instances. This could be neuroleptic malignant syndrome that would be related to a neuroleptic (citralpram) but which would have nothing to do with linezolid. This was not offered as a choice.

A 35-year-old male returns from a 2-day visit to rural Haiti where he was not cautious about food or water. He has voluminous diarrhea, nausea, but no fever, blood in stool, or abdominal cramp. Based on this gram stain of the organism, which of the following complications is most likely to occur. Renal failureThrombocytopeniaHypoglycemiaHepatic failureGastric ulcer

Correct answer: HypoglycemiaThis curved gram negative rod should suggest cholera in a patient with a classic syndrome, i.e. watery diarrhea, very recent travel to an endemic area (incubation period hours to 5 days), minimal fever or cramps. Hypoglycemia is a common complication of severe cholera. To confirm the diagnosis the colony could be cultured on a selective medium such as TCBS. This medium is excellent for identifying vibrios in general, which can then be specifically identified to the species level by other techniques. Therapy should focus on adequate rehydration and electrolyte repletion. Most strains are still sensitive to tetracyclines: one dose of doxycycline should suffice, but in some areas fluoroquinolones or macrolides may be necessary. Watery diarrhea could be caused by other gram negative rods, although they would generally not be curved. E. coli O157:H7 and related strains would be associated with hemolytic uremic syndrome with or without thrombocytopenia (HUS-TTP) but the clinical syndrome and gram stain are more typical of cholera. Gastric ulcer is a distractor: Helicobacter is not typically seen microscopically in stool, and is spiral shaped. Helicobacter in a gastric sample is associated with inflammation (peptic ulcer disease) and atrophy (gastric carcinoma or lymphoma). Correct Response

A 45-year-old, U.S.-born, Caucasian civil engineer returns to Washington, D.C. after a one-week trip to visit a hydroelectric project in Nigeria. He did not see any point in taking any type of malaria prophylaxis for such a short trip. The patient presented to the emergency room with a 2-day history of fever, and was now short of breath and confused. A peripheral blood smear showed Plasmodium falciparum: about 14% of his erythrocytes were infected. The patient was transferred to the ICU and IV quinidine begun while artesunate was being obtained from CDC. In the ICU the patient was found to have a serum creatinine of 2 mg/dL. The patient had a serum lactate of 3 mg/dL, hemoglobin of 12 g/dL and platelet count of 30,000/mm3. Four hours later, the patient was more confused, and was now hypotensive. On examination the patient was moderately obtunded, and still febrile to 38.7°C. A chest x-ray showed diffuse infiltrates consistent with capillary leak, atelectasis, or fluid overload. Parasitemia was 7%; hemoglobin 11 g/dL, and platelet count 100,000 /mm3. Assuming that his obtundation is related to his malaria, at this point, which of the following would be the first treatment to be given while awaiting stat labs? Prednisone NaloxoneIV Glucose Calcium Malarone

Correct answer: IV Glucose Patients with high levels of parasitemia due to falciparum malaria may get a variety of complications, including cerebral edema, pulmonary edema, hypoglycemia, severe hemolytic anemia, renal failure, thrombocytopenia and bacterial infection. The first thing to give this patient with the unexplained onset of obtundation is IV glucose. Hypoglycemia can complicate malaria or quinine therapy, and should be considered if the patient is confused, pending a rapid glucose determination . Antibacterial treatment could also be considered. A substantial number of patients (probably 25%) develop community-acquired or nosocomial bacterial infections: falciparum malaria predisposes to such infections. Thus, when patients with falciparum malaria develop shock, the clinician might consider sepsis due to a community-acquired or nosocomial infection. Prednisone has not been found to be helpful in the therapy of severe malaria: it may actually be harmful. Adding another antimalarial agent to quinine rarely is useful in the acute stage of malaria. Malarone is only available for oral use. Complicated falciparum malaria should be treated parenterally, as is this patient. Parasitemia has already fallen, indicating a therapeutic response to IV quinidine. There are many causes of obtundation in addition to hypoglycemia: cerebral infarct, cerebral bleed, bacterial meningitis, and acute electrolyte abnormalities are all considerations in addition to hypoglycemia. In this case naloxone is not indicated since there is no mention of the patient receiving narcotics. Correct Response IV Glucose

A 69-year-old female had malaise and fatigue for two to three months, and right upper quadrant pain for several weeks. When she developed fever, chills, and nausea, she came to the Emergency Department. Four months prior, she had traveled outside of the United States for the first time in her life to southern Africa for a game park tour, but had not been ill. She had complained of four to five non-bloody stools for several weeks prior to coming to the Emergency Department but that resolved. Physical Exam: T-102°F; BP 150/85; mild right upper quadrant tenderness without rebound Labs: WBC 16,000 (63% polys, 29% bands, 1% metamyelocyte); Hct 32%; T. Bili 1.7, ALT 82 U, AST 93 U Stool: negative for parasites, enteric pathogens CXR: small left effusion; CT shows a septate 15 cm diameter cystic lesion in the liver Which of the following is true? The CT scan can distinguish between an amebic and a bacterial liver abscess If the stool is negative for ameba, an amebic abscess is very unlikely If the amebic serology is positive, metronidazole therapy should be initiated to treat the liver lesion A percutaneous aspirate is contraindicated in this patient Absence of an abdominal focus of infection makes bacterial abscess unlikely

Correct answer: If the amebic serology is positive, metronidazole therapy should be initiated to treat the liver lesion This patient could have an amebic or a bacterial liver abscess. This patient could have a bacterial liver abscess, though the history and clinical findings, and the CAT scan, cannot distinguish between a bacterial and an amebic liver abscess. The abdominal source of a pyogenic liver abscess is often occult. Some pyogenic liver abscesses are also the result of hematogenous spread with an intraluminal agent. Since she is native to the United States, a positive amebic serology would be so suggestive that this is an amebic abscess and that therapy with metronidazole should be started. If there are ameba in the stool, the cysts must also be treated. The serology may remain positive for life. In Mexico, for example, 5-8% of the population is positive but have no active disease A large fraction of patients with amebic liver abscesses have no history of diarrhea, and no ameba detectable in their stool. A percutaneous aspirate and placement of a percutaneous drain is indicated here, both to establish the microbial agent responsible and to provide essential drainage whether this large abscess is bacterial or amebic. If the amebic serology were known to be positive, aspiration would not be indicated and unlikely to demonstrate trophozooites. If there was history consisted with exposure to echinococcosis, caution would be advised to prevent spread of scolices into the abdomen cavity during aspiration.

A 43-year-old woman residing in the District of Columbia was bitten on the hand by a feral cat that had taken up residence near her home. The bite occurred 7 hours ago, when the woman tried to feed the cat from her hand. The site has several puncture wounds and some edema and tenderness but no exudate. It did not appear to have entered a joint space. The patient was previously healthy. The woman's two children (ages 4 and 7 years) petted the cat but were not licked, scratched, or bitten. Which of the following should be recommended? If the cat can be monitored for 10 days and shows no signs of illness, rabies prophylaxis is unnecessary for anyone If the cat has escaped and cannot be found, rabies prophylaxis is unnecessary for anyone because rabies is so uncommon in this part of the US in cats If the cat has escaped and cannot be found, rabies immune globulin should be administered in one buttock, and rabies vaccine should be administered in the other buttock to the mother If the cat has escaped and cannot be found, rabies immune globulin should be administered in one buttock and the rabies vaccine should be administered in the other buttock to both children and the mother If the cat has escaped and cannot be found, rabies immune globulin is not necessary, but rabies vaccine should be administered

Correct answer: If the cat can be monitored for 10 days and shows no signs of illness, rabies prophylaxis is unnecessary for anyone Dogs and cats uniformly begin to become ill and die within 10 days (usually five-seven days) of the time the virus spreads from the animal's CNS to its salivary glands. For this reason, a healthy domestic dog, cat, or ferret that bites a person should be observed for 10 days. Animals that exhibit any sign of illness should be evaluated by a veterinarian and reported to the local health department. If the animal manifests signs of illness, it should be euthanized and its head shipped to an appropriate lab. If the animal remains healthy for the full 10 days, it did not have rabies virus in its saliva at the exposure time. Because this may be considered to be a provoked bite, it is unclear whether the animal is likely to be rabid. If the animal cannot be observed reliably, then the safest option is to give immune globulin as much as possible into the bitten hand, with the remainder in the buttock. A course of rabies vaccination should also be initiated, giving the initial injection in a site different from the immune globulin and repeated on days 3, 7, and 14. A fifth dose is no longer recommended. Rabies is known to exist in feral animals in the Mid-Atlantic area and the cat could have been bitten by a rabid animal. Cats are not often infected in the United States, but the potential exists. Prevalence of rabies in the area is not used to decide about prophylaxis. The children do not require prophylaxis because they were not bitten or exposed to the cat's saliva.

A 63-year-old man with HIV infection has received many different antiretroviral agents, with poor response to each related to poor adherence. He is currently on raltegravir, tenofovir and emtricitabine. He is currently on raltegravir, tenofovir (TDF) and emtricitabine with an HIV RNA level of 56,000 copies/ml. RT: M41L, L210W, T215Y, Y181C, Y188C Pro: I47A, I50L, V82A, L90M Which is the best answer regarding his potential to have developed drug resistance to raltegravir? Raltegravir resistance is linked to these protease genotypic mutations. Raltegravir resistance is linked to these reverse transcriptase genotypic mutations. The emerging virus is not likely to be raltegravir-resistant. The emerging virus is likely to have two or more raltegravir-resistance mutations.

Correct answer: If this patient fails a regimen containing raltegravir, there are likely to be two or more raltegravir resistance mutations. This patient had only RT- and protease-associated resistance mutation results; a genotype for integrase inhibitor-associated resistance mutations should also have been requested given that he experienced virologic failure on an integrase inhibitor-based regimen. There are three paths to raltegravir resistance (substitutions in Y143, Q148 and N155), when one of these is present, there is always at least one other integrase inhibitor substitution. Resistance mutations involving RTIs, NNRTIs or PIs do not predict raltegravir resistance, so that the first and second choices are incorrect. When patients fail raltegravir containing regimens, they often have raltegravir resistance with at least 2 mutations. Of note, dolutegravir retains susceptibility against the Y143 and N155 resistant strains, but loses activity with the Q148 substitution.

This is the modified acid fast smear of sputum from a 34-year-old woman with pneumonia. She had systemic lupus erythematosus and was receiving prednisone 60 mg daily. When she had been given trimethoprim- sulfamethoxazole last year, she had developed severe Stevens Johnson syndrome. Which of the following would you recommend? Imipenem plus amikacinIntravenous azithromycinClarithromycin and ethambutolCeftriaxoneIsoniazid plus rifampin plus pyrazinamide

Correct answer: Imipenem plus amikacin Presence of branching modified acid fast bacilli indicates nocardiosis, one of the causes of pneumonia in immunosuppressed patients. Trimethoprim-sulfamethoxazole is the preferred drug but reports of imipenem, with or without amikacin, have indicated efficacy. Azithromycin does not appear to be effective. There are a few reports of success with ceftriaxone but this may depend on the Nocardia species. Susceptibility testing is not clearly reliable and species identification is often not available at time of diagnosis, making ceftriaxone not recommended.

An injection drug user is admitted with fever for four days. Exam shows a grade IV aortic insufficiency murmur, and he is started on vancomycin 1gm q12h after three blood cultures are obtained. Methicillin-resistant Staphylococcus aureus is grown from all admission blood cultures, and repeat blood cultures on days two and three also grow MRSA. The vancomycin M.I.C. for his MRSA is 1.5. On day four of treatment he is short of breath, he has diffuse crackles on chest exam, and x-ray of the chest shows acute pulmonary edema. Which one of the following is the most important next step in management of this patient? Immediate aortic valve replacementValve replacement once blood cultures are negativeChange vancomycin to daptomycinCheck a vancomycin levelAdd rifampin

Correct answer: Immediate aortic valve replacement Severe heart failure in endocarditis is an immediate indication for surgery, even in patients who are still bacteremic. Delay in surgery increases mortality and infection risk for the new valve is small. Changes to antimicrobial therapy, even if potentially useful, would not supersede the need for surgery which may be life-saving. ACC/AHA Guideline-Surgery for Native Valve Endocarditis Strong Indication Valve stenosis or regurgitation leading to heart failure. Aortic or mitral regurgitation with hemodynamic evidence of elevated left ventricular end-diastolic or atrial pressures such as premature closure of the mitral valve with aortic regurgitation, rapid decelerating mitral regurgitation signal by continuous wave Doppler (v-wave cutoff sign), or moderate to severe pulmonary hypertension. Endocarditis due to fungal or other highly resistant organisms. Complications such as heart block, annular or aortic abscess, or destructive penetrating lesions such as fistula from the sinus of Valsalva to the right or left atrium or right ventricle, mitral leaflet perforation with IE of the aortic valve, or infection in annulus fibrosis. Less Compelling Indication Recurrent emboli and persistent vegetations despite appropriate antibiotic therapy. Possible Indication Mobile vegetations larger than 10 mm with or without emboli.

A 63-year-old male from Pennsylvania spent a month in Haiti, helping build a church. One day before returning from Haiti to the United States, he developed a sore throat. Upon his return, he stopped at an ER where he had a negative rapid test for Group A strep. Four days later (5 days after onset of illness), he returned to the emergency department with chills, fever, nausea, vomiting, stridor, aspirating fluid into his nose when he tried to drink and a gray adherent membrane over the anterior tonsillar pillars, tonsils and extending across the posterior soft palate and uvula. The patient had been fully immunized as a child and during his service in Vietnam in the 1970s but not subsequently. Which management would you recommend? Empirical therapy with valacyclovir pending results of viral cultures Piperacillin-tazobactam Immediate use of diphtheria antitoxin plus erythromycin ENT consultation for possible drainage of a peritonsillar abscess Erythromycin and a booster injection of DPT

Correct answer: Immediate use of diphtheria antitoxin plus erythromycin The adherent membranes and their location should suggest diphtheria and are not consistent with Streptococcal pharyngitis, peritonsillar abscess, Arcanobacterium haemolyticum infection or Lemierre's syndrome. Toxin-induced pharyngeal weakness leading to aspiration also suggests diphtheria. This patient has an illness compatible with diphtheria and he has been to a region where diphtheria is prevalent. He should be treated immediately with diphtheria antitoxin to prevent neuronal and cardiac toxicity. Erythromycin should be given, pending confirmation of the diagnosis. Special culture medium is needed to isolate Corynebacterium diphtheriae. Many adults in the United States are probably not protected against diphtheria if they have not been recently immunized. Testing of serum samples from participants in the Third National Health and Nutrition Examination Survey (1988-1994) indicated that the percentage of United States residents with protective levels (>0.1 IU/mL) of diphtheria antibodies decreased progressively with age, from 91% at ages 6-11 years to approximately 30% at ages 60-69 years. Routine use of Td (tetanus toxoid with low-dose diphtheria toxoid) or TdaP (with acellular pertussis) for patients with tetanus-prone wounds would be helpful in maintaining immunity against diphtheria in adults. In terms of prophylaxis, diphtheria-infected travelers returning to the United States with incubating or untreated disease can transmit C. diphtheriae to their close contacts. Antibiotic prophylaxis is recommended for close contacts after nasal and pharyngeal specimens for culture are obtained. Prophylaxis can be stopped if cultures are negative. Adolescent and adult contacts who have not received a dose of a diphtheria toxoid-containing vaccine during the preceding 5 years should be vaccinated.

A 37-year-old man with a history of intravenous drug abuse and HIV infection appeared in the emergency room with fever and pulmonary infiltrates. He is diagnosed with tuberculosis by smear and started on conventional 4 drug anti-tuberculosis therapy. Two weeks later, he was started by another physician on abacavir/lamivudine and double-dose dolutegravir. His CD4 was 60 cells/µL, and his viral load was 100,000 copies/µL, at the time ART was started. Eight weeks after starting ART (10 weeks after starting anti-TB therapy), he returns with new fever. Chest X-ray shows more extensive infiltrates, a new pleural effusion, and new mediastinal adenopathy. The sputum specimens are negative for AFB. Bronchoscopy shows no pneumocystis, fungus, or bacteria on direct smear, but the Gen probe remains positive for TB after one month of antituberculous therapy. The original culture has now been reported as positive for M. tuberculosis; susceptibility testing is pending but there is no rifampin or INH resistance by gene probe. This worsening clinical syndrome most likely represents: Drug-resistant tuberculosis Abacavir hypersensitivity syndrome BAL negative pneumocystis pneumonia Immune reconstitution syndrome A drug interaction between INH and abacavir

Correct answer: Immune reconstitution syndrome This question tests knowledge on both immune reconstitution syndrome and abacavir hypersensitivity reaction. Drug-resistant TB is possible based on the clinical presentation but the patient improved initially, and the smears have converted to negative. The Gen probe is expected to remain positive for weeks after the smears turn negative during successful therapy. Multidrug resistance would also be very unusual in someone who has a rifampin-sensitive strain which this patient appears to have by molecular probe. MDR and XDR by definition are rifampin-resistant but other combinations could theoretically occur with rifampin-sensitive strains, but this is most unlikely. However, when ART is initiated in close proximity to the initiation of anti-tuberculosis therapy for acute tuberculosis, immune reconstitution reactions are quite common (probably 30-40% when ARV is started within 2 months of anti-TB therapy). The timing of ART following a diagnosis of TB depends on the CD4 cell count: For <50, start as soon as possible, within 2 weeks; for >50, start within 8 weeks. This syndrome would be atypical for abacavir hypersensitivity syndrome, which typically occurs about one week after starting abacavir, most cases occur within 3 weeks, but can occur up to 2 months after the initiation of therapy. Typical cases have a triad of fever, constitutional manifestations, and gastrointestinal manifestations. Only about 30% have a rash, which is not usually an initial manifestation. Since pulmonary manifestations are not common, the patient above would not be likely to have abacavir hypersensitivity. The diagnosis is of this syndrome is clinical although HLA B 57 testing would provide supportive evidence. As a reminder, the use of rifampin necessitates doubling of the dolutegravir dose (to 50 mg twice daily) to overcome a significant drug-drug interaction.

This large banana-shaped organism is a malaria parasite in a blood smear. If you could find a vaccine that would specifically kill this stage, you would block which of the following events: Invasion of the liverGlomerulonephritisInfecting a mosquitoInvasion of RBCsCerebral Malaria

Correct answer: Infecting a mosquito This Plasmodium falciparum gametocyte is the form that infects mosquitos. A vaccine against this form would block transmission but not affect the course of malaria in the vaccinated patient.

This parasite, shown as an egg, was acquired from which source? Fresh human stoolIngesting contaminated earthEating poorly cooked porkEating poorly cooked beef

Correct answer: Ingesting contaminated earth Ascaris lumbricoides eggs need to mature in the earth before they are infectious. Infections typically are acquired when the egg is shed into the environment in human stool, and another human accidentally ingests the mature egg from dirt or raw produce, which then matures into a larva which penetrates the gut wall, circulates to the lung, ascends the tracheobronchial tree, is swallowed and matures in the GI tract as an adult worm. Humans may be symptomatic during migration through the lungs (eosinophilic pneumonia, Loeffler's syndrome). Worms in the GI tract usually produce few symptoms in adults, although occasional cases of mild abdominal discomfort or appendicitis occur. Intestinal blockade occurs primarily in children. If the ascaris ingested is from a dog or cat, the organism penetrates the intestine after ingestion and enters the blood stream but cannot complete its life cycle in the organ where it lodges. This produces visceral larva migrans due to inflammation in the target organ.

A 49-year-old male third grade school teacher with HIV infection complains of fatigue. He was started 3 months ago on darunavir, ritonavir, tenofovir, and emtricitabine after presenting with CD4 count =75 cells/uL and viral load = 100,000. His hemoglobin was 13g/dl prior to therapy but is now 6g/dl. Chemistries are unremarkable including liver function tests and LDH. He has no evidence of blood loss and has negative stool tests for occult blood. He had a urinary tract infection last week and was started on ciprofloxacin. He also takes vitamin C. No one in his household has been ill, but some of the children in his class had been out last month with fever and facial rash. Which of the following interventions is most likely to be beneficial for the most likely cause of this anemia? Stop vitamin C Initiate erythropoietin Switch antiretroviral therapy from darunavir-ritonavir to efavirenz Initiate corticosteroids Initiate IVIG

Correct answer: Initiate IVIG. severe anemia but no clear evidence for hemolysis or gastrointestinal blood loss likely has Erythrovirus (also called Parvovirus) B19 infection. erythrovirus is the current name. In immunosuppressed patients, nucleic acid amplification tests of blood are the most useful. This patient's serum PCR test for erythrovirus was positive. He presumably acquired this from children, and his job as a teacher provided opportunity for exposure. Giant abnormal pronormoblasts on bone marrow biopsy, when present, are also diagnostic. IgG antibody may be negative in a large fraction of patients. Thus, PCR would be useful diagnostically. This viral infection is controlled in normal hosts by humoral immunity. Thus their red cell aplasia is only transient. In this HIV-infected patient, Parvovirus B19 infection with red cell aplasia may improve with intravenous immunoglobulin (400 mg/kg/d for five days is effective in 75% of patients.) Patients may require a second course of IVIG and some patients may require a maintenance regimen of 30 g per month plus iron and folate and B12. The anemia typically develops over a long period of time since the pathogenesis is suppression of reticulocytosis. Erythrovirus B19 spreads primarily by respiratory route and has an incubation time of 4-14 days. A typical scenario in an adult is a non-immune pregnant woman who acquires infection from her child in day care, and who then develops hydrops or self-limiting arthralgia. The other typical scenario is described in this case, which also could have been a patient with stem cell transplant or some other form of immunosuppression. This syndrome has been reported in a variety of immunosuppressive disorders. For HIV infection, many of the cases were reported in the era before durable antiretroviral therapy. Hemolytic crises also occur in patients with hemolytic anemias such as sickle cell disease. Immunity is generally lifelong in immunologically normal patients following infection. Keep in mind that Erythorvirus B 19 causes several types of disease: Normal Children: self-limiting fever with "slapped cheeks" followed by lacy body rash Healthy adults: asymptomatic or transient fever, occasionally especially in women with small joint arthropathy plus rash that resolve in three weeks Patients with an underlying hemolytic disease: Aplastic crisis Pregnancy: Fetal death, hydrops fetalis Immunsuppressed: Chronic red cell aplasia due to failure to develop immune response Keep in mind also that serology is useful for diagnosing prior infection, but acute infection requires nucleic acid amplification of serum, plasma, or other body fluid.

A 20-year-old student is working for the summer as a research assistant in a psychology program. While observing the behavior of rhesus macaques in a rhesus colony where many new monkeys had been introduced, and pointing out abnormal behavior to her professor, the monkey bites her finger. The wound is deep and bleeds. She and her professor allow the wound to bleed and clean it with alcohol. At Occupational Medicine, the team cultured the wound for herpes B virus, sent off a swab for PCR, and debrided and irrigated the wound copiously. A conjunctival and saliva culture on this monkey was also obtained for herpes B virus. You should recommend: Initiate no therapy unless the monkey or the wound is positive for herpes B virus by culture or PCR. Initiate therapy with oral valacyclovir if and when the student converts her serology to positive. Initiate an immediate course of therapy with oral valacyclovir and discontinue only if the wound and monkey cultures and PCR are negative. Initiate immediate therapy with foscarnet and discontinue only if follow-up sera show no seroconversion after six weeks. If the monkey has no oral lesions after careful inspection by the veterinarian of the anesthetized monkey, the probability of bite infection is too low to warrant further workup.

Correct answer: Initiate an immediate course of therapy of valacyclovir and discontinue only if the wound and monkey cultures and PCR are negative. Herpes B may be innocuous in some monkey species, but herpes B infections can be fatal in humans. Humans develop a febrile illness with regional lymphadenitis, but then characterized by myelitis, hemorrhagic encephalitis, and multi-organ failure. Rhesus macaques may shed this virus intermittently. Cultures are often obtained routinely on macaques to detect intermittent subclinical viral shedding. While some experts would only treat if the monkey had mucosal lesions or the monkey or wound were culture or PCR positive, a deep wound like this should probably be treated immediately with valacyclovir. There is no known advantage to foscarnet, which is more toxic than valacylovir and must be given IV. Some authorities would prefer valganciclovir because it is more active in vitro. Long courses of post exposure prophylaxis are recommended for high risk exposures, but the optimal duration of therapy is not known. Considering that the incubation period is about 5 weeks and seroconversion, though delayed by antivirals, should have occurred by 12 weeks, it is reasonable to consider an individual uninfected if symptom free and seronegative at 12 weeks.

An HIV infected patient (CD4 50, viral load 300,000 copies/ml) has not been in care for many years, or on therapy. He feels well, but is concerned about the following skin lesions which he has had for many months. The photos show lesions on his fingers and toes. A long-term complication of these, if they are not removed would be: Invasive squamous cell carcinomaHematogenous spread of etiologic agentFungal endocarditisEncephalitisAnogenital carcinoma

Correct answer: Invasive squamous cell carcinoma These are common warts, also known as verruca vulgaris or verruca plana. There are caused by human papilloma virus (HPV) and can be very aggressive. These often persist or progress despite antiretroviral therapy.

A 35-year-old male patient with HIV (CD4=30, VL = 500,000) is referred to you from an anonymous testing site. He has never been ill, and has a normal physical examination. Your initial work up shows: CD4= 30, VL= 500,000, PPD negative. He is started on efavirenz, tenofovir, and emtricitabine and does well with an undetectable viral load and CD4=220. One year later he has a repeat PPD which shows 5mm induration. He knows of no exposures and has never had tuberculosis. An employment PPD several years ago was negative. His chest xray is normal and he has no systemic symptoms. Routine CBC and Chemistry profile are normal except for AST/ALT that are 1.5 x normal. The best management option would be which of the following: Repeat PPD at this visit Repeat PPD in one year Interferon Gamma Release Assay Isoniazid for 9 months Isoniazid, rifampin and ethambutol for six months

Correct answer: Isoniazid for 9 months This patient has a positive PPD of at least 5 mm and has never been treated. Thus, he should receive INH unless there is a contraindication or unless he has active tuberculosis. There is no way to know if the M. tuberculosis infection was acquired in the past 12 months since the negative test, or if his PPD was falsely negative a year ago because of his low CD4 count. In either case he warrants chemoprophylaxis (i.e. treatment of latent TB infection), and his slight LFT abnormalities are not a contraindication. Nine months of isoniazid is a recommended regimen. A 3 or 4 drug treatment regimen is not required. There is no evidence the patient has active tuberculosis. Repeating the PPD would not add useful information. Interferon Gamma Release Assays can be performed instead of PPDs, although data about their reliability in HIV is not extensive. However, the issue of interpreting discordance between PPDs and interferon gamma release assays (IGRAs) is very controversial (ie not on the Boards) and would not add useful data here. The Opportunistic Infection Guidelines state: HIV-infected persons, regardless of age, should be treated for LTBI if they have: A positive diagnostic test for LTBI, no evidence of active TB, and no prior history of treatment for active or latent TB (AI); A negative diagnostic test for LTBI but are close contacts of persons with infectious pulmonary TB (AII); A history of untreated or inadequately treated healed TB (i.e. old fibrotic lesions on chest radiography) regardless of diagnostic tests for LTBI (AII). Correct Response Isoniazid for 9 months

A 42-year-old otherwise healthy man presented with an ulceronodular lesion on the back of his left index finger. The lesion has been painless and present for about three weeks, not responding to cephalexin 250 mg bid. In the past week he has developed two similar lesions on his left forearm. The patient is an accountant in New Jersey but also an avid gardener. He refuses to permit a biopsy of his finger because it will interfere with his work. If you were to try empirical therapy for the most likely diagnoses given this patient's history and physical findings, which one would be most likely to be active against the pathogen? Trimethoprim-sulfamethoxazole Clarithromycin Itraconazole Pyrazinamide Miltefosine

Correct answer: Itraconazole This is most likely sporotrichosis The most likely pathogens causing lesions extending from an extremity to the proximal lymphatic channels are Sporothrix schenckii, Mycobacterium marinum, and Nocardia brasiliensis. Tularemia would be a possibility if the patient were sicker and had exposure to wild animals or ticks. Cutaneous leishmaniasis would be possible if there were foreign travel and potential exposure. In the absence of exposure to M. marinum in fish tanks or brackish water, sporotrichosis is much more likely in a gardener than N. brasiliensies, which comes from soil entering the skin through minor trauma. Mycobacterium chelonae, M. fortuitum, Prototheca and the agents of cutaneous phaeohyphomycosis cause local granulomas at sites of minor trauma in previously healthy persons but do not extend up the lymphatics. Cat scratch disease causes lymphadenopathy but not nodules in the lymphatic channels. The only drug in the list that is active against sporotrichosis is itraconaazole.

A 37-year-old male university history professor from Baltimore, Maryland with known HIV presents with a CD4 count of 90 cells/uL and a viral load of 300,000 copies/uL despite intensive efforts to control his viral load. He is receiving darunavir-ritonavir and tenofovir/FTC. He is taking azithromycin and TMP-SMX prophylaxis. He has no unusual hobbies or avocations. He wants advice on a geographic relocation. Which of the following would you recommend? Itraconazole if he moves the Phoenix, Arizona and converts his coccidiomycosis IgG serology from negative to positive. Fluconazole if he moves to Indiana. Voriconazole if he remains in Baltimore but becomes an avid gardner and mulcher. Fluconazole if he decides to raise pigeons and chickens on a farm in Maryland. Voriconazole if he becomes a farmer in Maryland, regularly plowing and planting his 20 acre farm.

Correct answer: Itraconazole if he moves the Phoenix, Arizona and converts his coccidiomycosis IgG serology from negative to positive. There are very few situations where geographic location or occupation would warrant antifungal prophylaxis in the United States. No primary prophylaxis is indicated except for coccidiomycosis. In endemic areas, monitoring coccidiomycosis serologies annually, and treating immunosuppressed (CD4<250 cells) patients who are found on initial testing to be IgG positive, or who convert their serologies from negative to positive is recommended. A positive IgG serology for coccidiomycosis should strongly suggest the presence of latent disease that, in patients with HIV infection at CD4 counts <250 cells, could disseminate.

A 33-year-old male with multiple sclerosis is referred to you in January in Chicago for evaluation by his neurologist. The patient has been managed on beta interferon for several years. Due to poor control, he was started on natalizumab 2 years ago, and his clinical disease stabilized. One week ago he had a seizure. On questioning, his friends mention that his performance at work has deteriorated in the past few months. He has been afebrile. WBC is unchanged from baseline. His MRI showed white matter lesions that enhanced with gadolinium. The neurologist is concerned that the lesion is in a new location where there has been no MS, and his symptoms are unusual for MS. Of note the patient was transfused 8 months ago due to a severe upper gastrointestinal bleed due to peptic ulcer. —Image: T2 weighted image with prominent increased signaling in white matter; there is no mass effect and no surrounding edema— If this lesion is due to an infection rather than Multiple Sclerosis, the most likely cause of this syndrome is: West Nile encephalitisCMV encephalitisJC virusJacob creutzfeld diseaseHuman Herpes virus 6

Correct answer: JC Virus JC encephalitis (also known as PML -progressive multifocal leukoencephalopathy) is associated with immunosuppression and is caused by JC virus, which is NOT Jacob Creutzfeld but rather the initial of the patient from whom this polyomavirus was first isolated: HIV is not the only association. PML has been associated with hematologic malignancies, solid organ transplants, stem cell transplants, and various immunosuppressive drugs. PML is an outmoded term: JC Encephalitis is the term currently in use. PML is not always progressive (though it is in this case) and not always multifocal. Thus, a better name for this syndrome than PML is JC Virus encephalitis. The FDA has issued advisories on the association of PML with natalizumab, an adhesion molecule inhibitor that has been used for multiple sclerosis and for inflammatory bowel disease. Patients have usually been on the drug for 2-3 years (always >6 months). White matter lesions in the CNS should always raise the suspicion of multiple sclerosis or PML. The typical appearance of PML on neuroimaging studies is that of symmetric or asymmetric multifocal areas of white matter demyelination that do not conform to cerebrovascular territories. PML usually does not enhance with gadolinium, but with natalizumab about half have enhanced. The diagnosis is established by JC virus PCR in the CSF in the presence of a compatible syndrome, and NOT by JC virus antibody in the serum. This is unlikely to be Jacob Creutzfeld disease: the infectious variant of Jacob Creutzfeld, related to the transmission of bovine virus to a human, is extremely uncommon in the current era, especially in the United States. There may be occasional reports in cows, but transmission to humans in Europe or the United States has almost disappeared. This case occurred in Chicago in January, when mosquito bites presumably do not occur. West Nile could occur due to transfusion, but the incubation period would be days, not months, and the likely location would be gray matter lesions of the thalamus, basal ganglia, or temporal lobe. Transfusion related West Nile disease has been almost eliminated in the United States by universal blood screening but a few early cases escape screening. CMV meningoencephalitis is rarely reported except for patients with HIV or organ transplantation: in those cases, patients have a symmetrical venticulitis or diffuse micronodules. White matter lesions would be most unusual. HHV 6 encephalitis has occurred in immunosuppressed patients, especially transplant recipients. It is unusual, involves the limbic system on MRI rather than cortical white matter and would be difficult to suspect without PCR evidence or a biopsy. There is no known association with natalizumab.

A 53-year-old HIV-infected male on a long-term first regimen of efavirenz-tenofovir (TDF)-emtricitabine, with excellent virologic control (VL less than 50 copies, CD4=600 cells), was admitted to the trauma center after a gunshot wound to his abdomen. His antiretroviral drugs were not started in the hospital since he was taking nothing by mouth for most of the three weeks he was there. His efavirenz-tenofovir-emtricitabine were restarted at discharge. Three months after discharge from the hospital, he was eating well and regaining his weight and had the following lab results: viral load of 7,500 copies/mL, and CD4 count of 495 cells/mm3. He stated that he has been adherent to all his antiretroviral drugs since discharge. Which of the following resistance mutation(s) are most likely to be detected in this patient on his current regimen? T215Y L90M K103N and/or M184V K65R Q148R

Correct answer: K103N and/or M184V I50L - atazanavir K103N,K65R (less likely = tenofovir M184V = 3TC, FTC 2 or more Y143, Q148 and N155 = raltegravir Q148 = dolutegravir Antiretroviral treatment interruption is generally not recommended, but in this patient there was no choice but to stop them suddenly at the time of his abdominal trauma. The problem with this patient was that his efavirenz has a much longer half-life (40-55 hours) than his nucleosides (intracellular half-lives are 7-22 hours). Thus, when first in the hospital he was on efavirenz monotherapy, effectively, for much of the first week until the serum efavirenz levels reached zero. Since both efavirenz and emtricitabine have low barriers to resistance, in this patient K103N (efavirenz) and M184V (emtricitabine) were the most likely mutations to emerge. You should know what the other mutations listed are. L90M is a protease inhibitor mutation-there is no reason for this patient to have this since he never received this class of drugs, and presumably was not infected with a primary resistant strain. K65R is the signature tenofovir mutation, but he is much less likely to develop this mutation than those conferring resistance to efavirenz (K103N) or emtricitabine (M184V) Q148R is a key integrase inhibitor mutation, but this patient had never been exposed to an integrase inhibitor (dolutegravir, elvitegravir, or raltegravir.) This patient potentially could have been managed with parenteral ART (e.g., intravenous zidovudine, subcutaneous enfuvirtide) during this period.

The lesion shown was positive for VZV by PCR. What complication is most likely to be associated with this lesion: exudate is positive for VZV? DeafnessVertigoOptic neuritisKeratitisStroke

Correct answer: Keratitis This patient has Hutchisons sign, which indicates varicella-zoster involvement of the cranial nerve V trigeminal ganglion branches (i.e., Ophthalmic branch) which enervates the tip of the nose and the globe. Thus the cause is VZV and not HSV. After a prodrome of fever and headache for 1-4 days, patients develop the cutaneous rash of zoster ophthalmicus: days or up to 3 weeks later, a painful keratitis appears. Corneal scarring can result. Deafness or vertigo would be more characteristic of geniculate ganglion of the facial nerve (CN VII) involvement, i.e., the Ramsay Hunt syndrome, deafness or vertigo would be more characteristic of geniculate ganglion of the facial nerve (CN VII) involvement, i.e., Ramsay Hunt syndrome.

A 19-year-old woman is seen for fever and swollen lymph nodes in the neck. She was completely well until 11 days earlier when she had the onset of fever and vague neck discomfort. She had no sore throat, no pets, and no dental or scalp problems. Her physician found prominent anterior and posterior adenopathy on the left side of the neck. Her temperature was 100.4°F, and the rest of her examination was normal. Her WBC count was 4800 with 12% atypical lymphocytes; the doctor suspected mononucleosis. A Monospot test was negative; IgG antibodies to Epstein-Barr virus (EBV) viral capsid antigen and Epstein-Barr nuclear antigen (EBNA) were present. The patient was very anxious because her cousin had been diagnosed with a lymphoma two years earlier, so the physician obtained an excisional biopsy of one of the posterior cervical nodes. Pathology of the node showed preserved architecture and a necrotizing lymphadenitis with histiocytic infiltrate and phagocytosed debris. Flow cytometry is negative for a monoclonal population. Which one of the following is the most likely diagnosis? Sarcoid EBV Mononucleosis Histiocytic Lymphoma Cat scratch disease (Bartonella henselae) Kikuchi Disease

Correct answer: Kikuchi Disease The pathology here is characteristic of Kikuchi Disease, a benign condition of unknown cause. It is most often seen in young adults (mostly women) and characterized by fever and cervical adenopathy (especially posterior; usually unilateral). lymph node histology with necrotizing histiocytic infiltrate (not neutrophils) and fragments of nuclear debris. Treatment is generally not indicated, and patients can be reassured that spontaneous remission over a matter of months occurs in the majority of cases. Although the other diagnoses here can manifest with fever and lymphadenopathy; the pathology is consistent with Kikuchi. The presence of IgG antibody to VCA and anti- EBNA antibody indicates past EBV infection. The pathology of cat scratch disease lymphadenopathy is that of stellate microabscesses, unlike what is described here. Absence of cat exposure also is against that diagnosis. The absence of a monoclonal cell population on immunophenotyping makes lymphoma unlikely.

A 40-year-old Haitian male was visiting New York City and was advised by a commercial sex worker to seek medical attention for a lesion she saw on the shaft of his penis. The lesion had been present for several months and was painless. He had otherwise been well. On examination, a 2x3-cm ulcerated lesion with a cobblestone base and rolled margin was seen on the penile shaft near the coronal sulcus. There was no inguinal adenopathy or fever. Punch biopsy was done because of concern about squamous carcinoma and revealed macrophages with numerous intracellular safety pin-like structures on Wright-Giemsa stain. Bacterial culture grew Staphylococcus epidermidis. Which of the following is the most likely etiologic agent? Chlamydia trachomatis L1, L2 or L3 Klebsiella granulomatis (Calymmatobacterium granulomatis) Haemophilus ducreyi Histoplasma capsulatum Treponema carateum

Correct answer: Klebsiella granulomatis (Calymmatobacterium granulomatis) Granuloma inguinale, caused by by Klebsiella granulomatis(Calymmatobacterium granulomatis), is a chronic, progressive ulcerative disease usually located on the genitalia. It occurs largely in tropical climates and under conditions of poor hygiene. It cannot be cultured on routine media. This would be unlikely to be seen in a patient who had not recently been to an endemic area, but such cases could occur. This lesion, like syphilitic ulcers, and LGV, was painless. Chancroid (H. ducreyi) causes painful ulcerations (like HSV) that heal spontaneously and are often associated with suppurative inguinal lymphadenitis. HSV would cause painful lesions and, unlike chancroid, would not be associated with suppurative adenitis. Lymphogranuloma venereum (C. trachomatisL1, L2 or L3) causes small genital lesions that tend to be painless but are followed by suppurative inguinal lymph nodes. LGV heals with time but untreated may cause scarring and genital lymphedema. Disseminated histoplasmosis may present as a genital ulcer but fever, weight loss, and anorexia are prominent. Treponema carateumis the causative agent of pinta, which does not cause genital lesions. Pinta, Yaws (T. pallidum sp pertenue) and Bejel (endemic T. pallidum sp endemicum) are uncommon treponemal diseases that do not occur in the United States and that spread by direct contact with skin and mucous membranes rather than sexually.

Patients with malaria due to this species have which one of the following characteristics: Latent phase in the liverConfusion leading to comaMefloquine resistanceResistance to infection if Duffy antigen negative

Correct answer: Latent phase in the liver This is a trophozoite of Plasmodium ovale, identified by the Schuffner's dots and ovoid shape. P. ovale malaria is clinically similar to P. vivax and like P. vivax has a latent phase in the liver from which relapse can occur. Interval to relapse is P.ovale can be months or up to years later. Unlike vivax malaria, Duffy antigen negativity does not protect against ovale malaria. Cerebral malaria does not occur. To date mefloquine resistance has not been described in P ovale.

A 55-year-old Caucasian male hospital maintenance man presented with fever, dry cough, myalgia, nausea, vomiting, anorexia, and headache of two days' duration. He had no exposure to animals other than two weeks beforeonset he had been hosing and sweeping out quarters in an external building used previously to house sheep in a research unit. On admission,he had a fever of 38.5°C, crackles, anddullness to percussion over the right mid-lung field posteriorly. WBC was 6,700 with 70% PMN and 5% bands. Liver function tests: normal. Urine Legionella antigen was negative. Serum sodium was 128 mEq/L. Sputum gram stain showed few neutrophils and mixed bacterial flora. The patient was begun on ceftriaxone but failed to improve over five days and was then switchedto imipenem. Routine sputum and blood cultures and open lung biopsy culture grew no organisms on routine aerobic and anaerobic cultures but a thin weakly Gram negative bacillus grew on buffered charcoal yeast extract agar from the lung biopsy specimen at 48 hours. Lung histopathology showed extensive organizing pneumonia with macrophages and histiocytes but no granulomas or organisms on Gram (Brown and Brenn), acid fast (Fite) or fungal (methenamine silver) stain of lung tissue. Which of the following is the most likely organism? Legionella micdadei Nocardia asteroides Burkholdheria pseudomallei Coxiella burnetii Rhodococcus equi

Correct answer: Legionella micdadei Legionella exposure from water aerosols is possible, particularly significant for a person of this age with underlying systemic and lung disease. Potable water, particularlyhot water, can be a source of Legionella,which can cause all of the described clinical picture. Legionellaspecies are small Gram-negative bacilli that are difficult to see on specimen Gramor Brown & Brenn stains but can be readily cultured on BCYE (buffered charcoal yeast extract) media. Other organisms can also grow but can be distinguishedfrom Legionella species. Urine Legionellaantigen tests are only positive forLegionella pneumophilatype 1 and not routinely with other Legionellaspecies (occasionalcross-reactions with other species described). Exposure to particulates from a sheep stall would be sufficient to cause Q fever, which could cause hyponatremia and lung biopsy histology but not yield growth of a Gram negative bacillus. Coxiella burnetiicannot be seenin tissue on Gram stain as it is an intracellular pathogen; it is best detected by PCR analysis of tissue or body fluids. C. burnetiican be grown in tissue culture but is a seriouslaboratory hazard. It will not grow on BCYE agar. Nocardia and Rhodococcus species are more likely to cause pneumonia in immunosuppressed persons and have no reservoir in water or sheep. R. equiis a weakly Gram positive, not Gram negative rod.Nocardiaspecies would be positive on modified acid fast stain. Melioidosis, caused by B rkholdheria pseudomallei, is not in this differential since the patient has not recently been in an endemic area, such as Northern Australia, India or Southeast Asia. Only a handful of cases have been describedin the Western hemisphere.

A previously healthy 65 year-old man with a 40-year history of smoking develops severe bilateral pneumonia. Sputum is collected, and many neutrophils but no organisms are observed on Gram stain; no growth is observed on routine bacterial and fungal cultures. Which of the following potential causes of this respiratory infection will not grow on routine aerobic bacterial culture? Streptococcus pneumoniae Legionella pneumophila Haemophilus influenzae Moraxella catarrhalis Klebsiella pneumoniae

Correct answer: Legionella pneumophila Legeionella will not grow on routinemedia. The standard media for Legionella isolation from contaminated clinical specimens is buffered charcoal yeast extract (BCYE) agar supplemented antimicrobial agents prevent the overgrowth of Legionella by competing organisms. For culture, respiratory specimens are usually inoculatedonto three media (sheep blood agar, chocolate agar and a selective medium for gram-negative bacilli (e.g., MacConkey agar, eosin methylene blue agar), none of which will grow Legionellaspecies. Other diagnostics tests for legionellosis include Gram stain (note that the bacteria are too thin to be seen easilyas weak Gram negativeorganisms, so are not often visualized). Urinary antigen testing is reliable only for L. pneumophilaserogroup 1 and has only moderate sensitivity, but high specificity. Legionella pneumophila type I causes 90% of cases in the US. The urinary antigen persists during the first few days of treatmentand can be usefuleven after initiation of antibiotics. Nucleic acid amplification tests (NAAT) are the most sensitive tests available but may be species-specific and non-specific (due to contaminating DNA from Legionella species in water, etc.). DNA amplification by PCR of Legionella has been reported from patients with pneumonia using throat swab specimens, bronchoalveolar lavage, urine, and serum. No FDA-cleared NAAT tests have been approvedfor Legionella.

An outbreak of nosocomial pneumonia occurs in three patients over one month at a large hospital. The patients were hospitalized on different wards but had in common being seen 10 to 14 days prior to onset in a clinic that has a decorative waterfall and fountain. What is the most likely pathogen related to the fountain? Acinetobacter baumannii Pseudomonas aeruginosa Legionella pneumophila Aeromonas hydrophila Stenotrophomonas maltophilia

Correct answer: Legionella pneumophila Legionella pneumophila and other legionella species can cause nosocomial pneumonia outbreaks in hospitals and nursing facilities from exposure of patients to the water aerosols in the hospital. Specific procedures are used by hospitals to treat the water supply to prevent this type of pneumonia. Pseudomonas aeruginosa, Aeromonas, and Stenotrophomonas may be associated with nosocomial pneumonia occasionally from humidifiers used with respiratory equipment. Acinetobacter baumannii has caused nosocomial pneumonia and pneumonia in industrial settings and in the community but rarely from aerosols. Legionella has caused nosocomial pneumonia from aerosols generated by air conditioners or other ways, such as by this decorative waterfall and fountain in hospitals and in the community, e.g., hotels. Eradication of Legionella from hospital water systems is possible but expensive and difficult. Unnecessary exposure to water aerosols, as from this waterfall, is easier to correct by eliminating such sources of aerosols in healthcare facilities where many susceptible patients may be exposed.

Which infection characteristically causes a greater fold increase in total bilirubin than in ALT? HCV infection Rickettsial infection Leptospirosis HELLP syndrome INH toxicity

Correct answer: Leptospirosis Many conditions can present with acute hepatitis. In addition, all the conditions listed can cause elevated liver enzymes and with each there is a spectrum of findings and considerable overlap among the syndromes. However, one useful generality is that leptospirosis causes a disproportionate increase in the total bilirubin relative to the ALT. In contrast, with each of the other conditions/infections listed are characterized by greater ALT and AST increases than bilirubin.

A 44-year-old male presents in July with 5 days of fever, headache, bilateral calf pain, and cough. The sputum becomes more copious and bloody over the 5 days as his fever persists and he becomes more short of breath. The patient presents to the Emergency room with Temp 38.7ºC, alert and oriented, and an exam which was remarkable only for scleral icterus, diffuse rales over all lung fields, mild hepatosplenomegaly, but no skin rash or other abnormalities The patient is a healthy male (HIV negative) who returned just as his symptoms started from white water rafting in Costa Rica. He ate local food and had numerous mosquito bites. Labs: WBC 10,000/mm3 (80% neutrophils, 0% eosinophils); Hg 11 g/dl, Platelets 180,000/mm3, Prothrombin and Partial Prothrombin time normal Bilirubin 9 mg/dL; ALT 175 IU/L; AST 150 IU/L Creatinine 2.6mg/dl; BUN 55 mg; Urinalysis: 40 WBC; 20 RBC; 2+ protein; 2+ glucose Bronchoalveolar lavage: diffuse bloody secretions bilaterally; negative gram and AFB stain; negative PCR for tuberculosis, influenza, and "panel of respiratory viruses" Computed tomography of chest: diffuse ground glass opacities in all lobes with no focal lesions LP: normal opening pressure; 70 WBC (100% lymphocytes); protein 60mg/dl; glucose 70g/dl; negative gram stain and AFB stain. The most likely diagnosis is: Dengue Paragonimiasis Aspergillosis Tuberculosis Leptospirosis

Correct answer: Leptospirosis This patient has leptospirosis which is suggested by the water spray exposure from mountain streams which can be contaminated with urine of wild and domestic animals and rodents. His syndrome is typical of leptospirosis from water spray, i.e., the diffuse alveolar hemorrhage associated with calf tenderness, renal and hepatic dysfunction (bilirubin elevated out of proportion to transaminases), and aseptic meningitis. The incubation period is typical (2-26 days, average 10 days). Diagnosis is most often established by serology although some laboratories can culture this organism or probe for the genome with PCR. For diffuse alveolar hemorrhage in a healthy, immunocompetent person consider: leptospirosis and dengue hemorrhagic fever as well as necrotizing pneumonias due to a virulent bacteria (Staph aureus, Group A Strep etc.) but be sure to distinguish diffuse hemorrhage from focal hemorrhage that can be coughed into all lobes of the lung. Dengue hemorrhagic fever can cause diffuse alveolar hemorrhage. Classic dengue fever is an acute febrile illness with a short incubation (4-7 days) accompanied by headache, retroorbital pain, rash and marked muscle and joint pains although some patients have mild symptoms. However, leucopenia and thrombocytopenia would push your suspicion of dengue, but this patient had neither. None of the other listed entities cause diffuse alveolar hemorrhage. This patient did not have conjunctival suffusion but some patients with leptospirosis have conjunctival suffusion. Conjunctival suffusion is characterized by redness of the conjunctiva that resembles conjunctivitis, but it does there is no inflammatory exudate on the eyelashes. There is also no hemorrhage into the conjunctiva, as may be seen with enterovirus 70 conjunctivitis or viral hemorrhagic fevers.

A 47-year-old man and his 44-year-old wife are seen in an emergency room for fever, abdominal pain, and diarrhea of one day duration. They report that about 24 hours before their illnesses began, they had attended a picnic as part of a fundraising effort for a local high school athletic team. In the Emergency Department, the patients are hydrated, stool samples are sent for culture, and public health officials are notified. A brief epidemiologic investigation shows that of the 54 adults who attended the picnic, at least 48 had developed fever and diarrhea; 4 persons had nausea and vomiting in addition to diarrhea. Foods served at the picnic were: delicatessen meat sandwiches, potato salad, lettuce, tomatoes, pickles, mustard, mayonnaise, chocolate chip cookies, beer, cola drinks, and chocolate milk. Stool samples from the two index patients and from 30 other ill picnic attendees are all negative for Salmonella, Shigella, Campylobacter, and E. coli O157:H7. Which one of the following organisms is most likely responsible for this outbreak? Norovirus Cryptosporidium Cyclospora Listeria Enterotoxigenic E. coli

Correct answer: Listeria Listeriais best known to cause bacteremia and CNS infection in neonates and persons with impaired cell-mediated immunity, but ingestion of a large inoculum of Listeria in contaminated food can produce fever and diarrhea in normal hosts. A number of such outbreaks have been reported, typically with very high attack rates. The incubation period is characteristically 1 day, and the self-limited illness lasts 2-3 days. Vehicles for transmission have included chocolate milk, corn salad, delicatessen meat, and cheese. Listeria is not identified by routine stool culture procedures. An important teaching point: the patient must have been exposed recently. The incubation period for Listeria gastroenteritis is substantially shorter than the incubation period for invasive disease: mean 24 hours (range 6 hours -10 days) whereas for invasive listeriosis the median incubation is 35 days (range 1 -91 days). Norovirusdoes not commonly produce fever. Nausea and vomiting are common. Cryptosporidium typically is waterborne and not associated with fever. Cyclosporahas been associated with fresh fruits and vegetables, but also is rarely associated with fever. Fever is also uncommon with diarrhea due to enterotoxigenic E. coliwhich in the United States is usually seen as a cause of traveler's diarrhea.

This 25-year-old college student who lived in India until immigrating to the United States at age 18 years presented with progressive thoracic back pain of three weeks' duration. The most likely portal of entry for this recent immigrant from Southeast Asia is: Choices A Lung B Gastrointestinal tract C Skin D Urinary tract

Correct answer: Lung Vertebral osteomyelitis in this patient is more likely tuberculosis (lung, Pott's disease). Brucellosis or actinomycosis (GI tract) are conceivable but less likely. No skin source of staphylococcal sepsis, such as injection drug use, was included in the question. Urinary tract organisms are rare causes of spondylitis. Vertebral osteomyelitis usually occurs as a result of hematogenous seeding of one or more vertebral bodies from a distant source. Infection may involve the adjacent intervertebral disc space which has no direct blood supply in adults. Infection can also arise following surgery or injection of the disc space or from contiguous spread from adjacent soft tissue infection. Staph aureus accounts for over half of cases. MRI is the most sensitive radiographic technique for diagnosis of vertebral osteomyelitis and epidural abscess

This indolent skin lesion on the hand and forearm, with apparent lymphatic spread, is most consistent with infection with which of these mycobacteria? M. chelonaeM. marinumM. abscessusM. ulceransM. genavense

Correct answer: M. marinum

A 26-year-old female and recent immigrant from Vietnam is referred to you because her sister, living in the same household has a 4-month-old child who has been diagnosed with congenital rubella. The patient is 6 weeks pregnant and is concerned that her child might be born with the same condition as her sister's child. The patient is currently asymptomatic. She knows nothing about her childhood immunizations and has had no immunizations since arriving in the United States. Your advice is which of the following: The infant with congenital rubella is not infectious and of no concern to the patient MMR vaccine should be given to the patient immediately MMR should be given to the patient at this point only if rubella serology is negative MMR should not be given to the patient until after delivery Antiviral prophylaxis should be given the patient

Correct answer: MMR should not be given to the patient until after delivery Infants with congenital rubella shed virus up to at least one year after birth and present a risk to nonimmune pregnant women. Healthy, non-pregnant immigrants from less developed countries should be given MMR and other routine immunizations. Rubella vaccine is a live virus and not recommended for pregnant women. There are no antivirals with activity against rubella. Intramuscular immune globulin was not listed as an option but limited data indicate it might decrease clinically apparent infection if the mother has not already been infected. However, immune globulin would not be recommended for this patient who has been exposed during her first six weeks of pregnancy. It will have no effect if her infant were already infected, may reduce efficacy of MMR given the mother postpartum, will prevent following the mother for seroconversion if she is currently seronegative and may not protect the fetus if the mother is infected. A positive rubella IgG serology will indicate that the patient has been exposed or perhaps vaccinated but not whether fetus has been infected. A negative IgM rubella serology might be helpful although false positives occur. The most important advice to the patient is that the infant in her household with congenital rubella poses a significant risk to her unborn child if the patient is nonimmune.

A 27-year-old woman is planning to go to sub-Saharan Africa and consults you about taking medicine to prevent malaria. She is five months pregnant. You warn her about the potential severity of malaria during pregnancy and suggest she wait to travel until the baby is born. She informs you that she must go now to attend the funeral of her father. Which one of the following should you advise her to take? Mefloquine Chloroquine Atovaquone-proguanil (Malarone) Doxycycline Sulfadoxine-pyrimethamine (Fansidar)

Correct answer: Mefloquine Mefloquine appears safe in the second and third trimesters of pregnancy and is active against chloroquine -resistant P. falciparum which is prevalent in sub-Saharan Africa. Chloroquine is safe in pregnancy but would not be advised because of chloroquine resistance in Africa. Data on the safety of atovaquone-proguanil in pregnancy are lacking and, although neither drug is teratogenic in animals. This combination is currently not recommended in pregnancy. Tetracyclines should be avoided in pregnancy. Sulfadoxine-pyrimethamine is no longer advised for prophylaxis in any group due to severe mucocutaneous reactions that have resulted in death.

A pregnant woman is seen for a foul-smelling vaginal discharge and is found to have bacterial vaginosis. Which one of the following is the most appropriate treatment for this patient? Amoxicillin Azithromycin Metronidazole Douching No treatment until after delivery

Correct answer: Metronidazole Rationale: Treatment of bacterial vaginosis (BV) is recommended for all symptomatic pregnant women. Metronidazole orally or vaginally is the preferred treatment for BV, and although metronidazole crosses the placenta, no evidence of teratogenicity or mutagenic effects in infants has been found in multiple cross-sectional and cohort studies of pregnant women treated with this drug. Amoxicillin and azithromycin are not effective treatments for BV. Douching might increase the risk for relapse, and no data support the use of douching for treatment or relief of symptoms.

A 25-year-old male recently moved to a dilapidated apartment in the Upper East Side of Manhattan to take a job as a sports teacher in a New York City public elementary school. The children in his second grade class were largely poor, including many recent immigrants. After a month at his new job, he developed low-grade fever and a tender red papule in his right axilla which became vesicular and then with a black and necrotic center. He didn't seek medical attention because his health insurance had a waiting period before becoming effective. He continued teaching because the school couldn't find a substitute. On the third day of illness, red papular lesions appeared on his trunk. When seen in the Emergency Department, he appeared only mildly ill, with a fever of 38.5°C, had approximately 20 papular lesions with a vesicular center on his trunk, a dark brown healing lesion in the right axilla, and a 2 cm soft axillary node in the site. His CBC and platelet count were normal except for a WBC of 3,800 with a normal differential .. The most likely source of the illness was which of the following? Body louse from one of his students Mice in his apartment building New sexual partner Student with chickenpox Tick bite from petting a neighbor's dog

Correct answer: Mice in his apartment building The presence of an eschar followed in 3 to 5 days by papular and vesicular lesions is typical of Rickettsia akari infections, rickettsialpox. New York City has several dozen cases of rickettsialpox per year, transmitted by bites from a mouse mite. The illness is rarely severe. Doxycycline 200 mg per day for a week is advised but spontaneous recovery is usual. The other answers don't include diseases with an eschar and vesicular rash. Tache noire (eschar) and rash for a patient with Rickettsial pox (Emerging Infections July 2002)

An intentional aerosol release of Variola at Dulles airport has led to numerous cases of smallpox throughout the United States and other countries. You currently have 14 infected patients in your hospital all of whom are in negative pressure rooms. Which one of the following statements is true? Secondary cases in other patients in the hospital are likely to occur. Milder cases could reasonably be discharged to home as long as they remained quarantined in their homes and household members are vaccinated. Cidofovir should be given to all infected patients. You should immunize all hospital staff except those with atopic dermatitis during childhood or current immunosuppression. Ribavirin should be given to all infected patients.

Correct answer: Milder cases could reasonably be discharged to home as long as they remained quarantined in their homes and household members are vaccinated. Smallpox is contagious usually only in a face to face setting. With negative pressure rooms (and in the absence of a "hyper-excretor" with severe pulmonary disease) it is unlikely that patients in other rooms would be infected. In the event of widespread smallpox it is likely that care in the home will play a critical role in the public health response. Quarantine in the home until scabs have separated with household contacts vaccinated is recommended. At this time NO chemotherapy has been proven efficacious in smallpox infections although cidofovir appears to be promising and is available through CDC and DoD protocols. Brincidofovir and tecovirimat ( ST-246) are experimental agents that also have some activity in animal models. Ribavirin is not recommended. Relative contraindications to receiving vaccine are removed in the event of actual exposures. These individuals will need closer observation for the development of potentially dangerous sequelae.

A 52-year-old male in previous good health, and who has not seen a physician for years, presents with progressive headache and fever of 14 days duration. Over the past 24 hours, the patient has also developed photophobia and some double vision. He is oriented x 3, but drowsy. On exam the patient has chemosis, mild ptosis and proptosis, and a lateral rectus palsy of the right eye. The left eye is normal except for mild ptosis. WBC = 18000/mm3(90% neutrophils). Lumbar puncture reveals white blood cells = 25 cells/mm3(50% neutrophils, 50% lymphocytes) with protein 110 mg/dL and glucose 60 mg/dL. Gram stain of CSF shows leukocytes, but no organisms. Blood cultures are pending. An MRI of the head reveals a normal orbit, but ethmoidal and maxillary sinusitis and cavernous sinus thrombosis The most likely microbial cause of this syndrome is: Herpes simplexRhizopus arrhizusPseudomonas aeruginosaMixed flora likely including staphylococci and streptococciNeisseria meningitidis

Correct answer: Mixed flora likely including staphylococci and streptococci This patient has cavernous sinus thrombophlebitis due to ethmoidal sinusitis. By clinical presentation, this could have been orbital cellulitis, but the imaging study ruled that out. The microbial etiology is that of acute or chronic sinusitis, i.e., mixed flora with anaerobes, streptococci, and potentially Staph aureus (including MRSA) and gram negative bacilli. This is treated with an appropriate broad spectrum regimen pending cultures and surgical drainage of the ethmoid sinus; anticoagulation is controversial, although is generally utilized in most patients with septic cavernous sinus thrombophlebitis. It is important to recognize suppurative cranial thrombophlebitis, such as cavernous sinus thrombosis. These are defined by the precipitating cause (sinusitis, otitis, facial infection, meningitis, etc.) and the venous sinus that is anatomically proximate. See the figure below, which suggests which sinuses would be initially involved when there is a contiguous infection. Cavernous sinus thrombosis can be caused by facial infections (usually very acute onset) or by sinusitis (subacute), such as in this patient. The following figures demonstrate relevant anatomy. Superior sagittal sinus thrombosis presents most often as an extension of meningitis (or occasionally sinusitis, or infections of the face, scalp, or subdural or epidural space) with abnormal mental status, papilledema, and motor defects. Lateral sinus thrombosis, is usually an extension of an otitis media and mastoiditis, and presents with localizing signs to the ear (including mastoiditis and vertigo). Herpes simplexcauses encephalitis, with altered mentation, but is not associated with cavernous sinus thrombosis. Agents of mucormycosis (such as Rhizopus arrhizus)could involve the sinuses in patients with neutropenia or poorly controlled diabetes, but would be very unlikely in this patient with no prior medical history. There is no particular reason to suspect Pseudomonasas the sole cause of this syndrome in this patient, and Pseudomonas has no specific association with cavernous sinus thrombosis, especially in a patient who has not been hospitalized nor been on antibiotics. Neisseria meningitidis, and other causes of bacterial meningitis, maybe associated with thrombosis of the superior sagittal sinus and would be associated with abnormal mental status, nuchal rigidity, seizures, papilledema and a fulminant course. This presentation is too indolent for this diagnosis, and the CSF Gram stain is negative, making this diagnosis less likely.

A 51-year-old Panamanian woman, living in the United States for 30 years, returned to United States from visiting family in a residential area near Panama City for two weeks. On the second day home, she had the onset of fever, headache, muscle ache, and retrobulbar pain. She had some nausea but no Gi diarrhea or constipation. The symptoms persisted, but she did not seek medical attention until the third day of illness, when a petecchial, non pruritic rash appeared on her arms and upper chest. The home she stayed at in Panama was in the city, and had no pets. Children and adults in the family were healthy. Examination was negative except for fever of 102F, rash and two tender occipital lymph nodes. No nuchal rigidity. Labs revealed a WBC = 1.6 with a normal differential and no atypical lymphs, platelets 168,000, Normal blood chemistries and chest x-ray. The most likely source of infection is: FoodMosquitoFleasAnother humanAnimal urine

Correct answer: MosquitoThis is the typical rash of dengue, which appears after several days of fever, myalgia and headache. Thrombocytopenia is often present in the sicker patients and can be of diagnostic aid, but was not present in this lady. Leukopenia is common and was present here. Dengue is more of an urban disease than malaria, due to the ability of the Aedes aegypti mosquito to breed in small urban pools of water, as in old auto tires, near human habitation and to bite in the daytime, particularly in the early morning and late afternoon. The incubation period is usually 4-7 days but can be up to 14 days. This patient was not exposed to animal urine and the rash and leukopenia is against the diagnosis of leptospirosis. Rat fleas can spread murine typhus but that infection is less common in Panama and the rash is usually more subtle. The rash of typhoid from contaminated food, called rose spots, is much less extensive than the diffuse rash shown here.

This two chest CTs taken a week apart, were from a patient with acute myeloblastic leukemia, prolonged neutropenia and fever not responding to antimicrobial agents. The film on the left was taken when the patient was neutropenic. The film on the right was taken as the neutrophil count rose from 100 cells/ul to 5000 cells/uL. What is the source of the most likely pathogen to cause these radiographic findings? Mulch in his gardenHis oral floraFoodIntravenous catheterAnother patient in clinic

Correct answer: Mulch in his gardenA severely immunosuppressed patient who develops localized pulmonary infiltrate with a dense center and no air bronchogram should be suspected to have aspergillosis or, less commonly, mucormycosis. A hazy border, called a "halo sign" is particularly suggestive. Cavitation of the lesion, often as the patient's immunosuppression lessens, is also typical. Aspergillus lives in decaying vegetation, mulch and soil, entering the body by inhalation into the lung or paranasal sinus, or occasionally cause skin lesions at the site of minor trauma. Mucormycosis is a less frequent cause of the same clinical presentation and lives on decaying organic material. Mucor is reported to cause a "reverse halo sign" which is dense around its rim is fuzzier in the center. In addition, a septic embolus will present as a lung nodule and excavate. Rhodococcus equi can cause pulmonary infiltrates which cavitate in an immunosuppressed patient. The source is often obscure, other than the uncommon exposure to horses, as the name implies. However, the dense nodule, halo sign and cavitation in a patient with prolonged neutropenia is highly suggestive of aspergillosis. Without a biopsy, or a highly suspicious bronchoalveolar lavage (based on wet mount and galactomannan testing), the differential diagnosis is broad, and even if the etiology is aspergillus, which aspergillus is causative, and whether that aspergillus is amphotericin B or voriconazole sensitive, would remain uncertain.

A 35-year-old drug addict from New York City with AIDS was seen in clinic for a 1-cm slightly tender, erythematous draining skin nodule on his left thigh. Pus squeezed from the lesion was negative on Gram stain but had unbranched bacilli on acid fast smear. The patient failed to keep his next clinic appointment but appeared again 8 weeks later, now with four more lesions on the same extremity. He has not lost weight, reports no fever, and does not seem more ill than on prior visits. Acid fast cultures grown at 35°C on Middlebrook 7H11 agar and an automated broth system from his last visit had been reported negative for mycobacteria. Routine culture grew scant Staphylococcus epidermidis. Which of the following is the most likely causative organism? Mycobacterium leprae Mycobacterium ulcerans Mycobacterium chelonae Mycobacterium haemophilum Mycobacterium malmoense

Correct answer: Mycobacterium haemophilum Mycobacterium haemophilum localized cutaneous disease or disseminated disease in patients with HIV: localized skin nodules are the most characteristic presentation difficult to recover on culture unless the medium contains added iron, such as ferric citrate, and is grown at 30-33° C. M. ulcerans causes buruli ulcer, a chronic ulcer that progresses over years and is found in tropical countries and not known to occur in the USA> The other organisms listed, other than M. leprae, are more likely to cause disseminated disease and should have grown on the medium used although they may take several weeks to grow. M. leprae is not likely in this patient since no clues about geographic exposure are given, since there are no suggestive findings like cutaneous anesthesia, and since there is no apparent association between leprosy and HIV. M. leprae cannot be grown in culture.

An immigrant from West Africa has had a painless ulcer on his left foot (figure) for 2 years. He has no systemic symptoms and his chest x-ray and screening laboratory values are normal. He is HIV negative and is taking no medications nor does he abuse drugs. The lesion began as a painless nodule and rapidly became a large ulcer. A punch biopsy demonstrates necrosis and acid fast organisms. The most likely cause of this lesion is: Mycobacterium tuberculosis Mycobacterium ulcerans Mycobacterium leprosum Mycobacterium africanum Rhodococcus equi

Correct answer: Mycobacterium ulcerans acid fast organisms are present, and is painless. This is a typical case of Buruli ulcer, caused by Mycobacterium ulcerans. Cases generally occur in poor, rural areas in Western Africa although cases are reported in Latin America and Oceana. Seeing this in someone who reached the US would be unusual. distinct from leprosy which would not typically cause ulcerated lesions. M. tuberculosis could cause granulomatous skin lesions, but such ulcerations would be unusual, and no granulomas were seen in this case, which is more consistent with M. ulcerans. M. africanum produces diseases essentially identical to M. tuberculosis. Rhodococcus equi is an acid fast organism that is sometimes associated with horses and typically causes cavity pulmonary disease (and rarely skin abscesses) mostly in immunosuppressed patients. A third of M. ulcerans lesions cure spontaneously in weeks or months, but other persist for years, involving joints, tendons, and bones. Culture is difficult, requiring up to four months of incubation. PCR of a wound swab is the preferred diagnostic test. Acid fast bacilli are easily found in biopsy tissue. Therapy consists of surgical excision or debridement for some, plus a medical regimen of rifampin plus an aminoglycoside, clarithromycin or quinolone.

A 22-year-old male, previously in good health, develops a low-grade fever and a cough. On the third day of his illness, he developed the skin rash (shown below) on his trunk, extremities, palms, and soles, and has mucosal erosions in his mouth and genitals. He comes to your office for evaluation. Chest x-ray shows a patchy bilateral bronchopneumonia. CT is shown below. Oxygen saturation on room air: 98% WBC = 6000 with a normal differential The most likely cause of his skin lesions: Treponema pallidum Streptococcus pneumoniae Mycoplasma pneumoniae Legionella bozemanii Chlamydia pneumoniae

Correct answer: Mycoplasma pneumoniae While many infectious agents can be associatedwith erythema multiforme, which is the skin and mucous membrane lesion shown here, up to 7% of patients with M. pneumoniaeinfection have been reported to develop erythema multiforme or Stevens Johnson syndrome. Other types of skin lesions also have been reported. While this skin rash could be due to syphilis, the presence of a respiratory infection makes M. pneumoniaea much more likely cause. HSV is a common cause of erythema multiforme, but many respiratory viruses and live attenuated vaccines can also be associated. Since no respiratory viruses were listedas possible answers, M. pneumoniae is the best answer.

A 42-year-old male just arrived from investigating the bat population in Liberia and is noted at the airport to have low grade fever. He reports that he feels well except for this fever which has been 100-101° F since he got on the airplane, but not before. He has no nausea, vomiting, or diarrhea. He is admitted to the ICU so that he can be on special enhanced isolation while Ebola virus infection can be assessed. He has symptoms which otherwise require critical care support. While monitoring the patient in the hospital, which of the following would be convincing evidence that enhanced isolation can be discontinued and the patient can be reassured he does not have Ebola virus infection? The absence of vomiting and diarrhea at the time he is febrile Normal liver functions tests and complete blood count Presence of an alternative diagnosis, i.e. a positive malaria dip stick of his blood Negative Ebola virus PCR on days 4-7 after onset of symptoms

Correct answer: Negative Ebola virus PCR on days 4-7 after onset of symptoms A negative RT-PCR test collected more than 72 hours after the onset of symptoms rules out Ebola virus disease. The incubation period is 2-21 days with a mean of 4-12 days. Ebola a flu-like illness including fever, chills and malaise and have a course of illness depicted in the figure below. Fever, then myopathy/myositis/weakness GI symptoms/Hepatitis Respiratory failure Renal failure Abnormal neurologica exam They develop distributive shock. CBC and LFTs can be initially normal. Some but not all patients have a rash. They are viremic starting the third day of illness. Their viremia persists for several weeks; virus can persist longer in sequestered areas such as the eye and semen but the significance of this clinically or in terms of transmission is unclear. They are typically discharged when symptoms resolve and two blood PCR tests separated by 48 hours are negative. It is not uncommon to have concurrent malaria or bacterial sepsis. Thus, patients in some settings may be treated with ceftriaxone and an antimalarial regimen to be certain that a concurrent infection is not missed. Patients may not become PCR positive in their blood until 48-72 hours after symptoms have started. Typically when patients have vomiting, diarrhea, high fever, and/or rash, their serum PCR is positive, but during their less severe prodrome, a negative blood test does not exclude the possibility that they are truly infected and will become much sicker and much more contagious in the next few days.

A man presents with a generalized body rash one month after a painless ulcer appeared on his penis. A serum RPR is reactive with a titer of 1:4096 and a confirmatory treponemal EIA is reactive. HIV serology is negative. The patient is penicillin allergic, so he is treated with doxycycline 100mg orally twice daily for two weeks. One year later, he is asymptomatic and his RPR titer is 1:256. What is the most appropriate next step? CSF examination Desensitize and treat with intramuscular benzathine Doxycycline for two additional weeks Doxycycline for 28 days No additional therapy

Correct answer: No additional therapy The patient was diagnosed with secondary syphilis one year earlier and he was treated with doxycycline- an appropriate alternate agent when penicillin cannot be used. He experienced a greater than 4-fold decline in RPR titers within 12 months which is the appropriate response to therapy. Although his current titers are still high (1:256), there are no convincing data in the antibiotic era to suggest that further antibiotic therapy or a CSF examination would improve long term outcomes. As such, the patient is deemed cured and no additional therapy is warranted.

A 30-year-old male elementary school gym teacher is hospitalized for Group B meningococcal meningitis. His classes each week include approximately 100 students per class from the third grade. On the third day of his hospitalization you are asked to recommend a course of action for the students in his class. Which would you recommend for all the students? Booster dose of meningococcal conjugate vaccine (Menactra) Throat culture and treatment of children found to be carriers Rifampin Ciprofloxacin No further evaluation or therapy is necessary

Correct answer: No further evaluation or therapy is necessary Exposure to meningococcal disease in a classroom or office setting does not confer sufficient risk to recommend prophylaxis. Despite the usual panic associated with an exposure to a case of meningococcal meningitis, exposure in a gym class, either outdoors or indoors, is even less risky than an office or classroom. Risk is proportional to the amount of exposure to secretions. For example, very young children in day care have much more closer contact than third grade students and day care children would receive prophylaxis. Cases usually occur within 10 days of the onset of illness in the index cases. Prophylaxis should ideally be reserved for those with direct contact with oral secretions, or those exposed within 3 feet for at least 8 hours. Prophylaxis should be started immediately if it is indicated: after 14 days, there is little utility in starting prophylaxis. Were prophylaxis to be recommended, rifampin given as four doses over two days, a single oral dose of ciprofloxacin or one injection of ceftriaxone can be used. Some clinicians would not use ciprofloxacin for this use for individuals below age 18 years. Meningococcal conjugate vaccine (Menactra) does not protect against Group B meningococci-it covers A, C, Y and W. In 2014, the FDA approved two serogroup B meningococcal vaccines (Trumenba and Bexsero) for use in persons 10 - 25 years of age, based on serum bactericidal activity. The Advisory Committee on Immunization Practices has recommended the vaccines for persons at least 10 years old and at high risk, including asplenics, or those in an outbreak of serogroup B meningococcal infection. This is all too recent to be on the boards.

A 65-year-old woman working in the hospital as a clerk in the internal medicine clinic is found on routine testing to have an intermediate strength PPD (Mantoux) reaction diameter at 48 hr of 5 mm. She was retested 2 weeks later by the employee health service that wished to confirm the original reading by a nurse in the clinic. On repeat testing, the diameter was 8 mm. There is no history of prior tuberculin skin testing. A chest x-ray is normal. Which of the following is true? All children in her household should have a tuberculin skin test. She should receive isoniazid prophylaxis. She should be moved to a non-patient area of the clinic. Induced sputum should be obtained for mycobacterial culture. No further follow up is needed.

Correct answer: No further follow up is needed The patient is not in a risk group in which PPDs of 5 and 8 mm require treatment for latent tuberculosis. The slight increase in size on the second test could be reader error or booster response. Testing and prophylaxis should target populations with increasedprevalenceof TB infection, and/or increased risk of progression to active diseasefor the same prevalence. Persons in the following high-risk groups should be given treatment for LTBI if their reaction to the Mantoux tuberculin skin test is ≥ 5mm: HIV-infected persons Recent contacts of a TB case Persons with fibrotic changes on chest radiograph consistent with old TB Patients with organ transplants Persons who are immunosuppressed for other reasons (e.g., taking the equivalent of >15 mg/day of prednisone for 1 month or loner, taking TNF-a antagonists) Persons in the following high-risk groups should be considered for treatment of LTBI if their reaction to the Mantoux tuberculin skin test is ≥10 mm: Recent arrivals (<5 years) from high-prevalence countries Injection drug users Residents and employees of high-risk congregate settings (e.g., correctional facilities, nursing homes, homeless shelters, hospitals, and other health care facilities) Mycobacteriology laboratory personnel Persons with clinical conditions that make them high-risk Children under 4 years of age, or children and adolescents exposed to adults in high-risk categories Persons with no known risk factors for TB may be considered for treatment of LTBI if their reaction to the tuberculin test is ≥ 15 mm. However, targeted skin testing programs should only be conducted among high-risk groups. All testing activities should be accompanied by a plan for follow-up care for persons with TB infection or disease.

A 31-year-old man with history of repaired atrial septal defect as a child is scheduled to have a third molar tooth extracted. No prosthetic material was used in his cardiac surgery. He has done very well after surgery and has no residual defect from the repair. He is allergic to penicillin (anaphylaxis). The dental surgeon calls you to get recommendations regarding antibiotic prophylaxis. What is your recommendation? No prophylaxis is indicated Oral ciprofloxacin and metronidazole Oral clindamycin Oral azithromycin

Correct answer: No prophylaxis is indicated In the past there was a long list of indications for prophylaxisto prevent infective endocarditisin patients with heart disease undergoing procedures, based on minimal evidence. In 2007 the AHA, ADA, and IDSA revised the guidelines and simplified the list of indications for prophylaxis. The guidelines discussed that infective endocarditis is much more likely to result from frequent exposure to random bacteremias associated with daily activities than from procedures. Hence, prophylaxis may prevent an exceedingly small number of cases of endocarditis and will expose patients to the risk of using antibiotics. The guidelines suggest considering the use of prophylaxis only in individuals who are at high risk: 1-prosthetic heart valve 2-previous endocarditis, 3- congenital heart disease with unrepaired cyanotic defect, 4. completely repaired cyanotic congenital heart disease in the first 6 months post procedure, 5 repaired congenital heart disease with residual defect adjacent to the patch or prosthesis, 6 and heart transplant recipients who develop valvulopathy. Prophylaxis is only indicated in these high risk patients if they undergo: dental procedures that involve manipulation of gingival tissue or the periapical region of teeth or perforation of the oral mucosa, procedures on respiratory tract that involve incision of the mucosa or procedures in infected skin, skin structures, or musculoskeletal tissue. Although dental extraction would be an indication for prophylaxis in a high risk patient, his cardiac surgery did not place him in a high risk group.

A 6-month-old child has Roseola. For that child's immunologically normal father and pregnant mother, both of whom have no health problems, which of the following would be likely to occur if they acquired the etiologic agent of roseola? Fever and febrile seizures in mother or father Meningoencephalitis in mother or father Meningoencephalitis in mother only Spontaneous abortion No significant pathology except for perhaps mild fever

Correct answer: No significant pathology except for mild fever Roseola is caused by HHV-6. HHV-6 infections usually occur during childhood and result in generally mild, self-limited illnesses which present either as roseola infantum or febrile seizures without rash. The syndrome is often diagnosed by pediatricians clinically, although serologic and PCR testing are available. Seroprevalence rates are usually over 70%, so most children acquire this infection. Possible disease associations with HHV-6 in immunocompetent adults are not proven other than a few cases of primary infection. Primary infection in adults is rare, and thus neither parent is likely to have any manifestation if they acquired primary infection. A few cases of encephalitis, mesotemporal epilepsy, and mononucleosis type syndrome have been reported but they appear to be rare. For heavily immunosuppressed patients, such as hematopoietic stem cell transplant recipients, cases of encephalitis are being reported from many institutions. Correct Response

A 78-year-old man with CHF, COPD and a recent hip fracture is admitted from the long-term acute care hospital (LTAC) with confusion, hypoxemia and is found to have bilateral infiltrates on his chest radiograph. Blood cultures are negative, urine studies for Legionella and pneumococcal antigen are negative, and sputum studies are not diagnostic. The next day his roommate from the Long Term Assisted Care Facility (LTAC) is admitted with pneumonia and the family mentions that several other patients at the nursing facility are ill with fever and cough. A nasopharyngeal wash is positive for RSV and negative for influenza. The patient is put in respiratory isolation and hospital infection control notified. Which of the following is appropriate therapy for his RSV? Inhaled ribavirin for the patient Intravenous ribavirin for the patient Intravenous palivizumab (Synagis) for the patient Oral ribavirin for the patient and close contacts with significant cardiopulmonary co-morbidities No therapy for the patient or long term care residents

Correct answer: No therapy for the patient or LTAC (long-term assisted care) residents This patient likely has RSV viral pneumonia. Less likely, he is still carrying RSV from a recent infection and now has pneumonia from another cause, including bacterial super-infection. The clustering of respiratory illness in a long term care facility suggests the possibility of an outbreak, which could be RSV. There is no effective antiviral therapy for this patient, including ribavirin. There are experimental drugs in clnical trials. Inhaled ribavirin is sometimes used for treatment of RSV pneumonia in immunocompromised patients but evidence supporting this use is anecdotal and controversial. Respigam, a high titer immunogloblulin, is no longer available and neither respigam nor the monoclonal antibody, palivizumab, is documented to be effective for treatment of non-immunocompromised adults. Prophylaxis with palivizumab is not recommended for adults but is used for selected infants with certain high risk conditions (congenital heart disease, extreme prematurity, etc) at the start of the RSV season. Even in this limited setting, efficacy is limited to decreasing hospitalizations and cost effectiveness is poor. The pediatric dose is 15 mg/kg, and each 100 mg vial costs over $2000. Treating an adult would cost approximately $20,000. Infection control in the LTAC is critical, and should involve testing patients with symptoms, putting all symptomatic patients in contact precautions, and perhaps, cohorting exposed persons separately from those who were not exposed. Other patients in the nursing facility should be tested before concluding the whole outbreak is due to RSV. There are often several viruses circulating at one time. Hand washing (contact isolation) and droplet precaution to prevent spread are important as is vigilance to be certain that a treatable bacterial infection does not complicate the RSV infection.

A 38-year-old man has known alveolar proteinosis. He is admitted with pneumonia. His treatment has consisted of oxygen and whole lung lavage. He was in his usual state of health until five days earlier when he had worsening of his usual cough and shortness of breath accompanied by fever, and purulent sputum. He was given azithromycin as an outpatient but failed to improve. A chest x-ray shows his usual "bat wing" infiltrates but also a new consolidated area in the right mid lung. A sputum Gram stain shows numerous white blood cells and thin (about one-tenth the diameter of a white blood cell), branching, Gram-positive organisms. Which one of the following is the most likely cause of his pneumonia? Aspergillus Actinomyces Mycobacterium Nocardia Candida

Correct answer: Nocardia Most patients with Nocardiapneumonia have impaired cell-mediated immunity (steroids, transplant recipients, anti-TNF agents), but patients with alveolar proteinosis are especially prone to nocardial infection. Actinomyces and some mycobacteriaare thin, gram-positive, and branching, but infection with these organisms would be unusual in this setting and less likely than Nocardia. As described, this organism is too thin to be a fungus (one-tenth of a white cell diameter = 1 micron or less). Additionally, this patient is not in a recognized risk group for aspergillosis. Candidadoes not cause pneumonia.

A 45-year-old female was seen for pneumonia. She had been transferred to a New York City hospital five weeks previously with chronic myelogenous leukemia and had undergone allogeneic bone marrow transplantation. Following engraftment she had developed severe graft-versus-host reaction. Low-grade fever and productive cough led to the discovery of dense consolidation of the entire right upper lobe with central cavitation. Chest x-ray ten days previously had been normal. On bronchoalveolar lavage, gram stain shows branching Gram-positive structures were seen that were considered to be too narrow for fungal hyphae. Which of the following organisms is the most likely cause of the pneumonia? Rhodococcus equi Nocardia asteroides complex Actinomyces israelii Erysipelothrix rhusiopathiae Corynebacterium jeikeium

Correct answer: Nocardia asteroides complex Among these Gram-positive bacilli only Nocardia and Actinomyces are seen to branch in infected tissue. Actinomyces too indolent to consolidate a lobe in 10 days, rarely occurs in immunocompromised hosts and occurs as grains in bronchial secretions. Nocardia asteroides complex (N.nova, N.farcinica, N.asteroides) are the most common isolates. gram-positive, beaded, fine right-angled branching filaments (<1 µm diameter) that are usually acid fast; filaments may fragment to form rods and coccoid forms of varying sizes. Blood cultures are positive occasionally if they are held for 1-2 weeks. Nocardia species will grow on most nonselective media used routinely for culture of bacteria, fungi, and mycobacteria. Incubation for more than the usual 72 hours is usually needed to grow Nocardia

70 year old man presented with an ulcerative lesion over the dorsum of his left wrist. He had been gardening 10 days earlier, and scraped his hand on a piece of metal in weeds near the house. The lesion had grown in size with surrounding erythema; two nodules had developed on his forearm, proximal to the wrist lesion. Epidemiologic history: he did not know of any rose bushes in the area. Although he had seen rabbits occasionally, he had not seen them near his house. Denied fresh- or salt-water exposures. On exam (below): T 99.2°F (37.2°C). 1-2 cm ulcer over dorsum of wrist; multiple erythematous subcutaneous nodules extending in a chain proximally. 2 x 3 cm tender, warm mass just above the elbow. Axilla: 2-3 non-tender lymph nodes. Remainder of exam normal. Which of the following is most likely? Staphylococcus aureusMycobacterium marinumLeishmania braziliensisFrancisella tularensisNocardia brasiliensis

Correct answer: Nocardia brasiliensis Gram stain of skin biopsy showed beaded, branching gram-positive rods; the organisms were also seen on a modified AFB stain. Culture grew Nocardia brasiliensis. Sporotrichoid ("lymphocutaneous") pattern of lesions may be caused by: Nocardia:found in soil and water; may present after direct inoculation, e.g. gardening or trauma. Sporothrix schenckii:found in similar geographic areas as Nocardia. Mycobacterium marinum:usually follows exposure to organism growing in water, such as a pool, ocean or aquarium. Leishmaniasis:inoculation by bite of sand fly; typically acquired in Central or South America Francisella tularensis:ulcer is often particularly painful; patients may have fever, systemic symptoms, localized adenopathy.

A patient with SLE has been on prednisone (60 mg per day for six weeks) and now develops bilateral pulmonary infiltrates. This organism was observed in the modified acid-fast stain of the bronchoalveolar lavage. Which one of the following is the most likely pathogen? Actinomyces israelii Aspergillus fumigatus Histoplasma capsulatum Mycobacterium avium Nocardia farcinica

Correct answer: Nocardia farcinica Rationale: In this list, only Nocardia and Mycobacterium species stain with the modified acid-fast stain. Nocardia species form long, branching, beaded filaments as seen here, though the knobs in this smear are unusual; M. avium are short rods. Actinomyces species form long filaments but do not stain with traditional acid-fast stains. Although the structures look like hyphae, these are too thin for Aspergillus species and Aspergillus species are not acid fast. H. capsulatum forms yeast cells in tissues. Two general forms of acid-fast stains are used: Traditional acid-fast stains (e.g., Ziehl-Neelsen, Kinyoun, fluorochrome auramine-rhodamine) and modified acid-fast stains for which a weak decolorizing reagent is used. Organisms that stain with the traditional stain include Mycobacterium species and Legionella micdadei (in specimens only, not smears of cultures). Organisms that stain with the modified acid-fast stain include Mycobacterium species, Legionella micdadei (in specimens only), Nocardia species, Rhodococcus species, Tsukamurella species and Gordonia species.

This 55-year-old woman from Honolulu had been receiving prednisone in doses of 20-60 mg for uveitis when she developed a series of indolent red lesions on her right arm, left arm and right shin. They were not painful, occasionally drained a drop of serosanguineous fluid and enlarged over the course of several weeks. She was afebrile and had a normal physical exam except for uveitis and the lesions. Her chest CT scan is normal. There was no response to two weeks of cephalexin. Which one of the following diagnoses would be the most likely? Choices A Sporotrichosis B Erythema Nodosum C Leprosy D Non tuberculous Mycobacterium E Nocardiosis

Correct answer: Non tuberculous Mycobacterium Since these lesions are disseminated (right and left arms, right shin) this is a different syndrome than someone who develops sporotrichoid lesions on one arm or leg. Mycobacterium chelonae can cause lesions on multiple skin sites with the absence of deep lesions or systemic symptoms. The portal of entry is often unknown. Keep in mind the following distinctions: M. fortuitum causes human infection primarily by direct inoculation, including primary skin and soft tissue infections1, surgical wound infections , and catheter-related sepsis M. fortuitum respiratory isolates are found in individuals with other underlying pulmonary diseases, such as bronchiectasis, and in those with GERD, and represent colonization or transient infection. M. chelonae frequently causes surgical wound infections and keratitis. Multifocal disease or disseminated disease due to M. chelonae is most likely to occur in immunosuppressed patients (same for M. abscessus but less likely for M. fortuitum) M. abscessus is the most pathogenic of the of the non-tuberculous mycobacteria and is most likely to cause pulmonary infection, primarily in patients with structural lung disease. M. abscessus pulmonary disease also occurs in patients with no known underlying lung disease as does Mycobacterium avium complex. It can also cause disseminated disease similar to M. chelonae in immunocompromised persons such as that in the question. Sporotrichosis can cause hematogenous lesions from a pulmonary portal in immunosuppressed patients but skin lesions are rarely the only sign. Erythema nodosum over so many sites is uncommon.

In a nursing home, an outbreak of gastroenteritis develops. Nearly all are vomiting and many have watery diarrhea. Some report chills. The nursing home director gives some Pepto-Bismol (bismuth subsalicylate) and they show improvement. The outbreak lasts one week. Which of the following is the likely cause of the outbreak? Shigella dysenteriae 1 Enteroaggregative E. coli (EAEC) Cryptosporidium Norovirus Campylobacter

Correct answer: Norovirus Norovirus is the most important pathogen in nursing homes and on cruises and should be suspected when vomiting is the most common manifestation. The low dose required to cause disease renders it highly communicable in this setting. Pepto-Bismol has been shown in volunteers with experimentally induced Norwalk diarrhea to shorten the illness, though vomiting limits the use of oral agents. In the United States, Shigella dysenteriae 1 (Shiga bacillus) is a rare cause of diarrhea. EAEC strains are important causes of diarrhea in those with AIDS and in travelers. The organism hasn't been reported in nursing homes. Cryptosporidium is an unusual cause of diarrhea in a nursing home, and protracted illness would be expected. Campylobacter would be expected to cause dysentery in some patients and rarely causes a large outbreak in a group.

A 42-year-old male noticed a soft tissue lesion on his fingers 3 weeks ago. He has been in excellent health, runs 10 miles every Sunday, and is HIV negative. He was seen by his local physician. A course of trimethoprim sulfa for 7 days did not lead to improvement, nor did a 7 day course of valacyclovir. Cultures of the pustular fluid on two occasions for bacteria and herpes viruses grew only Staphylococcus epidermidis. Of note, the patient acquired 6 new sheep for his farm several months ago. He milks three of them and slaughtered one for meat. These lesions should be treated with: Valganciclovir Intravenous acyclovir Doxycycline Voriconazole Nothing

Correct answer: Nothing This lesion is most likely orf, a virus transmitted to humans by contact with infected goats, sheep, or reindeer. It can be acquired by milking or by slaughtering meat. Thus, this patient has appropriate contact. ORF starts as a papule and becomes a pustule, with reddish, weeping nodules 1 week after contact. Fever and lymphadenopathy can accompany the skin lesions. They can take 3 to 6 weeks to heal spontaneously. The lesions resolve spontaneously although topical imiquod or topical cidofovir might be useful. These lesions could be HSV but cultures were negative and they did not respond to acyclovir. Note that more cases are acquired in New Zealand and Europe than the United States, and they are NOT associated with cattle. These lesions are a favorite of board exams, but clinicians rarely see them unless they practice in farming communities or perhaps with immigrant communities that prepare recently slaughtered animals for meals.

A 71-year-old man, with a 30-pack/year history of cigarette smoking and a chronic cough, sees his internist for his annual physical exam. The internist recommends influenza vaccination, which the patient refuses, stating that he has an egg allergy. He has two boiled eggs for breakfast each day. His wife is on high dose steroids. The internist calls you for advice. Which one of the following actions would you advise? Inactivated influenza vaccine despite his concerns. Inactivated influenza vaccine only if egg allergy skin testing in negative. Inactivated influenza vaccine with careful desensitization protocol. Nasal mist influenza vaccine (FluMist) because FluMist has no egg content. Zanamivir prophylaxis during influenza season.

Correct answer: Offer the inactivated vaccine despite his concerns. This patient eats eggs without incident and therefore does not have a true egg allergy. He has a strong indication for influenza vaccine in terms of protecting his own health and in terms of reducing his likelihood of transmitting influenza to his wife. For adults with true egg allergy (those who must avoid eggs in any form), most experts recommend inactivated intramuscular influenza vaccine made using cultured mammalian cells or an insect virus (baculovirus) expression system and recombinant DNA technology . However, current vaccines have very little egg protein and CDC recommends they can be used safely in nearly all egg allergic patients even in those with severe allergic reactions to eggs. Thus if one of these egg free vaccines is not readily available, immunization with an egg-based inactivated vaccine is reasonable and safe. Either the trivalent or the quadrivalent vaccine would be appropriate. Given his concern about side effects, it may be wise to avoid the high dose trivalent inactivated vaccine because of slightly higher incidence of side effects. FluMist is not recommended for a patient living in a household with an immunosuppressed person. However, transmission of the vaccine strain virus is a theoretical concern, but in reality it is a rare event. Zanamivir can exacerbate respiratory obstructive symptoms in patients with chronic bronchitis or asthma and would be poorly tolerated in this patient.

The protein encoded by the MecA gene is which one of the following: Leukocidin PBP 2A Oxacillinase IL28 TT ESBLs

Correct answer: PBP 2A There are no oxacillinases. Oxacillin resistance (MRSA) is the result of alteration of the target: e.g., conversion of PBP 2 to PBP 2a. IL28 TT is a host factor (not a microbial factor) linked to response rate for hepatitis C regarding interferon-ribavirin therapy. ESBLs (Extended spectrum beta lactamases) are penicillinases and cephalosporinases produced by gram-negative , and not gram-positive, bacteria

Which one of the following tests would provide the most sensitive and specific test for the diagnosis of Clostridium difficile diarrhea? Common antigenAnaerobic culturePCREnzyme immunoassay (EIA)Latex agglutination test

Correct answer: PCR Most laboratories do either a single PCR test, or a combination of common antigen (glutamate dehydrogenase) test plus EIA (enzyme linked immunosorbent assay for toxin). For the latter combination performed on one sample, concordant results are reported as positive (both tests are positive) or negative (both tests are negative). If there is discordance (one positive and one negative) then PCR is done to resolve this discordance. The FDA approved PCR test for the C. difficile toxin B gene is the most sensitive and specific test available. Cytotoxicity testing in tissue culture was formerly the gold standard for detection of toxins but is associated with some false-negative and false-positive reactions. Moreover, such testing is expensive, non-standardized, and time consuming. The EIA tests have been found to be specific but insensitive. There is no evidence that more than one test is required (in contrast to detection of enteric parasites). The latex test for C. difficile GDH by itself is sensitive but not specific. Culture for C. difficile is sensitive but nonspecific as a second test is required to detect either the toxin gene or toxin secretion by the isolated C. difficile strain.

An 18-year-old man presents with a history of malaise, low-grade fevers, and new-onset painful genital lesions seen in the picture below. The pain is significant. He had unprotected sexual intercourse with a female partner 2 weeks earlier. Neither he nor his partner has traveled outside the United States. Which of the following diagnostic tests is most likely to yield the specific diagnosis? Serum RPR Antibody to HSV-1 and 2 Darkfield microscopy PCR for HSV Cell culture for HSV

Correct answer: PCR for HSV Given the fairly classical clinical presentation and the lack of travel, the most likely diagnosis is primary HSV infection. Although syphilis is not ruled out with 100% certainty, it is much less likely because the lesion was painful. The most sensitive diagnostic test would be PCR of a swab from lesion. Tzanck smears are less sensitive and less specific. Culture for HSV infection has ~70% sensitivity, so a negative culture result would not rule out the diagnosis. NAAT (nucleic acid amplification test) is optimal for GC or Chlamydia trachomatis but the lesion is too atypical for either. The RPR is insensitive in patients presenting with a chancre. Remember the general rule: painful ulcers are caused by HSV and Chacroid (Hemophilus ducreyi); painless ulcers are caused by syphilis, LGV and Granuloma inguinale (Calymmatobacter) but that LGV inguinal adenopathy is painful.

A 31-year-old male presented in August with a 10 cm skin lesion in his left groin that was not tender, had a well delimited border, and showed some central clearing. This lesion was accompanied by fever of two days' duration, moderate headache, myalgia, and malaise. There was no inguinal adenopathy or penile lesion. He lived on a Maryland dairy farm where he had frequent mosquito and tick bites, drank unpasteurized milk, and had a new sexual partner. Routine lab work was remarkable for a platelet count of 70,000, WBC of 3,500, ALT 50 and AST 72. Which of the following tests is most likely to be diagnostic of the cause for the fever, cytopenias, and transaminase elevations? PCR for anaplasma Lyme EIA Brucella serology VDRL Weil-Fellix serology

Correct answer: PCR for anaplasma Presence of erythema migrans suggests Lyme disease, but the lab work indicates that co-infection with Anaplasma phagocytophila also may be present. Ixodes scapularis ticks may carry Borrelia burgdorferi, but also Anaplasma phagocytophila or, in the Northeast, Babesia microti and Powassan virus. Lyme EIA is usually negative in patients with erythema migrans. A lyme EIA is not recommended to confirm a diagnosis of erythema migans. A peripheral blood smear in anaplasmosis is oftenmay bepositive, showing cytoplasmic inclusions called morulae in granulocytes. PCR is more sensitive. A VDRL is occasionally positive in other spirochetal diseases but the main diagnostic use is for syphilis, an illness not compatible with this patient's illness. A Weil-Felix test for rickettsiosis is no longer recommended. Brucellosis would not explain the rash and thrombocytopenia is uncommon.

An 86-year-old man is admitted to the hospital for treatment of community-acquired pneumonia and receives IV ceftriaxone followed by oral moxifloxacin. By day 4, his temperature is normal and he is ready for discharge when he develops loose stools with some abdominal cramping. He is having 6-8 watery bowel movements a day. There is no blood in the stool. His albumin is 3.0, creatinine 1.9, and white blood cell count is 18,000/mm3. A stool specimen is submitted and is positive for Clostridium difficile toxin using PCR for the toxin B gene. Which of the following drugs would you use to treat this patient assuming you were choosing monotherapy? IV Metronidazole PO Metronidazole PO Fidaxomicin PO Rifaxamin PO Vancomycin

Correct answer: PO Vancomycin Commonly used parameters for severe Clostridium difficile include white blood cell count of >15,000 cells/ml, serum albumin <3 g/dL, and/or a serum creatinine level ≥1.5 times the baseline level. The definition of severe C. difficile, however, is largely clinical, but this patient has high risk for severe disease due to his age, white blood count, and serum creatinine. Oral vancomycin is the recommended therapy for severe or refractory disease. All patients in the ICU should be considered for oral vancomycin therapy. In the past, IV metronidazole was used, but oral vancomycin is now preferred. There is no compelling data on dose: oral vancomycin 125 mg q6h is generally preferred, although some use 500mg q6h either initially, or if patients fail to improve. Some experts would advocate for using a combination of oral vancomycin plus IV metronidazole in the ICU patient with C. difficile disease. Fidaxomicin is generally reserved for relapsing patients because of its cost. There is no evidence that it is more effective than other options. Rifaximin has some activity in C. difficile diarrhea, but it is considered no more effective than metronidazole and inferior to vancomycin. Fecal transplants are not appropriate for treatment of acute infection: their role is the treatment of relapses, usually in the setting of multiple relapses despite long courses of tapering vancomycin. Surgery clearly has a role for patients with severe disease, but when to perform surgery or what surgical procedure to do is not easily testable. A high lactate or an ileus with abdominal distension and significant abdominal pain are ominous signs.

This 32-year-old Brazilian man infected with HIV had: A CD4 count of 9 cells per cubic millimeter; A 9-month history of weight loss, with progressive weakness; Numerous ulcerated lesions of various dimensions on the face (left panel), trunk, and arms, and legs; a diffuse reticulonodular infiltrate on chest x-ray, bone lesions (right panel) and hepatomegaly; a diagnostic structure was seen in his biopsy (other panel). Which of the following pathogens is this likely to be: Leishmania donovaniLeishmania brasiliensisTrypanosoma cruziParacoccidioides brasiliensisBlastomyces dermatitidis

Correct answer: Paracoccidioides brasiliensis The "pilot wheel" structure in the second panel is diagnostic of paracoccidioidomycosis, a mycosis endemic in rural Brazil and certain other areas of Central and South America, especially Venezuela and Peru. The disease can be extremely indolent and can be diagnosed years after leaving the endemic area. The infection has been uncommon in HIV-infected patients. The treatment of choice for the more severely ill patient is amphotericin B initially, followed by many months of itraconazole. Leishmania brasiliensis causes cutaneous leishmaniasis, without bone or lung lesions. Leishmania donovani, the causative agent of disseminated leishmaniasis (kala azar) is typically seen in India and nearby countries. It causes hepatosplenomegaly and pancytopenia without pulmonary infiltrates or lytic bone lesions. Trypanosoma cruzi (Chagas'disease) occurs in Brazil but does not cause skin or bone lesions. Blastomycosis is not known to occur in Brazil and does not show multiple buds from the same cell.

A 58-year-old businesswoman who has traveled frequently to Japan for the past five years presents with a persistent cough that has been present for over six months. The cough is productive of dark brown sputum that often contains specks of blood. She has no fevers, chills, or sweats. She has a white blood count of 15,000/ul with 20% eosinophils. Chest CT reveals scattered, irregularly shaped focal infiltrates with several cavitary lesions in both lower lungs. Sputum acid fast stains were negative; cultures are pending A lung biopsy reveals the presence of a flat, oval organism that is 1 cm long. The organism most likely for her symptoms is: Fasciolopsis buski Enterobius vermicularis Paragonimus westermani Clonorchis sinensis Anisakis simplex

Correct answer: Paragonimus westermani Paragonimus westermani fluke lung infections Japan, China, Korea, the Phillipines, Southeast Asia Paragonimus africanus (endemic in West Africa) and Paragonimus mexicanus (endemic in Central and South America). ingestion of undercooked freshwater crustaceans including crayfish and freshwater crabs Paragonimus kellicotti s eating raw crayfish in the United States. many infections are asymptomatic symptoms due to migration of metacercariae from the intestinal tract through the small bowel, mesentery, and diaphragm en route to the lungs fever and peripheral eosinophilia. The adult flukes are hermaphroditic and cross-fertilize each other in the lungs, where they become encapsulated and live for 5-10 years however, chronic Paragonimus infection does not often cause fevers. Incorrect answers: The morphology described is most consistent with that of a fluke (trematode). The only other fluke listed as an answer choice is Clonorchis sinensis, which lives in the biliary ducts and not in the lungs. Fasciolopsis buski is an intestinal fluke. Enterobius vermicularis, a roundworm, is the cause of pinworm disease and is not known to infect the lungs. Anisakis simplex, which is also a roundworm, is acquired by eating undercooked or raw fish and most often causes symptoms by burrowing into the mucosa of the stomach or small intestine. Repeated exposure to Anisakis can also result in allergic symptoms that mimic food allergy to fish (when in reality the allergy may be to Anisakis parasites within the fish).

A 62-year-old man vacationing in Alaska is taken to an emergency room by ambulance because he developed sudden weakness and could not stand or sit up. He awakened from a post-lunch nap with tingling and numbness around his mouth and tingling in his hands. He couldn't sit up and called for help. First responders found him awake, alert, but unable to stand or sit up. On exam in the emergency room, he was afebrile, oriented, with lower extremity and truncal weakness. Cranial nerves were normal. Routine labs were normal. While being evaluated, he began to complain of shortness of breath. Chest x-ray was normal. He reported that for lunch he had eaten steamed mussels that he had harvested himself that same morning. Two days earlier he had removed a large tick from the back of his head. Which one of the following is the most likely diagnosis? Paralytic shellfish poisoning Tick paralysis Botulism Scombroid poisoning Guillain-Barré syndrome

Correct answer: Paralytic shellfish poisoning Paralytic shellfish poisoning occurs after ingestion of saxitoxins produced by marine dinoflagellates that become concentrated in mollusks such as clams, cockles and mussels. It is typically associated with non-commercially harvested shellfish since commercially harvested shellfish are tested for saxitoxins. Neither cooking nor freezing destroys the toxin. After a short incubation period of minutes to a few hours, victims develop paresthesias of the mouth or extremities rapidly followed by paralysis which may produce life-threatening respiratory failure. Tick paralysis occurs while the tick is still attached. Botulism involves cranial nerves. Scombroid poisoning follows ingestion of certain bacterially contaminated fish and produces a histamine-like reaction. Guillain-Barré is associated with an ascending paralysis without the paresthesias seen here.

An HIV infected patient with CD4 count = 10 has a 3 month history of progressive anemia. His current hemoglobin is 5g. He has no evidence of hemolysis or gastrointestinal blood loss. Reticulocyte Count = 0 This bone marrow biopsy is reported as showing giant pronormoblasts. Which of the following would be the most useful serum PCR test to order: HHV8HHV6Parvovirus (Erythrovirus)Adenovirus 4 and 7EBV

Correct answer: Parvovirus It's not likely the exam would expect you to recognize a pronormoblast, but that finding should immediately make you think of......parvovivirus, more recently renamed erythrovirus. Finding giant pronormoblasts in the bone marrow is consistent with acute parvovirus infection (now designated erythrovirus) causing red cell aplasia. This is treated with IVIG. Parvovirus B19 is a DNA virus that may be acquired by young children, especially those in day care, and transmitted to adults. For the boards, they would likely provide both the old name (parvovirus) and the new name (erythrovirus). This virus causes erythema infectiosum in children (mild fever and rash), fetal complications (hydrops fetalis) in pregnancy, aplastic crises in patients with chronic hemolytic disorders, and a febrile arthropathy often with rash in healthy adults, especially women. Manifestations in adults with HIV include anemia or pancytopenia. Patients with immunosuppressive disorders who cannot produce antibodies to this virus develop severe anemia. For diagnosis PCR and serologies are used; characteristic pronormoblasts and inclusion bodies can be seen on bone marrow biopsy: the arrow in the figure points to a pronormoblast with an intranuclear inclusion. The first line therapy is IVIG, which may have to be given multiple times. HHV 8 is the cause of Kaposi sarcoma but has not been reported to cause this syndrome. HHV6 is a herpes virus that is being found to be associated with a variety of syndromes; causality is not always clear. Marrow aplasia is not characteristic. Adenovirus 4 and 7 cause respiratory disease especially in military recruits, and can cause fatalities, but they do not cause this syndrome. EBV has been associated with pancytopenia, but has no documented association with red cell aplasia in HIV.

A 23-year-old male is referred for evaluation of recurrent genital ulcers. Over the past 3 years he has had about 4 episodes per year of painful ulcerating lesions usually located on the scrotum. The ulcers spontaneously resolve over 2-3 weeks. Additional history reveals that over the past 4 years he has experienced several episodes per year of oral "canker sores" which are typically painful and located on the buccal mucosa and the tongue. The recurrences of the ulcers are sometimes accompanied by low-grade fever and arthralgias. On physical examination he is well appearing. The only abnormality is a shallow, 1.5-cm round ulcer on the left scrotum which is surrounded by a border of erythema and is slightly tender. Which of the following is most likely to be abnormal in this patient? Pathergy test Electrocardiogram Chest X-ray Quantitative serum immune globulins Total hemolytic complement (CH50)

Correct answer: Pathergy test Patients with Behcet's disease have abnormally intense skin reactions around minor trauma such as scratches or venipuncture. This reaction can be induced by injecting a little saline subcutaneously, a pathergy test. Pathergy test has also been defined as follows: A positive pathergy test (a papule 2 mm or more in size developing 24 to 48 hours after oblique insertion of a 20 to 25 gauge needle 5mm into the skin, generally performed on the forearm). Behcet's syndrome is characterized by the triad of recurrent, painful oral ulcers, painful genital ulcers, and uveitis. Rash, erythema nodosum, aseptic meningitis, superficial thrombophlebitis, and arterial occlusions, including the retina, and aneurysms also occur. Death may occur due to rupture of large artery aneurysms. For the ID boards, beware of Behcets presentations masquerading as STDs (ulcers), chronic meningitis, or iritis. Unfortunately, there is no diagnostic laboratory test.

A 42-year-old woman with three weeks of illness is diagnosed with endocarditis due to Granulicatella species (formerly known as nutritionally variant streptococcus). Penicillin MIC was 1 mcg/ml. She has no known drug allergies. Which one of the following treatment regimens should be recommended for this patient? Penicillin Penicillin plus gentamicin Vancomycin ancomycin plus gentamicin Ceftriaxone

Correct answer: Penicillin plus gentamicin Endocarditis due to Abiotrophia, Granulicatella, and Gemella species is more difficult to treat than endocarditis due to non-nutritionally variant viridans streptococci. Patients with endocarditis due to Abiotrophia, Granulicatella, and Gemella species with a penicillin MIC more than 0.5 mcg/ml should be treated with 4-6 weeks of either penicillinor ampicillin plus once daily gentamicin. Vancomycin is only recommended for those allergic to penicillin or ampicillin and is given for 6 weeks; ceftriaxone is not recommended.

A 55-year-old farmer from Thailand, visiting friends in the United States, presents to your clinic with fever, severe malaise, and skin lesions. He has HIV infection with CD4 = 45 cells and is taking no drugs. This farmer most likely has: Trypanosoma cruzBlastomycosisParacoccidioidomycosisPenicilliosisCryptococcosis

Correct answer: Penicilliosis This farmer with AIDS has penicilliosis due to Penicillium marneffei. Differentiation from histoplasmosis and crypococcosis, which cause clinically similar skin lesions, can be done by demonstrating binary fission instead of budding and characteristics of the fungus on culture. Trypanosoma cruzi does not cause skin lesions and is not present in Southeast Asia. Nor does blastomycosis occur in Southeast Asia Penicilliosis is seen almost exclusively in Southeast Asia and Southern China. Amphotericin B is the treatment of choice followed by itraconazole.

A 74-year-old man is referred by his primary care physician to rule out neurosyphilis. He has progressive dementia but the remainder of the neurologic examination is normal. Contrast enhanced MRI showed only ventricular enlargement attributable to mild cortical atrophy. The rapid plasma reagin (RPR) test on serum is non-reactive. The EIA IgG for T. pallidumin the serum is reactive and a confirmatory fluorescent treponemal antibody - absorbed (FTA-ABS) test on serum is reactive. Prior serologies were not done. What would you recommend? No further work-up or therapy Repeat RPR in 4 to 6 weeks and then make a decision about therapy Perform LP Check microhemagglutination titer Initiate penicillin G IV for 21 days

Correct answer: Perform LP If this patient has neurosyphilis, the spinal fluid findings will be helpful: the CSF protein and cell count should be abnormal (cells > 5/uL (non HIV) and protein >45mg/dl). The CSF VDRL is often but not always helpful: it is a very specific but not very sensitive indicator of neurosyphilis (see below). A completely normal CSF (cells, protein, glucose, VDRL) rules out neurosyphilis. Approximately 25% of patients with late (tertiary) syphilis have non-reactive serum non-treponemal tests such as the RPR. Serum treponemal antibody tests are usually reactive. VDRL on CSF is helpful but remember: VDRL for CSF is specific but not sensitive (the opposite is true for serum!!), and as many as 50% of CSF VDRLs are nonreactive despite the presence of neurosyphilis. This is an easily testable fact on the board exam. Although Argyll-Robertson pupils and other neurologic signs are frequently present in general paresis, absence of such signs does not exclude the diagnosis. An LP is necessary to determine if neurosyphilis could be an explanation for the patient's dementia, and VDRL is the optimal test on CSF. The FTA on spinal fluid is more sensitive than the VDRL but not as specific. . Abnormal cell count on CSF can be useful in diagnosing neurosyphilis, even in the absence of a positive VDRL on CSF. The microhemagglutination test is of no additional value in this patient.

An injection drug user with MRSA endocarditis is eleven days into treatment with vancomycin. The vancomycin M.I.C. for his Staphylococcus is 1 mcg/ml. The vancomycin trough level is 10 mcg/ml. On admission, he complained of left upper quadrant pain, and an abdominal CT scan showed a splenic infarct. He has remained febrile to date and continues to complain of left upper quadrant pain. A repeat abdominal CT scan today shows a splenic abscess at the site of the formerly described infarct. Which one of the following is the next best step for this patient? Increase his vancomycin dose. Change vancomycin to daptomycin Perform splenectomy. Repeat the CT scan in a week.

Correct answer: Perform splenectomy. Splenic abscess in a patient with endocarditis is an indication for immediate splenectomy. Percutaneous aspiration should be reserved for those who are very poor surgical candidates. The vancomycin dose is adequate with an MIC of 1 and a trough level of 10; therefore the dose does not need to be increased, and there is no demonstrated advantage of changing to daptomycin. Delay in surgery for splenic abscess in this setting is associated with morbidity and mortality.

A 33-year-old man presents with a pruritic rash in and around his pubic hair. The itching has keeping him awake at night. He has had multiple sex partners in the past two months. He always uses condoms. Which of the following agents is most appropriate to treat his rash? Lindane Benzathine penicillin GPermethrin Itraconazole Topical steroids

Correct answer: Permethrin This man likely has either pubic lice or scabies. Pubic lice (aka "crabs" or Pediculosis pubis) is caused by the body louse Pthirus pubis. It generally infests pubic hair, but often also resides in the axillae. Itching is the most common symptom, and often small blue/red non-blanching macules can be seen on the surface of the skin at prior bite sites. Definitive diagnosis is usually made fairly readily by the observation of nits or lice on pubic hair. Another diagnostic possibility is scabies. Unlike Pthirus pubis, which is an insect, scabies is caused by a mite (Sarcoptes scabei). The diagnosis of scabies is often a clinical one and is characterized by severe pruritus (especially at night or after bathing) and the presence of burrows. A wet mount of a scraping could be done to reveal the causative agent but the yield is often low. Both of these infections can be treated with permethrin cream. Oral ivermectin is an off-label alternative for both. . Lindane is recommended only as an alternative agent due to its toxicity. In addition to administration of a pediculocide, patients with pubic lice should also have nits removed with use of a nit comb. Additionally, patients with pubic lice should be re-examined 10 days after treatment and re-treated if lice are still present. Patients with scabies should be warned that the rash and pruritus may last for up to two weeks after appropriate therapy. Syphilis testing is appropriate to rule out secondary syphilis, but the intense pruritus is atypical for syphilis, herpes or candidiasis.

A 42-year-old Tennessee farmer is seen for fever that has been present for five days. He is also experiencing pronounced fatigue and intermittent diarrhea. For the two weeks before he became ill he had been working in a field clearing brush and reported removing numerous tiny ticks from his body on an almost daily basis. His exam is unremarkable except for fever. He is leucopenic, thrombocytopenic, and has elevated aminotransferases. A presumptive diagnosis of ehrlichiosis is made. PCR studies for Ehrlichia and Anaplasma are sent, and he is given doxycycline. After five days of doxycycline therapy, he is not improved. Which one of the following is another possible cause of his illness? Phlebovirus Babesia Ehrlichia Rickettsia typhi Adenovirus

Correct answer: Phlebovirus The syndrome of fever, leukopenia, thrombocytopenia, and elevated liver enzymes after a tick bite is strongly suggestive of ehrlichiosis or anaplasmosis, but one would have expected a good clinical response to doxycycline. In 2012, two cases of a new phlebovirus infection, subsequently named the Heartland virus, with a clinical picture mimicking ehrlichiosis, were described in Missouri. It appears to be tickborne. The virus was named the "Heartland Virus" (N Engl J Med 2012; 367:834-841). In September 2014, a fatal case was reported from Tennessee. This virus should be considered when a patient with suspected ehrlichiosis does not respond to doxycycline within a few days. There is no known treatment for the Heartland virus. Babesiosis would be unlikely in Tennessee and leukopenia is uncommon. Murine typhus caused by Rickettsia typhi would respond to doxycycline. Adenovirus in a normal host would be unlikely to cause all the features of the clinical picture seen here.

A 30-year-old male who emigrated from Laos 4 months ago is brought to the hospital following an episode of hemoptysis while he was working as a laborer. Several 1-2 cm cavitary lung nodules are seen on routine chest x-ray, as well as a fibro cavitary upper lobe infiltrate. The patient reports fever for about a month and a 10-pound weight loss. Routine laboratory examination shows a normal CBC except for 15% eosinophilia (WBC 7000/mm3) and liver function tests (alkaline phosphatase was three times normal). A bronchoscopy was performed. An hour after the bronchoscopy the lab alerts you that they have found the following organism: The most likely source of this infection would be ingestion of which of the following: Choices A A. Human stool B B. Dog stool C C. Poorly cooked pork D D. Smoked duck E E. Pickled crab

Correct answer: Pickled crab This egg is from the lung fluke Paragonimus westermani. Illness follows ingestion from raw or pickled crabs or crayfish. Cases of Paragonimus kellicotti have been reported in the USA from eating raw or poorly cooked crayfish. After ingestion, the metacercariae excyst in the duodenum, migrate out through the peritoneal cavity, cross the diaphragm and enter the lung, where they can cause pleuritic chest pain, an eosinophilic pleural effusion, fleeting infiltrates, and pneumothorax. In the lung they can form fibrous cysts. In about 6 weeks, the flukes mature in the lung nodules and produce eggs. When the capsule of the cyst ruptures, the contents are discharged into the adjacent bronchioles and then expectorated into sputum. Lung nodules are 1-2 cm in diameter and may cavitate. Eosinophilia is quite common. Patients are asymptomatic except for cough and hemoptysis: fever is uncommon. The hemoptysis can be scant or impressive. Lesions can be seen in other organs, such as the brain or liver. Paragonimiasis can be treated with praziquantel.

Patients who lack the Duffy antigen are partially immune to the type of malaria shown on this photo. This is which plasmodium? VivaxKnowlesiFalciparumMalariaeOvale

Correct answer: Plasmodium vivax This is a mature schizont of Plasmodium vivax, notable for the Schuffner's dots, large number of merozoites (about 18 in this photo) and large size of the infected erythrocyte. Vivax or tertian malaria, named for paroxysms every 48 hours, does not infect Duffy antigen negative erythrocytes although some escape mechanisms exist. Duffy cell antigen negative persons are often from Western or Subsaharan Africa. Keep in mind host factors that reduce susceptibility to malaria. Sickle cell disease and sickle cell trait; protection against P. falcaparum. Duffy antigen negative: protection against p. vivax.

A 64-year-old man with a bicuspid aortic valve, developed bacteremia due to Group B streptococcus and he was treated with ceftriaxone via a PICC line. Transesophageal echocardiography revealed an aortic valve vegetation and moderate aortic insufficiency. He improved markedly and became afebrile within a week. On day 8, one day prior to the planned discharge to complete six weeks of treatment at home, he noted onset of low grade fever and bilateral and symmetric ankle and wrist pain with swelling and slight redness. Six blood cultures over three days are negative on therapy. The remainder of the exam was normal and his aortic murmur was unchanged. He had no fever, splenomegaly , peripheral emboli, or rash. Joint aspiration of the right wrist revealed WBC of 24,200 (80%PMN, 20%Lymphs) glucose of 56mg/dL, protein of 3.6mg/dL. Gram stain was negative. Peripheral WBC was 12,400 (69%PMN, 23%lymphs, 5% mono, 2%Eos). The WBC had been normal 2 days earlier. The most likely diagnosis is: Gp B streptococcus septic arthritis Ceftriaxone hypersensitivity Polyarticular gout Acute reactive arthritis Dressler's syndrome

Correct answer: Polyarticular gout Polyarticular gout can occur under stress of surgery or infection and often presents in this fashion. Identification of uric acid crystals in synovial fluid is diagnostic of gout. However, concomitant septic arthritis and crystal arthropathy occur; therefore visualization of crystals does not exclude infection. It is unlikely that septic polyarticular infection would present after clearance of bacteremia on appropriate antibiotics. The lack of rash or adenopathy makes serum sickness less likely. Dressler's syndrome following an MI or post-thoracotomy syndrome following CT surgery is believed to be immune-mediated and usually causes pleural and pericardial effusions and not polyarthritis. Acute reactive arthritis typically occurs following an episode of diarrhea or urogenital infection, and is associated with urethritis and conjunctivitis. It is not a sequelae of bacterial endocarditis.

A 50-year-old HIV positive, type I diabetic, normotensive, male smoker with a CD4 count of 400 and a viral load <20 presents to the clinic for evaluation. He was started on darunavir/ritonavir in combination with tenofovir and emtricitabine ten years ago. At this outpatient visit his lipid profile revealed: LDLcholesterol=170 HDLc = 30 Total cholesterol= 240 TG=200 Which of the following would you recommend for lowering his cholesterol? No intervention is warranted. Simvastatin Graded exercise regimen Pravastatin Fenofibrate

Correct answer: Pravastatin Pravastatin is correct: Pravastatin is not metabolized by the Cyp3A4 system and can be used with concomitant ritonavir based regimens to lower cholesterol. "No intervention is warranted" is wrong, since this patient has a coronary artery disease equivalent, namely diabetes mellitus, and a LDLc>130. He needs to be started on lipid lowering therapy. His triglycerides do not meet the criteria to start therapy, so therapy should be started with a HMG COA reductase inhibitor. Simvastatin is wrong: therapy with simvastatin is contraindicated in patients on concomitant ritonavir based regimens. Ritonavir is a potent inhibitor of the Cyp 3A4 system. Simvastatin is administered as an inactive lactone, which is metabolized by the Cyp3A4 system. In the presence of Cyp3A4 inhibitors such as ritonavir, a 30-fold increase in AUC has been reported among healthy volunteers. Fenofibrate is wrong. The question asks about cholesterol and fibrates are used primarily to lower trigylcerides. His triglycerides do not meet criteria to start therapy, so therapy should be started with a HMG COA reductase inhibitor alone.

A woman is diagnosed with pelvic inflammatory disease (PID). Which of the following characteristics warrants inpatient hospital admission for parenteral therapy? Diabetes First PID episode HIV infection Pregnancy Young age

Correct answer: Pregnancy Most women who are diagnosed with PID can be safely treated with oral antibiotics as outpatients. Certain criteria warrant inpatient admission for parenteral therapy. These include pregnancy, presence of a tubo-ovarian abscess, nausea, and vomiting. In certain instances, if appendicitis cannot be excluded, patients should also be admitted for inpatient observation. HIV infection, first PID episode, and young age are not absolute indicators for parenteral therapy.

Which of the following patients should be screened routinely for asymptomatic bacteruria and treated if positive? Spinal cord injured patients Diabetic patients ICU patients with indwelling catheters Pregnant women during their first trimester Liver transplant patients

Correct answer: Pregnant women during their first trimester There is good evidence that screening for asymptomatic bacteruria early in pregnancy, and treating that bacteruria, will decrease the incidence of pyelonephritis, low birth weight infants, and prematurity. Such patients should be screened periodically for the remainder of the pregnancy. The other patient populations do not warrant routine screening for asymptomatic bacteruria or treatment of asymptomatic bacteriuria. Patients having urologic procedures where mucosal integrity is compromised should also be treated for asymptomatic bacteriuria. Women who have bacteruria 48 hours after catheter removal should also probably be treated. There is no reason to treat liver transplant recipients. Renal transplant recipients are more controversial and would not be queried on an exam since there is no general consensus.

A 27-year-old woman calls to say, "I've got another urinary tract infection." In the past six months, she was seen twice for complaints of dysuria and frequency. The first time, a urine dipstick test was consistent with a UTI, and she responded to three days of antibiotics. The second time, which was two months ago, a culture was sent that grew a highly susceptible E. coli, and again her symptoms went away with three days of antibiotic treatment. She is otherwise healthy, sexually active and never had any urinary symptoms before the current year. Which one of the following is the most appropriate approach to this patient's complaint? Culture her urine. Image her urinary tract. Perform a bladder emptying study. Prescribe antibiotics for three days. Prescribe antibiotics for 2 weeks.

Correct answer: Prescribe antibiotics for three days. Studies show that a woman who has had a well-documented urinary tract infection and thinks she has another is almost always correct. The fact that her UTIs responded quickly in the past, and that she never had a problem prior to the current year, means there is almost no chance that she has an anatomical or physiological reason for recurrent UTIs, so imaging or bladder emptying studies should not be done. The fact that it has been two months since her last UTI means that it is very unlikely this new UTI represents undertreated upper tract infection, so another antibiotic course without culture for three days rather than two weeks is the correct approach.

A 36-year-old woman comes to the walk-in clinic complaining of 2 days of sore throat and fever. Her past medical history is unremarkable. She is a preschool teacher. On examination, she has a temperature of 101.3°F, and has an exudate in the pharynx and anterior tender cervical adenopathy. Rapid streptococcal test is negative. You suspect group A streptococcal pharyngitis and send a culture. She is allergic to penicillin, and is started on azithromycin. Two days later she calls you back reporting that her symptoms are not any better. The culture confirms group A Streptococcus. Because she is not improving you ask the laboratory to runs susceptibilities and the organisms comes back as moderately resistant to erythromycin but susceptible to clindamycin What is the most likely mechanism of resistance to azithromycin in this patient? Presence of the ermB gene Presence of the mefA gene Decreased permeability of the outer cell envelope Mutation in the ferredoxin gene Enzymatic inactivation in the cytoplasm

Correct answer: Presence of the mefA gene Acquired resistance to macrolides is an increasingly common phenomenon, and the two most common mechanisms are target site alteration and export of the drug. Target site alteration occurs when a methylase encoded by the ermB gene alters the macrolide binding site on the ribosome. This change leads to the MLSb phenotype which produces high degree of resistance to macrolides, lincosamides(e.g., clindamycin), and streptogramin B. Export of the drug is produced by a macrolide efflux pump encoded by the mef A (macrolide efflux) gene. This change leads to the M phenotype, with low to moderate degree of macrolide resistance, and no resistance to lincosamides or streptogramin B. In the United States, the most common mechanism of resistance is mediated by the mef-A gene (M phenotype). In Europe there is a much higher frequency of ermB (MLSb phenotype). Mutations in the ferredoxin gene produce resistance to metronidazole and enzymatic modification is not a mechanism of resistance to macrolides in group A Streptococcus.

A 58-year-old male underwent a right knee replacement 6 months earlier for severe osteoarthritis. He did well, but one month prior to admission, he was in a motor vehicle accident and had some trauma to the knee. He subsequently had pain in the knee and later noted a draining hole in his right knee near the surgical site. He was febrile to 39oC and examination of the right knee revealed some swelling and erythema, and a sinus tract that was draining seropurulent material. Cultures of the drainage revealed methicillin-resistant Staphylococcus aureus. What would be the most appropriate approach in treating this patient's infection? 6-week course of parenteral antimicrobial therapy followed by life-long oral antimicrobial therapy Surgical drainage of the knee with prosthesis retention followed by a 6 month course of antimicrobial therapy Prosthesis removal and immediate reimplantation of a new prosthesis followed by a 6 week course of antimicrobial therapy Prosthesis removal and reimplantation of a new prosthesis after 2-4 weeks of antimicrobial therapy followed by 6 months of antimicrobial therapy Prosthesis removal and reimplantation of a new prosthesis after 6 weeks of antimicrobial therapy

Correct answer: Prosthesis removal and reimplantation of a new prosthesis after 6 weeks of antimicrobial therapy prosthetic joint infections, removal of the prosthesis, any debris in the operative site and all the cement, and implantation of a new prosthesis after 6 weeks of systemic antimicrobial therapy In patients with a knee prosthesis, an antibiotic impregnated polymethacrylate spacer is often inserted after the prosthesis removal to prevent contracture of the joint space. This longer initial course (6 weeks) of antimicrobial therapy is especially recommended in those with difficult to treat microorganisms such as methicillin-resistant Staphylococcus aureus(which this patient has), other multidrug-resistant organisms, Enterococcusspecies, and fungi; antimicrobial therapy is discontinued 2 weeks before reimplantation to obtain reliable tissue specimens for culture at the time of reimplantation. If these cultures are positive, prolonged antimicrobial therapy is given after reimplantation. Simple surgical drainage with retention of the prosthesis and antimicrobial therapy is only successful in 14-68% of cases. Debridement with retention, however, can be considered in patients with early postoperative or acute hematogenous infection, clinical symptoms and signs <1 week, stable implant, and good soft tissue with no sinus tract. Correct Response Prosthesis removal and reimplantation of a new prosthesis after 6 weeks of antimicrobial therapy

Colistin is being used increasingly to treat resistant Acinetobacter baumannii and Pseudomonas aeruginosa infections. Which one of the following is predictably resistant to colistin? Escherichia Enterobacter Salmonella Proteus Klebsiella

Correct answer: Proteus Klebsiella, Salmonella, Haemophilus, and most Stenotrophomonas species. predictably resistant to colistin include Proteus, Providencia, Morganella, Serratia, and Burkholderia species, along with all Gram-positive bacteria. Polymyxins which include colistin(polymyxin E) and polymyxin B, bind to cell wall lipopolysaccharide. In some bacteria, the binding of the polymyxin fails to occur. A helpful acronym is M(A)PPS for bacterial genera frequently resistant to polymyxin antibiotics: M = Morganella A = "and" P = Providencia P = Proteus S = Serratia

Which of the following would be the mostly likely pathogen in this rapidly expanding skin lesion in a febrile neutropenic patient with acute leukemia? The skin biopsy with sutures is seen at the center of the lesion. Nocardia asteroidsStreptococcus pyogenesBorrelia burgdoferiPseudomonas aeruginosaStreptococcus anginosus

Correct answer: Pseudomonas aeruginosa This lesion is typical of early ecthyma gangrenosum, typically presenting in neutropenic patients and classically due to Pseudomonas aeruginosa. The sharp border and pale center comes from invasion of blood vessels in the dermis. Ecthyma gangrenosum are caused by perivascular bacterial invasion of the media and adventitia of arteries and veins that then produce secondary ischemic necrosis. Ecthyma refers to an ulcerative lesion that extends through the epidermis and deep into the dermis and appears as a punched-out ulcer surrounded by raised violaceous margins. While pseudomonas Pseudomonas is a classic cause, identical lesions can be caused by Staphylococcus aureus, other gram negative bacilli, fusariumFusarium, mycobacteria Mycobacteria and many other bacterial, fungal, and viral organisms. Many patients with such lesions do not have positive blood cultures. The etiologic agent may be recoverable from lesion exudate or a punch biopsy. Lyme lesionsi.e.erythema migrans(Borrelia burgdorferi) or streptococcal cellulitis are sharply circumscribed but don't have an association with neutropenia. Nocardiosiscauses subcutaneous abscesses. Streptococcus anginosus and Streptococcus pyogenesdo not typically cause ecthyma gangrenosum.; or otherskin lesions.

This young lady's hot tub rash, occurring mostly under her bathing suit and appearing the day after exposure, was most likely due to which one of the following organisms: Aeromonas hydrophiliaPseudomonas aeruginosaStenotrophomonas maltophiliaStaphylococcus aureusMycobacterium marinum

Correct answer: PseudomonasPseudomonas can cause a follicular rash (folliculitis) in exposed skin after hot tub bathing, often localized under the swim trunks. Staphylococcal folliculitis is more purulent and usually occurs in areas with hair, particularly with shaving, not bathing. The other organisms do not cause folliculitis. Hot tub folliculitis and hot foot syndrome are benign, self-limited skin lesions caused by Pseudomonas aeruginosa that occur following exposure to contaminated water. P. aeruginosa folliculitis typically develops 8 to 48 hours after exposure to contaminated water. The folliculitis consists of tender and pruritic papules, papulopustules, or nodules associated with malaise and low grade fever. The lesions are self resolving although an oral quinolone is often helpful in accelerating resolution.

This patient with profound, prolonged neutropenia had the rapid onset of fever and these painful skin lesions. The most likely test to reveal the etiology is: Blood culturePunch biopsySerum galactomannanBlood PCR

Correct answer: Punch biopsy This is ecthyma gangrenosa, usually due to a mould, Pseudomonas aeruginosa or occasionally another Gram negative bacillus. Blood cultures are rarely positive with aspergillosis or mucormycosis but may be positive with fusariosis or bacteria. Immediate punch biopsy with culture, Gram stain and impression smear using Calcofluor stain for hyphae can be done in hours while waiting for blood culture results.

A 24-year-old active duty Marine sergeant was seen in clinic because he had noted dark urine and yellow discoloration of his sclerae which had been increasing over 10 days. For the past month he had noted low grade fever, anorexia, malaise and dull pain in his upper abdomen. At the onset of symptoms he hadcompleted a rigorous field training in Quantico, Virginia with exposure to mud and numerous insect bites. Recently he has noted erythematous papules and macules on his trunk, palms and soles A year prior, HIV had been diagnosed and antiretrovirals begun. His clinic visits and drug intake were erratic, his CD4 was 430 /mcl and viral load 63,000 two months prior. Current antiretrovirals were efavirenz, emtricitabine and tenofovir. Liver function tests found a bilirubin of 8.6 mg/dl, alkaline phosphatase of 1200, AST of 120 and ALT of 135. CBC was normal. On exam he was afebrile and felt well but had salmon colored macules on his trunk, palms and soles. The most useful diagnostic test would be: Hepatitis C IgG serology Hepatitis A IgM serology Leptospira agglutinin serology Wright stain of peripheral blood RPR

Correct answer: RPR Hepatitis during secondary syphilis is notable for the disparity between the high elevations of the alkaline phosphatase and bilirubin compared to aminotransferases. Although the same pattern of liver function tests is found in leptospirosis, which could have been acquired by wading in streams, leptospirosis is an acute infection, not the indolent process described here. Viral hepatitis and hepatic abnormalities due to stopping tenofovir in a patient with hepatitis B have more striking elevations of aminotransferases than alkaline phosphatase. Acute hepatitis A or B from a sexual partner would be expected also to cause anorexia and malaise as well as more abnormal aminotransferases. He likely was immunized to both, however. Hepatitis is not a prominent finding in tick borne diseases, including ehrlichiosis and Rocky Mountain spotted fever, compared to rash, headache and fever.

A 24-year-old backpacker presents to your office 2 months after having sustained a penetrating, now well healed, but visible dog bite on the leg during a trek in Nepal. The dog disappeared. The backpacker has never received rabies vaccine during his life. What is the recommendation based on CDC guidelines? Rabies cell culture vaccine -either the CDC or WHO regimen Rabies vaccine plus Human Rabies Immunoglobulin into the bite site on the first day of treatment Rabies vaccine plus Human Rabies Immunoglobulin into the bite site at day 7 of the vaccine series Human rabies immune globulin followed in one month by rabies vaccine Inform the patient that it is too late for prophylaxis to be useful

Correct answer: Rabies cell culture vaccine 1.0mL, IM, at days 0,3,7,14 plus Human Rabies Immunoglobulin into the bite site at day 0 Rabies can incubate for a year or more. If risk exposure has occurred, it is never too late to start post-exposure prophylaxis. According to both WHO and CDC Guidelines, a decision to vaccinate is a decision to use RIG as the vaccine will not kick in for several weeks. RIG should not be given if vaccine has previously been started 7 or more days previously. WHO still recommends the classic Essen regimen; CDC has dropped the Day 28 dose.

A 25-year-old woman in Dallas, Texas has come to the hospital for antenatal care. She reports no prior illnesses and has never been tested for HIV. Her lab report is as follows: Pregnancy test: Positive Ultrasound: Normal 10 week gestation CD4 Count: 490 cells/uL Viral Load: 200,000 (pan sensitive on genotyping) CBC and Chem 12: Unremarkable The obstetrician seeks your advice. You would recommend: Elective caesarian section regardless of viral load on antiretroviral therapy Single-dose nevirapine at time of delivery for mother and single dose for infant within two hours of delivery, but no other antiretroviral regimen during pregnancy. Lopinavir-ritonavir, tenofovir-emtricitabine to start immediately Raltegravir and tenofovir disoproxil fumarate (TDF)-emtricitabine to start immediately.

Correct answer: Raltegravir, tenofovir, lamivudine to start immediately This woman requires ART to prevent transmission to her fetus and to benefit her own health. Most clinicians would also consider it imperative to start her on ART to benefit her. Atazanavir + ritonavir or darunavir + ritonavir or raltegravir would be reasonable choices in combination with tenofovir DF-emtricatabine. Of these, the only one offered in this question is D, raltegravir. Single dose nevirapine is indicated only in resource-limited settings where combination therapy is not feasible. There is no role for single drug therapy in the United States. C-Section would pose no benefit if the mother has a good response to ART and has a viral load less than 50 in the third trimester. An elective C section is indicated if the mother cannot obtain a viral load less than 1000 copies on ART prior to going into labor. Currently, there are insufficient data to recommend the use of TAF, dolutegravir, bictegravir or elvitegravir to be started during pregnancy but if the patient is on these drugs when found to be pregnant, it is reasonable to continue them. Note that in pregnancy, CD4 absolute counts are likely to be lower than pre-pregnancy. The CD4 percentage is a better reflection of immune competence. This decrease in absolute CD4 count is probably due to altered blood volume and volume of distribution.

A 30-year-old Peace Corps Volunteer returns to the United States after working in rural West Africa for two years. She has recurrent diarrhea and abdominal pain. You suspect a parasitic pathogen. Stools are negative for Cryptosporidium and Giardia. Which of the following tests is best to rule out Entamoeba histolytica infection? Rapid stool antigen test based on Gal/GalNac lectin Microscopic examination looking for motile trophozoites Serology Fecal leukocyte test Fecal hemoccult test

Correct answer: Rapid stool antigen test based on Gal/GalNac lectin Entamoeba histolytica stool antigen assays are commercially available (Techlab) to make an accurate diagnosis of intestinal or hepatic amebiasis based on the Gal/GalNac lectin. Serology has some role: antibodies may appear as early as 7 days. However, a positive IgG serology does not distinguish acute disease from disease that occurred and resolved months or years before. A negative serology is thus useful, but that was not a choice above. Microscopic examination of the stool is no longer performed for amebiasis given low sensitivity which is a low as 25% or specificity as low as 10%. Microscopy confuses E. histolytica with the identical appearing and much more common, non-pathogenic parasites such as E. dispar. There is no reliable method to culture the protozoan from stools. Fecal leukocytes and fecal hemoccult test are too insensitive to rule out amoebiasis.

A 24-year-old female in a monogamous relationship has a vaginal discharge that is diagnosed as trichomoniasis based on microscopic examination of a wet smear of vaginal secretions. She is treated with one dose of metronidazole, and the discharge dissipates. However, 3 weeks later she has a recurrence. She is retreated with one dose of oral metronidazole and improves. Now, however, the ob-gyn is calling you because she is in the office with a third recurrence. The motile organisms on smear are clearly Trichomonas vaginalis. You would recommend: Obtain culture and metronidazole susceptibility testing for the Trichomonas Treat with a topical metronidazole gel Treat with clindamycin gel Treat with a longer course of oral metronidazole, i.e., 14-28 days Retreat the patient with single dose oral metronidazole but also treat the sexual partner

Correct answer: Retreat the patient with single dose oral metronidazole but also treat the sexual partner. This patient very likely is being re-infected by her sexual partner. Both the patient and her partner need to be treated simultaneously. Although not offered as a choice here, a single dose of tinidazole 2 gm is often selected over metronidazole. Tinidazole is more expensive but at least as effective and better tolerated than metronidazole. Topical therapy or a longer course of therapy is not indicated because she has responded to each course of oral metronidazole. This patient's response to therapy is not the problem.

A 29-year-old man is referred to you for evaluation of fever and a rash, which have lasted 5 days. He returned from vacationing in South Africa 3 days ago. While there he spent most of his time at the beaches around Capetown. On the last day of his vacation, he had the onset of fever, mild headache, and myalgias. These symptoms persisted and were accompanied by photophobia and the development of a diffuse papular rash last night. He was sexually active while on vacation with two different female partners. His past medical history is unremarkable and he is taking no medications. On examination, temperature is 100.6°F, BP 110/78 mm Hg, pulse 94/min, respirations 14. There is a diffuse, papular erythematous rash on the trunk that extends onto the extremities. There are five dark red, 0.5-1.0-cm lesions on the right lower extremity. A few shotty cervical and inguinal lymph nodes are palpable bilaterally. The conjunctivae are mildly injected. The oropharynx is normal as are the ears and nose. The remainder of the examination is normal. Which of the following is the most likely cause of this patient's current illness? Rickettsia prowazekii Rickettsia rickettsii Rickettsia africae Measles Treponema pallidum

Correct answer: Rickettsia africae The combination of a rash associated with a tache noire or eschar in a febrile patient should always suggest a vector-borne disease. In this case, the recent travel to South Africa especially with exposure to the tall grasses around the beaches is characteristic of R. africae infection. R. conorii causes an identical disease but is uncommon in South Africa. The syndromes caused by R. conorii and R. africae are often hard to distinguish. A study in NEJM (May, 2001) suggested that R. africae is much more common than R. conorii among visitors to South Africa. Both organisms cause self-limiting diseases with short incubation periods although fatalities do occur in 2-3% of R. conorii infections. Both are often associated with rashes although a substantial fraction of patients infected with R. africae have no rash, or the rash appears late. R. africae infection characteristically has multiple tache noires (eschars) while R. conorii infections usually have only one tache noire. R. rickettsii causes RMSF, a severe disease that does not occur in South Africa. There is no eschar. R. africae shares considerable homology with R. rickettsii, but it causes a much milder illness that is usually self-limited. Primary HIV infection would be a consideration in this patient except for the tache noire. Syphilis and measles would also be a consideration were it not for the eschar. Measles is also accompanied by coryza, cough and conjunctivitis. Of note, the Rickettsia mentioned here have different vectors; Amblyomma ticks (R. africae), lice (R. prowazekii), and Dermacentor ticks (R. rickettsii). R. prowazekii (epidemic typhus) is louse-borne and very rare among vacationers. Here is a tache noire (eschar) (A) and a rash-skin lesion (B) of the tick bite for R. africae: (Emerging Infections, 2009)

A 43-year-old man recently emmigrated from Russia and applied for a job at your hospital as a venipuncture technician. Initial occupational health evaluation revealed a positive PPD of 18mm at 72 hours. He was completely asymptomatic, with no cough, weight loss, malaise or fever. The patient reports that he had BCG as a young student. He was given isoniazid because of a positive PPD in the past in Russia but had to stop after 2 weeks due to "severe liver toxicity." His chest X-ray showed a left lung granuloma, but he was never told he had positive TB smears or cultures. CT scan shows calcified lymph nodes, apical pleural thickening, and a few small granulomas in both lungs. To prevent the development of active disease in this patient with apparent latent infection, you would recommend: Begin low doses of isoniazid with gradual dose escalation and, if tolerated, treat for 9 months. Rifampin and pyrazinamide for 2 months. Rifampin for 4 months. Rifampin and ethambutol for 9 months with pyrazinamide for the first two months. Order a Quantiferon-TB Gold in-tube test before determining treatment.

Correct answer: Rifampin for 4 months This patient needs prophylaxis (i.e. treatment for latent TB infection). His chest xray is consistent with prior tuberculosis. Infant BCG immunization can cause a positive tuberculin skin test, but this patient is too old for that BCG effect to persist. BCG immunization will not cause a positive Quantiferon or T-SPOT test, but in this case, IGRA results would not alter the appropriate management strategy. If the patient could tolerate isoniazid, the preferred regimen would be 12 weeks of isoniazid plus rifapentine, administered once weekly. CDC may still recommend that even this be given by directly observed therapy, but that is controversial. The issue in this question is to know options when INH appears to be contraindicated. Age is no longer a factor in choosing isoniazid but isoniazid hepatotoxicity can be severe on rechallenge. Rifampin for 4 months is indicated. Though there is less experience with this regimen for latent tuberculosis, using isoniazid or isoniazid plus rifapentine incurs a serious risk of isoniazid hepatotoxicity. The combination of rifampin and pyrazinamide is no longer recommended because of hepatotoxicity. This two drug regimen was effective but has increased potential for toxicity. There is no indication for a three drug regimen in the absence of active tuberculosis. Were there active TB disease, culture and susceptibility testing would be important to exclude MDR tuberculosis acquired in Russia. Correct Response Rifampin for 4 months.

This 38-year-old homeless Houston male had the sudden onset in August of fever, headache, nausea, chills, myalgia and rash that spread from his trunk to hands and feet. Nausea, vomiting and cough developed over the ensuing three days despite therapy with ceftriaxone then he slowly improved. Which of the following would be the most likely source of infection: Rodent fleaDog tickDeer tickRodent mite

Correct answer: Rodent flea The diffuse macular or maculopapular rash in a mildly ill patient with fever, headache and nausea would be characteristic of endemic (murine) typhus due to Rickettsia typhi or rarely Rickettsia felis. Disease has mostly been identified in patients in contact with rats and their fleas. Most cases of R. typhi in the USA have been reported from South Texas but infection is worldwide. R. felis is transmitted by cat or opossum fleas. Rash only develops in a minority of patients and there is no eschar at the bite site, making diagnosis difficult.

A 22-year-old sexually active woman who came to New York City two days ago from Puerto Rico is seen for bilateral ankle arthritis and nodules on the legs. Her illness began two weeks ago with migratory pain involving both knees symmetrically which spontaneously resolved. She tohen developed bilateral ankle pain and swelling. On exam, she has tender reddish-purple nodules over the anterior lower legs and clear evidence of bilateral ankle arthritis with effusions. Her CBC is normal. Chest x-ray shows hilar adenopathy. Which one of the following is the most likely diagnosis? Rheumatic fever Dengue fever Gonorrheal arthritis Sarcoidosis Tuberculosis

Correct answer: Sarcoidosis This patient has migratory arthritis which might make one think of disseminated gonorrhea since you are given gratuitous information about her sexual history. The key to this question is the hilar adenopathy and the skin lesions which are nodules rather than petechial lesions typical of disseminated gonorrhea. Gonococcemia may be accompanied by arthritis which is not symmetrical and not associated with hilar adenopathy or erythema nodosum. Lofgren's Syndrome, a form of sarcoidosis, is characterized by hilar adenopathy, erythema nodosum and arthritis, typically of the lower extremities. Rheumatic fever causes a migratory polyarthritis following streptococcal pharyngitis, but is not associated with hilar adenopathy. Dengue viral infection causes joint pain (and is sometimes called "break-bone fever" for that reason) and is endemic to Puerto Rico but does not cause erythema nodosum or hilar adenopathy. Primary tuberculosis can cause hilar adenopathy without pulmonary infiltrate and erythema nodosum but not the bilateral arthritis seen here.

This is the pruritic rash on the arm of an 18-year-old counselor at a summer camp in northern Michigan. He and another fellow camp counselor developed this rash after swimming in the lake at the camp a few days previously. The young campers who also went swimming with him did not develop the rash. This was their first year at camp but the two councilors had both attended camp last year. Which of the following organisms is the most likely cause of this rash? Leptospira speciesPseudomonas speciesBorrelia speciesSchistosome species

Correct answer: Schistosome speciesThis is swimmer's itch, due to swimming in water contaminated with avian schistosomes. The cercaria burrow into the skin and die. Persons previously exposed have an intense allergic reaction to the tiny schistosomes. In the case cited above, the councilors from the prior year were sensitized from swimming in contaminated water the prior summer. Humans are not the preferred host for these parasites, and thus the cercaria cannot migrate to the liver or genitourinary tract as with Schistosoma mansoni, hematobium, or japonicum. Swimmers itch can be seen in many areas of the US. No specific treatment is indicated. Antihistamines and topical steroids are useful. Symptoms abate after 1-2 weeks. Superinfections of the abraded skin with bacteria can occur.

A group of adults in Detroit, Michigan, develop an illness during a banquet in which they are served mahi-mahi, mushrooms, wonton soup and corn with margarine. Eight of 16 persons eating the meal have symptoms beginning within 20 minutes after eating fish that tastes peppery and unusual. They experience a burning mouth, feel flushed, have a headache, and feel dizzy. Two experienced an urticarial reaction. The illness lasted 10 hours. What was the likely cause of the outbreak? Ciguatera Chinese restaurant syndrome (MSG) Staphylococcal food poisoning Scombroid Mushroom poisoning

Correct answer: Scombroid Scombroid is the cause of the outbreak. Scombroid is due to the release of histamine like substances by Proteus and Klebsiella (peppery taste) when the fish is not well refrigerated. It is treated with antihistamines. Incorrect Answers: Most outbreaks of ciguatera occur in coastal areas (Caribbean or Hawaii). The lack of neurologic findings points to a disease other than Ciguatera. MSG could cause this type of syndrome in a few patients, but not half the exposed persons, and the urticarial reaction is not consistent with MSG. Staphylococcal food poisoning would produce vomiting without other findings and the very short incubation period is inconsistent with Staphylococcal food poisoning Mushroom poisoning is a cholinergic poisoning with visual disturbances and dry mouth without the other findings here.

A family, including two parents and two children all complain of severe itching of the anal areas. Several months prior to onset, the children completed summer camp in Massachusetts and during that time their parents traveled for two weeks in Croatia. Their pediatrician suspects a parasitic infection. Which test would you order to diagnose the most likely parastic cause? Eosinophil count Scotch tape test Stool for ova and parasites Wet mount smear of perianal skin scraping Serology

Correct answer: Scotch tape test Enterobius vermicularis (pinworm) is the likely cause of the complaint. It can easily spread within the family. The test of choice is the scotch-tape test for eggs used at the anal area on arising. Stool tests for parasites are only positive in 10 to 15% of cases. Eosinophilia is uncommon and non-diagnostic. There is no routine serology available for pinworm. A serologic test for strongyloidiasis is available and this disease can cause perianal lesions with intense infection. However, strongyloidiasis is not spread within a family although there could conceivably be a common source. Wet mount smears of skin scrapings are good for the diagnosis of ringworm, candidiasis or scabies but perianal localization would be very unusual for scabies and neither ringworm nor candidiasis would spread like this. Adults of Enterobius are 2-13 mm (i.e. often less than 1 cm) in length, i.e., very small. Do not worry about identifying the worm, but be aware of its size (2-13 mm) compared to ascaris which are 15 to 35 cm, i.e., much larger. The following diagram is provided to remind you of the enterobius life cycle.

A trauma surgeon in New Orleans consults you because of failure of post-hurricane soft tissue wounds to heal after surgical debridement. Necrosis reappears post-debridement along the wound margins despite daily trips to the OR. Antibiotic therapy consists of the combination of meropenem and vancomycin.. Deep wound samples taken during debridement were sent to the lab and are negative on Gram stain and have only a light growth of mixed flora susceptible to the patient's current antibiotics. He is concerned that the increasing size of the lesions is going to make skin coverage difficult or impossible. Operating room photos show gray, non-bleeding wound margins prior to debridement and red bleeding margins after debridement. Which one of the following should you suggest to him? X-rays to look for gas pockets deep in the tissue. Add empiric colistin in case carbapenemase producing bacilli have been missed by the micro lab. Send tissue for multiplex PCR looking for bacteria (16s rRNA) Send tissue from wound margins to pathology for fungal stains and culture.

Correct answer: Send tissue from wound margins to pathology for fungal stains Massive soft tissue injury from blast injury, hurricanes, tornadoes, or other trauma in immunocompetent persons can cause invasive angiocentric mold infections that invade blood vessels and lead to tissue necrosis. Aspergillosis is usually identified on fungal culture. In contrast, Mucor is more likely to be seen than grown in this setting. Biopsy is not frequently done in these situations but can show hyphae invading blood vessels and diagnose mucormycosis by the characteristic hyphae. Gas in tissues in such cases is usually the result of debridement so x-rays are less helpful. Anaerobic bacteria could be present but the tissue gram stain and culture do not suggest anaerobic infection, although information is not given as to whether the wound cultures were handles appropriately to find anaerobes: meropenem is active against all anaerobes. Clostridia are sufficiently aerotolerant to survive delayed receipt in the micro lab. Micro labs can detect carbapenemase production and that should be requested. Colistin is rarely indicated without an otherwise resistant organism. PCR of wounds is relatively useless as a screening test because a lot a hospital flora is present in these wounds as surface contamination.

A 43-year-old HIV infected man returned for a routine visit to evaluate his antiretroviral therapy. He had achieved no detectable viral load and a stable CD4 count of 350 cells/mm3 on his current regimen of dolutegravir, abacavir and lamivudine. He took a variety of alternative medications but no other pharmaceuticals. His only complaint was redness and pain in both eyes of several days' duration, associated with blurred vision and photophobia that made it difficult to drive his car to the construction site where he worked. He had no scratchy feeling or pain on opening and closing his eyes. http://lifeinthefastlane.com/uveitis/ He was otherwise asymptomatic, with no fever or respiratory symptoms. He was sexually active with occasional anonymous partners. On exam, both eyes had conjunctival injection around the corneal limbus without exudates. Fundoscopic examination found the retinas to be without exudates but the view was somewhat hazy because of inflammatory reaction in the anterior chamber. He had a small hypopyon. With the current information, which of the following approaches is likely to be the most helpful in explaining his ocular symptoms? Conjunctival culture on Thayer Martin mediumSerology for syphilisGenProbe of conjunctival swab for Chlamydia trachomatisSerum PCR for ToxoplasmaPCR for CMV on anterior chamber aspirate

Correct answer: Serology for syphilis Bilateral perilimbal conjunctival injection in the absence of conjunctival exudate should suggest iritis or episcleritis. This patient has syphilitic iritis. Photophobia can occur but pain is not a major feature of iritis. There is an anatomic differential to begin the evaluation involving keratitis, conjunctivitis, iritis, anterior uveitis and episcleritis. This patient has the clinical findings of iritis, not conjunctivitis or keratitis or retinitis. The red ring around the cornea (ciliary flush) indicates iritis. There is no scratchy feeling or conjunctival discharge of conjunctivitis and not the intense pain of keratitis. Herpes viruses, Lyme, syphilis, TB, sarcoid, and acute reactive arthritis (formerly known as Reiter's syndrome) are causes of iritis. In many patient populations this is more likely to be non infectious than infectious with many autoimmune causes. A serologic test for syphilis should be considered in any sexually active patient who presents with findings compatible with iritis, episcleritis or uveitis. A lumbar puncture could be considered to rule out neurosyphilis in patients with syphilitic iritis, although ocular syphilis is treated the same as neurosyphilis. Ocular involvement is present in about ten percent of patients with secondary syphilis and can be the only presenting finding. Ocular involvement also occurs in congenital and tertiary syphilis. Conjunctivitis is usually associated with exudates on the lid margins and diffuse injection of the palpebral conjunctivae as well as the bulbar conjunctiva. Gonococcal or bacterial conjunctivitis is acute, purulent and very painful. Chlamydial or viral conjunctivitis is typically more a discharge with more irritation than pain. Viral keratitis is typically very painful and is associated with perilimbal injection, with haze in the anterior chamber as infection progresses. Drug reactions can cause iritis, such as with rifabutin, but efavirenz is not known to cause iritis. CMV and Toxplasma cause retinitis, not iritis. Retinitis is not associated with pain and not associated with red eye syndromes. Iritis can lead to posterior synechiae, which are bands of scar tissue that connect the posterior surface of the iris to the anterior surface of the lens.

Epidemiologic studies in a community outbreak of diarrhea with fever suggest a person-to-person spread of illness where family and others develop illness after one to three days after contact with illness cases. Which one of the following organisms do you suspect as the most likely cause? Salmonella enterica (non typhi) Vibrio cholerae Shigella sonnei Enterotoxigenic Escherichia coli (ETEC) Enterotoxigenic strain of Staphylococcus aureus

Correct answer: Shigella sonnei For bacterial diarrhea transmitted person to person, think of Salmonella typhi carriers and Shigellosis. Only Shigellosis was offered as an answer above. Salmonella typhi is also uncommon in the United States with only 200 to 300 cases per year but....this could still be on the board exam! Shigellosis is the third most common bacterial cause of diarrhea in the US after Salmonella and Campylobacter: there are 450,000 cases annually, compared to only 10 cases per year of cholera. Most cases of Shigellosis are related to day care centers, residential centers, and food borne outbreaks. Of the pathogens listed only Shigella can be transmitted to people in doses less than 100 viable organisms and thus is a likely cause of the outbreak described above. During outbreaks of Shigella secondary spread person-to-person is common. Fever is seen in patients infected by a pathogen causing intestinal inflammation or bacteremia. Not all of the choices, however, cause inflammation and fever and even with Shigellosis, a substantial fraction of patients have no fever. V. cholerae causes a secretory diarrhea without fever. There are very few cases in the US per year (only about 10) and some of these are in returning travelers. Moreover, cholera is spread by contaminated food and water rather than person to person spread. Thus, cholera is not a likely source. Staphylcoccal food poisoning is caused by ingestion of toxin in food secondary to improper hygiene during preparation. The toxin induces vomiting and dry heaves without fever. This is also not spread person to person, and thus Staph toxin is not likely. Salmonella characteristically causes illness after ingestion of > 1,000 organisms and, V. cholerae and ETEC require ingestion of > 1,000,000 organisms: thus person to person transmission of any of these organisms is not common. For bacterial diarrhea transmitted person to person, think of Salmonella typhi carriers and Shigellosis.

A 30-year-old male with a history of obstructive hydrocephalus had implantation of a ventriculoperitoneal shunt. Three months after implantation, he developed headache and nausea over a period of about one week; he denied fever. On physical examination, he was afebrile with normal vital signs. He was awake, but somewhat lethargic. There was no tenderness or erythema along the area of the implanted shunt. His abdominal examination was normal. Neurosurgery is consulted and cerebrospinal fluid removed from the shunt reveals a WBC count of 500/mm3with 80% segs, glucose of 45 mg/dL, and protein of 50 mg/dL. Gram stain was negative, but cultures grew Staphylococcus epidermidis. In addition to administration of intravenous vancomycin, which of the following is the most appropriate management of this patient? Intraventricular vancomycin through the implanted shunt Shunt removal and immediate implantation of a new shunt Shunt removal, external drainage, and re-shunting after cultures are negative Add oral rifampin to intravenous vancomycin Change vancomycin to trimethoprim-sulfamethoxazole

Correct answer: Shunt removal, external drainage, and re-shunting after cultures are negative This patient has a cerebrospinal fluid (CSF) shunt infection. Staphylococci are the most likely infecting agents (55-95% of cases), with most caused by coagulase-negative staphylococci. The most common clinical symptoms are headache, nausea, lethargy and altered mental status; fever is reported in 14-92% of cases. Numerous methods of treating CSF shunt infections have been reported, but no randomized, prospective trials have been performed. The principles of antimicrobial therapy are generally the same as for acute bacterial meningitis. Direct instillation of antimicrobial agents into the ventricles (i.e., through an external ventriculostomy or shunt reservoir) is occasionally needed for difficult to eradicate infections, but the indications for intraventricular administration are not well-defined. Attempts to treat Staph epidermidis CSF shunt infections with use of systemic antimicrobial agents alone (given by the intravenous and/or intraventricular route) rarely have been successful. Combining removal of shunt hardware with immediate shunt replacement and intravenous antimicrobial therapy cures approximately 65-75% of patients. Antimicrobial use with removal of all components of the shunt along with some component of external drainage appears to be the most effective treatment, with treatment success usually >85%.

A 42-year-old male presents with a deformed foot and draining sinuses. The patient is a native of Mexico, where he did manual labor. He noted swelling 2 years ago in Mexico after dropping a heavy wooden box on his foot. He received various treatments, but his foot has become more and more deformed. He is now in the United States visiting family, and comes to you for advice. He is afebrile, with normal vital signs. He looks well. His exam is normal except for his left foot, which is swollen and firmly indurated, particularly over the metatarsals, but not very tender, with several draining sinuses. His routine lab work, including CBC, is normal. An x-ray of his foot shows soft tissue destruction without boney destruction. The appropriate approach would be: An empiric course of trimethoprim-sulfamethoxazole An empiric course of amphotericin An empiric course of itraconazole Smear and culture of the sinus discharge before initiating therapy Surgical biopsy and culture of deep tissue

Correct answer: Smear and culture of the sinus discharge before initiating therapy entity can be caused by filamentous branching bacteria such as Nocardia, Streptomyces, or Actinomadura. The entity can also be caused by fungi such as Scedosporium apiosperumum, Madurella, and Fusarium. The distinction between chronic osteomyelitis and mycetoma is easier in this case because of the absence of boney destruction. This patient has a mycetoma, or Madura foot, which is characterized by a slowly progressive, painless destruction of the subcutaneous tissue that dissects up fascial planes and sometimes involves contiguous bone. This is caused by inoculation of the organism by minor trauma into the subcutaneous tissue, usually the foot. Persons picking up firewood and carrying it on the back or on the head may have lesions on the hands, upper back or top of the scalp. There are tiny grains in the draining material that represent clumps or organisms. This entity can be caused by filamentous branching bacteria such as Nocardia, Streptomyces, or Actinomadura. The entity can also be caused by fungi such as Scedosporium apiosperumum, Madurella, and Fusarium. Thus, empiric therapy would be inappropriate until the causative organism was known to be a fungus or a bacterium. This distinction can be made by microscopic examination of a grain in the sinus drainage. There is a role for surgical biopsy if there is no sinus drainage, but grains with the causative organism should be sought first in sinus tract drainage if sinuses are present. Biopsy can miss the diagnosis if multiple sections are not examined because grains are infrequent in the tissue.

A 34-year-old woman in Columbus, Ohio was admitted to the hospital because of high fever, prostration, and extreme malaise of increasing severity over the past week. Her past history was notable for Crohn's disease being treated with adalimumab (Humira) for the past two months. Prior prednisone therapy had been discontinued. She was born in Nicaragua but had lived in the United States with her husband and children for the past five years, working in a daycare center. On examination, she was flushed and dyspneic, with pulse oximetry at 92% saturation. Chest x-ray showed a faint diffuse infiltrate. Admission studies found her long standing anemia has worsened, with a hematocrit of 25%, platelet count 30,000, WBC 2,500 with a normal differential, alkaline phosphatase 250, ALT 120, AST 89 and creatinine 2.0. She was transferred to intensive care and given intravenous cefipime and levofloxacin plus oral doxycycline. Admission and subsequent daily blood cultures remained negative. At the end of the first week, micafungin was begun because yeast cells were seen in her peripheral blood smear. The most likely source of her infection was which of the following: a human in Nicaragua a human in her day care center Her intestinal tract pigeon droppings soil

Correct answer: Soil Patients receiving TNF-alpha inhibitors are at increased risk of disseminated, severe intracellular infections, including tuberculosis and histoplasmosis. Histoplasma is the most likely organism to be seen in a peripheral smear: this is likely from soil that contains bird or bat droppings, is aerosolized, and inhaled. Although the clinical picture is compatible with either infection, yeasts in the peripheral blood smear provide strong evidence for acute disseminated histoplasmosis. Candidemia and Cryptococcus can rarely be seen as yeast on smear, but would have grown from routine blood cultures and would not explain her pancytopenia.Viral associated lymphocytic hemophagocytosis can present as fever and pancytopenia, but would not explain yeasts in the blood stream. An echinocandin such as micafungin, is not effective and immediate institution of amphotericin B is indicated. Azotemia, probably from her histoplasmosis, would suggest use of liposomal amphotericin B (AmBisome) would be better tolerated than conventional amphotericin B.

An otherwise healthy 58-year-old woman who lives in Wisconsin presents with progressive nodular lesions on her right forearm. She has recently acquired a kitten. These lesions have been present for approximately six weeks, and have increased in number as they have progressed from her finger to the forearm. She works at a local flower store. She visited Brazil 3 months ago and spent 2 weeks in the Amazon basin. She gives no history of fever or constitutional symptoms. Her exam is remarkable for several subcutaneous lesions from the right hand extending to the forearm. There is no associated lymphadenopathy or lymphangitic streaking. The lesions are somewhat painful to palpation, and some of these nodules have spontaneously suppurated and drained. Which of the following is the most likely diagnosis? Sporothrix schenckii Leishmania brasiliensis Bartonella henselae Prototheca wickerhamii Mycobacterium marinum

Correct answer: Sporothrix schenckii The syndrome of nodular lymphangitis is defined by marching subcutaneous lesions, intervening normal skin, often without specific lymph node involvement. Common causes include Sporothrix species, Mycobacterium marinum, and Nocardia brasiliensis. Cutaneous leishmaniasis, cat scratch disease and protothecosis uncommonly have proximal lesions along the extremity. Cat scratch often has a suppurative proximal lymph node but the proximal lesion in the photo is not in a lymph node but along the lymphatic drainage. The diagnosis is usually based on culture, histopathology or, in the case of leishmaniasis, by PCR. Even though another diagnosis is possible, the most likely diagnosis in this case is sporotrichosis because of the epidemiology and proximal lesion.

This hand MOST LIKELY came from a 24-year-old-female who was just recovering from which of the following: Streptococcal toxic shockStaphylococcal toxic shockKawasaki's syndromeKukuchi syndromeVibrio vulnificus sepsis

Correct answer: Staphylococcal toxic shock Desquamation often follows resolution of staphylococcal toxic shock but not the other entities except Kawasaki's disease, a disease extremely rare in adults. S. aureus toxic shock is usually associated with menstruation, a surgical wound, or a staphylococcal abscess. Patients with Staphylococcal toxic shock usually have fever >38.9°C, hypotension, diffuse erythroderma, desquamation (unless the patient dies before desquamation can occur), and involvement of at least three organ systems. Cultures of wounds or other sites are usually positive. Blood cultures are rarely positive. The desquamation occurs 1-2 weeks after onset of the syndrome. The clue to toxic shock is the rapid onset of shock symptoms in young, healthy people. This syndrome needs to be distinguished from Group A streptococcal toxic shock which is most often associated with severe pain and tenderness at an infected site of local trauma and the early appearance for renal dysfunction. Patients with streptococcal TSS may require immediate surgical debridement of the involved site. Blood cultures are usually positive (60% in streptococcal toxic shock vs 5% in staphylococcal toxic shock).

A 36-year-old man with lymphoma was admitted and his 3rd cycle of chemotherapy begun. Within 4 days of admission, he developed pneumonia and was treated with vancomycin, ceftazidime and tobramycin. Sputum revealed a heavy growth of Staphylococcus aureus, methicillin resistant (MRSA),with a vancomycin MIC of 4 mcg/ml (intermediate resistance). Blood cultures were negative. The isolate was susceptible in vitro to linezolid, daptomycin, tigecycline and rifampin. The tobramycin and ceftazidime were discontinued. After another 2 days, it was apparent that the patient was not responding clinically, the vancomycin trough serum concentration is 12 mcg/ml, and the Gram stain of a new tracheal aspirate showed many Gram positive cocci in cluster. At this point, which one of the following changes in his treatment regimen would you recommend? Start Linezolid and stop vancomycinStart Daptomycin and stop vancomycinStart Tigecycline and stop vancomycinIncrease dose of Vancomycin to achieve trough levels of 15-20 mcg/mlAdd Rifampin and continue current vancomycin dose.

Correct answer: Start Linezolid and stop vancomycin VISA strains (vancomycin intermediate resistant S. aureus; MIC 4-8 mcg/ml) are unlikely to respond to vancomycin at any dose. Options for the treatment of VISA pneumonia include: linezolid, telavancin, and ceftaroline. Daptomycin is not indicated for the treatment of pneumonia based on clinical trails wherein patients with S. pneumonia CAP failed, presumably due to binding of the drug to pulmonary surfactant. Whether, daptomycin would fail treatment of S.aureus pneumonia is unknown as Staph aureus infections are necrotizing and destroy surfactant while S.pneumoniae infection does not result in degradation of surfactant. In patients with right-sided endocarditis and embolic S.aureus pneumonia, daptomycin seemed efficacious. None the less, probably it is good to avoid treatment of pneumonia with daptomycin until more data are available. Linezolidhas demonstrated efficacy in the therapy of MRSA infections, including pneumonia. Linezolid is bacteriostatic and avoided if there is a possibility of endocarditis. Many clinicians would not use linezolid to treat Staphylococcal bacteremia. After 2 weeks of therapy, there is risk of bone marrow suppression and peripheral neuropathy. Tedizolid is similar to linezolid and, like oritavancin and dalbavancin, was approved for complicated skin and skin structure infections; so far, there are little or no data on the therapy of pneumonia or bacteremia with these newer agents. Ceftaroline, is the first cephalosporin with activity vs. MRSA, including VISA strains. Ceftaroline was found not inferior to ceftriaxone in treating methicillin susceptible Staph aureus pneumonia. Telavancin is a newer lipoglycopeptide that is bactericidal for MSSA and MRSA and is approved for treatment of hospital acquired pneumonia. It would be a reasonable option if the isolate were susceptible (likely for VISA, much less likely for VRSA) but was not offered as a treatment option. Tigecycline has undesirable pharmacokinetics with very low serum concentrations. . Many treatment failures are documented. Vancomycin dose: Increasing the dose of vancomycin to trough blood levels of15-20 mcg/ml may increase the possibility of nephrotoxicity (evidence reviewed in Antimicrob Agents Chemother 2008; 52:1330-6) but as noted above is not likely to be effective against a VISA strain of Staph aureus. The addition of rifampinto antimicrobial therapy of susceptible staphylococcal infections remains controversial as to therapeutic efficacy, adds the risk of many drug-drug interactions, and may result in rifampin resistance. Rifampin was not recommended in the recent IDSA guidelines for MRSA infections except for infections involving implanted prostheses (Clin Infect Dis 2011;52:1-38).

A 46-year-old man in excellent health presents to his physician with a 2-week history of "very bad flu" and is found on exam to have a rash and oral ulcers. Work-up includes a negative HIV antibody test, a negative monospot test, and negative hepatitis A, B, and C serologies. His HIV RNA (qualitative) is positive and a quantitative test shows 3.5 million copies/ml. CD4 cell count is 835 and an HIV genotype test is sent. The patient requests immediate treatment to reduce his symptoms and reduce his risk of transmitting the infection to his HIV-negative partner. Which of the following is the best choice for you to initiate immediately while you wait for additional information? No therapy is indicated for acute infection Start tenofovir AF /emtricitabine and dolutegravir Start tenofovir DF/emtricitabine/efavirenz Start tenofovir DF/emtricitabine + raltegravir Start tenofovir AF/emtricitabine + darunavir/ritonavir

Correct answer: Start tenofovir/lamivudine and dolutegravir Treating this patient with acute HIV infection with antiretroviral therapy will decrease his symptoms, reduce damage to tissues, and his risk of transmitting HIV sexually. Without an available genotype result, treatment guidelines recommend starting an HIV integrase-based regimen. This is a key part of this question. The integrase inhibitor dolutegravir (or bictegravir) are best used with tenofovir AF -emtricitabine since integrase inhibitor resistance in the community is still rare. Also, both drugs have a high barrier to resistance. If a substitution at Q148 is present, this would decrease the activity of dolutegravir in a subsequent regimen. Raltegravir is a reasonable choice as well, but requires a higher pill burden and has a lower barrier to resistance. TDF or TAF are likely equally efficacious but most clinicians lean toward TAF owing to the lower likelihood of long-term renal or bone toxicities. Drug resistance to NNRTI (both nevirapine and efavirenz) occurs in the community, if the genotype ultimately demonstrates wild-type virus, a change to a simpler regimen could be done. Protease inhibitors (e.g., darunavir) can work and has a low likelihood of high - level resistance, but darunavir is less well tolerated than InSTI agents (bictegravir and dolutegravir), which are the optimal choices for this patient. Darunavir-ritonavir also has much higher likelihood of drug-drug interactions with other medications the patient may be on owing to the strong inhibition of CYP3A4.

A 37-year-old male construction worker from Mexico was referred because of new skin lesions, arhthralgias and fever. The patient had been diagnosed with lepromatous leprosy 4 months earlier and started on dapsone, rifampin, and clofazimine. He was found to be HIV positive, with a CD4+ count of 350/cu mm and a viral load of 50,000 copies/ml. Antiretrovirals were not begun. A few days prior to consultation, the patient had the onset of fever, arthralgias and new skin lesions. On examination his temperature was 39.5°C. In addition to his prior skin nodules and plaques, he had several new, tender, red, nodular lesions on his face and anterior aspect of his lower extremities. Routine CBC and chemistries were unremarkable. A CRP was 45. The most likely course of action to benefit this patient is which of the following: Hold dapsone and rifampin Start thalidomide Start ethambutol Stop clofazimine Biopsy a skin lesion for culture

Correct answer: Start thalidomide erythema nodosum leprosum (ENL), complication first year of treatment of lepromatous lepromatous or borderline lepromatous leprosy. Despite concerns about birth defects in pregnant women, thalidomide remains a useful drug for controlling this reaction. Corticosteroids also have a role, particularly if neuritis is worsening, but that option was not provided in the answers above. ENL should be distinguished from reversal reactions, which are increases in erythema and size of prior lesions, usually with minimal if any systemic symptoms. Holding the dapsone or rifampin will not alter the course of ENL. Clofazimine can cause skin pigmentation but not skin lesions. There is no role for ethambutol in the treatment of leprosy. This patient should start antiretrovirals when the ENL is controlled. Biopsy of a lesion to determine the number and integrity of the leprosy bacilli will not be helpful in establishing the cause of the lesion. This bacillus cannot be cultured

A 55-year-old man with a history of prostate cancer was diagnosed with cystitis and started empirically on ciprofloxacin. Twelve days later, he developed a non-pruritic rash, fever, malaise, and painful lesions in his mouth and eyes (figures shown). The skin of his abdomen, legs, and back had similar lesions, with detached skin involving approximately 10% of his body surface area. Which of the following is the most likely diagnosis? Bullous impetigo Paraneoplastic pemphigus Dermatitis herpetiformis Disseminated herpes zoster Stevens Johnson Syndrome

Correct answer: Stevens Johnson Syndrome Stevens-Johnson syndrome (SJS) and toxic epidermal necrolysis (TEN) are severe idiosyncratic reactions characterized by mucocutaneous lesions leading to necrosis and sloughing of the epidermis. SJS and TEN are distinguished by the percentage of body surface involved, with SJS producing skin sloughing in less than 10% of the body surface, and TEN in more than 30% (SJS-TEN overlap 10 to 30% skin sloughing of body surface area). These disorders frequently present with a prodrome of malaise and fever, followed by the rapid onset of a diffuse rash, with macules and plaques and some degree of skin sloughing. Mucosal membranes (oral, genital, ocular) are affected in 90 to 100% of patients. Medications are the most common cause of SJS and TEN (the most common are: sulfonamides, β-lactams, allopurinol, antiepileptics, NSAIDs). Other etiologies include infections (SJS more common than TEN), vaccinations, systemic diseases, and exposures to chemical agents. Skin biopsy is useful in the diagnosis. In this case the rash is diffuse, but skin sloughing is limited to less than 10% of body surface area. Incorrect answers Paraneoplastic pemphigusis indolent in onset. Dermatitis herpetiformisis intensely pruritic and less extensive. This entity looks like herpes simplex but is autoimmune and is not caused or related to a Herpes virus. Disseminated herpes zoster typically occurs in immunocompromised individuals. Clinical findings include diffuse vesicles or ulcers, but desquamation does not occur. Bullous impetigois never this extensive and doesn't involve the mucous membranes.

A 40-year-old Caucasian woman, raised in the Philippines, saw a dermatologist because of multiple enlarging skin lesions over her face and body. A diagnosis of multibacillary lepromatous leprosy was confirmed by skin biopsy which demonstrated numerous acid fast bacilli. Her routine laboratories were not markedly abnormal. The patient was begun on rifampin, dapsone, and clofazimine. You are asked to see the patient 2 weeks later because she is very fatigued with dyspnea on exertion. Examination reveals skin lesions which she states are unchanged, tachycardia of 120/min, and mild scleral icterus. Labs at this time reveal: WBC: 4000 (62% PMN, 32% lymphs, 3% eos, 3% monos) Hgb: 6.4 gm/dL; (prior Hg was 12.6); Retic. Count 11% Platelets: 120,000/mm3 Creatinine: 1.3 mg/dL AST 34 U/L ALT 41 U/L Alkaline Phosphatase: 105 U/L Bilirubin: 6.5 (direct: 1.2) LDH: 564 U/L (prior week LDH = 250 U/L) The patient's oxygen saturation on room air is normal. Which one of the following should be done? Start prednisone Administer methylene blue Start thalidomide Stop dapsone Stop clofazimine

Correct answer: Stop Dapsone This is a case of dapsone-related hemolysisin a patient with glucose-6-phosphate dehydrogenase deficiency. Note the anemia, high reticulocyte count , and high LDH. This is not erythema nodosum leprosum (ENL). G6PD (glucose 6-phosphate dehydrogenase) deficiency in a patient receiving dapsone can cause severe hemolysis. All patients should be tested for G6PD deficiency prior to using dapsone. Hemolysis due to G6PD deficiency is more common in dark-skinned patients but the hemolysis is mild in severity; the hemolysis is less common, but more severe in Caucasians. Dapsone also can cause methemoglobinemia of sufficient degree to cause hypoxemia and fatigue/weakness but methemoglobinemia does not lead to anemia. Further, the patient's oxygen saturation is unremarkable. Thus, as a dapsone related toxicity, one must distinguish between G6PD related hemolysis and methemoglobinemia. In some patients the G6PD test is falsely negative due to lysis of all the older cells. Nonetheless, patients are anemic, have elevated indirect bilirubin, and have a normal oxygen saturation. In contrast, in patients with methemoglobinemia, the patients are not necessarily anemic, and the oxygen saturation is discordantly low compared to the pO2. Methylene blue is an appropriate acute intervention for methemoglobinemia caused by dapsone. Regarding the other answer choices, none of the other agents cause severe hemolysis. The patient has no evidence of a reversal reaction (reversal reaction characterized by lesions getting worse rather than better presumably due to more intense immunologic reaction) or ENL (erythema nodosum leprosum) with nodules and leukocytosis that would warrant treatment. Thalidomide is generally avoided in women of child-bearing age, but is the treatment of choice for ENL. High-dose prednisone is an alternative to thalidomide. Use of clofazamine has resulted in marked reduction of the incidence of ENL.

A 34-year-old woman from Connecticut was started on doxycycline two weeks ago for erythema migrans accompanied by fever and myalgia. Today, on her 14th day of treatment, she calls to say her fever and rash are gone but she is still not feeling well and wants to know what to do. She has severe fatigue and is unable to work a full 8 hour day and has a poor appetite. She has not regained the five pounds she lost. Her vital signs and physical examination are unremarkable. CBC and comprehensive chemistry panel detect no abnormalities. Which one of the following is the best next step for this patient? Continue the doxycycline for an additional two weeks. Switch to ceftriaxone to be administered for an additional 4 weeks. Add azithromycin for an additional two weeks. Stop the doxycycline. Perform an echocardiogram.

Correct answer: Stop the doxycycline. Doxycycline is appropriate therapy for early Lyme disease that is clearly the case here. Treatment duration of 10 days to 14 days has been studied and is equivalent to longer durations of therapy. Studies have shown that at the end of 10 day or two week course of therapy, as many as 1/3 of patients are still symptomatic, but further treatment is no better when compared to stopping at the end of the 10 or 14 days. There is no reason here to add or switch or continue antimicrobial therapy. Cardiac involvement would be extremely unlikely this early in the disease course given early treatment. If one were concerned about cardiac involvement, the best test would be an EKG to look for heart block, not an echocardiogram initially as myocarditis causing lower ejection fractions is fairly rare compared to Electrophysiologic dysfunction.

A 42-year-old man from New York City developed fever, dyspnea, and increasing pulmonary infiltrate four weeks post-cadaveric single lung transplant. He had been receiving standard 3 drug immunosuppression, but has also required high dose steroids for acute organ rejection. He received standard anti-infective prophylaxis. On bronchoscopy, diffuse alveolar hemorrhage was noted from both lungs. Biopsy of the transplanted lung showed no evidence of rejection. BAL stains for bacteria, fungi and mycobacteria were negative. PCR of blood for CMV was negative. The transplant center was notified that the recipient of the other lung had developed a similar syndrome. The donor was a 20-year-old recent immigrant from Guatemala who died of a gunshot wound. His mother thought he had been healthy. Assuming this infection was acquired from the transplanted lung, which organism appears most likely: Balamuthia mandrillaris Rabies Cryptococcus neoformans Nocardia brasiliensis Strongyloides stercoralis

Correct answer: Strongyloides stercoralis asymptomatic in the donor lungs, taking four weeks to become manifest, and causing diffuse pulmonary hemorrhage. Strongyloides larvae could have been migrating through the lungs of the donor, passed from the lungs to the intestinal tract of the recipient and begun the expansion of the larval population migrating through the recipient's lungs, causing the syndrome of hyperinfection in this immunosuppressed patient. The BAL might have had larvae, had they been sought (wet prep of sputum or BAL for ova and parasite). There are several cases similar to this in the literature: Strongyloidesinfection should be considered in the appropriate populations of donors and recipients. Preemptive treatment with ivermectin can prevent active disease. Balamuthia, rabies, and Cryptococcushave been transmitted and caused encephalitis and/or disseminated infection in organ donor recipients, but not diffuse alveolar hemorrhage. Nocardiawould be unlikely to be asymptomatic in the donor and also unlikely to cause alveolar hemorrhage.

A 35-year-old male presented with left-sided weakness and a seizure. He was found to have HIV (CD4=15 cells, Viral Load = 300,000 copies). A diagnosis of cerebral toxoplasmosis was made. He was started on sulfadiazine plus pyrimethamine, and two weeks later was started on dolutegravir + emtricitabine-tenofovir. Two weeks later, after taking part in a 4-hour political rally where it was very hot and he walked three miles, he presented to the Emergency Room with left flank pain and nausea. His serum creatinine had doubled and an ultrasound shows mild left hydronephrosis. The most likely cause of this syndrome would be related to which of the following: Toxoplasmosis Ureteral calculous (Calcium or Urate) Sulfadiazine Pyrimethamine Tenofovir

Correct answer: Sulfadiazine most likely has a renal obstruction due to crystallized sulfadiazine. poorly hydrated predisposes to crystallization in the urine. High dose sulfadiazine can cause crystal induced interstitial nephritis, obstruction due to sludge composed of crystals, or calculi. Treatment includes high urine flow, urine alkalinization, and in some cases urological procedures. Drugs that cause crystals include sulfadiazine, indinavir, acyclovir, and atazanavir. Tenofovir can cause renal toxicity, but not obstruction. Early in therapy acute renal failure occasionally occurs. Later in therapy tubular dysfunction (Fanconi syndrome) may occur. Ureteral stone always possible but less likely in the clinical setting described

This 71-year-old woman with well controlled diabetes mellitus had the onset of severe pain and a red, tender rash in her calf. She was not aware of preceding trauma. She is applying topical antifungal cream for known Tinea pedis. In the ED She was given cephalexin. The pain persisted she returned to the emergency room about 48 hours after onset. Her temperature was 102°, BP 80/50. WBC was 15,000 with 80% PMN and 5% bands. The periphery of the areas of inflamed skin remained tender but the central portions of the inflamed areas were less tender than previously. Red streaks were seen in soft tissue of the thigh. Her antibiotic regimen is switched to vancomycin, clindamycin, and piperacillin-tazobactam. Which of the following is the most likely diagnosis? Soft tissue x-ray MRI Needle aspiration Surgical consultation Addition of IV metronidazole to regimen

Correct answer: Surgical consultation Although ultrasound and MRI imaging may help determining the presence of infection in the fascial plane, definitive diagnosis is surgical exploration with debridement of any areas of necrotizing fasciitis identified. The clinical picture is most consistent with S.pyogenes infection of the epidermis, dermis, and subcutaneous fat: i.e. erysipelas. The portal of entry is the Tinea pedis. The red streaks represent streptococcal lymhangiitis. Progression of erysipelas into the fascial plain can cause destruction of the neurovascular bundle with subsequent necrosis of the overlying skin manifest as reduced pain , a purple hue to the skin , and ischemic bleb formation. The other causes of necrotizing fasciitis do not fit the clinical setting. Clostridium perfringens causes nec. Fasciitis but occurs most trauma in the presence of a local anaerobic environment. Subcutaneous gas formation occurs. S. aureus, as usual, forms pockets of purulence some of which may extend into the fascial plane. Meleney described necrotizing Fasciitis in patients with a mixture of aerobic and anaerobic bacteria after surgery: e.g., leakage of bowel content intothe abdominal wall. Imaging of the leg with soft tissue xray or MRI could be useful to delineate whether there was air present in the soft tissues, and to assess the anatomy of this process. However, this patient most likely has necrotizing fasciitis, and urgent surgical intervention should be the top priority along with the immediate initiation of broad spectrum antibiotics. Necrotizing soft tissue infections come in a variety of forms and include necrotizing forms of cellulitis, myositis, and fasciitis. All of these syndromes are characterized clinically by fulminant tissue destruction, systemic toxicity, and high mortality. Necrotizing soft tissue infections are divided into two distinct bacteriologic entities: type I (polymicrobial infection) and type II (group A streptococcal infection although other organisms can cause similar syndromes For the type I infections, at least one anaerobic species (usually Bacteroides, Clostridium, or Peptostreptococcus) is isolated in combination with one or more facultative anaerobic streptococci (other than group A) plus members of the Enterobacteriaceae (eg, Escherichia coli, Enterobacter, etc) Obligate aerobe, such as Pseudomonas aeruginosa, is only rarely part of such a mixed infection. Necrotizing fasciitis of the head and neck is usually caused by mouth anaerobes (e.g.Fusobacteria, anaerobic streptococci, or Bacteroides) . Fournier's gangrene is caused by facultative organisms (E. coli, Klebsiella, enterococci) along with anaerobes. Type II necrotizing fasciitis is generally mono-microbial due to group A Streptococcus (also known as hemolytic streptococcal gangrene). MSSA or MRSA may sometimes also be involved. Aeromonas hydrophila has been associated with traumatic lesions in fresh water, and Vibrio vulnificus can cause necrotizing fasciitis in association with seawater injuries or among patients with cirrhosis who ingest raw oysters. Necrotizing fasciitis involves deeper tissues than necrotizing cellulitis: the fascia and fat are involved. The area may be firm to palpation, but is associated with extreme pain and rapid progression with purple discoloration leading to bullae and necrosis. As the syndrome progresses, the area may become anesthetic. Necrotizing fasciitis of the neck results from a breach in oropharynx mucous membrane integrity in the setting of a dental infection or an oral procedure that disrupts mucosa Necrotizing fasciitis of the perineum (Fournier's gangrene) occurs due to a breach in the integrity of the gastrointestinal or urethral mucosa, especially in diabetic men. Metronidazole is unlikely to add significant antimicrobial coverage over and above the piperacillin tazobactam. The issue here is broad spectrum coverage pending surgical intervention and delineation of the precise causative agents.

A 39-year-old female received allogeneic hematopoietic cell transplantation for acute myelogenous leukemia, from an unrelated donor, using a myeloablative conditioning regimen. Her post-transplant course was complicated by skin and gut graft vs host disease which was treated with methylprednisolone (2 mg/kg). She had a positive CMV PCR on day 40, at 5000 copies (log10 4.82 IU/ml), and was treated with induction doses of ganciclovir (5 mg/kg twice daily, Cr 1.0). During the second week of induction dosing of ganciclovir, her absolute neutrophil count declined to 200/mm3; ganciclovir was changed to foscarnet (90 mg/kg twice daily). Two weeks after switching to foscarnet, her serum CMV PCR was undetectable but she developed vaginal pain, with visible ulcers on her vulva and adjacent to her urethra. She has no concurrent diarrhea, rash, or oral mucositis. WBC is now 5000 with 70% neutrophils. What is the best therapeutic intervention for her vaginal ulcers at this point? Continue foscarnet but change to once daily maintenance. Continue foscarnet and add topical trifluridine to vulva. Continue foscarnet and add topical corticosteroids to vulva. Switch foscarnet to acyclcovir. Switch foscarnet to ganciclovir.

Correct answer: Switch foscarnet back to ganciclovir. This patient likely has genital ulcers caused by foscarnet. Genital ulcers can occur during foscarnet therapy, likely as a result of contact dermatitis caused by high concentrations of foscarnet in the urine. Changing to once daily foscarnet is a less favorable option than changing to a different drug. Other causes of genital mucosal lesions include HSV; however, breakthrough Herpes simplex vaginitis/cervicitis is highly unlikely for someone receiving high dose foscarnet. A negative PCR for HSV on a swab of an ulcer would also help exclude HSV. She is unlikely to have graft vs host involving her vulva, since she has no other manifestations: no diarrhea, no oral mucositis, and no rash. The appropriate first step is to switch to an alternative anti CMV drug. There is no role for topical agents. Since the ANC had improved on foscarnet, switching to valganciclovir or maintenance once daily intravenous ganciclovir is a reasonable choice. If the ANC drops again, support with hematopoietic growth factors may be needed to stabilize the counts. Given the improvement in her CMV viral load, she doesn't need to go back to the higher induction dosing of ganciclovir or valganciclovir. Valacyclovir activity against CMV is inadequate for treating or maintaining reactivation.

A 63-year-old male underwent allogeneic stem cell transplant for chronic myelogenous leukemia 120 days ago. He has had multiple episodes of graft-versus-host disease. His absolute neutrophil count hovers between 750 and 1000 cell/µL. He has received multiple pulses of corticosteroids as well as maintenance doses of cyclosporine and prophylactic doses of trimethoprim-sulfamethoxazole. The patient develops a fever, patchy pulmonary infiltrates and hypoxia. He is intubated and undergoes bronchoscopy. The micro lab reports that branched hyphae are present on wet mount of the BAL. No pneumocystis was seen. Liposomal amphotericin (5 mg/kg/day) is started. Five days later, the lab reports that the BAL culture is growing Scedosporium apiospermum. Culture for cytomegalovirus is positive but PCR of peripheral blood for CMV is undetectable. The patient is still febrile and the pulmonary status has deteriorated. At this point, you would recommend: Raise the dose of liposome amphotericin B to 10 mg/kg Switch to ganciclovir Switch to fluconazole Switch to voriconazole Add caspofungin

Correct answer: Switch to voriconazole This mold is one of the few pathogens that are inherently resistant to amphotericin B but susceptible to itraconazole and voriconazole. Primary resistance to amphotericin is typical for Aspergillus terreus, Scedosporium spp., and Trichosporon spp. For the Candida spp., primary resistance is noted most often for Candida lusitaniae. Development of resistance in isolates of normally susceptible species is infrequent but has been described for most common pathogens. Voriconazole is FDA approved as salvage therapy for S. apiospermum, is available for intravenous administration and would be preferred in this patient. Viral culture of BAL commonly grows CMV. With a negative blood PCR in this patient population, CMV pneumonia in this patient is unlikely. Echinocandins are not recommended alone or in combination for scedosporiosis.

A 27-year-old accountant from New Orleans comes to clinic for recurrent unexplained fevers of ten years' duration. His symptoms started at age 17, he had three to four episodes every year, and the episodes lasted for six to 10 days. Associated symptoms include muscle aches, and sometimes red painful eyes and abdominal pain. Sometimes his cervical lymph nodes will swell with an attack but without a sore throat. He had a negative laparotomy for suspected appendicitis at age 18. No one in his family has had similar episodes. His fever and pain have always responded to short courses of antibiotics and, on one occasion, prednisone. A prolonged trial of colchicine had no effect on his attacks. He underwent an extensive work up in the past that included blood cultures; chest, abdomen and pelvis CT scans; immunoglobulin IgG, IgM, IgA and IgD assays; and peripheral blood smears for malaria. All tests were negative. On exam he is febrile, has conjunctivitis, a blotchy red maculopapular rash on the trunk, and tenderness to palpation of the muscles in his right leg. WBC is 16,000 with a normal differential, CRP 34, ESR 110. Blood chemistries and chest x-ray are normal. Which of the following is the most likely diagnosis? Adult Still's disease Familial Mediterranean fever Hyper IgD syndrome TNF-receptor associated periodic fever (TRAPS) Immunoglobulin subtype deficiency

Correct answer: TNF-receptor associated periodic fever (TRAPS) The familial febrile syndromes or autoinflammatory syndromes are rare inherited disorders characterized by recurrent episodes of fever and localized inflammation. The distinction between TRAPS and FMF is subtle but it's worth knowing why this patient has TNF Reception Associated Periodic Fever (TRAPS) rather than Familial Mediterranean Fever (FMF): These disorders are not autoimmune in nature They are caused by mutations in inflammatory mediators or receptors. Periodic fevers include autosomal recessive disorders such as familial Mediterranean fever (FMF), and hyper IgD syndrome, and autosomal dominant disorders such as TNF-receptor-associated periodic syndrome (TRAPS). There are other syndromes that are usually seen in pediatric population such as Muckle-Wells syndrome or familial cold autoinflammatory syndrome. The autoinflammatory syndromes that may present in adolescents, or young adults, are TRAPS, FMF, and hyper IgD syndrome. TRAPS was initially reported in families of Irish/Scottish ancestry but has been found in patients of many other ancestries and ethnicities. Some patients with TRAPS lack a family history. TRAPS is caused by mutations in the TNF receptor. The median age of onset is 3 years, ranging from 2 weeks to 53 years. TRAPS is characterized by recurrent episodes of fever, abdominal pain, localized myalgia or muscle cramping, and an erythematous tender rash. Abdominal pain is common and may lead to surgery.Other manifestations include conjunctivitis, lymphadenopathy, chest and genitourinary pain. The symptoms increase over 1-3 days, then after a number of days gradually resolve lasting on average more than 5 days. The disease can be complicated by AA amyloidosis. Some features that help characterize TRAPS as opposed to FMF are the: Ethnicity (Scottish or Irish for TRAPS vs. Jewish, Turkish, Armenian or Arab for FMF) Pattern of inheritance (autosomal dominant vs. recessive) Duration of symptoms (several days for TRAPS vs. 2 days for FMF) Presence of myalgias and conjunctivitis with TRAPS Response to medications (response to steroids or TNF-alpha blockers in TRAPS and to colchicine in FMF). Both disorders can be complicated by amyloidosis. Other Incorrect Answers The hyper IgD Syndrome (HIDS) has occurred in patients of Northern European ancestry and is caused by mutations in the mevalonate kinase gene. Symptoms resemble TRAPS and FMF, with periodic fevers often with abdominal pain and arthralgia. One distinguishing feature is elevation of serum IgA and IgD concentrations, which was not a feature of the case described here. Adults Stills' disease can have intermittent fever, often with arthralgias, evanescent rash and elevated erythrocyte sedimentation rate (ESD) and serum ferritin. The short duration of attacks and the pain in the abdomen and muscles are not seen in Adult Still's disease. Immunoglobulin subtype deficiency with normal total IgG would not predispose to recurrent infections. Fevers of unknown origin, as in this case, would not suggest immunoglobulin deficiency. Correct Response TNF-receptor associated periodic fever (TRAPS)

A 57-year-old man seeks attention for intermittent fevers that have been present for more than 35 years. The febrile episodes began many years ago when he was a college student, continued to occur every one to two months, and last 4-6 days. During febrile episodes, he experiences extreme fatigue and "can't do anything." Between episodes he feels well and is a productive businessman. His only associated symptom or sign is a rash that typically occurs as a large, red, irregularly defined patch on his right buttock that extends down onto his posterior thigh. During the early years of his illness, he sought medical attention repeatedly and was hospitalized twice. No specific abnormalities were found and no diagnosis was made. Laboratory studies were always normal except for a persistently elevated sedimentation rate. After about ten years, he decided he would "just have to live with it" and stopped seeing physicians for febrile episodes. He is looking for answers again because his only child, a college student, has recently developed a similar illness. Which one of the following is the most likely diagnosis? Familial Mediterranean Fever TRAPS (tumor necrosis factor receptor associated periodic syndrome) Cyclic neutropenia Hyperimmunoglobulin D syndrome Chronic granulomatous disease

Correct answer: TRAPS (tumor necrosis factor receptor associated periodic syndrome) TRAPS is a rare, autosomal dominant disorder in which there is a defect in the gene that encodes the receptor for tumor necrosis factor. Patients may present from infancy to the 50s. Recurrent fevers over many years occurring every 5-8 weeks and lasting several days, in the absence of any evidence of infection are typical. Other features that may be present include myalgias, conjunctivitis, periorbital edema, abdominal pain and monoarticular arthritis. Rash is common, usually manifesting as one or several erythematous patches that may spread distally down an extremity. A minority of patients will develop amyloidosis. Familial Mediterranean Fever (FMF), an autosomal recessive disorder, may also present in adult life and is characterized by episodic attacks of fever accompanied by abdominal pain, or pleurisy, or arthritis. Rash is very rare in FMF. The exam is not likely to ask you to distinguish FMF and TRAPS. In cyclic neutropenia monthly episodes of profound neutropenia occur lasting about a week. Diagnosis typically is made in childhood. The patients are usually African American. Hyperimmunoglobulin D, an autosomal recessive periodic fever syndrome, is characterized by attacks of fever accompanied by chills, enlarged cervical nodes and sore throat, aphthous ulcers, and a pleomorphic rash; IgD levels are elevated. Chronic granulomatous disease is a disorder of neutrophils in which intracellular killing of certain organisms is impaired leading to recurrent infections, not unexplained fever.

You are consulted to see a 31-year-old woman on the neurology service who was admitted yesterday after an apparent transient ischemic episode. She was febrile on admission and reported having had fever for more than a week along with night sweats. On review of systems, she noted a five pound weight loss in the last week along with pain in both calf muscles after walking about a half mile. She works in a shelter for homeless people. On exam, she has a temperature of 101.6°F; pulse 100; BP 84/66. There is no rash and no murmur. She is tender bilaterally over her carotid arteries and has diminished peripheral pulses throughout. Her neurological exam is normal. Blood cultures from admission are negative at 24 hours. Chest x-ray and routine labs are normal except for a WBC count of 12,300 with 77% polymorphonuclear neutrophils. Which one of the following is the most likely diagnosis? Culture-negative endocarditis Temporal arteritis Moyamoya disease Takayasu's arteritis Atrial myxoma

Correct answer: Takayasu's arteritis Takayasu's arteritisis a large-vessel vasculitis of the aorta, its main branches, and pulmonary arteries. Most patients are young, otherwise healthy women who present with fever, weight loss, sweats, and myalgias along with TIAs, visual changes, strokes, asymmetrical blood pressure, carotid tenderness, extremity claudication, and diminished pulses. Diagnosis is made by arteriography or PET scan and treatment is with steroids. Temporal arteritis typically occurs in older persons and is associated with headache and visual changes. Common laboratory findings are marked elevation in inflammatory markers and anemia, which are variably present in Takayasu's arteritis. Moyamoya diseaseis confined to the cerebral circulation and results in ischemic strokes. Moyamoya disease is a rare, progressive cerebrovascular disorder caused by blocked arteries in the the basal ganglia. The name "moyamoya" means "puff of smoke" in Japanese and describes the look of the tangle of tiny vessels formed to compensate for the obstruction. The incidence is higher in Asian countries than in Europe or North America. The disease primarily affects children, but it can also occur in adults presenting as hemorrhagic stroke. In contrast to Takayasu's arteritis, fever is not seen with this disease. Therapy is surgical. There is no murmur to suggest endocarditis here and endocarditis would not explain claudication and carotid tenderness. Atrial myxomacan mimic endocarditis, producing fever and embolic phenomena but would not explain the diminished pulses, claudication, or carotid tenderness.

A 45-year-old married physician has been out of the work force for 15 years since her residency in New York City, while she raised her three children. She is returning now to patient care. She has Marfan syndrome, and is on warfarin anticoagulation for a mechanical aortic valve and prosthetic aortic graft. At her hospital orientation, she is found to have 12 mm of induration around her PPD (purified protein derivative) intradermal skin test at 48 hours after placement of the test. Her chest X-ray is negative. Her last PPD when she was a resident was negative but since then she has volunteered as a care provider at a homeless shelter that serves a clientele of undocumented individuals many of whom are from Latin America. You should advise her to: Repeat the PPD skin test in 1 to 3 weeks. Take a 6-month self-directed course of isoniazid. Take a 9-month self-directed course of isoniazid. Take a 12-week self-directed course of weekly isoniazid and rifapentine. Perform an interferon-γ release assay.

Correct answer: Take a 9-month self-directed course of isoniazid. This physician has obviously had potential exposures since her last PPD. Epidemiologically, she could have been infected with M. tuberculosis by one of her clients at the shelter. There two major issues here: 1) is the patient really latently infected; 2) is a rifapentine containing preventive regimen the best option for this patient? The use of directly observed therapy (DOT) is also an issue, but is more controversial. Tuberculosis skin tests are read at 48-72 hours; so, a 48 hour interval is acceptable. Tests may be repeated in 1-3 weeks if initially negative, to look for the "booster" phenomenon (an amnesic response), but this test is already positive and therefore does not need re-testing. Treating an individual for a positive PPD skin test is done based on the likelihood that the test represents latent tuberculosis infection. If present, latent tuberculosis has about a 2-5% of reactivating during the individual's lifetime. The likelihood of a positive test representing latent disease is based on the amount of induration and pre-test probability of infection. In this case - a physician with plausible patient exposure -10 or more mm of induration would be considered a positive reaction. Then the question is the appropriate preventive regimen; 9 months of isoniazid, unless there is concern for drug resistance based on exposure history, will reduce the risk of reactivation of tuberculosis by more than 90%. The 12-weekly course of isoniazid and rifapentine that is currently is recommended should be administered as directly observed therapy (DOT), although there may be different recommendations and practices about using DOT for this regimen. Importantly, especially for this patient, rifapentine has numerous drug interactions (warfarin, certain antidepressants, certain antihypertensives, oral contraceptives, methadone, etc.) and would be a poor choice for someone relying on warfarin for anticoagulation. In some hospitals, when prior BCG vaccination is likely, employees would be screened with an interferon-gamma release assay. The Quantiferon assays and T-SPOT assay have encountered problems of false positives in hospital screening in the United States, but the same could be said of tuberculin skin tests. Jereb et al, MMWR 2011:60(48);1650-4. Correct Response Take a 9-month self-directed course of isoniazid.

A 52-year-old male received a heart transplant two years ago and is doing well on cyclosporine, mycophenolate and prednisone maintenance therapy. His 12-month-old son is scheduled for routine immunizations including varicella vaccine. The father, who grew up in rural Mexico, is seronegative for varicella. He never received varicella vaccine and does not remember ever having chickenpox or zoster. What should you advise regarding management? The child should not be immunized. The child should be immunized and the father given VariZig (zoster immune globulin). The child should be immunized and removed from contact with the father if the child develops a rash associated with the vaccine The child should be immunized at which time the father should be given a 10-14 day course of valacyclovir. The father should be immunized prior to immunizing the child.

Correct answer: The child should be immunized and removed from contact with the father if the child develops a rash associated with the vaccine. The child should be immunized both to protect the child from chickenpox and to protect the father: if the child were unimmunized and introduced acute varicella into the household, the father could have a severe or fatal illness. The child is at exceedingly low risk for transmitting disease due to the vaccine strain even if the child were to develop a few cutaneous lesions from the vaccine. It is not necessary to give the father zoster immune gobulin or valacyclovir.

A 22-year-old male is seen in the emergency room with severe right testicular pain and fever. He had been healthy, working as a camp counselor in upper New York State. Eight days previously he had developed a fever and bilateral facial swelling involving his parotid glands. He was sent home from camp, felt better, but now, 5 days after onset of his facial swelling, he presents with unilateral testicular pain and high fever. He has been sexually active at camp with several female councilors, and has been hiking and swimming daily. He received all of his usual childhood immunizations, including MMR, according to his mother. Physical examination is normal, including rectal exam, except for exquisite right testicular tenderness. The left testicle was normal. He has minimal bilateral parotid swelling and no inguinal lymphadenopathy. CBC and Chemistry profile are unremarkable, as is a urinalysis. Which of the following is the most likely cause of orchitis in this vaccinated adult: mumps Chlamydia trachomatis Neisseria gonorrheae Escherichia coli Coxsackie virus

Correct answer: The orchitis is likely due to mumps. The important distinction in this case is recognizing that acute orchitis is usually viral but epididymitis is usually chlamydia or gonorrhea in young men and urinary tract bacteria in older men. If the distinction between orchitis and epididymitis is not clear on physical examination, ultrasound may be needed. This is a typical case of mumps orchitis: this complication typically occurs several days after the onset of parotitis and is unilateral in 70% of cases. Keep in mind that the effectiveness of the mumps component of the MMR vaccine is lower than that of the measles and rubella components. Estimates of the effectiveness of the mumps vaccine have varied in previous studies, ranging from 73% to 91% after 1 dose and from 79% to 95% after 2 doses. Coxsackie virus is a rare cause of orchitis and would not explain this patient's parotitis. Bacterial orchitis is rare but most often urinary tract organisms and associated with high fever. Chlamydia and gonorrhea cause epididymitis but rarely orchitis

A healthy, 20-year-old college student comes to the college health service with a 2-day history of fever, myalgias, and sore throat. There is no history of drug allergies and he is otherwise well. Examination is remarkable for an oral temperature of 39°C, enlarged tonsils with exudates and a few palpable, freely movable lymph nodes bilaterally in the posterior and anterior cervical triangles. A rapid strep test is negative, so he was only treated symptomatically. However, one day later the throat culture report indicated light growth of Group A streptococcus and the serum Mono Spot test is reported as negative. The student is contacted and requested to return to the health service on day four of his illness, but is reluctant because he is feeling better. Which of the following statement is true? The student should be treated with penicillin V to reduce the likelihood that rheumatic fever will occur. The student should be treated with cephalexin to reduce the likelihood that rheumatic fever will occur. Antibiotics should be withheld because it is too late to prevent nonsuppurative complications. The student is colonized, not infected, and requires no antibiotic therapy. The student's throat should be recultured and if still positive, penicillin should be given.

Correct answer: The student should be treated with penicillin V to reduce the likelihood that rheumatic fever will occur. The rapid test is highly specific, but only approximately 70% sensitive: cultures are done to pick up the 30% missed by the rapid tests. There is no easy and immediate way to tell who is colonized and who has disease. ASO or Streptozyme tests are insensitive early in the course of infection. Remember, there are four reasons to treat streptococcal pharyngitis: to reduce the duration of symptoms; to reduce suppurative complications; to reduce non-suppurative complications such as rheumatic heart disease; to reduce transmission to close contacts. If therapy is not started within the first two days of symptomatic illness, the duration of symptoms is not likely to be reduced by therapy. However, there are other reasons, per above, to treat. Therapy should be offered during the first 8-10 days after infection in order to reduce the likelihood of rheumatic heart disease as a sequelae; there is no effect on the incidence of glomerulonephritis. Admittedly, there is very little rheumatic disease anymore in the US, for reasons that are not clear, but this is still the recommendation. Early intervention can also reduce the duration of symptomatic pharyngitis. Density of growth on routine cultures is not adequate to distinguish colonization from infection. Repeat throat cultures are helpful in diagnosis of relapsed streptococcal pharyngitis but not in the initial therapeutic decision. First generation cephalosporins are effective in eradicating streptococcus from the pharynx but are not proven to prevent rheumatic fever.

A patient comes to see you because he was told his Chagas serology was positive when he went to donate blood. He is a healthy 30-year-old school teacher who grew up in Brazil. His exam, chest x-ray, and EKG are completely normal. Which of the following advice would you provide regarding the positive serology: The test may be a true positive but needs confirmation by a second test.The test is likely a true positive but, because he has no cardiac or esophageal disease, he is safe to donate blood.The test is likely a true positive but parasitemia only occurs acutely within the first few months after infection and thus he is safe to donate since he has not been to Brazil in many years.Even if the test were a true positive, treatment is not a consideration because there are no effective drugs for Trypanosoma cruzi.The test is likely a false positive because he has no evidence of cardiomyopathy or megaesophagus.

Correct answer: The test may be a true positive but needs confirmation by a second test. In the United States there are an estimated 100,000 immigrants with chronic Chagas disease. Immigrants from some areas have very high seropositive rates (i.e. >10%) and these individuals must be assumed to have low-level parasitemia even if they are asymptomatic with no signs of cardiac or esophageal disease. Many are identified when the offer to donate blood (or organs for transplant) and are found to be seropositive. These individuals have the potential to transmit Chagas by blood or organ donation, although surprisingly few such transmissions have been documented to occur. Because the ELISA test used to screen blood donors is sensitive but lacks specificity, positive tests should be confirmed by a second test which uses another method, such as the radioimmune precipitation assay (RIPA). The second choice is incorrect: he may well be parasitemic even if his examination and imaging and EKG are normal. The third choice is incorrect. This test could well be true positive in this native Brazilian since, as noted above, a substantial number of persons have chronic infection and chronic parasitemia without clinical manifestations. The fourth choice is incorrect because there are effective drugs. Benznidazole has proven efficacy against Chagas disease for an asymptomatic individuals and children with acute infection.

A 49-year-old female from Las Vegas presented to an Emergency Room. In the ER, she complained of fatigue and headache for 4 days, with low-grade fevers. Physical exam was remarkable only for modest splenomegaly and for some petechiae and a fever of 38.3°C. The patient is an avid hiker, drinks from mountain streams, spends weekends at her log cabin in the Nevada mountains, and had several anonymous sexual encounters over the past month at night clubs in Las Vegas. One week previously, she had a similar febrile episode that was more severe but resolved after 3 to 4 days. She did not seek medical attention, but comes in now that she is again ill. Hematocrit was 38% Platelet count was 25,000/µL. White blood count was 18,000/µL. Peripheral smear contained numerous spirochetes Which of the following is the likely source of infection? Man Tick Louse Mosquito Water

Correct answer: Tick This patient acquired Borrelia hermsii from a tick bite when she was asleep in her cabin. These soft-bodied ticks differ from hard-bodied ticks as the bites are usually painless. These ticks feed on numerous bite sites in an evening so some blood may be noticedon skin or bedclothes, but no eschar develops. Rodents and other small animals are often reservoirs. The incubation period is 3-12 days. Patients present with fever, headache, arthralgias, and myalgias. The platelet count is usually low, butthe WBC is normal or elevated. Diagnoses can be established by blood smear, culture, PCR or serology, but the serologies (including Western blot) often cross react with other Borrelia including Borrelia burgdorferi, the agent for Lyme disease. Typically blood smear examination for spirochetes offers the quickest mode of diagnosis as PCR, serology, andculture are only donein research or reference laboratories. Episodes last 1-3 days andmultiple relapses can occur at 4-14 day intervals. Doxycycline or penicillin for 5-10 days iseffective about 80% of the time, i.e, relapses may occur. Jarisch-Herxheimer reactions are common. Louse-borne relapsing fever does not occur in the United States. Syphilis and leptospirosis, also caused by spirochetes, don't fit this scenario.

A 21-year-old college student was admitted to the hospital in July with a fever of 40°C, dry cough, dyspnea and prostration of increasing severity over the past 24 hours. He had been previously healthy and was spending his summer on Martha's Vineyard as a groundskeeper for a golf course. He was living in a beach house with some college friends, with no pets in the house. He did not use alcohol or tobacco and had no history of recent travel. On admission he had crackles over both lung fields but no other abnormalities on physical examination. Bilateral pneumonia was present on chest x-ray. WBC was 4,100 with a normal differential. Hemoglobin was 15 and platelets 150,000. His AST and ALT were 2 x normal. He said his coworker at the golf course had been admitted with a similar illness the prior day to another hospital. Among the possible inhalation exposures that must be considered in the management of this patient, which of these is the most likely? Ticks or small rodents ground up by the lawn mower Cleaning out rodent nests from under the tool shed Scraping pigeon droppings off the patio Scrubbing the shower floor at the swimming pool Spreading compost on the flowers

Correct answer: Ticks or small rodents ground up by the lawn mower The geography should suggest Tularemia as more likely than some of the other options, especially hantavirus pulmonary syndrome (rodent nests) that would be more likely in the Southwest. Tularemia is usually acquired from an infected small animal through cutaneous inoculation of the animal's blood into a small cut or from a tick bite. However, machines that cut grass and brush can aerosol tissue or ticks from infected animals and cause pulmonary tularemia. When there is aerosol exposure, pulmonary infiltrates are bilateral, the wbc is often normal, and the transaminases are often mildly elevated. Diagnosis is usually made retrospectively by serology but effective treatment requires an aminoglycoside or perhaps a fluoroquinolone. Rodent nests can be a source of hantavirus pulmonary syndrome through the urine and feces. In this case there is no hemoconcentration, or biphasic illness, or geographic clue to make this the most likely diagnosis Pigeon droppings can be found to contain Cryptococcus but rarely appear to trace to cases of cryptococcosis. Cryptococcal pneumonia in previously healthy persons is indolent and without many systemic symptoms. Aerosolized water can be a source of Legionnaires' disease. Outbreaks of Legionnaires' disease are uncommon: since this is an outbreak, and there are no specific clues, Legionnaires' is a less likely cause than tularemia. Compost can lead to aspergillosis in patients with impaired immunity. Aspergillus does not cause invasive pulmonary disease in immunocompetent patients.

A 64-year-old healthy man with benign prostatic hypertrophy and Type 2 diabetes mellitus who had mild hemorrhagic stroke one month ago presented to the emergency room with high fever, chills, and difficulty urinating. His temperature was 40.2° C, blood pressure 80/50 mm Hg, pulse 124/min. The remainder of the exam was normal except for mild confusion and a tender distended urinary bladder to palpation. Laboratory Tests: WBC: 22,000/mm3 (94% PMN's, 6% lymphs) Hgb:12.4, platelets 164,000/mm3 Creatinine: 4 mg/dL Glucose 140 mg/dL Liver function tests: normal Urinalyisis: 100-200 WBC's The patient is volume resuscitated, but requires levophed to maintain an adequate blood pressure. Which of the following treatments on admission has been most clearly shown to improve survival for this febrile, hypotensive syndrome? Timing of initial effective antimicrobial therapy Low-dose dopamine Corticosteroids Central venous catheter to measure and allow titration of oxygen saturation Tight glucose control

Correct answer: Timing of initial effective antimicrobial therapy Of the 5 choices, only effective antimicrobial therapy achieved as early as possible has improved survival. There is clear data that the sooner effective antimicrobial therapy is administered for patients in septic shock, the better the outcome. Low-dose dopamine has no benefit in terms of survival. Steroids may make hypotension less refractory, but there is no evidence they improve outcome Tight glucose control is harmful and is no longer recommended. Swan ganz catheters and central venous catheters for monitoring venous oxygen saturation do not alter outcome for patients with septic shock although they can be useful in highly selected situations.

A 40-year-old previously healthy businessman returns from a trip with a localized rash that developed the previous day after he slept in a hotel. The rash is shown below. He found bugs on his mattress, which he put into a container to show you and which are shown below: Which of the following would be the most appropriate to prevent morbidity from this bite? DoxycyclineAmpicillinAcyclovirIvermectinTopical antihistamines or topical steroids

Correct answer: Topical antihistamines or topical steroids This 5mm insect is consistent with a bed bug, which has never been proven to transmit any infection. Thus A-D are incorrect. Bed bugs can produce rashes which develop within 30 minutes of the blood meal and persist for up to a week. The only useful therapy is topical antipruritic or anti-inflammatory products. Bed bugs usually live within 3-6 feet of humans in mattresses and other loci, and can live for up to a year. Bed bugs avoid light, and are rarely found on humans. Eradication is by cleaning and pesticides. The common bed bug (Cimex lectularius) is a wingless, red-brown, blood-sucking insect that grows up to 7 mm in length and has a lifespan from 4 months up to 1 year. Bed bugs hide in cracks and crevices in beds, wooden furniture, floors, and walls during the daytime and come out at night to feed on their preferred host, humans. Bed bug bites can result in small groups of extremely pruritic, erythematous papules or wheals that represent repeated feedings by a single bed bug. On occasion, more severe manifestations include grouped vesicles, giant urticaria, and hemorrhagic bullous eruptions. Bites are managed symptomatically with topical compounds including, topical corticosteroids and oral antihistamines. Bed bugs have never been shown to transmit disease to humans.

A 36 year old woman presented with a fever and skin eruption two weeks after starting lamotrigine for depression. She had also had a mild, non productive cough for about ten days preceding the initiation of lamotrigine for which she was given trimethoprim-sulfamethoxazole by her family physician. On examination she has a temperature of 38.3C, oral ulcers, and ulcerating skin lesions over 75% of her body. Her conjunctiva are inflamed. Her lungs are clear as is her chest radiograph. Her CBC shows a slight leukocytosis Arch Dermatol. 2008;144(6):724-726 The most likely diagnosis is: Erythema multiforme Toxic epidermal necrolysis Scalded skin syndrome Disseminated herpes simplex

Correct answer: Toxic Epidermal necrolysis Stevens-Johnson syndrome and toxic epidermal necrolysis are severe mucocutaneous reactions, usually triggered by medications started within the past 3 weeks. Patients have extensive necrosis and detachment of the epidermis. Mucosal and ocular involvement are characteristic. This patient likely had this reaction to either the trimethoprim-sulfamethoxazole or the lamotrigine. SJS and TEN are variants of a disease continuum which are distinguished by severity as measured by body surface area involved. They present with fever and malaise before the skin and mucous membrane lesions become apparent. SJS : skin detachment is <10 percent of the body surface Mucous membranes are affected in over 90 percent of patients, (ocular, oral, and genital). TEN involves detachment of >30 percent of the body surface area. Mucous membranes are involved in over 90 percent of patients. The drugs which most commonly trigger this are the following, with reactions occurring in the first 8 weeks of drug use Allopurinol Anticonvulsants Sulfonamides Lamotrigine NSAIDs Mycoplasma pneumoniae infection is the most common infectious trigger of SJS and TEN, This patient has a history that could be compatible with mycoplasma, and this infection could trigger either SJS or TEN, or erythema multiforme, but these lesions are not erythema multiforme (see below) SJS and TEN skin lesions typically begin with ill-defined, coalescing erythematous macules with purpuric centers, although many cases of SJS/TEN may present with diffuse erythema Lesions are tender and skin pain can be prominent and out of proportion to the cutaneous findings. They start on the face and thorax and do not involve the palms and soles. Patients are often described as showing Nikolsky sign (gentle rbbing on the skin loosens the epidermis. Mucosal involvement and ocular involvement occur in 90 percent of cases of SJS/and oral mucosa and the vermilion border are almost invariably involved. Treatment is supportive with emphasis on fluid and electrolytes, avoiding infection, and treating promptly any infections that occur. IVIG and corticosteroids have not been demonstrated effective although some clinicians use them. This is not erythema multiforme. Erythema multiforme usually presents with typical target lesions. Bullae and epidermal detachment are usually limited and involve less than 10 percent of the body surface area This is not Staphylococcal scalded skin syndrome. Staphylococcal scalded skin syndrome is caused by epidermolytic toxins produced by certain strains of Staphylococci but is usually seen in neonates and young children.. This presents with generalized erythema followed by the development of blisters and desquamation The mucous membranes are not involved. Scalded skin histology reveals sloughing of only the upper layers of the epidermis, in contrast with the subepidermal split with full thickness epidermal necrosis observed in SJS/TEN.

A 69-year-old male presents with a cough, unexplained weight loss of ten pounds, and night sweats for four months. Chest x-ray shows that he has a right upper lobe lesion with a cavity. He is intermittently febrile to 38°C, but has no other physical findings. CBC and chem-12 are normal. Three sputums and two bronchoalveolar lavages grow M. fortuitum but are negative for fungus. Cytologies on the BAL were negative. He does not smoke and has always worked as a lawyer. He has always lived in Boston and has not traveled outside the United States in the past few years. At this point, the best course of action is: Send urine for Histoplasma antigen Lung biopsy Empiric INH, rifampin, PZA, ethambutol, and then biopsy if no response in 3 months Treat when susceptibility testing of M. fortuitum is available

Correct answer: Treat when susceptibility testing of M. fortuitum is available The clinical findings in this patient are consistent with a variety of diagnoses, including squamous cell carcinoma, fungal pneumonia, or mycobacterial pneumonia. Cavitary pulmonary histoplasmosis usually occurs in patients with centrilobular emphysema and is rare in New England. Efforts to grow fungi or find tumor cells in the BAL have failed. Although concern about the diagnosis remains, it is likely that this patient is infected with M. fortuitum. Mycobacterium avium-intracellulare complex, Mycobacterium kansasii and the Mycobacterium abscessus group are the most commong nontuberculous mycobacteria causing lung disease in the United States, but M. fortuitum can cause cavitary disease in the lungs in previously healthy individual, especially in those with a history of GERD. Therapy can wait for the results of susceptibility tests since results can be available in a several days for these rapid growers. Isolates are often susceptible to ciprofloxacin, sulfamethoxazole, clarithromycin, azithromycin and doxycycline. Note, however, a large percentage of these isolates will display inducible macrolide resistance. A regimen with 2 to 3 oral drugs for 12 or more months would be recommended. Cefoxitin, imipenem, and amikacin are active agents and combination therapy would be reasonable if the patient were sick, which this patient appears not to be, or if the patient had a cavity, which this patient has. These IV drugs are not offered in this question as an option, but they could be used. A regimen for tuberculosis would not be effective in M. fortuitum disease.

A 48-year-old insurance company executive is seen for six days of unremitting fever and "aching all over." He says his leg muscles hurt when he walks, his arms hurt when he combs his hair, and his "eyes hurt" when he looks from side to side. He has no joint pain and no respiratory, urinary or gastrointestinal symptoms. On examination his temperature is 101.2°F. He has periorbital edema, a conjunctival hemorrhage, and subungual splinter hemorrhages in five fingers. There is tenderness on palpation of the trapezius and gastrocnemius muscles. Two months ago he went to Arizona on a business trip and played golf on a desert course. Two weeks ago he returned from a ten day vacation in Texas where fished for and ate fresh trout and hunted and ate wild boar, cooked rare. His hemoglobin is 14.2, his WBC is 18,000 (69% segs, 12% lymphs, 14% eos); his platelet count is normal. Which one of the following is the most likely explanation for his illness? ChikungunyaCoccidioidomycosisDengueEosinophilic granulomatosis with polyangiitis (Churg -Strauss syndrome)Trichinellosis

Correct answer: Trichinellosis Muscle pain and tenderness, splinter and conjunctival hemorrhages, periorbital edema and ocular pain, and eosinophilia are all cardinal features of trichinellosis (formerly trichinosis). Fever is often present and may be high. Trichinellosis has been reported worldwide; the prevalence is highest in China, Thailand, Mexico, Argentina, Bolivia, the former Soviet Union, and other parts of Central Europe. Infection is acquired by eating meat containing cysts of Trichinella spp., usually from inadequately cooked pork from domestic pigs outside the United States - domestic pigs in the United States are rarely the cause in recent years. Wild carnivorous animals harbor and perpetuate infections: walrus, bear, cougar, and wild boar. Undercooked wild game has emerged in recent years as a predominant source of infection in the United States; during 1997-2001, pork products were associated with 40% of cases, and wild game was associated with 60% (most frequent was bear meat). The incubation period is generally 7 to 30 days-this is an important clue here. Albendazole or mebendazole is the therapy of choice. Steroids may be used in severe cases. The other diseases listed explain some but not all the features of this patient's illness. Polyarteritis nodosa may cause fever, muscle pain and weakness. Dermatomyositis causes skin changes but not like this. Coccidioidomycosis, acquired in Arizona, can cause eosinophilia and fever, but is typically associated with pulmonary disease and would not generally cause periorbital edema or muscle tenderness. Eosinophilic granulomatosis with polyangiitis could explain blood eosinophilia and fever but asthma, a characteristic feature, is missing. As Dengue, Chikungunya, and Zika viruses spread to regions in the US, they will become increasingly part of the differential diagnosis, but the blood counts do not suggest these viral diseases, i.e., there are no cytopenias. These viral diseases would also not cause eosinophilia, periorbital edema, or subungual hemorrhages.

A 51-year-old woman is seen for a vaginal discharge. The discharge has been present for a week and is grayish-white. She recently began to use a vaginal spermicide. She says there is an unpleasant odor to the discharge. Intercourse is associated with dyspareunia. There is no pruritus. On exam a thin grayish vaginal discharge is noted. The vaginal pH is 5. A whiff-amine test is positive (fishy odor when 10% KOH added to sample of discharge). Numerous neutrophils are seen on Gram stain of the vaginal secretions. Assuming there is only one diagnosis for her symptoms, which of the following is most likely? Bacterial vaginosis Candida vaginitis Chlamydia cervicitis Spermicide hypersensitivity Trichomonas vaginitis

Correct answer: Trichomonas vaginitis This is a difficult question that tests your knowledge that bacterial vaginosis is NOT associated with inflammatory cells on the vaginal discharge nor is there usually dyspareunia. Grayish white discharge, vaginal pH >4.5, and a positive whiff test are typical of bacterial vaginosis, but are also seen with trichomoniasis. In symptomatic women with trichomoniasis, wet smears of vaginal secretions demonstrate motile trichomonads in 60-70% of patients and allow rapid confirmation of the diagnosis. Women with trichomoniasis have evidence of inflammation (neutrophils on microscopy) and often have dyspareunia; these two features are not usually present with bacterial vaginosis. Hypersensitivity to spermicides is rare and associated with evidence of inflammation without increase in vaginal pH.

A 7-year-old girl living in El Paso, Texas developed mild abdominal pain and eosinophilia. Bacterial culture of a stool specimen was negative. Ova and parasite (O&P) exam was positive for the organism in the figure. Which of the following organisms is responsible for her infection? Ascaris lumbricoidesEnterobius vermicularisNecator americanusToxocara canisTrichuris trichiura

Correct answer: Trichuris trichiura This patient is infected with T. trichiura, the whipworm. Infection is initiated by ingestions of infectious, embryonated eggs. After passing through the stomach, the egg hatches releasing the larvae that migrate to the large intestine. The adult worm attaches to the cecum or colon and begins egg laying. The eggs have a characteristic morphology: 50 to 55 µm long, 22 to 24 µm wide, barrel-shaped with a thick shell and clear plugs at each end. Infections are usually asymptomatic but adults may develop rectal prolapse.

A patient from Brazil with HIV infection, CD4=35 cells, Viral Load =90,000 copies/uL, presents with headache, a seizure, and the CT shown here. The diagnosis of toxoplasma is made presumptively. An LP is performed very cautiously: when the CSF is examined, the Toxoplasma PCR is negative but the organism below is seen. This is most likely which of the following: ToxoplasmaLeishmaniaTrypanosomeWuchereriaPlasmodium

Correct answer: Trypanosome The organism is Trypanoma cruzi, which infects person who are exposed to the feces of the reduvid bug. Chagas disease in patients with HIV and low CD4 counts can present identically to cerebral toxoplasmosis. Organisms can be seen in CSF, peripheral smear, or tissue biopsy. Consider this in patients from South and Central America, but especially Brazil. Reactivation of chronic T. cruzi infection has been reported to occur in patients with immunosuppression due to malignancy, chemotherapy, immunosuppressive regimens for solid organ or bone marrow transplantation, or HIV/AIDS Reactivation may occur in the absence of specific symptoms. Transplant recipients present with febrile episodes, rejection, or dermatologic manifestations, i.e. inflammatory panniculitis and skin nodules T. cruzi organisms may be visible in skin lesions. Chagomas in the CNS also occur. Often circulating trypomastigotes can be detected by light microscopy of fresh whole blood or buffy coat For persons with HIV/AIDS, T. cruzi activation occurs in about 20% of patients with HIV disease, low CD4 counts and chronic Chagas disease, ie poisitive Chagas serology. Patients can be parasitemic with or without systemic manifestaions. The most common manifestations of symptomatic T. cruzi reactivation in patients are meningoencephalitis or brain abscesses (chagoma which present like CNS toxoplasmosis. Acute myocarditis also occurs as do a variety of skin lesions.

A 38-year-old man from Nantucket was admitted three days ago with a dense pneumonia and a pleural effusion. He has been treated with azithromycin and ceftriaxone but is not improving. His illness began abruptly two days PTA with fever and chills; he began to cough the following day and developed pleuritic pain. He has scant sputum production. On exam, he is febrile with a normal pulse; there are crackles and percussion dullness in the right chest. The WBC is normal; the pleural fluid is exudative with a lymphocyte predominance. Admission sputum culture grew normal flora; the pleural fluid was sterile. He is an accountant, has no recent travel history, and no exposure to ill persons. His only possible animal exposure occurred 4 days before he became ill when he was mowing his lawn, and some kind of small animal darted out from behind a bush and was pulverized by the mower blades. Which one of the following is the most likely diagnosis? Primary tuberculosis Q fever Mycoplasma pneumonia Plague Tularemia

Correct answer: Tularemia This patient likely has tularemia pneumonia acquired through inhalation when an infected rabbit or rodent was aerosolized by his mower. Rapid onset, scant sputum, pleural involvement (exudative and lymphocytic effusion), temperature-pulse disparity, absence of visualization of the organism on sputum gram stain, and negative sputum cultures (many strains needed cysteine added to media for growth), and normal WBC count are all common findings. Primary TB can cause a lymphocytic pleural effusion but wouldn't likely be associated with a dense pneumonia and this kind of abrupt onset with chills. Q fever and Mycoplasma pneumonias may be associated with temperature-pulse disparity but the abrupt onset and pleural involvement would be unlikely. Plague pneumonia patients are desperately ill with pronounced leukocytosis.

This otherwise healthy patient, who has never left the Midwestern part of the United States, has a chronic leg ulcer. He has had a negative histopathology on biopsy for fungi, mycobacteria, Nocardia, or viral inclusions. This otherwise healthy patient with a chronic leg ulcer is most likely to have: Common variable immunoglobulin deficiencyLupus erythematosusHepatitis CUlcerative colitisMycobacterium ulcerans

Correct answer: Ulcerative colitis This is a typical lesion of pyoderma gangrenosa. Lesions characteristically are painful, irregular in shape, and have undermined borders. They can occur anywhere on the body but appear most often on the legs. The most common underlying disease is inflammatory bowel disease, more often due to ulcerative colitis than Crohn's, and skin lesions may precede the onset of bowel disease. Common variable hypogammaglobulinemia would present with a history of recurrent respiratory infections; not a chronic leg ulcer. Lupus can be associated with skin lesion but not a deep painful ulcer of this type. Cryoglobulinemia due to hepatitis C is associated with palpable purpura, not a single large ulcerative lesion. Mycobacterium ulcerans infection is not seen in the U.S.

A 31-year-old female (EBV IgG negative) with acute myelogenous leukemia-M2 in relapse was given a T cell depleted myeloablative allogeneic hematopoietic stem cell transplant from an EBV antibody positive donor. She engrafted with 100% chimerism and was doing well on day 79 with a WBC of 3,700/cu ml at which time she developed fever, severe sore throat and tender cervical lymphadenopathy. She had been receiving cyclosporine and prophylactic trimethoprim-sulfamethoxazole and valacyclovir. Cultures of the pharynx for HSV and Streptococcus pyogenes were negative. High resolution chest CT found a 3cm well defined nodule in the left upper lobe. Aspiration of a lymph node found predominately B cells with plasmacytoid differentiation, numerous mitotic figures and polyclonal light chain expression. Quantitative PCR of peripheral blood found 3,200,000 genome copies of EBV per ml and positive IgG antibody to EBV VCA. PCR of peripheral blood was negative for CMV. Which of the following best describes this patient's condition? Infectious mononucleosis Uncontrolled proliferation of EBV infected B cells EBV infection that will respond to ganciclovir B cell lymphoma EBV infection that will respond to cidofovir

Correct answer: Uncontrolled proliferation of EBV infected B cells This patient has post-transplant lymphoproliferative disorder (PTLD), a condition occurring in severely immunosuppressed patients. Fever, lymphadenopathy and lymphomatous infiltrates in lymph nodes, spleen, liver, bone marrow, kidney and lung are typical in this syndrome. Immune mechanisms controlling proliferation of EBV infection in her donor B cells are deficient, resembling the "immortalization" of B cells when infected with EBV in vitro. Treatment with antivirals has no effect. Reduction of immunosuppression in solid organ transplants can lead to improvement. In stem cell transplants, Rituximab has been used as therapy.

A 33-year-old primary school teacher comes to your office with fever and abdominal pain for 3 weeks which has persisted during the first three weeks of school. He has been in excellent health but is feeling increasingly fatigued since returning from a summer vacation in Mexico where he learned to make cheese. Evaluation shows peritoneal signs. After a CT scan, a peritoneal biopsy is performed: A smear is positive for acid fast bacilli which the microbiology lab later identifies by gene probe as M. tuberculosis. You begin the patient on a standard regime of isoniazid, rifampin, pyrazinamide and ethambutol. Three weeks later the laboratory reports that the isolate is resistant to pyrazinamide but sensitive to other drugs. Which of the following is likely source of this infection? Unpasteurized milk Inhalation exposure to an infected human Contaminated water Inhalation of contaminated earth Recurrence of a childhood infection

Correct answer: Unpasteurized milk Isolated PZA resistance should raise the possibility that the isolate is M. bovis and not M. tuberculosis, especially in someone who has traveled to Mexico recently and ingested potentially unpasteurized milk products. M. bovis is usually PZA resistant although some reports claim that a fraction of isolates are sensitive. M. bovis often causes abdominal lymphadenitis, and peritonitis. However, pulmonary involvement is not that uncommon. Person to person spread can occur, especially with HIV infected patients, but most cases are due to ingestion of raw milk or milk products produced outside the United States. There have been cases of M. bovis in the United States, especially in Southern California and New York. The majority are acquired outside the United States, often in Mexico. The commonly used gene probes, and conventional cultures, do not distinguish M. tuberculosisfrom other members of the M. tuberculosiscomplex, i.e. M. bovis, M. africanum, M.microti. Thus, M. bovisisolates are often reported as "M. tuberculosiscomplex"

A 55-year-old male CNN reporter arrived at Kennedy Airport in New York yesterday after spending two months in Somalia interviewing refugees about secret laboratories that terrorists might be building to produce agents of bio-terrorism. Some of the individuals who he interviewed had febrile illnesses with rash. For about one day, he had myalgias, low-grade headaches, and fever. Today, his wife noted a rash and he came to your office for evaluation. He feels like he only has a mild case of the "flu" and would like to go in to the office to catch up on his work. He reports that he was bitten by insects in Somalia, had one new sexual partner there, and took mefloquine and trimethoprim-sulfamethoxazole for prophylaxis of malaria and diarrhea, respectively. On physical examination, he is febrile to 38.5° C, has injected conjunctiva, and has lesions on his trunk and face, which are quite numerous; there are a few on his hands and legs but none on his palms and soles. Some lesions are pustular, some are vesicular. They started on his trunk. The most appropriate therapy for this patient would be: Valacyclovir Valganciclovir Foscarnet Ribavirin Cidofovir

Correct answer: Valacyclovir This patient most likely has chickenpox. About 10% of individuals over age 15 years are susceptible, and this patient may never have had the varicella vaccine. The incubation of chickenpox is 10-20 days; the incubation of smallpox is typically about 12 days, so this patient could have been exposed to either in Somalia. Since there have been no known cases of smallpox since 1977, exposure could only have occurred if a bio-terrorist had the virus and allowed the virus to be aerosolized, intentionally or unintentionally infecting lab workers or intended targets. Patients with smallpox usually feel very ill for 2-3 days prior to their rash: at the time of their prodrome they usually are so ill they stay in bed, which reduces contagion: this patient's prodrome and current illness are mild in that he wanted to go to work. In smallpox, the lesions are deep, all develop in 1-2 days, they start on the face and oral mucosa rather than the trunk, and they occur on the palms and soles. Thus, this patient almost certainly would have varicella. Treatment with valacyclovir would be an option, and post-exposure prophylaxis for those with whom he came into close contact should be considered. Characteristic of Lesions Chickenpox vs. Smallpox Lesion Characteristic 1_Depth relatively superficial 2_Deeply embedded in the skin; feel like firm round objects Lesion Distribution Centripetal (directed from periphery toward center), with rash much denser over the trunk than on the face and extremities; almost never found on the palms or soles Centrifugal (outward from center), with dense concentration on the oral mucosa and the face, spreading to the extremities, including the palms and soles Lesion Evolution New lesions appear in crops every few days for up to 6 days All lesions appear during a 1- to 2-day period Lesion Maturation Lesions at very different stages of maturation, with vesicles, pustules, and scabs present at the same time on adjacent sites (i.e., asynchronous) Lesions evolve at the same rate, so are all at the same stage of development on any given part of the body (i.e., synchronous)

Strains of Staphylococcus aureus have been isolated that are highly resistant to vancomycin (VRSA) The mechanism of resistance in these strains is mediated by which of the following gene clusters? mecAvanAvanBvanCileS-2

Correct answer: VanA Several S. aureus isolates have been isolated with an MIC of at least 32 mg/mL to vancomycin. These isolates are also typically methicillin resistant. These are called VRSA. The vanA gene cluster from Enterococcus species encodes a change in the cell wall target of vancomycin. It appears that VRSA is the result of the transfer of the vanA gene from vancomycin resistant enterococci to S. aureus. mecA confers methicillin resistance. The ileS-2 gene confers mupirocin resistance. Options for treatment of VRSA are not well defined but might include daptomycin, linezolid and ceftaroline.

A 43-year-old male teacher, previously in good health, presents with 4 days of fever and a sudden onset of headache, altered consciousness, and hemiparesis. There is no obvious source of the fever, and the patient has no unusual risk factors such as HIV infection or intravenous drug abuse. In the Emergency Room a CAT scan and then a CT angiogram is ordered, and a ruptured middle cerebral aneurysm is found which is described by the radiologist as "mycotic" based on its location and the characteristics of its wall. Pending neurosurgical intervention and cultures, the antibiotic(s) most likely to be active against the causative organism is/are: Fluconazole Liposomal Amphotericin B Vancomycin +/- gentamicin Ciprofloxacin plus gentamicin Azithromycin

Correct answer: Vancomycin +/- gentamicin This patient most likely has bacterial endocarditis, with common causes being Staph aureus (MSSA or MRSA), Viridans viridans streptococci, or Enterococci. Thus vancomycin is the most likely of the listed drugs to be active for this patient and is the usual recommendation as empiric therapy for endocarditis pending blood culture results: if this turned out to be MSSA, gentamicin would be optional; if this were MRSA, gentamicin would not be indicated per AHA guidelines. Mycotic aneurysms are uncommon complications of endocarditis that result from septic embolization of vegetations to the arterial vasa vasorum or the intraluminal space, with subsequent spread of infection through the intima and outward through the vessel wall. Arterial branching points favor the impaction of emboli and are the most common sites of development of mycotic aneurysms. They occur most frequently in the intracranial arteries, followed by the visceral arteries and the arteries of the upper and lower extremities The name mycotic aneurysm here is confusing since mycotic refers to fungi...except in this context! The term mycotic aneurysm was originally used for aneurysms associated with bacterial endocarditis because at autopsy these looked like "fresh fungus vegetations." Despite this, the majority of mycotic aneurysms are caused by bacteria and not by fungi, especially in this patient who has no risk for fungemia. Mycotic aneurysms usually occur where there is pre-existing pathology: atherosclerotic plaques, prosthetic grafts, aortic aneurysms. However, in the case of bacteremias, the arterial wall can become infected and the wall ultimately may dilate into an aneurysm. Such aneurysms can occur anywhere and the manifestations are the result of local dilatation, inflammation, or rupture. Intracranial mycotic aneurysms are usually clinically silent. Hemorrhage results in severe headache of sudden onset with rapid deterioration in the level of consciousness as in this patient. The time interval from diagnosis of endocarditis to the onset of hemorrhage is variable (0 to 35 days), with a mean of 18 days. This patient needs a neurosurgical consult to determine if a procedure is needed to repair the rupture (there is no consensus on how to manage cases), and standard therapy for endocarditis. However, in the case of bacteremias, the arterial wall can become infected and the wall ultimately may dilate into an aneursym. Such aneurysms can occur anywhere and the manifestations are the result of local dilatation, inflammation, or rupture. Ciprofloxacin might suggest Pseudomonas, which is a rare case of endocarditis primarily in intravenous drug abusers or occasionally in patients with indwelling catheters. The optimal drug is controversial. Ciprofloxacin might also suggest Salmonella, a cause of aortic aneurysms which are not common in the United States, although cephalosporins would probably be preferred. Azithromycin might suggest Bartonella endocarditis, but this entity is rare, being seen occasionally in patients exposed to lice, and has no particular association with mycotic aneurysms. However, in fact, tetracyclines are preferred for bartonella endocarditis.

A previously healthy 21-year-old male is admitted with the onset earlier that day of severe headache, followed by vomiting and lethargy. In the emergency room he was found to be poorly responsive to verbal stimuli, had nuchal rigidity, normal optic fundi and an otherwise normal neurologic examination. Blood cultures, WBC and differential were obtained and emergency CT requested prior to planned lumbar puncture. What regimen would you recommend? Vancomycin and ceftriaxone Vancomycin, ceftriaxone and dexamethasone Vancomycin, ampicillin and ceftriaxone Vancomycin, ampicillin, ceftriaxone and dexamethasone Ceftriaxone alone

Correct answer: Vancomycin, ceftriaxone and dexamethasone The presence of pneumococci with reduced susceptability to penicillin has made use of vancomycin plus ceftriaxone routine as a part of the initial, empirical regimen for patients with suspected pneumococcal meningitis. Addition of ampicillin to treat possible listeriosis is advisable for elderly and immunosuppressed adults, but not healthy young patients as this one. Addition of high-dose corticosteroid has remained controversial, but one study has convinced many of the experts that 4 days of dexamethasone 10 mg IV q6h, begun before or simultaneous with the administration of antibiotics, decreases mortality in patients with purulent meningitis. If the lumbar puncture indicates that purulent meningitis is not present, the dexamethasone can be stopped, and the diagnosis and therapy can be reassessed.

A 35-year-old Egyptian male, studying in Boston, was seen because of microscopic hematuria picked up on a routine physical examination. He underwent cytoscopy. A biopsy is shown of his bladder wall, where a roughened, dull surface was found in several areas of the bladder. He likely acquired this infection from which of the following sources? Wading in the Nile RiverEating poorly cooked lambDrinking well waterInsect biteEating human stool

Correct answer: Wading in the Nile River The eggs of Schistosoma haematobium are recognizable by the spike at the tip, not on the side like Schistosoma mansoni. The eggs originate in the adult schistosome living in a vein of the vesicle or pelvic plexus and migrate into the wall of the bladder, causing hematuria, and are then excreted into the urine. Excreted eggs hatch into miracidia which then infect snails which release motile cercariae. Infection is acquired when wading in water in which infected snails have produced cercariae that swim until they can invade the intact skin of the host. Parts of the Nile River have been notable for the population of infected snails and persons with schistosomiasis, both mansoni and haematobium.

What is the most likely source of the organism seen in this intestinal biopsy? WaterHamburgerRaspberriesHuman handsUnrefrigerated processed meat

Correct answer: WaterCryptosporidiosis is mostly commonly acquired in the United States through recreational water (waterparks, community swimming pools) or drinking water. The parasites, usually Cryptosporidium parvum or Cryptosporidium hominis, undergo their life cycle in the superficial cytoplasm of intestinal epithelial cells. The photomicrograph shows several oocysts. Once excreted in the feces, only a few oocysts are sufficient to infect another human. The low inoculum and prolonged viability allows water to be the usual vehicle of transmission, not person to person spread. Inhalation infection is unknown and food transmission appears to be uncommon. This parasite can be transmitted in day care from children's feces to day care provider hands and subsequent ingestion.

This 67-year-old man was brought to the hospital by the police in Washington, DC in because he was sleeping on a grate in bitter cold weather and, when asked to move along by the police, began muttering incoherently. In the emergency room he was combative and had to be restrained. He was admitted for observation and had numerous skin lesions such as the one shown. Which of the listed tests is most likely to be informative? Wet mount of skin scrapingFungal culture of skin scrapingAcid fast smear of skin scrapingSerum VDRLHIV ELISA

Correct answer: Wet mount of skin scraping The crusted, extensive skin lesions are typical of Norwegian scabies, a condition usually found in immunosuppressed patients, raising the possibility that this man has AIDS. The diagnosis is best made by wet mount of a skin scraping. The Sarcoptes scabiei mites and their feces are abundant in the lesions. Although this man may have HIV infection, HIV infection itself would not explain the lesions. Skin lesions of secondary syphilis are always a consideration, but are not typically pruritic and hyperkeratototic. Mycobacterium marinum and Mycobacterium chelonae are never hyperkeratotic, like the lesions shown.

A patient presents with 3 weeks of fever, 20lb weight loss, and cough. His chest radiograph shows a right upper lobe and middle lobe infiltrate but no cavity. A sputum specimen had been sent from the Emergency Department earlier in the day, and you are called by the lab - that sputum was negative for acid fast bacilli by fluorescent smear microscopy, but positive for Mycobacterium tuberculosis by GeneXpert MTB/RIF molecular testing; rifampin resistance was not detected by GeneXpert MTB/RIF, and culture results are not yet available. He is also found to be HIV infected (VL=400,000 copies/uL and CD4=10 cells/uL). Pending further drug-resistance testing results, he is started on isoniazid , rifampin, pyrazinamide, and ethambutol which is scheduled for two months to be followed by isoniazid and a rifamycin for an additional four months HIV therapy is deferred. The patient is homeless and there is no place to send him. Thus he will in in the hospital until he can come off isolation and be placed in a group home or shelter. When can this patient be taken off respiratory isolation in the hospital, assuming that his fever resolves and his cough improves? When three consecutive sputum smears are negative and he has received at least 2 weeks of TB treatment When three consecutive sputum cultures for TB are negative When three consecutive PCRs for TB are negative When his cough has resolved completely regardless of sputum results After 2 weeks of directly observed therapy regardless of sputum results

Correct answer: When three consecutive sputum smears are negative and he has received at least 2 weeks of TB treatment In this patient, the GeneXpert result confirms the diagnosis of pulmonary tuberculosis. GeneXpert MTB/RIF did NOT detect mutations associated with rifampin resistance. Given the very high negative predictive value of GeneXpert MTB/RIF for detection of rifampin resistance, this patient's M. tuberculosis is very likely to be susceptible to rifampin. Thus, conventional 4-drug TB treatment with rifampin, isoniazid, pyrazinamide, and ethambutol is indicated and should be initiated. While the patient is hospitalized, respiratory isolation should be continued until he has had three consecutive negative AFB sputum smear results, plus demonstrated clinical improvement, plus had a minimum of two weeks of antituberculosis treatment. This conservative approach is recommended by the CDC* based on uncertainty about (and individual variability in) how much TB treatment is required to render a patient non-infectious. Observational data show that untreated individuals with smear-negative TB can transmit M. tuberculosis to others. Nucleic acid amplification tests (e.g. GeneXpert MTB/RIF, PCR tests) may remain positive for months after initiation of effective treatment, and therefore should not be used routinely to assess infectiousness or response to treatment. Sputum cultures typically remain positive for several weeks after initiation of effective treatment, even though the patient may not be infectious to others. Negative sputum cultures are not typically a requirement for discontinuation of respiratory isolation (although this very stringent standard is sometimes used for patients with highly drug-resistant pulmonary TB who for whatever reason require ongoing hospitalization/stay in healthcare facility). There is no evidence that HIV infected persons with TB are any more infectious than TB patients who are HIV uninfected. In terms of general management, you should be aware that, after a diagnosis of active tuberculosis is established, or when tuberculosis is very high in the differential diagnosis, antituberculous therapy should be initiated. Initiation of multidrug oral therapy with isoniazid (INH), a rifamycin (eg, rifampin or rifabutin), pyrazinamide, and ethambutol is appropriate for adults a) whose GeneXpert MTB/RIF is positive for MTB but "rifampin resistance not detected", and b) in whom no rapid susceptibility test results are available but the risk of drug-resistance is considered to be low. Pending susceptibility results, isoniazid (plus pyridoxine=vitamin B6), a rifamycin, pyrazinamide, plus ethambutol should be administered for two months followed by isoniazid and a rifamycin for an additional four months. These drugs and the planned duration of therapy do not differ between HIV positive and HIV negative patients. Treatment should be given in association with the local health department, and directly observed therapy (DOT) should be used. With regard to dosing frequency, daily treatment should be used in this patient with low CD4 counts; twice-weekly treatment should NOT be used in this patient; three times per week DOT could be used by the health department if necessary to facilitate outpatient DOT. For patients who are hospitalized/in healthcare settings, daily (not intermittent) TB treatment always should be used. For patients with pulmonary TB and CD4 cell counts <50 cells/mm3, ART should be initiated within two weeks after initiation of TB treatment. This approach reduces the combined risk of an AIDS-defining illness and death, despite increased risk for TB-IRIS. (For patients with pulmonary TB and CD4 counts ≥50, ART should be initiated within 8 to 12 weeks after initiation of TB treatment.) During treatment of pulmonary tuberculosis, sputum should be obtained for AFB smear and culture at monthly intervals until two consecutive cultures (not PCRs) are negative to assess the adequacy of treatment (adherence and drug resistance) and to plan the duration of therapy. * https://www.cdc.gov/mmwr/preview/mmwrhtml/rr5417a1.htm?s_cid=rr5417a1_e

A patient presents with 3 weeks of fever, 20lb weight loss, and cough. His chest radiograph shows a right upper lobe and middle lobe infiltrate but no cavity. A sputum specimen had been sent from the Emergency Department earlier in the day, and you are called by the lab - that sputum was negative for acid fast bacilli by fluorescent smear microscopy, but positive for Mycobacterium tuberculosis by GeneXpert MTB/RIF molecular testing; rifampin resistance was not detected by GeneXpert MTB/RIF, and culture results are not yet available. He is also found to be HIV infected (VL=400,000 copies/uL and CD4=10 cells/uL). Pending further drug-resistance testing results, he is started on isoniazid, rifampin, pyrazinamide, and ethambutol which are scheduled for two months to be followed by isoniazid and a rifamycin for an additional four months HIV therapy is deferred. The patient is homeless and there is no place to send him. Thus, he will stay in the hospital until he can come off isolation and be placed in a group home or shelter. When can this patient be taken off respiratory isolation in the hospital, assuming that his fever resolves and his cough improves? When three consecutive sputum smears are negative, there is clinical improvement, and he has received at least 2 weeks of TB treatment When three consecutive sputum cultures for TB are negative When three consecutive PCRs for TB are negative When his cough has resolved completely regardless of sputum results After 2 weeks of directly observed therapy regardless of sputum results

Correct answer: When three consecutive sputum smears are negative, there is clinical improvement, and he has received at least 2 weeks of TB treatment In this patient, the GeneXpert result confirms the diagnosis of pulmonary tuberculosis. GeneXpert MTB/RIF did NOT detect mutations associated with rifampin resistance. Given the very high negative predictive value of GeneXpert MTB/RIF for detection of rifampin resistance, this patient's M. tuberculosisis very likely to be susceptible to rifampin. Thus, conventional 4-drug TB treatment with rifampin, isoniazid, pyrazinamide, and ethambutol is indicated and should be initiated. While the patient is hospitalized, respiratory isolation should be continued until he has had three consecutive negative AFB sputum smear results, plus demonstrated clinical improvement, plus had a minimum of two weeks of anti-tuberculosis treatment. This conservative approach is recommended by the CDC* based on uncertainty about (and individual variability in) how much TB treatment is required to render a patient non-infectious. Observational data show that untreated individuals with smear-negative TB can transmit M. tuberculosisto others. Nucleic acid amplification tests (e.g. GeneXpert MTB/RIF, PCR tests) may remain positive for months after initiation of effective treatment, and therefore should not be used routinely to assess infectiousness or response to treatment. Sputum cultures typically remain positive for several weeks after initiation of effective treatment, even though the patient may not be infectious to others. Negative sputum cultures are not typically a requirement for discontinuation of respiratory isolation (although this very stringent standard is sometimes used for patients with highly drug-resistant pulmonary TB who for whatever reason require ongoing hospitalization/stay in healthcare facility). There is no evidence that HIV infected persons with TB are any more infectious than TB patients who are HIV uninfected. In terms of general management, you should be aware that, after a diagnosis of active tuberculosis is established, or when tuberculosis is very high in the differential diagnosis, anti-tuberculous therapy should be initiated. Initiation of multidrug oral therapy with isoniazid (INH), a rifamycin (eg, rifampin or rifabutin), pyrazinamide, and ethambutol is appropriate for adults a) whose GeneXpert MTB/RIF is positive for MTB but "rifampin resistance not detected", and b) in whom no rapid susceptibility test results are available but the risk of drug-resistance is considered to be low. Pending susceptibility results, isoniazid (plus pyridoxine=vitamin B6), a rifamycin, pyrazinamide, plus ethambutol should be administered for two months followed by isoniazid and a rifamycin for an additional four months. These drugs and the planned duration of therapy do not differ between HIV positive and HIV negative patients. Treatment should be given in association with the local health department, and directly observed therapy (DOT) should be used. With regard to dosing frequency, daily treatment should be used in this patient with low CD4 counts; twice-weekly treatment should NOT be used in this patient; three times per week DOT could be used by the health department if necessary to facilitate outpatient DOT. For patients who are hospitalized/in healthcare settings, daily (not intermittent) TB treatment always should be used. For patients with pulmonary TB and CD4 cell counts <50 cells/mm3, ART should be initiated within two weeks after initiation of TB treatment. This approach reduces the combined risk of an AIDS-defining illness and death, despite increased risk for TB-IRIS. (For patients with pulmonary TB and CD4 counts ≥50, ART should be initiated within 8 to 12 weeks after initiation of TB treatment.) During treatment of pulmonary tuberculosis, sputum should be obtained for AFB smear and culture at monthly intervals until two consecutive cultures (not PCRs) are negative to assess the adequacy of treatment (adherence and drug resistance) and to plan the duration of therapy. Positive sputum culture at two months should prompt repeat drug susceptibility testing of that isolate. * https://www.cdc.gov/mmwr/preview/mmwrhtml/rr5417a1.htm?s_cid=rr5417a1_e

A 56-year-old housewife living in rural Colorado is hospitalized for pneumonia. Her husband came home that evening and found her prostrate on the couch. The husband says she has been healthy. When he left that morning, she was well and was peering into the throat of the family cat which seemed to be sick, coughing or choking. The cat normally roams outside and she said she feared the cat had eaten something that got stuck in its throat. Her exam: Temp 104.2F; P 140, BP 90/70; RR 36. She looks quite ill and is coughing frequently, producing bloody, purulent sputum. Crackles in chest bilaterally. WBC 29,500 with 70% segs and 15% bands; platelets 86,000. Which one of the following is the most likely cause of her pneumonia? Yersinia Legionella Hantavirus Bartonella Adenovirus

Correct answer: Yersinia This severe illness is most consistent with plague pneumonia due to Yersinia pestis. Most human plague follows bites from rodent fleas with subsequent regional lymphadenopathy; pneumonic plague often follows inhalation of infected aerosol droplets from humans or animals. Infected cats often appear to be choking. In primary plague pneumonia the incubation period is hours to a few days, and the clinical onset is sudden with SOB, high fever, pleuritic pain, and cough producing bloody sputum. Pneumonic plague can be primary or secondary. Primary pneumonic plague can be acquired by inhalation of respiratory secretions or aerosolized droplets from infected animals or humans, or by laboratory exposure. Secondary pneumonic plague is more common and arises via hematogenous spread of bacteria from a bubo or other source. . Legionella and hantavirus can cause severe pneumonia, but bloody sputum would be very unlikely. Hantavirus results from exposure to rodent droppings and often is associated with a profound thrombocytopenia. Bartonella is carried by cats, but the cats are not ill, and Bartonella doesn't cause pneumonia. Adenovirus can cause hemorrhagic pneumonia in immunosuppressed individuals, but would be quite unlikely in this patient.

Many important pathogens require specialized media for recovery. The laboratory must be notified if a specific organism is suspected, otherwise the organism may not be detected. Which one of the following organisms can grow on sheep blood agar, the usual standard media? Yersinia pestis Neisseria gonorrhoeae Haemophilus influenzae Francisella tularensis Legionella pneumophila

Correct answer: Yersinia pestis Y. pestis is a member of the family Enterobacteriaceae and grows on most nonselective media and media for Gram-negative bacilli (e.g., MacConkey agar). The other organisms listed have selective growth requirements. N. gonorrhoeae and H. influenzae grow on chocolate agar but not blood agar. F. tularensis requires cysteine and grows on chocolate agar but not blood agar; it grows best on buffered charcoal yeast extract (BCYE) agar; however, L. pneumophila, requires cysteine and iron; grows best on buffered charcoal yeast extract [BCYE] agar. Some notable other organisms that require specialized growth media include Bordetella pertussis, which grows on Regan Lowe and Bordet-Gengou media, Campylobacter species, which grow on Campy agar in microaerophilic atmospheres, and Leptospira species which grows in Fletcher's medium

This 18-year-old Navajo man was brought to the Four Corners Hospital in Arizona because of high fever and the lesions shown. He looks quite ill. His mother thought he might have been bitten by a rat while he was sleeping, because he awoke crying of pain because of the abdominal lesion. She had seen a dead rat in the garage a few days prior. On exam, he had a temp of 40ºC and the lesions seen. Both the lesion and the axillary area was were very tender. Gram stain of the skin lesion found no organisms. The most likely pathogen is which of the following: Yersinia pestisStreptobaccilus moniliformisSpirillum minusEikenella corrodensPasteurella multocida

Correct answer: Yersinia pestisHigh fever, a local lesion and a tender swollen lymph node (bubo) with prostration suggests plague, tularemia or a staphylococcal abscess, the latter two not being on the list above. Staphylococcal furuncles would have been at the site of minor trauma and more indolent. Eikenella and Pasteurella multocida (as opposed to Pasturella pestis now known as Francisella pestis) typically do not give a bubo, though they may cause sepsis from a dog or cat bite. Spirillum minus is a rare disease from rat bites in Japan. This is not seen in the US. Streptobaccilus also causes rat bite fever, without a bubo but often with rash and arthritis. Plague is important to diagnose because the best drug is gentamicin, not something typically used for skin and lymphatic infection. This man presumably was bitten on the abdomen by a flea from a rat, developed the abdominal lesion, and then the bubo.

A 26-year-old man presents to the emergency room with one day of right lower quadrant pain. He reports fever and one episode of nausea and vomiting. He has been in good health except for self-limited mild-moderate diarrhea 7 to10 days prior. He denies any past medical history. He is a college school student and lives in rural Mississippi. He has no history of alcohol or illegal substances. He frequently eats raw oysters and chitterlings (cooked intestines of young pigs). He has a pet turtle. On examination, he is febrile to 101°F, other vital signs are within normal limits except for a mild tachycardia. He has decreased bowel sounds and right lower quadrant tenderness with mild rebound and guarding. His WBC is 14000. CT of the abdomen reveals a thickened terminal ileum and enlarged mesenteric lymph nodes. There is no apparent edema or inflammation of the appendix. Which of the following organisms would most likely be a specific cause of this clinical syndrome? Vibrio vulnificus Salmonella enterica Shigella sonnei Campylobacter jejuni Yersinia pseudotuberculosis

Correct answer: Yersinia pseudotuberculosis Mesenteric adenitis, with or without terminal ileitis can produce a syndrome that is clinically indistinguishable from appendicitis (pseudo-appendicitis). On exploratory laparotomy, the appendix usually looks normal in these patients. The most common reported bacterial causes of mesenteric adenitis are Yersinia enterocolitica and Yersinia pseudotuberculosis. Yersiniosis is a zoonosis and is associated with exposure to swine, cattle, and rarely with contaminated water. Unpasteurized milk has been the source of several reported cases. Salmonellosis can be acquired from pet turtles or food sources. Salmonella, Campylobacter and Shigella do not cause terminal ileitis or mesenteric adenitis. Vibrio vulnificus can be acquired from oysters but causes sepsis in patients with severe liver disease and not mesenteric adenitis.

A 42-year-old woman was diagnosed with HIV 10 months ago in India and was started on an antiretroviral regimen at that time. She mentions that for the last month it has been difficult for her to climb stairs and get up from chairs, and she feels overall weak. She denies myalgias. Her strength was normal before starting medications. On exam, the strength in her hip flexors is markedly decreased (3+/5), and the sensory exam is normal including deep tendon reflexes. Her creatine kinase level is 550 (twice normal), but her chemistry and hematology panels are otherwise unremarkable. Her electrolytes including calcium and phosphate and lactic acid and urinalysis are normal. Which of the following medications is the most likely explanation for her symptoms? Raltegravir Lopinavir-ritonavir Zidovudine atazanavir Rilpivirine

Correct answer: Zidovudine Zidovudine is used uncommonly today in the U.S. because of toxicities. Myopathy is a relatively frequent adverse effect of zidovudine (AZT), seen in as many as 17% of patients treated with zidovudine for prolonged periods of time. The symptoms include weakness, mainly in the lower extremities, difficulty rising from chairs or climbing stairs, and myalgias with symmetric proximal weakness especially in hip flexors. The CPK levels are usually mildly elevated (median 500), but can reach very high levels. Muscle biopsy reveals characteristic findings. The pathogenesis is thought to be related to mitochondrial toxicity through inhibition of gamma;-DNA polymerase and energy shortage in the muscle. Most patients improve within several months of stopping zidovudine. There is no hypophosphatemia, glycosuria or proteinuria to make you think about tenofovir producing renal tubular dysfunction with phosphaturia and muscular toxicity. However, if a similar question comes up on the exam, don't miss a tenofovir induced Fanconi syndrome leading to hypophosphatemia.

A 32-year-old female presents to the clinic for evaluation. She has been on tenofovir (TDF)/emtricitabine/ efavirenz for 18 months. She admits to frequent inadherence to her regimen and at this visit has a CD4 count of 198 and a viral load of 27,500 copies. A genotype is done and reveals the following mutations: Nucleosides: K65R, M184V Non nucleosides: K103N Which of the following regimens would be the best choice for this patient? Tenofovir, emtricitabine, efavirenz Zidovudine, lamivudine, dolutegravir Tenofovir, emtricitabine, darunavir, ritonavir Abacavir, lamivudine, dolutegravir

Correct answer: Zidovudine, lamivudine, dolutegravir The K103N mutation rules out efavirenz as a viable option, and thus the first choice is incorrect. K65R confers high level resistance to tenofovir, thus making choices 1 and 3 incorrect . M184V (associated with emtricitabine and lamivudine resistance) also weakens the effect of abacavir, especially when a K65R is present. The best nucleoside for use when a K65R mutation is present is zidovudine. While it is less well tolerated, it is the best choice in this setting. Either boosted darunavir or dolutegravir should work here, but the only answer with zidovudine as an option is answer 2. M184V if present, will "resensitize" (re-confer some virologic activity) for both zidovudine and tenofovir, and while 3TC activity is reduced it will still have some activity. Thus the best regimen for this patient must not contain efavirenz, tenofovir, or abacavir and should contain zidovudine in preference to the other nucleosides, which leaves the second choice as the best answer.

A 44-year-old HIV positive male user of IV drugs has been treated with efavirenz, tenofovir plus emtricitabine for one year. At the time of initiation of antiretroviral therapy has was seronegative for HCV Ab as well as for HBsAb, but he was seropositive for HBcAb. He repeatedly refused hepatitis B immunization even though he is sexually active without practicing safe sex. He returns for routine monitoring. He is asymptomatic but is noted to have an ALT of 543 IU/L, AST 346, with normal alkaline phosphatase and bilirubin. Other lab tests: Plasma HIV RNA: <50 copies/uL; CD4 count 454/mm3 (baseline 210/mm3) Anti-HBc positive, antiHBs negative, HBsAg negative. What is the most likely cause for his asymptomatic elevation in liver function tests? HBV-related IRIS Efavirenz Acute HCV infection HBV breakthrough (resistance to tenofovir/emtricitabine)

Correct answer: acute HCV infection There are recent outbreaks of HCV infection among HIV positive MSM and this is a typical case. He has never had convincing evidence of HBV infection: an isolated HBc could conceivably represent chronic HBV infection (this is controversial without HBsAb and without plasma HBV DNA), but even if he did have chronic HBV, a flare (or immune reconstitution syndrome) while on tenofovir plus emtricitabine would be unlikely since both are active against HBV, and resistance to either or both is unusual. Efavirenz, like many drugs, can cause hepatitis, but this toxicity is uncommon with efavirenz. Correct Response Acute HCV infection

A 56-year-old man six months post-heart transplant complicated by steroid-induced diabetes and transplant rejection, requiring rituximab three months prior, sustained a laceration on the lower leg after cutting his leg on a lawn mower blade. Three days later, he developed erythema of the tissues surrounding the lesion associated with increasing pain and fever. He was started on Vancomycin empirically. Despite the vancomycin, the cellulitis progressed and he developed a black eschar over the wound (see image). There was no crepitus. A punch biopsy of the wound was performed in the emergency department and the KOH preparation of the tissue biopsy is shown below. The best therapeutic intervention, in addition to debridement, would be: Intravenous liposomal amphotericin B aloneIntravenous voriconazole aloneIntravenous liposomal amphotericin B plus deferoxamineIntravenous voriconazole plus deferoxamineOral posaconazole alone

Correct answer: amphotericin B alone This is cutaneous mucormycosis which is caused by fungi from the subphylum Mucoromycotina. Note the broad hyphae and absence of septae. This calcofluor stain under UV light would show septae very clearly but none are present. Mucoromycotina are ubiquitous in nature, and can be found on decaying vegetation and in the soil. Infection of the skin and soft tissues usually results from inoculation of the fungal spores into the dermis and accordingly is almost always associated with trauma or wounds. Cutaneous mucormycosis usually appears as a single, painful, indurated area of cellulitis that develops into an ecthyma-like lesion. Patients who have suffered trauma with an open wound that was contaminated with spores can develop rapidly progressive tissue necrosis reflecting the presence of ischemic infarction. Dissemination and deep tissue involvement are unusual complications of cutaneous mucormycosis, but need to be considered in the immunocompromised host. Radiographic imaging is useful to help determine the extent of involvement, but should not delay surgical intervention. Treatment of mucormycosis involves a combination of surgical debridement of involved tissues and antifungal therapy. Amphotericin remains the drug of choice for mucormycosis; most experts would use a lipid formulation to deliver high doses with less nephrotoxicity, particularly in a transplant recipient who is likely also receiving a calcineurin inhibitor. Posaconazole could potentially be used in the intravenous form or the new tablet formulation, however, the in vitro activity of posaconazole is not predictable. Further, posaconazole is probably not as effective as amphotericin B for the therapy of mucormycosis and is not FDA-approved for primary or salvage therapy of mucormycosis. Deferasirox is not useful in treating mucormycosis, based on a small clinical trial.

A 40-year-old patient with HIV (CD4= 200) has just returned from a beach vacation in New York State and now presents with fever, profound shock, and a hemoglobin of 6 g/dl. He is intubated and started on pressors. Lab values suggest severe hemolysis. His peripheral smear is shown in the figure. The treatment of choice for the most likely cause of this patient's illness is: Choices A Doxycycline B Imipenem C atovaquone and azithromycin D E Mefloquine Artemether-ljumefantrin (Coartem

Correct answer: atovaquone and azithromycin This is babesiosis: the best treatment option for babesiosis is atovaquone plus azithromycin Exchange transfusion is useful when disease is severe. Fever and hemolysis should raise a limited differential including thrombotic thrombocytopenic purpura (TTP) and E. coli induced hemolytic uremic syndrome, but in this case malaria and babesiosis are suggested by the smear. A good morphologist can recognize this as Babesia, although the lack of foreign travel helps considerably. These organisms look like plasmodia in that they are intraerythrocytic rings, but malaria and Babesia can be distinguished by morphology, PCR, or serology. This is Babesia acquired probably on the Eastern coast such as Shelter Island or Martha's Vineyard from the Ixodes scapularis tick; there is also B. divergens on the West coast and there is Babesia in Europe. Babesia can be especially severe in patients with splenectomy or advanced age or immunosuppression including that due to HIV infection. Watch also for cases in patients who have received TNF blocking agents or rituximab.

A 19-year-old college student was brought to the hospital emergency room because of fever, headache, and extreme malaise of one day's duration. While waiting to be seen, these skin lesions appeared, and the patient was found to have developed shock. While venous access for fluid resuscitation was being obtained, the patient had a cardiac arrest and was intubated and resuscitated. What preventive measure should be advised for the team who intubated and resuscitated the patient? Throat cultureDoxycyclineCiprofloxacinAmpicillinImmunization booster

Correct answer: ciprofloxacin The rash is characteristic of meningococcal sepsis. Personnel exposed to respiratory secretions of this patient during resuscitation should receive antimicrobial prophylaxis. Other emergency room personnel would not need prophylaxis. Throat cultureis not rapid or sensitive enough to be useful to detect early colonization or infection. Ampicillindoesn't penetrate nasopharyngeal secretions and has not been found to be effective. Doxycyclineis not useful for meningococcal prophylaxis, but would be useful for therapy if this patient had Rocky Mountain Spotted Fever. Either rifampin600 mg q12 hours for four doses, ceftriaxone250 mg IM once, azithromycin500 mg po once, or ciprofloxacin500 mg po once would be sufficient for the resuscitation team members. Rifampin is now rarely used due to the need for multiple doses and the potential for drug interactions.

A rash on an adult that came bumps, flattening out, and become hemorrhagie. was found on a stuporous adult one morning. He had appeared well the night before other than some "flu like" symptoms. Blood cultures of this patient are likely to grow which of the following: Choices A Gram negative cocci B Gram positive cocci C Gram negative bacilli D Gram positive bacilli

Correct answer: gram negative cocci This patient has purpura fulminans, usually due to Neisseria meningitidis. Purpura are nonblanchable, hemorrhagic skin lesions that result from the leakage of red blood cells into the skin There are many processes that can cause similar lesions Idiopathic thrombocytopenic purpura (ITP} Thrombotic thrombocytopenic purpura ( TTP} Warfarin and Heparin Cryoglobulins Calciphylaxis Sepsis and DIC due to a wide variety of bacteria Purpura fulminans is a severe complication of meningococcal disease characterized by the acute onset of cutaneous hemorrhage and necrosis due to infection of capillary endothelial cells with subsequent vascular thrombosis and disseminated intravascular coagulopathy. The first manifestations is usually cutaneous pain followed by erythema and petechiae. Ecchymoses develop and evolve into painful indurated, purple papules with erythematous borders and then evolve into necrosis and bullae and vesicles. Patients often have concurrent meningitis. Meningococcal disease is one of the most devastating infections. Disease due to Neisseria meningitidis attacks young, previously well individuals and can progress over hours to death. Mortality can be very high and long-term sequelae can be severe. The mortality and morbidity from meningococcal disease has changed very little since the 1950s. Keep in mind that Complement deficiency involving early and late components of the complement system have been associated with increased susceptibility. Eculizimab, a monoclonal antibody complement inhibitor used for treatment of hemolytic uremic syndrome and paroxysmal nocturnal hemoglobinuria, predisposes to meningococcemia. there may also be an association between HIV infection and meningococcemia but this is controversial

A 23-year-old nurse, 8 weeks pregnant, sought advice from her obstetrician. For the past two weeks she has been taking care of a hospitalized child with sickle cell disease and aplastic crisis. For the past five days she has had low grade fever, headache, the mildly pruritic rash shown here and aching joints with stiffness in her hands and feet. She had all the usual childhood vaccinations, was taking no medications, lived alone with her husband, and had no pets. The major concern for her unborn infant would be which of the following: deafnesshydrops fetalisthrombocytopeniacongenital heart diseasemental retardation

Correct answer: hydrops fetalis Patients with chronic hemolytic diseases, such as sickle cell disease, are susceptible to complications of Parvovirus B 19 infections (also called Erythrovirus B19) which manifests as severe anemia or aplastic crisis. Patients with immunosuppression such as AIDS patients and transplant recipients can develop chronic infection with Parvovirus B19 and can shed virus in their secretions, presenting risk of infection to their contacts. Although many adults have serologic evidence of prior infection, previously uninfected pregnant women may transmit the virus to the fetus if they are acutely infected during pregnancy. Arrest of erythrocyte production leads to severe anemia and congestive heart failure in the fetus. This leads to hydrops fetalis. Intrauterine transfusion may salvage the pregnancy but fetal loss is a serious risk. The other fetal abnormalities are characteristic of congenital rubella (deafness and congenital heart disease), toxoplasmosis (hydrocephalus, retinitis, disseminated disease) or cytomegalovirus infection (disseminated disease)

A 28 year female injection drug user is admitted for tricuspid valve endocarditis. Transthoracic echocardiogram shows presence of a 1 cm mobile mass on the tricuspid valve. Two months ago she was treated with vancomycin for four weeks for bacteremia caused by methicillin-resistant Staphylococcus aureus (vancomycin MIC=1 mcg/ml, daptomycin MIC = 1 mcg/ml). Three of three blood cultures are again positive for MRSA (vancomycin MIC=4 mcg/ml, daptomycin MIC=1 mcg/ml). Daptomycin 8 mg/kg IV once daily is administered. After 8 days of daptomycin, two of two blood cultures, obtained because of recurrent fevers, are positive for MRSA (vancomycin MIC=4 mcg/ml, daptomycin MIC=4 mcg/ml). Which one the following genes is mediating the increase in daptomycin MIC? dapR mecA mecC mprF vanA

Correct answer: mprF There are only a few bacterial resistance genes you need to know. There is no such thing as a dapR gene in staphylococci. mecA and mecC mediate methicillin/beta-lactam class resistance, and do not confer resistance to other drug classes. vanA mediates vancomycin resistance (MIC > 8) not resistance to daptomycin and in any case, with a vancomycin MIC=4 this strain has the phenotype of vancomycin intermediate S. aureus (VISA) which is genetically distinct from, and not mediated by, vanA. Daptomycin non-susceptible strains of S. aureus usually have a mutation in mprF, which encodes a membrane protein that modifies membrane lipid phosphatidylglycerol (PG) by enzymatic transfer of L-lysine to PG, imparting a net positive charge. The mutation enhances this transfer activity, thereby increasing outer membrane charge positivity over what is normally present. This interferes with binding of daptomycin, which is complexed with Ca++ and also positively charged, to the membrane as a result of charge repulsion. Correct Response mprF

This MRI came from a 66-year-old male taking hydroxycholoroquine and steroids for rheumatoid arthritis for 5 years and who had a three month history progressive right sided weakness and a one month history of progressive aphasia. He was afebrile. A noncontrast CT is shown on the left and a T2-weighted noncontrast MRI on the right. The most likely cause among the following is: herpesvirusenteroviruspolyoma viruslymphomatoxoplasmosis

Correct answer: polyoma virus Progressive multifocal leukoencephalopathy is a slowly progressing disease of immunosuppressed patients, resembling multiple sclerosis on brain MRI because the majority of the lesions are in the white matter and don't enhance on MRI or CT with contrast. Lesions on MRI are white (dense) on T2 imaging and usually don't enhance with contrast. JC, a polyoma virus that causes PML, can be detected in the CSF of many PML patients by PCR. The preferred term is now JC encephalitis instead of PML. JC encephalitis (PML) has also been reported in patients treated with HIV and efalizumab, belatacept, fludarabine, infliximab, rituximab, mycophenolate, natalizumab, and glucocorticoids. Many such patients had an underlying hematologic malignancy or collagen vascular disease. The current patient was on long term corticosteroids. Herpes simplex and enteroviruses can causes acute encephalitis or meningitis but not with this insidious progression and afebrile course. Toxoplasma lesions are enhancing on MRI, not confined to white matter, favor the basal ganglia and are not so indolent as in this case.

A 76-year-old HIV positive man on his first ART regimen, tenofovir/emtricitabine/elvitegravir/cobicistat, for a year. HIV RNA originally 123,000 copies/ml, then decreased to less than 20 copies/ml by 6 months. Most recently, HIV RNA was 23,000 copies/ml, repeated 2 weeks later after adherence counseling at 34,000 copies/ml. What lab tests would you now order? Drug level testing Routine genotype testing Routine genotype testing plus integrase genotype testing Integrase testing alone CCR5 tropism testing

Correct answer: routine genotype testing plus integrase genotype testing. This patient appears to be experiencing virologic failure (2 confirmed HIV RNA levels >500 copies/ml) and needs further evaluation with a genotype of reverse transcriptase/protease (the routine test) and, because he's on elvitegravir (an integrase inhibitor), a genotype of integrase. Most likely, he will have the M184V substitution of reverse transcriptase, associated with emtricitabine drug resistance. The integrase genotype likely also will demonstrate substitutions in integrase, such as N155H, Y143C, T66I or in this case, S147G, which is unique for elvategravir. Importantly, dolutegravir retains activity against these viral strains. If a substitution at Q148 is present, this would decrease the activity of dolutegravir in a subsequent regimen.

A 55-year-old man with AIDS who previously experienced treatment failure due to suboptimal adherence on two prior antiretroviral regimens: first, tenofovir/emtricitabine/efavirenz for 2 years, and then abacavir/lamivudine and boosted atazanavir for 4 years. He subsequently took a "drug holiday" six months prior to following up with his physician. Work-up now reveals CD4 cell count 203; HIV RNA 89,000; genotype and phenotype wild-type virus; and tropism test reveals R5 virus. He is ready to start a new regimen and says he's "found religion" and will be adherent. Which of the following is the optimal management for efficacy and safety and convenience? Restart tenofovir/emtricitabine/efavirenz Restart abacavir/lamivudine + boosted atazanavir Start Etravirine + darunavir (boosted) + dolutegravir Start zidovudine + lamivudine + etravirine Start Etravirine + darunavir (boosted) + enfuvirtide

Correct answer: start etravirine + darunavir (boosted) + raltegravir This treatment-experienced patient likely has significant drug resistance (despite his current negative drug resistance tests which were sent following a drug holiday, i.e. without significant drug pressure allowing the emergence of wild-type virus). The drug history must be considered when interpreting genotype or phenotype data. Drug-resistance testing in the setting of virologic failure should be performed while the person is taking prescribed ARV drugs or, if not possible, within four weeks after discontinuing therapy. On the basis of history alone, one would guess he likely has the M184V mutation (associated with emtricitabine and lamivudine drug resistance), possibly the K103N mutation (associated with efavirenz drug resistance), and possibly some protease inhibitor resistance mutations. Restarting either of his prior regimens makes less sense given the likelihood for drug resistance (particularly the M184V). Thus, any choice with efavirenz, atazanavir, emtricitabine/lamivudine, or tenofovir is not a "best" answer. Treatment guidelines suggest including two - or preferably three - fully active drugs in the next regimen. Of the listed choices, etravirine (an NNRTI not affected by the K103N mutation), darunavir (a PI with activity against many PI-resistant viral strains and dosed twice-daily in the setting of pre-existing PI resistance mutations) and dolutegravir (integrase inhibitor with full activity in this integrase inhibitor-naïve individual) would likely be active agents. For the final choice, enfuvirtide, while active in this fusion inhibitor-naïve individual, requires twice daily subcutaneous injections and is thus not a convenient option. Correct Response Start Etravirine + darunavir (boosted) + dolutegravir

A 26-year-old man with recently diagnosed HIV disease is eager to start antiretroviral therapy. Initial work-up reveals HIV RNA 2.5 million copies/ml, CD4 1011, HLA-B*5701 positive, genotype with L63P in the protease. His only medication is a fluticasone inhaler for asthma. He prefers one pill, once-daily therapy, if possible. Which of the following regimens do you recommend? tenofovir/emtricitabine + darunavir/ritonavir tenofovir/emtricitabine/elvitegravir/cobicistat tenofovir/emtricitabine/rilpivirine tenofovir/emtricitabine + dolutegravir abacavir/lamivudine + efavirenz

Correct answer: tenofovir/emtricabine + dolutegravir. The best choice of the options listed is the 3-drug regimen of tenofovir/emtricitabine + dolutegravir. Choices 1 and 2 both have a pharmacologic booster (ritonavir in 1, cobicistat in 2) which can increase the systemic levels of fluticasone and can lead to Cushing's syndrome. Choice 3, the rilpivirine-based regimen, and choice 5, the abacavir-containing regimen, would not be appropriate with a baseline HIV RNA level of >100,000 copies/ml; also abacavir is contraindicated with HLA-B*5701 positive.

A 13 year old girl developed a pruritic rash on her foot after moving to rural northeast Florida. Which of the following helminths is the most likely cause of the rash (serpeginus rash on the foot)? A. Enterobius vermicularis B. Ascaris lumbricoides C. Trichuris trichiura D. Toxocara canis E. Anyclostoma braziliense

Cutanous larvae migrans Penetrating the skin: Ancylostam brazilienses How do people get infected with nematodes? 1. Eating eggs in fecally contaminated food or soil Ascaris, Trichuris, Enterobius, and Toxocara 2. Direct penetration of larvae through skin Hookworms, Strongyloides 3. Eating food containing infectious larvae Trichinella, Angiostrongylus, Anisakis 4. Vector transmission Wuchereria, Brugia, Oncho, Loa

52 y/o M +digital/oral ulcers every 4 month one ANC of 100 + spleen tip Dx? Rx?

Cyclic or Acute neutropenia Dx-molecular testing Rx- G-CSF

-23 y/o F - +Diarrhea after leaving Nepal - no response to aug or metro - ill for 5 week, +abd pain, 15 lb wt loss - stool - for Shi/Sal/Cam by Cx, and E.histo, Giardia, and Cryptospori by EIA Dx? Rx?

Cyclospora / TMP-SMX, 2nd choice Cipro 3 "chronic" infectious diarrhea 1- cyclospora 2- giardia (should have responded to metro [ though 10% do not - use tinidazole instead], and neg EIA) 3- post-infectious irritable bowl

Persistent diarrhea (≥ 14 days) • Diagnostic challenges Negative test Giardia, EIA/PCR for E. histolytica, acid fast staining not routine, multiplex PCR solves • Cryptosporidum Animals reservoir, water vehicle of transmission • E. histolytica produces liver abscess most importantly in males Serology helpful in hepatic abscess as stools

Cyclospora > cryptosporidum Sporulation required for Cyclospora for infectivity

35F c/o 5 wks progressive cough with paroxysms, pleuritic pains, chest tightness and dyspnea Tried albuterol and amox/clav without benefit PMH: early menopause, recurrent UTIs, insomnia, depression SH: Born in India, in US x 14 yrs Meds: estrogen replacement, vaginal estrogen + nitrofurantoin (started 2 mos prior), venlafaxine PE: afebrile, RR 14, 02 sat 93% RA No lymphadenopathy, no JVD Lungs: fine, basilar crackles bilateral Cor: no murmur Ext: no edema 11.8 160K 63N, 36L, 1E Which treatment decision will mostly likely lead to resolution of patient's symptoms A. Azithromycin x 5 days B. INH, RIF, PZA, ETB C. Ceftaroline x 7 days D. Discontinue nitrofurantoin E. Prednisone 60 mg daily with taper

D May present like atypical pneumonia or interstitial fibrosis • Acute Symptoms start ~8d after starting drug Fever, dyspnea and cough • Subacute (after 1 month) Similar but less prominent fever • Chronic 47% in one large study >12 mos on drug • Hypersensitivity reaction (early) Acute & subacute Ground glass findings May have eosinophilia (peripheral or BAL) • Later presentations Interstitial pneumonitis Bronchiolitis obliterans Mixed ground glass, fibrosis, consolidation

68 y. o. butcher who is an avid hunter presents with dementia progressing over 4 months, myoclonus, MRI below, periodic sharp waves on EEG. Acquisition of this illness was most likely due to: T2 Flair A. Contact with elk brains B. Contact with sheep brains C. Contact with pork brains D. A spontaneous event

D Prion Diseases: Transmissible Spongiform Encephalopathies • Spontaneous (N=~6000 worldwide per year) - Sporadic Creutzfeldt-Jakob disease (sCJD) • Associated with specific ingestion - Beef from cows with Bovine Spongiform Encephalopathy • Denoted "Variant CJD", "vCJD" (N ~ 220 total cases) - Human brains • Kuru (N= ~2700 total cases) • Associated with a medical procedure (N ~ 450 total cases) - Iatrogenic - Denoted "iCJD" • Hereditary (N ~600-900 worldwide per year) - Familial (fCJD) - Gerstmann-Straussler-Sheinker (GSS) - Fatal Familial Insomnia (FFI) - Fatal Sporadic Insomnia (FSI) Rapid progression • Classic Triad - Dementia - Myoclonus - EEG: periodic sharp waves • CSF - 14-3-3 protein (elevated in HSV encephalitis) - Supportive evidence but not pathognomonic

35 yo M Seabee in excellent health (HIV negative) has been building bridges in Uganda. Over three days he develops acute headache, fevers, rigors, and abdominal pain. WBC 28,000 with 63% eosinophilia. AST and ALT are both twice normal Most likely diagnosis? A. Entamoeba histolytica B. Strongyloides stercoralis C. Wuchereria bancrofti D. Schistosoma mansoni E. Paragonimus westermani

D. Schistosoma mansoni - acute - 6-8 week after infection - worms releasing eggs - lateral spine, hepatitis Shistosoma haematobium - terminal spine - cysitis • Fresh water exposure in an endemic region. • Clinical syndrome compatible with acute schistosomiasis (F, abd pain, myalgias, eosinophilia) • Clinical syndrome compatible with chronic schistosomiasis (abdominal pain, blood in stool, loose stools, evidence of portal HTN, hematuria, eosinophilia) entamoeba histolytica -protozea, liver abscess, but no eosinophelia strongyloides stercoralis - isn't a terrible guess - quiet ill -immunocompromised wuchereria bancrofti - lymphatic filariasis - pulmonary symptoms paragonimus westermani - pulmonary disease

A 47‐year‐old male with acute myeloid leukemia and an neutrophil count below 100/mcl for the past three weeks has been febrile for 10 days, first treated with piperacillin‐tazobactam Had been on prophylactic micafungin but a blood culture is growing a yeast on Gram stain. The most likely echinocandin‐resistant yeast is which of the following: A. Candida parapsilosis B. Candida glabrata C. Candida auris D. Trichosporon asahii E. Candida lusitaniae

D. trichosporon asahii echinocandin resistance is cryptococcus trichosporon moulds other than aspergillus

A 25 y/o female suffers a cat bite on the forearm. She presents one hour later for care. If no antibacterial is administered, the percentage of such patients that get infected is: A. 0-10 % B. 10-30 % C. 30-70 % D. 70-100 %

D: 70-100% most common organims is pasteurella multocida -also anerobes -bartonella henselae (cat scratch dis) -rabies virus -s.aureus -streptococcal disease in saliva of >90% of cats and over 80% of wounds get infected pasturella canis in saliva of 50% of dogs and only 10% get infected all amoxicillin sensitive, but keflex resistent

37 M carpenter +painless, firm, nonfluctuant subQ mass in submandibular 4weeks no F/sore throat/NS/Wt loss/cough/ cervical LN CXR normal biopsy: chronic inflammation without granuloma formation, matt like collection of filamentous gram positive rods D/P/T

D: Actinomycosis - not mycobacterial (granulomatous), and systemic S&S and more cervical P: Actinomyces israelii - sulfur granules, are small yellow bodies found in puss termed in 19th century T: 4-6 week of IV PCN or CFX then 6-12 months of PCN V or Amoxicillin - PCN allergy: tetracycline

45 y/o M w/ splenectomy +fever, flu-like, HA x 5 days 2 days ago oval red rash, 10cm on back of thigh, started on doxycycline to ER looking ill, HR 100, BP 98/70, F 101 homogenous rash on right thigh, and left upper quadrant WBC 3.3K, Hgb 4.7, Plt 105, Tbilli 3.8 --> ARDS, AKI, DIC Dx? (lyme, francisella, babesia, anaplasma, rikettsia)

D: Babesiosis (babesia microti), transmitted via ixodes scapularis (black-legged/deer tick) 1) Anemia - hemolytic, with elevated bilirubin and LDH - not caused by anaplasmosis nor lyme disease 2)ARDS 3) AKI 4) DIC 5) Thrombocytopenia splenectomized at risk; encapsulated bacteria S.pneumoniae H.influenza N. meningitidi and other pathogens: Capnocytophaga canimorsus and intraerythrocytis parasite lyme/babesia co-infeciton is common RMSF (rickettsia rickettsi) can cause sever infection, but not this degree of anemia; trasmitted via Demacentor (dog) tick ;wil have diffuse maculopapular / petechial rash, not erythema migrans ixodes scapularis in NE can trasnmit / incubation period: Borrelia miyamotoi (relapsing fever), Powassan / deer tick virus (agent of encephalitis) Anaplasma phagocytophilum (Human granulocytis anaplasmosis HGA), 5 d - 2 week Babesia microti (1-6 weeks) Lyme disease, 1-2 weeks (range 3-30 dyas) *human monocytic ehrlichiosis (HME) due to Ehrlichia chaffeensis, transmitted by different vector, Ambylomma americium (lone start tick)

62 M w/ RA on anti-TNF, allergic to PCN +HA (acute), 101.2F, stiff neck -prior to starting anti-TNF, negative PPD and CXR -WBC 12K - CSF 642 cells (62% neut), RBC 44, Glu 40, Prot 195, GS small GPR

D: Bacterial Meningitis - due to anti-TNF should consider pneumocytosis, histo, and listeriosis P: GPR: listeriosis T: PCN allergy: TMP-SMX

75 M +confused, T38.5C LP shows 1100WBC (90% neut) GS of CSF: GPR

D: Bacterial meningitis - >50 y/o - immuno suppressed P: Listeria monocytogenes - short, coccoid, not stain drakly on GS - more likley than bacillus, corynebacterium, or erysipilothrix T: Ampicillin , some experts will add gentamicin - PCN allg: TMP-SMX

35 M IVDU injected black tar in february rapidly over hours: vision changes , then wekaness in arms and legs, difficulty breathing - no GI symp, no fever, normal HR & BP, tender at puncture site , pupils dilated, CN III, IV,VI impaired both sides

D: Botulism - CN abnormalities and progressive descending paralysis - incubation of wound botulism > food borne, ~ 10das P: Clostridium botulinum T: heptavalant botulinum antitoxin (HBAT)

61 M , hunted swine, dressin animals, cut his hand; also ate some of the boiled meat +F x10 days (high 103F) + HA, myalgia, anorexia

D: Brucellosis - prolonged incubation period - common in swine slaughterhouses / also bison in western US P: Burcella Suis - Trichinellosis possible but no eosinophillia or muscle tenderness - Tulermia possible but presentation too prolonged, and associated with rabbits &rodents - Incubation too long for leptospirosis or Qfever (2-4 weeks, and normal x-ray) - feral swine could be a source of trichinellosis by ingesting contaminated meet, but boiling kills it , and after 10 days should have had eiosinophillia T: 1st line Doxy 45 dayy + (Streptomycin first 2-3 weeks or gentamicin first 7-10d) for osteoarticular dx 2nd line doxy + rifampin x6 weeks (or cipr/doxy or doxy/rifampin) 3rd line TMP-SMX

49 y/o M +enlarged lymph node in right axilla - firm, moveable, and slightly tender + linear scratches from a new kitten (3 weeks new) s/p node resetion + granulomas w/ neutrophlic absesses and necrosis 2 month ago, visit africa, ate local food, cut right hand on plant leaf, trailed right hand in river; he's an avid gardner roses and orchids D/P/T

D: Cat Scratch Disease (regional adenopathy with necrotizing granulomas with neutrophilic abscesses) DDx: rose gardner - sporotrichosis; nodules along lymphatic, but nodes not enlarged, usually ulcer ulcer with nodules along lymphatics also with M. marinum and N. brasiliensis, but lymph nodes not enlarnged P: Bartonella hensalae T: Mostly self-limiting but antibiotics recommended 2014 IDSA Azithromycin 5 days other options Doxy +/- rifampin or gentamicin

52 M homeless +F/C/loss of appetite/ left arm&leg weakness +38.3, disheveled, conjuctival petechiae, hemorrhagic lesion in right retina, 2/6 sys murmur +lice in clothes +oscilating mass on TTE - BCxs negative

D: Culture negative infective endocarditis -trench fever transmitted by lice P: Bartonella quintana - fastidious, slow rowing GNR -lysis-centrifugation culture on chocolate agar helpful - coxiella burnetti also cause of CNIE, but no aerosol inhalation of goat/sheep/cattle in story T:

28 y/o soldier returned from Afghanistan and Iraq 2 months ago; many insect bites ; fails to repsond to 6 weeks of oral antibiotics large deep ulcer; indurated boarder no F/c/ns Disease, pathogen, treatment?

D: Cutanous leishmaniasis / transmitted via Sand fly 90% in middle east / south america) / biopsy to diagnos shows macrophages with intracellular organisms P: L. Major and L. Tropica L. mexicana T: most ulcers heal without therapy paromomycin / fluconazole / antimonials

27 y/o M w/ splenectomy, + usual vaccination +dog bite, laceration on leg 5 days ago +delerium, F (103.4) + WBC 24.7 +rod-shaped bacteria on blood smear D/P/T

D: Dog bite septicemia (usually alcoholic or asplenic) P: Capnocytophaga canimorsus - Pasteurella less virulant

52 homeless, depressed, suicide jump off a bridge into canal in ohio, suffered a compound fracture 2 weeks later, +Fever, but ex-fix of femur in good appearance WBC 10K stable then + HA, +bilirubin rise to 4 and Cr up to 4.4 BCx -ve, Renal biopsy = inflammation and spirochete fragments

D: Leptospirosis - acquired intact skin exposued to urine of infected animals - also inhalation and aerosol of infected water (white water rafting) - more elevation of billirubin than AST/ALT other spirochetes: - rat bite fever: spirillum minus; Syphillis - trepenoma; and lyme/relapsing disease - borrelia spp P:Leptospira interrogans T: PCN 1MUq4h / CFX 1gq24h 5-7 d Doxy/ minocycline

31 y/o F +fever / neurological symptoms - first n/v, then myalgia/HA; now dipolopia - exam: +febrila 102.4, + nuchal rigidity, 6th cranial nerve palsy - MRI: inflammation of pons and medulla D/P/Tx

D: Rhombencephalitis - Biphasic, F/HA/N/V for 3-4 days, then sudden onset of progressive asymmetrical CN deficits, cerbeallar signs, and hemiparesis w or w/o meningeal sign - Too rapid for TB or crypto; strep pnuemo never parenchymal dx P: Listeria monocytogenes Tx: Amp 2g IV q4h, or PCN 24MU/d; or TMP-SMX 15-20 mg /kg in 3-4 doses

44 M chronic hepatitis C and cirrhosis +Fever, abd pain, distension x 2 days + ascites, enlarged liver, WBC 20K (90%neut), Cr 2.0 + paracentesis WBC2400(74%neut), GNR on GS plus antibiotics what else impoves survival?

D: SBP 1)ascetic fluid infection 2)no intra-abdominally surgically treatable source P: GI flora T: CFX or Pip-tazo

21 M +F, red tender patch on his anterior lower leg - no nodularity, fluctance, drainge or evidence of lymphangitis which oral therapy?

D: community acquired SSTI - must consider MRSA since it can mimic strep - also must consider GAS, which are resistant to TMP-SMX and doxy P: GAS and CA-MRSA T: TMP-SMX and amoxillin

30 y/o M + single painless well-demarcated genital ulcer with heaped up borders x 1 week hx of anaphylaxis to PCN T?

D: early syphilis / syphilitic chancre (too acute to b granuloma inguinale) P: Treponema pallidium T: favored alternate to treat in non-pregnant person (data on HIV-infected is limited) is doxycyline x2 week

Alot of people who shared dinner casserole, macroni, vegtebale, beef, turkey and gravey +N/Abd pain/Diarrhea onset 11 hours better within 24 hours D/P/T

D: enterotoxin induced illness, "food poisoning" - > 10^6 CFU/g of vegetative cells ingested - food cooled too slowly or reheated insufficiently - enterotoxin in small intestine 6-24 hours P: Clostridium perfiningens - could be Bacillus cereus (N/V/D in 24H, very quick) - not S. aureus (usually within ~6hrs symptoms, no diarrhea with s. aureus) - Ecoli, listeria, norovirus long incubation time, >24H, listeria will have fever, norovirus will have secondary cases T: supportive care

18 y/o F, healthy, returned form northern arkansas camping - removed 2 ticks from legs, 9 days ago - red ovoid rash about 7 cm / with central clearing - clinical manifestation most likely to occur? (arthritis / heart block / shock / hepatitis / or nothing)

D: erythema migrans of southern tick associated rash illness (STARI) - mild constitutional/systemic complains (HA's, Aches, Fatigue, Nausea) - from bite of lone star tick (ambylomma americanum) - does not have any seqelae of B. Burgdorferi - no serological test

six friends, ages 19-21 +rash for 1 day - pruritic & papulo-pustular on trunk and buttocks and lower extremity - 2 with tender red purple nodular lesion on soles of their feet - ate burger / beer/ hotdog/rasperries/strawberries/ pineapples - spent 2 hours in hot tub with abrasive bottom, wore swimsuit D/P/T?

D: folliculitis (hot tub buns) - often in areas covered by bathing suit -incubation 1-2days -self-limited, resolve within a week w/o abx - could be MRSA or candida but not after exposure to water source P: psuedomonas aeruginosa - cyclospora, a protozon acquire dby eaitng contaminated rasberries, results in GI illness not rash T: supportive

40 M Syncope, Bradycardia Tick bites Weeks of fatigue myalgia started on doxy plan for PPM placement D/P/T

D: lyme carditis -myocardial inflammation on gallium scan or heart biopsy - heart block persists for >week P: Borrelia burgdorferi T: no hard evidence for CFX, but most recommend over doxy - no need for PPM, maybe a temporary pacemaker

81 M +F, RUQ pain + CT: large hepatic abscess + stones in gallbader +s/p surgery, intra-op puss noted + anerobic culture +GNR - aeorbic NGTD which abx? Levo/clinda/cipro/amp/erta

D: pyogenic abscess P: anaerobic GNR - Bacteroides possibly T: ertapenem - bactroides resistance to clindamycin has increased now at 25%

38 M w/ chronic diarreha, generalize hyperpigmentaiton of his skin for 1 year, 3 months of arthralgia in knees hands, ankles shoulder wrist + anemia, elevated ESR, diffuse hypergammaglobulinemi ? whipple what test most sensitive and specific for whipple's disease?

D: whipple's disease - diagnose with ampilification of gene target in tissue - not observed on GS, does not grow in cultures - PAS staining has low sensitivity - Electron microscopy not specific P: Tropheryma whippeli T: PCN (endocar) CFX (CNS) 2-4 weeks if PCN allg: Meropenem then TMP-SMX for 1 year, if TMP-SMX allergy then doxycycline

42 yr WF with Crohn's disease taking adalimumab is admitted to a Chicago hospital because of 6 weeks of low grade fever, pancytopenia and a 10 pound weight loss. Hydrocortisone 200 mg daily was begun for low serum cortisol not responding to Cortrosyn stimulation. Micafungin was given for yeasts seen in peripheral blood smear that were not growing on routine culture. The most helpful diagnostic test would be which of the following: A. Fungal blood culture B. CT of abdomen C. PPD D. Bone marrow aspirate E. Urine for Histoplasma antigen

E. Urine for Histoplasma antigen Disseminated histoplasmosis Best test for rapid diagnosis: urine histoplasma antigen bone marrow aspirate... can't see histoplasmosis Limitations: false positives in blastomycosis and coccidioidomycosis

You have been monitoring a 36 year old HIV+ man with CD4 ~350, VL 236,000 who is now ready to start ART. Which regimen is best? A. zidovudine/lamivudine + darunavir (boosted) B. tenofovir/emtricitabine/rilpivirine C. abacavir + emtricitabine + efavirenz D. abacavir/lamivudine + atazanavir (boosted) E. tenofovir/emtricitabine + dolutegravir

E.tenofovir/emtricitabine + dolutegravir A- zidovudine - toxic drug we do not use anymore B - attractive choice, rilpivrine at the exterem of HIV, CD4 < 200 or VL >100,000 C/D - abacavir with any drug except dolutegravir a suboptimal regimen

Question #4 A 35 y/o male suffers a clenched fist injury in a barroom brawl. He presents 18 hrs later with fever and a tender, red, warm fist wound. A gram stain of the bloody exudate shows a small gram-neg rod with some coccobacillary forms. The aerobic culture is positive for viridans streptococci. Which one of the following organisms is the likely etiology? A. Viridans streptococci? B. Eikenella corrodens? C. Peptostreptococcus? D. Fusobacterium species?

Eikenella corrodens Anaerobic small gram-negative bacillus Susceptible to: penicillins, FQs, TMP/SMX, Doxy, and ESCs. Resistant to: Cephalexin, clinda, erythro, and metronidazole

A patient has catheter-related Staphylococcus aureus bacteremia. An E test is done to determine vancomycin susceptibility. What is the mechanism of the resistance demonstrated by this E test? Beta lactamaseEfflux pumpD- lactate replacement of D-alanine in S. aureus cell wallExcess D- alanine binding cell wall binding sites: i.e., thick cell wall

Feedback Correct Answer: Excess D- alanine binding cell wall binding sites: i.e. thick cell wall This E test shows some colonies of S. aureus growing within the area where most S. aureus have been inhibited by vancomycin. Were it not for these few colonies, this S. aureus would have an MIC of about 2 mg/ml. Strains of S. aureus with this mechanism of resistance are referred to as vancomycin (glycopeptide) intermediate S. aureus.

A 72-year-old woman with COPD is admitted with shortness of breath, tachypnea and low-grade fevers. CXR shows a diffuse infiltrate. Blood cultures, urinary antigen studies for pneumococcus and Legionella are normal and the BNP is not elevated. A senior resident orders multiplex respiratory viral testing over the objection of the attending. It comes back positive for RSV Which of the following is appropriate: Begin inhaled ribavirin Place your patient in contact isolation Begin therapy with RSV immune globulin (Respigam) Give your patient palivizumab (Synagis) Ignore the RSV since the patient has pneumonia and treat with piperacillin-tazobactam and vancomycin

Feedback Correct Answer: Place your patient in contact isolation Your patient likely has RSV viral pneumonia. She is of the right age group and underlying lung disease may be a risk factor. There is no specific treatment, but she is infectious and nosocomial transmission is a significant concern. Per the CDC Isolation Guidelines: "...RSV transmission is effectively prevented by adherence to Standard plus Contact Precautions.... It is important to remind healthcare personnel that even if Droplet Precautions are not recommended for a specific respiratory tract pathogen, protection for the eyes, nose and mouth by using a mask and goggles, or face shield alone, is necessary when it is likely that there will be a splash or spray of any respiratory secretions or other body fluids as defined in Standard Precautions....During the seasons when RSV, human metapneumovirus, parainfluenza, influenza, other respiratory viruses, and rotavirus are circulating in the community, cohorting based on the presenting clinical syndrome is often a priority in facilities that care for infants and young children.... However, when available, single patient rooms are always preferred since a common clinical presentation (e.g., bronchiolitis), can be caused by more than one infectious agent." The first choice is incorrect. There are no data supporting a role for inhaled ribavirin except limited data among transplant patients The third choice is incorrect. RSV immune globulin is not effective in the treatment of RSV disease. In patients with hypogammaglobulinemia, replacement therapy with IVIG is usually recommended. The fourth choice is incorrect. Palivizumab is used for prophylaxis in a very limited subset of infants at high risk of severe RSV disease but even there the efficacy is modest and cost effectiveness is poor. It has not been shown to have any effect in the treatment of RSV disease. The last choice is incorrect. Viral bacterial co-infection is certainly possible, but this patient does not have severe pneumonia or risk factors for Pseudomonas so standard therapy for CAP would be appropriate. In this setting, procalcitonin would be very helpful. A very low procalcitonin with clear evidence of RSV infection and a compatible presentation would make bacterial co-infection very unlikely and antibiotics could be stopped.

Drug A was tested in a randomized non-inferiority trial against drug B with a power of 0.8, alpha of 0.05 and delta of 0.1. Which of the following results would justify a claim that drug A was superior to drug B? Results are shown as the proportion of successes with regimen A minus regimen B with the 95% confidence interval around the difference. 0.20 +/- 0.25 -0.15 +/- 0.10 -0.05 +/- 0.20 0.10 +/- 0.10 0.20 +/- 0.10

Feedback Correct answer: 0.20 +/- 0.10 The lower boundary of the 95% confidence interval has to be at or above the delta to claim superiority. Only in one answer is the difference minus the 95% confidence interval a positive number of at least the delta of 0.1. (0.20 - 0.1 = 0.1). The investigators chose a delta of 0.1 in their study design because they considered that a difference of less than 0.1 (10%) would be clinically inconsequential. With the bottom result in the list, drug A was 20% better with a 95% probability that the difference was at least 10%.

The effect of iron overload on the prevalence of infection with a specific bacterium was estimated by comparing the prevalence of infection in patients who did not have iron overload, using a national database of hospital discharge diagnoses. The prevalence of the infection in patients with iron overload was 2 per 1000 persons. The prevalence of the infection in those without iron overload was 0.5 per 1000 persons. The hazard ratio of iron overload for this infection was which of the following: 4 2.5 2 0.4 0.2

Feedback Correct answer: 4.0 The hazard ratio is the 1/prevalence without iron overload divided by 1/prevalence with iron overload, or (1/0.005)/(1/0.002) or 2000/500=4.

65 yr wm admitted with cryptococcal meningitis, seizures, diabetes mellitus and granulomatosis with polyangiitis. Given conventional amphotericin B, flucytosine, phenytoin, glipizide, prednisone and cyclophosphamide. By the third week of treatment, his creatinine had risen to 3 mg/dl. By the fourth week his WBC had fallen to 1.2K, platelets 60K and diarrhea began. The cause of his WBC falling to 1.2K, platelets 60K and copious diarrhea is most likely which of these drugs? A. flucytosine B. phenytoin C. glipizide D. cyclophosphamide E. cytomegalovirus

Flucytosine accumulates during azotemia and causes marrow suppression and colitis. Glipizide and phenytoin don't accumulate in azotemia or cause this toxicity. Cyclosphosphamide at therapeutic doses may cause marrow suppression but not colitis. CMV is an unlikely pathogen in this patient

Entamoeba histolytica • strictly human pathogen - food/water contaminated with human feces wide range of clinical presentations • asymptomatic • traveler's diarrhea (a common cause) • colitis (can be lethal) sharp abdominal pain bloody diarrhea fever flask-shaped ulcerations onset can occurs weeks to months after travel Diagnosis E. histolytica trophozoites with ingested RBCs. Serology • very helpful in amebic liver abscess (95% sensitive) • helpful (about 85% sensitive) in intestinal amebiasis Treatment tinidazole or metronidazole followed by intraluminal agent (paromomycin)

Giardia intestinalis (prior G. lamblia, G. duodenalis) Flagellated protozoan • fecal/oral via ingestion of cyst form in food/water • cyst is chlorine resistant • cysts from humans (beavers, muskrats) Disease in U.S. • most common parasitic infection in the U.S (20k cases reported/year, likely 2M) a leading cause of traveler's diarrhea Symptoms • intermittent watery diarrhea weeks to months • foul smelling stools, flatulence, "sulfur burps" Giardia At risk populations • international travelers • swimming in lakes/streams, outdoor survival/camping • infants in daycare • child care workers • immunoglobulin deficiencies (esp CVID) • HIV when CD4 < 100 Diagnosis • stool antigen test • recently approved stool multiplex PCR Treatment tinidazole, metronidazole (off-label), nitazoxanide, and albendazole (off label)

A 41 homeless male with HIV presents for initial evaluation. His CD4 count is 141 (9%), HIV VL 11,580, crcl 90. He is currently taking no medications reliably. He reports a recent history of injecting drug use with heroin, which is being treated with methadone maintenance for which he is on a stable dose under directly observed therapy. The patient reports previous attempts at treatment with antiretrovirals but was poorly adherent due to "life circumstances". Since he failed a raltegravir based regimen in the past, and has raltegravir mutations (Q148R and N155H) another physician elected to initiated efavirenz-TAF-emtricitabine. He is monitored daily for ART adherence at his shelter, and he appears to be taking his drugs. Ten days after initiating treatment the patient returns to your office. He reports several days nausea, vomiting, abdominal cramping, diarrhea, rhinorrhea, and irritability but no fever. Laboratory results are pending What is the most likely cause of this new syndrome A. Hepatitis A B. Hepatitis E C. Campylobacter enteritis D. Methadone withdrawal E. Immune reconstitution syndrome (IRIS)

Given reports of abdominal cramping, diarrhea, rhinorrhea, and irritability, the patient seems to be exhibiting signs of opioid withdrawal. This is likely due to increased clearance of methadone, which can result from interaction with efavirenz. Efavirenz (EFV),and lopinavir/ritonavir (LPV/r) have been associated with significant decreases in methadone levels (Efavirenz can reduce methadone levels by 51%) and other protease inhibitor combinations have also been implicated such as darunavir- ritonavir. Patients and substance use disorder treatment facilities should be informed of the likelihood of this interaction. The clinical effect is usually seen after 7 days of coadministration and may be managed by increasing the methadone dosage, usually in 5-mg to 10-mg increments daily until the desired effect is achieved. The integrase inhibitors, TAF and FTC do not interact with methadone. However, due to his prior exposure to raltegravir, integrase inhibitors were not an option for this patient. The clinical picture is not consistent with IRIS. Furthermore, IRIS with a CD4 count >100 is not likely. Hepatitis A occurs in persons with poor hygiene. There have been outbreaks among homeless persons, including those with HIV infection, but without laboratory values its difficult to attribute this to any form of viral hepatitis

25 y/o black women w/ fatigue IVDU no edema CR 8.4, anemia, mild aceidemia in ER 10 weeks normal renal funciton UA high grade proteinuria US of kidnesys, normal to increase, size; no obstruciton rapid HIV test + Dx?

HIVAN

38 year old woman from Jamaica presents with weakness, unsteadiness of several months duration and has recently developed incontinence. Neurologic exam notes hyperreflexia ankle clonus, and positive Babinski reflex WBC = 7500 cells/ul CD4 T cell = 1000 cells/ul CSF cell count: 10 cells/mm3 (lymphocytes ) CSF protein: 75 mg/dl T2 weighted MRI image is shown The etiologic agent associated with this illness is also associated with A. Acute T cell leukemia B. Multiple sclerosis C. Variant Creutzfeldt‐Jacob D. Hemorrhagic cystitis

HTLV-I Tropical Spastic Paraparesis /HTLV-1 Associated Myelopathy • Epidemiology • <1% of HTLV‐I develop HAM/TSP • The second most common neurologic syndrome in Jamaica after stroke • Latency may be short‐‐several years • Female predominance • Presentation • Spastic paraparesis • Lower>upper • Proximal>distal • Bladder disturbance • Hyperreflexia • Positive Babinski reflex • Differential Diagnosis • Cord compression • B12 deficiency • Syphilis • HIV‐1 myelopathy • M

37 year old Jamaican female with diffuse pruritic rash (right), bone pain with lytic bone lesions. WBC: 50,000, 90% lymphocytes Which is most likely cause of her presentation? A. HTLV‐I B. HTLV‐II C. HIV‐1 D. HTLV‐IV

HTLV‐Iacute T cell Leukemia (ATL) Long Latency (>30 years) • Small pediatric series in SA • Epidemiology • Approximately 1% of HTLV‐ I infected adults • M>F (Japan); M=F (Jamaica) • Associated syndromes • Infectious • TB, MAC, Leprosy • PCP • Recurrent Strongyloides • Scabies esp. Norwegian scabies • Noninfectious‐hypercalcemia+lytic bone lesions • Therapy • Cytotoxic chemotherapy • AZT+Ifn • Transplant • Mogamulizumab (Poteligeo, anti CCR4 monoclonal) APPROVED in Japan for ATL • Lenalidamide

30 y/o nurse, for seasonal influenza live vairus vacicnt rather than killed vaccine, "hates needles" works for cancer hospital on the SCT unit has chronic exacerbation of acute bronchitis advise? He should get the live vaccine and continue working. He should get the live vaccine but not work in the first week after vaccination. He should not get the live vaccine because he has chronic bronchitis. He should not get the live vaccine because it is no longer recommended for adults but he should definitely get the inactivated influenza vaccine.

He should not get the live vaccine because it is no longer recommended for adults but he should definitely get the inactivated influenza vaccine. Beginning 2016-17 flu season, the CDC no longer recommends live virus vaccine, poor efficacy in the prior two flu seasons. In addition, there is a tiny risk of transmission the infection to immunosuppressed patients in the week after vaccination. Chronic bronchitis in an adult is not a contraindication to the live virus vaccine and rather is a good reason to be vaccinated against influenza. Thus, this person should get the inactivated vaccine in order to reduce the likelihood he will suffer from influenza morbidity and to reduce the likelihood of his transmitting influenza to his contacts including his patients.

22 y/o F, vacation to mexicon 1 week ago +fever x2 days, +jaundice and fatigued Dx with hepatitis A lives with roommate conuslted by roomate about vaccination (has had MMR, DPT and meningococcal), unclear about the rest best protect the roomamte? Hepatitis A hyper-immune globulin Immune serum globulin Ribavirin Hepatitis A Vaccine Nothing is required.

Heaptitis A vaccine The patient has acute hepatitis A and could transmit it to her roommate. Many adults in this country have been vaccinated for hepatitis A, but the roommate is uncertain, and it's probably not wise to wait for an IgG result to come back from the lab. Within two weeks of exposure, hepatitis A vaccine confers equal protection against hepatitis A as does immune serum globulin (ISG) to patients ages 1 to 40. Vaccination would be preferred, because the protection from vaccination would be lasting, unlike ISG. There is no hyperimmune globulin for hepatitis A. Ribavirin is not useful for HAV prophylaxis. Immune serum globulin would be preferred for contacts who are not likely to respond to immunization (e.g., HIV with low CD4 counts or other severe immunosuppression), or patients with severe liver disease or older individuals. For exposures more than two weeks prior, prophylaxis is not likely to be effective, though vaccination could be considered to protect against future exposures, in case infection did not result from the current exposure.

50 yo F with a history of psoriatic arthritis 2 wks of cough, dyspnea, fever, and malaise been treated with prednisone and methotrexate for several years, and had started inflixamab about 10 months prior to this illness. On exam, she appeared short of breath. RR 32. 02 sat 96% on 100% FiO2 supplemental 02. Lungs: coarse rales bilaterally, decreased breath sounds at bases. CXR: bilateral pulmonary infiltrates. She underwent lung biopsy

Histoplasmosis after TNF- antagonists Blood, BAL cultures: Histoplasma capsulatum Histoplasma: most prevalent endemic mycosis in US Disseminated histoplasmosis reported in patients treated with TNF blockers Infections among patients treated with TNF inhibitors Narrow-based budding of Histoplasma (2-5 microns) Mycobacterial: TB, non-tuberculous mycobacteria Bacterial: listeriosis, nocardiosis Viral: HBV reactivation; possibly zoster Fungal infections: PCP, histoplasma, coccidioides, cryptococcus, aspergillus, candida Broad based budding of Blastomyces 10-15 microns Spherule of Coccidioides (25-100 microns) "Pilot wheel" of Paracoccidioides (10-20 microns

33 y/o M, 3 weeks ago business trip to bangkok, thialand now 2 days of +fever, severe myalgia, +HA and retro-orbital pain WBC 2.7, Hgb 13.2, Plt 80, PT/PTT wnl CXR-normal what rules out dengue? Location of his travel Lack of exposure to nocturnal mosquitos Incubation period Lack of skin rash Normal PT and PTT

Incubation period incubation Dengue 4 - 7 days although incubation periods of 3-14 days have been reported. Incubation periods of >14 days rule out Dengue. Dengue is common in Thailand and Americas, Africa, and Asia. Transmission in the United States is rare and occurs along the border of Mexico, Puerto Rico, and Key West, Florida. transmitted by two types of Aedes mosquitos: these are daytime biters that adapt easily to urban environments, so this patient had ample exposure. These mosquitos also bite at night when lights are on. Skin rash occurs in 50% of patients, often occurring 1-2 days after the onset of fever. Thus, the absence of rash is not helpful diagnostically. With classic dengue, thrombocytopenia and leucopenia are common, but coagulation abnormalities are not. Dengue hemorrhagic fever is a more severe form of disease caused by this virus: gross bleeds from the GI tract, vagina, or other mucosal surface can occur, but clotting factor abnormalities are not as typical as thrombocytopenia and capillary leak (effusions in abdomen, chest etc.). The incubation period here is key: for diseases in returning travelers that present with fever within 3 weeks of exposure in the tropics, think of: Most important diseases occurring within 21 days of exposure: Malaria (12-35 days after mosquito bite) Dengue (3-14 days) Typhoid (5-21 days) Also for short incubation consider: Salmonella Leptospirosis Plague Typhus Japanese encephalitis Yellow Fever Viral hemorrhagic fevers (Ebola 2-21 days) Zika virus (2-14 days) Chikungunya (2-4 days)

Naegleria fowleri • warm freshwater exposure • enters through olfactory neuroepithelium • fulminant meningoencephalitis • immunocompetent children/young adults Acanthamoeba • found in soil and water • enter through lower respiratory tract or broken skin • subacute granulomatous encephalitis • immunocompromised hosts • chronic granulomatous keratitis (contact lens, LASIK) Balamuthia mandrillaris • likely enters through lower respiratory tract or broken skin • transmission by solid organ transplantion has been reported • subacute granulomatous encephalitis • normal and immunocompromised hosts

Intestinal Apicomplexa: clinical clues Cryptosporidium • watery diarrhea of several weeks • cattle workers and daycare outbreaks • cysts are resistant to chlorine (water supply outbreaks) --> #1 cause of water park/swimming pool outbreaks - 4UM Cyclospora cayetanensis - self-limited immunocompetent BUT can last up to 10 weeks! • food associated outbreaks: raspberries, lettuce, herbs • esp. Nepal, Peru, Guatemala -10um Cytoisospora belli • no animal reservoirs known • watery diarrhea • may be associated with a peripheral eosinophlia! (the ONLY intestinal protozoa that does this) -20um

► 28 yr old male presents with temp 39oC, diffuse myalgia, headache, malaise. Returned 2 days ago from "Iron Man" race with running, biking, swimming in lake, climbing in Hawaii. Numerous mosquito bites. Exam: Conjunctival suffusion but no other localizing findings. ► WBC 14,500 with 80%PMN, no eos or bands. Platelets 210k. ► Bili 2.4, ALT 45, AST 52, Alk Phos 120, Cr 1.6. Hct 45%. BC neg. UA: normal Most likely diagnosis: A. malaria B. dengue C. ehrlichiosis D. leptospirosis E. Zika

LEPTOSPIROSIS ► Spirochetes excreted in urine of infected host & able to survive in wet environment ► Exposed intact skin to animal urine in water: veterinarians, farmers, loggers, triathletes, white water rafting, trapping ► Urine from cows, pigs, dogs, raccoons, rats, mice. Summer & early Fall. ► Fever, myalgia, headache (aseptic meningitis late in course) ► Conjunctival suffusion, +/- rash ► In severe cases: jaundice, azotemia, pulm. hemorrhage ► Lab: serology by agglutination test, culture urine in Fletcher's medium Molecular/PCR/NGS emerging ► Rx: doxycycline for outpatients, IV penicillin for inpatients Jarisch-Herxheimer in first 2 hr.

A 35 yo man of Ethiopian descent cut his left thumb with a knife while slaughtering a lamb as part of Easter festivities. He washed the wound with water and applied lemon juice and alcohol. One week later, he developed swelling and tenderness and a fluctuant lesion at the site. Two weeks after the injury, he underwent incision and drainage; cultures grew S. aureus (oxacillin sensitive). Treated with cephalexin but did not improve. A:Botryomycosis due to S. aureus B. Nocardia C. Brucella D. Orf E. Salmonella

Lesion removed surgically. Pathology: hyperkeratosis, epidermal necrosis, dermal infiltrate of mixed inflammatory cells; surface keritonocytes with eosinophilic inclusions PCR testing at CDC + for orf virus DNA. Appearance consistent with ecythma contagiousum Orf (contagious ecthyma) Zoonotic infection caused by a dermatropic parapox virus (ds DNA) of goats and sheep Transmitted by contact with infected animal or fomites Animal handlers; children after visiting petting zoos, livestock fairs Clusters reported after Eid, other festivities involving lamb sacrifice (Passover, Easter) 3-7 d incubation period. Progression: macule or papule nodule with red center, white halo and peripheral erythema ulcerative lesion regenerative papilloma. Most resolve in 4-8 wk Human-to-human transmission not reported Protective immunity incomplete; persons can be infected multiple times

Which one of the following susceptibility interpretations is applicable to methicillin-resistant Staphylococcus aureus (MRSA)? ceftaroline resistant?

MRSA are resistant to all available beta-lactam agents (i.e., penicillins, cephalosporins, carbapenems) with the exception of ceftaroline. Resistance is mediated by a change in the penicillin-binding proteins such that beta-lactamase inhibitors do not overcome this type of resistance. mecA is the gene most commonly associated with this resistance; however, new resistance genes, mecB and mecC, have been recently reported.

W. bancrofti and B. malayi • Asymptomatic microfilaremia • Lymphangitis • retrograde (filarial lymphangitis) • bacterial skin/soft tissue infections (dermatolymphangioadenitis) • Lymphatic obstruction • Lymphedema, elephantiasis, hydrocele, chyluria Tropical pulmonary eosinophilia • Paroxysmal nocturnal asthma • Pulmonary infiltrates • Peripheral blood eosinophilia (>3,000/mm3) • Elevated serum IgE • Rapid response to anti-filarial therapy Likely due to excessive immune response to microfilariae in lung

Manifestations of Onchocerciasis Skin: nodules, pruritus, rash, depigmentation, lichenification Manifestations of Onchocerciasis • Eye: punctate keratitis, sclerosing keratitis, chorioretinitis diagnosis, serology and skin snips

37 y/o F , DM, hx of genital herpes and live kidney tx 12 months ago; cadav pancrease 6 m/a; off TMP-SMX 7 m/a due to rash fever + 2 week productive cough, - F/NS/dyspnea VSS on RA, WBC6.5, Cr 1.3, CXR wnl *CT = nodularity *Gram stain sputum = gram positive rods, filamentaous rod, beaded, weakly +AFB Pathogen? Tx?

N.asteroides 90% (lung +/- CNS), N. brasiliensis = mycetoma treatment: Severe: Imipenem + Amikacin Mild to moderate: TMP-SMX 15mg/kg

31 y/o M with tetralogy of fallot as a chil, to have colonscopy + PCN allergy PPX?

No PPX PPX for high risk : 1)prosthetic heart valve 2) previous endocarditis 3) congenital heart disease (CHD) with unrepaired cyanotic defect 4) repairs cyanotic CHD in first 6 months 5) repaired CHD with residual defect near patch or prosthesis 6) heart transplant recipients who develop valvulopathy undergoing: 1) dental procedure (manipulation of ginivial tissue or the periapical region) 2) procedures on respiratory tract involving incision of the mucosa or procedures in infected skin, or MSK tissue 3) no GI or GU (consider PPX in enterococcus infection with above high risk) procedures

A 50 y/o female alcoholic suffered a provoked dog bite. It was cleansed, tetanus toxoid given, and the dog placed under observation. The patient is post-elective splenectomy for ITP. She received pneumococcal vaccine one year ago. One day later, the patient is admitted to the ICU in septic shock with severe DIC and peripheral symmetric gangrene of the tips of her fingers/toes. Which one of the following is the most likely etiologic bacteria? A. Pasteurella canis B. Capnocytophaga canimorsus C. Fusobacterium sp. D. Bartonella henselae

Only 2-10 % get infected Potential pathogens from Dog's mouth: Pasteurella canis, Capnocytophaga canimorsus Human skin: S. aureus, S. pyogenes Capnocytophaga is an important cause of overwhelming sepsis in splenectomized patients Capnocytophaga suscept. to AM/CL, PIP/Tazo, Penicillin G, and clindamycin; resistant to TMP/SMX, keflex and maybe vanco

A 66 yr old male with neutropenia following chemotherapy for lung cancer, serum creatinine 5 mg/dl, and congestive heart failure is found to have a Scedosporium apiospermum lung abscess. Which of the following would be preferred? A. Anidulafungin B. Itraconazole C. Micafungin D. Oral voriconazole E. Liposomal amphotericin B

Oral voriconazole is preferred. No dose adjustment in azotemia. Itra is relatively contraindicated in patients with heart failure because of negative inotropic effects. Ampho has little efficacy in Sc. apiospermum Echinocandins are not active in scedosporiosis

An 18 year old man presents with a history of malaise, low-grade fevers, and new-onset painful genital lesions seen in the picture below. He had unprotected sexual intercourse with a female partner 2 weeks earlier. Neither he nor his partner has traveled outside the United States. Which of the following diagnostic tests is most likely to yield the specific diagnosis? A. Serum RPR B. Serum FTA-Abs C. Darkfield microscopy D. Glycoprotein-G 1 serum antibodies E. PCR on lesion swab

PCR on lesion swab • Historically, culture of HSV in cell culture was the gold standard. Using daily cultures to detect viral shedding resulted in 4-7% of all days being positive. • Use of PCR has supplemented culture and detects shedding in up to ~25% of days. More recent data show intermittent shedding on the same day. • A culture isolate of virus is required to test for resistance • Serology can be used to assess prior exposure to HSV. The distinction between HSV glycoprotein 1 and 2 is diagnostic.

TULAREMIA ► Highly infectious gram-negative coccobacillus Francisella tularensis ► Ticks (Dermacentor variabilis (dog tick), Amblyomma americanum) ► Deerflies, rabbits, squirrels, muskrats, beavers. ► Hunters skinning animals (old days) ► Farmers, veterinarians ► Red tender local lymph node inoculation site may form ulcer ► Ulceroglandular > glandular >> oculoglandular, pharyngeal, typhoidal, pneumonic = Bioterrorism, landscapers, mowers incubation period: 3-5 days but up to 3 weeks ► DX: Serology. ► Culture of F. tularensis is lab hazard. Neg routine culture, needs chocolate agar. ► RX: Streptomycin, gentamicin, cipro in milder cases (Europe) Prophylaxis (bioterrorism) doxycycline

PLAGUE ► Yersinia pestis ► New Mexico, California, Arizona & Colorado Rodent flea bite Prairie dogs ► Fever, nausea & swollen, painful lymph nodes ► Sepsis, pneumonia-hematogenous or aerosol in crowded conditions Gram negative coccobacillus ► Bipolar-staining bacilli ► Safety pin appearance Yersinia pestis: lab hazard ► Treatment: Streptomycin >> doxy, cipro

A 65‐year old chairman of medicine at a medical school with 15 days of diarrhea, passing 4‐8 watery stools per day without fever or passage of bloody stools. He has not traveled and had an initial workup for diarrhea: standard stool culture and an order for parasites that includes a screen for Giardia, Cryptosporidium and Entamoeba. Which of the following is the best next approach? A. Collect 3 stools for parasites by EIA B. Collect 3 stools for parasites by PCR C. Perform multiplex PCR for enteric viral, bacterial and parasitic pathogens D. Ask the laboratory to perform acid‐fast staining of stool for parasites E. Give the patient 1,000 mg azithromycin in single dose

Perform multiplex PCR for enteric viral, bacterial and parasitic pathogens afb staining in stools lack sensitivity

40 y/o F 6 days of fevers and chills. a severe headache and myalgias. hiked in New Hampshire. She denied history of tick bites. She had a dog but no other animal exposures. T 103.5, BP 104/50, HR 122. No rash or adenopathy. Remainder of exam normal. WBC 2.3 (51% P, 29% bands, 14% L, 4% atypical lymphocytes); Hct 39%; Platelets 44. ALT 90. AST 100. Bilirubin normal. CXR normal. periphreal blood smear showing morulae in WBC's A:Meningococcemia B. Anaplasmosis C. Histoplasmosis D. Babesiosis E. "Spotless" Rocky Mountain Spotted Fever

Peripheral blood smear showed morulae inside white blood cells, consistent with anaplasmosis. Diagnosis confirmed with PCR testing. Treated with doxycycline, symptoms resolved. Caused by Anaplasma phagocytophilum transmitted by tick Ixodes scapularis. New England, north central states and West Coast. Incubation: 1 to 2 wks Sx: fevers, chills, malaise, myalgias and headache. Rash <10%. Labs: leukopenia, thrombocytopenia, elevated Dx: visualization of intraleukocytic bacteria (morulae) on blood smear (present in 20-80% of cases); serology (paired); serum PCR Treatment: doxycycline Rule out coinfection with Lyme, Babesia (same vector). Meningococcemia: patient did not have meningeal signs or rash to suggest acute meningococcemia; did not have arthritis/tenosynovitis/rash to suggest chronic meningococcemia Histoplasmosis: patient not immunosuppressed, which predisposes to disseminated histo; CXR was not abnormal (infiltrates often present in histo) Babesia: ring-forms in red cells, not white cells "Spotless" Rocky Mountain Spotted Fever: would not explain the morulae in her WBC

A 34-year-old HIV-negative nurse sustains a needlestick from an HIVpositive patient who has not taken ART for 2 years. Which of these do you recommend? A. No ART - low-risk exposure. B. Post-Exposure Prophylaxis (PEP) with tenofovir (TDF)/emtricitabine. C. PEP with tenofovir (TDF)/emtricitabine and an integrase inhibitor. D. PEP with tenofovir (TDF)/emtricitabine and a protease inhibitor

Postexposure prophylaxis (PEP) for occupational exposure: • Assess nature of exposure: source fluid (blood, blood tinged fluid, vaginal secretions), volume of fluid, type of exposure, timing • Assess exposure source; HIV and hepatitis testing • Testing (baseline, 6 + 12 wks + 6 months with standard HIV Ab or 6 wks + 4 months if new HIV Ab/p24 test used) and counseling • Offer 4 weeks of rx for recognized transmission risk • start ASAP (within 72 hours) • tenofovir (TDF)/emtricitabine + dolutegravir (not in women in early pregnancy or sexually active and not on birth control) or raltegravir • adjust regimen for possibility of resistance in source patient • f/u within 72 hours

• 25 yo Male makes an appointment to request PrEP • Generally feels well • No significant PMHx • No medications • Has ~2 - 3 different partners per month (sometimes more) • Mostly unprotected oral and rectal intercourse (insertive and receptive) • Two episodes of syphilis in the last year • One episode of GC / Chlamydia He is HIV neg and has normal kidney function. You prescribe daily: A. Tenofovir DF / emtricitabine (FDC) B. Tenofovir AF / emtricitabine (FDC) C. Tenofovir DV / emtricitabine / efavirenz (FDC) D. Tenofovir AF / emtricitabine / bictegravir (FDC) E. Would not use PrEP in this setting

PrEP for populations whose annual HIV incidence is at least 2% • HIV-seropositive and -negative individuals should be reminded that condoms are required to prevent acquisition of non-HIV STIs • Quarterly screening for asymptomatic STIs for all populations with high rates of bacterial STIs and incomplete condom use • Daily TDF/emtricitabine for men and women and transgender individuals at risk of sexual exposure and people who inject drugs • 1-week lead-in time with daily dosing for rectal, penile, and vaginal exposures, with daily TDF/emtricitabine to ensure adequate tissue levels are achieved • At PrEP discontinuation, TDF/emtricitabine should continue for 1 week after the last sexual exposure Pericoital TDF/emtricitabine PrEP (on-demand, event-driven, or "2-1-1") dosing is an alternative to daily PrEP for MSM with infrequent sexual exposures. This regimen is not recommended in other risk groups or in patients with active HBV infection because of the risk of hepatitis flare and hepatic decompensation • If intercourse is planned in the context of 2-1-1 PrEP regimen, the first (double) dose of TDF/emtricitabine should be taken closer to the 24-hour precoital time than the 2-hour time • TDF/lamivudine, TAF/emtricitabine, and TDF alone are not recommended for PrEP • TDF-based PrEP is not recommended in persons with CrCl <60 mL/min

30 yo woman with HIV (CD4 cell count 20, not on therapy) presented with gradual onset of word-finding difficulties, expressive aphasia and right upper extremity weakness over 4 weeks. She lived in New England. No recent travel or known insect bites. Not sexually active On exam, she was afebrile. She had oral thrush. She had difficulty naming objects and right-sided weakness. Studies: WBC count of 2.2 (44% P, 45% L) MRI: Abnormal T2 signal involving white matter, left fronto-parietal region. No enhancement, edema, mass effect An arbovirus B. A polyomavirus C. A herpes virus D. A spirochete E. A dematiaceous fungus

Progressive multifocal leukoencephalopathy CSF JC virus positive Demyelinating disease of central nervous system caused by reactivation of JC virus, a polyoma virus Immunocompromised hosts (heme malignancy; HIV, natalizumab, rituxamab) Rapidly progressive focal neurologic deficits, usually due to cerebral white matter disease. Rx: reversal of immunodeficiency. In HIV+ patients: antiretroviral therapy Arbovirus, such as West Nile Virus: Unlikely because of no confusion, headache, meningeal signs, paralysis. Herpes virus, such as HSV: temporal lobe Spirochetal infection, such as syphilis: central nervous system gumma or stroke-like syndrome (meningovascular disease) Dematiaceous fungus: no risk factors (e.g. adjacent paranasal sinus infection, penetrating trauma); lack of enhancement of brain lesion on head imaging

60 y.o. with known COPD and CHF admitted with CAP Diagnostic "bundle" performed and then empiric Ceftriaxone and Azithromycin started Within 24 hrs: Influenza A and S.pneumoniae detected in airway Oseltamivir started EKG: Non-specific ST,T abnormalities, QTc 440 msec Atrial fibrillation with RVR treated with Amiodarone Haloperidol PRN for mild delirium Furosemide for CHF On day 2: Torsade de Pointe and then Fatal Cardiac Arrest Which one of the following was the most likely cause of the fatal cardiac arrthymia ? A. Amiodarone, Haloperidol, and azithromycin B. Occult coronary artery disease C. Influenza myocarditis D. Furosemide-induced hypokalemia

Prolonged QTc and Risk with Macrolides and Fluoroquinolones Can be congenital or acquired Use caution if: Family history of unexplained sudden death Electrolyte abnormalities (low K and/or Mg) Concomitant drug(s) that inhibit cytochrome P450 enzymes &\or prolong QTc (e.g., Haldol) Underlying known cardiac disease QTc Drug list: Azithromycin added to ceftriaxone therapy of pneumococcal pneumonia to modulate the innate immune response Azithromycin added to anti-pseudomonas therapy to inhibit quorum sensing toxin production Azithromycin is Not added to empirically treat concomitant "atypical" pathogen Azithromycin is Not added to enhance anti-pneumococcal activity of the ceftriaxone

A 22 year old previously healthy male contractor returned from Afghanistan one week prior to presentation. He had a three day history of fever, myalgia, arthralgia, mild headache & cough. He had vomited once & had mild midepigastric, nonradiating pain. ► The facility he was hired to guard was adjacent to the path that the local sheep & goat herders used on their way to market & he had purchased a wool rug from one of the locals. He remembers shaking it hard to get rid of the dust. ► He reported that some members of his guard unit also had flu-like illness from which they recovered without treatment. ► Examination was normal except for a variable temperature up to 102oF ► WBC 3.3K, platelets 121K, creatinine 1.2, AST 144, ALT 154, alk phos 88, total bilirubin 0.6 ► Admission chest xray was normal ► Ceftriaxone was begun but the patient remained febrile & had the chest CT shown on the next slide Which of the following is the most likely diagnosis? A. brucellosis B. anthrax C. leptospirosis D. Q fever E. Visceral leishmaniasis

Q FEVER ► Coxiella burnetii: tiny cocco baccilus Infects cows, sheep, goats, cats, etc . ► Spores survive in straw, manure, meat, parturient tissue for months. Aerosol, ingest raw milk ► Acute pneumonia (in half cases), fever, headache, hepatosplenomegaly ► Chronic endocarditis on native or prosthetic valves ► Granulomatous hepatitis ► DX: serology, valve PCR or immunocytochemistry ► RX: acute: Doxycycline or levofloxacin or azithromycin ► Chronic: doxycycline plus hydroxychloroquine

A 45 y/o USA homeless male presents with fever and severe polymyalgia. On physical exam, animal bite marks found around his left ankle. A faint rash is visible on his extremities. Within 24 hrs, blood cultures are positive for pleomorphic gram-negative bacilli. Which one of the following is the most likely diagnosis? A. Pasteurella multocida? B. Haemophilus parainfluenza? C. Spirillum minus? D. Streptobacillus moniliformis?

Rat bite fever USA: Streptobacillus moniliformis; Asia: Spirillium minus Bites or contaminated food/water S.moniliformis: Fever, extremity rash(Mac./Pap, pustular, petechial, purpuric Symmetrical polyarthralgia Treatment: Penicillin or doxycycline

A 51 year‐old male with an 18 year history of well controlled HIV was seen on a routine visit and found to have a rising creatinine. One month prior, his antiretroviral medication has been changed at his request for a once daily regimen. His viral load had been undetectable and his CD4 750 while taking emtricitabine‐ tenofovir disoproxil fumarate (TDF)‐lopinavir‐ritonavir. He was HLA‐B *5701 negative. His new regimen for the past month was abacavir/dolutegravir/lamivudine. His serum creatinine one month prior had been stable at 1.45 mg/dl but after one month had risen to 1.79 mg/dl. His urinalysis now is still normal as was his serum phosphate and electrolytes. He had no unusual physical activity, continuing his desk job. He was taking no other medications and had no history of renal stones, hypertension or diabetes mellitus. What is needed now to address his rising creatinine? A. Nothing B. Renal Ultrasound C. Urine culture D. Change back to his prior regimen E. CT of the abdomen and pelvis

Rationale: A slight rise (0.1‐ 0.2 mg/dl) is expected from dolutegravir because it decreases creatinine clearance without an effect on glomerular filtration rate. Dolutegravir affects the function of the organic cation transporter 2 (OCT2) in the renal tubular cell which effluxes creatinine. The creatinine rise is seen in the first month of treatment and then the serum creatinine stabilizes.

Law enforcement officers brought to the emergency room three suspects who appeared to be preparing a bioterrorism weapon in the house where they were apprehended. In the course of the arrest, one of the suspects blew a white powder into the face of one of the arresting officers. After decontamination, this officer and the suspects are brought for observation and chemoprophylaxis because the suspects alleged that this powder, which is being investigated, is anthrax. The appropriate hospital isolation precautions for these persons is which of the following: Routine hand hygiene and gloves Surgical mask, gown and gloves N95 mask, gown and gloves Powered air-purifying respirator (PAPR), gown and gloves Powered air-purifying respirator (PAPR), disposable scrubs, gown, gloves and shoe covers

Routine hand hygeiene and gloves

26 y/o 20 week pregnant female (2nd preganancy) - HIV neg; screen Toxo IgG IFA @ week 6 is 1:160 - her first pregnancy titer was 1:80 - normal PE, except 1 cm ant & post cervical LN - couple of days of malaise - has cat exposure What to do?

She can't transmit disease since she acquired infection prior to this pregnancy; unless HIV + or transplan patients Titer has to change 4-fold If sero-convert IgG - to IgG + need amniocentesis Acute TOxo = mostly asymptomatic but can be febrile cervical adenopathy or mononucleosis without sore throat

50 yo F ulcerated lesion on left thumb enlarged over several months despite several courses of antibiotics no sore throat, fever, chills, dyspnea or cough Three months before, travelled to Ecuador, where she stayed in an ecotourism hotel near a river. No known fresh- or salt-water exposure. Reported seeing several kinds of insects and receiving several bites. No known animal exposures or tick bites. Patient appeared well. T 98.1. Raised ulcerated lesion on thumb with a violaceous border A. Cutaneous leishmaniasis B. Mycobacterium marinum C. Sporotrichosis D. Pyoderma gangrenosum E. Tularemia

Skin biopsy showed amastigote, with kinetoplast in a vacuole. Culture of tissue from skin biopsy in Schneider's Media revealed promastigotes. PCR of tissue: Leishmania guyanensis. Mycobacterium marinum: patient did not have known freshor salt-water exposure; she did not have nodular lymphangitis Sporotrichosis: no known exposures to soil or thorn; she did not have nodular lymphangitis Pyoderma gangrenosum: patient did not have known inflammatory bowel disease or other underlying pre-disposing condition; ulcerative PG usually occurs on lower extremities, trunk Tularemia: no animal or tick exposure; no systemic symptoms; no adenopathy

Trichuris trichiura (whipworm) 4cm long nematode Life cycle: Fecal-oral In heavy infections: - loose and frequent stools - tenesmus - occ blood to frank blood - in heavily infected children: rectal prolapse Dx: eggs are football shaped with two polar plugs

Strongyloides stercoralis (can complete lifecycle in host!) Usual manifestations GI: mild abdominal/epigrastric pain Pulm: wheezing, transient infiltrates Skin: urticarial rashes, larva currens Hyperinfection syndrome immunocompromised state steroids, TNF-inhibitors, HTLV-1, malignancy, malnutrition....NOT HIV large burden of parasites GI: Nausea, vomiting, abdominal pain, diarrhea, erosions b/c millions of larvae in intestinal mucosa Pulmonary: diffuse infiltrates, wheezing, dyspnea, cough Systemic: fever and hypotension due to gram negative sepsis Strongyloides stercoralis Diagnosis • stool o/p (sensitivity is low - 30-60%) • serology Treatment of choice: ivermectin Prevention in pts from endemic countries who are about to be immunosuppressed • Empirically treat, or check serology and treat if positive.

26 year old presents with cryptococcal meningitis and newly diagnosed HIV (Rx with AMB +5FC; to fluconazole) HIV RNA 740,000; CD4= 23 cells/ul Baseline labs: CSF: 2 lymphocytes / protein 54 / glu 87 (serum 102) OP = 430 mm H20 Started on TAF/FTC /Bictegravir at week 2 Returns 6 weeks later, Fever 103 and a mass in supraclavicular region (3 x 4 cm) Which of the following is the most likely cause of the new mass? A. B Cell Lymphoma B. Multicentric Castleman's Disease C. IRIS reaction to cryptococcus D. Mycobacteria Avium Complex E. Bacterial Abscess from prior PICC line

TESTING POINT: Every ID physician should know that IRIS can occur to pre-existing OI in setting of recent potent ARV administration - B-cell lymphoma, no other symptoms -castleman usually axillary - MAC IRIS can occur - coudld have bacterial abscess Immune Reconstitution Inflammatory Syndrome Occurs 4 - 12 weeks after initial ARV administration Most often in patients with advanced HIV infection High viral load / low CD4 count TB, MAC, crypto, PML, KS are most common OIs Is NOT related to type of ARV therapy

45 year old recently diagnosed with HIV HIV RNA 140,000; CD4= 230 cells/ul Baseline labs: Hb 11.2 gm/dl; AST 310 / ALT 120 140|101 | 5 Gluc 100 4.2 | 28 |1.1 eGFR = 65 ml/min Started on TAF/FTC+ Dolutegravir; No other medications Returns 4 weeks later, labs unchanged except creatinine now 1.3 mg/dl (eGFR 55) Which of the following is the most likely cause of her increased creatinine / reduced eGFR? A. Glomerular lesion B. Proximal Tubule damage C. Proximal Tubule inhibition D. Distal Tubule damage E. Distal Tubule inhibition

TESTING POINT: Every ID physician should know that cobicistat inhibits a proximal tubule enzyme that excretes Creatinine Anion/cation active tubular transports Cobicistat, tenofovir and dolutegravir interfer with acitve transport of creatine

Which of the following is the highest relative risk for his Acute MI? A. Cigarette smoking B. Lipid levels (LDL level of 180 / HDL 30) C. Abacavir use D. Lack of use of aspirin E. HIV infection

TESTING POINT: Every ID physician should know that the usual risk factors are still the most important predictors of CV disease -Age < 42 - smoke high RR

46yowf c/o (CD4 582, VL 387 6/10/13) c/o 1 week cramps in calves, tingling in hands, feet Today awoke and can't move except hands/feet No F/C, chest pain, SOB, incontinence + chronic diarrhea 4x/day Chronic fatigue, poor appetite Meds TDF/FTC/EFV (2008), prior AZT/3TC, ATV-RTV zoloft, buproprion, norco, prilosec, trazodone, pravachol ibuprofen VS: T 98.2 P 79 BP 112/73 RR 16, O2 sat 97% Pertinent findings Neuro: CNII-XII intact, strength 1+ all extremities except 4+ hand/wrist and ankles. Nl reflexes. Alert, oriented. Labs Cr 1.0, phos low , UDS + cocain Ca 8.3 Phos 1.8 Mg 2.1 Lactate 1.5 CK 186 UDS +cocaine/benzo/opiate UA: 1.015 pH 6.5 2+ pro Neg: gluc/ketones Which of the following is the most likely diagnosis? A. Cocaine toxicity B. Nucleoside-induced myopathy (ragged red fiber disease) C. Serotonin Syndrome D. Statin toxicity E. Fanconi's syndrome

Testing point: Every ID physician should be able to recognize TDF induced Fanconi's Syndrome A- Cocaine toxicity: Muscle toxicity can occur; usually cardiac or rhabomyolysis. B. Nucleoside-induced myopathy (ragged red fiber disease): ZDV induced; chronic, not acute C. Serotonin Syndrome: mental status normal, no shivering / fever, myoclonus D. Statin toxicity: CPK normal E. Fanconi's syndrome Type II RTA - associated with tenofovir Generalized proximal tubule dysfunction Hypophosphotemia, renal glucosuria, hypouricemia, aminoaciduria Not all have present at once Osteomalacia can occur Recovery is the rule; can take months

25 year old man presents with newly diagnosed HIV Had an episode c/w acute seroconversion syndrome 4 months ago Initial HIV RNA 40,000; CD4 443 cells/ul He wants to start ARV therapy A baseline genotype is ordered that shows an M184V mutation. Which of the following drugs will have reduced susceptibility with this mutation? A. Efavirenz B. Zidovudine C. Tenofovir D. Etravirene E. Emtricitabine

Testing point: Every ID physician should know that M184V reduces FTC/3TC activity increases efavirenz, zidovudine, tenofovir susceptibility

55 year old man presents with R hip pain H/o COPD requiring steroids frequently HIV diagnosed 17 years ago On TDF / FTC / EFV for 10 years; originally on IND / AZT / 3TC Initial HIV RNA 340,000; CD4 43 cells/ul Now HIV RNA < 50 c/ml; CD4 385 cells/ul Electrolytes NL; Creat 1.3; Phos 3.5 Ca 8.5 Mg 2.1, alk phos 130; U/A neg R Hip film unremarkable Which if the following is the most likely underlying cause of his hip pain? A.Osetonecrosis of Femoral Head B.Fanconi's syndrome C.Vitamin D deficiency D.Tenofovir bone disease E.Hypogonadism

Testing point: Every ID physician should know that Osteonecrosis is a common syndrome with long-standing HIV Osetonecrosis of Femoral Head: Prior steroid / PI use - requires hip replacement B.Fanconi's syndrome: Assoc with TDF; but Phos, creat, and U/A nl. C.Vitamin D deficiency: VERY common; often overlooked. Leads to osteoporosis / malacia/ fx D.Tenofovir bone disease: Happens, but rare. Occurs in unusual places (1st rib) usually fractures. E.Hypogonadism: Could be a contributing factor, but Vit D deficiency much more common cause of Fx

What vector is most likely to transmit an infection which would produce this EKG acutely in the state of Maryland? MosquitoFlyTickFleaBed bug

The EKG shows second degree heart block with a 3:1 capture ratio. Heart block is a common cardiac complicationof Lyme disease, starting from days to weeks after a bite by an infected Ixodid tick. If EKGs were performedon all early Lyme patients, then first- or second-degree block would identifiedfrequently,butsince this is usually asymptomatic, this transient state that resolves with antibitoic therapy is rarely known. Third-degreeblock that is most commonly diagnosedcardiac complication because of symptoms associated with a heart rate in the 30s. Heart block also occurs in acute rheumatic fever and chronic Chagas' disease. Chagas' cardiomyopathy can occur years after the bite of a Triatome (reduviid) bug in South America but is very rare in US residents. There are a few cases transmitted in the United States, e.g., in Texas, California, Louisiana, and Tennessee. However, there are 300,000 people in the US who mightdevelop heart block after acquiring the disease while residing in Latin America. Heart failure usually accompanies cardiac conduction abnormalities in Chagas' cardiomyopathy. The other insect vectors listed are not associatedwith diseases that cause AV block.

23 y/o monogamous M, comes for HPV vaccine, same relationship with same man x3years, no allergies and never received HPV vaccine He is too old for the catch-up schedule for the HPV immunization of boys. If he is monogamous, he is at low risk for the acquisition of HPV and does not need to be immunized. He should begin immunization with the nine-valent vaccine. He should begin immunization with either the bi-valent or the nine valent vaccine.

The nine-valent vaccine (Gardasil 9) protects against genital warts as well as lessening the chance of transmitting viruses that cause cervical, vulvar and anal cancer. The bivalent vaccine is only for cervical HPV and is not recommended for men. Catch-up vaccination is currently underway for males 13 to 21 years of age. Catch-up immunization is also recommended for men who have sex with men through age 26.

32. You are called to see a patient in the ICU with ARDS (adult respiratory distress syndrome). She is being treated with imipenem, vancomycin, and ciprofloxacin for sepsis of unknown etiology. The patient is a previously healthy woman who presented to the ER in Washington, DC in April with fever, chills, mylagias, and dehydration. She had just returned from her country cabin in the rural California mountains. Because her evaluation was non-specific other than mild LFT abnormalities, she was rehydrated and sent home without antimicrobial therapy. The next day she returned with fever, hypotension, and respiratory failure requiring intubation. Laboratory testing revealed a WBC of 3400/µL, hemoglobin level of 11.4 g/dL, and platelet count of 19000/µL. The following was seen on peripheral smear (see photo): The most likely reservoir of this pathogen was: Choices A Another human B Aquatic birds C Lake Fish D Rodents E Deer

The organism in the blood stream is a spirochete: The only ones seen in the bloodstream in the United States are the agents of tick borne relapsing fever, including B. hermsii. Spirochetes include the genera Spirochaeta, Treponema, Borrelia, and Leptospira. Leptospira - not in blood Syphilis - not in the blood lyme diseae - Why did the patient not report a tick bite? B. hermsii is transmitted to humans through the bite of Ornithodoros ticks. These ticks typically feed for less than 30 minutes and usually at night; consequently, most patients do not recall being bitten. Rodents are vertebrate reservoirs for the disease, and cabins or homes located at elevations of 2,000-- 7,000 feet in coniferous forests are common exposure sites. Outbreaks associated with such sites have been reported from 11 Western states. In other regions Borrelia recurrentis (louse-borne relapsing fever) might be seen in a blood smear.

A 59-year-old woman presents to her physician's office complaining of dysuria, hematuria, and frequency of 3 days duration. The patient is frustrated by these recurrent symptoms as they are significantly impacting her quality of life. She presented with similar symptoms three previous times over the past 6 months and was diagnosed with a urinary tract infection at each visit. She was treated with Bactrim (TMP/SMX) and her symptoms resolved. A urine culture obtained at the time of her first symptomatic presentation yielded an antibiotic-susceptible E. coli. The culture was obtained to confirm the diagnosis. It was the first time that she had experienced a symptomatic urinary tract infection. No cultures were obtained at the subsequent visits given the identical clinical presentations. Her past medical history was significant for hypercholesterolemia, hypertension, and recurrent otitis media as a child. She had a normal Papanicolaou smear 6 months earlier. Her physical examination demonstrated no costovertebral angle tenderness and mild suprapubic tenderness. A speculum examination revealed a pale, smooth, and dry vaginal epithelium with patchy erythema, areas of friability, and a yellow malodorous vaginal discharge. Vaginal pH was 6.0 and a wet smear demonstrated rounded parabasal epithelial cells with large nuclei and many inflammatory cells. Gram staining of a vaginal smear demonstrated a mixed flora of gram-negative rods and gram-positive cocci. After treating her acute cystitis, which of the following interventions is most likely to decrease her future risk of recurrent urinary tract infections? Antibiotic suppression Boric acid vaginal suppositories Metronidazole therapy for bacterial vaginosis Topical vaginal estrogen cream Urological assessment for pelvic organ prolapse

The patient has had recurrent urinary tract infections. Given her age, she is likely post-menopausal. Her examination is consistent with vulvovaginal atrophy: a pale smooth and dry vaginal epithelium, an inflammatory discharge, high pH, and evidence of dysbiosis (a shift from a lactobacillus-dominant microbiota to a diverse dysbiotic microbiota). This clinical entity is the result of a hypoestrogenic state. Treatment with topical estrogen has been shown to improve vulvovaginal atrophy and decrease the risk of recurrent urinary tract infections (systemic estrogen therapy has not been shown to reduce the risk of recurrent cystitis). Antibiotic suppression in this case is not warranted, as it does not address the underlying risk factor for these recurrent bouts of cystitis. Boric acid vaginal suppositories have been used to treat vulvovaginal candidiasis but not atrophic vaginitis. This patient's clinical findings are most consistent with vulvovaginal atrophy rather than bacterial vaginosis (in general, BV does not cause a dry vaginal epithelium, areas of mucosal friability, or significant inflammation). While it is often difficult to differentiate BV from atrophic vaginitis in clinical practice, the findings described in this case favor the diagnosis of atrophic vaginitis. There is no mention of pelvic organ prolapse on physical examination.

A 65 yo American male has had unprotected sex with men for many years. The HIV‐1/2 ELISA is reactive and western blot positive. Viral RNA level is <20 copies/ml and CD4 count is 700 cells/µl Which of the following is most likely: A. The patient is in the window period of HIV‐1 infection. B. The patient is chronically infected with HIV‐1 but is a long term non progressor. C. The patient is infected with non‐B subtype of HIV‐1 D. The patient is not infected with HIV‐1 or ‐2, I.e. HIV tests are false positive.

The patient is chronically infected with HIV‐1 but is a long term non progressor. • Represents authentic HIV infection • Slow progression • Associated with specific HLA subtypes, e.g., B57 • SOME non‐subtype B HIV are not well quantified by HIV viral load testing • rarely <20 c/ml when viremia is present A - it never happens, viral is always a million, 500,000. C- our tests pick up both HIV-1/2 and all subtypes D - ELISA and western blot + then he is infected

A 29-year-old anthropologist presents with a chief complaint of recurrent soft tissue swellings. He spends several months a year in tropical areas of Cameroon. For the past three months he has noticed occasional focal swellings of his hands and forearms. These are about 4-6 cm in diameter, last for about a day and then spontaneously resolve. Giemsa stain of a filtered daytime blood sample shows the following. Which of the following should be assessed prior to initiating therapy? Lumbar puncture Quantitative microfilaria count Stool exam for ova and parasites Brain MRI Glucose-6-phosphate dehydrogenase level

The patient's symptoms are consistent with Calabar swellings due to Loa loa. This is further supported by the finding of microfilaria in a daytime blood sample. The treatment of choice for L. loa is diethylcarbamazine (DEC). However, DEC can cause significant adverse effects, including death from encephalitis and shock, in patients with very high circulating microfilaria levels (>2,500 microfilariae/ml). This is believed to be due to a severe inflammatory response against dying microfilariae. Thus, high L. loa microfilaria levels are a contraindication for DEC treatment. Thus, before using DEC, the circulating level of microfilaria can be reduced with apheresis or albendazole. Antihistamines and corticosteroids have been used to limit post-treatment reactions, but do not prevent serious side effects such as encephalopathy.

A 37-year-old man presents asking about HIV PrEP (Pre-Exposure Prophylaxis). He is in a monogamous relationship with his HIV- husband of 7 years. He is HIV antibody negative on rapid testing and his creatinine clearance is calculated at >100 cc/min. What do you recommend? No PrEP start tenofovir DF once daily start tenofovir DF/emtricitabine before and after sex start tenofovir DF/emtricitabine/elvitegravir/cobicistat once daily

This man does not appear to be at risk of acquiring HIV, so PrEP would not be appropriate. PrEP is FDA-approved and CDC-recommended for sexually active adults at high risk for HIV. If his circumstances changed, PrEP could be appropriate with tenofovir/emtricitabine taken once daily. Although the IPERGAY study suggested that episodic PrEP could be effective, this strategy is not yet FDA approved. Tenofovir DF alone was only shown effective in a single study; the 4-drug regimen is for treatment, not prevention. Correct Response No PrEP

What complication is it most likely to be associated with lesion on tip of nose? A. Deafness B. Vertigo C. Optic neuritis D. Keratitis E. Stroke

This patient has Hutchison's sign, which indicates involvement of the cranial nerve V, i.e. ophthalmic branch of the trigeminal nerve, which inervates the tip of the nose and the globe. After a prodrome of fever and headache for 1-4 days, patients develop a cutaneous rash. Days or up to 3 weeks later, the sclera and cornea can be involved. Thus, keratitis is the correct answer. Deafness or vertigo would be more characteristic of geniculate ganglion (CN VII) involvement, i.e. Ramsay Hunt, which is a polyneuropathy involving the cranial nerve VII, and then often involves VIII, IX, X. Thus A and B are not the best answers

What complication would you be most concerned of vesicular lesions near the ear about? A. Facial paralysis B. Keratitis C. Encephalitis D. Optic neuritis E. Oculomotor palsies

This patient has Ramsay Hunt syndrome (Herpes zoster oticus), caused by VZV reactivation in the geniculate ganglion, i.e. zoster of CN VII, presenting with severe ear pain and reduced hearing or deafness. When vesicle are seen in the auditory canal, abnormalities in cranial nerves VII, and sometimes VIII, IX or X, can occur. Thus A, facial paralysis is the best answer. Acyclovir is usually recommended although its not clear if it's effective. The facial paralysis is more severe and less likely to resolve than the usual HSV related Bells Palsy. • Keratitis would be more typical of a lesion on the tip of the nose, or zoster ophthalmicus involving the CN V ophthalmic branch. • Encephalitis can be caused rarely by VZV and would not be the best answer. Stroke syndromes due to carotid intimal involvement are associated with zoster, and often with cranial nerve V (trigeminal involvement), but are not offered as an answer

This Haitian farmer was visiting his family in the United States when he sought help for this painless firm foot swelling of ten years duration. The slide shows the foot x‐ray of the foot and a biopsy of the lesion stained with H&E. The most likely organism is which of the following: A. Coccidioides posadasii B. Burkholderia pseudomallei C. Staphylococcus aureus D. Madurella mycetomatis E. Mycobacterium ulcerans

This patient has a mycetoma, or Madura foot, which is characterized by a slowly progressive, painless destruction of the subcutaneous tissue that dissects up fascial planes and sometimes involves contiguous bone. This is caused by inoculation of the organism by minor trauma into the subcutaneous tissue, usually the foot. Persons picking up firewood and carrying it on the back or on the head may have lesions on the hands, upper back or top of the scalp. There are tiny grains in the draining material that represent clumps or organisms. This entity can be caused by filamentous branching bacteria such as Nocardia, Streptomyces, or Actinomadura species. The entity can also be caused by fungi such as Scedosporium apiospermum, Madurella species, and Fusarium species. Thus, empiric therapy would be inappropriate until the causative organism was known to be a fungus or a bacterium. This distinction can be made by microscopic examination of a grain in the sinus drainage. The biopsy shows a grain with numerous hyphae (Madurella species is a fungus). Mycobacterium ulcerans causes a chronic ulcer (Buruli ulcer) without grains or bone invasion.

. A 27-year-old previously healthy woman from Philadelphia awakened this morning with blurred vision in the left eye. She had no previous eye problems. She has no eye pain, no trauma history, no fever and no other symptoms except for mild malaise over the past 4 days. She works as an assistant in a nursing home. Her yearly PPD skin tests have been negative. She has had a single sexual partner for the past 3 years; she believes he is monogamous as well. She traveled to Ohio to visit a cousin one year ago; there is no other travel history. She owns a cat that she got as a kitten 3 months ago. She has no allergies and takes no medicines. She is afebrile and her examination including neurological exam is unremarkable except for decreased vision in the left eye. You send her to the eye clinic for a careful examination which is said to show mild papilledema with stellate (star burst) retinal edema in the macula. Which one of the following is the most likely cause of her eye problem? Choices A Cytomegalovirus B Treponema C Mycobacterium D Bartonella E Toxoplasma

This patient has neuroretinitis as manifested by acute visual loss from optic nerve edema associated with macular edema. Bartonella henselae, the cause of cat scratch disease, is one of the most common causes of this rare problem. The stellate exudates in the macular area of the retina along with optic nerve edema are characteristic. Diagnosis is by serology. Patients are usually healthy, present with unilateral visual impairment, and have a history of cat exposure. Most recover completely with or without treatment. Cytomegalovirus retinitis is seen almost exclusively in those with severely impaired cell-mediated immunity (e.g. advanced HIV infection or less commonly organ transplant recipients or those receiving certain lymphocyte depleting biologics). Mycobacterium tuberculosis may cause retinal exudates as part of disseminated disease but there are no other features here to suggest that problem. Toxoplasma chorioretinitis may occur in normal hosts due to reactivation of congenital infection; retinal lesions are characteristic and not stellate in form.

A patient with HIV infection (CD4 count of 15 cells/µL, VL 2 million) has a 3-year history of a recurrent perianal herpes simplex that had in the past responded to acyclovir or valacyclovir but recurred repeatedly. On this occasion, the painful ulcer has not responded to a 10-day course of acyclovir 400 mg TID followed by a 10-day course of valacyclovir 1g bid. The patient has been adherent to his regimens. If virus DNA sequencing for resistance mutations cannot be obtained, the best therapeutic option would be: Intravenous acyclovir Oral famciclovir Oral valganciclovir Intravenous ribavirin Intravenous foscarnet

This patient is likely infected with a thymidine kinase (tk)-deficient Herpes simplex virus, and the virus is resistant to high-dose acyclovir and to valacyclovir (a pro-drug of acyclovir), famciclovir (a pro-drug of penciclovir) and ganciclovir or valganciclovir. Foscarnet and cidofovir are both effective in tk-deficient HSV, but cidofovir is much more toxic than foscarnet. Topical cidofovir is effective in some cases, but topical form of this drug is not commercially available. Ribavirin has no activity for Herpes viruses.

A 53 y/o male construction worker has sudden onset of pain in his left calf. Within hours the skin and subcutaneous tissue of the calf are red, edematous and tender. Red "streaks" are seen spreading proximally. A short time later, patient is brought to the ER Confused, vomiting, and hypotensive. Temp is 40°C with diffuse erythema of the skin. Oxygen sat. is 88% (room air). WBC is 3000 with 25% polys and 50%band forms. Platelet count is 60,000. Which one of the following is the most likely complication of the erysipelas? A. Bacteremic shock due to S. pyogenes? B. Toxic shock due to S. pyogenes? C. Bacteremic shock due to S. aureus? D. Toxic shock due to S. aureus?

Toxic Shock Syn. (TSS): Staph. Versus Strep. Feature Staphylococcal Streptococcal staph aureus toxic shock syndrome has colonizaatio Tampon, surgery; -colonization - no focal pain - no tissue necrosis or inflammatoin - n/v, renal failure and DIC - erythoderma - rare bacteremia mortality <3% Strep Cuts, Burns, Varicella, erysipelas - has focal pain -common sign of tissue necrosis/inflammation - n/v, renal failure/DIC - less common erythoderma - 60% bacteremia 0 Mortality <3% 30-70%

• You are asked to see a 25 yo pregnant woman admitted to the ICU with "severe Varicella and viral pneumonia vs 2° bacterial pneumonia" requiring intubation • She had been ill for ~1 week prior to the onset of her rash with fever, malaise, headache and myalgias • Her lesions started on her face, hands and feet 3 days ago at which point she rapidly developed cough and dyspnea. Admitted the previous night. • She is currently on high dose IV acyclovir, ceftriaxone and vancomycin • The chief resident took this photo while the patient was still in the ER • A scraping of the one lesion was sent for DFA for HSV and VZV but the tech was not available last night • Even with no further tests available what concerns you about the dx of Varicella? •

Variola smallpox has been going on for a week, prodormal, not usually like varicella varicella typicall starts on trunk and later moves to the exteremiteis small pox synchronus, all lesions same site can appear on palm/soles , but never in chicken pox reservoir - human transmission person to person via respiratory droplets incubatio 7-17 days prodrome 1-4 days prior, sick, but not transmissible 2-4 days start having rash , papules, vesicles, pustules, scabs, painful lesions

48 yo Male presents with newly diagnosed HIV infection • Asymptomatic except for weight loss / fatigue • Initial: HIV RNA 160,000 c/ml CD4 count 221 cells/ul • Other labs are normal; Started on ARV Rx • Returns for a 3 month follow up visit • HIV RNA < 20 c/ml; CD4 390 cells/ul Assuming he remains undetectable, you tell him that his risk of transmitting HIV to his seroneg partner via sex is: A. Virtually zero risk (< 0.2%) B. Very low risk (< 2%) C. Possible (<10 %) D. It depends on which ARV regimen he's on

Virtually zero risk (< 0.2%) 548 heterosexual and 972 discordant gay couples followed up to 8 years Seropositive partner had VL < 200 c/ml 77,000 sexual acts without condoms Zero transmissions (from seropositive partner) Upper bound of 95% CI: 0.23 /100 CYFU Sexual Transmission from a person with Undetectable Viral Load is Effectively Zero

Ampho: not Scedosporium (Pseudallescheria boydii), Candida lusitaniae, Asperillus terreus Only ampho for mucormycosis Fluconazole: not Candida krusei , Candida auris, +/‐ Candida glabrata Echinocandins: not Trichosporon or crypto Know mechanisms of action: glucan, sterol, cell membrane, DNA synthesis Flucytosine WBC & plt fall, diarrhea, hepatitis

Voriconazole: phototoxicity, periostitis, hallucinations Azole interactions: Increases other drug levels: cyclosporine, tacrolimus, serolimus, steroids, imatinib, etc. Decrease azole levels: phenytoin, rifampin, etc

48 yo man admitted thru the ER the previous night with fever, chest pain and leukocytosis but no infiltrate on CXR. • Dx "Febrile bronchitis" • Begun on oral azithromycin • ID Consult called when admission blood cx are reported positive for GNRs (Gram negative rods) 6h post admission. • You arrive on the ward just as the patient is being transferred to the ICU for hypotension • While the patient is transferred you look at the peripheral smear that has been reported by the night tech as "GNRs" • When you point out that they look like GPRs (Gram positive rods) he says "probably a diphtheroid contaminant" • In 4/4 bottles in 6 hours?! • What is your differential dx?

When they say GPR, think.. Bacillus anthracis SuspectAnthrax? • Skin lesions or flu sx with potential exposure • Painless skin lesion with black eschar and surrounding edema • Unexplained sepsis, respiratory failure with: • large pleural effusion • bloody effusion • wide mediastinum • GPRs in blood cultures, CSF or (rarely) sputum specimen Definitive: Compatible clinical illness plus: 1) Positive culture from skin lesion, blood, CSF, pleural fluid ‐ grows rapidly on standard media and automated BCx systems ‐ Blood cx likely positive early in inhalation illness 2) Two positive non‐culture tests: ‐ PCR ‐ gamma-phage ‐ serology

A 52 yo diplomat and her 48 yo husband returned from Algeria yesterday and have both been admitted with cough, fever, WBCs >18k and pulmonary infiltrates. • Presumptive dx from the ER ?pneumococcal pneumonia • IV azithromycin initiated Sputa stains are shown • Many PMNs • Gram neg coccobacillary organisms 1‐2 μ long • Characteristic appearance ONLY on Wright‐Giemsa or Wayson's staining • NOT on Gram's stain

Yersinia pestis -gram negative, cococcbacilli -bipolar saftey pin (on giemsa or wayson stain) - vector is flea - priarie dogs - extremely sensitive to environment degradation (UV light) - strict respiratory isolation for 48 hrs (negative pressure room), untill the sputum is negative or treated with antibiotics - inform the lab -close contacts need post-exposure ppx with cipro or doxy

39 y.o F 4 days fever, leukocytosis 3 1/2 weeks of fever accompanied with arthralgia knee wrists and ankles during the first week T104.2 tonsilar swelling and erythema with tender cervical LN spleen palpable R wrist is swollen and painful rash noted on trunk and extermities most prominaently under the breast and in are of her underwear waistband ferritin 3600 (nl40-200) WBC 32 K (89%neut) ALT / AST 3x normal ESR and CRP 5x normal throat and bcx negative

adult still's diseae evanescent salmon colored , waxing and waning rash multisystem illness elevated ferritin (but can happen with HLH) Koebner phenomenon = rash eliciated by stroking skin or area of pressure

25 year old black woman presents with fatigue History of IV Heroin use Exam no edema Work up in ER shows creatinine 8.4 BUN 79; mild anemia; mild acidemia In ER 10 weeks earlier; normal renal function U/A high grade proteinuria US of kidneys: Normal to increase size; no obstruction Rapid HIV test positive Which of the following is the most likely cause of her renal failure? A. Volume depletion / ATN B. Heroin Associated Nephropathy C. HIVAN D. Membranous glomerulonephritis E. Tenofovir Toxicity (PrEP)

answer - C, rapid, high grade proteinuria A. Volume depletion / ATN: Tubular damage doesn't lead to high-grade proteinuria B. Heroin Associated Nephropathy: Typical nephrotic syndrome; slower progression to ESRD C. HIVAN: High-grade proteinuria; nl to large kidneys; NO EDEMA; rapid progression to ESRD D. Membranous glomerulonephritis: Less common; needs rule out. More typical nephrotic syndrome E. Tenofovir Toxicity (PrEP): Proximal tubule disease; low grade proteinuria, if at a A.An ACE inhibitor: Important adjunct, but less effective than ARV therapy B.Corticosteroids: Doesn't really work C.High Molecular Weight Dextran: No indication D.Antiretroviral Therapy: HIV directly infects kidney parenchymal cells; ARV Rx critical E.A calcium channel blocker: No Effect

A 28 year old woman returns after studying mosquito breeding habits in Honduras for one year. She reports intermittent abdominal pain and diarrhea for several months. Stool ova and parasite exam is positive for the presence of a ciliated single cell organism. What is the most likely diagnosis? A. Balantidium coli B. Entamoeba histolytica C. Giardia lamblia D. Dientamoeba fragilis E. Endolimax nana

answer A Balantidium coli • the only ciliated pathogen of humans! • largest protozoan pathogen of humans! (up to 200 m long) • found worldwide, • eating food/water contaminated with pig feces • asymptomatic can cause colitis with abdominal pain, weight loss, +/- diarrhea (especially in malnourished and immunocompromised) • Treatment: tetracycline (!) or metronidazole

35 yo female s/p heart transplant in France 90 days prior presented with fever, dyspnea and a diffuse pneumonia on chest CT. • She was receiving prednisone, tacrolimus & mycophenolate. • Both recipient & donor were CMV negative; she was not on CMV prophylaxis. • She was on inhaled pentamidine for PCP prophylaxis Trimethoprim-sulfamethoxazole was started empirically and she began improving. Bronchoalveolar lavage ( BAL) was negative for: pneumocystis by direct fluorescent antibody stain & PCR, fungi by calcifour white / potassium hydroxide stain, mycobacteria by AFB smear, bacteria by Gram stain, and respiratory viruses by multiplex PCR Routine bacterial BAL and blood cultures were negative. Assuming trimethoprim-sulfamethoxazole was causing her improvement, which additional test on the BAL might have explained her improvement? A. PCR for CMV B. PCR for toxoplasmosis C. PCR for tuberculosis D. Galactomannan E. Cold enrichment culture for Listeria

answer is B Acquired from donor, reactivation, blood transfusion or ingestion of contaminated food or water • Donor seropositive/Recipient seronegative at high risk • HEART > LIVER > KIDNEY TRANSPLANT • DIAGNOSIS: • PCR • Giemsa smear of BAL • Brain aspirate for tachyzoites • Immunoperoxidase stain of endocardial biopsy or other tissue • TREATMENT: sulfadiazine-pyrimethamine-leucovorin

numbness of lips burning and tingling of his extremitites and abdominal pain and vomititng 30 minutes after a meal in jamaica progressing to respiratory failure ddx? What is the likely diagnosis? A. Scombroid B. Paralytic shellfish poisoning C. Ciguatera D. Neurotoxic shellfish poisoning E. Monosodium glutamate toxicity

ciguatera

46 y/o healthy lives in conneticut s/p tick bite 2 days ago in may ? questions about lyme prophylaxis? when to give single dose doxy?

criteria for doxy prophylaxis: - tick ID as adult or nymph ixodes scapularis (black legged/ transmitts lyme /ehrlichia/babesia) - attached >=36 hours (outdoor exposure or engorged tick) - can take abx within 72 hours of tick removal - >=20% tick infection with B. burgdorferi - non-pregnant or breastfeeding (to take doxy) timeline: erythema migrans ~ 10days to develop IgM 7-14 days, IgG 2-6 weeks

44 yr previously healthy male accountant in Washington DC presented with the acute onset of confusion that was preceded by three months of headache. Cranial MRI was normal. Lumbar CSf had an opening pressure of 350mm CSF, WBC 250/cu mm, glucose 22 mg /dl, protein 125 mg/dl and cryptococcal antigen titer 1:512. Liposomal amphotericin B was begun at 5.0 mg/kg IV daily. On the third day of treatment he complained that the room was too dark and was found to have visual acuity of hand motion only in both eyes. The most important next step in this patient is which of the following: A. start flucytosine B. start fluconazole C. Start acetazolamide (Diamox) D. Begin daily lumbar punctures E. Start dexamethasone

daily lumbar punctures Measures to reduce intracranial pressure Daily lumbar punctures (guidelines) CSF shunt Diamox useless, can worsen renal tubular acidosis from ampho B Corticosteroids controversial.

38 y/o healthy female 1 day h/o sore throat and fever childhood history of anaphylaxis T 102.3 HEENT tonsilar erythema with purulence neck tender b/l anterior LAN Labs: rapid antigen diagnostic test is negative what to treat with? cephalexin none doxycycline clindamycin levofloxacin

do nothing modified centor criteria C- can't cough +1 E- exudate +1 N-neck adenopathy +1 T - temperature elevation +1 or -age less than 15 +1 - age > 44 ; - 1

A 55-year-old treatment-naïve man with HIV disease, CD4 320 and HIV RNA 67,000 cps/ml Lab testing reveals: toxoplasma Ab+; CMV Ab+; HAV total Ab+; HBV surface Ag+, core Ab+, surface Ab-; HCV Ab-; RPR NR Of the following, which ART regimen would you recommend? A. abacavir/lamivudine/dolutegravir B. abacavir/lamivudine + atazanavir (boosted) C. tenofovir (TAF or TDF)/emtricitabine + zidovudine D. tenofovir (TAF or TDF)/emtricitabine + darunavir (boosted)

enofovir (TAF or TDF)/emtricitabine + darunavir (boosted) hepatitis B surface Ag + Some ART has activity against HBV • lamivudine (3TC), emtricitabine (FTC), tenofovir (TDF or TAF) • Some HBV drugs have activity against HIV • entecavir McMahon NEJM 2007;356:2614 • If treatment started, treat both optimally • 2 active agents for HBV • 3 active agents for HIV • e.g. TDF*/FTC + 3rd drug

19 y/o college student sore throat, fever x 3 days not toxic fatigued the rash show in the phot0 , nonpruritic T 101F, some tonsillar exudate bilaterally and rash a rapid strep test was negative so a throat culture obtained and treament withheld next day culture reported as having no beta-hemolytic streptococci

gram (+) bacillus - Arcanobacterium haemolyticum, formerly known as Corynebacterium hæmolyticum, is a species of bacteria classified as a gram-positive bacillus. It is catalase-negative, aerobic, beta-hemolytic, and not motile occurs in young and children - sore throat and rash - looks like scarlet's fever (no strawberry tongue) - except GPB - hard to culture out - not beta-hemolytic -2-3 days to grow - easy to treat (b-lactam, macrolide) - get well so fast

You are asked to see a patient in the Bone Marrow Transplantation Unit for fever, neutropenia and pleuritic pain. The patient, a 45‐year‐old woman, had an allogeneic stem cell transplant five weeks ago for acute leukemia. She has been receiving prophylaxis with acyclovir and fluconazole. Three days ago she developed fever without localizing signs or symptoms. At that time, she had failed to engraft and remained neutropenic with an ANC of zero. Cefepime was begun. Fever persisted and vancomycin was added. On day 5 of fever, she complained of right‐sided pleuritic pain. She has no cough. A chest CT scan showed a peripheral right‐sided lung lesion which was described by the radiologist as having a "halo sign," that is, a nodular dense consolidation with hazy ground glass infiltrate surrounding it. Which one of the following is the most likely cause of the lung problem? A. Aspergillosis B. Candidiasis C. Legionellosis D. Bronchiolitis obliterans E. Diffuse alveolar hemorrhage

halo sign (but not absolutely diagnostic) of Aspergillosis. Aspergillus species can form a "halo sign" (central dense consolidation surrounded by ground glass infiltrate) seen in -invade and thrombose blood vessels. The nodule is fungus and infarcted lung; the halo is due to hemorrhage into the alveoli. The dense nodule often cavitates (Figure B) with effective antifungal therapy, especially when the neutrophil count returns, as shown below. Keep in mind that this is different from the "reversed halo sign". The reverse halo sign is a dense margin of a pulmonary infiltrate with a less dense center and has been reported to be associated with mucormycosis. Candida almost never causes pulmonary disease in any patient, and thus is not the best answer Cryptococcus is not likely is a patient taking fluconazole prophylaxis, and is not associated with a halo sign. Bronchiolitis obliterans is graft versus host disease, occurs late, usually after 100 days, wheezing bilateral ground glass infiltrates, but not nodules Diffuse alveolar hemorrhage occurs within the first 100 days post‐transplant, and presents with bilateral, diffuse ground glass. To make this diagnosis the bronchoalveolar lavage should be bloody throughout the lavage, and other causes of hemorrhage should be excluded.

A 52-year-old woman is admitted for progressive SOB, is intubated, undergoes BAL and is found to have PCP. HIV Ab test is positive, CD4 103, HIV RNA 135,000 copies/ml. She is day 4 of IV trimethoprim-sulfa and corticosteroids and still intubated. When should she start ART? A. Immediately B. In the next 2 weeks C. After completing 21 days of trimethoprim-sulfa D. At her first outpatient clinic visit

in the next 2 weeks - intubated, can't be confident of delivery of medications -drug drug interactions - C/D too long - need to be treated in setting of her acute OI •282 patients with treatable OI diagnosed within 14 days randomized to start ART within 48 hours vs. after 4 weeks • most common OI: PCP (63%) • AIDS progression/death: immediate rx (14%) vs delayed rx (24%) • No differences in safety/toxicity, IRIS, or week 48 responses

55 y/o microscope repairman had an aquarium at home with tropical fish biopsied

mycobacterium marinum - culture at low temperature

which combinaiton have been shown to improve outcome with patient with S.auresu bacteremia or NVE ASP + gentamicin ASP + rifampin vanco + B-lactam no combination

no combination therapy shown to improve mortlaity upfront monotherpay is the way no high quality RCt has ever demonstrated improved outcomes of combination ABx therapy over monotherpay

83‐Year‐Old Man with Bloody Diarrhea Develops Renal Failure • He has a one week history of diarrhea with stools containing blood; he undergoes colonoscopy which looks like ischemic colitis • As his diarrhea improves his urine output decreases • Serum creatinine is 9, platelet count of 50,000, hematocrit 20 and LDH 1,000. • Stool culture on Sorbitol MacConkey Agar grows no sorbitol‐negative E. coli and stool sample is positive for Shiga toxin 2 by EIA • He is treated with Eculizumab, a humanized monoclonal antibody inhibits the terminal sequence of complement What is the likely cause of dysentery and renal failure in the elderly man? A. Ischemic bowel disease B. Non‐O157 Shigatoxin producing E. coli (STEC) C. O157:H7 strain of STEC D. Shigella dysenteriae 1 (Shiga bacillus) E. Campylobacter jejuni

non-0157 shigatoxin producing Ecoli can be differentiated by sorbitol testing

A 25-year-old African-American male turkey farmer from rural North Carolina was taken to the Emergency Department in July because of fever and headache of approximately 48-hours'duration. His farm had a few cows and dogs. They drank unpasteurized milk from their cows. Occasionally he noted ticks on his body, but could not remember when the last time was. They didn't slaughter animals on the farm, but sent the turkeys elsewhere for processing. He sometimes walked around in stagnant water in the fields where he grazed the cows. He had been healthy and took no medications. On examination: His temperature was 40°C, pulse 110 and BP 90/60. He was obviously ill and groaning from the headache but oriented x 3. No rash was seen. Slight, but definite nuchal rigidity was found but no other neurologic signs. Blood cultures were drawn and ceftriaxone 2 gm q12h IV begun. Lumbar puncture found WBC 40/cu mm, all lymphocytes and monocytes. CSF protein was 45 mg/dL and glucose 55 mg/dL. CSF Gram stain was negative. WBC was 2,500/cu mm with a normal differential. Platelet count was 70,000/cu mm and hemoglobin 13 gm%. Routine chemistries showed aminotransferases were slightly elevated, 1.5 times the upper normal limit. Which of the following is the most likely source of this infection? Turkeys Mosquitoes Dog ticks Unpasteurized milk Stagnant water

orrect Answer: Dog ticks The presence of leukopenia and thrombocytopenia raised the possibility of Rocky Mountain spotted fever and ehrlichiosis from dog ticks. Patients with Rocky Mountain spotted fever are generally more ill than those with ehrlichiosis, so the former is more likely in this patient. Despite the name, Rocky Mountain Spotted Fever, the majority of cases occur in South Central and Southeast states, including North Carolina. Rash in spotted fever is often delayed, only occurring in the first three days in approximately half the cases. This rash is easy to miss in African Americans. The dog tick, Dermacentor variabilis, is a major vector of Rocky Mountain Spotted Fever in the Eastern United States. Monocytic ehrlichiosis is probably at least as common as Rocky Mountain Spotted Fever, and the two diseases resemble each other closely, except that rash is unusual in monocytic ehrlichiosis. The latter is carried by the lone star tick, Amblyomma americanum, which has dogs as its usual host. Whether the patient was infected with Rickettsia rickettsii, the cause of spotted fever, or Ehrlichia chaffeensis, the cause of monocytic ehrlichiosis, the most likely source was a dog tick. Treatment of either would be doxycycline. Headache, fever, and CSF pleocytosis would be consistent with leptospirosis, which could have been acquired by wading in stagnant water of his fields. Leukocytosis is common but leukocytopenia is rare in leptospirosis. Mosquito-borne viral encephalitis (West Nile, Eastern Equine, etc.) could occur in July but the normal mental status and hematologic abnormalities are atypical. The clinical picture is inconsistent with psittacosis from turkeys and with brucellosis or other infections acquired by ingestion of unpasteurized milk.

The probable source of this infection is which of the following? Female anopheles mosquitoMale anopheles mosquitoA deer tickA dog tick

orrect answer: A deer tick The organisms are Babesia microti, a hemoprotozoon of rodents, acquired by humans from bites of deer ticks. Babesia have a different developmental form than Plasmodium falciparum, resembling a "maltese cross" which are four dots in a cluster. (See arrow in photo). Infection is often mild but asplenic patients may have severe and even fatal infections. This distinction based on morphology can be difficult, but this "maltese cross" is typical enough to be testable. PCR on blood is commonly used to establish the specific diagnosis before or after an intraerythrocytic form is seen on a blood smear. Babesia is also transmitted by transfusion of whole blood and can appear weeks after transfusion.

A 74-year-old female presented with a 4-month history of fatigue and one month of dull, non-radiating lumbar back pain. She had low-grade fever four months ago which resolved with three weeks of amoxicillin-clavulanate, given for sinusitis. She has lost 10 pounds of weight, has sweating at night and recently noticed a tender swelling on her finger. She had a mitral valve repair with a prosthetic ring implanted 20 years prior and has mitral regurgitation on transthoracic echocardiogram, unchanged from 8 months prior. The finger nodule in the above photo, is suggestive of which of the following: Staphylococcal felonOsler's nodeRheumatoid nodulePolymyalgia rheumaticaJaneway lesion

orrect answer: Osler's node This is a typical appearance of an Osler's nodeand should suggest the diagnosis of subacute bacterial endocarditis, . which this patient had. Relatively uncommon clinical manifestations that are highly suggestive of IE include: Osler nodes - Tendersubcutaneous violaceous nodules most often seen on the pads of the fingers and toes, but also occur on the thenar and hypothenar eminences Janeway lesions - Nontender erythematous macules on the palms and soles Roth spots - Hemorrhagic lesions of the retina with pale centers Rheumatologiccomplaints are common in SBE and can mislead the diagnosis, particularly in the elderly, who may have preexisting cardiac murmurs. Staphylococcal felonis an infection of the pulp of the terminal digit, arising by direct inoculation. Polymyalgic rheumatica does not cause finger lesions. Nothing in the history is consistent with rheumatoid arthritis. Janeway lesionsoccur in SBE but are nontender red macules, most often on the palm. (Photo courtesy of Dr. Jillian Raybould, Georgetown Univ.)

42 y/o M 3rd episode of cellulitis 2 pervious same ear 2 and 5 months ago , resolving slowly to abx several year history of chronic nasal stuffiness afebrile right auricle is inflamed and tender dx?

relpasing polychondirtis -immune mediated condition with inflammation of cartilaginous -clinical dx buzz word: sparing ear lobe parasternal joint involvement califlower ear

28 year old HIV+ man on TDF/emtricitabine + atazanavir/ritonavir for 2 years with HIV RNA <50 cps/ml and CD4 200s300s presents for routine follow-up; labs reveal HIV RNA 102 cps/ml and CD4 352. What do you recommend? A. Obtain genotype. B. Obtain genotype and phenotype. C. Repeat HIV RNA at next visit. D. Change regimen to abacavir/lamivudine/dolutegravir to improve adherence.

repeat HIV RNA at next viist - a Blip, worry if > 200 or >500 Virologic failure Immunologic failure • VL undetectable - drug resistance unlikely • VL <200 cps/ml - controversial; one large retrospective analysis found no increased risk of failure • VL persistently >200 cps/ml - drug resistance often associated (particularly >500 cps/ml) • Caution with change to newer VL assays and blips • Associated factors: ▫ CD4 <200 at ART initiation ▫ older age ▫ co-infections ▫ meds ▫ persistent immune activation ▫ loss of regenerative potential ▫ other reasons • No consensus on definition or treatment

68 y/o M with CKD immigrated from brazil to the US, underwent cadaveric renal transplant prior to trasnpalnt had episodes of recurrent epigastric pain WBC 6.5 with 15% eosinophils after transplant immunosuppresed 1 month later with hA meningismus and AMS, and a T of 39 LP WBC 2500 (98% neutrophils) glucose 20, protein 450 on empirica abx, with vanc/amp/cfx BCx and CSF grew Ecoli what to test for?

serioal stool exam

hospitalized pt with inter-current influenza A , tx promptely with oseltamivir should be placed on: standard percuations in any room/ private room contact, droplet or airbonre percuations?

standard percuations in a private room

3 non-family members begin vomititng 2 hours after eating a local italian resturant

staphylococcal enterotoxin from resturant c. perfingins 8-16hrs norovirus 24-48hrs shigella - longer

40s M a 1 week history of rash, described as "small red bumps" on trunk and upper extremities. Complained rash was pruritic, and worse with stress, friction, or trauma. Patient denied fever, headache, other symptoms. Denied sick contacts or exposure to children. Stated he was sexually active with men. He denied any new medications, recent travel, or known tick bites Afebrile Erythematous, 3-5 mm papular lesions on abdomen, upper extremities, thighs. One isolated lesion noted on each palm WBC 6.9 (Polys 51%, Lymphs 42%, atypical lymphs 4%). CXR: normal Referred to Dermatology Noted to have new lesions Skin biopsy performed Differential Diagnosis A. Acute HIV-1 infection B. Syphilis C. Epstein-Barr virus (EBV) infection D. Rocky mountain spotted fever E. Atypical measles

syphillis Granulomatous infiltrate with scattered plasma cells centered on nerve fibers Syphilis antibody positive. Rapid plasma reagin (RPR) 1:64 Admitted to unprotected sexual activity with new male partner 3-4 months ago Treated with two weekly injections of benzathine penicillin Rash faded, patient improved Acute HIV infection: diffuse maculopapular rash; not typically with lesions on palms/soles Acute EBV: rash may occur after amoxicillin (no such history in this case); no pharyngitis sx. RMSF: erythematous or petechial rash; begins in an acral distribution, then spreads to trunk. In this case, no other symptoms (e.g. headache) to suggest RMSF Atypical measles: often presents with fever, respiratory symptoms, abnormal CXR; all absent in this case

A 39-year-old man with HIV disease, CD4 298, HIV RNA 23,000 cps/ml, never on ART is diagnosed with pulmonary TB. The plan is to start INH, RIF, PZA, and ETH pending susceptibilities. He agrees to start ART and genotype is wild-type. Which ART regimen do you recommend? A. tenofovir/emtricitabine/efavirenz B. tenofovir/emtricitabine + atazanavir (boosted) C. tenofovir/emtricitabine + atazanavir (unboosted) D. tenofovir/emtricitabine + darunavir (boosted)

tenofovir/emtricitabine/efavirenz - drug drug interactions w/ rifampin - do not use rifampin with any protease inhibitor • Include a rifamycin in the regimen. • rifampin • significantly ↓ ALL PIs - cannot use together • ↓ RAL and DTG concentrations (need to ↑ RAL to 800 mg bid, DTG to 50 mg bid) • ↓ NNRTI concentrations: EFV 600 (or 800) mg daily • rifabutin: preferred; more manageable drug interactions with protease inhibitors • For IRIS, continue both ART and TB meds while managing the syndrome. • Treatment support, including DOT of TB rx is strongly recommended.

• The patient had been to her doctor 3 times over the past 8 months with this pruritic and mildly painful rash on her right buttock. She had been told that it was an irritation from riding a bicycle. • What is the key to the diagnosis? ▫ A. the fact that lesions recurred ▫ B.site of involvement is not unusual ▫ C.trauma can induce reactivation

the fact that lesions reoccurred

47 y/o M with AML and neutrophile count of 100 for past 3 week sbeen febrile tx with PIp-tazo , on micafungion bcx growing a yeast on gram stain most likely echocandin resistant yeast?

trichosporin sahii

56 y/o came 2 weeks ago from safari, in east africa game park now fever, mildly prurit rash, malaise, and easy fatigue of one week duration blood smear shown: trypasonmiasis

trypasynomiasis -from tstete fly (african sleeping disease) -shawman's disease not in peripheral

Paragonimus westermani "lung fluke" eggssnailsfreshwater crabs and crayfish Ingestion of undercooked seafood Adults migrate to LUNGS, frequent EOSINOPHILIA Symptoms: • fever, cough, diarrhea during acute migration • later, may have chest pain as worms migrate through lungs • can develop chronic pulmonary symptoms Dx: Sputum and/or stool exam for eggs. NOTE: Recent cases of Paragonimus kellicotti acquired in U.S. by ingestion of raw crayfish in rivers in Missouri

uunlikely to be tested Intestinal Flukes Fasciolopsis buski ("Giant Intestinal Fluke" 2cm w x 8 cm) • acquisition: eating encysted larval stage on aquatic vegetation • symptoms: usually asymptomatic • can cause diarrhea, fever, abdominal pains, ulceration, and hemorrhage Dx: eggs in stool Metagonimus yokagawi (2.5mm x 0.75mm) • acquisition: eating larvae in undercooked fish • symptoms: diarrhea and abdominal pain

45 y/o M 6 week of post SCT for myelodsyplasia w/ hx of chronic hep c d/c home to florid aon cyclosospirine, mycophenylate, prednisone, bactrim , ctilaopram and voriconazole, diffuse nonprurritic erythema developed over his sun exposed skin... cause?

voriconazole

21 y/o M, healthy with moderate severe asthma a positive skin test for egg, told by his allergist never to eat eggs which vaccine contains sufficient egg? Measles, mumps, rubella (MMR) Yellow fever Polio Varicella HPV

yellow fever contraindications for yellow fever vaccination is a severe allergic reaction to eggs, chicken proteins or gelatin. The other vaccines contain either no egg or, in the case of MMR, such minute quantities (picrograms) that they are not relevant. The most widely used, inactivated intramuscular influenza vaccines have little egg in current preparations.There are, however, two eggfree options. Note that allergy to gelatin is a concern, as most of the virus immunizations contain gelatin (bovine or porcine, which cross react), as does live oral typhoid. Histories of allergies to thiomerosol, neomycin, yeast, or latex are rarely relevant.

1)Scrub typhus - orienttia tsutugamushi - vector chiggers - japan / east asia - eschar, draining lymph node 2) Louse-borne relapsing fever Boriellia recurentis -body louse 3) Tick-borne relapsing fever - borrelia hermsi -borrelia turicatae - spirochetes -soft ticks - west coast/ moutnatins - jarisch-heimer reaction - dx giemsa stain - finding spirochetes 4) Boutonneuse (Mediterranean) fever - rickettsia conorii 5) Louse-borne epidemic typhus rickettsia prowazekii 6) Endemic (murine) typhus rickettsia typhi flea (rat, cat) milder 7)R. parkeri; american boutenneuse fever" eschar lone star tick spotted fever serology 8) R. africae multiple inoculation eschars safari, africa ambyloma tick 6-7 days 9) R. Akari, Ricketsiopox -eschar, no vesicular or pox virus - urban environment - mouse mites 10) R. Prowzeskii (epidemic typi) -flying squirls - spread by lice/louse , WWII - lethal 11)ehrlichia chaffeensis - Lone star tick - thrombocytopenia - luekopenia - elevated LFT - morulae in monocyte 12) anaplasma phagocytophillum - ixodes scapularis - not as severe - leukopenia (neutrophils lower) - morulae seen more frequently 13) babesia microti - ixodes scapularis - nantucket, martha's vinyard - most common cause blood transfusion - babesia PCR - tx w/ azithromycin plus atovaquone; or clindamycin and quinine 14 ) STARI Lone star tick (Ambyloamata) B. lonestarii (single case) Unclear if doxycycline needed 15) B miyamotoi - ixodes tick - spirochetes in CSF - low WBC and Plt, Increased as LFT 16) heartland virus - missourie, tenesse, arkensa - lone star tick - HME

• "Flu-like illness" (e.g. fever, headache, myalgia) thrombocytopenia Leukocytosis or leukopenia Elevated LFTs Doxycycline

• Candidiasis - Small, tender (very painful) papules • Herpes - vesicular • Aspergillus - ulcerative, necrotic (very little erythema) • Other filamentous fungi (Fusarium, P. boydii) - Multiple, erythematous, different stages - positive blood cultures • P. aeruginosa - Ecthyma gangrenosum

• Fusarium - can started as an onychomycosis that progresses -Invasive pulmonary disease with skin lesions • Locally invasive infections in neutropenic patients - Keratitis - Onychomycosis

Instead of healing, as shown on the last slide, the lesions progress despite antiviral therapy. The most likely cause for disease progression is a deficiency or alteration of: A. Ribonucleotide reductase B. Reverse transcriptase C. Protease D. Thymidine kinase E. DNA polymerase

• Three types of acyclovir resistant viruses: ▫ thymidine kinase negative ▫ thymidine kinase altered substrate ▫ DNA polymerase mutations • All populations of HSV contain viruses with resistant genotypes • Progressive disease has been limited to the immunocompromised host, especially HSCT recipients • Three normal hosts with documented ACV resistant virus


Set pelajaran terkait

Group Sickness and Accident Insurance

View Set

Agency Law - Introduction, Types, Duties

View Set

contract law end of chapter quiz

View Set

Abeka Science: Order & Design ( 7th Grade) Test 9 Chapters 7-9

View Set

Week 4 Qualitative Research (Part 1)

View Set

Medication Administration Assessment

View Set

Genética. El genoma humano. Capítulo 2

View Set